Anesthesiology Examination and Board Review [7th Edition] 9780071801539

The Single-Best Review for the American Board of Anesthesiology Written Examinations—Completely Updated to Reflect the N

1,300 200 7MB

English Pages 476 Year 2014

Report DMCA / Copyright

DOWNLOAD FILE

Polecaj historie

Anesthesiology Examination and Board Review [7th Edition]
 9780071801539

  • Commentary
  • NO BOOKMARKS

Table of contents :
1. Physics and Chemistry
2. Anesthesia Equipment
3. Circulation
4. Respiration
5. Nervous System
6. Renal, Hepatic, Endocrine, Hematologic, and Metabolic Systems
7. Pharmacology
8. General Anesthesia
9. Regional Anesthesia
10. Anesthesia for Cardiothoracic and Vascular Surgery
11. Anesthesia for Neurosurgery
12. Obstetric Anesthesia
13. Pediatric Anesthesia
14. Respiratory Therapy and Critical Care
15. Acute and Chronic Pain
16. Complications of Anesthesia and Quality Assurance

Citation preview

McGraw-Hill SPECIALTY BO ARD REVIEW

Seventh Edition

Anesthesiology

Examination & Board Review Ed ited by Mark D ershw itz, M.D ., Ph.D . Professor and Vice Chair of Anesthesiology Professor of Biochem istry & Molecu lar Pharm acology University of Massachu setts Med ical School and J. Matthias Walz, M.D . Associate Professor of Anesthesiology and Su rgery University of Massachu setts Med ical School McGraw -H ill N ew York

N ew York

Chicago San Francisco Athens Lond on Mad rid Milan N ew Delhi Singap ore Syd ney Toronto

Mexico City

Copyright © 2014 by McGraw-Hill Education. All rights reserved. Except as permitted under the United States Copyright Act of 1976, no part of this publication may be reproduced or distributed in any form or by any means, or stored in a database or retrieval system, without the prior written permission of the publisher. ISBN: 978-0-07-180153-9 MHID: 0-07-180153-7 The material in this eBook also appears in the print version of this title: ISBN: 978-0-07-177076-7, MHID: 0-07-177076-3. eBook conversion by codeMantra Version 1.0 All trademarks are trademarks of their respective owners. Rather than put a trademark symbol after every occurrence of a trademarked name, we use names in an editorial fashion only, and to the bene t of the trademark owner, with no intention of infringement of the trademark. Where such designations appear in this book, they have been printed with initial caps. McGraw-Hill Education eBooks are available at special quantity discounts to use as premiums and sales promotions or for use in corporate training programs. To contact a representative, please visit the Contact Us page at www.mhprofessional.com. Notice Medicine is an ever-changing science. As new research and clinical experience broaden our knowledge, changes in treatment and drug therapy are required. The authors and the publisher of this work have checked with sources believed to be reliable in their efforts to provide information that is complete and generally in accord with the standards accepted at the time of publication. However, in view of the possibility of human error or changes in medical sciences, neither the authors nor the publisher nor any other party who has been involved in the preparation or publication of this work warrants that the information contained herein is in every respect accurate or complete, and they disclaim all responsibility for any errors or omissions or for the results obtained from use of the information contained in this work. Readers are encouraged to con rm the information contained herein with other sources. For example and in particular, readers are advised to check the product information sheet included in the package of each drug they plan to administer to be certain that the information contained in this work is accurate and that changes have not been made in the recommended dose or in the contraindications for administration. This recommendation is of particular importance in connection with new or infrequently used drugs. TERMS OF USE This is a copyrighted work and McGraw-Hill Education and its licensors reserve all rights in and to the work. Use of this work is subject to these terms. Except as permitted under the Copyright Act of 1976 and the right to store and retrieve one copy of the work, you may not decompile, disassemble, reverse engineer, reproduce, modify, create derivative works based upon, transmit, distribute, disseminate, sell, publish or sublicense the work or any part of it without McGraw-Hill Education’s prior consent. You may use the work for your own noncommercial and personal use; any other use of the work is strictly prohibited. Your right to use the work may be terminated if you fail to comply with these terms. THE WORK IS PROVIDED “AS IS.” McGRAW-HILL EDUCATION AND ITS LICENSORS MAKE NO GUARANTEES OR WARRANTIES AS TO THE ACCURACY, ADEQUACY OR COMPLETENESS OF OR RESULTS TO BE OBTAINED FROM USING THE WORK, INCLUDING ANY INFORMATION THAT CAN BE ACCESSED THROUGH THE WORK VIA HYPERLINK OR OTHERWISE, AND EXPRESSLY DISCLAIM ANY WARRANTY, EXPRESS OR IMPLIED, INCLUDING BUT NOT LIMITED TO IMPLIED WARRANTIES OF MERCHANTABILITY OR FITNESS FOR A PARTICULAR PURPOSE. McGraw-Hill Education and its licensors do not warrant or guarantee that the functions contained in the work will meet your requirements or that its operation will be uninterrupted or error free. Neither McGraw-Hill Education nor its licensors shall be liable to you or anyone else for any inaccuracy, error or omission, regardless of cause, in the work or for any damages resulting therefrom. McGraw-Hill Education has no responsibility for the content of any information accessed through the work. Under no circumstances shall McGraw-Hill Education and/or its licensors be liable for any indirect, incidental, special, punitive, consequential or similar damages that result from the use of or inability to use the work, even if any of them has been advised of the possibility of such damages. This limitation of liability shall apply to any claim or cause whatsoever whether such claim or cause arises in contract, tort or otherwise.

From M .D.: For Renée, Phil, Eli, & Sally

From J.M .W.: To my wonderful wife Joanna, and my children Emily, Hannah, and Lucy

This page intentionally left blank

Co nte nts

Co ntributo rs ........................................................................................................................................... ix Pre fac e to the 7 th Editio n ..................................................................................................................... xi Intro duc tio n ......................................................................................................................................... xiii Part I: Basic Topics in Anesthesiology................................................................................................................1 1. Physics, Chemistry, and Mathematics

Ma r k Der sh wit z, M.D., Ph .D. Questions ...........................................................................................................................................................3 Answ ers and Exp lanations .............................................................................................................................9 2. Anesthesia Equipment

Ma r k Der sh wit z, M.D., Ph .D. Questions .........................................................................................................................................................13 Answ ers and Exp lanations ...........................................................................................................................19 3. Circulation

Th eod or e Al st on, M.D., Ph .D. Questions .........................................................................................................................................................25 Answ ers and Exp lanations ...........................................................................................................................35 4. Respiration

St avr os G. Memt soudis, M.D., Ph .D. Questions .........................................................................................................................................................41 Answ ers and Exp lanations ...........................................................................................................................52 5. N ervous System

Joh n J. A. Ma r ot a , M.D., Ph .D. Questions .........................................................................................................................................................61 Answ ers and Exp lanations ...........................................................................................................................68 6. Renal, Hepatic, Endocrine, Hematologic, and Metabolic Systems

Renée M. Goet zl er , M.D., M.P.H. a nd Ma r k Der sh wit z, M.D., Ph .D. Questions .........................................................................................................................................................75 Answ ers and Exp lanations ...........................................................................................................................88 7. Pharmacology

J. Mat t h ia s Wa l z, M.D. a nd Ma r k Der sh wit z, M.D., Ph .D. Questions .........................................................................................................................................................95 Answ ers and Exp lanations ......................................................................................................................... 116 v

vi

Conte 1: P hants rma cology

8. General Anesthesia

Ma r k Der sh wit z, M.D., Ph .D. Questions .......................................................................................................................................................133 Answ ers and Explanations .........................................................................................................................138 9. Regional Anesthesia

Joh n G. Ant ona ka kis, M.D. a nd Br ia n D. Sit es, M.D. Questions .......................................................................................................................................................143 Answ ers and Explanations .........................................................................................................................151 10. Practice Test

Th e Aut h or s Questions .......................................................................................................................................................155 Answ ers and Explanations .........................................................................................................................176 Part II: Advanced Topics in Anesthesiology .................................................................................................189 11. Pharmacology

J. Mat t h ia s Wa l z, M.D. a nd Ma r k Der sh wit z, M.D., Ph .D. Questions .......................................................................................................................................................191 Answ ers and Explanations .........................................................................................................................200 12. Regional Anesthesia

Joh n G. Ant ona ka kis, M.D. a nd Br ia n D. Sit es, M.D. Questions .......................................................................................................................................................207 Answ ers and Explanations .........................................................................................................................212 13. Anesthesia for Cardiothoracic and Vascular Surgery

Ch r ist ia n Mül l er , M.D. a nd J. Mat t h ia s Wa l z, M.D. Questions .......................................................................................................................................................215 Answ ers and Explanations .........................................................................................................................230 14. Anesthesia for N eurosurgery

Joh n J. A. Ma r ot a , M.D., Ph .D. Questions .......................................................................................................................................................239 Answ ers and Explanations .........................................................................................................................247 15. Obstetric Anesthesia

Rona l d B. Rubin, M.D. Questions .......................................................................................................................................................253 Answ ers and Explanations .........................................................................................................................270 16. Pediatric Anesthesia

Rebecca Za nconat o, M.D. Questions .......................................................................................................................................................281 Answ ers and Explanations .........................................................................................................................296

Conte nts

vii

17. Anesthesia for Miscellaneous Procedures

El ea nor M. Duduch , M.D. a nd Ma r k Der sh wit z, M.D., Ph .D. Questions .......................................................................................................................................................307 Answ ers and Exp lanations .........................................................................................................................318 18. Critical Care Medicine

Robin Guil l or y, M.D. a nd Ra e M. Al l a in, M.D. Questions .......................................................................................................................................................331 Answ ers and Exp lanations .........................................................................................................................347 19. Acute and Chronic Pain

Sa l a h a din Abdi, M.D., Ph .D. Questions .......................................................................................................................................................363 Answ ers and Exp lanations .........................................................................................................................373 20. Practice Test

Th e Aut h or s Questions .......................................................................................................................................................385 Answ ers and Exp lanations .........................................................................................................................410 References.............................................................................................................................................................431 Index ......................................................................................................................................................................433

This page intentionally left blank

Co ntributo rs Salahadin Abdi, M.D ., Ph.D . Professor and Chair Dep artm ent of Pain Med icine Division of Anesthesiology and Critical Care University of Texas MD And erson Cancer Center H ou ston, TX

Renée M. Goetzler, M.D ., M.P.H. Associate Program Director Carney H osp ital Associate Clinical Professor of Med icine Tu fts University School of Med icine Boston, MA

Rae M. Allain, M.D . Associate Director of Surgical Critical Care Dep artm ent of Anesthesiology & Pain Med icine St. Elizabeth’s Med ical Center Associate Professor of Anesthesiology Tu fts University School of Med icine Boston, MA

Robin K. Guillory, M.D . Co-Med ical Director of the N eu roscience ICU University of Lou isville H osp ital Assistant Professor of Anesthesiology Dep artm ent of Anesthesiology and Periop erative Med icine University of Lou isville School of Med icine Lou isville, KY

Theodore Alston, M.D ., Ph.D . Dep artm ent of Anesthesia, Critical Care and Pain Med icine Massachusetts General H osp ital Assistant Professor of Anesthesia H arvard Med ical School Boston, MA John G. Antonakakis, M.D . Staff Anesthesiologist Portsm ou th Regional H osp ital Portsm ou th, N H Mark D ershw itz, M.D ., Ph.D . Professor and Vice Chair of Anesthesiology Professor of Biochem istry & Molecu lar Pharm acology University of Massachu setts Med ical School Worcester, MA Eleanor M. D uduch, M.D . Resid ency Program Director Clinical Associate Professor of Anesthesiology University of Massachu setts Med ical School Worcester, MA

John J. A. Marota, M.D ., Ph.D . Dep artm ent of Anesthesia, Critical Care and Pain Med icine Massachusetts General H osp ital Assistant Professor of Anesthesia H arvard Med ical School Boston, MA Stavros G. Memtsoudis, M.D ., Ph.D ., F.C.C.P. Clinical Professor of Anesthesiology and Pu blic H ealth Weill Cornell Med ical College Director of Critical Care Services H osp ital for Sp ecial Su rgery N ew York, N Y Christian Müller, M.D . Director of Card iac Anesthesia Assistant Professor of Anesthesiology University of Massachu setts Med ical School Worcester, MA Ronald B. Rubin, M.D ., F.A.C.O.G. Assistant Professor of Anesthesiology University of Massachu setts Med ical School Worcester, MA ix

x

Contributors

Brian D . Sites, M.D . Associate Professor of Anesthesiology and Orthoped ic Su rgery Director of Regional Anesthesiology Geisel School of Med icine at Dartm outh College H anover, N H J. Matthias Walz, M.D . Associate Professor of Anesthesiology and Su rgery University of Massachusetts Med ical School Worcester, MA

Rebecca M. Zanconato, M.D . Assistant Professor of Anesthesiology and Ped iatrics University of Massachu setts Med ical School Worcester, MA

th

Pre fac e to the 7 Editio n The 7th ed ition of The M cGraw-Hill Board Review of A nesthesiology is tim ed to coincid e w ith the m ost su bstantial changes ever m ad e by the Am erican Board of Anesthesiology in the certification exam ination process. Beginning w ith the grou p of resid ents that begin anesthesiology training in 2013, the w ritten exam ination is being d ivid ed into tw o p arts. Resid ents w ill take the exam ination on Basic Top ics follow ing the first year of resid ency training, and w ill take the exam ination on Ad vanced Top ics tw o years later follow ing com p letion of the resid ency. Thu s, this book has been com p letely red esigned and rew ritten to conform to these changes im p lem ented by the Board . Once again, the su bsp ecialty areas w ithin anesthesiology w ere w ritten by colleagu es w ith sp ecial exp ertise in these field s. We are p leased by the ad d ition of several new contribu tors. Drs. Brian Sites and John Antonakakis w rote the chap ters on regional anesthesia, Dr. Stavros Mem tsou d is on resp iration, Dr. Christian Mü ller on card iothoracic anesthesia, Dr. Rebecca Zanconato on p ed iatric anesthesia, Dr. Salahad in Abd i on acu te and chronic p ain, and Dr. Eleanor M. Du d u ch on a n ew ch ap ter on an esth esia for m iscellan eou s p roced u res. As in the last ed ition, Dr. Ronald Ru bin, one of the few p hysicians in the United States fu lly trained and certified in both anesthesiology and obstetrics & gynecology w rote the chap ter on anesthesia for obstetrics, Dr. Ted Alston on circu lation, Dr. John Marota on the nervou s system , Dr. Renée Goetzler, an internist, on the hep atic, renal, m etabolic, and hem atologic system s, and Dr. Rae Allain on critical care, this tim e accom p anied by Dr. Robin Gu illory. The reference list w as exp and ed and m ost of the references are also available online via the w ebsite AccessAnesthesiology (w w w.accessanesthesiology.com ). Becau se of the su prem e im portance of p harm acology to the practice of anesthesiology, w e believe every anesthesiologist shou ld ow n a com prehensive reference. In the United States, the

clear choice is Good m an and Gilm an, n ow in its 12th ed ition and ed ited by Brunton, et al. Thu s the m ajority of the pharm acological qu estions in this book are referenced to it. Ped iatric and obstetric anesthesia rem ain im portant p arts of every resid ent’s training and a large p ortion of m any p ersons’ p ractices. Because the ped iatric and obstetric sections in the m ajor references are abbreviated , m any of the ped iatric and obstetric qu estions are referenced to the textbooks ed ited by Coté, et al. and Chestnu t, et al., resp ectively. For this edition, the Introduction was also extensively rewritten to include new information from the American Board of Anesthesiology on the aforementioned changes in the certification process as well as on the Maintenance of Certification in Anesthesiology (MOCA) process. MOCA is mandatory for diplomates with time-limited certificates issued after 2000 who wish to maintain their certification. This book contains 1500 questions and answers and m any m ore references. There w ill be errors d esp ite the best efforts of the au thors and the ed itors to prevent them. We are very grateful to those persons who previously contacted us to report errors they had found in previous editions. Once again, we would very much appreciate having any and all mistakes brought to our attention, and we have listed our email addresses below to facilitate communication. We w ould like to acknow ledge the contributors to previous ed itions of this book: Drs. Paul Alfille, Thomas Beach, William Denman, Peter Foley, Philippa Groves, Grace Harrell, and Bobbie Jean Sw eitzer. We w ould also like to thank our families for tolerating our absences necessitated by the preparation of this latest ed ition. Mark Dershw itz m ark.d ershw itz@u m assm ed .ed u J. Matthias Walz m atthias.w alz@u m m assm em orial.org Worcester, Massachu setts Ap ril, 2013 xi

This page intentionally left blank

Intro duc tio n If you are p lanning to p rep are for the Am erican Board of Anesthesiology (ABA) w ritten exam inations, includ ing MOCA (see below ), then The M cGraw-Hill Board Review of A nesthesiology, 7th ed ition, is d esigned for you . H ere, in one p ackage, is a com prehensive review resou rce w ith 1,500 Board typ e m u ltip le-choice qu estions w ith referenced , p aragraph-length d iscu ssions of each answ er. Org anizatio n o f this Bo o k The McGraw-Hill Board Review of Anesthesiology, 7th edition, is divided into two parts, covering the material that will appear on the Basic and Advanced written examinations, respectively. The Basic examination is taken after completion of the first (CA-1) year of anesthesiology residency training, while the Advanced examination is taken after completion of the third and final (CA-3) year of residency training. Successful passage of the Basic examination is required before a cand id ate can register for the Advanced examination. Su ccessfu l p assage of th e Ad van ced exam in ation is required before a candidate can register for the Applied (formerly called the oral) examination. Both the Basic and Advanced examinations are computer-based examinations taken at a standardized testing center. Complete information on the ABA examination process is found in the Booklet of Information published yearly by the ABA. While the Booklet was formerly available in hard copy, it is currently only available via download in PDF format from the ABA website, www.theaba.org.

A-t ype (single-best a nsw er) quest ions. This type of qu estion presents a problem or asks a question and is follow ed by fou r or five choices, only one of w hich is entirely correct. The d irections p reced ing this typ e of qu estion w ill generally ap pear as: DIRECTION S (Qu estion 1): Each of the nu m bered item s or incom p lete statem ents in this section is follow ed by answ ers or by com p letions of the statem ent. Select the ON E lettered answ er or com p letion that is BEST in each case. An exam p le for this item typ e is 1. A previou sly healthy 22-year-old m ale is u nd ergoing su rgical rem oval of a loose bod y in his knee u nd er general anesthesia. Ap p roxim ately one hou r after beginning anesthesia, the p atient becom es severely hyp ertensive. The m ost likely cause of the hyp ertension is (A) (B) (C) (D) (E)

p heochrom ocytom a m alignant hypertherm ia light anesthesia thyroid storm d ru g overd ose

Que s tio n Type s

In this typ e of qu estion, choices other than the correct answ er m ay be partially correct, bu t there can only be one best answ er. In the question above, the key w ord is “m ost.” Althou gh all of the options listed m ay cau se intraop erative hyp ertension, and all m u st be consid ered in the d ifferential d iagnosis, the m ost likely etiology of intraoperative hypertension is light anesthesia, or answ er (C).

There are three d ifferent typ es of qu estions fou nd on the Basic and Ad vanced exam inations. Each of these qu estion types is also fou nd in this book. In testing p arlance, they are referred to as A-typ e, G-set, and R-type qu estions.

G-set quest ions. A G-set begins w ith a case p resentation, a figu re, a d iagram , or a related item . This is then follow ed by a series of A-typ e qu estions that all p ertain to the case, figu re, d iagram , etc. An exam p le of this item typ e is

xiii

xiv

Introduction

TABLE 1. S TRATEGIES FOR ANS WERING S INGLE-BES T ANS WER QUES TIONS * 1. Re me mbe r tha t only one choice ca n be the corre ct a nswe r. 2. Re a d the que s tion ca re fully to be s ure tha t you unde rs ta nd wha t is be ing a s ke d. 3. Quickly re a d e a ch choice for fa milia rity. (This importa nt s te p is ofte n not done by te s t ta ke rs.) 4. Go ba ck a nd cons ide r e a ch choice individua lly. 5. If a choice is pa rtia lly corre ct, te nta tive ly cons ide r it to be incorre ct. (This s te p will he lp you le s s e n your choice s a nd incre a s e your odds of choos ing the corre ct choice /a nswe r.) 6. Cons ide r the re ma ining choice s a nd s e le ct the one you think is the a nswe r. At this point, you may wa nt to quickly s ca n the s te m to be s ure you unde rs ta nd the que s tion a nd your a nswe r. 7. Click on the corre ct a nswe r on the compute r s cre e n. (Eve n if you do not know the a nswe r, you s hould a t le a s t gue s s ; you a re s core d on the numbe r of corre ct a nswe rs, s o do not le ave a ny bla nks.) *Note tha t s te ps 2 through 7 s hould ta ke a n ave ra ge of a bout a minute. The a ctua l exa mina tion is time d for a n ave ra ge of a bout one minute pe r que s tion.

Directions: u se the follow ing scenario to answ er Qu estions 1-2: A 55-year-old w om an p resents to the p ain clinic w ith the com p laint of interm ittent ep isod es of electric shock-like pain over the right sid e of her nose and cheek. These ep isod es are m ost likely to occu r w hen she is ap p lying m akeu p , althou gh they m ight also be p recip itated by a strong breeze blow ing on her face. 1.

The m ost likely d iagnosis is (A) (B) (C) (D) (E)

2.

occip ital neu ralgia trigem inal neu ralgia m eralgia p aresthetica facial neu ralgia glossop haryngeal neu ralgia

The d ru g of choice for m anaging her sym p tom s is (A) (B) (C) (D) (E)

d iazep am m orp hine carbam azep ine flu oxetine chlorp rom azine

R-t ype (ext ended ma t ching) quest ions. These are m u ltiple-choice questions organized into sets that u se one list of options for all item s in the set. An exam ple of an R-set is

DIRECTIONS (Questions 1-3): Each group of items below consists of lettered headings follow ed by a list of numbered phrases or statements. For each numbered phrase or statement, select the ONE lettered head ing or component that is most closely associated w ith it. Each lettered heading or component may be selected once, more than once, or not at all. (A) (B) (C) (D) (E) (F) (G) (H )

α 1-ad renocep tor agonist α 1-ad renocep tor antagonist α 2-ad renocep tor agonist α 2-ad renocep tor antagonist β 1-ad renocep tor agonist β 1-ad renocep tor antagonist β 2-ad renocep tor agonist β 2-ad renocep tor antagonist

For each m ed ication, select the m ost accu rate d escrip tion of its recep tor activity. 1. 2. 3.

atenolol albu terol clonid ine

Answ ers, Explanations, and References At the end of each chap ter, there is a section containing the answ ers, exp lanations, and references to the qu estions. This section (1) tells you the answ er to each qu estion; (2) gives you an explanation/ review of w hy the answ er is correct, backgrou nd inform ation on the subject m atter, and w hy the other answ ers are incorrect; and (3) tells you w here you can find m ore in-d epth inform ation on the subject m atter in other reference books. We encou rage you to u se this section as a basis for fu rther stu d y and u nd erstand ing. If you choose the correct answ er to a qu estion, you can then read the exp lanation (1) for reinforcem ent and (2) to ad d to you r know led ge abou t the su bject m atter (rem em ber that the exp lanations u sually tell not only w hy the answ er is correct, bu t also w hy the other choices are incorrect). If you choose the w rong answ er to a question, you can read the exp lanation for a learning/ review ing d iscu ssion of the m aterial in the question. Furtherm ore, you can note the reference cited (e.g., 5:345), look u p the fu ll sou rce in the References on page 431 (e.g., Longnecker DE, et al., ed s. A nesthesiology, 2nd ed . N ew York: McGraw -H ill, 2012), and refer to the p age cited (p. 345) for a m ore in-d ep th d iscu ssion.

Introduction

Practice Tests In the tw o 150-qu estion Practice Tests at the end of Parts I and II, the qu estions cover all of the topics in the p reced ing chap ters in that Part. This form at m im ics the actu al exam and enables you to test you r skill at answ ering questions in all of the areas u nd er sim u lated exam ination cond itions. How to Us e this Bo o k There are two logical ways to get the most value from this book. We shall call them Plan A and Plan B. In Plan A, you go straight to the Practice Test and com p lete it accord ing to the instru ctions. Analyze your areas of strength and w eakness. This w ill be a good ind icator of you r initial know led ge of the subjects and w ill help to id entify specific areas for p rep aration and review. You can now u se the p reced ing chapters to help you im prove you r relative w eak points. In Plan B, you go throu gh the first nine chap ters in Parts I or II checking off you r answ ers, and then com paring your choices w ith the answ ers and d iscu ssions in the book. Once you have com p leted this process, you can take the Practice Test to see how w ell prep ared you are. If you still have a m ajor w eakness, it shou ld be ap p arent in tim e for you to take rem ed ial action. In Plan A, by taking the Practice Test first, you get qu ick feed back regard ing you r initial areas of strength and w eakness. You m ay find that you have a good com m and of the m aterial ind icating that p erhaps only a cu rsory review of the preced ing chapters is necessary. This, of cou rse, w ould be good to know early in you r exam p rep aration. On the other hand , you m ay find that you have m any areas of w eakness. In this case, you cou ld then focu s on these areas in you r review, not ju st w ith this book, bu t also w ith the cited references and w ith you r cu rrent textbooks and jou rnals. It is, how ever, u nlikely that you w ill not d o som e stud ying prior to taking the ABA exam (especially since you have this book). Therefore, it m ay be m ore realistic to take the Practice Test after you have review ed the first nine chapters in Parts I or II (as in Plan B). This w ill p robably give you a m ore realistic type of testing situ ation since very few of u s ju st sit d ow n to a test w ithou t stu d ying. In this case, you w ill have done some review ing (from superficial

xv

to in-d epth), and your Practice Test w ill reflect this stu d ying tim e. If, after review ing the preced ing chapters and taking the Practice Test, you still have som e w eaknesses, you can then go back to the preced ing chapters and su pplem ent your review w ith you r texts.

The Ame ric an Bo ard o f Ane s the s io lo g y History Anesthesiology is a relatively new specialty, and therefore the ABA is also relatively you ng. The Am erican Society of Anesthesiologists had its beginnings in 1911 in the N ew York area w hen tw o group s m erged to form the N ew York Society of Anesthetists. The grou p grew and by 1935 w as national in scop e. In 1936 the nam e w as changed to the Am erican Society of Anesthetists, and the society w as incorp orated . The Am erican Society of Anesthesiology, Inc. grew ou t of a com m ittee rep resenting the Am erican Society of Anesthetists, Inc., the Am erican Society of Regional Anesthesia, Inc., and the Section on Surgery of the Am erican Med ical Association. In 1937, the Am erican Board of Anesthesiology, Inc., w as form ed as an affiliate of the Am erican Board of Su rgery, Inc. In 1941, the board w as ap p roved as a separate entity. “The ABA m ission is to ad vance the highest stand ard s of the practice of anesthesiology. The ABA exists in ord er to:* A. Ad vance the highest stand ard s of p ractice by fostering lifelong ed u cation in anesthesiology, w hich the ABA d efines as the p ractice of m ed icine d ealing w ith bu t not lim ited to: (1) Assessm ent of, consu ltation for, and p rep aration of, patients for anesthesia. (2) Relief and p revention of p ain d uring and follow ing su rgical, obstetric, therap eu tic and d iagnostic p roced u res. (3) Monitoring and m aintenance of norm al p hysiology d uring the periop erative or periproced u ral period . (4) Managem ent of critically ill p atients. (5) Diagnosis and treatm ent of acu te, chronic and cancer related pain.

xvi

Introduction

(6)

Managem ent of hosp ice and p alliative care. (7) Clinical m anagem ent and teaching of card iac, p u lm onary, and neu rologic resu scitation. (8) Evalu ation of resp iratory fu nction and ap p lication of resp iratory therap y. (9) Cond u ct of clinical, translational and basic science research. (10) Su p ervision, teaching and evalu ation of perform ance of both m ed ical and allied health p ersonnel involved in p erioperative or p erip roced u ral care, critical care, pain m anagem ent, and hospice and p alliative care. (11) Ad m inistrative involvem ent in health care facilities and organizations, and m ed ical schools as ap prop riate to the ABA’s m ission. B. Establish and m aintain criteria for the d esignation of a Board certified and su bspecialty certified anesthesiologist as d escribed in the ABA’s Booklet of Inform ation. C. In form th e Accred itation Cou n cil for Grad u ate Med ical Ed u cation (ACGME) concerning the training requ ired of ind ivid uals seeking certification as su ch requ irements relate to resid ency and fellow ship training program s in anesthesiology. D. Establish and cond u ct those p rocesses by w hich the Board m ay ju d ge w hether a p hysician w ho voluntarily applies shou ld be issu ed a certificate ind icating that the requ ired stand ard s for certification or m aintenance of certification as a d ip lom ate of the ABA in anesthesiology or its su bspecialties have been m et. A Board certified anesthesiologist is a p hysician w ho p rovid es m ed ical m anagem ent and consu ltation d u ring the p eriop erative p eriod , in p ain m ed icine and in critical care m ed icine. At the tim e of ap p lication and at the tim e of initial certification, a d ip lom ate of the Board m u st p ossess know led ge, ju d gm ent, ad ap tability, clinical skills, technical facility and p ersonal characteristics su fficient to carry ou t the entire scop e of anesthesiology p ractice in d ep en d en tly, w ith ou t accom m od ation or w ith reasonable accom m od ation. An

ABA d ip lom ate m u st logically organize and effectively p resent rational d iagnoses and ap p rop riate treatm ent p rotocols to p eers, p atients, their fam ilies and others involved in the m ed ical com m u nity. A d ip lom ate of the Board can serve as an exp ert in m atters related to anesthesiology, d eliberate w ith others, and p rovid e ad vice and d efend op inions in all asp ects of the sp ecialty of anesthesiology. A Board certified anesthesiologist is able to fu nction as the lead er of the anesthesiology care team . Becau se of the natu re of anesthesiology, the ABA d iplom ate m u st be able to m anage em ergent life-threatening situ ations in an ind ep end ent and tim ely fashion. The ability to ind ep end ently acqu ire and p rocess inform ation in a tim ely m anner is central to assu re ind ivid ual responsibility for all asp ects of anesthesiology care. Ad equate p hysical and sensory facu lties, su ch as eyesight, hearing, sp eech and coord inated fu nction of the extrem ities, are essential to the ind ep end ent perform ance of the Board certified anesthesiologist. Freed om from the influ ence of or d ep end ency on chem ical su bstances that im p air cognitive, physical, sensory or m otor function also is an essential characteristic of the Board certified anesthesiologist. E. Serve the p u blic, m ed ical p rofession, health care facilities and organizations, m ed ical schools, and licensing board s by provid ing the nam es of p hysicians certified by the Board . Certification Requirements* At the tim e of certification by the ABA, the cand id ate m u st: A.

H old an u n exp ired licen se to p ractice m ed icine or osteop athy in at least one state or ju risd iction of the United States or p rovince of Canad a that is p erm anent, u ncond itional and u nrestricted . Fu rther, every United States and Canad ian m ed ical license the ap p licant hold s m u st be free of restrictions. Cand id ates for initial certification and ABA d ip lom ates have the affirm ative obligation to ad vise the ABA

Introduction

B. C.

D. E. F.

of any and all restrictions p laced on any of their m ed ical licenses and to p rovid e the ABA w ith com p lete inform ation concerning su ch restrictions w ithin 60 d ays after their im p osition or notice, w hichever first occurs. Su ch inform ation shall inclu d e, bu t not be lim ited to, the id entity of the State Med ical Board im p osing the restriction as w ell as the restriction’s d u ration, basis, and sp ecific term s and cond itions. Cand id ates an d d ip lom ates d iscovered not to have m ad e d isclosu re m ay be su bject to sanctions on their cand id ate or d ip lom ate statu s. H ave fu lfilled all the requ irem ents of the continu u m of ed u cation in anesthesiology. H ave on file w ith the ABA a Certificate of Clin ical Com p eten ce w ith an overall satisfactory ratin g coverin g th e final six-m on th p eriod of clinical anesthesia trainin g in each anesthesiology resid en cy p rogram . H ave satisfied all exam ination requ irem ents of the Board . H ave a p rofessional stand ing satisfactory to the ABA. Be capable of perform ing ind epend ently the entire scop e of anesthesiology practice w ithou t accom m od ation or w ith reasonable accom m od ation.

Althou gh ad m ission into the ABA exam ination system and su ccess w ith the exam inations are im p ortant step s in the ABA certification p rocess, they d o not by them selves gu arantee certification. The Board reserves the right to m ake the final d eterm ination of w hether each cand id ate m eets all of the requirem ents for certification, includ ing A, E and F above, after su ccessfu l com pletion of exam inations for certification. ABA certificates in anesthesiology issu ed on or after Janu ary 1, 2000 are valid for 10 years after the year the cand id ate p asses the exam ination for certification. ABA certificates are su bject to ABA rules and regu lations, inclu d ing its Booklet of Inform ation, all of w hich m ay be am end ed from tim e to tim e w ithou t further notice. A p erson certified by the ABA is d esignated a d iplomate in pu blications of the Am erican Board of Med ical Specialties and the Am erican Society of Anesthesiologists.

xvii

Maintenance of Certification in Anesthesiology Program* The ABA issues a certificate that is valid for 10 years to d iplom ates certified on or after Janu ary 1, 2000. They m u st satisfactorily com p lete the requ irem ents of MOCA before their tim e-lim ited certificate expires to m aintain d iplom ate status in the specialty. MOCA is a 10-year p rogram of ongoing self-assessm ent and lifelong learning, continu al p rofessional stand ing assessm ent, p eriod ic self-d irected assessm ents of practice perform ance and quality im provem ent, and an exam ination of cognitive exp ertise. A d ip lom ate’s MOCA cycle begins the d ay after the ABA aw ard s initial certification or m aintenance of certification in the specialty. The ABA aw ard s a certificate for Maintenance of Certification in the sp ecialty of Anesthesiology w hen a d ip lom ate has com pleted all MOCA p rogram requirem ents w ithin the p reced ing 10 years. At the tim e of com pletion of m aintenance of certification, the d iplom ate m ust be cap able of p erform ing ind ep end ently in the sp ecialty or su bspecialty, w ithout accom m od ation or w ith reasonable accom m od ation. Althou gh ad m ission into the MOCA program and su ccess w ith com p onents of the p rogram are im p ortant step s in the ABA m aintenance of certification process, they d o not by them selves gu arantee m aintenance of certification. The Board reserves the right to m ake the final d eterm ination of w hether each d ip lom ate m eets all of the requ irem ents for m aintenance of certification, includ ing Professional Stand ing and the ability to perform ind epend ently in the sp ecialty or su bsp ecialty, w ithou t accom m od ation or w ith reasonable accom m od ation, before aw ard ing m aintenance of certification. ABA m aintenance of certification certificates are su bject to ABA ru les and regu lations, inclu d ing its Booklet of Inform ation, all of w hich m ay be am end ed from tim e to tim e w ithou t fu rther notice. Physicians should m aintain com petency in the follow ing general areas: p atient care, m ed ical know led ge, p ractice-based learning and im provem ent, interp ersonal and com m u nication skills, p rofessionalism , and system s-based practice. The MOCA requ irem ents for Professional Stand ing, Lifelong

*The Am erican Board of Anesthesiology, Inc., Booklet of Inform ation, Febru ary, 2013.

xviii

Introduction

Learning and Self-Assessm ent (LLSA), Cognitive Exp ertise, and Practice Perform ance Assessm ent and Im p rovem ent (PPAI) are d esigned to p rovid e assessm ents of these six general com p etencies. A. PART I: PROFESSION AL STAN DIN G ASSESSMEN T ABA d iplom ates m u st hold an active, u nrestricted license to practice m ed icine in at least one ju risd iction of the United States or Canad a. Th e ABA assesses a d ip lom ate’s Professional Stand ing continu ally. ABA d ip lom ates have the affirm ative obligation to ad vise the ABA of any and all restrictions p laced on any of their m ed ical licenses and to p rovid e the ABA w ith com p lete inform ation concerning su ch restrictions w ithin 60 d ays after their im p osition. Su ch inform ation shall inclu d e, bu t not be lim ited to, the id entity of the m ed ical board im p osing the restriction as w ell as the restriction’s d uration, basis, and specific term s and cond itions. Diplom ates d iscovered not to have m ad e d isclosu re m ay be su bject to sanctions on their d ip lom ate statu s. Professional Stand ing accep table to the ABA is a p rerequisite qu alification for cognitive exam ination and for m aintenance of certification. B. PART II: LIFELON G LEARN IN G AN D SELF-ASSESSMEN T ABA d iplom ates shou ld continu ally seek to im p rove the qu ality of their clinical p ractice and p atient care throu gh self-d irected p rofessional d evelopm ent. This should be d one throu gh self-assessm ent and learning opportu nities d esigned to m eet the d ip lom ate’s need s and the MOCA requ irem ent for Lifelong Learning and Self-Assessm ent (LLSA). LLSA requ irem ents by certification year are available on the ABA w ebsite at w w w. theABA.org. The LLSA requ irem ent for m aintenance of certification is 250 cred its for continu ing m ed ical ed ucation (CME) activities. (1) All 250 cred its m u st be Category 1 cred its for ACCME-ap p roved p rogram s or activities.

(2)

From 2006 to 2012, no m ore than 70 cred its for CME program s and activities m ay be com pleted in the sam e calend ar year. Effective as of 2013, no m ore than 60 cred its for CME p rogram s and activities m ay be com p leted in the sam e calend ar year. MOCA participants w ill have to com p lete som e CME activity in at least five years of each 10-year MOCA cycle and are encou raged to com p lete som e CME activity in each of the six general com p etencies for physicians.

Self-Assessm ent CME Cred it Requ irem ents (1) All new ly certified d ip lom ates and n on-tim e lim ited d ip lom ates w ho enrolled in the MOCA program from Janu ary 1, 2008, to Decem ber 31, 2009, are requ ired to com p lete 60 Category 1 cred its of ABA-app roved self-assessm ent activities once d u ring th eir 10-year MOCA cycle. A list of the ap p roved activities is available on the ABA w ebsite at w w w.theABA.org. (2) All diplomates certified on January 1, 2010, or after and non-tim e lim ited d iplom ates w ho enroll in the MOCA p rogram after Janu ary 1, 2010, are requ ired to com p lete 90 Category 1 cred its of ABA-ap proved self-assessm ent activities once d u rin g their 10-year MOCA cycle. A list of the ap p roved activities is available on the ABA w ebsite at w w w.theABA.org. Patient Safety CME Cred it Requirem ents (1) All new ly certified d ip lom ates and n on-tim e lim ited d ip lom ates w ho enroll in the MOCA p rogram after Janu ary 1, 2008, are requ ired to com p lete 20 Category 1 cred its of Patient Safety CME. A list of the approved activities is available on the ABA w ebsite at w w w.theABA.org. CME sp on sors m ay su bm it CME activities an d cred its to th e ABA electron ically for ABA d ip lom ates. ABA d ip lom ates m ay self-rep or t th eir CME activities an d cred its to the

Introduction

ABA electronically. Whereas providerreported CME activities do not require verification by the ABA, self-reported CME activities are subject to audit and verification by the ABA within three years of their submission. Therefore, diplomates must keep documentation of every self-reported CME activity for at least three years after they submit it to the ABA for LLSA credit. C. PART III: COGN ITIVE EXPERTISE ASSESSMEN T Dip lom ates w ho p articip ate in MOCA m u st d em onstrate their cognitive exp ertise by p assing an ABA exam ination ad m inistered via com pu ter u nd er secure, proctored , stand ard ized testing cond itions. Abou t 75% of the test item s are based on general anesthesia topics, and the rem aind er of the exam ination is approxim ately evenly d istributed am ong the follow ing areas: p ed iatric anesthesia, card iothoracic anesthesia, neu roanesthesia, obstetric anesthesia, critical care m ed icine and pain m ed icine. Dip lom ates m ay satisfy the exam ination requirem ent no earlier than the seventh year of their 10-year MOCA cycle. Exam ination p rerequ isites for the p u rp ose of satisfying the MOCA program requ irem ent are: (1) Professional stand ing accep table to the ABA. (2) Practice Perform ance Assessm ent and Im p rovem en t (PPAI) p articip ation accep table to the ABA. (3) Dip lom ates m u st com p lete h alf (125 cred its) of the total CME requirem ent. Requ irem ents by certification year are available on the ABA w ebsite at w w w. theABA.org. There is no lim it to the nu m ber of tim es d ip lom ates m ay take the MOCA exam ination to satisfy the m aintenance of certification requ irem ent. The ABA w ill inform registered exam inees of the p roced u re for making an exam ination ap p ointm ent ap p roxim ately fou r m onths prior to the exam ination d ate. The MOCA Cognitive Exam ination is ad m inistered tw ice each year. Test d ates

xix

are available on the last page of the Booklet of Inform ation. H ow ever, for the m ost current test d ates p lease visit the ABA w ebsite at w w w.theABA.org, w hich is the official sou rce of ABA test d ates and d ead lines. Th e ABA m u st receive all d ocu m en tation it requ ires to m ake a d ecision abou t a d ip lom ate’s eligibility for exam in ation by October 31 of th e p reced in g year for th e w in ter exam in ation s an d by Ap ril 30 of th e exam ination year for th e su m m er exam in ation s. These d ead lines are absolu te, and the ABA m u st have d ocu m entation that the d iplom ate has m et all of the prerequ isites by the ap p rop riate d ead line. When the ABA d oes not have the required d ocum entation by the ap p rop riate d ead line, it w ill evalu ate the d ip lom ate’s eligibility for the next MOCA exam ination. It ultimately is the responsibility of the diplomates to assure that the ABA receives documentation in a timely manner that they have met all of the MOCA examination prerequisites. D. PART IV: PRACTICE PERFORMAN CE ASSESSMEN T AN D IMPROVEMEN T ABA d ip lom ates shou ld be continu ally engaged in a self-d irected p rogram of Practice Perform ance Assessm ent and Im p rovem ent (PPAI). The PPAI requ irem ent consists of the follow ing activities: (1) Case Evalu ation: A fou r-step p rocess w here d ip lom ate’s assess their p ractice and im p lem ent changes that im p rove p atient ou tcom es. A case evalu ation m ay be com p leted in a sp ecialty or su bspecialty of anesthesiology. Instru ctions and exam ples are available on the ABA w ebsite at w w w. theABA.org. (2) Sim u lation Ed u cation Cou rse: A contextu al learning opp ortunity to assess and im p rove one’s p ractice in areas su ch as crisis m anagem ent in a sim u lation setting at an ASA-end orsed center. A sim u lation ed ucation cou rse m ay be com p leted in a sp ecialty or su bsp ecialty of anesthesiology. Inform ation on ASA-end orsed sim ulation centers

xx

Introduction

Prac tic e pe rfo rmanc e as s e s s me nt and improve me nt re quire me nts by ye ar in MOCA c ycle Ye a r ce rtifie d

1

2

3

4

5

6

200 0-2003

Atte s ta tion

2004-2007

Atte s ta tion

2008 o r late r

*Ca s e eva lua tion or s imula tion

7

8

9

10

Atte s ta tion Ca s e eva lua tion or s imula tion *Ca s e eva lua tion or s imula tion Atte s ta tion

*Comple te both a ca s e eva lua tion a nd s imula tion cours e during your 10-ye a r MOCA cycle. One a ctivity mus t be comple te d be twe e n ye a rs 1 to 5, a nd the s e cond be twe e n ye a rs 6 to 10.

is available on the Maintenance of Certification p age of the ABA w ebsite at w w w.theABA.org. (3) Attestation: The ABA solicits references to verify d ip lom ate’s clinical activity and p articip ation in p ractice im provem ent activities. During their 10-year MOCA cycle, d ip lom ates m u st com plete the PPAI activities as d efined in the above chart, based on the year they w ere certified . Diplomates certified between January 1, 2001 and Decem ber 31, 2007, have the option of com p leting a Sim ulation Cou rse in lieu of com p leting an Attestation. Diplomates certified between January 1, 2004 and Decem ber 31, 2007 w ho elect to com p lete a Sim ulation Cou rse in lieu of an Attestation m ust com plete a Case Evaluation in years 6-10. Evid ence of one PPAI activity accep table to the ABA is a prerequ isite for the MOCA Cognitive Exam ination. Ad d itional inform ation abou t the ABA’s PPAI process and requirem ents by year certified can be found on the ABA w ebsite at w w w.theABA.org. Reciprocity For D iplomates: Diplom ates m ay com p lete one Part IV activity throu gh any other ABMS Board and su bm it it to the ABA to fu lfill the MOCA Part IV Case Evalu ation requ irem ent. Docu m entation of com pletion of the activity m u st be su bm itted w ith d ip lom ates’ requ ests for Part IV cred it. All d iplom ates enrolled in MOCA m u st com p lete a Sim u lation Ed u cation Cou rse and an

Attestation as d efined by their specific program requ irem ents. E. MOCA CYCLE DURIN G AN D AFTER TRAN SITION PERIOD The transition from a volu ntary recertification exam ination p rogram to MOCA began in Janu ary 2004. The volu ntary recertification exam ination end ed w ith the ad m inistration of the Decem ber 2009 Recertification Exam ination. (1) Dip lom ates certified before 2000 have a certificate that is not tim e-lim ited . They d o not have to com p lete the MOCA p rogram to m aintain certification. They m ay, how ever, volu ntarily p articip ate in the MOCA p rogram . The first tim e they ap p ly for MOCA they m ay com plete the p rogram in as soon as tw o years. They m ay com plete the exped ited MOCA program only once; thereafter, the 10-year MOCA p rogram is their only op tion. Dip lom ates certified before 2000 w ho choose to com plete their first MOCA program w ithin five years of their enrollm ent m ust com p lete tw o PPAI activities, an attestation and either a case evaluation or sim u lation ed u cation cou rse; those w ho choose to com plete the p rogram w ithin 6-10 years of their enrollm ent m ust com plete all three PPAI activities. The Professional Stand ing assessm ent is continu al. They can satisfy LLSA requ irem ents on the basis of CME activities com p leted after certification and w ithin the past 10 years. They can take a secure examination when they have satisfied

Introduction

all of the p rerequisite requ irem ents by the ap propriate d ead line. (2) The MOCA p rogram is the only op tion to m aintain certification for ABA d iplom ates certified in or after 2000. The ABA au tom atically enrolls d ip lom ates in MOCA w hen they are aw ard ed tim elim ited certification or w hen they su ccessfu lly com plete each MOCA cycle. They have to m aintain Professional Stand ing acceptable to the ABA and satisfy the Cognitive Exam ination requ irem ent. Ad d itionally, a. Dip lom ates certified in 2000, 2001, 2002, or 2003 w ere issu ed a tim e-lim ited certificate before the MOCA p rogram w as available. For these d ip lom ates, the LLSA requ irem ents for the secu re exam ination p rerequ isite and for the aw ard ing of m aintenance of certification are prorated , and , the PPAI requirem ent consists of the ABA obtaining attestations and evid ence of the cand id ate’s clinical activity and ongoing p ractice p erform ance assessm ent and im p rovem ent in Years 5 and 9 of their MOCA cycle. b. The MOCA p rogram w as available w hen d ip lom ates w ere issu ed a tim e-lim ited certificate in 2004, 2005, 2006, and 2007. For these d ip lom ates, the LLSA requ irem ents for the secure exam ination prerequ isite and for the aw ard ing of m aintenance of certification are not prorated . For PPAI, the ABA w ill obtain attestations and evid ence of the cand id ate’s clinical activity and ongoing program of practice perform ance assessm ent and im p rovem ent in Year 5 of the cand id ate’s MOCA cycle, and they have to com plete one of tw o PPA I activities (case evaluation or sim u lation ed u cation) in Years 6 throu gh 10. c. Dip lom ates enrolled in MOCA from 2000 throu gh 2007 m ay

F.

xxi

com p lete a sim u lation cou rse in p lace of p rovid ing references to su p p ort their attestations. In ord er for the d iplom ate to receive PPAI cred it for the cou rse, the sim ulation cou rse m u st be com p leted at an Am erican Society of Anesthesiologists (ASA) end orsed sim ulation center and all follow u p m ust be com pleted w ithin the requ ired tim efram e. Once all p ortions of the sim u lation cou rse are com plete, the ASA w ill report the com pletion to the ABA on behalf of the d iplom ate. d . The LLSA requ irem ents for the secu re exam ination p rerequ isite and for the aw ard ing of m aintenance of certification are not p rorated for d iplom ates issu ed a tim e-lim ited certificate in or after 2008. For PPAI, these d ip lom ates have to com p lete three PPAI activities (case evalu ation, sim u lation ed u cation, and an attestation). Dip lom ates m ay visit the ABA w ebsite at w w w.theABA.org or contact the ABA office for ad d itional inform ation regard ing their MOCA program requ irem ents. EN ROLLMEN T APPLICATION PROCEDURE Diplomates are automatically enrolled in MOCA w hen they are aw ard ed time-limited primary certification in anesthesiology and again w hen they successfully complete each MOCA cycle, includ ing an exped ited MOCA cycle. The ABA automatically enrolls d iplomates w ith a non-time limited primary certificate in MOCA upon their completion of the MOCA program in 2005 or thereafter. All other ABA diplomates have to apply to the ABA to enroll in MOCA. Dip lom ates not au tom atically enrolled in MOCA m ay electronically enroll at any tim e via the ABA w ebsite at w w w.theABA. org. Exceptions to this requirem ent w ill be consid ered u pon w ritten requ est. Written requ ests are to be ad d ressed to the ABA Secretary and m u st inclu d e the basis for the requ ested excep tion.

xxii

Introduction

Ap p licants m u st p rovid e inform ation abou t all their m ed ical licenses and cu rrent contact inform ation (e.g., p ostal ad d ress) to com p lete the ap p lication p rocess. It ultimately is the responsibility of every applicant to assure that the ABA receives all required information.” Inform ation concerning the Board and its requ irem ents m ay be obtained from The Am erican Board of Anesthesiology, Inc. 4208 Six Forks Road , Su ite 900 Raleigh, N C 27609-5735 Telep hone: (866) 999-7501 FAX: (866) 999-7503 Web: w w w.theABA.org Examination Preparation Id eally, prep aration for the exam ination shou ld begin d u ring training. Du ring this p eriod , tim e sp ent in a system atic approach to the bod y of know led ge w ill be w ell rew ard ed . A Content Ou tline of the In-Training Exam ination, by the Am erican Board of Anesthesiology and the Am erican Society of Anesthesiologists, covers the know led ge that shou ld be gained d u ring resid ency training. It serves as a grid for the qu estions

to be covered in the exam ination and w as u sed by all of this book’s au thors to choose top ics to be covered by each chap ter ’s qu estions. A cop y of the m ost cu rrent Content Ou tline m ay be obtained from the ABA’s w ebsite at w w w.theABA.org. Textbooks provid e a good background of inform ation. Pop ular com prehensive texts inclu d e those ed ited by Longnecker, et al., and Miller, et al. Many su bspecialty texts are available in the areas of pain m anagem ent, critical care, card iothoracic, obstetric, and p ed iatric anesthesia. The best sou rces of cu rrent inform ation are jou rnals and m eetings. The new est d ata and latest techniqu es are rep orted in these tw o sources, and one shou ld be fam iliar w ith them . An invalu able sou rce of inform ation is the Annu al Refresher Cou rse given each year in conju nction w ith the ASA m eeting in October. Lectu res are given throu ghou t the m eeting. A syllabu s is available covering all the lectu res and is su p p lied electronically to attend ees. Format of Examinations The Basic and Ad vanced exam inations of the Am erican Board of Anesthesiology are of the objective, m u ltip le-choice form em p loying the sam e qu estion types u sed in this book. They are com p uter-based exam inations taken at a stand ard ized testing center.

PART I

Bas ic To pic s in Ane s the s io lo g y

This page intentionally left blank

CHAPTER 1

Phys ic s , Che mis try, and Mathe matic s Que s tions DIRECTION S (Qu estions 1-34): Each of the nu m bered item s or incom p lete statem ents in this section is follow ed by answ ers or by com p letions of the statem ent. Select the ON E lettered answ er or com p letion that is BEST in each case. 1. A p atient w ith severe p u lm onary fibrosis w ishes to travel on an airp lane. What am ou nt of su p p lem ental oxygen w ill be need ed to m aintain the arterial oxygen concentration at abou t 70 m m H g? Assu m e the airp lane cabin is p ressu rized at 580 m m H g and that the p atient’s alveolar-arterial oxygen grad ient is 150 m m H g. (A) (B) (C) (D) (E)

40% 50% 60% 70% 80%

2. A clinician w ishes to p erform a single-breath ind u ction w ith sevoflu rane and exceed the m axim u m sevoflurane concentration su p plied by the vap orizer. She fills a 3-L reservoir bag w ith oxygen and ad d s 2 m L of sevoflu rane liqu id to the bag. What w ill the concentration of sevoflurane vapor be in the reservoir bag? Assu m e the op erating room is at sea level and the room tem p eratu re is 20°C. The id eal gas constant is 0.082 L-atm -°K-1-m ole -1, the specific gravity of sevoflurane is 1.52, and its molecular w eight is 200 g/ m ol. (A) (B) (C) (D) (E)

12.2% 13.5% 14.8% 16.1% 18.5%

3. In the typ ical op erating room , p atients lose heat via all of the follow in g m ech anism s EXCEPT (A) (B) (C) (D) (E)

su blim ation cond u ction convection rad iation evap oration

4. An an esth esiologist p lan s a clin ical stu d y comparing a new antiemetic w ith ond ansetron. 100 w om en u nd ergoing elective laparoscopic tu bal ligation w ill be rand om ized to receive either the new antiem etic or a stand ard 4-m g d ose of ond ansetron. The p rim ary ou tcom e variable is the num ber of w om en that vom it p ostop eratively in each grou p . In all likelihood , the best statistical test to com p are the tw o treatm ents w ill be (A) (B) (C) (D) (E)

Stu d ent’s t-test for u np aired d ata Stu d ent’s t-test for p aired d ata analysis of variance chi squ ared Wilcoxon rank-su m test

DIRECTION S: Use the follow ing scenario to answ er Qu estions 5-7: The stand ard extension cord u sed in a p articu lar hosp ital is 15 feet long and has a cap acitance of 0.05 µF. The capacitive reactance of each extension cord w hen u sed w ith 60 H z p ow er is 53 kilohm s. Three of these extension cord s are p lu gged into a circuit protected by the sam e circu it breaker and line isolation m onitor. Assu m e that the three extension cord s are the only sou rces of capacitance and cap acitive reactance on the circu it.

3

4

1: P hys ics, Che mis try, a nd Ma the ma tics

5. Wh at is the valu e of the cap acitan ce in th e circuit? (A) (B) (C) (D) (E)

0.0167 µF 0.025 µF 0.05 µF 0.10 µF 0.15 µF

9. The electrical cu rrent, often called the “let-go cu rren t,” above w h ich con traction of th e finger flexors is u nable to be overcom e by volu ntarily contracting the finger extensors is ap p roxim ately (A) (B) (C) (D) (E)

6. What is the valu e of the capacitive reactance in the circuit? (A) (B) (C) (D) (E)

17.7 kilohm s 26.5 kilohm s 53 kilohm s 106 kilohm s 159 kilohm s

7. Will the line isolation m onitor alarm ? (A) N o, becau se the leakage cu rrent is less than 1 m A. (B) N o, becau se the leakage cu rrent is less than 5 m A. (C) N o, becau se the leakage cu rrent is less than 10 m A. (D) Yes, becau se the leakage cu rrent is greater than 5 m A. (E) Yes, becau se the leakage cu rrent is greater than 10 m A. 8. In ord er to p erform a m eta-an alysis, a researcher (A) m u st show significance at a very high p robability level (e.g., p < 0.001) (B) com bines the resu lts from several sim ilar stu d ies (C) need s to enroll at least 1,000 p atients in a stud y (D) d esigns a stu d y that takes p lace at num erou s (e.g., > 100) ind epend ent clinical sites (E) stu d ies nu m erou s ind ep end ent p aram eters that evaluate an intervention (e.g., d ru g treatm ent)

0.15 m A 1.5 m A 15 m A 150 m A 1.5 A

DIRECTION S: Use the follow ing table to answ er Qu estions 10-12:

Vis c o s ity Gas o r vapo r (c e ntipo is e s )

Mo le c ular we ig ht (g /mo l)

De ns ity (kg /m 3 )

Bo iling po int (C)

Argon

0.022

39.9

1.66

−186

Ca rbon dioxide

0.0145

44

1.82

−57

Die thyl e the r

0.00075

74

3.08

35

He lium

0.0188

4

0.166

−269

Nitroge n

0.0175

28

1.16

−196

Nitrous oxide

0.0133

44

1.83

−88

Oxyge n

0.020

32

1.33

−183

Xe non

0.022

131

5.46

−108

10. Du e to an allergic reaction, a p atient has severe tracheal ed ema that u niformly narrow s the trachea from the vocal cord s to the carina. Com pared w ith breathing 100% oxygen, for a given in sp iratory force, airflow w ill be increased to the greatest d egree by d iluting the oxygen w ith (A) (B) (C) (D) (E)

heliu m nitrogen argon nitrou s oxid e xenon

11. A patient w ith an intralum inal tracheal tum or has severe narrow ing of the airw ay resulting in an air p assage resem blin g a p in h ole. Com pared w ith breathing 100% oxygen, for a given in sp iratory force, airflow w ill be increased to the greatest d egree by d iluting the oxygen w ith

Que s tions : 5–18

(A) (B) (C) (D) (E)

d iethyl ether carbon d ioxid e heliu m xenon nitrou s oxid e

12. Ordinarily it is not permissible to use a rotameter d esigned for one gas to control the flow of another gas. In a military field hospital w ithout spare parts, a biom ed ical engineer w ishes to can nibalize other equ ip m ent to rep lace a broken oxygen flow m eter. Althou gh not a p erfect m atch, the best replacem ent for the broken oxygen rotam eter w ou ld be one d esigned to control the flow of (A) (B) (C) (D) (E)

argon heliu m nitrogen nitrou s oxid e carbon d ioxid e

13. Regard ing the critical tem p erature of nitrou s oxid e in an E cylind er attached to an anesthesia m achine in an operating room , the critical tem p erature (A) is not relevant in an op erating room environm ent (B) is the tem p eratu re in the tank that p erm its nitrou s oxid e to exist as a liqu id (C) cannot be exceed ed w ithou t risking tank ru ptu re (D) is the tem p eratu re above w hich nitrou s oxid e cannot exist as a liquid (E) is the tem p eratu re at w hich nitrou s oxid e liqu id becom es a gas 14. An anesthesia tech accid entally left the oxygen flow m eter set at 15 L/ m in on a Frid ay aftern oon an d the oxygen flow ed at this rate throu gh the absorber all w eekend . On Mond ay m orn in g, w h en 10% d esflu ran e in 100% oxygen is ad m inistered , the p atient m ay be exp osed to a toxic concentration of (A) ozone (B) p hosgene (C) carbon d ioxid e

5

(D) carbon m onoxid e (E) flu orid e 15. A cylind er of oxygen has an internal volum e of 6 L and a pressure of 1700 psi. H ow many liters of oxygen w ill this tank supp ly at sea level? (A) (B) (C) (D) (E)

660 L 680 L 694 L 706 L 716 L

16. A patient is und ergoing operative repair of an arm fractu re. The anesthesiologist is stand ing fou r feet from the p atient’s arm . The su rgeon is u sing flu oroscop y to visu alize the fractu re. Assu m e the am ount of rad iation to w hich the anesthesiologist is exp osed is x. If the anesthesiologist m oves to a new p osition eight feet from the arm , then the am ou nt of rad iation exp osu re w ill d ecrease to ap p roxim ately (A) (B) (C) (D) (E)

0.75x 0.5x 0.33x 0.25x 0.125x

17. The absorption of one molecule of carbon d ioxid e by sod a lim e cau ses the net p rod u ction of how m any m olecu les of w ater? (A) (B) (C) (D) (E)

0 1 2 3 4

18. The cu rrent d elivered to the p atient by an electrocau tery d evice d iffers from the cu rrent sup p lied by an electrical u tility in its (A) (B) (C) (D) (E)

cap acitance frequ ency am p erage voltage p ow er

6

1: P hys ics, Che mis try, a nd Ma the ma tics

19. A solution of intravenou s fluid has the follow ing com p osition: 100 m M glu cose 120 m M sod iu m chlorid e 10 m M potassiu m chlorid e 5 m M calcium chlorid e 4 m M m agnesiu m chlorid e The osm olality of this solu tion is ap p roxim ately (A) (B) (C) (D) (E)

239 m Osm / 278 m Osm / 287 m Osm / 378 m Osm / 387 m Osm /

kg kg kg kg kg

H 2O H 2O H 2O H 2O H 2O

DIRECTION S: Use the follow ing figu re to answ er Qu estion 20:

21. The protection d evice know n as a grou nd fau lt interru p ter is rarely u sed in op erating room s becau se it (A) p rotects against m icroshock bu t not m acroshock inju ry (B) d oes not fu nction w ell in a w et environm ent (C) d isconnects p ow er to life su p p ort equ ip m ent (D) cannot be tested by the u ser for p rop er fu nctioning (E) d oes not have an au d ible alarm 22. The Dop p ler effect is the p rincip le behind n u m erou s m on itorin g d evices. Wh en th e sou rce of an au d io w ave m oves tow ard the observer, the sou nd (A) (B) (C) (D) (E)

am p litu d e d ecreases frequ ency d ecreases frequ ency increases p hase d ecreases p hase increases

23. If tw o d ifferent tissu es have the sam e p artial pressure of an anesthetic gas, the concentration of the gas in the tissu es w ill

20. In an intu bated p atient connected to an anesthesia m achine, m echanical d ead sp ace consists of the volu m e of the end otracheal tu be proxim al to the teeth plu s w hich one (or m ore) of the volum es ind icated in the figure? The d ifferent volu m es are ind icated by the d ifferent shad ings as show n in the legend in the low er left of the figu re. (A) (B) (C) (D)

1 1+2 1+2+3 1+2+3+4

(A) differ according to each tissue’s blood flow (B) differ according to each tissue’s partition coefficient (C) vary accord ing to the atm osp heric p ressu re (D) not d ep end on bod y tem p eratu re (E) be the sam e 24. When the relative hu m id ity is 100% then (A) relative hu m id ity and absolu te hu m id ity are equ al (B) relative hu m id ity and sp ecific hu m id ity are equ al (C) the relative hu m id ity w ill d ecrease if the tem peratu re d ecreases (D) a patient’s perspiration will not evaporate 25. Th e N ation al Fire Protection Association m an d ates w hich on e of these stan d ard s for op erating room s?

Que s tions : 19–30

(A) Isolated pow er is required in all operating room s (B) Isolated pow er is required in areas d esignated as w et locations (C) In the event of a pow er failu re, em ergency p ow er m u st becom e op erative w ithin 1 second (D) When isolated pow er is p resent, a line isolation m onitor m u st also be p resent DIRECTION S: Use the follow ing scenario to answ er Qu estions 26-27. A u niversity end ocrinology clinic d iagnosed pheochromocytom a in seven p atients in the preced ing year. The m ean arterial p ressu res (MAP), in m m H g, of the seven p atients at initial presentation w ere 120, 130, 135, 145, 150, 150, and 164. 26. The m ed ian of the MAP valu es is (A) (B) (C) (D) (E)

130 142 145 150 155

27. The sam p le stand ard d eviation of the MAP valu es is (A) (B) (C) (D) (E)

3.8 7.2 11.1 14.7 22.4

28. Which one of the follow ing intravenou s solu tions is a colloid ? (A) (B) (C) (D) (E)

lactated Ringer solu tion 0.9% saline 7.5% saline 5% albu m in 5% glucose

29. Of the follow ing intravenous solutions, the one w hose osm olality is m ost d ifferent from that of plasm a is (A) lactated Ringer solu tion (B) 5% glucose

7

(C) 2.5% glu cose + 0.45% saline (D) 0.9% saline (E) 5% glu cose + 0.45% saline 30. If the skin resistance betw een a p erson’s arm s is 6000 ohm s, and the p erson’s right arm is connected to an earth grou nd , and 120 V are applied to his/ her left arm, the cu rrent flow ing from arm to arm w ill be (A) nearly 0 if the 120 V are su p p lied by a grou nd ed electrical system (B) 20 m A if the 120 V are su p p lied by a grou nd ed electrical system (C) 20 m A if the 120 V are su pp lied by an isolated electrical system (D) 50 m A if the 120 V are su p p lied by an isolated electrical system (E) 50 A if the 120 V are su p p lied by an isolated electrical system DIRECTION S (Qu estions 31-34): Each grou p of item s below consists of lettered head ings follow ed by a list of nu m bered p hrases or statem ents. For each nu m bered phrase or statem ent, select the ON E lettered head ing or com p onent that is m ost closely associated w ith it. Each lettered head ing or com p on ent m ay be selected on ce, m ore than on ce, or n ot at all. (A) (B) (C) (D) (E) (F) (G) (H ) (I) (J) (K) (L) (M)

Volt Am p ere Jou le H enry Farad Ohm Cou lom b Watt Sievert Gray H ertz N ew ton N o u nit

For each m easu rem ent, select the ap p rop riate u nit.

8

1: P hys ics, Che mis try, a nd Ma the ma tics

31. The am ou nt of rad iation to w hich a p atient is exposed w hile u nd ergoing a CT scan. 32. The baricity of a local anesthetic solu tion for spinal anesthesia.

33. The electrical power used by a warming blanket. 34. The electrical imped ance of an anesthesiologist m easu red from the person’s left hand to right hand .

Answe rs a nd Expla na tions

1. (B) The patient’s alveolar oxygen concentration is estimated from the alveolar gas equation: P A O 2 = F IO 2 × (Patm − P H 2 O ) − Pa C O 2 × F IO 2

1 − F IO 2 + RQ

Assum ing a valu e of 0.8 for the resp iratory quotient, the patient’s alveolar oxygen concentration w ould be ap p roxim ately 222 m m H g w hen breathing 50% oxygen. The patient’s arterial oxygen concentration w ou ld therefore be app roxim ately 72 m m H g. (5:459) 2. (A) A volu m e of 2 m L of sevoflu rane liqu id has a m ass of 3.04 g corresp ond ing to 0.0152 moles. The volume of an ideal gas, V, is nRT/ P = (0.0152) × (0.082) × (273 + 20) = 0.365 L. A volum e of 0.365 L of sevoflu rane vap or in the 3-L reservoir bag rep resents a concentration of 12.2%. (5:627)

5. (E) The three extension cord s rep resent cap acitors connected in p arallel. When cap acitors are connected in parallel, their capacitances are ad d ed . (5:372) 6. (A) The three extension cord s rep resent reactances connected in p arallel. When three id entical reactance values are connected in parallel, the resu lting reactance value is one-third that of each ind ivid ual u nit. (5:372) 7. (D ) The leakage cu rrent d u e to the extension cord s w ill be given by Ohm ’s law, I = E/ Z, w here I is cu rrent in am p eres, E is voltage in volts, and Z is im ped ance in ohm s. In this circuit, since the only contribution to im ped ance is cap acitive reactance, the p otential leakage cu rrent is 6.8 m A at 120 V. Since the typical line isolation m onitor w ill alarm w hen the p otential leakage current is above 5 m A, the line isolation m onitor w ill ind eed alarm . (5:376-8)

3. (A) The fou r mechanism s of heat loss are cond u ction, convection, rad iation, and evap oration . Su blim ation is th e p h ase tran sition betw een a solid and a gas. (5:259)

8. (B) In a m eta-analysis, several stu d ies that m easured sim ilar outcome variable(s) are combined , typically to increase the statistical pow er that is d erived from a larger nu m ber of stu d y su bjects. (www.cochrane.org/handbook/915-whatdoes-meta-analysis-entail)

4. (D ) Of the tests listed , only the chi squ ared text com p ares p rop ortions (in this case, the p rop ortion of patients vom iting) in tw o popu lations. Stu d ent’s t-test and analysis of variance compare means in tw o populations, w hile the Mann-Whitney test is a nonparam etric test that assesses w hether one p op u lation has larger valu es than another. (4:1202-11)

9. (C) When electrical cu rrents of 10-20 m A are ap p lied to th e u p p er extrem ity, su stained m uscle contraction occu rs of a m agnitud e that cannot be overcom e. If the ind ivid u al is hold ing onto a w ire, he or she p robably w ill not be able to let go. (5:374-5)

9

10

1: P hys ics, Che mis try, a nd Ma the ma tics

10. (D ) Lam inar flow in a tu be is governed by Poiseuille’s law that relates lam inar flow to the 4th p ow er of the rad iu s: πpr 4 Flow = 8v l w here p is the p ressu re, r is the rad iu s of the tu be, v is the viscosity of the gas, and l is the length of the tu be. Since flow is inversely p roportional to viscosity, the flow w ill be increased to the greatest d egree by d ilu ting oxygen w ith nitrous oxid e, the gas that has the low est viscosity am ong the choices. (5:622) 11. (C) Gas flow throu gh a pinhole resu lts in turbu lent flow that is inversely p rop ortional to d ensity. Thu s, heliu m , the least d ense gas of th ose listed , w ou ld p rovid e th e greatest increase in flow w hen u sed to d ilu te oxygen. (5:622) 12. (B) The proper functioning of a rotameter tube d ep end s on lam inar flow in the tu be that is governed by Poiseu ille’s law. Therefore the closest rep lacem ent rotam eter w ou ld be for heliu m , the gas w ith the viscosity closest to that of oxygen. (5:623) 13. (D ) The critical tem p eratu re of nitrou s oxid e is the tem p eratu re at w hich it boils into a gas, regard less of the p ressu re in the tank. For nitrou s oxid e, that tem p eratu re is 36.5°C, a tem peratu re that cou ld easily be reached in an op erating room w ithou t air con d itionin g. (5:618, 622) 14. (D ) Desflurane, but not sevoflurane, may react w ith d ry sod a lim e to yield a p otentially toxic concentration of carbon m onoxid e. The reaction of trichloroethylene, an obsolete anesthetic agent, yield ed p hosgene. (5:614) 15. (C) At constant tem p eratu re, the p rod u ct of the p ressu re and volu me of a gas is constant (Boyle’s law ). Thu s, the volu m e of gas at sea level w here the atm ospheric p ressu re is 14.7 p si w ill be (1700 psi) × (6 L)/ (14.7 psi) = 694 L. (5:618)

16. (D ) Rad iation intensity d ecreases as a function of the squ are of the d istance. Thu s, by d ou bling the d istance betw een the p atient and the anesthesiologist, the am ount of the rad iation exp osu re w ill d ecrease to one-fou rth its p reviou s valu e. (5:385) 17. (B) A m olecu le of carbon d ioxid e reacts w ith one m olecu le of w ater to form carbonic acid . The reaction of one m olecu le of carbonic acid w ith tw o m olecu les of sod iu m hyd roxid e p rod uces tw o m olecules of w ater. Therefore, there is the net p rod u ction of one m olecu le of w ater. (5:641) 18. (B) The stand ard frequ ency u sed by electrical u tilities in the United States is 60 H z, w hile the cu rrent d elivered by an electrocautery d evice is in the range of 105 – 106 H z. 19. (E) Osm olality is the concentration of osm otically active particles per kilogram of solvent. At both room and bod y tem peratu res, the d ifference betw een 1 L of w ater and 1 kg of w ater can be ignored . Glucose d oes not d issociate in solu tion so its osm otic concentration is its m olar con centration. Sod iu m chlorid e and p otassiu m chlorid e each d issociate into tw o osm otically active p articles, w hile calciu m ch lorid e and m agnesiu m chlorid e each d issociate into three osm otically active particles. The total osm olality is therefore approximately 100 + 240 + 20 + 15 + 12 = 387 m Osm / kg H 2O. (5:511) 20. (A) Mechanical d ead sp ace in this p atient is the sum of the volume of the end otracheal tube p roxim al to the teeth p lus the volum e of the arm of the Y-connector. N one of the other volumes in the circle system contribute to mechanical d ead sp ace. 21. (C) A grou nd fault interru pter provid es excellent protection against injury from macroshock (bu t not m icroshock). It has a “test” bu tton to confirm proper functioning. Althou gh it d oes not have an au d ible alarm , that is not the reason w h y it is rarely u sed in op erating room s. A grou nd fau lt interru p ter p rotects the u ser by d isconnecting p ow er to the d evices

Answe rs : 10–34

11

connected to it. These d evices m ay inclu d e life su p p ort equ ip m ent in an op erating room . (5:379).

145 because there are three valu es low er (120, 130, and 135) and three valu es higher (150, 150, and 164).

22. (C) The d efinition of the Dop p ler effect is that w hen the sou rce of an au d io w ave m oves tow ard the observer, the frequ ency of the sou nd increases. (5: 430)

27. (D ) The sam ple stand ard d eviation, s, is given by the follow ing equ ation: N

s = 23. (B) The d efinition of partition coefficient is the ratio of the concentrations of a su bstance in tw o contiguou s com partm ents at stead y state (i.e., w hen the partial pressure of the substance is equ al in both com p artm ents). Partition coefficients vary accord ing to tem p eratu re bu t not accord ing to atm osp heric p ressu re. (5:600) 24. (D ) Absolu te hu m id ity is the am ou nt of w ater vapor in air, in u nits of m ass/ volu m e. Specific hu m id ity is the ratio of the m ass of w ater vapor in a volum e of air to the m ass of that volum e of air. Relative hu m id ity is the ratio of the partial pressure of w ater vapor in air to the saturated (maxim um) partial pressure of w ater vapor at the sam e tem peratu re and pressu re. Since the m axim u m w ater vap or content of air increases w ith tem p eratu re, if the tem p eratu re of a saturated volume of air is d ecreased , w ater vapor w ill cond ense into d roplets form ing fog, bu t the relative hu m id ity w ill rem ain 100%. When a patient is p laced in an atm osp here of 100% relative hu m id ity, persp iration w ill not evap orate and there w ill be no evap orative heat loss (as long as the volum e of the satu rated air is large enou gh su ch that the patient’s bod y tem p eratu re d oes not raise the air tem p erature by cond u ction or convection). 25. (B) Isolated p ow er is requ ired only in w et areas. Line isolation m onitors are not required , nor is the instantaneou s (i.e., < 1 sec) availability of em ergency backu p p ow er. (5:378, 380) 26. (C) The m ed ian valu e is the valu e that falls in th e m id d le of th e su bjects’ valu es. In th is grou p of seven p atients, the m ed ian valu e is

2 ( x − ) x ∑i =1 i

N −1

w here N is the num ber of observations, –x is the m ean, and x i rep resents each ind ivid u al observation. The su m of the squares of the d ifferences is 1298, thu s the sam p le stand ard d eviation is 14.7. 28. (D ) Colloid solu tions contain solu tes of high m olecu lar w eight (e.g., > 30 kDa). (5:200) 29. (E) The osmolality of 5% glucose + 0.45% saline is ap p roxim ately 50% greater than that of plasma. The other four solutions have approximately the same osmolality as plasma. (5:535) 30. (B) Cu rrent w ill flow only if both the p atient and the electrical system are grou nd ed . The m agnitu d e of the cu rrent is given by Ohm ’s law that states that cu rrent (in am p s) equ als the voltage (in volts) d ivid ed by the resistance (in ohm s). Thu s, the cu rrent w ill equ al 120 V/ 6000 ohm s or 0.02 A, that is 20 m A. (5:362-3, 376-8) 31. (I) Th e u n it of rad iation exp osu re is th e Sievert. (5:385) 32. (M) Baricity is the ratio of the sp ecific gravity of the local anesthetic solution to the specific gravity of spinal fluid at the same temperature. Since it is a ratio, it has no u nits. (5:794). 33. (H) The u nit of electrical p ow er is the Watt. (5:372) 34. (F) The u nit of electrical resistance is the Ohm . (5:372)

This page intentionally left blank

CHAPTER 2

Ane s the s ia Equipme nt Que s tions DIRECTION S (Qu estions 35-75): Each of the nu m bered item s or incom p lete statem ents in this section is follow ed by answ ers or by com p letions of the statem ent. Select the ON E lettered answ er or com p letion that is BEST in each case. 35. An anesthesiologist is ad m inistering isoflu rane via a variable byp ass vaporizer in Denver (altitu d e 1,609 m ). The fresh gas flow rates are 1 L/ m in oxygen and 2 L/ m in nitrou s oxid e. When the vaporizer is set to d eliver 2% isoflu rane, the concentration of isoflu rane p resent at the com m on gas ou tlet is approxim ately (A) (B) (C) (D) (E)

1.1% 1.6% 2% 2.7% 3.4%

36. A patient is being m onitored w ith a bispectral ind ex (BIS) m onitor. When the valu e for BIS is 60, it m eans that the p atient (A) has about a 60% p robability of being aw ake (B) has a very sm all (less than 1%) p robability of having recall of intraop erative events (C) d oes not requ ire ad d itional op ioid (D) if not p harm acologically p aralyzed , w ill not m ove in resp onse to su rgical incision (E) is less likely to be aw ake than if the BIS value w ere 50

37. An op en w aste-gas scavenging system (A) m u st have a negative p ressu re relief valve (B) m u st have a p ositive p ressu re relief valve (C) m u st be connected to a sou rce of vacu um (D) d oes not need a reservoir (E) cannot be sim u ltaneou sly connected to the APL and ventilator relief valves 38. Which one of the follow ing statements is TRUE regard ing blood p ressu re cu ffs? (A) The blad d er length shou ld be 50% of the lim b circu m ference. (B) The blad d er w id th shou ld be 40% of the lim b circu m ference. (C) A cu ff d esigned for a thigh cannot be u sed on a large arm . (D) A cu ff d esigned for an arm cannot be u sed on a sm all thigh. (E) A cu ff w hose blad d er is too narrow for the lim b w ill give an erroneou sly low blood p ressu re. DIRECTION S: Use the follow ing figure to answ er Qu estions 39-40:

13

14

2: Ane s the s ia Equipme nt

39. The system d ep icted in the figu re is the (A) (B) (C) (D) (E)

Georgia valve Jackson–Rees system T-p iece Bain circu it Map leson A system

40. A d isad vantage of the circuit in the figure is the (A) inability to u se sp ontaneou s ventilation w ith the system (B) requ irem ent for low flow (C) inability to scavenge w aste gases (D) p resence of overflow valve farther from the patient (E) kinking of inner d elivery tu be 41. A resid ent notices an increasing d iscrepancy betw een the Paco 2 and the end -tid al carbon d ioxid e. Factors that increase this d ifference inclu d e ALL of the follow ing EXCEPT (A) (B) (C) (D)

m ism atch of ventilation and p erfu sion w heezing high fresh gas flow rates high card iac ou tp u t

42. An anesthesia technician is transporting several Alad in isoflu rane vap orizer cassettes on a cart. After one of the cassettes accid entally tips over onto its sid e (A) the w ick w ill becom e satu rated (B) the cassette m ust be retu rned to the factory (C) the concentration of the vap or w ill be higher than calculated (D) the cassette shou ld be flu shed w ith oxygen at 10 L/ m in for 10 m in (E) the cassette m ay be p u t into u se im m ed iately 43. Which one of the following situations regarding in traop erative m on itorin g is m an d ated by th e Am erican Society of Anesthesiologists’ Stand ard s for Basic Intraoperative Monitoring?

(A) A p atient having an ingu inal herniorrhap hy u nd er local anesthesia w ith m onitored anesthesia care m u st have a resid ent or attend ing anesthesiologist or nu rse anesthetist p resent in the operating room at all tim es d u ring the su rgical proced u re. (B) A patient w ho und erw ent a thoracotom y and w ed ge resection yesterd ay and w ho is receiving a continu ou s ep id u ral infu sion of 0.1% bu p ivacaine m ust have his heart rate taken and record ed every five m inu tes. (C) A patient having a total knee replacem ent u nd er spinal anesthesia m u st have the exp ired carbon d ioxid e m easu red . (D) A p atient having intraop erative rad iation therap y u nd er general anesthesia m u st have a resid ent or attend ing anesthesiologist or nurse anesthetist p resent in the op erating room at all tim es d u ring the p roced u re. (E) A w om an in labor w ith an ind w elling ep id u ral catheter and receiving a continuous epid ural infusion of local anesthetic m u st have her blood pressure taken and record ed every five m inu tes. 44. An anesthesia circu it is connected to a circle system an d the com bin ed volu m e of both is 6 L. The fresh gas flow is 3 L/ m in. After isoflu rane 2% is tu rned on, how long w ill it take the concentration in the circu it to reach 1.96% isoflu rane (i.e., 98.1% of the concentration set on the vap orizer?) (A) (B) (C) (D) (E)

0.5 m in 1 m in 2 m in 4 m in 8 m in

45. A negative-p ressu re leak test (A) is accom p lished by having the clinician app ly m ou th suction to the tu bing connected to the com m on gas ou tlet (B) reliably find s leaks in the carbon d ioxid e absorber

Que s tions : 39–52

(C) is only ap p rop riate for anesthesia m achines containing a check valve d ow nstream from the vap orizers (D) m u st be p erform ed w ith the anesthesia m achine’s m aster sw itch tu rned on (E) m ay d etect internal vap orizer leaks 46. All anesthesia m achines app roved for cu rrent u se have check valves located at the (A) com m on gas ou tlet (B) exp iratory connection on the circle system (C) vap orizer ou tlet (D) w aste gas scavenging ou tlet 47. A 56-year-old w om an w ith breast cancer metastatic to the spine need s an MRI to evaluate the metastases. Because she cannot lie flat comfortably, general anesthesia is planned . Anesthesia equ ip m ent that is UN SAFE to u se in a room containing a fu nctioning m agnetic resonance im aging m achine inclu d es (A) (B) (C) (D)

laryngeal m ask airw ay fiberop tic oxim eter p robe blood p ressu re cu ff Macintosh laryngoscop e w ith p lastic hand le (E) end-tidal carbon dioxide sampling tubing 48. The m ajor com p onent, by w eight, in cu rrently u sed carbon d ioxid e absorbents is (A) (B) (C) (D) (E)

bariu m hyd roxid e calciu m hyd roxid e p H ind icator p otassiu m hyd roxid e sod iu m hyd roxid e

49. The form ation of carbon m onoxid e in a carbon d ioxid e absorber is facilitated by the presence of ALL of the follow ing com pou nd s EXCEPT (A) (B) (C) (D)

d esflu rane p otassiu m hyd roxid e sod iu m hyd roxid e w ater

15

50. A patient is being ventilated w ith a trad itional anesthesia m achine ventilator u sing the follow ing set p aram eters: Tid al volu m e = 500 m L Ventilatory rate = 10/ m in I:E ratio = 1:3 Oxygen ow = 1 L/ m in N itrou s oxid e ow = 2 L/ m in The resu lting m inu te ventilation is ap p roxim ately (A) (B) (C) (D) (E)

5 L/ m in 5.25 L/ m in 5.5 L/ m in 5.75 L/ m in 6 L/ m in

51. The tu bing that connects the APL (ad ju stable p ressu re lim iting, or p op -off) valve to the w aste gas scavenging system is the same diameter as the tu bing or the connector that is connected to the (A) (B) (C) (D) (E)

circle system com m on gas ou tlet end otracheal tu be reservoir bag ventilator p ressu re relief valve

52. The anesthesia machine checkout instructions p u blish ed by the Am erican Society of Anesthesiologists in 2008 include ALL of the following steps EXCEPT (A) assessm ent of the com p etency of the u nid irectional valves in the breathing circuit (B) calibration of the oxygen analyzer (C) confirm ation that the carbon d ioxid e absorbent is not exhau sted (D) d eterm ination that the vap orizers can d eliver agent (E) d eterm ination of the internal p ressu re of the attached oxygen cylind er

16

2: Ane s the s ia Equipme nt

53. Th e typ ical au tom atic n on in vasive blood p ressu re m easu rin g d evice m ost accu rately d eterm ines (A) (B) (C) (D)

DIRECTION S: Use the follow ing figu re to answ er Qu estion 58:

cessation of Korotkoff sou nd s d iastolic blood p ressu re m ean arterial p ressu re systolic blood p ressu re

54. The d evice u sed to red u ce the p ressure of a gas from a com p ressed gas cylind er to a u sable, nearly constant p ressu re is (A) (B) (C) (D) (E)

a gau ge a flow m eter an ind icator a regu lator a check valve

55. The p osition best tolerated by the su rgical patient is the (A) (B) (C) (D) (E)

lithotom y p osition p rone p osition horizontal su p ine p osition Trend elenbu rg p osition Fow ler p osition

56. If a nitrou s oxid e tank is contam inated w ith w ater vapor, ice w ill form on the cylind er valve as a resu lt of the (A) (B) (C) (D) (E)

latent heat of vap orization sp ecific heat vap or p ressu re low p ressu re of the nitrou s oxid e am bient tem p eratu re

57. The ind icator in carbon d ioxid e absorbent is (A) (B) (C) (D) (E)

m ethylene blu e ethyl violet brom thym ol blu e p henolp hthalein cresol p u rp le

58. The oxygen flow m eter shou ld be at p osition (A) (B) (C) (D) (E)

A B C D p osition is not critical

59. An anesthesiologist is u sing an anesthesia machine equipped w ith an oxygen proportioning system , as w ell as an extra flow m eter that is su p p lied by a heliu m tank. A hyp oxic gas m ixtu re m ay be ad m inistered to the p atient in any of the follow ing situ ations EXCEPT (A) the gas su p p lied by the w all oxygen outlet is not oxygen (B) there is a leak in the oxygen flow m eter (C) the heliu m flow m eter is ad ju sted to an excessively high flow rate (D) the nitrou s oxid e flow m eter is ad ju sted to an excessively high flow rate 60. A resid ent is teaching a m ed ical stu d ent to intu bate for the first tim e. She hand s her a laryngoscop e hand le w ith a Macintosh blad e and tells her that in ord er to u se the Macintosh blad e p rop erly, the (A) (B) (C) (D) (E)

tip is ad vanced into the vallecu la ep iglottis is lifted d irectly laryngoscop e is held in the right hand p atient shou ld be told to say, "Ah." blad e enters the m ou th on the left sid e

Que s tions : 53–67

61. A nitrous oxid e tank contains gas at a pressure of 750 psi. When the last d rop of liqu id nitrou s oxid e evaporates (A) the p ressu re w ill fall rap id ly (B) the rate of p ressu re fall is d ep end ent on the rate of flow (C) the p ressu re w ill begin to rise as the gas expand s (D) the p ressu re w ill be zero (E) the pressu re w ill rem ain at 750 psi u ntil the tank is em p ty 62. A p atient w ith a p rior history of aw areness u nd er anesthesia is to be m onitored w ith a BIS m onitor. This m onitor m ost reliably ind icates the d ep th of anesthesia w hen the p atient is anesthetized w ith (A) (B) (C) (D) (E)

oxygen p lu s high-d ose fentanyl ketam ine nitrou s oxid e xenon p rop ofol

63. When the oxygen flu sh valve is pressed (A) oxygen at the p ressu re of the com pressed oxygen cylind ers is app lied to the breathing system (B) the oxygen flow rate is accu rately d isplayed by the oxygen rotam eter (C) excessive agent concentration m ay be applied to the breathing system if the vap orizer is on (D) barotrau m a m ay resu lt if the APL valve is op en an d con n ected to an op en scavenging system (E) the ventilator shou ld not be in the insp iratory p hase of p ositive p ressu re ventilation 64. The d iam eter ind ex safety system (DISS) (A) prevents attachment of gas-administering equ ip m ent to the w rong typ e of gas (B) prevents incorrect yoke-tank connections (C) consists of qu ick-connectors typ ically m ou nted on the w all or hanging from the ceiling

17

(D) is fou nd on the w all end , bu t not the m achine end , of gas hoses connected to anesthesia m achines (E) is p rohibited on the ou tlet connections of portable gas tanks 65. In the su p ine p osition, all of the follow ing areas should be p ad d ed EXCEPT (A) (B) (C) (D) (E)

occip u t elbow s antecu bital fossae heels p op liteal fossae

66. The TRUE statem en t regard in g ECG lead p lacem ent is (A) lead I d isp lays the ECG signal record ed from the left arm and right arm . (B) lead II d isp lays the ECG signal record ed from the left arm and right leg. (C) lead III d isp lays the ECG signal record ed from the left leg and right leg. (D) lead V1 d isp lays the ECG signal record ed from a u nip olar electrod e p laced in the 2nd intercostal sp ace at the right m id axillary line. (E) lead V6 d isplays the ECG signal record ed from a u nip olar electrod e p laced in the 4th intercostal sp ace at the anterior axillary line. 67. The laryngeal m ask airw ay (A) p rotects against asp iration as w ell as an end otracheal tu be (B) can be u sed to prevent airw ay obstru ction d uring m onitored anesthesia care (C) p erm its p ositive p ressu re ventilation (D) requ ires a flexible fiberop tic laryngoscope for proper p lacem ent (E) is m ore easily p laced w hen an assistant ap p lies cricoid p ressu re

18

2: Ane s the s ia Equipme nt

68. View ing the w aveform of the capnograph m ay d etect all of the follow ing scenarios EXCEPT (A) (B) (C) (D) (E)

d isconnection of w all oxygen su p p ly exhau stion of carbon d ioxid e absorbent p u lm onary em bolu s w heezing esop hageal intu bation

69. All of the follow ing are tru e of the lithotom y position EXCEPT (A) (B) (C) (D) (E)

the back is su p ine the knees are flexed the hip s are flexed the legs are internally rotated there are few hem od ynam ic d isad vantages to the p osition

70. The isolated p ow er su p p ly system u sed in op erating room s requ ires that (A) the m etal p ortions of the op erating table be connected to earth grou nd (B) the p atient be insu lated from the m etal p ortions of the op erating table (C) the anesthesia m achine and anesthesia m onitors be connected to d ifferent electrical circu its than electrocautery d evices (D) cond u ctive flooring be u sed in the op erating room (E) a transform er be connected betw een electrical equ ip m ent in the op erating room and the electric p ow er su p p lied by the utility com p any 71. A fiberop tic bronchoscop e is u sefu l for airw ay m anagem ent of the p atient w ith a d ifficu lt airw ay, and the suction channel of the scop e m ay facilitate intu bation by facilitating all of the follow ing actions EXCEPT (A) ad m inister oxygen (B) ventilate the p atient d u ring bronchoscop y

(C) ad vance the scop e over a w ire inserted p ercutaneou sly throu gh the larynx (D) sp ray local anesthetic (E) asp irate tracheal secretions 72. Pu lse oxim eters (A) are based on the Bernou lli p rincip le (B) can d ifferentiate carboxyhem oglobin from oxyhem oglobin (C) cannot d ifferentiate m ethem oglobin from oxyhem oglobin (D) u se a single w avelength of red light 73. Gases and vap ors that m ay be m easu red by infrared sp ectrom etry inclu d e ALL of the follow ing EXCEPT (A) (B) (C) (D) (E)

halothane isoflu rane carbon d ioxid e nitrogen nitrou s oxid e

74. The fail-safe valve on an anesthesia m achine (A) p revents d elivery of a hyp oxic m ixtu re (B) p revents nitrou s oxid e flow u nless the oxygen flow m eter is on (C) p revents high p ressu re in the breathing circu it from reaching the vaporizers (D) requ ires that oxygen tanks be fu ll (E) is op en if oxygen p ressu re is p resent 75. Venou s air em bolism m ay be rap id ly d etected by ALL of the follow ing m ethod s EXCEPT (A) alteration in heart sou nd s d etected by a precord ial Dop pler (B) d ecrease in exp ired carbon d ioxid e on the capnom eter (C) increase in exp ired nitrogen m easu red by m ass sp ectroscop y (D) d ecrease in oxygen satu ration ind icated by the p u lse oxim eter

Answe rs a nd Expla na tions

35. (D ) At the red u ced barom etric p ressu re in Denver (about 630 m m H g), the concentration of isoflu rane d elivered by a variable byp ass vap orizer w ill be slightly higher th an th e vaporizer setting. (5:599) 36. (B) The BIS m onitor m easu res the d ep th of hypnosis and the BIS valu e can be u sed to p red ict the probability that a patient w ill be aw ake and responsive or w ill have recall at a given point in tim e. The BIS valu e is not a m easu re of the likelihood of m ovem ent or of the m agnitu d e of noxiou s stim uli requiring opioid therap y. Du ring su rgery, the typ ical BIS target valu e is betw een 40 and 60. At a BIS valu e of 60, the probability of resp onsiveness to verbal com m and is abou t 20%, w hile the probability of recall is less than 1%. (5:482, 692) 37. (C) An op en w aste-gas scavengin g system requ ires a reservoir bu t no valves. It m u st be connected to a sou rce of vacu u m to actively w ithd raw w aste gases. It is u su ally connected to both the APL and ventilator relief valves via a ”Y” connector. (5:655-6) 38. (B) The blad d er length shou ld be at least 80%, and the w id th 40%, of the lim b circu m ference. If the blad d er is too sm all, a larger p ressu re w ill be need ed to occlu d e flow and an erroneou sly high blood pressure read ing w ill resu lt. The labeling of a cu ff as ”ad u lt” or ”thigh” is a general gu id eline; the cu ff size sh ou ld be chosen to m atch a particular lim b size, regard less of w hether the lim b is the arm or leg. (Circulation 1980; 62:1146A -1155A )

39. (D ) The circu it show n is a Bain circu it that is a mod ification of the Mapleson D system. (5:639) 40. (E) The inner tu be m ay becom e d isconnected or it may kink, lead ing to excessive rebreathing of exhaled gases or d isru p tion of fresh gas flow. The Bain circu it p erm its sp ontaneou s ventilation, requires relatively high fresh gas flow s, perm its easy w aste gas scavenging, and p ositions the p op -off valve aw ay from the p atient for easier ad ju stm ent. (5:639) 41. (D ) The d ifference betw een arterial and m easu red en d -tid al carbon d ioxid e w ill be increased by ventilation-p erfu sion m ism atch (e.g., em boli, d ecreased card iac ou tpu t), prolonged exp iratory p hase (e.g., w heezing), and high fresh gas flow rates that d ilu te exp ired carbon d ioxid e. (5:465-7) 42. (E) The Alad in vap orizer is not of the variable bypass type. When the cassette is d isconnected from the vap orizer, it m ay be transp orted in any position w ithou t hazard . (5:635) 43. (A) Stand ard I of the ASA Stand ard s for Basic In traop erative Mon itorin g m an d ates th at ”qu alified anesthesia p ersonnel shall be p resent in the room throu ghou t the cond u ct of all general anesthetics, regional anesthetics, and monitored anesthesia care. In the event there is a d irect know n hazard (e.g., rad iation) to the anesthesia p ersonnel that m ight requ ire interm itten t rem ote observation of th e p atient, som e p rovision for m onitoring the p atient m u st be m ad e.” Th e stan d ard s ”are n ot intend ed for ap p lication to the care of the obstetric p atient in labor or in the cond uct of

19

20

2: Ane s the s ia Equipme nt

pain m anagem ent." The patient having sp inal anesthesia m ay have his or her ventilation "evaluated by continual observation of qualitative clinical signs," accord ing to Stand ard II. (5:88-90) 44. (E) The tim e constant of the system is the volume divided by the fresh gas flow, i.e., 6 L ÷ −

t τ

3 L/ m in, or 2 m in. Since 1 − e = 1 − e −1 = 0.63, after one tim e constant the isoflu rane concentration w ill be 63% of the valu e set on the vap orizer. After tw o tim e constants, the percentage is 85%; after three tim e constants, 95%; after fou r tim e constants, 98%; and after five tim e constants, 99.3%. (5:601) 45. (E) The negative-p ressu re leak test is a u niversal test for leaks in the low -pressu re circuit of anesthesia m achines, regard less of w hether the low -p ressu re circu it contains a check valve. Perform ing the test requ ires ap plying negative pressu re w ith a su ction bu lb to the com m on gas ou tlet (from w hich the carbon d ioxid e absorber has been d isconnected ) w ith the m achine’s m aster sw itch turned off. With the su ction bu lb collapsed , each vaporizer is ind ivid u ally tu rned on and reinflation of the suction bu lb ind icates an internal leak in that vaporizer. (5:659-60) 46. (B) Unidirectional check valves are required on the insp iratory and expiratory connections on the circle system to p revent rebreathing of exhaled gas. Som e anesthesia m achines have check valves at the com m on gas ou tlet or at vaporizer ou tlets, but they are not necessary. (5:636-7, 640) 47. (D ) Ad ministering anesthesia and / or monitoring a patient d uring an MRI scan is a challenge. Laryngeal m ask airw ays are safe. An oxim eter probe w ith a fiberop tic cable to the m onitor is safe, w hile an oxim eter probe connected w ith an electrical cable m ay cau se a bu rn at the probe site. Blood pressure cuffs are safe as long as the tubing connectors are not ferrom agnetic and p lastic gas sam pling tu bing is safe. While plastic laryngoscope blad es are read ily available, the light sou rces requ ire batteries that are

ferrom agnetic and shou ld not be brou ght into the room containing the MRI machine. (5:1273; A nesthesiology 2009; 110:459-79.) 48. (B) While som e absorbents have low p ercentages of sod iu m or p otassiu m hyd roxid e, m ost of the absorbent consists of calcium hyd roxid e. Bariu m hyd roxid e is no longer u sed . (5:641) 49. (D ) The form ation of carbon m onoxid e resu lts w hen d esflu rane reacts w ith a strong base (i.e., sod iu m or potassiu m hyd roxid e, but not calcium hyd roxid e) in the absence of w ater, i.e., in absorbent that has been d ried by being flu shed w ith d ry gas. (5:641) 50. (D ) In a typ ical anesthesia ventilator, d u ring insp iration, the p ressu re relief valve is closed so that the tid al volume d elivered to the patient is the sum of the set tid al volume plus the fresh gas flow that occu rs d u ring insp iration. In this case, the total fresh gas flow is 3,000 m L/ m in or 50 m L/ sec. With a ventilator rate of 10/ m in, each breath lasts for 6 sec. With an I:E ratio of 1:3, the insp iratory p hase lasts for 1.5 sec. Therefore, the set tid al volu m e is au gm ented by fresh gas flow ing for 1.5 sec, or 75 m L/ breath. (5:650) 51. (E) The w aste gas scavenging system is connected to the APL valve and the ventilator p ressu re relief valve w ith 19- or 30-m m tu bing. The circle system connectors and the connection for the reservoir bag are 22 m m , w hile the comm on gas outlet and end otracheal tube connections are 15 m m. All of these measu rem ents are internal d iam eter (ID). (5:655) 52. (D ) Confirm ation of the ability of the vap orizers to d eliver volatile agent is not one of the listed step s. (5:662) 53. (C) Au tom atic noninvasive blood p ressu re d evices u se oscillom etry and d irectly m easu re m ean arterial pressu re that is the p ressu re at w hich the cu ff p ressu re flu ctu ations are m axim al. Each m anu factu rer em ploys its ow n, and u nique, algorithm to estim ate systolic and d iastolic pressu res. These d evices d o not listen for Korotkoff sou nd s. (5:406)

Answe rs : 44–64

54. (D ) The mechanism to red u ce pressu re of a gas to a useful pressu re is a regu lator. A gauge is a d evice to m easure the pressure. The flow m eter is a d evice to m easu re the flow being d elivered . A check valve is a d evice to allow flow in one d irection only. (5:619) 55. (C) The horizontal su p ine p osition is the one best tolerated , bu t even that one has its p roblems. Merely because the patient d oes not have to be m oved shou ld not cau se one to be com p lacent w ith p osition in g. Pressu re p oints shou ld be pad d ed , the sup erficial nerves protected , and the eyes protected . (5:362-3) 56. (A) The nitrou s oxid e tank contains a liqu id , and in ord er for it to becom e vap orized , heat m ust be sup plied . As the cylind er is opened , heat is removed from the cylind er and from the air in the im m ed iate vicinity. The tem p eratu re falls, cau sing cond ensation. (5:626) 57. (B) The ind icator in carbon d ioxid e absorbent is ethyl violet. (5:641) 58. (D ) Placem ent of the oxygen flow m eter in the p osition nearest to the com m on gas ou tlet w ill avoid d elivery of a hyp oxic m ixtu re shou ld a crack occu r in the flow m eter tu bing. If a leak occurs in this position, all com p onents of the gas w ill leak ou t the crack in the tu bing. (5:622) 59. (D ) An oxygen p rop ortioning system m echanically connects the oxygen and nitrou s oxid e flow controls so that as the nitrou s oxid e flow is increased , or the oxygen flow is d ecreased , the flow of the other gas is changed to m aintain a m inim u m oxygen concentration of 25%. The system d ep end s u p on oxygen being correctly su p p lied to the oxygen flow control. Therefore, increasing the nitrou s oxid e flow w ill not, by itself, resu lt in a hyp oxic m ixtu re. Any ad d itional flow controls (e.g., heliu m , air, carbon d ioxid e) are not linked to the oxygen flow control, and increasing their flow cou ld lead to a hyp oxic m ixtu re. Oxygen leaking from the oxygen flow m eter is not d etected by the proportioning system and could also result in a hyp oxic m ixtu re. (5:624)

21

60. (A) The Macintosh blad e is u sed by placing it in the vallecu la (the sp ace betw een the base of the tongu e and the ep iglottis). The laryngoscope is held in the left hand , and the blad e enters the m ou th on the right. Becau se of the w id esp read availability of fiberop tic laryngoscopes, it is now uncommon to use a Macintosh blad e for an aw ake intu bation. (5:563) 61. (B) When the last d rop of nitrous oxid e liqu id evaporates, the tank is ap proxim ately 16% fu ll. The p ressu re fall w ill d ep end on the size of the tank and the rate of flow. (5:622) 62. (E) The BIS monitor has been valid ated primarily w ith anesthetic techniqu es based exclu sively or p rim arily on p rop ofol or volatile anesthetics. It d oes not reliably ind icated d epth of anesthesia w hen the anesthetic techniqu e is based exclu sively or p rim arily on ketam ine, nitrou s oxid e, xenon, or op ioid s. (5:610) 63. (E) The oxygen flu sh valve can ap p ly oxygen to the breathing system at the p ressu re of the p ip eline su p p ly (e.g., ap p roxim ately 50 p si) p ossibly cau sing barotrau m a. If the APL valve is op en and the scavenging system is of the open type, barotrau ma is minimized . If oxygen is bein g su p p lied by th e gas cylin d ers m ou nted on the anesthesia m achine, the cylind er p ressu re is d ecreased to ap p roxim ately 45 p si in th e an esthesia m achin e, an d th is p ressu re is ap p lied to th e breath in g circu it w h en th e oxygen flu sh valve is p ressed . Du ring the insp iratory p hase of p ositive p ressu re ventilation, both the ventilator relief and p op -off valves are closed , an d p ressing th e oxygen flu sh valve w ill ap p ly high p ressu re to th e breath in g circu it. Th e oxygen flu sh valve is p ositioned d istally to the rotam eters an d vap orizers in an an esth esia m ach in e. (5:621, 631) 64. (A) The system is based on m atching sp ecific bores and d iam eters that are assigned to the specific gases. The DISS should prevent attachm ent of gas ad m inistration equ ipm ent to the w rong gas. This is not the protective system for cylind er-yoke attachm ents that u se the p in ind ex safety system (PISS). DISS connectors

22

2: Ane s the s ia Equipme nt

m ust be screw ed on and are therefore not consid ered "quick-connectors." They are required on the m achine end , and are optional on the w all en d , of h oses con n ectin g an esth esia m achines to the w all. A gas-sp ecific "qu ickconnector" is an alternative for the w all connection. DISS connectors are also som etim es fou nd on the gas ou tlets of p ortable gas tanks. (5:619) 65. (C) Pressure d amage may occur on the occiput, elbow s, and heels after prolonged proced u res. Pad d ing the p op liteal fossae m ay d ecrease postop erative stiffness and back pain. There is little ind ication to p ad the antecu bital fossae. (5:362-3) 66. (A) Lead I is d escribed correctly. Lead II is left arm and left leg, lead III is right arm and left leg, V1 is p laced in the 4th intercostal space to the right of the sternum , and V6 is p laced in the 5th intercostal space at the m id -axillary line. (6:1832) 67. (C) The laryngeal m ask airw ay p erm its p ositive pressu re ventilation if the fit and seal of the airw ay are ad equ ate for a p articu lar p atient. It is p ositioned blind ly in a p atient u nd er general anesthesia. If anesthesia d epth is inad equ ate, laryngospasm may resu lt. There is no p rotection against asp iration. (5:555-6) 68. (A) Exhau stion of the carbon d ioxid e absorbent w ill increase the am ou nt of rebreathing of exh aled gas an d in crease th e in sp ired carbon d ioxid e con cen tration . Th is resu lt w ou ld be d isp layed as an increasing baseline on the cap nom eter w aveform . A p u lm onary em bolu s typically prod u ces a su d d en d ecrease in end -tid al CO 2. Wheezing ap p ears as a p rolon ged ap p roach to p lateau d u rin g exp iration. An esop hageal intu bation resu lts in no (or m inim al) m easu red carbon d ioxid e. If the w all oxygen su p p ly is d isconnected , oxygen w ill be su p p lied by the gas cylind ers on the anesthesia m achine, if their valves are op en. If the cylind ers are closed or em p ty, all fresh gas flow from th e com m on gas ou tlet w ill cease. (5:465-6)

69. (D ) In the lithotom y p osition, internal rotation of the legs stretches the com m on p eroneal nerve arou nd the head of the fibu la cau sing a nerve p alsy to be m ore likely. H em od ynam ic stability is preserved because the legs are elevated above the heart. (5:363-5) 70. (E) In an isolated p ow er su p p ly system , an isolation transform er is connected betw een the electric pow er su pp lied by the utility com pany and the electrical outlets in the operating room. The safety provid ed by this system d epend s on the patient and any w ires or m etallic objects in contact w ith the patient not being connected to earth ground . Cond uctive flooring w as used in the p ast to avoid static electricity sparks that cou ld cause fires or explosions in the presence of flam m able anesthetics. (5:375-8) 71. (B) The suction port of a fiberoptic bronchoscope is d esigned p rim arily to asp irate secretions. H ow ever it m ay also be used to ad minister oxygen to a spontaneously breathing patient or to spray local anesthetic in the airway or on the vocal cords. It may also be used to pass the scope over a retrograd e w ire. The patient cannot be ventilated via this port. (5:567-9) 72. (C) Pu lse oxim eters u se tw o w avelengths of light—one red and the other infrared —to estim ate the p ercentage of hem oglobin in the oxygen ated form by ap p lying Beer ’s law that relates absorbance of light by a chem ical to its concentration. Both carboxyhem oglobin and m ethem oglobin are m isinterpreted as oxyhem oglobin by p u lse oxim eters. (1:670-1; 4:877-9; 5:1213-4, 1448-9) 73. (D ) N itrou s oxid e and all of the volatile anesthetics absorb infrared light. N itrogen and oxygen m ay be d eterm ined by m ass spectroscopy or Raman spectroscopy. Oxygen may also be m easu red by electrochem ical or galvanic d etectors. (1:669-70; 4:869-70; 5:1213, 1454) 74. (E) The fail-safe valves on anesthesia m achines are on if oxygen pressure is present. It is important to u nd erstand that this valve is strictly p ressu re-related . As long as there is oxygen p ressu re in the m achine, the valves w ill be on,

Answe rs : 65–75

p erm itting flow of other gases such as nitrou s oxid e, carbon d ioxid e, air, heliu m , nitrogen, etc. It is therefore p ossible to still d eliver a hyp oxic m ixtu re to the p atient even w ith a fu nctioning fail-safe valve. In contrast, new er m achines have p rop ortioning system s that ensu re a m inim u m concentration of oxygen d elivered at the com m on gas ou tlet. (1:569-70; 4:1014; 5:277)

23

75. (D ) A d ecrease in oxygen satu ration m ay be a late m anifestation of venous air em bolism, and shou ld not be the m ethod u pon w hich d etection is relied . The precord ial Doppler, capnom eter, and m ass sp ectrom eter w ill all ind icate the presence of a venou s air em bolism qu ickly and w ith high sensitivity. (1:766; 4:1635-7; 5:2290)

This page intentionally left blank

CHAPTER 3

Circ ulatio n Que s tions DIRECTION S (Qu estions 76-149): Each of the nu m bered item s or incom p lete statem ents in this section is follow ed by answ ers or by com p letions of the statem ent. Select the ON E lettered answ er or com p letion that is BEST in each case. 76. A hypertensive w om an is w ell controlled by am lod ipine and qu inapril, and she is p lanned for cholecystectomy. According to the American College of Card iology/ Am erican H eart Association (ACC/ AH A) Gu id elin es for Periop erative Card iovascu lar Evaluation and Care for N oncard iac Su rgery, her antihyp ertensive m ed ications shou ld be (A) (B) (C) (D)

continued during the perioperative period held on the m orning of su rgery held for 24 h before su rgery rep laced by intravenou s m ed ications on d ay of su rgery (E) red u ced by 50% for 24 h before surgery 77. A 62-year-old w oman is seen in the preoperative clinic p rior to u nd ergoing a total knee replacement. Routine laboratory results include a serum potassium concentration of 5.7 mEq/ L. This could be an effect of w hich one of these med ications? (A) (B) (C) (D) (E)

hyd rochlorothiazid e bu m etanid e ethacrynic acid triam terene chlorthalid one

(A) d ecreases u rine p rod u ction (B) is p rod u ced by the kid neys (C) is often at a norm al level d esp ite heart failu re (D) is a p rod u ct of im m u nological B-cells (E) is a m arker of inflam m ation and m yocard ial d am age in heart failu re 79. A 43-year-old w oman is in the PACU follow ing facial cosmetic surgery w hen suddenly her card iac rhythm changes to supraventricular tachycardia. The PACU resident decides to administer ad enosine. The effects of this med ication usually last for just a few second s, but may be prolonged by the prior administration of (A) (B) (C) (D) (E)

caffeine theobrom ine ep inep hrine d ip yrid am ole ketam ine

80. A 70-year-old m an u nd erw ent coronary su rgery w ith the aid s of card iop u lm onary byp ass an d h ep arin . H is ch an ce of p ostop erative h ep arin-ind u ced throm bocytop enia (H IT) is ap p roxim ately (A) (B) (C) (D) (E)

2% 20% 50% 80% 99%

78. A 58-year-old w om an has a p reop erative B-typ e n atr iu retic p ep tid e (BN P) lev el of 200 p g/ m L fou r years after aortic valve rep lacem ent. This su bstance 25

26

3: Circula tion

81. A 72-year-old w om an takes asp irin and clop id ogrel becau se of the recent insertion of a d ru g-elu ting coronary stent. She has a red u cible ventral hernia that w ill requ ire rep air at som e p oint, bu t the su rgeon w ants her to stop the clop id ogrel a w eek p rior to the su rgery. Based on the 2007 ad visory from the Am erican College of Card iology and the Am erican H eart Association, it is recom m end ed that she d elay elective rep air of her hernia u ntil how long after her stent proced u re? (A) (B) (C) (D) (E)

1 m onth 3 m onths 6 m onths 12 m onths 24 m onths

82. H igh satu ration of hem oglobin w ith oxygen is planned in a p atient w ith sickle cell d isease. A factor that tend s to increase the affinity of hem oglobin for oxygen is (A) (B) (C) (D) (E)

acid osis hyp ertherm ia carbon m onoxid e hyp ercarbia d ip hosp hoglycerol

83. A 70-year-old m an w ith a p alp able p u lsatile abd ominal mass und erw ent CT evaluation and w as fou nd to have an aortic aneurysm 4.5 cm in d iam eter. H is ap p roxim ate annu al risk of ru ptu re is (A) (B) (C) (D) (E)

0.5-5% 10-20% 20-40% 30-50% 50-70%

84. A 25-year-old m an has su ffered hem orrhagic trau m a in a m otor vehicle accid ent. H e bears blood grou p A and can receive (A) (B) (C) (D) (E)

w hole blood of typ e AB red cells of typ e A or O FFP of any typ e red cells of any typ e w hole blood of typ e O

85. A 30-year-old m an has fainted several tim es and has fam ilial hypertrop hic obstructive card iom yop athy. H em od ynam ic goals d u ring anesthesia for ingu inal hernia rep air includ e (A) (B) (C) (D) (E)

d ecreased p reload d ecreased afterload d ecreased contractility increased heart rate low hem atocrit

86. A 60-year-old patient w ith aortic valve area of 0.8 cm 2 is planned for laparoscopic cholecystectomy. In an effort to optimize hemod ynamics in aortic stenosis, the goals generally includ e (A) (B) (C) (D) (E)

low p reload low afterload low heart rate low hem atocrit high p u lm onary vascu lar resistance

87. A 60-year-old m an w ith mean arterial pressu re of 50 m m H g is consid ered for intraaortic balloon cou nterp u lsation. The d evice can be beneficial in the follow ing cond itions EXCEPT (A) (B) (C) (D) (E)

aortic regu rgitation mitral regu rgitation ventricu lar sep tal d efect myocard ial ischem ia myocard ial stu nning

88. A p reop erative p atient is noted to have elevated mean red cell volum e. This could ind icate the follow ing EXCEPT (A) (B) (C) (D) (E)

aplastic anem ia anem ia of chronic liver d isease iron d eficiency anem ia anemia of folate or vitamin B12 deficiency chem otherap y

89. A 50-year-old w ith von Willebrand d isease requires thyroid ectomy. H emostatic aid s might inclu d e the follow ing EXCEPT (A) d esm op ressin (DDAVP) (B) cryop recip itate (C) aminocap roic acid

Que s tions : 81–97

(D) tranexam ic acid (E) vitam in K 90. A critically ill p atient is monitored w ith the aid of a d orsalis ped is arterial catheter after failure of a fem oral catheter. As the pressu re m onitoring site m oves d istally from the aorta the (A) (B) (C) (D) (E)

p u lse p ressu re increases u p stroke is sooner m ean p ressu re d ecreases d iastolic p ressu re increases d icrotic notch is earlier

91. A 65-year-old m an has a type II aortic aneu rysm . In the Craw ford classification of thoracoabd om in al aortic an eu rysm exten t, th e follow ing are true EXCEPT

27

arou nd the baseline, and interm ittent sinu s rhythm with T-wave alternans. This arrhythmia (A) (B) (C) (D) (E)

m ay be elicited by p rocainam id e is a typ e of su p raventricu lar tachycard ia can be cau sed by hyp erm agnesem ia is characterized by a short QT interval is m ore frequ ent in m en than w om en

94. Pain control fails to im p rove the tachycard ia of a p atient in the PACU. At equ ianalgesic d oses, the opioid m ost likely to increase heart rate is (A) (B) (C) (D) (E)

m orp hine m ep erid ine fentanyl su fentanil hyd rom orp hone

(A) type I d oes not involve the renal arteries (B) the renal arteries are involved in types II, III, and IV (C) typ e IV involves the entire d escend ing thoracic aorta (D) typ e II arises near the left su bclavian and reaches the abd om inal bifu rcation (E) typ e III arises in the m id p ortion of the d escend ing thoracic

95. A 60-year-old m an is su sp ected of acu te coronary synd rom e. Myocard ial oxygen d em and m ay be d ecreased by

92. Hypoxia general elicits vasodilation in patients. However, hypoxic vasoconstriction is prominent and clinically important in the

96. A 70-year-old w om an is u nd er anesthesia in p rep aration for coronary su rgery. Coronary p erfu sion p ressu re is increased as a result of

(A) (B) (C) (D) (E)

lu ngs brain heart liver skin

93. A 64-year-old male w ith a past med ical history significant for coronary artery d isease and peripheral vascular d isease is found unresponsive in his bed . It is tw o d ays since he und erw ent right fem oro-popliteal bypass grafting, and his postoperative course had been complicated by ventricular arrhythmias. Upon arrival of the rap id resp onse team , the airw ay is secured , and an ECG is obtained that show s a p olym orp hic tachycard ia that is oscillating

(A) (B) (C) (D) (E)

tachycard ia d ecreased p reload increased afterload increased contractility increased w all tension

(A) increased d iastolic blood p ressu re (B) increased left ventricu lar end -d iastolic p ressu re (C) systolic hyp ertension (D) tachycard ia (E) hyp ocap nia 97. A patient w ith m ild aortic stenosis is u nd ergoing sp lenectom y. At a constant stroke volu m e, card iac outpu t is (A) (B) (C) (D) (E)

p roportional to resistance a linear fu nction of heart rate d ep end ent on blood volu m e related to p otassium concentration inversely proportional to arterial blood p ressure

28

3: Circula tion

(D) 120,000/ m m 3 (E) 1,000,000/ m m 3

98. You r p atient is in norm al sinu s rhythm . The intrinsic rate of the sinoatrial nod e is generally (A) (B) (C) (D) (E)

20 to 40 beats p er m inute 40 to 60 beats p er m inute 70 to 80 beats p er m inute 80 to 100 beats p er m inute 100 to 120 beats p er m inute

DIRECTION S: Use the follow ing figu re to answ er Qu estions 103-106: 1

2

m u l o o

k

e

v

3

(A) (B) (C) (D) (E)

efficacy of p u m p fu nction arrhythm ias ischem ia electrolyte d istu rbance p acem aker fu nction

100. You r p atient has a history of d eep venou s throm bosis and p u lm onary em bolism . An end ogenous inhibitor of intravascu lar coagulation is (A) (B) (C) (D) (E)

factor V Leid en activated p rotein C brad ykinin H agem an factor argatroban

101. A 40-year-old skier is h yp oten sive after a femur fracture and is suspected of fat em bolus. Other clinical m anifestations of fat em boli inclu d e the follow ing EXCEPT (A) (B) (C) (D) (E)

p etechiae hyp oxem ia confu sion brad ycard ia cyanosis

102. A p reop erative p atient is noted to have a low p latelet cou n t. Th e m in im u m n u m ber of p latelets need ed for su rgical h em ostasis is ap p roxim ately (A) 10,000/ m m 3 (B) 50,000/ m m 3 (C) 80,000/ m m 3

S

t

r

99. Electrocard iograp hic m onitoring is ind icated d u ring anesthesia to d etect all of the follow ing EXCEPT

e

Norma l

B C

4

A LVEDP

103. Left ventricu lar end -d iastolic p ressu re is m ost closely app roxim ated by (A) (B) (C) (D) (E)

arterial blood p ressu re central venou s p ressu re p u lm onary artery systolic p ressu re p u lm onary cap illary w ed ge p ressu re right atrial p ressu re

104. The cu rves m arked 3 and 4 in the grap h show n cou ld be seen in p atients (A) (B) (C) (D) (E)

w ith increased contractility on d igoxin or ep inep hrine w ho are in heart failu re w ho have been treated w ith glu cagon w ho resp ond to increased left ventricu lar end -d iastolic p ressu re (LVEDP) w ith increased stroke volu m e

105. If a p atient’s statu s is rep resented by p oint A on cu rve 4 in the grap h show n, then ad m inistration of intravenou s flu id m ay (A) (B) (C) (D) (E)

shift the p atient to the norm al cu rve shift the p atient to p oint B or C take the p atient to p oint B only have no effect shift the p atient to cu rve 2

Que s tions : 98–115

106. Ad m inistration of an inotrop e to a p atient at p oint B on cu rve 4 is intend ed to (A) (B) (C) (D) (E)

shift the p atient to cu rve 3 shift the p atient to p oint A shift the p atient to p oint C m aintain the p atient at p oint B shift the p atient to cu rve 1

107. A 5-foot, 7-inch, 40-year-old m an w eighing 320 p ou nd s is p lanned for lap aroscop ic gastric byp ass. Morbid obesity is associated w ith (A) (B) (C) (D) (E)

d ecreased card iac ou tp u t hyp ertension d ecreased p u lm onary artery p ressu re d ecreased blood volu m e d ecreased card iac w orkload

108. A new born has persistent patency of the d uctus arteriosus. This may close in response to (A) (B) (C) (D) (E)

hyp oxem ia alp rostad il (p rostagland in E1) hep arin ind om ethacin nitrou s oxid e

109. You are h yp erven tilatin g a n eu rosu rgery patient to an arterial Pco 2 valu e of 28 m m H g. The coronary circu lation resp ond s to hyp erventilation w ith (A) (B) (C) (D)

no change an increase in flow a d ecrease in flow a transient increase follow ed by an intense vasod ilatation (E) intense vasoconstriction 110. You r p atient’s blood p ressu re is 160/ 100. The m ean pressu re is approxim ately (A) (B) (C) (D) (E)

110 m m 120 m m 130 m m 140 m m 150 m m

Hg Hg Hg Hg Hg

29

111. A sm all bu bble is noted in the saline-filled p lastic tu bing connected to an arterial catheter of you r patient. The bubble (A) (B) (C) (D) (E)

is not significant lead s to an artificially high read ing affects only the d iastolic p ressu re lead s to a d am p ing of the tracing has a greater effect on the m ean blood p ressu re

112. Und er norm al circu m stances, the p lasm a p rotein (colloid ) osm otic pressu re is (A) (B) (C) (D) (E)

0 mm Hg 5 mm Hg 20 m m H g 50 m m H g 100 m m H g

113. Pu lm onary ed em a m ay resu lt from all of the follow ing EXCEPT (A) (B) (C) (D) (E)

altered p erm eability d ecreased pu lm onary capillary pressu re d ecreased oncotic p ressu re increased negative airw ay p ressu re head injury

114. You encou nter a cyanotic p atient. N et right-toleft shu nt is a featu re of (A) (B) (C) (D) (E)

atrial sep tal d efect ventricu lar sep tal d efect p atent d u ctu s arteriosu s tetralogy of Fallot increased ventricu lar filling p ressu re

115. You notice an increase in heart size before revascularization of a patient und ergoing coronary surgery. If the size of the ventricle increases (A) w all tension need ed to p u m p the sam e am ou nt of blood is less (B) less oxygen is need ed to p u m p blood (C) the heart becom es m ore efficient (D) w all tension w ill increase p rop ortionally w ith the rad iu s (E) w all tension w ill be p rop ortional to w all thickness

30

3: Circula tion

116. You r goal is to m aintain coronary blood flow w hile a p atient w ith coronary d isease u nd ergoes cholecystectom y. The flow (A) (B) (C) (D) (E)

is ind ep end ent of the systolic p ressu re is not affected by hu m oral agents is increased by a slow heart rate is increased by a fast heart rate occu rs alm ost entirely in systole

117. You are m onitoring a p atient w ith the aid of a p u lm onary artery catheter. Pu lm onary artery p ressu re (A) increases p assively w ith increases of card iac ou tpu t (B) rem ains constant w ith change of card iac output (C) is not im p ortant to p u lm onary vascu lar resistance (D) is not d ep end ent on the rad iu s of the vessels (E) d ep end s entirely on card iac ou tp u t 118. You are evalu ating the d egree of congestive heart failu re in a p reop erative p atient w ith d ysp nea on exertion. Increased sym p athetic tone in congestive heart failu re m ay be ind icated by (A) (B) (C) (D) (E)

m em ory loss w eakness fatigu e confu sion anxiety

119. A 56-year-old fem ale is u nd ergoing left p neu m onectom y for lung cancer. A few hou rs after the su rgery, the patient is noted to have progressive hyp oxem ia, and hyp otension requ iring vasop ressor su p p ort. A stat transthoracic echocard iogram obtained at the bed sid e is suggestive of right-heart failure. The next best step in m anagem ent w ou ld be (A) ad m inistration of selective p u lm onary vasod ilators (B) afterload red u ction w ith infu sion of intravenou s nicard ip ine

(C) end otracheal intu bation and high levels of positive end -expiratory pressu re (PEEP) (D) a red u ction in FIO 2 (E) end otracheal intu bation and p erm issive hyp ercarbia 120. Card iop u lm onary byp ass is initiated in a 70-year-old man planned for coronary su rgery. Unexp ected hyp otension (m ean aortic p ressu re of 30 m m H g) is rap id ly encou ntered and m ay be d u e to all of the follow ing EXCEPT (A) (B) (C) (D) (E)

hem od ilu tion d ecreased catecholam ines aortic stenosis p ersistent system ic to p u lm onary shu nt aortic d issection

121. Your patient requires cardiopulmonary bypass. Myocard ial p reservation d u ring that p roced u re m ay includ e all of the follow ing EXCEPT (A) m aintaining the ventricle in a d istend ed state (B) card iop legia (C) nearly continu ou s p erfu sion (D) avoid ance of ventricu lar fibrillation (E) maintaining the heart at a low temperature 122. Follow ing breast surgery under general anesthesia, your patient in the recovery room exhibits elevated blood pressure read ings of about 190/ 110 mm H g. Your approach should be (A) treat the blood p ressu re w ith sm all d oses of an antihypertensive m ed ication (B) d o nothing, bu t w ait to see if the hyp ertension is a transient problem associated w ith em ergence from anesthesia (C) exam ine the p atient for evid ence of hyp oxia or hyp ercarbia (D) recheck the blood p ressu re you rself to m ake su re the cu ff is the correct size (E) arrange a card iology consu ltation

Que s tions : 116–129

123. A 97-year-old w om an u nd ergoes bow el resection und er general anesthesia. Card iovascu lar changes that occur w ith ad vancing age are (A) (B) (C) (D) (E)

d ecreasing blood p ressu re increase in card iovascu lar reserve loss of elasticity of the vascu lar tree increased nu m ber of m yofibrils increase in card iac ou tp u t

124. Poor right ventricular contractility is noted in the op erating room soon after m itral valve rep lacem ent. The right ventricle (A) has little fu nction in the ad u lt p atient (B) is very easily cooled in the p atient u nd ergoing card iopu lm onary byp ass (C) can overcom e p u lm onary vascu lar resistance very effectively, and , therefore, is not of concern in term inating card iop u lm onary bypass (D) is m ore likely to be inju red by intracoronary air than is the left ventricle d u ring card iopulm onary bypass (E) is u naffected by PEEP 125. A preoperative patient is planned for su rgery promp ted by w orku p of hypertension. A cause of system ic hypertension that is am enable to su rgical correction is (A) (B) (C) (D) (E)

essential hyp ertension second ary ald osteronism renal p arenchym al d isease p heochrom ocytom a long-stand ing renal artery stenosis

DIRECTION S: Use th e follow in g scen ario to answ er Qu estions 126–127. A 24-year-old fem ale is ad m itted for evalu ation of chest p ain. The p ain is sharp and aggravated by breathing. A chest x-ray show s an enlarged heart, ECG show s d ecreased voltage, and there is a p ericard ial friction ru b. Pericard iocen tesis d em onstrates p u ru lent flu id . Serial x-rays show w id ening of the card iac shad ow, and the p atient is sched u led for p ericard ial d rainage.

31

126. On p hysical exam ination, you w ou ld expect to find (A) a w ater-ham m er p u lse (B) an increased card iac ou tp u t (C) d istend ed neck veins that flatten in the sitting position (D) d ecreased arterial p ressu re (E) low central venou s p ressu re 127. A p u lsu s p arad oxu s is a p ulse that has the follow ing p rop erties EXCEPT that it (A) red u ces in am p litu d e >10 m m H g on insp iration (B) can occur in hem orrhage and in severe obstru ctive lung d isease (C) is an exaggeration of the norm al respiratory effect on the arterial p ulse (D) is stronger on inspiration (E) is associated w ith tam ponad e, p ulm onary em bolism , and tension p neum othorax 128. A 68-year-old is noted to have an abnorm al Allen test before card iac surgery. The test is (A) u sed to assess ad equ acy of rad ial artery p erfu sion (B) of little p red ictive valu e (C) p ositive if good flow occu rs (D) negative if good flow occu rs (E) ind ep end ent of hand p osition 129. Central venou s cannu lation is ind icated in all the follow ing proced u res EXCEPT (A) a su rgical p roced u re in w hich there is an u nu sual position, e.g., head -d ow n p osition (B) p atients in shock (C) total p arenteral hyp eralim entation (D) intravenou s ad m inistration of vasop ressors (E) inad equ ate venou s access in a p atient w ith extensive bu rns

32

3: Circula tion

130. A 42-year-old patient w ith a p ast m ed ical history of insu lin d ep end ent d iabetes m ellitu s and end -stage renal d isease requ iring hem od ialysis is ad m itted to the intensive care u nit w ith signs and sym p tom s of sep tic shock second ary to com m u nity acqu ired p neu m onia. The p atient has very p oor p erip heral access, an d a d ecision for p lacem en t of a central venou s catheter, as w ell as an arterial catheter for blood p ressu re m onitoring, is m ad e. With resp ect to p otential sites of vessel cannu lation it is tru e that (A) chylothorax is com m on w ith right internal ju gular cannu lation (B) brachial p lexu s trau m a is com m on w ith the antecubital approach for central venou s cannu lation (C) the risk of p neu m othorax is low w ith the su bclavian approach (D) the Allen test rules out ischemic complications related to a rad ial arterial catheter (E) the d orsalis p ed is is not the artery of choice for blood pressure m onitoring in a patient w ith d iabetes 131. You are asked by a member of the orthopedic team to d o a preop erative evalu ation on an 87-year-old fem ale ad m itted from the em ergency department for left femur fracture. The patient has a heart murmur, and an echocard iogram obtained by the medical consultant shows aortic stenosis w ith a valve area of 0.6 cm 2. The patient w ith severe aortic stenosis usually has (A) a rap id ly d eteriorating cou rse once sym ptom s are present (B) a large left ventricu lar cavity (C) low voltage criteria on the electrocard iogram (D) p rotection against ischem ia d u e to the large ventricle (E) a very com p liant ventricle 132. A brid ge-to-transplant patient w ith biventricular heart failu re is stabilized by p lacem ent of a left ventricu lar assist d evice and by ad m inistration of inhaled nitric oxid e. The inhaled d ru g has the follow ing p rop erties EXCEPT that it

(A) m ay cau se m ethem oglobinem ia (B) often low ers m ean p u lm onary artery p ressu re (C) exhibits virtu ally no system ic vasod ilation (D) often d ecreases system ic oxygenation (E) increases vascu lar sm ooth m u scle cyclic GMP 133. Follow ing card iac su rgery, a patient is noted to exhibit equ alization of d iastolic filling p ressu res (central venou s, right ventricu lar d iastolic, pu lm onary artery d iastolic, pu lm onary capillary w ed ge) that tend s to occu r in (A) (B) (C) (D) (E)

m itral stenosis p u lm onary em bolu s tricu sp id regu rgitation card iac tam p onad e atrial fibrillation

134. Vitam in K (A) is requ ired for synthesis of fu nctional clotting factors VII, IX, X, and II (p rothrom bin) (B) antagonizes hep arin (C) p otentiates w arfarin (D) antagonizes p rotam ine su lfate (E) is a p rom inent citru s p rod u ct 135. A p atient in renal failu re is receiving erythropoietin in p rep aration for card iac surgery. This substance (A) (B) (C) (D) (E)

is norm ally p rod u ced by the liver increases p latelet p rod u ction increases red blood cell p rod u ction increases w hite blood cell p rod u ction is a breakd ow n p rod u ct of hem e

136. A 20-year-old tachycard ic p atient im p roves follow ing an intravenou s d ose of ad enosine. The d ru g (A) inhibits p hosp hod iesterase (B) is inactivated by catechol O-m ethyltransferase (C) d ecreases atrioventricu lar cond u ction

Que s tions : 130–145

(D) is a β -ad renergic agonist (E) is p rim arily cleared by the kid neys 137. A p reoperative 3-year-old exhibits ”tet sp ells.” An agent that may red u ce right-to-left shu nt in tetralogy of Fallot is (A) (B) (C) (D) (E)

sod iu m nitrop ru ssid e labetolol p rop ranolol p henylep hrine d iltiazem

138. Seven years after heart transp lantation, a 60-year-old w om an is p lanned for ap pend ectom y. Th e transp lanted , d en ervated h eart resp ond s to (A) (B) (C) (D) (E)

circu lating catecholam ines circu lating acetylcholine neu rom u scu lar blocking agents vagal stim u lation m u scarinic blocking agents

139. A 58-year-old m an w ith chest p ain exhibits elevation of the ST segments of precord ial ECG lead s. This find ing can occu r in all of the follow ing cond itions EXCEPT (A) (B) (C) (D) (E)

Prinzm etal’s angina (coronary sp asm ) acu te p ericard itis m yocard ial infarction m yocard ial contu sion d u ring norm al exercise in an ind ivid u al w ithou t coronary d isease

140. You con sid er verap am il for a tach ycard ic patient. The d ru g m ay exacerbate all of the follow ing d iseases or sym p tom s EXCEPT (A) (B) (C) (D) (E)

sick sinus synd rom e atrial fibrillation atrioventricular block Wolff–Parkinson–White synd rom e congestive heart failure

141. You m easu re card iac ou tp u t in ord er to calcu late system ic vascu lar resistance in a sep tic patient. Card iac ou tp ut is

33

(A) (B) (C) (D)

id entical in the left and right heart u su ally 2 to 3 L/ m in in a 70-kg m an u naffected by blood volu m e the p rod u ct of heart rate and stroke volum e (E) the p rod u ct of heart rate and m ean arterial p ressu re 142. You consid er red u ction of m yocard ial contractility in a card iac p atient. Contractility is d ecreased by (A) (B) (C) (D) (E)

sym p athetic stim u lation p arasym p athetic tone m ilrinone ad m inistration norep inep hrine ep inep hrine

DIRECTION S: Each grou p of item s below consists of lettered head ings follow ed by a list of num bered p hrases or statem ents. For each nu m bered p hrase or statem ent, select the ON E lettered head ing or com p onent that is m ost closely associated w ith it. Each lettered head ing or com p onent m ay be selected once, m ore than once, or not at all. (A) (B) (C) (D) (E) (F) (G) (H )

atrial fibrillation hypertrophic obstructive cardiomyopathy p ericard itis aortic regu rgitation p u lm onary em bolism Eisenm enger synd rom e tricu sp id regu rgitation m yocard ial infarction

For each p atient w ith a card iac sym p tom or sign, select the m ost likely m echanism cau sing the sym ptom s or sign. 143. A 50-year-old m an has d ysp nea on m ild exertion, bound ing pulses, and a d iastolic murmu r. 144. A 40-year-old w om an w ith a long-stand ing heart m u rm u r has shortness of breath, cyanosis, and a systolic heart m urm ur 145. A 20-year-old m an becom es p rofou nd ly hypotensive after m od est intraoperative bleed ing.

34

3: Circula tion

146. A 70-year-old w om an w ith an irregu lar p u lse su ffers sud d en loss of the right low er quad rant of the visual field . 147. A 25-year-old heroin u ser h as p erip heral ed em a. 148. A 70-year-old w om an d evelop s acu te d ysp nea one w eek after hip rep lacem ent. She is

tach yp n eic an d cyan otic. A ch est x-ray is u nrem arkable. 149. A 20-year-old man has chest pain, a friction rub on chest au scu ltation, and J-point elevation on m ost ECG lead s.

Answe rs a nd Expla na tions

76. (A) Antihypertensives are recom m end ed to be continu ed . In p articu lar, beta-blockers and clonid ine are continu ed becau se of the potential for significan t w ith d raw al synd rom es. (5:743) 77. (D ) H yperkalemia is associated w ith amilorid e, sp ironolactone, and triam terene. The other choices tend to d ecrease potassiu m . (5:743) 78. (E) A low BN P level m ight be u sefu l to select ou t high-risk patients w ho d o not need echocard iogram s to rule ou t ventricu lar d ysfunction. Preoperative levels may predict myocard ial inju ry and hosp ital length of stay. It m ay be a u sefu l m arker of p eriop erative d eterioration. (5:109) 79. (D ) Dip yrid am ole blocks reu p take of ad enosine, d elaying its clearance and p otentiating its effects. Caffeine and theobrom ine are com p etitive inhibitors of ad enosine action. (5:765) 80. (A) The frequ ency of H IT after card iac su rgery is abou t 2%. H ow ever, 25-50% of p atients d evelop hep arin-d ep end ent antibod ies p ostoperatively over the first 5-10 d ays after exp osure. If this im m u nological response strongly activates p latelets and coagu lation, it cau ses the p rothrom botic d isord er know n as H IT. Antibod y assays d o not necessarily ind icate H IT, but negative assays are helpfu l to rule out the p roblem . (5:900) 81. (D ) Antip latelet therap y is m ore im p ortant than tim e betw een stent p lacem ent and su rgery in red u cing stent throm bosis. Althou gh

d u al an tip latelet con tin u ation for 12 m on th s is m an d atory, clop id ogrel treatm en t beyon d 12 m onths m ay not be necessary. (5:106) 82. (C) The other factors facilitate release of oxygen from hem oglobin. (5:460) 83. (A) The first fou r percentages are respectively for 4.0-4.9, 6.0-6.9, 7.0-7.9, and >8 cm . (5:1024) 84. (B) A patient bearing blood group A can receive w hole blood typ e A, red cells of typ e A or O, p lasm a of type A or AB, any cryop recipitate or p latelets of any type. (6:951) 85. (C) Intraoperative hemod ynam ic goals inclu d e increased p re-, and , afterload , and d ecreased contractility, and heart rate. (5:910) 86. (C) H igh p re- and afterload and low heart rate can be salutary, as long as the low rate d oes not com prom ise card iac outpu t. (5:910) 87. (A) The balloon w orsens aortic regu rgitation. (6:2235-6) 88. (C) The anemia of iron d eficiency is microcytic. (5:199) 89. (E) The von Willebrand factor is not sensitive to vitam in K. The sensitive factors are II, VII, IX, X, C, and S. (5:205, 6:980-1) 90. (A) Th e m ean p ressu re is fairly u n iform throughout the arterial tree, but the pulse pressu re is higher in the foot than in the w rist or aorta. (5:406-7)

35

36

3: Circula tion

91. (C) Type IV arises at the level of the d iaphragm. (5:917-8) 92. (A) The vasoconstrictor response of the pu lmonary circu lation d iffers from that of other tissu es in ord er to m atch lu n g p erfu sion to ventilation. (5:967-9) 93. (A) The d escription of the arrhythm ia is consistent w ith torsad es d es pointes. The phrase w as taken from a ballet-d ancing m aneu ver. It is a typ e of p olym orp hic ventricu lar tachycard ia ch aracterized by a lon g QT in terval. Procainam id e can prolong the QT interval and elicit torsad e. Magnesiu m can be therap eu tic. The arrhythm ia occu rs m ore frequ ently in w om en than in m en. (6:54, 175-6, 1884, 1891) 94. (B) Meperidine tends to decrease cardiac output and increase heart rate. Other µ -receptor agonists tend to d ecrease heart rate. (1:715-6) 95. (B) Myocard ial oxygen con su m p tion is increased by tachycard ia. It cau ses increased d em and for oxygen at the sam e tim e that it lead s to d ecreased oxygen su p p ly by d ecreasing coronary blood flow. Increased afterload lead s to increased w all tension, both of w hich requ ire m ore oxygen. Increases in contractility requ ire m ore oxygen. (5:903) 96. (A) Coronary p erfu sion (CPP) occu rs m ostly d u ring d iastole. CPP = DBP − LVEDP, w here DBP is d iastolic blood p ressu re and LVEDP is left ventricu lar end -d iastolic p ressu re. As DBP rises or LVEDP falls, the flow w ill increase. Tachycard ia d ecreases tim e of perfusion, since d iastole is shortened . Systolic hyp ertension d ecreases p erfu sion, since the ventricle con tracts hard er, allow ing less p erfu sion d u ring systole. (5:902) 97. (B) At a constant stroke volume, card iac output is a linear fu nction of heart rate. The other factors are im portant in the generation of card iac outpu t, bu t they are d irect or ind irect d eterm inants of stroke volu m e: resistance (afterload ), arterial pressu re, and blood volum e (p reload ). Card iac ou tp u t is not d irectly related to p otassiu m concentration. (6:1856)

98. (C) The intrinsic rate of the sinoatrial nod e is generally 70 to 80 beats per m inu te. Since it has a faster rate, it is the d om inant pacem aker. The m ore cau d al a p acem aker cell is located in the cond u ction system, the slow er its intrinsic rate. The atrioventricu lar nod e hasan intrinsic rate of 40 to 60 beats p er m inu te. (6:1867-71) 99. (A) The ECG cannot d etect efficacy of pu m p fu nction. The ECG m ay show a norm al sinu s rhythm d u rin g p u lseless electrical activity (electrom ech an ical d issociation ) w h en th e p u m p is not w orking effectively at all. The ECG can also be observed for changes ind icative of electrolyte d istu rbance, esp ecially of calciu m and p otassiu m . Ischem ia m ay be d etected by changes in the tracing. Pacem aker function may be checked by observing the tracing w hile tu rning the p acem aker on and off (e.g. w ith a m agnet). Arrhythm ias are d etected by the ECG. (5:89) 100. (B) En d oth elial th rom bom od u lin con verts intravascular thrombin from a clotting enzyme into an activator of p rotein C. The activated protein C has anticoagulant activity through its d estru ction of factors V and VIII. Factor V Leid en is a single p oint m u tation that m akes the m olecu le resistant to d estruction by activated protein C and therefore increases the risk of intravascu lar throm bosis. Argatroban is a synthetic inhibitor of throm bin. Activated H agem an factor initiates the intrinsic blood clotting cascad e and activates prekallikrein to p rod u ce brad ykinin. (1:860, 883) 101. (D ) Fat em boli m ay be seen w ith fractu res. Fat em bolism synd rom e is associated w ith p etechiae, hyp oxem ia, confu sion and cyanosis. Tachycard ia is often p resent. (5:1203) 102. (B) The m inim u m nu m ber of p latelets need ed for norm al coagu lation is controversial, bu t is frequ en tly cited as 30,000–50,000/ m m 3. (5:1446) 103. (D ) Pu lmonary capillary w ed ge p ressu re is the best estim ate of left ventricu lar end -d iastolic p ressu re. By m easu ring changes in card iac ou tp u t and p ressu res, a ventricu lar fu nction

Answe rs : 91–117

curve can be d raw n and therap eu tic interventions evaluated . Right heart pressu res d o not necessarily p rovid e accu rate estim ates of left heart pressures and function. Therefore, central venous pressu re gives the w orst approxim ation of left sid e fu nction. (5:414-5) 104. (C) The cu rves are rep resentative of p atients w ho are not resp ond ing to increases in LVEDP w ith increases in stroke volu m e. These d o not show any effect of increased contractility or any effect of inotrop ic intervention. In su ch a p atient, inotrop e ad m inistration m ay be the next logical step. (1:792) 105. (B) A p atient on su ch a low cu rve m ay respond to fluid by ad vancing to point B, but the patient m ay also react ad versely and fall even farther d ow n on the curve. This w ould be represented by p oint C. (1:792) 106. (A) The inotrope ought to improve contractility. (1:792) 107. (B) Severe hyp ertension is seen in 5% to 10% of patients w ith morbid obesity. Mod erate hypertension is seen in 50% of p atients. The card iac ou tpu t is increased , as is the pu lm onary artery p ressu re. Blood volum e is increased , and the card iac w orkload is increased . (5:305) 108. (D ) The d u ctu s norm ally closes in resp onse to elevated arterial oxygen tension and p atency may be maintained deliberately with alprostad il (PGE1). (5:1181) 109. (C) Coronary circu lation reacts to carbon d ioxid e in a m anner sim ilar to cerebral circu lation. Although this m ay be salu tary in the instance of head trau m a, in the patient w ith chest pain and ischem ia, hyp erventilation m ay lead to fu rther d ecrease of coronary flow. The m agnitud e of the d ecrease is not large. (5:902) 110. (B) An estimate for mean arterial pressure is: MAP = d iastolic + (systolic − d iastolic)/ 3. (5:1369) 111. (D ) The pressure w aveform may be affected by the presence of even small bubbles. The bubble,

37

being very com p liant, lead s to a d am p ing of the trace and a read ing that hovers arou nd the m ean. Both systolic and d iastolic pressure w ill be affected . (5:407) 112. (C) The p lasm a oncotic p ressu re is im p ortant in maintaining the fluid balance in the capillaries. The balance betw een the d riving p ressu re and th e on cotic p ressu re p reven ts tissu e ed em a. The d elicate balance can be d istu rbed by either increases or d ecreases in p ressu re or increases or d ecreases in oncotic p ressu re. (5:338-9) 113. (B) Pu lm onary ed em a resu lts from an increase in p u lm onary cap illary p ressu re. Pu lm onary ed em a m ay also resu lt from n eu rological inju ry. N egative airw ay p ressu re p u lm onary ed em a has been rep orted in the p atient w ith airw ay obstru ction w ho is breathing against a closed glottis or an occlu d ed end otracheal tube. (5:422-3) 114. (D ) There is increased p u lm onary blood flow in the other lesions. (5:928-32) 115. (D ) As the size of the ventricle increases, the radius increases, and the w all tension increases. The law of LaPlace states that m ore tension is need ed to generate the sam e p ressu re as the rad iu s increases. This m akes the heart m ore inefficient and requ ires m ore oxygen to p u m p blood . (1:749 ) 116. (C) Coronary p erfu sion is im p roved by slow heart rates, since m ost perfu sion occu rs d uring d iastole. The coronary flow is not ind epend ent of systolic pressure, since areas of the subend ocard iu m are p oorly p erfu sed d u ring systole. (6:1998) 117. (A) As the card iac ou tp u t increases, p u lm onary artery p ressu re increases. Other factors also are involved in pulmonary artery pressu re, e.g., the rad iu s of the vessels. Vasoconstriction and vasod ilatation of the p u lm onary vessels occu r w ith changes of card iac ou tpu t to regu late p u lm onary vascu lar resistance. (5:412-15, 932, 969)

38

3: Circula tion

118. (E) The anxiety seen in p atients w ith congestive heart failu re is d u e to increased sym p athetic activity. The increased sym p athetic activity is a com pensatory m echanism , and it is not d irectly related to the low flow state, as are the other op tions. Mem ory loss, w eakness, fatigu e, and confu sion are d irectly cau sed by low ou tp u t. (6:1904-5) 119. (A) H yp oxia, hyp ercarbia, and PEEP increase the afterload of the failing right heart. After pneu monectom y, a healthy right ventricle m ay fail w hen su d d enly challenged w ith increased afterload d ue to the d ecrease in cross-sectional area of the p ulm onary vascu latu re, and resu lting pulmonary hypertension. Selective pulm onary vasod ilators su ch as inhaled nitric oxid e or inhaled epoprostenol can be useful to red uce PVR. (5:1005) 120. (C) H ypotension persisting after initiation of byp ass m ay be a sign of aortic insu fficiency. The initial hem od ilu tion m ay cau se d ecreased catecholam ines and d ecreased viscosity. These shou ld be transient. If p ersistent, one m u st look for other problem s, e.g., a persistent shunt or aortic d issection. (5:923) 121. (A) Allow ing the ventricle to becom e overd istend ed can be d etrim ental, and m ost centers u se a ventricu lar vent to d ecom p ress it. The ventricle should be kept in a nonbeating state and kep t cold to low er oxygen consu m p tion. Fibrillation sh ou ld be avoid ed , sin ce th e oxygen cost is great. (5:908) 122. (C) The recovery room p atient w ith hyp ertension shou ld be exam ined to rule out hypoxia or hyp ercarbia. If p resent, these m u st be treated , and this takes preced ence over other consid erations. After you have ruled out those problem s, you can p roceed w ith a m ethod ical assessment of the problem: retake the pressure, and if it is not life threatening, you m ay w ant to w atch it or treat it w ith a d ru g. In som e cases, a consu ltation m ay be in ord er. (5:1281, 1291-2) 123. (C) The vascu lar tree becom es less elastic w ith age. Blood p ressu re rises w ith age that m ay be

a reflection of th e loss of elasticity. Th e nu m ber of m yofibrils d ecreases, and the card iac ou tp u t d ecreases. Th e card iovascu lar reserve d ecreases as a reflection of the other changes. (5:279-80) 124. (D ) Righ t ventricu lar fu nction is receiving mu ch more attention, and the interd epend ence of the tw o ventricles is being ap preciated to a greater d egree. Blood flow occu rs d u ring systole and d iastole. Since there is good collateral flow, it is hard er to cool the right ventricle. Pu lm onary vascu lar resistance is hard to overcom e, since the ventricle m u scle is not as w ell d evelop ed . PEEP w ill be transm itted to the righ t ven tricle, d ecreasin g ven ou s retu rn . (5:924; 6:2076) 125. (D ) Pheochrom ocytom a is often am enable to su rgical correction. Other cau ses of second ary hyp ertension, second ary ald osteronism and renal parenchymal d isease, may not be so amenable. The p atient w ith renal artery stenosis m ay be treated su rgically if the cond ition is d iagnosed early. If later, su rgery m ay be of no help . (5:1125) 126. (D ) A w ater-ham m er p u lse is seen in the p atient w ith aortic insu fficiency. The p atient w ith p ericard ial effu sion w ill exhibit p u lsu s p arad oxu s. The card iac ou tp u t is d ecreased . The neck veins are d istend ed and stay d istend ed in the su pine position, since little drainage can occu r. Blood p ressu re w ill d ecrease. (6:2221) 127. (A) Pu lsu s p arad oxu s is an exaggeration of the norm al variation in the p u lse w ith insp iration. The pu lse red u ces in am plitu d e on inspiration. (6:1824-5) 128. (B) The Allen test is u sed to assess the ad equacy of u lnar artery flow w hen cannu lating the rad ial artery. Stu d ies have show n poor correlation w ith su bsequ ent problems. The reason for this is that m any of the p roblem s are em bolic in natu re that cannot be p red icted w ith an Allen test. In ad d ition to the lack of d ata show ing correlation, there is confu sion as to w hat is p ositive or negative. It is better to

Answe rs : 118–141

d escribe the result of the test. Since the position of the w rist is im p ortant, one m ust ascertain that the w rist is not hyper extend ed w hen perform ing the test. (5:409) 129. (A) A central venou s line is not ind icated because the patient is in an abnormal position. It is in d icated in p atien ts w ith sh ock. Hyperalimentation, inotropic drugs, and irritating solutions should be infused through a central venou s catheter. Patients w ith lack of p erip heral large bore IV access, w ho are expected to require substantial fluid resuscitation, such as bu rn p atients, shou ld und ergo placement of a central venous catheter. (5:412) 130. (E) Pneumothorax is a common problem w ith subclavian cannulation. Chylothorax is a comp lication of left internal ju gu lar cannu lation. There should be no brachial plexus trauma with the antecubital approach. Allen’s test is not a foolproof method to determine the patency of the ulnar arch. The d orsalis ped is artery should be avoid ed in the p atient w ith d iabetes or peripheral vascular d isease. (5:408-9) 131. (A) The p atient w ith aortic stenosis has a rap idly d eteriorating course once angina, syncope, and congestive heart failu re occu r. The size of the ventricu lar m uscle m ass increases, but the cavity size d oes not change. The increased size also rend ers the m u scle m ore p rone to isch em ia. Fast heart rates w ill cause a low card iac ou tp u t, since ventricu lar filling w ill be com p rom ised . In ad d ition, coronary p erfu sion w ill su ffer. Low heart rates can also be d evastating, since the stroke volu m e is fixed . The m ost com m on problem is that of tachycard ia. (5:63) 132. (D ) Inhaled nitric oxid e (N O) d ilates blood vessels in ventilated regions of the lu ngs and can thereby im p rove the m atching of p erfu sion to ventilation. While intravenou s vasod ilator s can w or sen v en tilation :p er fu sion m atch in g an d so w orsen system ic arterial oxygen ation , in h aled n itr ic oxid e often im p roves system ic oxygen ation . In h aled nitric oxid e exhibits little or no system ic vasod ilation becau se th e m olecu le is rap id ly d estroyed in the system ic circu lation. Reaction

39

w ith hem e is one m echanism of N O d estru ction, and hem oglobin oxid ation occu rs concom itantly. In m ost clinical situ ation s, N O benefits occu r at d oses that d o not cau se significant m ethem oglobin levels. (1:558-9; 5:969) 133. (D ) The filling pressu res are elevated and tend to equalize in pericard ial constriction and pericard ial tam ponad e. (5:458) 134. (A) The synthesis of vitamin K-d epend ent clotting factors is inhibited by w arfarin. Protam ine antagonizes hep arin. (1:860-1) 135. (C) The anem ia of chronic renal failu re stem s from d iminished prod u ction of erythropoietin, a p rotein available for therap eu tic p u rp oses. (1:1068-70) 136. (C) Ad enosine m ay norm ally regu late coronary blood flow. It m ay interru p t reentrant su p raven tricu lar tachycard ia. It is not an ad renergic catechol. It is inactivated by ad enosine d eam inase. (1:834) 137. (D ) Increased peripheral resistance may red uce cyanosis in tetralogy of Fallot. (5:928) 138. (A) Because of esterase activities of the blood, th ere is n o circu latin g acetylch olin e. β -ad renocep tor antagonists slow the transplanted heart, but antimuscarinic agents do not speed the transplanted heart. (5:1093) 139. (E) With the exception of normal exercise, all of the cond itions listed can cau se ST elevation. In the case of transm ural ischem ia and infarction, this m ay be follow ed by the occu rrence of Q-w aves. Electrocard iographic changes associated w ith norm al exercise m ay inclu d e d ow nw ard d isplacement of the J point. (6:1831, 1836, 1971) 140. (B) Verap am il m ay increase the heart rate in case of su p raventricu lar tachycard ia d u e to WPW synd rom e. (1:833) 141. (D ) Card iac ou tp ut is a fu nction of m any com p onents of the circulatory system . The typ ical ou tp u t is 5 L/ m in in a healthy resting ad u lt.

40

3: Circula tion

Ou tp ut is affected by blood volu m e. Card iac ou tpu t is the p rod uct of heart rate and stroke volu m e. Com m u nications of the bron chial artery cap illaries w ith those of the pu lm onary veins is a reason for left ventricu lar ou tp u t exceed ing the right. Left-to-righ t sh u n ting through an atrial septal d efect cou ld increase right ou tp u t over left. (5:418-22, 604) 142. (B) Myocard ial contractility is affected by both neu ral and hu m oral factors. Parasym pathetic tone and m yocard ial ischem ia both d ecrease con tractility, w h ereas sym p ath etic stim u lation and inotrope ad m inistration increase contractility. (1:178; 6:1804)

initially causing right-to-left shunting. Examples inclu d e sep tal d efects and p atent d u ctu s. (6:1923-4) 145. (B) In th is card iom yop ath y, ven tricu lar obstruction is w orsened by hypovolem ia and by sym p athetic tone and can be relieved by beta-ad renergic blockers, phenylephrine, and / or volum e ad m inistration. (6:1968-9) 146. (A) Throm bi often form in the left atriu m d u ring fibrillation, and these can em bolize. (6:3274-5) 147. (G) The tricu sp id valve m ay have been d am aged by end ocard itis. (6:1930, 1948)

143. (D ) The p atient m ight have regu rgitation cau sed by an ascend ing aortic aneu rysm . (6:2061-3)

148. (E) The embolic source is likely venous thrombi of the legs. (6:2090, 2173)

144. (F) The cyanosis ind icates a right-to-left shunt. This could be the result of increased pulmonary vascu lar resistan ce in resp on se to a lesion

149. (C) The p roblem can follow resp iratory infections and m ay resp ond to COX inhibitors. (6:1830, 1971-3)

CHAPTER 4

Re s piratio n Que s tions DIRECTION S (Qu estions 150-235): Each of the num bered item s or incom plete statem ents in this section is follow ed by answ ers or by com p letions of the statem ent. Select the ON E lettered answ er or com pletion that is BEST in each case. 150. At functional resid u al cap acity (FRC) (A) no fu rther insp iration is p ossible (B) chest w all elastic forces are greater than the lu ngs elastic recoil (C) the p ressu re d ifference betw een the alveoli and the intrap leu ral sp ace is zero (D) the total p u lm onary vascu lar resistance is very high (E) the p ressu re d ifference betw een the alveoli and atm osp here is zero 151. All statem ents in regard to the sensation of d yspnea are correct EXCEPT d yspnea m ay be: (A) influ enced by chem orecep tors that are located in the m ed u lla (B) m ed iated by J-recep tors d u ring p u lm onary ed em a (C) d u e to hyp erinflation of the lu ng (D) m ed iated by m etaborecep tors located in m u scle (E) d u e to low card iac ou tp u t in ind ivid u als w ith obesity 152. During mechanical ventilation proximal airway pressure w ill increase w ith (A) d ecreases in tid al volu m e (B) increases in low er resp iratory system com pliance (C) increases in resp iratory flow (D) d ecreases in PEEP (E) d ecreases in airw ay resistance

153. A 60-year-old man w ith COPD and an 80-packyear sm oking history is sched u led to u nd ergo p u lm onary fu nction testing. Measu rem ents that can be obtained by spirom etry inclu d e all of the follow ing EXCEPT (A) (B) (C) (D) (E)

tid al volu m e resid u al volu m e exp iratory reserve volu m e insp iratory reserve volu m e vital cap acity

154. A d ou ble-lu m en tu be is u sed for anesthesia in p atients w ith severe (A) (B) (C) (D) (E)

asthm a hem op tysis em p hysem a heart failure tracheal stenosis

155. Carbon d ioxid e transp ort involves all of the follow ing EXCEPT (A) (B) (C) (D) (E)

w ater bicarbonate ion carbonic anhyd rase hem oglobin carboxyhem oglobin

156. In the norm al u p right lu ng (A) the blood flow is greatest at the ap ex (B) the ventilation is greatest at the ap ex (C) the ventilation/ p erfu sion (V/ Q) ratio is higher at the ap ex (D) ventilation is u niform (E) the P O 2 is low er at the ap ex com p ared to the base 41

42

4: Re s pira tion

157. A 57-year-old w oman is und ergoing thoracotomy for resection of her left low er lobe because of a small-cell tumor. She has a right-sid ed d ouble-lumen endotracheal tube in place. All of the following are effective ways to improve oxygenation d uring one-lung ventilation EXCEPT (A) lu ng recru itm ent m aneu ver in the d epend ent lu ng (B) d ecrease blood flow in the d ep end ent lu ng, i.e., p lacem ent of a ligatu re on the p u lm onary artery (C) increase blood flow and p erfu sion to the d epend ent lu ng (D) increase Pao 2 in the nond ep end ent lu ng (E) PEEP to the d ep end ent lu ng 158. If one m easu red p leu ral p ressu re in a stand ing hu m an, one w ou ld find that the p ressu re w as (A) (B) (C) (D) (E)

highest at the ap ex of the lu ng highest at the base of the lu ng equ al at all levels u nrelated to bod y p osition com p letely u np red ictable from one level to another

159. The LaPlace law is im p ortant in p u lm onary physiology because it d escribes (A) the p rop erties of gas m ixtu res (B) the angles of the bronchi (C) the bu cket hand le m ovem ent of the ribs d u ring ventilation (D) the p ressu re relationship s w ithin the alveoli (E) resistance in large airw ays 160. Surfactant is a su bstance that (A) is p rod uced in the liver of the new born (B) is im portant in new borns bu t has little im p ortance in the ad u lt (C) is p rod uced by the basem ent m em brane of the lu ng (D) low ers surface tension in the alveoli (E) is a long-chained carbohyd rate m olecu le

161. Th e fu n ction al resid u al cap acity (FRC) is d efined as the com bination of (A) tid al volu m e and resid u al volu m e (B) tid al volu m e and exp iratory reserve volum e (C) tid al volu m e and insp iratory reserve volum e (D) resid u al volu m e and exp iratory reserve volum e (E) vital cap acity less the closing volu m e 162. A 16-year-old girl w ith asthm a is to u nd ergo rou tine pu lm onary function testing. In ord er to m easu re the FEV1 d uring spirom etry, she (A) is asked to inhale forcefu lly (B) exhales forcefu lly from total lu ng cap acity to resid u al volu m e (C) is asked to breathe slow ly at norm al tid al volu m es (D) forcefu lly exhales from total lu ng cap acity to resp iratory reserve volu m e (E) is asked to forcefully exhale for 1 second 163. A p reviou sly healthy 46-year-old m an d eveloped gallstone pancreatitis and then requ ired m echanical ventilation to m aintain ad equ ate oxygenation. The ICU team d ecid ed that his case of ad u lt resp iratory d istress synd rom e w as best m anaged w ith high-frequ ency jet ventilation. This ventilatory m od e (A) can p rovid e ventilatory su p p ort only w hen a tightly sealed airw ay is established (B) cannot be app lied for su rgical p roced u res (C) can be u sed to p rovid e em ergency ventilation after cricothyroid cannu lation (D) creates a jet d rag effect, p reventing second ary gases from entering the airw ay (E) is only necessary d u ring rigid bronchoscop y 164. Factors contribu ting to increased airw ay p ressu re und er anesthesia inclu d e all of the follow ing EXCEPT

Que s tions : 157–171

(A) (B) (C) (D) (E)

m u scle p aralysis of the chest w all a decrease in functional residual capacity the su p ine p osition the p resence of an end otracheal tu be controlled ventilation

165. Distribu tion of ventilation in the lu ng is su ch that (A) the ap ical p ortions are better ventilated (B) the dependent areas are better ventilated (C) the central or hilar areas are better ventilated (D) all areas are ventilated equ ally (E) ventilation is not affected by p osition 166. The level of arterial Pco 2 (Paco 2) (A) d ep end s on m inu te ventilation only (B) is ind ep end ent of CO 2 p rod u ction (C) is not d ep end ent on d ead sp ace ventilation (D) varies d irectly w ith CO 2 p rod u ction and inversely w ith alveolar ventilation (E) d ecreases w hen d ead space ventilation increases 167. A healthy 26-year-old m an u nd erw ent op en herniorrhap hy. H e had a Class I p reop erative airw ay exam ination and the m ed ical stu d ent intu bated h im on her first attem p t. Wh en d ecid ing w hen to extu bate him , you shou ld consid er that (A) laryngosp asm is of no concern, as it only occurs d u ring ind uction (B) laryngosp asm shou ld be m anaged w ith positive p ressu re ventilation, oxygen, su ctioning of oropharyngeal secretions, and , in severe cases, a sm all d ose of IV su ccinylcholine (C) airw ay obstru ction only occu rs in the first 4 to 6 m in after extubation (D) extu bation d u ring d eep anesthesia carries the risk of a profou nd card iovascu lar response (E) in p atients w ho w ere d ifficu lt to intu bate, extu bation shou ld be carried ou t d u ring d eep anesthesia

43

168. A 70-year-old m an u nd erw ent left carotid end arterectomy tw o years ago and is about to have the same operation on his right sid e. Follow ing this right-sid ed surgery, he w ill (A) have no resp iratory changes (B) show no change in arterial carbon d ioxid e (C) resp ond to hyp oxia w ith hyp erventilation (D) be m ore su scep tible to hyp oxem ia (E) alw ays d evelop hyp ertension 169. Mechanism s that m ay cau se hypoxem ia und er anesthesia includ e all of the follow ing EXCEPT (A) hyp oventilation (B) hyp erventilation (C) increase in fu nctional resid u al cap acity (FRC) (D) su p ine p osition (E) increased airw ay p ressu re 170. A healthy 44-year-old w om an und erw ent ablation of a d ysrhythm ia focu s. The p roced u re requ ired tw o h ou rs, an d th e card iologist requ ested ap nea d u rin g p eriod s of im age acqu isition. You hyp erventilated her throu ghou t the proced u re. N ow that she is extu bated and breathing room air, she w ill (A) retu rn to norm al p aram eters w ithin 30 m in (B) rem ain hyp ocarbic for 2 h (C) p ossibly becom e hyp oxem ic if not treated w ith oxygen (D) becom e hyp oxem ic and hyp ercarbic (E) be w ell oxygenated if the air exchange is u nim paired by d ru gs 171. All of the follow ing lead to d ecreases in lu ng com pliance, EXCEPT (A) (B) (C) (D) (E)

p neu m othorax em p hysem a p u lm onary fibrosis p neu m onectom y hyp erinflation

44

4: Re s pira tion

172. The ventilatory resp onse to Paco 2 (A) (B) (C) (D) (E)

is ind ep end ent of hyp oxem ia has a m ajor p erip heral com p onent is d ep ressed by m etabolic acid em ia is u naffected by op ioid antagonists is au gm ented by norep inep hrine

173. When consid ering oxygen transp ort in the lu ng, the LEAST im p ortant cau se of hyp oxem ia is (A) (B) (C) (D) (E)

ventilation/ p erfu sion m ism atch d iffu sion barrier venou s ad m ixtu re bronchial artery blood flow altitu d e

174. In p u lm onary fu nction testing, carbon m onoxid e d iffu sing cap acity (DLCO) (A) is greater than fu nctional resid u al cap acity (FRC) (B) is u nchanged in anem ia (C) is increased in p u lm onary fibrosis (D) estim ates the gas transfer ability of the lu ng (E) estim ates the d ead sp ace ratio (VD / VT) of the lung 175. All of the follow ing lead to d ecreases in chest w all com p liance, EXCEPT (A) (B) (C) (D) (E)

chest w all ed em a thoracic d eform ities flail chest ventilator d yssynchrony abd om inal d istension

176. The oxyhemoglobin dissociation curve describes the relationship of oxygen saturation to oxygen tension. All of the following are true EXCEPT that (A) at an oxygen tension of 60 m m H g, the satu ration is ap proxim ately 90% (B) the cu rve is shifted to the left w ith a m ore acid ic pH (C) the cu rve is shifted to the right w ith an increase in carbon d ioxid e tension

(D) the cu rve is shifted to the left w ith a d ecrease in tem perature (E) the cu rve is shifted to the right w ith increased levels of 2,3-DPG 177. The d efinitive test of ad equ acy of ventilation is (A) (B) (C) (D)

listening to the esop hageal stethoscop e w atching the rise and fall of the chest analyzing arterial blood gases m easu ring tid al volu m e w ith a spirom eter (E) u sing an ap nea m onitor 178. Pu lm onary vascu lar resistance (A) is entirely d ep end ent on the card iac ou tpu t (B) is entirely d ep end ent on the p ressu re in the p ulm onary artery (C) is equ al to p ressu re d ivid ed by rad iu s of the artery (D) d ep end s on the state of vasom otor tone, flow, and p ressu re (E) is not affected by card iac ou tp u t 179. The w ork of breathing (A) can be excessively high d u ring SIMV (sp ontaneou s interm ittent m and atory ventilation) (B) is solely d u e to airw ay resistance (C) is solely d u e to elastic forces (D) is at its low est at a resp iratory rate of 25 breaths per m inu te (E) is increased in the p atient w ith restrictive d isease if the respiratory rate is increased 180. Anatom ic d ead sp ace (A) (B) (C) (D)

is ind ep end ent of lu ng size is abou t 1 m L/ kg bod y w eight is not affected by equ ip m ent com bined w ith alveolar d ead sp ace constitu tes p hysiologic d ead space (E) is of less im p ortance in the new born than the ad u lt

Que s tions : 172–188

181. The term P 50 in reference to the oxyhemoglobin d issociation cu rve (A) refers to the p osition on the cu rve at w hich the P O 2 is 50 m m H g (B) norm ally has a valu e of 27 m m H g (C) d escribes an enzym e system in hem oglobin (D) is constant (E) is affected only by typ e of hem oglobin 182. All of the follow ing are frequ ently fou nd in carbon m onoxid e poisoning EXCEPT (A) (B) (C) (D) (E)

seizu res and com a lactic acid osis d esatu ration by p u lse oxim etry carboxyhem oglobin norm al Pao 2

183. The carotid bod ies p rim arily (A) (B) (C) (D) (E)

resp ond to elevated Pco 2 resp ond to Svo 2 resp ond to Pao 2 signal the m ed u lla via the vagu s nerve resp ond to hyd rogen ions

184. H yp ercap n ia u n d er an esth esia m ay be a resu lt of (A) (B) (C) (D)

hyp erventilation d ecreased d ead sp ace ventilation d ecreased carbon d ioxid e p rod u ction u se of an Ayre T-p iece at less than p eak insp iratory flow rate (E) increased p u lm onary artery flow 185. All of the follow ing statem ents abou t the d iap hragm are tru e EXCEPT (A) (B) (C) (D)

it is innervated via the vagu s nerve it has no fixed insertion it is m ainly active in insp iration it has an equ al m ix of slow tw itch and fast tw itch fibers (E) it is d eficient in stretch recep tors

45

DIRECTION S: Use the follow ing figu re to answ er Qu estions 186-189: Zone 1

P A > P pa > P pv

Zone 2

P pa > P A > P pv

Zone 3

P pa > P pv > P A

Zone 4

P pa > P IS F > P pv > P A

A - a lve oli pa - pulmona ry a rte ry

pv - pulmona ry ve in IS F - inte rs titium

186. In Zone 1 of the lu ng (A) (B) (C) (D) (E)

no air is m oving circu lation is highest venou s p ressu re is high d ead sp ace is high shu nting is high

187. In Zone 2 of the lu ng (A) there is good blood flow regard less of ventilation (B) venou s p ressu re is high (C) d ead sp ace is high (D) the p u lm onary vessels are collap sed (E) the blood flow is d eterm ined p rim arily by p u lm onary artery p ressu re and alveolar pressure 188. In Zone 3 of the lu ng (A) blood flow is governed by the arteriovenou s p ressu re d ifference (B) d ead sp ace is high (C) there is high alveolar p ressu re (D) venou s p ressu re is very low (E) little blood flow occu rs

46

4: Re s pira tion

189. All of the follow ing are tru e EXCEPT (A) Zone 2 is the ”waterfall” region of the lung (B) Zone 1 w ill increase in hyp ovolem ic shock (C) Zone 4 w ill increase w ith lym p hatic blockage (D) these zones w ou ld m atter less if w e cou ld breathe in w ater (E) these zones are ind ep end ent of gravitational effects 190. H yp oventilation in the recovery room (A) shou ld alw ays be treated w ith op ioid reversal (B) is com m on after inhalation anesthesia (C) is u ncom m on after u p p er abd om inal p roced u res (D) is best d etected by p u lse oxim etry (E) is alw ays accom p anied by increases in blood p ressu re 191. All of the follow ing statem ents abou t p reoxygenation are tru e EXCEPT (A) 80% of the nitrogen in the fu nctional resid u al cap acity (FRC) is being rep laced w ith oxygen d u ring p reoxygenation (B) d u ring p reoxygenation, nitrogen is elim inated rap id ly, d ep end ent on the volu m e of the breaths (C) p reoxygenation p reced ing ind u ction of general anesthesia can su stain vital organs for u p to 15 m in even w ithou t active ventilation (D) p reoxygenation shou ld alw ays be consid ered before ind u ction of general anesthesia (E) p reoxygenation shou ld be carried ou t over 2-3 m in, or as a series of fou r vital capacity breaths 192. Which one of the follow ing statem ents is true about Typ e II alveolar cells? (A) They p rod u ce su rfactant. (B) They are the m ajor com p onent of gas exchange.

(C) They line the cap illary end otheliu m . (D) They can be rep laced by Typ e I cells. (E) They are m igratory and p hagocytic. 193. A 56-year-old fem ale p atient is sched u led for em ergent exp loratory lap arotom y for acu te bow el obstruction. After rapid sequence ind uction of general anesthesia, the patient is noted to have regurgitation of gastric contents d uring d irect laryngoscopy. Regard ing perioperative asp iration of gastric content, all of the follow ing are tru e, EXCEPT (A) the severity of sym p tom s d ep end s on the type and volu m e of m aterial asp irated (B) initial m anagem ent com p rises su ctioning, ad m inistration of bronchod ilators, su pplem ental O 2, and ICU transfer (C) bronchoscop y m ay be of benefit to rem ove p articu late m aterial (D) p u lm onary lavage w ith large volu m es of saline should be carried out rep eated ly (E) ad m inistration of em p irical antibiotic is not recom m end ed 194. Regard ing hypoxem ia d u ring the p ostop erative period , w hich one of the follow ing is true? (A) H yp oxem ia necessitates reintu bation in m ost cases. (B) H yp oxem ia is rarely cau sed by d ecreased ventilatory d rive. (C) Op ioid antagonists shou ld be avoid ed , as they im p ed e pain therap y. (D) The incid ence of p ostop erative hyp oxem ia is relatively ind epend ent of surgical site. (E) Analgesics can enhance p ostop erative resp iratory m echanics in som e cases. 195. Com p aring infant (8 yr) airw ays, (A) the angle betw een trachea and right bronchu s is sm aller in infants (B) the narrow est p osition of the airw ay is glottis in infants and cricoid cartilage in ad u lts

Que s tions : 189–202

(C) only ad u lts have a p rom inent p rotru sion of the cornicu late and cu neiform tubercles into the laryngeal ad itu s (D) the angle betw een trachea and left bronchus rem ains u nchanged (E) the ep iglottic cross-section shap e rem ains u nchanged 196. If a patient is allow ed to breathe 100% oxygen u nd er anesthesia (A) areas of atelectasis w ill d isap p ear (B) bow el d istention w ill d ecrease (C) the P O 2 w ill rise d u e to increased d ead space (D) lu ng u nits w ith low ventilation/ p erfu sion (V/ Q) ratios m ay becom e shu nt u nits (E) the oxygen tension w ill rise d u e to an increase in fu nctional resid u al cap acity (FRC) 197. All of the follow ing statem ents are tru e abou t the esop hageal-tracheal airw ay, Com bitu be, EXCEPT (A) the com bitu be enters the trachea in app roxim ately one third of cases (B) the d evice has tw o lu m ens, one op ening at the d istal end , one at fenestrations betw een the balloons (C) ventilation takes p lace u sing the m ore proximal lu m en in the m ajority of cases (D) the com bitu be d oes not need to be rep laced even if it enters the trachea (E) it is intend ed to establish em ergency airw ay access if the op erator is not able to perform face m ask ventilation or conventional intubation 198. Ind ep end ent risk factors for d ifficu lt m ask ven tilation in clu d e all of th e follow in g, EXCEPT (A) (B) (C) (D) (E)

sleep ap nea lim ited m and ibu lar p rotru sion bod y mass ind ex 55 years

47

199. A 46-year-old w om an w ith a history of sclerod erm a for m ore than tw enty years is being evaluated in the preoperative clinic. She w ou ld be exp ected to have all of these p u lm onary m anifestations of sclerod erm a EXCEPT (A) (B) (C) (D) (E)

increased com p liance d iffu se fibrosis d ecreased vital cap acity hyp oxem ia increased VD / VT ratio

200. All of the follow ing statem ents are tru e abou t alveoli EXCEPT (A) they are (B) they are lar cells (C) they are alveolar (D) they are alveolar (E) they are

100–300 m icrons in d iam eter m ostly lined w ith Typ e I alveop artially lined w ith Typ e II cells p artially lined w ith Typ e III cells su rrou nd ed by cap illaries

201. Au to-PEEP (p ositive end -exp iratory p ressure) (A) d ecreases end -exp iratory lu ng volu m e (B) can be m easu red by ap p lying an exp iratory pau se in m echanically ventilated p atients (C) p rom otes venou s retu rn (D) d ecreases w ith increasing resp iratory rate (E) d ecreases w ith increasing m inu te ventilation 202. Chest w all com p liance (A) norm ally is 200 mL/ cm H 2O (B) d ecreases in the setting of a flail chest (C) is increased in p atients w ith kyp hoscoliosis (D) is increased in p atients w ith abd om inal d istension (E) is increased in m orbid ly obese p atients

48

4: Re s pira tion

203. The w ork of breathing (A) involves both resistive and elastic w ork (B) is exp end ed m ostly in exp iration (C) to overcom e elastic forces is d ecreased w hen breathing is d eep and slow (D) to overcom e resistive forces is d ecreased w hen breathing is fast and shallow (E) is exp end ed m ostly in insp iration DIRECTION S: Use the follow ing figu re to answ er Qu estions 204-205:

206. Functional resid ual capacity (FRC) can be measured by (A) (B) (C) (D) (E)

u se of an esop hageal balloon com p u ted tom ograp hy sp irom etry inert gas d ilu tion techniqu e bioim p ed ance

207. End -tid al CO 2 is increased by (A) bicarbonate ad m inistration (B) a circu it leak arou nd the end otracheal tube cu ff (C) hyp otension (D) card iac arrest (E) intracard iac air em bolism 208. H yp oxic p u lmonary vasoconstriction

204. All of the follow in g are TRUE of th e flow volu m e loop show n in the figure EXCEPT (A) the X axis is volu m e (B) the Y axis is p ressu re (C) vital cap acity is the d istance from p oint 1 to point 3 (D) a breath p roceed s throu gh p oints 1, 2, 3, and 4, in that ord er (E) there is no air leak 205. In this flow -volu m e loop (A) there is evid ence of tracheal stenosis (B) point 2 is at m axim u m expiratory volu m e (C) point 3 is at m axim u m inspiratory volu m e (D) one can d eterm ine insp iratory reserve volu m e (E) one can d eterm ine fu nctional resid u al capacity (FRC)

(A) lead s to an increase in the shu nt fraction (B) is increased w ith increases in p u lm onary artery p ressu re (C) is increased w ith increases in central blood volu m e (D) is d ecreased w ith the u se of sod iu m nitropru ssid e (E) is increased w ith the u se of inhaled nitric oxid e 209. Cond itions aggravated by hypercapnia include all of the follow ing EXCEPT (A) (B) (C) (D) (E)

elevated intracranial p ressu re right-to-left card iac shu nts p u lm onary hyp ertension card iac d ysrhythm ia ARDS

210. The p atient w ho is hyperventilated to a Pco 2 of 20 m m H g u nd er anesthesia w ill have (A) (B) (C) (D)

increased cerebral blood flow increased ionized calciu m increased oxygen d elivery to the tissu es increased ventilation/ p erfu sion (V/ Q) m ism atch d u e to inhibition of hyp oxic p u lm onary vasoconstriction (E) increased resp iratory d rive

Que s tions : 203–219

211. Sp ecific effects of anesthesia on control of breathing inclu d e a d ecreased resp onse to all of the follow ing, EXCEPT (A) (B) (C) (D) (E)

carbon d ioxid e hyp oxem ia m etabolic acid em ia ad d ed airw ay resistance external stim u li

212. Factors lead ing to p u lm onary ed em a inclu d e all of the follow ing, EXCEPT (A) (B) (C) (D) (E)

increased cap illary p ressu re d ecreased oncotic p ressu re lym p hatic insu fficiency increased cap illary p erm eability hyp erinflation d u ring ventilation

49

(D) increasing CC above the tid al volu m e p lu s FRC resu lts in areas of atelectasis (E) com p ared to ad u lts, you ng child ren are less likely to su ffer atelectasis from tid al breathing occu rring from end -exp iratory lu ng volu m es close to closing cap acity 216. Du ring anesthesia, the d iap hragm assu m es a m ore cep halad p osition becau se of all of the follow ing, EXCEPT (A) p aralysis from m u scle relaxants (B) increased end -exp iratory tone of the abd om inal w all (C) su rgical retraction (D) second gas effect (E) p neu m op eritoneu m 217. Forced exhaled vital cap acity (FVC)

213. The resp iratory qu otient (A) d ep end s on the CO 2 ou tp u t and O 2 u p take (B) is independent of the metabolic substrate (C) d ep end s on the O 2 u p take and card iac ou tpu t (D) d ep end s on the CO 2 ou tp u t and m etabolic equ ivalent (E) is alw ays 0.8 214. Positiv e en d -exp iratory p ressu re (PEEP) u su ally (A) d ecreases fu nctional resid u al cap acity (FRC) (B) d ecreases com p liance (C) d ecreases w ork of breathing (D) increases w ork of breathing (E) d ecreases lu ng volu me 215. In com p aring closing capacity (CC) and fu nctional resid u al capacity (FRC), (A) obesity increases both CC and FRC (B) increasing FRC relative to CC results in areas of low ventilation/ perfusion (V/ Q) (C) anything that d ecreases CC below FRC resu lts in areas of atelectasis

(A) (B) (C) (D)

m ay vary w ith p atient coop eration is m easu red in the first second is a measure of inspiratory reserve volume is affected by restrictive d isease in the first second (E) can on ly be m easu red u sin g bod yp leth ysm ograp hy or the inert gas d ilu tion techniqu e 218. The m ed u llary chem orecep tors are m axim ally stim u lated by (A) (B) (C) (D) (E)

low oxygen tension reflex activity from the d iap hragm carbon d ioxid e ond ansetron hyd rogen ion

219. The com p osition of alveolar gases d iffers from that of inhaled gas. Concerning this, all of the follow ing are tru e, EXCEPT (A) oxygen is being absorbed from the alveoli (B) carbon d ioxid e is being ad d ed to the alveoli (C) w ater vap or is being ad d ed (D) nitrogen is taken u p by the alveolar capillaries (E) inhalational anesthetic agents are absorbed from or eliminated into the alveoli

50

4: Re s pira tion

220. The endotracheal tube position in a female adult patient is evaluated by the follow ing: examining the patient, the numerical markings on the tube, and the chest x-ray. Which one of the follow ing is consistent w ith the proper position? (A) The left sid e is ventilated better than the right sid e. (B) The tip of the tube is 30 cm from the u pper front teeth. (C) The tip of the tu be overlies the 6th thoracic vertebra. (D) Both sid es ventilate equ ally. (E) The id eal tu be p osition is ap p roxim ately 2 m m above the carina. 221. Du rin g m ech an ical ven tilation in sp iratory airw ay resistance (A) cannot be estim ated (B) is d ecreased w ith the u se of longer tu bes (C) is ind ep end ent of end otracheal tu be d iam eter (D) is typ ically low er com p ared w ith exp iratory airw ay resistance (E) is ind ep end ent of end otracheal tu be length 222. A right shift in the oxyhemoglobin d issociation cu rve is cau sed by (A) (B) (C) (D) (E)

d ecreased tem p eratu re hyp ercarbia alkalosis the p resence of fetal hem oglobin hyp ocarbia

223. Factors that increase the incid ence of p ostop erative p ulmonary comp lications inclu d e all of the follow ing, EXCEPT (A) (B) (C) (D) (E)

u pp er vs. low er abd om inal surgery chronic obstructive pu lm onary d isease heart failure asthm a longer surgical d uration

224. All of the follow ing statements are tru e of closing cap acity EXCEPT it is

(A) the lu ng volu m e at w hich the onset of airw ay closu re is d etected (B) increased in sm oking (C) greater than resid u al volu m e (D) greater than closing volu m e (E) a sm aller fraction of the total lu ng capacity in infants, com pared to ad u lts 225. Transpu lm onary pressure (A) m easu res intralu ng p ressu re (B) is equ al to intrap leu ral p ressu re at FRC (C) is a grad ient betw een the airw ay op ening and the alveolar p ressu re (D) increases w ith increasing lu ng volu m e (E) is ind ep end ent of tid al volu m e 226. All of the follow ing statem ents are tru e in interpreting pulmonary function tests EXCEPT (A) vital cap acity m easu rem ent is not a tim ed m easurem ent (B) sp irom etry fails to d etect early d isease in sm all airw ays (C) m axim al breathing cap acity is d ep end ent on coop eration (D) the FEV1 w ill d etect restrictive d isease (E) absolu te lu ng volu m es cannot be d eterm ined using stand ard sp irom etry 227. A p atient w ho arrives in the recovery room after a general anesthetic shou ld (A) be sed ated to p revent overt p ostop erative stress (B) alw ays be encou raged to lie on the back for easier access to the airw ay (C) be closely observed for resp iratory d epression (D) be given op ioid s at fixed intervals (E) alw ays be m aintained on arterial blood p ressu re m onitoring, u sing a rad ial artery catheter 228. A 14-gau ge catheter is inserted through the cricothyroid m em brane and attached to a w all oxygen sou rce in su ch a w ay that oxygen can be d elivered intermittently. With this technique

Que s tions : 220–235

(A) (B) (C) (D) (E)

p neu m othorax is inevitable ad equ ate oxygenation is p ossible gastric d ilatation is a hazard larynx inju ry is im p ossible p revention of hyp ercap nia is p ossible

229. Patients w ith pneu m oconiosis, e.g., asbestosis or silicosis, often requ ire su rgery on other organs. In the p reop erative assessm ent, one m ust recognize that (A) the lu ng volu m es w ill be increased (B) the x-ray abnorm ality fu lly reflect the fu nctional changes (C) early airw ay closu re is the hallm ark (D) fibrosis u su ally is p resent (E) FEV1/ FVC is regu larly d ecreased in these patients 230. Vital cap acity inclu d es all of the follow ing, EXCEPT (A) (B) (C) (D) (E)

tid al volu m e insp iratory reserve volu m e exp iratory reserve volu m e fu nctional resid u al cap acity (FRC) closing volu m e

231. H yp oxem ia m ay occu r u n d er an esth esia becau se of all of the follow ing, EXCEPT (A) blood loss (B) increased release of oxygen from hem oglobin to the tissu es (C) d ep ressed m yocard ial fu nction (D) shu nting (E) airw ay obstru ction 232. Diffu sion hyp oxia (A) is d u e to a large volu m e of nitrou s oxid e in the lu ngs (B) is d u e to a large volu m e of carbon m onoxid e in the blood stream

51

(C) is d u e to the second gas effect (D) can occu r u p to 48 h after su rgery (E) d oes not resp ond to oxygen supp lem entation 233. Pneum othorax m ay be d ue to all of the follow ing EXCEPT (A) alveolar ru p tu re (B) chest w all trau m a (C) connection betw een the d istal airw ay and the pleu ral sp ace (D) extrathoracic tracheal p u nctu re (E) a break in the p arietal p leu ra 234. When assessing the acutely hypoxemic patient, causes that m ay be im portant are all of the follow ing EXCEPT (A) (B) (C) (D) (E)

hyp oventilation hyp op erfu sion ventilation/ p erfu sion (V/ Q) m ism atch intracard iac shu nts abnorm al d iffu sion

235. Total pu lm onary com p liance (A) is m easu red by d ivid ing p ressu re by volu m e (B) is u su ally d ecreased in eld erly p atients (C) involves the lu ng only (D) is ind ep end ent of p reviou s breaths (E) is increased by su rfactant

Answe rs a nd Expla na tions

150. (E) At FRC, there is no p ressu re d ifference betw een the alveoli and atm osp here. Fu rther ventilation is p ossible, since FRC is the volu me that exists at the end of a norm al tid al volu m e. Elastic forces of the lu ng and chest w all are balanced . The total p u lm onary vascu lar resistance is at its low est at FRC. The grad ient betw een the alveoli and the intrap leu ral sp ace w ill be non-zero, balancing the elastic tension of the lung. (6:2087-8) 151. (E) Dysp nea associated w ith obesity is likely d u e to m u ltiple problem s, includ ing high card iac ou tp ut and d ecreased com p liance of the chest w all. Chem orecep tors are activated by hyp oxem ia, acu te hyp ercarbia and acid osis and are located in the m ed u lla and carotid bod ies. J-recep tors are activated d u ring the accu m u lation of p u lm onary ed em a. Changes in the biochem ical m ilieu of skeletal m u scle can activate m etaboreceptors that can contribute to the sensation on d ysp nea. (6:277-9) 152. (C) In ventilated p atients p roxim al airw ay p ressu re increases w ith in creases in flow, higher tid al volu mes, low er respiratory system com pliance, higher airw ay resistance, higher insp iratory flow, higher PEEP, and p resence of auto-PEEP. (5:470) 153. (B) Sp irometry can only d etect relative volu me changes. In ord er to d eterm ine absolu te volum es inclu d ing resid u al volu m e and to calcu late total lu ng cap acity, other m ethod s like inert gas d ilu tion or bod y p lethysm ograp hy have to be ap p lied . (6:2091)

52

154. (B) End obronchial intu bation w ith a d ou blelu m en tu be allow s selective lu ng ventilation an d isolation of a bleed ing lu n g segm ent. When one lu ng contains either blood or infectiou s secretions, isolation of the lungs should be applied in ord er to prevent spillage of blood or secretions into the unaffected lung. (5:417, 963) 155. (E) Carboxyhem oglobin is involved in the transport of carbon m onoxid e. Carbon d ioxid e exists as d issolved , bicarbonate and carbam inohem oglobin. (5:461, 525, 1336) 156. (C) The V/ Q ratio is higher at the ap ex and thus P O 2 is higher in this location as w ell. Blood flow is greatest at the base. (5:959) 157. (B) Measu res to im p rove oxygenation d u ring one-lu ng ventilation inclu d e, am ong others, im p roving V/ Q d istribu tion and / or blood flow in the d ep end ent lu ng (lu ng recru itm ent m aneu ver, PEEP, p rostacyclin, nitric oxid e), increasing Pao 2 (throu gh CPAP) or d ecreasing blood flow in the non-d ep end ent lu ng (ligatu re). A ligatu re p laced on the p u lm onary artery of the d ep end ent lu ng w ou ld resu lt in m assive shu nting and p rofou nd hyp oxem ia. (5:970) 158. (B) As an air-flu id m ixtu re, the lu ng tend s to sag w ith gravity, cau sing a gravity-ind u ced p leu ral p ressu re grad ient. Pleu ral p ressu re increases by app roxim ately 0.25 cm H 2O w ith every centim eter from apex to base. Thus, it is low est at the ap ex and highest at the base. Pleu ral p ressu re is related to bod y p osition becau se in d ifferent p ositions, d ifferent areas

Answe rs : 150–169

53

of the lung w ill be d epend ent. Pleural pressure variation is cau sed by the hyd rostatic pressu re exerted by gravity. (5:959)

injector throu gh the cricothyroid m em brane. An escap e p athw ay for the injected gases m u st be p resent, how ever. (5:1419)

159. (D ) The LaPlace law, P = 2T/ R, states that the p ressu re w ithin an elastic sp here is d irectly p rop ortional to the tension of the w all and inversely p rop ortional to the rad iu s of the cu rvatu re. In this case, the sp here is the alveolu s. Alveoli are lined w ith a film of su rfactant that lend s stability to the alveoli by d ecreasing the su rface tension as the rad iu s of the alveolu s becom es sm aller. Withou t this ability to vary su rface tension, sm all alveoli, w hich have a sm aller rad iu s of cu rvature and thu s a higher p ressu re, w ou ld em pty into large alveoli and alveolar stability w ou ld be lost. (6:2154)

164. (A) Muscle paralysis w ill d ecrease chest and abd om inal w all tone, im proving com p liance. The d ecrease in FRC and the su pine position w ill move the patient to a less compliant region of the lung volume relationship. The end otracheal tube increases airw ay resistance, w orsening dynamic compliance. Controlled ventilation changes the p ressu re grad ient entirely; the airw ay pressu re m u st be su p ra-atm osp heric instead of sub-atmospheric. (5:362, 471, 1406)

160. (D ) Surfactant is a substance containing d ipalmitoyl lecithin prod uced by the type II alveolar ep ithelial cells of the lu ng. The su bstance is im p ortant in ad u lts as w ell as new borns, p rovid ing alveolar stability. It is 90% lip id and 10% p rotein. (5:250; 6:2206) 161. (D ) The FRC is composed of expiratory reserve volu m e and resid u al volu m e. Tid al volu m e p lus insp iratory reserve volu m e com prise the insp iratory cap acity. The other op tions are not d esignated cap acities. (5:256) 162. (B) In ord er to d eterm ine the FEV1, the p atient is asked to forcefu lly exhale from total lu ng cap acity to resid u al volu m e. While the FEV1 m easu res the volu m e exhaled after the first second of the m aneuver, the patient is asked to com plete the exhalation process. This allow s am ong others for the calcu lation of the FEV1/ FCV ratio. (5:131) 163. (C) Du ring high frequ ency jet ventilation (H FJV), gas is injected into the trachea u nd er high p ressu re, creatin g a ”jet d rag” effect entraining (not p reventing the entry of) second ary gases, thus provid ing ventilation to a non-sealed airw ay. It has been u sed su ccessfu lly in su rgeries w here establishing a closed airw ay is not possible, e.g. u pp er airw ay su rgery. Sim p le H FJV system s have been u sed to p rovid e em ergency ventilation by placing an

165. (B) The d ep end ent areas are better ventilated , since the alveoli in the d ep end ent areas are sm aller and m ore com pliant. (5:959) 166. (D ) Arterial CO 2 rises w ith CO 2 p rod u ction and d ecreases w ith increases in alveolar ventilation. As d ead sp ace ventilation d oes not contribu te to gas exchange, the larger the ratio of d ead sp ace ventilation to total ventilation becom es the higher Paco 2 rises. (5:460) 167. (B) Laryngospasm is common after extubation, esp ecially in child ren. It shou ld be treated as ou tlined in option B. Delayed airw ay obstru ction can occur, for instance as a result of laryngeal edema. Extubation during d eep anesthesia m inim izes the card iovascu lar resp onse, bu t increases the risk of airw ay com p lications. It shou ld not be attem p ted in patients w ho w ere d ifficu lt to intu bate, as reintu bation m ight becom e necessary. (5:575) 168. (D ) After bilateral carotid end arterectom y, a p atient w ill be m ore susceptible to hypoxem ia becau se of bilateral d enervation of the carotid bod ies. The p atient w ill not resp ond to hyp oxem ia w ith hyp erventilation. In ad d ition, the resting Pco 2 is elevated , and the resp onse to sm all d oses of op ioid s m ay be accentu ated . H yp ertension is not a constant find ing in these p atients. (5:1021) 169. (C) Anesthesia u su ally cau ses a d ecrease in FRC that lead s to hyp oxem ia. All of the other options can lead to hypoxemia: hypoventilation

54

4: Re s pira tion

by d ecreased FRC and increased shu nt, hyp erventilation by shift of the oxyhem oglobin d issociation cu rve to the left and d ecreased card iac ou tpu t, su pine position by d ecreased ventilation and d ecreased FRC, and increased airw ay pressu re by change in the ventilationperfu sion relationship s. (5:363, 459, 550) 170. (C) Patients w ho are hyperventilated for long p eriod s of tim e have their carbon d ioxid e stores d epleted . Postoperatively, these patients hyp oven tilate in an effort to restore th eir carbon d ioxid e and , in d oing so, m ay becom e hyp oxem ic if not given su pp lem ental oxygen. (5:460) 171. (B) N orm al lu ng com p liance is 100 m L/ cm H 2O and is d ecreased by p u lm onary ed em a (card iogenic or noncard iogenic), p neu m othorax, lu ng consolid ation, atelectasis, pu lm onary fibrosis, p neu m onectom y or lu ng resection, bronchial intubation, and hyperinflation. Lung comp liance is increased w ith em physem a and flail chest. (5:471) 172. (E) The CO 2 response curve is shifted to the left by n orep inep h rine, acid osis, an d hyp oxia. N aloxon e w ill reverse op ioid d ep ression . Peripheral chem orecep tors contribu te 15% of the control. (5:324, 529, 612, 710) 173. (B) Diffusion is rarely the lim iting com p onent of oxygen transport. H igh altitu d e low ers the Pao 2. The other factors contribu te to shu nt. (6:287) 174. (D ) DLCO is a m easu re of the d iffu sing cap acity of the lu ng. Increased hem oglobin and p u lm onary blood volu m e w ill affect it. It is not a lu ng cap acity in the sam e sense as FRC. (5:953; 6:2093) 175. (C) All cond itions lead to d ecreased chest w all com pliance through either increased m uscular tone or m echanical im p airm ent of elasticity, except flail chest that lead s to increased chest w all com p liance throu gh instability (5:471) 176. (B) The cu rve is shifted to the right w ith acid osis. The other options are correct. (5:459; 6:853)

177. (C) All of the factors cited are p resu m ptive evid ence of ventilation. The only su re m easu re of d em onstrating effective gas exchange is the analysis of blood gases, sp ecifically carbon d ioxid e. (5:459) 178. (D ) Pu lm onary vascu lar resistance is the resu lt of card iac outpu t, the state of vasom otor tone, and p ressu re. A change in card iac ou tpu t w ill norm ally be follow ed by changes in the rad ius of the vessels to allow m aintenance of norm al p ressu re. The resistance is equ al to p ressu re d ivid ed by flow. (6:1856) 179. (A) As the m and atory ventilatory rate is red u ced d u ring SIMV, the w ork of breathing for both m and atory and sp ontaneou s breaths increases. The optimum rate is about 15 breaths p er m inu te in norm al ad u lts. In the p atient w ith restrictive d isease, short shallow breaths d ecrease the effort. In general, hu m ans ad ju st their breathing pattern to m inim ize the w ork of breath in g w h ile m ain tain in g ad equ ate ventilation. (5:1410) 180. (D ) An atom ic d ead sp ace in creases w ith increased lung volume. The normal d ead space is ap p roxim ately 2 m L/ kg. Equ ip m ent d ead space m ay greatly increase the am ount of d ead sp ace that can be d isproportionately large in infants. (5:460) 181. (B) P 50 is the P O 2 level on the oxyhem oglobin d issociation curve at w hich hem oglobin is 50% satu rated . The norm al valu e is 27 m m H g and m ay change d u e to the influ ence of factors that cause a shift in the oxyhemoglobin d issociation curve. (5:200, 274) 182. (C) Carbon m on oxid e bin d s h em oglobin avid ly, form ing carboxyhem oglobin. Becau se oxyh em oglobin an d carboxyh em oglobin absorb light at the sam e w avelength, it is not p ossible to d etect carbon m onoxid e poisoning w ith p u lse oxim etry. (5:1336) 183. (C) The chem orecep tors of the carotid bod y sense Pao 2 (less than 65 m m H g) and send afferen ts via th e glossop h aryn geal n erve. (5:251; 6:277)

Answe rs : 170–197

184. (D ) H yp ercap nia m ay resu lt from rebreathing of CO 2 w hile using an Ayre T-piece at low fresh gas flow. The other op tions all cau se d ecreased carbon d ioxid e tension. (5:638) 185. (A) The d iap hragm is innervated by th e p hrenic nerve. (5:952) 186. (D ) In Zone 1, the alveolar p ressure is higher than the arterial or venou s p ressu re; therefore, these areas act as d ead space u nits. (5:460, 960) 187. (E) In Zone 2, the blood flow is d ep end ent on arterial p ressu re and the alveolar p ressu re. These change w ith the statu s of ventilation. Venou s p ressu re is still not a d eterm inant of blood flow in this zone. (5:414, 460, 960) 188. (A) In Zone 3, blood flow is d eterm ined by the arteriovenou s pressu re d ifference. Dead sp ace is low, since the u nits are being p erfu sed , alveolar pressure is not high, and the venou s pressu re is low er than arterial bu t higher than alveolar pressu re. (5:414, 460, 960) 189. (D ) Gravity is the cause of the hyd rostatic pressu re grad ient in the arterial and venous circu lations. If the d ensity of the alveolar gas w ere closer to blood , the p ressu re grad ient in the alveoli would be closer to the vascular gradient. (5:414, 460, 960) 190. (D ) Resid u al anesthetic, both inhaled and intravenou s, inhibits hyp oxic d rive. It m ay or may not respond to opioid reversal, d epend ing on the anesthesia techniqu e em p loyed . The patient w ith hypoventilation m ay be hyp otensive. The best w ay to id entify the hypoxia is a continuou s convenient m onitor: p ulse oxim etry. It is im portant to note that patients receivin g a high con centration of su p p lem en tal oxygen might not d isplay oxygen d esaturation even in the setting of hypoventilation. (5:1286) 191. (C) All of th e facts are tru e excep t C. Preoxygenation can su stain vital organs for up to approxim ately 8 m in. (5:551) 192. (A) Type I alveolar cells form the m ajority of the alveolar gas exchange surface. When injured ,

55

Typ e II cells create new Typ e I cells. Typ e II cells prod uce surfactant. Type III cells are alveolar m acrophages. (6:2206) 193. (D ) All of the op tions are ap p rop riate, excep t pulmonary lavage w ith large volumes of saline that is believed to cau se d etrim ental effects. (5:1288) 194. (E) H ypoxem ia can be treated by su pplem enting the sp ontaneou sly breathing p atient w ith O 2 in m ost cases. A m ajor reason for p ostop erative h yp oxem ia is resid u al resp iratory d ep ression from h yp n otics or an algesics, n ecessitatin g ap p lication of antagon ists in som e cases. H ow ever, im proved pain control by ad m inistration of op ioid s m ight facilitate d eep breathing and thu s p revent hyp oventilation and atelectasis. The su rgical site, esp ecially if thoracic or upper abd om inal, is know n to have a m ajor influ ence on p ostop erative hyp oxem ia. (5:1286) 195. (D ) Infant (8 yr)

Angle be twe e n tra che a a nd right bronchus (de g)

30

20

Angle be twe e n tra che a a nd le ft bronchus (de g)

45

45

Na rrowe s t pos ition of the a irway

Cricoid ca rtila ge

Glottis

P rotrus ion of cornicula te a nd cune iform tube rcle s into a ditus

P romine nt

Minima l

S ha pe of e piglottic cros s -s e ction

Ome ga -s ha pe d

Cre s ce nts ha pe d or fla t

(5:549) 196. (D ) When p atients breathe an increased FIO 2, there is an increased am ou nt of shunt present d u e to absorp tion atelectasis. This occu rs in all air spaces. The lu ng u nits w ith low V/ Q ratios have a greater tend ency to collap se. (5:970) 197. (A) The Com bitu be enters the trachea in abou t 4-6% of cases and can then still be used to ventilate the lu ngs, by u sing the d istal rather than the m ore proxim al lu m en. All other facts are tru e. (5:558)

56

4: Re s pira tion

198. (C) Ind ep end ent risk factors for d ifficu lt m ask ventilation inclu d e age old er than 55 years, bod y m ass ind ex greater than 30 kg/ m 2, facial hair, lim ited m and ibu lar p rotrusion, abnorm al neck anatom y, sleep ap nea, and a history of snoring. (5:549) 199. (A) The p u lm onary involvem ent in sclerod erm a resu lts in restrictive d isease. The chest w all and the lu ng itself both contribu te to the restriction. There is fibrosis and a d ecreased vital capacity lead ing to d ecreased arterial satu ration. The VD / VT ratio increases. (6:2757) 200. (D ) Type III alveolar cells are macrophages that are not stationary elem ents of the alveolar lining. (6:2206) 201. (B) Au to-PEEP occurs if the expiratory phase is term inated p rem atu rely. When this occu rs, alveolar p ressu re d oes not equ ilibrate w ith p roxim al airw ay p ressu re at end -exhalation an d gas trap p in g resu lts. It in creases en d exp iratory lu ng volum e and can be m easu red by ap p lying an expiratory p au se in m echanically ventilated p atients. The p ressu re m easu red at the end of this m aneu ver that is in excess of the PEEP set on the ventilator is au toPEEP. Au to-PEEP m ay red u ce venou s retu rn and increases w ith increasing resp iratory rate and m inu te ventilation. (5:470) 202. (A) Chest w all com p liance is calcu lated from the change in esop hageal p ressu re (p leu ral pressu re) d u ring p assive inflation. Chest w all com p liance norm ally is 200 m L/ cm H 2O, and can be d ecreased by abd om inal d istension, chest w all ed em a, chest w all bu rns, thoracic d eform ities (e.g., kyp hoscoliosis), in obese p atients, and throu gh an increase in m u scle tone (e.g., a p atient w ho is d yssynchronou s w ith the ventilator). Chest w all com p liance is increased w ith flail chest and p aralysis. (5:471) 203. (A) The w ork of breathing involves both elastic and resistive w ork. If breathing is d eep and slow, m ore effort is expend ed in overcom ing elastic w ork. Most of the breathing effort is exp end ed in inspiration. (6:2089)

204. (B) A flow -volum e loop has flow on the y-axis and volume on the x-axis. An air leak w ould be m anifest by an op en curve. (5:997) 205. (C) Dynam ic tracheal stenosis w ould be seen w ith in sp iratory or exp iratory flatten in g, d epend ing on w hether the affected area w as extrathoracic or in trath oracic, resp ectively. Point 2 is the p oint of p eak insp iratory flow, w hile m axim u m exp iration is at point 1. Point 3 is at m axim u m insp iration, and p oint 4 at p eak exp iratory flow. Since a flow -volu m e loop is d one from m axim u m exp iration to m axim u m insp iration and back, the point at rest, FRC, can n ot be d eterm in ed . Withou t know ing the resid u al volu m e, FRC, or tid al volume, none of the classical lung volumes can be d eterm ined . Vital capacity, the sum of inspiratory reserve, exp iratory reserve, and tid al volumes can be d etermined , but it is a capacity, not a volu m e. (5:997; 6:2086) 206. (D ) The FRC m easu rem ent is p ossible throu gh the u se of bod y p lethysm ograp hy or a trace gas (su ch as heliu m ) d ilu tion. The resid u al volu m e com p onent of the FRC cannot, by d efinition, be measured with spirometry. An esop hageal balloon is used to estimate pleural pressure, and is not relevant to this measurem ent. (6:2084) 207. (A) The breakd ow n of bicarbonate w ill lead to CO 2 p rod u ction and thu s increased ETCO 2 levels. Any d ecrease in sam pling, red u ced p rod u ction or d elivery of CO 2 to the lu ngs w ill d ecrease ETCO 2. (5:466) 208. (D ) Vasod ilators attenu ate the hyp oxem ic p u lm onary vasoconstrictive resp onse. Anything that increases pressure in the pulm onary circulation w ill d ecrease hyp oxic p u lm onary vasoconstriction, includ ing increases in pulm onary p ressu re, central blood volu m e, and arterial p ressu re. (5:969) 209. (E) All of the cond itions mentioned are aggravated by hypercapnia, partly through increased catecholam ines, resp iratory acid osis, and increased p lasm a p otassiu m , excep t ARDS, w here a certain degree of permissive hypercapnia can au gm ent u nload ing of oxygen to the

Answe rs : 198–221

tissues by virtue of a rightw ard shift of the oxygen-hemoglobin dissociation curve. (5:532, 551) 210. (D ) H yp ocap nia lead s to d ecreased cerebral blood flow, d ecreased ionized calciu m , and d ecreased oxygen d elivery to the tissu es. Resp iratory d rive is d ecreased . There is an inhibition of hypoxic p u lm onary vasoconstriction lead ing to V/ Q m ism atch. (5:970) 211. (D ) The anesthetized p atient has d ecreased ability to respond to increased CO 2, low oxygen, m etabolic acid em ia, and external stim uli. The ability to resp ond to ad d ed airw ay resistance is not lost. (5:612) 212. (E) Su rgery can cau se cap illary d am age from rou gh hand ling and can interru pt lym p hatic d rainage. Inju d iciou s flu id ad m inistration can ad d m ore fluid than the pulmonary circu lation can hand le. H yperinflation m ay cau se parenchym al lung d am age, bu t usually not pulm onary ed em a. (5:422; 6:2232) 213. (A) The resp iratory qu otient is the rate of CO 2 output d ivid ed by the rate of O 2 u ptake. It may vary w ith the m etabolic substrate and , therefore, w ill not alw ays be 0.8. (5:1468) 214. (C) PEEP increases FRC and d ecreases the w ork of breathing. Com p liance is increased , and the lu ng volum e is increased . (5:470, 651) 215. (D ) The FRC/ CC relationship is im p ortant in d eterm ining if the airw ay w ill rem ain op en. If CC is below FRC, lu ng areas w ill stay op en. If CC is above FRC, lu ng areas w ill collap se w ith each breath. If CC is above FRC p lu s tid al volum e, lu ng areas w ill stay collap sed . In obesity, FRC is d ecreased , w hile CC stays constant. You nger child ren are m ore p rone to exp eriencing airw ay closu re and alveolar collap se w ith atelectasis becau se the end -exp iratory lu ng volum e from w hich tid al breathing occurs is close to closing capacity. (5:253, 302) 216. (D ) When p aralyzed , the d iaphragm is p u shed cephalad by abd om inal contents in the sup ine and lateral p ositions. Und er sp on taneou s, anesthetized breathing, active exp iration w ith

57

increased abd om inal tone also p u shes the d iap hragm cephalad . Pneu mop eritoneu m , retraction, and p acking in the abd om en can also d ecrease the d iap hragm ’s d escent. The second gas effect is the concentrating effect of the initial up take of nitrou s oxid e on the other alveolar gases. (5:605, 1041) 217. (A) Th e forced vital cap acity (FVC) is the volum e of gas that can be exhaled from m axim al inhalation. The resu lts vary w ith patient coop eration. The FEV1 is that p ortion of the p roced ure p erform ed in the first second . The FEV1 is greatly red u ced in obstru ctive d isease. Insp iratory reserve volu m e is only a p ortion of the FVC. Unlike resid u al volu m e, FVC can be d eterm in ed u sin g stan d ard sp irom etry. (6:2084) 218. (E) The p rim ary stim ulu s is pH . Carbon d ioxid e reacts w ith w ater to form hyd rogen ions; thu s, CO 2 is involved ind irectly. H ypoxia acts p erip herally at the level of the carotid bod ies. Ond ansetron is an antiem etic agent. (5:528) 219. (D ) The com p osition of alveolar gas d iffers from that of the inhaled gas since there is active CO 2 p rod u ction, O 2 consu m p tion, u p take or elim ination of inhalational anesthetic agents (d ep end ing on concentration on either sid e), and hu m id ification. N itrogen u nd ergoes no m etabolism and insignificant excretion, so at equ ilibriu m , no net transfer occu rs across the alveolar m em brane. (5:459, 596) 220. (D ) Equ al ventilation of both sid es is consistent w ith p rop er tu be p lacem ent, althou gh it d oes n ot gu aran tee it. Un equ al ven tilation obviou sly d em onstrates p oor tu be p lacem ent, or lu ng p athology. The d istance from the lip s to the carina in an ad u lt fem ale is abou t 25 cm . Therefore, the tu be d istance of 30 cm from the u p p er teeth is excessive. The carina lies at the lev el of th e 4th –5th th or acic v er tebr a. Therefore, the tu be p lacem ent by chest x-ray also is incorrect. (5:573) 221. (D ) Airw ay resistance is typ ically higher d u ring exhalation than inhalation. Factors influ encing resistance inclu d e the d iam eter

58

4: Re s pira tion

an d len gth of th e en d otrach eal tu be. In m echanically ventilated patients the resistance can be estim ated as the d ifference betw een p eak and plateau p ressure d ivid ed by inspiratory flow. (5:472) 222. (B) A right shift in the oxyhem oglobin d issociation cu rve is cau sed by hypertherm ia, acid osis, hypercarbia, and increases in DPG. Fetal hem oglobin w ill shift this cu rve to the left. (5:460) 223. (D ) The rate of pu lm onary com plications correlates strongly w ith all these factors, excep t asthm a. The existence of chronic or acu te pu lm onary d isease and the specific location of the incision can influ ence the level of p ostop erative pulm onary d ysfunction. H ow ever, risk of com p lication s is su rp risin gly low in w ellcontrolled asthm a and in patients treated preoperatively w ith corticosteroid s. (5:63) 224. (E) All of the op tions regard ing closing cap acity are tru e, excep t E. Closing cap acity is the su m of closing volu m e and resid ual volum e. As a fractional p art of the total lung capacity, it is d ecreased d u ring infancy and child hood . (5:255) 225. (D ) Transp u lm onary p ressu re is the p ressu re grad ient across the lu ng m easu red as the p ressu re d ifference betw een the airw ay op ening and the pleu ral su rface, and therefore is associated w ith tid al volu m e and lu ng volu m e. It is zero at FRC. (5:1415) 226. (D ) Vital cap acity is the fu ll exhalation from total lu ng cap acity w ithou t tim e lim it for the m aneuver. Closing capacity is the earliest test of sm all airw ay d isease, but requires gas analysis. Maxim al breathin g cap acity is effort d ep end ent. FEV1 is altered in obstructive d isease. To d eterm ine absolu te lu ng volu m es as op p osed to relative volu m e changes, bod y plethysm ography or an inert gas d ilu tion techniqu e is requ ired . (6:2209) 227. (C) Patients in the recovery room d o not norm ally requ ire sed ation. Op ioid s shou ld be u sed carefu lly in an am ount that w ill treat the

p ain bu t still allow th e p atient to breathe d eep ly and m obilize secretions, rather than at fixed intervals. Positioning is depend ent on the typ e of su rgery p erform ed ; invasive arterial blood p ressu re m onitoring is not ind icated in low -risk p atients after low -risk su rgery. (5:87) 228. (B) This d escribes a m ethod of em ergency ventilation that can be used to oxygenate a patient ad equ ately. If there is no obstru ction above, p neu m othorax is u nlikely bu t one of the p ossible com p lications of the p roced u re. Gastric d ilatation d oes not occu r, since the air w ill be vented to the atm osp here. Inju ry to the su rrou nd ing stru ctu res (larynx, esop hagu s, trachea) are p ossible com p lications. Ventilation m ay be inad equ ate to p revent hyp ercap nia. (5:572) 229. (D ) Pneumoconiosis is associated w ith fibrosis. Since a restrictive com p onent is p resent, the d iap hragm m ay m ove very little. The x-ray m ay look m u ch w orse than the fu nctional state. Decreased FEV1/ FVC is a m easu re for obstru ctive lu ng d isease. (6:2121) 230. (D ) Vital cap acity includ es tid al volu m e, closing volu m e, and insp iratory and exp iratory reserve volu m es bu t not all of FRC. FRC inclu d es resid u al volu me. (5:256; 6:2209) 231. (B) All of the factors cited m ay be involved in causing hypoxemia, except B. Decreased release of oxygen from hem oglobin to the tissu es is a cause of hypoxia. Blood loss, card iac outpu t, and shunting are involved in oxygen transport; airw ay obstru ction is involved in ventilation. (5:459) 232. (A) Diffu sion hyp oxia is d u e to an ou tp ou ring of nitrou s oxid e from the circu lation into the alveoli th at d isp laces oxygen and lead s to hyp oxia. It can easily be treated and p revented throu gh oxygen su p p lem entation. It occu rs only d uring the phase of nitrous oxid e elimination, m inutes after emergence from anesthesia. The second gas effect occu rs d u ring ind u ction and refers to enhanced u ptake of a second volatile anesthetic in the presence of nitrous oxid e.

Answe rs : 222–235

Carbon m onoxid e d oes not cau se d iffu sion hyp oxia. (5:605, 612) 233. (D ) Pneum othorax is cau sed by a loss of the sealed , negative pressu re in the pleu ral sp ace. Gas tracking throu gh fascial planes, a connection to the airw ays, a connection throu gh the chest w all, or intrathoracic tracheobronchial inju ry can all cau se p neu m othorax. Chest w all traum a can either prod uce a penetrating injury to the chest w all, or airw ay ru ptu re, or a rib fractu re, that in tu rn can p u nctu re the lu ng. (5:857; 6:2181) 234. (E) H ypoventilation, low card iac ou tpu t, V/ Q m ism atch, and intracard iac or p u lm onary shunting are problem s that are of the highest

59

im p ortan ce. Abn orm al d iffu sion is rarely im p ortant in this situ ation, and is associated w ith chronic lung d iseases, such as sarcoidosis. (5:459) 235. (E) Total p u lm onary com p liance is volu m e d ivid ed by p ressu re. N et p u lm onary com p liance rem ains relatively u nchanged in eld erly p eople d ue to a com bination of loss of tissue elasticity and stiffening of costochond ral joints, com pared to you nger ind ivid uals. Pu lm onary com p liance involves the lu ng and the chest w all. There is hysteresis in the alveolar exp ansion, so hyp oventilation and atelectasis w ill d ecrease com p liance. Com p liance is increased by su rfactant. (5:280, 470)

This page intentionally left blank

CHAPTER 5

Ne rvo us S ys te m Que s tions DIRECTION S (Qu estions 236-289): Each of the nu m bered item s or incom p lete statem ents in this section is follow ed by answ ers or by com pletions of the statem ent. Select the ON E lettered answ er or com pletion that is BEST in each case.

(A) exp osu re of bod y su rfaces to room tem peratu re air (B) ad m inistration of room tem p eratu re intravenou s flu id s (C) ad m inistration of m u scle relaxants (D) low hu m id ity of insp ired gases (E) high insp ired oxygen concentration

(A) (B) (C) (D) (E)

thiop ental m ethohexital ketam ine p rop ofol m id azolam

DIRECTION S: Use the follow ing figu re to answ er Qu estions 240-243:

P

237. Of the cranial contents contribu ting to intracranial p ressu re (ICP), w h ich one has th e sm allest volu m e?

239. A 22-year-old m an has an u nid entified parietal m ass and presents for open brain biopsy. H e has som e sym p tom s associated w ith a m ild increase in intracranial p ressu re. Which one of the follow ing ind u ction agents m ay increase his intracranial p ressu re even fu rther?

)

236. A 44-year-old w om an is u nd ergoin g total p elvic exen teration for m etastatic ovarian cancer. Several hou rs into the p roced u re her bod y tem p eratu re h as d rop p ed to 34.6°C d espite measures to keep her w arm. Core bod y tem p eratu re m ay d ecrease intraop eratively becau se of all the follow ing factors EXCEPT

(C) the p reganglionic neu rotransm itter is acetylcholine (D) target organ recep tors are only ad renergic (E) effects are m ed iated entirely throu gh cyclic AMP

C I ( e r u s s e r p l a i c

r

a

n

C

a

brain tissu e tissu e w ater venou s blood arterial blood cerebrosp inal flu id (CSF)

I

n

t

r

(A) (B) (C) (D) (E)

D

238. In the sym p athetic nervou s system (A) p reganglionic fibers synap se only in p aravertebral sym p athetic ganglia (B) p reganglionic cell bod ies are located throughou t the spinal cord

A

B Volume

61

62

5: Ne rvous S ys te m

240. Betw een A and B on the cu rve (A) there is a sm all ICP increase w ith increased intracranial volu m e (B) a sm all ICP increase ind icates p oor com pensatory effect (C) p ressu re increases are com p ensated for by increased blood flow (D) intracranial contents d o not effect ICP (E) com p ensatory m echanism s are not fu nctional 241. Intracranial p ressu re m easu rem ents sim ilar to those show n in the graph are obtained from a (A) (B) (C) (D) (E)

need le in the cau d al canal need le in the cervical ep id u ral sp ace catheter positioned in the lateral ventricle catheter p ositioned in the carotid sip hon catheter p ositioned in the ju gu lar bu lb

242. A patient at position B on the cu rve w ou ld be exp ected to (A) have clinical m anifestations of increased ICP (B) m ove tow ard p osition C w ith hyperventilation (C) have u nilateral m yd riasis (D) m ove tow ard p osition A w ith ad m inistration of isoflu rane (E) m ay benefit from ad m inistration of intravenou s hyp ertonic saline 243. At p osition C on the cu rve, a p atient (A) m oves tow ard position D w ith hyp erventilation (B) m oves tow ard position D w ith cou ghing (C) w ill not benefit from intravenou s m annitol (D) w ill likely be hyp otensive (E) w ill likely be tachycard ic 244. The ventilatory control centers (A) contain no exp iratory neu rons (B) receive no m echanical sensory inp u t (C) receive input from airway oxygen receptors

(D) are located in the m ed u lla and p ons (E) are activated by p erip heral CO 2 tension 245. Absorp tion of cerebrosp inal flu id (CSF) takes p lace throu gh (A) (B) (C) (D) (E)

ep end ym al cells arachnoid villi the p ia m ater the foram en of Monro the foram en of Magend ie

246. The sm allest nerve fiber, a p ostganglionic fiber associated w ith slow cond u ction, is the (A) (B) (C) (D) (E)

A-alp ha fiber A-beta fiber A-gam m a fiber B fiber C fiber

247. Integrity of all of the follow ing stru ctu res can be m onitored by sensory-evoked p otentials EXCEPT the (A) (B) (C) (D) (E)

d orsal colu m ns cerebellu m thalam u s vestibu lar-cochlear nerve inferior collicu lu s

248. A 61-year-old m an has d evelop ed a resting trem or in his right hand . H is neu rologist believes it is consistent w ith early Parkinson d isease. H e is therefore likely to have d ysfu nction w ithin the (A) (B) (C) (D) (E)

cerebellu m lateral ventricle p ons basal ganglia aqu ed uct of Sylvius

249. A nuclear group in the brain involved in transmission of sensory information to the cortex is the (A) cerebellu m (B) cau d ate (C) hyp othalam u s

Que s tions : 240–258

(D) thalam u s (E) hip p ocam p u s 250. All of the follow ing cranial nerves contain p arasym pathetic efferent fibers EXCEPT the (A) (B) (C) (D) (E)

ocu lom otor nerve (III) trigem inal nerve (V) facial nerve (VII) glossop haryngeal nerve (IX) vagu s nerve (X)

251. The electroencep halograp hic (EEG) w aveform w ith a frequ ency range of 8 to 13 H z is associated w ith (A) (B) (C) (D) (E)

alp ha activity beta activity gam m a activity d elta activity theta activity

252. In face of a d eclining su p p ly of nu trients (A) neu ronal fu nction d eteriorates in an allor-none fashion (B) there is no reserve below the norm al level of cerebral blood flow (C) irreversible neu ronal d am age occu rs w ith EEG evid ence of ischem ia (D) the actu al CBF level at w hich neu ronal fu nction d eteriorates varies w ith anesthetics (E) an isoelectric EEG ind icates irreversible neu ronal d am age 253. The su bstantia gelatinosa (A) has the highest concentration of op ioid receptors in the sp inal cord (B) is located in the lateral colum ns (C) is p art of the d orsal colu m n (D) is in the m otor area of the brain (E) is in the ventral colu m n 254. The ocu locard iac reflex involves all of the follow ing structu res EXCEPT (A) vagus nerve (B) trigem inal ganglion

63

(C) op hthalm ic d ivision of cranial nerve V (D) ocu lom otor nerve (E) brain stem 255. H orm onal p rod u cts of the anterior p itu itary inclu d e all of the follow ing EXCEPT (A) (B) (C) (D) (E)

grow th horm one lu teinizing horm one antid iu retic horm one follicle-stim u lating horm one thyrotrop in

256. After bringing a trau m atized p atient to the em ergency d ep artm ent, the p aram ed ic team reports that his Glasgow Coma Score is 11. This assessm ent is based on (A) eye op ening, verbal resp onse, and m otor resp onse (B) assessm ent of knee jerk and other m otor reflexes (C) assessm ent of p u p il size and brain stem reflexes (D) assessm ent of resp iration and au tonom ic brain stem fu nctions (E) assessm ent of EEG 257. An increase in Pa CO 2 from 40 to 50 m m H g w ill increase cerebral blood flow (A) (B) (C) (D) (E)

not at all 1 to 2 m L/ 100 g/ m in 5 to 10 m L/ 100 g/ m in 10 to 20 m L/ 100 g/ m in 25 to 50 m L/ 100 g/ m in

258. The oxygen reserves of the brain are (A) infinite (B) cap able of m aintaining fu nction for 25 m in (C) greater u nd er anesthesia (D) very low (E) carried p rim arily in the cerebral hem isp heres

64

5: Ne rvous S ys te m

259. Cerebral p erfu sion p ressu re m ay be estim ated by (A) (B) (C) (D) (E)

MAP + ICP SBP − ICP MAP − ICP MAP + CVP − ICP SBP − CVP MAP: m ean arterial blood pressu re ICP: intracranial p ressu re SBP: systolic arterial blood pressure CVP: m ean central venou s p ressu re

DIRECTION S: Use the follow ing case to answ er Qu estions 260-262: A 47-year-old m an is brou ght to the interventional rad iology su ite for insertion of a su prapu bic catheter to provid e long-term blad d er d rainage. H e su stained a C6 fractu re w ith transection of the cervical cord at age 23 in a m otor vehicle accid ent. H e takes no m ed ications bu t has had several u rinary tract infections treated w ith antibiotics. H e has no m otor function or sensation below the shou ld ers. H e is sed ated w ith 2 m g m id azolam and 50 m cg fentanyl. 260. In the patient w ith an inju ry at the C 6-7 level, one w ou ld exp ect (A) a m ajor loss of d iap hragm atic p ow er (B) no effective ventilation and ventilator d epend ency (C) d ifficu lty sw allow ing and p u lm onary aspiration (D) im p aired alveolar ventilation (E) p rofou nd brad ycard ia 261. On instillation of the blad d er w ith saline, the blood pressure rises from 128/ 75 to 221/ 120 m m H g and the heart rate falls from 87 to 45 bp m . What is the likely cau se of these changes to blood p ressu re and heart rate? (A) (B) (C) (D) (E)

resp onse to p ain au tonom ic hyp erreflexia m ictu rition reflex resp onse to cold solu tion in the blad d er anxiety

262. This event cou ld have been p revented by (A) (B) (C) (D) (E)

p retreatm ent w ith beta blockers lithotom y p osition instillation of bod y tem p eratu re flu id light general anesthesia sp inal anesthesia

263. A 38-year-old w om an is in the neu rological intensive care u nit follow ing ru p tu re of an aneu rysm of the right m id d le cerebral artery. A CT scan show s extensive subarachnoid hemorrhage. Which one of the follow ing find ings w ou ld be consid ered to be LEAST likely? (A) (B) (C) (D) (E)

hyp onatrem ia d ysrhythm ias norm al ICP cerebral vasosp asm ECG abnorm alities consistent w ith ischem ia

264. The basilar artery is form ed by the m erger of (A) (B) (C) (D)

the vertebral arteries branches of the internal carotid artery branches of the external carotid artery the anterior and p osterior com m u nicating arteries (E) the anterior and p osterior sp inal arteries 265. The celiac p lexu s (A) contains visceral and som atic afferent and efferent fibers (B) receives efferents from both sym p athetic and p arasym p athetic ganglia (C) innervates abd om inal viscera (D) is form ed from nerve roots from T7 throu gh T10 (E) m ay be blocked to relieve intractable angina p ain 266. Cerebrosp inal flu id (CSF) (A) is form ed in the choroid p lexu s by a p assive p rocess (B) fu nctions as a cu shion for the brain (C) is p rod u ced at a rate of abou t 100-200 m L/ h

Que s tions : 259–274

(D) p rod u ction is affected to the least extent by enflu rane of all of the volatile agents (E) norm ally contains p rotein at abou t onetenth the plasm a concentration 267. N eu rotran sm itters in th e cen tral n ervou s system that exert an inhibitory action on postsynaptic neurons inclu d e (A) d op am ine, glycine, serotonin, γ -am inobu tyric acid (GABA) (B) acetylcholine, d op am ine, histam ine (C) d op am ine, ep inep hrine, glu tam ate (D) acetylcholine, γ -am inobu tyric acid (GABA), glu tam ate (E) d op am ine, serotonin, ep inep hrine, histam ine 268. Low cerebral blood flow (CBF) m ay d isru p t ad equ ate brain p erfu sion, and one of the m ain d eterm inants of CBF is the cerebral m etabolic rate for oxygen (CMRO 2). Which one of the follow ing statem ents regard ing CMRO 2 and CBF is TRUE? (A) In adults, the normal rate of neuronal oxygen utilization is 30-40 mL/ 100 g/ min. (B) Blood flow in the brain is ind ep end ent of CMRO 2. (C) CMRO 2 reflects the am ou nt of oxygen u tilization in neu rons to prod uce ATP. (D) Oxygen u tilization is higher in the w hite m atter as com pared to the gray m atter. (E) Blood flow in the brain is cou p led to CMRO 2 in a ratio of 5:1. 269. A typical neuron has (A) (B) (C) (D) (E)

nu m erou s branching d end rites three or fou r axons to cond u ct im p u lses no nu cleu s in the cell bod y a d end ritic zone d evoid of recep tors a m yelin sheath that covers the entire cell

270. Propofol (A) has little effect on SSEP or MEP (B) can d ecrease CMRO 2, CBF, and ICP (C) d oes not cross the blood brain barrier

65

(D) increases the frequ ency of EEG (E) has little effect on seizu re activity 271. All of the follow ing are tru e concerning hyp ertherm ia EXCEPT that it (A) increases ICP (B) increases cerebral m etabolic d em and (C) w orsens ou tcom e from cerebral ischem ia (D) d ecreases cerebral blood flow (E) increases card iac ou tp u t 272. A typical reflex arc includ es all of the follow ing EXCEPT a (A) (B) (C) (D) (E)

sense organ afferent neu ron efferent neu ron synap se on a p erip heral effector ascend ing axon w ithin the ventral colum n

273. The electroencep halogram (EEG) (A) alone is su fficient for the d iagnosis of brain d eath, becau se a flat line is synonym ou s w ith d eath (B) show s d ecreased cortical activity w ith hyp ercapnia (C) is u naffected by hysterical seizu res (D) is u naffected by hyp otherm ia (E) is not u sefu l in the d iagnosis of a convu lsive d isord er 274. A 15-year-old girl is undergoing surgery to correct scoliosis. Intraoperative monitoring includes motor evoked potentials. These potentials (A) test integrity of the d escend ing corticospinal tracts (B) are u naffected by m u scle relaxants (C) cannot be p erform ed u nd er anesthesia w ith a volatile agent (D) are increased d u ring hyp otherm ia (E) are p rod u ced by electrical stim u lation w ithin the cerebellu m

66

5: Ne rvous S ys te m

275. When the craniu m is open, the ICP is (A) increased by volatile anesthetic agents (B) red u ced by hyp erventilation (C) d ep end ent u p on the p osition of the head relative to the heart (D) equ al to am bient p ressu re (E) cannot be m easu red by a ventricu lar catheter 276. The act of vom iting is an integrated activity that inclu d es (A) activation of the vom iting center in the cerebral cortex (B) op ening of the glottis (C) closing of the esop hageal and gastric card iac sphincters (D) activity m ed iated by cranial nerves VIII and XI (E) activation of the chem orecep tor trigger zone in the area postrem a 277. The com p ressed sp ectral array (CSA) record ing of the electroencep halogram (EEG) (A) is m ore accu rate than a stand ard EEG tracing (B) p resents d ata in a form at of am p litu d e, tim e, and frequ ency (C) is m ore accu rate for d eterm ination of sud d en events (D) requ ires no training for interp retation (E) shou ld be in u se for all neu rosu rgical proced ures 278. Which one of the follow ing is N OT involved in sensory transm ission? (A) (B) (C) (D)

d orsal root ganglion spinothalam ic tract p arietal cortex ventral posterior and ventral lateral nuclei of the thalam us (E) p recentral gyrus

279. In the ad u lt, the sp inal cord end s (A) at the low er bord er of the second sacral vertebra (B) at the low er bord er of the first lu m bar vertebra (C) at a segm ent in the low er lu m bar region d ep end ing u p on the p atient’s height (D) m id w ay betw een the 3rd and 4th lu m bar vertebra (E) at the lu m bosacral ju nction 280. The norm al brain (A) has a constant m etabolic rate (B) m aintains nearly constant blood flow betw een m ean arterial blood p ressu res of 80 to 120 m m H g (C) requ ires only glu cose and oxygen to m aintain fu nction (D) cou p les regional blood flow to m etabolic d em and (E) requ ires fru ctose for energy 281. The blood –brain barrier (A) (B) (C) (D) (E)

p erm its free p assage of bicarbonate ion d oes not inclu d e the brainstem is im p erm eable to carbon d ioxid e is p erm eable to m annitol is com p osed of tight ju nctions in the vascular end othelium

282. H yperventilation m ay lead to (A) cerebral vasod ilatation (B) red u ced Pa CO 2 in CSF (C) shift of the oxyhem oglobin d issociation cu rve to the right (D) increased card iac ou tp u t (E) m etabolic alkalosis 283. In administration of anesthesia to a patient with Arnold–Chiari malformation, all of the following considerations are important EXCEPT (A) cou ghing on the end otracheal tu be should be prevented (B) a m od erate increase in m ean arterial p ressu re m ay lead to su barachnoid hem orrhage

Que s tions : 275–289

(C) p ostop erative resp iratory d ep ression m ay be encou ntered (D) a coexistent syrix in the sp inal cord is com m on (E) extrem e flexion-extension of the neck shou ld be avoid ed 284. In the p atient w ith am yotrop hic lateral sclerosis, the anesthetic plan should includ e (A) succinylcholine but not non-depolarizing m uscle relaxants (B) non-d ep olarizing m u scle relaxants bu t not su ccinylcholine (C) aw areness of the p atient’s resp iratory lim itation (D) avoid ance of resp iratory d ep ressants (E) avoid ance of isoflu rane 285. Loss of cerebral au toregu lation (A) w ill cau se the cerebral blood p ressu re to fall w ith system ic hyp ertension (B) affects the entire brain at the sam e tim e (C) m ay be focal (D) only affects the low er end of the blood pressu re range (E) has no clinical significance 286. A 15-year-old girl w ith a history of m yotonic d ystrop hy is sched u led for tonsillectom y and ad en oid ectom y u n d er gen eral an esth esia. Consid erations regard ing her anesthesia m anagem ent inclu d e (A) p ostop erative hyp ertherm ia m ay p recip itate a m yotonic resp onse (B) m yotonia can be relieved by nond ep olarizing relaxants

67

(C) m echanical ventilation is contraind icated (D) m yotonia can be p recip itated by succinylcholine (E) neostigm ine p revents m yotonia 287. A 26-year-old m an is sched u led to u nd ergo elective resection of an arteriovenou s m alform ation. In planning the anesthesia, the anesthesiologist shou ld be prepared to m anage (A) blood pressu re d uring ind uction w ithin a very narrow range (B) m alignant brain sw elling (C) barbitu rate-ind u ced com a to d ecrease oxygen d em and prior to resection (D) vasospasm d uring su rgery treated w ith d eliberate hyp ertension (E) elevated ICP treated w ith the head -u p p osition 288. Brain tu m ors affect intracranial p ressu re by all of the follow ing p rocesses EXCEPT (A) (B) (C) (D) (E)

increasing intracranial tissu e content increasing cerebral m etabolic rate altering cerebral blood volu m e obstru cting CSF flow increasing interstitial flu id volu m e

289. Which is the m ost profou nd chem ical stim u lus for regu lation of cerebral blood flow ? (A) (B) (C) (D) (E)

m etabolic alkalosis hyp otherm ia hyp ertherm ia carbon d ioxid e hyp ercalcem ia

Answe rs a nd Expla na tions

236. (E) Oxygen concentration d oes not influ ence bod y tem p eratu re. Exp osu re to low am bient tem peratu re m ay cau se hypotherm ia. This is a problem esp ecially in sm all child ren becau se of their large bod y su rface to m ass ratio. Cold intravenou s flu id s and d ry insp ired gases are sou rces of heat loss and cause a fall in bod y tem peratu re. Mu scle relaxants prevent shivering, a p rincip al m eans by w hich the bod y m ain tain s a n orm al tem p eratu re. (1:528; 5:1507-9; 6:165-6)

239. (C) Ketam ine cau ses increases in intracranial p ressu re, cerebral m etabolism , and cerebral blood flow. It is relatively contraind icated in p atients w ith an intracranial m ass or increased in tracran ial p ressu re. Th e oth er in d u ction agen ts w ill d ecrease in tracranial p ressu re. (1:538-9; 5:699-700) 240. (A) On the flat p art of the com pliance cu rve, there is a sm all rise in pressure for increases in volum e. In this area of the curve, there is good com pensation. As one m oves to the right, there is a su d d en increase in p ressu re w ith sm all changes in volu m e. (5:873, 880; 6:2256-7)

237. (D ) Und er normal conditions, intracranial arterial blood volu m e is about 7 to 8 m L and constitu tes abou t 15% of the 50 m L of total intracranial blood volum e. Conceptually, brain tissue can be d ivid ed into solid m aterial (about 168 g or 12% of intracranial contents) and tissue w ater (about 1092 g or 78% of intracranial con tents). In tracran ial CSF volu m e is about 75 m L. Althou gh occu pying the sm allest volu m e w ithin the craniu m , cerebral blood volu m e is altered rapid ly by physiological and pharm acological intervention. (5:871-3; 6:2254)

241. (C) ICP is m easu red typ ically w ith a ventricu lostom y catheter p ositioned in the lateral ventricle or from a fiberotic intrap arechym al ICP m onitor. ICP only can be d eterm ined accu rately w ithin the craniu m; m easu rem ents from the spine m ay not accurately reflect intracranial events. Pressu re d eterm ined w ithin vascu lar stru ctu res w ill not reflect ICP. (5:875, 880; 6:2255)

238. (C) Preganglionic neu rons are located in the thoracic and lumbar spinal cord (T1 through L3) and synapse upon postganglionic nerves either in p aravertebral sym p athetic ganglia or in p lexi ad jacent to organs of innervation. Excep t for sw eat gland s that are cholinergic, target organ recep tors are ad renergic. β -ad renergic recep tors m ed iate their effects throu gh cyclic AMP; α-ad renergic recep tors act via a m ore com plex second m essenger system involving G proteins. All preganglionic neurons are cholinergic. (1:171-4; 5:1574; 6:3351)

242. (E) Althou gh this p atient is ap p roaching the steep p ortion of the intracranial com p liance curve, they may not manifest signs of increased ICP. Unilateral m yd riasis often occu rs w ith brain d istortion from extrem ely high ICP or herniation. H yp erventilation red u ces intracranial blood volu m e and p ressu re by vasoconstriction of cerebral arteries. Isoflu rane, in contrast, increases cerebral blood flow by causin g cerebral vasod ilation an d raises ICP. H yp ertonic saline red u ces ICP by red u cing brain w ater content. (5: 537-8, 880; 6:2255-7)

68

Answe rs : 236–254

243. (B) This patient is on the steep portion of the intracranial com p liance cu rve. Since com p ensatory mechanisms have been exhausted , small increases in the volume of intracranial contents (su ch as an increase in blood volu m e d u e to coughing) w ill prod uce large increases in intracranial pressure. Red uction in brain volum e by d ecreasing brain w ater content w ill m ove the p atient tow ard p osition A. The Cu shing’s resp onse to elevated ICP is hyp ertension, and som etim es brad ycard ia. (5:880, 893; 6:1868, 2256-7) 244. (D ) The ventilatory control centers, located in the pons and m ed ulla, integrates inform ation from p erip h eral m echan ical recep tors and p erip heral an d central ch em osen sors. The apneu stic center contains both insp iratory and exp iratory neu rons. Perip heral oxygen sensors are located in the carotid bod y (not the airw ay). Recep tors for h yd rogen ion are cen trally located ; w hen the hyd rogen ion concentration increases, ventilation is stim u lated . (5:528-9; 6:277, 288) 245. (B) The absorp tion of CSF takes p lace throu gh the arachnoid villi. The epend ym al cells and the p ia m ater are not involved . The foram ina of Monro and Magend ie are cond uits for CSF flow. (5:871-3) 246. (E) A very small postganglionic fiber w ith slow cond uction is the C fiber. (5:1520; 6:93, 3352) 247. (B) The cerebellum is not monitored by sensoryevoked p otentials. The d orsal colu m ns of the sp inal cord transm it p rop riocep tion and are assessed by som atosensory-evoked p otentials. The vestibu lar-cochlear nerve (cranial nerve VIII) and the inferior colliculus are assessed by brainstem au d itory-evoked p otentials. Peaks correspond ing to thalam ic nuclei are d etected w ith both. (5:484-6) 248. (D ) Parkinson d isease is d ue to loss of d opaminergic cells in the basal ganglia of the brain. A non -intention trem or m ay be a sym p tom . (5:149; 6:3317)

69

249. (D ) The thalam u s is located at the base of the brain an d con tain s m an y d istin ct n u clear grou p s. Sensory inform ation is p rocessed here before cond u ction to the cortex. Distinct p eaks corresp ond ing to the thalam u s are fou nd in som atosen sory an d brain stem au d itoryevoked p otentials. (5:484-8; 6:187) 250. (B) Th e trigem in al nerve contain s som atic afferent fibers from the face and su p p lies m otor innervation to the m u scle of m astication. The ocu lom otor nerve contains parasym p ath etic efferen ts th at con trol p u p illary constriction; the facial nerve contains parasym p athetic fibers su pp lying the su bm and ibu lar gland ; the glossop haryngeal nerve su p p lies the parotid gland ; and the vagu s nerve sup p lies the heart, resp iratory system , and gu t. (5:1208; 6:224, 243, 3352) 251. (A) Alpha activity has a frequency range of 8 to 13 H z. Beta activity has a frequ ency of greater than 13 H z. Theta rhythm has an activity of 4 to 7 H z. Delta rhythm has an activity of less than 4 H z. There are no gam m a w aves in the EEG. (5:475-7) 252. (D ) In hu m ans u nd ergoing carotid end arterectom y, volatile anesthetics have been show n to alter the cortical blood flow level at w hich EEG evid ence of ischem ia first d evelop s; halothane had the least effect, isoflu rane red u ced the threshold m ost, and enflurane w as interm ed iate betw een the tw o. Although neu ronal fu nction deteriorates progressively w ith d ecreasing blood flow, not u ntil CBF is less than half the norm al level (22 m L/ 100 g/ m in) d oes EEG evid ence of ischem ia m anifest. The EEG is isoelectric at 15 m L/ 100 g/ m in, bu t irreversible d am age d oes not occu r u ntil flow falls below 6 m L/ 100 g/ m in. (5:478-9, 1019; 6:2255) 253. (A) The su bstantia gelatinosa has the highest concentration of opioid receptors in the spinal cord . This area is in the d orsal horn of the spinal cord . (5:798, 1306-8, 1520-1, 1603-4) 254. (D ) The ocu locard iac reflex is chacterized by brad ycard ia (even to th e p oin t of asystole) p rod u ced by p ressu re or traction on the eye.

70

5: Ne rvous S ys te m

The afferent lim b of the reflex is com p osed of fibers from the op hthalm ic d ivision of the trigem inal nerve that synap se u p on brain stem neu rons and u ltim ately increase vagal efferent activity. Activation of the reflex can occu r from m anip u lation of any of the stru ctu res that carry afferent im pu lses, inclu d ing the trigem inal ganglion and the long ciliary nerves that innervate the globe. The oculomotor nerve is not involved in the reflex. (5:866, 1220; 6:3351-2) 255. (C) Antid iu retic horm one (ADH ) and oxytocin are secreted by the p osterior pitu itary. Grow th horm one, leutenizing horm one, follicle-stim u lating horm one, thyrotrop in, p rolactin, and ad renocorticotrop in are all secreted by the anterior pitu itary. (5:1127-31; 6:2876-7) 256. (A) The Glasgow Com a Score is based on the ability to open the eyes and verbal and m otor resp onses. Resp onses to knee jerk, p u p il size, resp iration, and EEG are assessed as p art of a general neu rologic exam ination bu t are not part of the Glasgow Com a Score. (5:1328, 1355; 6:3381) 257. (D ) Cerebral blood flow increases 1 to 2 m L/ 100 g/ m in w ith each 1 m m H g rise in Pa CO 2 in the normal physiologic range of Pa CO 2. For this reason it is important to have good airw ay control at all tim es w hen d ealing w ith p atients w ith increased intracranial p ressu re. (5:874; 6:2255-6) 258. (D ) The oxygen reserves are very low. The reserves are not changed und er anesthesia, bu t oxygen use is d ecreased . Since oxygen reserves are so low, the brain is su bject to hyp oxia w ith any bout of ischem ia. (5:1464-6; 6:2257) 259. (C) Cerebral p erfu sion p ressu re is the fu nd am ental concept und erlying the au toregu lation curve. It represents the blood pressure available for global p erfu sion of the brain. Becau se the brain is enclosed in the craniu m , w hich fu nctions as a noncom p liant container, CPP is d efined as the p ressu re of the blood entering the cranium (MAP) minus the pressure exerted w ithin the craniu m (ICP). This assu mes that central venous pressure is less than intracranial

p ressu re, as is the case w hen the head is elevated and there is no resistance to venous outflow from the craniu m . (5:871; 6:2255) 260. (D ) The p hrenic nerve arises from cervical segm ents C3–C5 and su p p lies m otor innervation to the d iap hragm . The intercostal m u scles receive m otor innervation from nerves originating in the thoracic cord . Althou gh d isru ption of the cervical cord betw een C6 and C7 w ou ld com p rom ise ventilation , som e d iap h ragm atic fu n ction w ou ld rem ain intact. Sym p athetic su p p ly to the heart is from the sympathetic ganglia that receive afferents from the thoracic cord below the injury; parasym pathetic innervation is from the vagu s and is u naffected by the inju ry. In the absence of h yp oxia, no d istu rban ce in h eart rate is exp ected . Cranial n erves IX and X su p p ly motor and sensation to the pharynx and glottis. (5:828; 6:2182-5) 261. (B) Au ton om ic hyp erreflexia occu rs w ith sp inal lesions above T5. Any noxiou s stim u lation (e.g., urinary catheter insertion or blad d er d istention) may lead to hypertension accompanied by sw eating and brad ycard ia. Anxiety or pain w ou ld p rod u ce hyp ertension and tachycard ia. (5:1138-9; 6:3356) 262. (E) Disru p tion of noxiou s afferent signals em anating from the blad d er to the intact low er spinal cord can prevent the au tonom ic hyp erreflexic resp onse. This can be accom p lished w ith d eep gen eral an esth esia or region al an esthesia. (5:1138-9; 6:3356) 263. (C) After su barachnoid hem orrhage, rhythm d istu rbances and ischem ic ECG changes are com m on. Sp asm of the cerebral arteries can resu lt in d ecreased cerebral blood flow w ith resulting cerebral ischem ia. H yponatremia can occur from SIADH (synd rome of inappropriate secretion of antid iu retic horm one). The m ass effect of hem atom a in the su barachnoid sp ace as w ell as obstru ction of CSF egress com m only lead s to hyd rocep halou s. (5:85-7; 6:2261-5) 264. (A) The left and right vertebral arteries m erge to form the basilar artery. (5:870)

Que s tions : 255–276

71

265. (C) The celiac p lexu s is a m ajor abd om inal p lexus com posed of a num ber of ganglia and innervates m ost of the abd om inal contents. Sym p athetic nerve fibers are d erived from T5 through T12 and convey only visceral afferent and efferent in form ation w ith no som atic inp u ts. (5:1134, 1574; 6:3352)

271. (D ) The consequences of hyperthermia inclu d e in creased cerebral blood flow an d ICP. Metabolic d em and is increased and cerebral ischem ia has been show n to have a w orse ou tcom e d u ring hypertherm ia. Card iac outpu t is increased to meet metabolic demand. (5:1469-71; 6:143-5)

266. (B) The m ajority of CSF is form ed in the choroid p lexu s by filtration and by active transport, the latter responsible for abou t tw o-third s of the total am ount. The active transport process requ ires energy. Transep end ym al d iffu sion from the brain interstitium also contributes to total volu m e. Continu ou s p rod u ction at a rate of 30-40 m L/ h resu lts in a com p lete tu rnover three tim es d aily. The brain and sp inal cord essentially float in this flu id . Enflu rane acts to increase p rod u ction and d ecrease absorption. The norm al p rotein concentration in CSF is about one-hund red th the plasma concentration. (5:873; 6:3435, 3600)

272. (E) A reflex arc inclu d es a sense organ, an afferent neu ron and efferent neu ron that form a synap se on a p erip heral effector su ch as a m uscle. Reflexes can be m ed iated by d escend ing inputs from higher centers w ithin the CN S. The ventral colu mn contains d escend ing fibers of the corticosp inal tract. (1:261, 265, 5:489-90)

267. (A) The major inhibitory transmitter in the central nervous system is GABA, bu t other transm itters m ay have inhibitory effects as w ell. Acetylcholine, glutam ate, histam ine, and epinep hrine are excitatory. (5:323, 474-5, 585-7; 6:3523) 268. (C) In ad u lts, the norm al rate of neu ronal oxygen u tilization is 3.0 to 3.8 m L/ 100 g/ m in that is cou pled to blood flow of 50 m L/ 100 g/ m in, resu lting in a ratio of 15:1. This cou p ling of CMRO 2 to CBF cau ses an increase in CBF d u ring neu ronal activity. Oxygen u tilization is higher in the gray m atter as com pared to the w hite m atter. (5:873-4) 269. (A) The typ ical neu ron has nu m erou s d end rites that branch out and a d end ritic zone that is rich in receptors. The neuron has a single cell bod y w ith an active nu cleu s and only one axon. The myelin sheath covers the axon only in m yelinated neu rons. (5:474-5, 1518-21; 6:95) 270. (B) Prop ofol d ecreases the frequ ency of EEG and d im inishes both the SSEP and MEP. It d ecreases cerebral metabolism, blood flow, and ICP. It is an effective anticonvu lsant. (5:875-6)

273. (C) The EEG is not sufficient for the d iagnosis of brain d eath. Brain d eath requires a careful history and p atient assessm ent. Patients w ho are hyp otherm ic or d eep ly anesthetized w ith barbitu rates, volatile agents, or p rop ofol m ay d isplay a flat EEG. Cortical activity increases w ith hyp ercap nia. H ysterical seizu res d o not p rod u ce a seizu re sp ike p attern on EEG. (6:2252-3) 274. (A) Motor evoked p otentials are p rod u ced by d irect transcranial electrical stim ulation of the m otor cortex; action potentials in contracting m uscles are m easured p eripherally. They test integrity of the ventral sp inal cord that contain s th e corticosp in al tracts. MEPs are d epressed by hypotherm ia, m u scle relaxants, and high concentrations of volatile anesthetics. (5:484-8) 275. (D ) ICP is prod uced w ithin a closed craniu m . Op ening of the craniu m at su rgery relieves intracranial p ressu re and ICP equ als am bient p ressu re or zero. While p ositioning of the head and anesthetic agents imp act up on brain relaxation, they d o not alter ICP. (5:880) 276. (E) The act of vom iting requ ires the activation of the vom iting center located in the reticu lar form ation of the low er m ed u lla near the area p ostrem a. It requ ires the opening of the esophageal and gastric card iac sp hincters. The p rocess is m ed iated by cran ial n erves IX an d X.

72

5: Ne rvous S ys te m

It m ay be initiated by stim u lation of the chem oreceptor trigger zone. (5:711; 6:301) 277. (B) The com p ressed sp ectral array p resents d ata from the EEG in a form at of am p litu d e, tim e, and frequency. Althou gh this form at is m ore convenient, som e sp ecificity and accu racy are lost. The CSA is not as accu rate for sud d en events. In spite of its convenience, a certain am ou nt of training and fam iliarity are need ed to interpret the tracing accurately. CSA is not necessary for every anesthetic. (5:1019) 278. (E) The precentral gyrus is p rim arily involved w ith m otor fu nctions. Sensation involves the d orsal root ganglion cell, the lateral sp inothalam ic tract, the thalam ic nu clei, and the p ostcentral gyru s. (5:1520-21; 6:187-8) 279. (B) The sp inal cord end s at the low er bord er of the first lum bar vertebra in the ad u lt. The su barachnoid sp ace continu es fu rther cau d ally and end s at S2. Although in child ren the cord end s in the low er lu m bar region and m oves rostral as the child reaches ad ulthood , the position of the end of the cord is not d epend ent u pon p atient height. (5:786) 280. (D ) In the norm al brain, m etabolic rate and blood flow are relatively constant. The au toregu latory range in w hich constant blood flow is m aintained occu rs betw een a m ean arterial blood pressu re of ap proxim ately 50 to 150 m m H g. Fru ctose is not a requ ired energy sou rce of the brain. While the brain requ ires only glu cose and oxygen to m aintain energy su pp ly, this is insu fficient to m aintain the stru ctu ral and fu nctional integrity. (5:871-4; 6:2255-7) 281. (E) The blood -brain barrier is p resent in all regions of the brain and sp inal cord and is form ed by tight ju nctions betw een vascu lar end othelial cells and foot p rocesses of the glia. Carbon d ioxid e freely crosses the blood -brain barrier; bicarbonate is charged and crosses the blood -brain barrier m ore slow ly. Mannitol is exclu d ed from norm al brain. (5:538, 874-5, 880; 6:2254-6)

282. (B) H yp erventilation red u ces the Pa CO 2 in CSF. It d ecreases card iac ou tp u t and constricts cerebral vascu latu re. The oxyhem oglobin cu rve is shifted to the left. The red uction in Pa CO 2 prod u ces resp iratory alkalosis; p rolonged hyp erventilation p rod u ces loss of bicarbonate ion and m etabolic acid osis. (5:528-9) 283. (B) The Arnold –Chiari m alformation is characterized by brain stem com pression from herniation of the cerebellar tonsils throu gh the foram en m agnu m and cau d al d isp lacem ent of the brain stem . H yd rocep halu s is com m on from obliteration of the foram ina of Lu shka and Magend ie that d rain the fou rth ventricle. In creases in in tracran ial p ressu re m ay be encou ntered . Extrem e flexion-extension of the neck m ay increase brain stem com p ression. Postop erative resp iratory and hem od ynam ic p atterns shou ld be m onitored closely becau se of possible brain stem com pression. The synd rom e is com m only associated w ith syrinx. (5:251, 1180, 1193; 6:127) 284. (C) The p atient w ith am yotrop hic lateral sclerosis is chronically w eak and has m uscle w asting from loss of d escend ing inp u t in the spinal cord . Succinylcholine should be avoid ed . N ond ep olarizing m u scle relaxants m ay have p rofou nd im p act; if requ ired , they shou ld be u sed in low d oses. If the p atient has any resp iratory com prom ise p ostop eratively, m echanical ventilation is ind icated . Althou gh these p atients m ay exhibit exaggerated changes in hemod ynam ics in response to inhalational anesthetics, there is no sp ecific recom m end ation to avoid these agents. (5:139, 500-1; 6:3345-7) 285. (C) Loss of cerebral au toregu lation w ill cau se cerebral blood flow to vary d irectly w ith blood pressure. This may be focal and only affect part of the brain. Loss of au toregu lation has great clinical significance w ith respect to blood pressu re and cerebral ischem ia. (5:873-4; 6:2255) 286. (D ) The patient w ith m yotonic d ystrophy presents an anesthetic p roblem w ith regard to m uscle relaxation. While succinylcholine m ay cau se m yotonia, m yotonic ep isod es are not n ecessarily reversed by n on d ep olarizin g

Que s tions : 277–289

m u scle relaxants. Fu rtherm ore, neostigm ine m ay aggravate m yotonia. Postoperative shivering in the recovery room m ay ind u ce m yotonia. At the end of the proced ure, m echanical ventilation is preferable to reversal of m uscle relaxants. (5:144-5; 6:3487-9) 287. (B) Arteriovenou s m alform ations are highflow, low -p ressu re lesions, and hyp ertensionind uced hem orrhage is rare, if not nonexistent. Vasosp asm is also rare. Intraop erative hem orrhage can be severe if ru p tu re occu rs. In ad d ition, reperfusion breakthrou gh (d evelopm ent of severe brain sw elling as a resu lt of the su d d en alteration in blood flow and perfusion p ressure in the brain parenchyma surrou nd ing the m alform ation) d u ring resection can resu lt in severe intraop erative p roblem s. Treatm ent m ay inclu d e ind u ced hyp otension and highd ose barbitu rates. (5:887-8; 6:3298)

73

288. (B) Becau se intracranial contents are not com p ressible, any increase in the m ass of intracranial m aterial w ill result in a rise in intracranial p ressu re u nless the volu m e of another com partment d ecreases. Ed ema in tissue surrou nd in g th e tu m or in creases in terstitial flu id volu m e and contribu tes to the rise in ICP. Cerebral blood volume may be either increased or d ecreased by tumor grow th. H yd rocephalus can resu lt from tu m or obstru ction of CSF ou tflow from the brain. (5:80-3; 6:2254-7) 289. (D ) The m ost p rofou nd chem ical stim u lu s affecting cerebral blood flow is carbon d ioxid e. Changes in electrolytes and acid -base balance m ay have an effect, but the p red om inant effect is from Pa CO 2. H ypothermia d ecreases cerebral blood flow by d ecreasing cerebral m etabolic rate. (5:873-4; 6:2255-7)

This page intentionally left blank

CHAPTER 6

Re nal, He patic , Endo c rine , He mato lo g ic , and Me tabo lic S ys te ms Que s tions DIRECTION S (Qu estions 290-365): Each of the num bered item s or incom plete statem ents in this section is follow ed by answ ers or by com pletions of the statem ent. Select the ON E lettered answ er or com pletion that is BEST in each case. 290. A 50-year-old p atient u nd erw ent a thyroid ectom y that finished at 1000, and at 2200 he comp lained to the nurse of d ifficu lty in breathing. She took his blood pressu re that w as m od erately elevated above previous d eterm inations, bu t she also noticed that his w rist flexed w hen the blood pressu re cu ff rem ained inflated . The cause of the strid or is probably (A) (B) (C) (D) (E)

vocal cord p aralysis p artial vocal cord p aralysis laryngeal ed em a cervical hem atom a hyp ocalcem ia

291. A 62-year-old m an w ith chronic renal failu re w ho is receiving d ialysis three tim es w eekly suffers a fracture of the hum erus in a fall. H e is brou ght to the op erating room for an op en red u ction of the fractu re on the evening of a d ay on w hich he m issed his d ialysis d ue to the inju ry. Exp ected abnorm alities in this p atient inclu d e all of the follow ing EXCEPT (A) (B) (C) (D) (E)

m etabolic acid osis hyp erkalem ia u rem ia throm bocytop enia hyp ervolem ia

292. A previou sly healthy 25-year-old m an su ffers a ru p tu red sp leen d u rin g a ru gby gam e. During the emergency splenectomy, he is given ten u nits of p acked red blood cells. After achieving hem ostasis, an accep table hem atocrit valu e of 27%, and eu volem ia via the ad m inistration of n orm al saline, the m ost likely electrolyte abnorm ality is (A) (B) (C) (D) (E)

hyp ercalcem ia hyp erm agnesem ia hyp op hosp hatem ia hyp okalem ia hyp erkalem ia

293. A 50-year-old alcoholic m an w ith a long history of tobacco abuse and w ho is chronically d yspneic at rest is in the recovery room after an em ergency ap p end ectom y. H e is receiving su pplem ental oxygen by m ask and appears to be hyp oventilating. An arterial blood gas is obtained that show s pH = 7.19, P O 2 = 85 m m H g, and P CO 2 = 90 m m H g. This state can best be d escribed as (A) p u re resp iratory acid osis (B) com bined resp iratory acid osis and m etabolic acid osis (C) resp iratory acid osis w ith com p ensating m etabolic alkalosis (D) m etabolic acid osis w ith com p ensating resp iratory alkalosis (E) p u re m etabolic acid osis

75

76

6: Re na l, He pa tic, Endocrine, He ma tologic, a nd Me ta bolic S ys te ms

294. A hyp otensive, com atose 47-year-old m an is brou ght to the op erating room after being struck by a bus. H e is emaciated and has a long history of alcoholism . It is imp ortant to ad m inister thiam ine to this p atient becau se thiam ine d eficiency m ay (A) be the cau se of the observed com a (B) be the cause of the observed hypotension (C) be p recip itated by the ad m inistration of glu cose-containing solu tions (D) cau se rhabd om yolysis if su ccinylcholine is given (E) p otentiate the card iovascu lar d ep ressant effects of volatile anesthetics 295. A 59-year-old woman requires treatment for nonH od gkin’s lymphoma. Antineoplastic chemotherapy with which one of the following agents may cause a peripheral neuropathy? (A) (B) (C) (D) (E)

m ethotrexate d oxoru bicin bu su lfan bleom ycin vincristine

296. A 20-year-old college stu d ent is fou nd d ead in his d orm itory room after consu m ing a large am ou nt of alcohol. When a lethal am ou nt of ethanol is ingested , the toxic effect of ethanol u sually lead ing to d eath is (A) hyp otension d u e to vasod ilation (B) hyp otension d u e to d ecreased card iac output (C) seizu re (D) ap nea (E) ventricu lar arrhythm ia 297. A 73-year-old patient is u nd ergoing im p lantation of a hip p rosthesis u nd er general anesthesia. She has been taking p red nisone for rheum atoid arthritis for 6 m onths. Du ring the p roced u re, there is a su d d en d rop in blood pressure. The first step to be taken is to (A) ad m inister hyd rocortisone 100 m g intravenou sly (B) establish the cau se of the hyp otension

(C) cancel the p roced u re (D) begin an infu sion of p henylep hrine (E) d iscontinu e all anesthetic agents 298. A 72-year-old m an w ith esop hageal carcinom a has been receiving total p arenteral nu trition w ith a solution containing 20% glucose and 4% am ino acid s for several w eeks p rior to su rgery throu gh a central venou s catheter in the left su bclavian vein. The patient is und ergoing a left thoracotom y for resection, and d u ring the p roced ure the m ed ical stu d ent hold ing the rib retractor accid entally rem oves the su bclavian catheter. Regard ing the infusion of total parenteral nutrition, the most appropriate maneuver to p erform d u ring the su rgical p roced u re w ou ld be to (A) d o nothing (B) p osition the p atient su p ine, insert a right su bclavian catheter, and restart the infu sion of p arenteral nu trition (C) restart the p arenteral nu trition solu tion via a perip heral intravenou s catheter (D) ad m inister 5 u nits of regu lar insu lin intravenou sly and m easu re the seru m glu cose concentration in 15 m in (E) begin an infu sion of 10% glu cose via a perip heral vein 299. A 36-year-old nu rse in good health is stu ck w ith a u sed need le d u ring the u nsu ccessfu l resu scitation of a h om eless m an w ith an u nknow n m ed ical history. She is at potential risk of all of these d iseases from exp osu re to a need le contaminated w ith blood or tissu e from an infected patient EXCEPT (A) (B) (C) (D) (E)

Creu tzfeld t–Jakob d isease hep atitis A hep atitis C cytom egaloviru s syp hilis

300. A 29-year-old anesthesia resid ent is fou nd d ead of an overd ose in his call room one morning. The follow ing statements are true concerning su bstance abu se am ong anesthesiologists EXCEPT

Que s tions : 294–306

(A) alcohol and fentanyl are the d ru gs m ost likely to be abu sed (B) a p rop er d ru g accou ntability system w ill prevent m isu se (C) an ind ivid u al confrontation w ith the d ru g u ser is not recom m end ed (D) the relap se rate of anesthesiology resid ents allow ed to reenter a p rogram is abou t 66% (E) it is the p olicy of the ASA to treat su bstance abuse d isord ers as a d isease 301. A 37 year-old -w om an w ith a history of sclerod erm a is brou ght to the hospital after a m otor vehicle accid ent and requ ires an em ergency lap arotom y for m anagem ent of intraabd om inal bleed ing. Problem s that m ay be encou ntered in sclerod erm a in clu d e all of th e follow ing EXCEPT (A) (B) (C) (D) (E)

lim ited m ou th op ening arterial d ilatation p u lm onary fibrosis contractu res p ericard ial effu sion

302. A 2-year-old child w ith H u rler synd rom e (gargoylism), a disturbance of mucopolysaccharide metabolism, needs anesthesia for an umbilical hernia repair. Anesthesia for such patients is complicated by all of the following EXCEPT (A) (B) (C) (D) (E)

d w arfism m acroglossia hyp ertelorism hep atosp lenom egaly short neck

303. An 80-year-old p atient has a norm al valu e of 1.0 m g/ d L for seru m creatinine. Com p ared w ith a 20-year-old p atient of the sam e w eight and w ith the sam e seru m creatinine value, the 80-year-old p atient has ap p roxim ately w hat fractional valu e of creatinine clearance? (A) (B) (C) (D) (E)

0.1 0.2 0.5 0.7 0.9

77

304. A 40-year-old alcoholic m an w ith a m assive gastrointestinal bleed is treated w ith a variety of blood com p onents. The blood com ponent w ith the least risk of transm itting hepatitis C is (A) (B) (C) (D) (E)

cryop recip itate fresh frozen p lasm a p acked red blood cells frozen w ashed red blood cells 5% albu m in

305. A 64-year-old man with a history of stable exertional angina for 3 years presents for preoperative evaluation before an elective hip replacement for severe degenerative arthritis. In terms of his preoperative medications or his need for additional evaluations prior to surgery, he (A) need s a card iac catheterization (B) need s a d obu tam ine stress echocard iogram (C) need s a m yocard ial p erfu sion scan (D) shou ld be taking a beta-blocker u nless there is a specific contraind ication (E) shou ld d iscontinu e his statin p reop eratively 306. You are the anesthesiologist assigned to the p reop erative testing area for the d ay. All of the follow ing statem ents regard ing stop p ing or continu ing m ed ications are tru e EXCEPT (A) p atients w ith coronary artery d isease shou ld id eally never stop asp irin becau se of a rebou nd p henom enon resu lting in an increased risk of p eriop erative card iac com p lications; risk of bleed ing versu s throm bosis need to be assessed (B) statins shou ld be continu ed and m ay even exert a p rotective effect in p atients u nd ergoing vascu lar surgery (C) m etform in shou ld be stop p ed 2 w eeks p reop eratively becau se of the risk of m etabolic acid osis (D) it is ad visable to stop fu rosem id e a d ay before su rgery becau se of an increased risk of hyp ovolem ia (E) birth control p ills shou ld be continu ed on the d ay of su rgery

78

6: Re na l, He pa tic, Endocrine, He ma tologic, a nd Me ta bolic S ys te ms

307. A 79-year-old m an w ith coronary artery d isease d evelop s acu te kid ney in ju ry after a carotid end arterectom y. The circulation to the kid ney is (A) au toregu lated over a m ean arterial p ressu re range of abou t 80 to 160 m m H g (B) not regu lated by neu ral factors (C) innervated by sym p athetic nerves originating in T2-T3 (D) not affected by ep inep hrine (E) constricted by p rostagland in E2 308. A 33-year-old w om an takes clarithrom ycin chronically for sinu sitis. After falling on ice, she su ffers a com p lex fractu re of her ankle. In the em ergency d ep artm ent, her ankle is too sw ollen to p erm it im m ed iate operative repair so it is casted and she is sched u led for surgery in a w eek. She is given a fentanyl p atch, 25 m cg/ h, for p ain. The follow ing m orning, she is fou nd d ead in bed . The m ost likely m echanism for her d eath is (A) excessive fentanyl effect d u e to excessive d ose (B) excessive fentanyl effect d u e to im p aired m etabolism (C) m yocard ial infarction (D) p u lm onary em bolu s of clot originating in inju red leg (E) bone m arrow em bolu s 309. A morbidly obese 42-year-old woman is scheduled for bariatric surgery. Which one of the following statements is true about the perioperative management of a morbidly obese patient? (A) Obese p atients have d ifferent fasting gu id elines com p ared to non-obese p atients. (B) N eck circu m ference is the single best p red ictor of p roblem atic intu bations in obese patients. (C) Postop erative continu ou s p ositive airw ay p ressu re increases the incid ence of m ajor anastom otic leakage after gastric byp ass su rgery.

(D) Patients w ith obesity-hyp oventilation synd rom e have a d ecreased sensitivity to the resp iratory d epressant effects of general anesthetics. (E) Su ccinylcholine shou ld be d osed based on lean bod y w eight in obese p atients. 310. A 59-year-old m an w ith m od erate chronic obstru ctive p u lm onary d isease (COPD) is sched u led for a nephrectom y for renal cell carcinom a. All of the follow ing statem ents abou t p atients w ith chronic pu lm onary d isease are correct EXCEPT (A) a p atient w ith severe bu t stable COPD shou ld rou tinely have preoperative pu lm onary fu nction tests before m ajor abd om inal or pelvic su rgery (B) to p revent intraop erative bronchosp asm in p atients w ith asthm a, p retreatm ent w ith a β 2-ad renocep tor agonist and / or an anticholinergic agent is ind icated (C) in p atients w ith severe asthm a, p rop ofol and ketam ine are the ind u ction agents of choice (D) ketam ine has little effect on ventilatory d rive in patients w ith COPD (E) in p atients w ith COPD, longer-acting neu rom uscu lar blockad e is associated w ith a greater risk of p ostop erative p u lm onary com p lications than u se of shorter acting neu rom u scu lar blockers 311. The liver affects glu cose m etabolism by all of the follow ing m echanism s EXCEPT (A) (B) (C) (D) (E)

glycogen storage glu coneogenesis glycogenolysis insu lin p rod u ction conversion of galactose to glu cose

312. A 55-year-old woman with recurrent nephrolithiasis is treated with extracorporeal shock wave lithotripsy. The shock wave during extracorporeal shock wave lithotripsy is timed to coincide with a particular point in the ECG, and occurs (A) at the p eak of the P-w ave. (B) 100 m sec after the p eak of the P w ave

Que s tions : 307–318

(C) 20 m sec after the peak of the R w ave (D) 200 m sec after the p eak of the R w ave (E) at the p eak of the T w ave 313. A 60-year-old m an w ith a history of w ellcontrolled typ e 2 d iabetes and a m yocard ial infarction 7 years ago w ith no recent card iac sym p tom s p resents for p reop erative evalu ation prior to prostatectom y. All of the follow ing statements about perioperative card iac risk assessm ent are tru e EXCEPT (A) m ost ischem ic events occu r p ostop eratively rather than intraop eratively (B) in term s of card iac risk stratification for noncard iac su rgery, intrathoracic and intrap eritoneal su rgeries are d eem ed interm ed iate risk (C) accord ing to the Am erican College of Card iology/ Am erican H eart Association (ACC/ AH A) gu id elines, patients w ith asym ptom atic typ e 2 d iabetes have the sam e risk of perioperative card iac com plications (PCC) as a p atient w ith a p reviou s MI (D) card iom egaly by itself is not a risk factor for PCC (E) renal insu fficiency is one of the five perioperative risk factors in the ACC/ AH A gu id elines 314. A 40-year-old alcoholic m an is fou nd to have m assive ascites. Ascites (A) follow s chronic d ecreased p ortal vein pressu re (B) follow s p eriod s of hyp eralbu m inem ia (C) is usually accompanied by hypernatremia (D) m ay have an ad verse card iop u lm onary effect (E) shou ld be rem oved rap id ly to avoid reaccu m u lation 315. An eld erly w om an is brou ght into the hosp ital after being fou nd on the floor of her ap artment. She is fou nd to have p rerenal failu re and her u rine w ill (A) be p ositive for nitrites (B) be concentrated

79

(C) have a sp ecific gravity of ap p roxim ately 1.010 (D) be excreted in large am ou nts (E) have a red d ish tinge d u e to p resence of red blood cells DIRECTION S: u se the follow ing scenario to answ er Qu estions 316-317: A 46-year-old w om an w ith m yxed em a is sched u led for em ergency intraabd om inal su rgery. 316. A find ing consistent w ith the m yxed em atou s state is (A) (B) (C) (D) (E)

fine, soft hair m oist skin brad ycard ia heat intolerance p itting ed em a of the eyelid s

317. Flu id and blood rep lacem ent in this p atient (A) shou ld be gu id ed by blood p ressu re (B) shou ld be gu id ed by electrocard iograp hic voltage (C) does not differ from that in normal patients (D) shou ld be gu id ed by invasive arterial and central venou s p ressu re m onitoring (E) shou ld be accom p anied by rap id restoration of the eu thyroid state 318. A 75-year-old w om an w ith aortic stenosis and d epression treated w ith fluoxetine presents for aortic valve replacem ent surgery. All of the follow ing statements about d epression are correct EXCEPT (A) St. John’s w ort is an herbal rem ed y often taken by patients to treat d ep ression (B) su ccessfu l treatm ent of d ep ression has been show n to correlate w ith a red u ced risk of p ostop erative d eath (C) it is estim ated that m ore than 75% of hosp italized eld erly patients su ffer from d epression (D) d ep ression m ay be a sym p tom of hyp ocalcem ia (E) d ep ression m ay be a sym p tom of Cu shing synd rom e

80

6: Re na l, He pa tic, Endocrine, He ma tologic, a nd Me ta bolic S ys te ms

319. A 60-year-old w om an p resents w ith ep isod ic hypertension to 180/ 105 associated w ith head ache and p rofu se sw eating. A d iagnosis of pheochrom ocytom a is m ad e. She (A) requ ires im m ed iate su rgery (B) shou ld be treated for 10 to 14 d ays w ith an α -ad renocep tor antagonist (C) can be anesthetized regard less of the level of blood p ressu re read ings (D) is u su ally hyp ervolem ic (E) shou ld have a Sw an-Ganz catheter in p lace preoperatively DIRECTION S: u se the follow ing scenario to answ er Qu estions 320-321: A 52-year-old w om an w ith a one-year history of typ e 2 d iabetes controlled on m etform in is sched uled for a partial colectom y for recu rrent d iverticu litis. 320. The m ost im p ortant goal in the treatm ent of a mid d le-aged d iabetic patient u nd ergoing anesthesia is to (A) (B) (C) (D) (E)

keep blood su gar in the norm al range p revent glycosu ria p revent hyp oglycem ia p revent ketoacid osis p revent acetonu ria

321. H yp oglycem ia in the aw ake p atient (A) is id entical to that in the anesthetized p atient (B) is characterized by brad ycard ia (C) is characterized by hyp ertension (D) is d u e to ketoacid osis (E) is characterized by a m arked p arasym p athetic resp onse 322. You are called to see a p atient on the m ed ical service in need of a cholecystectom y for acu te cholecystitis. The p atient w as ad m itted w ith abd om in al p ain an d h yp erosm olar com a. H yperosm olar com a (A) u su ally occu rs in you ng p eop le (B) occu rs at osm olar levels of 150 to 175 m Osm / L

(C) occu rs in the absence of ketonem ia (D) is u su ally accom p anied by oligu ria (E) requ ires treatm ent w ith large d oses of insu lin 323. A 33-year-old w om an w ith typ e 1 d iabetes w ho u ses an insu lin p u m p p resents for su rgical d rainage of a p ilonid al abscess. She has been fasting since the night before su rgery. All of the follow ing statem ents regard ing p erioperative m anagem ent of this p atient are correct EXCEPT (A) in general, it is best to m aintain the basal infu sion rate of insu lin (B) sched u led p rep rand ial insu lin bolu ses shou ld be om itted (C) glu cose levels shou ld be m onitored at frequ ent intervals (D) she shou ld resu m e her u su al d iet and insu lin therap y regim en as soon as possible p ostop eratively (E) u p take of insu lin from the p u m p is not affected by alterations in tissu e perfu sion 324. A 26-year-old otherw ise healthy female trauma patient is brou ght to the op erating room w ith m assive inju ries and requ ires m u ltip le u nits of blood to attempt to maintain euvolemia. Which one of the follow ing statem ents abou t blood transfusions in her is correct? (A) Cross-m atching is necessary for fresh frozen p lasm a (FFP). (B) Most p latelet transfu sions are given to treat d ilutional throm bocytopenia after m assive transfu sion. (C) H u m an error is the root cau se of m ost fatal hem olytic transfu sion reactions. (D) A unit of packed red blood cells should increase the hemoglobin level by 3 g/ dL. (E) FFP transfu sion shou ld be gu id ed by the IN R valu e.

Que s tions : 319–331

325. A 47-year-old p atient is to be op erated on for a tum or of the sm all bow el. In the p reoperative interview, a history of flu shing, d iarrhea, and joint p ain is elicited . There are also sym p tom s com p atible w ith congestive heart failu re. A likely d iagnosis is (A) (B) (C) (D) (E)

Zollinger-Ellison synd rom e carcinoid synd rom e p heochrom ocytom a Peu tz-Jeghers synd rom e ad renal tu m or w ith m etastasis

326. A 59-year-old m an w ith cirrhosis need s hip rep lacem ent su rgery for severe osteoarthritis. Which one of the follow ing statem ents regard ing cirrhotic p atients is correct? (A) (B) (C) (D) (E)

The serum albumin level will be elevated. Excessive sod iu m is lost in the u rine. Pancu roniu m is m ore effective. Serum gamma globulin level will be low. Less thiopental is required for induction.

327. The op tim al anesthetic regim en for a 50-yearold patient w ith fulminant hepatic failure from hepatitis C u nd ergoing liver transplantation (A) (B) (C) (D) (E)

w ill avoid fentanyl w ill avoid nond ep olarizing relaxants is a balanced techniqu e w ill avoid halogenated hyd rocarbons w ill d ep end on the cau se of the liver failu re and the p atient’s statu s

328. A 35-year-old w om an w ith hyperp arathyroid ism u nd ergoes a p arathyroid ectom y. Featu res of hypocalcem ia relevant to the perioperative p eriod inclu d e all of the follow ing EXCEPT (A) hyp ocalcem ia p red isp oses to laryngosp asm (B) hyp ocalcem ia can p resent as change in m ental status (C) hyp ocalcem ia can lead to d ifficu lt to treat hyp ertension in the p ostop erative setting

81

(D) one gram of calciu m chlorid e solu tion p rovid es three tim es the am ount of elem ental calciu m p resent in one gram of calcium glu conate (E) concentrated calciu m solu tions m ay be cau stic to p eripheral veins 329. A 43-year-old m ale p atient w ho has had a transsp h enoid al hyp op hysectom y for acrom egaly several years ago p resents for cholecystectom y. Wh ich on e of th e follow in g hormones should be given in the perioperative p eriod ? (A) (B) (C) (D) (E)

ACTH TSH vasop ressin cortisol insu lin

330. A 39-year-old d iabetic m an is sched u led for a p od iatric proced ure. Which one of the follow ing statements is TRUE regard ing the effects of m ed ications on blood su gar? (A) Propranolol potentiates the hyperglycem ic response to stress. (B) Lisinop ril increases insu lin requ irem ents in the d iabetic. (C) H yd rochlorothiazid e p otentiates the hyp oglycem ia p rod u ced by glybu rid e. (D) Atenolol d ecreases insu lin release from the pancreas. (E) Pred nisone increases the blood glu cose concentration. 331. Cryop recip itate contains all of the follow ing clotting factors EXCEPT (A) (B) (C) (D) (E)

factor VIII factor IX factor XIII von Willebrand factor fibrinogen

82

6: Re na l, He pa tic, Endocrine, He ma tologic, a nd Me ta bolic S ys te ms

332. The treatm ent of a hem olytic transfu sion reaction in a m id d le-aged fem ale p atient w ho received blood d u ring a su rgical p roced u re m ay involve the im m ed iate ad m inistration of all of the follow ing EXCEPT (A) (B) (C) (D) (E)

crystalloid intravenou s flu id s fu rosem id e hyd rocortisone sod iu m bicarbonate m annitol

333. An eld erly m an w ith u rosep sis d evelop s d issem inated intravascu lar coagu lation (DIC). Wh ich on e of th e follow ing statem ents is FALSE regard ing DIC? (A) DIC is u su ally d u e to the abnorm al consu m ption of clotting factors. (B) DIC m ay be treated w ith hep arin. (C) Gram -negative end otoxem ia is a com m on cau se of DIC. (D) Abnorm alities in laboratory tests inclu d e a p rolonged p rothrom bin tim e and d ecreased valu es for p latelet cou nt and p lasm a fibrinogen. (E) Regard less of the etiology, therap y of DIC is d irected tow ard rep lacem ent of clotting factors and inhibition of the clotting cascad e. 334. A mid d le-aged m an w ith long-stand ing hypertension and chronic renal insu fficiency on a beta-blocker and simvastatin requires an intraop erative angiogram w hile u nd ergoing a vascu lar p roced u re. What sh ou ld be d on e to d ecrease h is risk of d evelop ing contrastind u ced acu te tu bu lar necrosis? (A) Ad m inister a flu id bolu s of norm al saline p rior to the proced u re. (B) Stop his statin a w eek before su rgery. (C) Start a d iu retic on the m orning of surgery. (D) Start a calciu m channel blocker im m ed iately after the p roced u re. (E) Start a beta-blocker at a low d ose one w eek before su rgery.

335. Which one of the follow ing p atients requ ires stress d oses of steroid s on the d ay of su rgery to d rain a p ostop erative w ou nd infection a w eek after a p artial colectom y for recu rrent d iverticu litis? (A) A p atient w ho u ses top ical steroid s d aily for eczem a on both elbow s as need ed . (B) A p atient w ho u ses inhaled bu d esonid e d aily in the sp ring and fall for allergyrelated asthm atic sym p tom s. (C) A p atient w ho took oral p red nisone for three w eeks to treat her severe p oison ivy 9 m onths ago. (D) A p atient w ho w as treated w ith one d ose of d exam ethasone last w eek for p ostoperative nau sea. (E) A p atient treated w ith steroid s for lym p hom a 3 years ago. 336. Periop erative m anagem ent of a 47-year-old m an w ith carcinoid synd rom e u nd ergoing a p rostatectom y shou ld inclu d e (A) infu sion of ep inep hrine to treat bronchosp asm (B) avoid ing the u se of vasop ressin to treat intraop erative hyp otension (C) the p referential u se of m u scle relaxants in the benzylisoqu inoline class as opposed to those in the steroid class (D) ad m inistration of octreotid e preoperatively (E) starting a calciu m channel blocker one w eek before su rgery 337. All of the follow ing facts abou t antid iu retic horm one (ADH ) are tru e EXCEPT (A) release is u nd er control of osm orecep tors in the hypothalam u s (B) release is from the p osterior p itu itary (C) release is inhibited by increased stretch of the atrial barorecep tors (D) acts on the p roxim al convolu ted tu bu le (E) cau ses increased reabsorp tion of free w ater from the kid ney

Que s tions : 332–344

338. A 50-year-old m an w ith lon g-stan d in g obstru ctive sleep ap nea w ho u ses a CPAP m achine interm ittently at hom e and need s a cholecystectom y (A) w ou ld benefit from a short-acting benzod iazep ine given at bed tim e for the w eek before su rgery (B) shou ld be kep t intu bated and m echanically ventilated for the night follow ing su rgery (C) is at high risk for d evelop ing p ostop erative hypoxem ia (D) w ou ld u su ally requ ire large d oses of opioid s to treat p ostop erative pain (E) shou ld com p letely stop u sing CPAP for a w eek prior to su rgery 339. The p eriop erative m anagem ent of a m id d leaged m an w ith gou t u nd ergoing knee replacem ent shou ld inclu d e the avoid ance of (A) (B) (C) (D) (E)

local anesthetics in the am id e class su ccinylcholine β-ad renergic antagonists hyp ovolem ia etom id ate

340. Which one of the follow ing abnormalities d oes not cau se a hypercoagulable state? (A) (B) (C) (D) (E)

Factor V Leid en Protein S d eficiency Protein C d eficiency Antithrom bin III d eficiency Von Willebrand d isease

341. You are called to see a 57-year-old patient in the recovery room w ith a seru m p otassiu m concentration of 5.9 m Eq/ L after a colonic resection for colon cancer. Treatm ent of hyp erkalem ia inclu d es all of the follow ing EXCEPT (A) elim ination of exogenou s sou rces (B) correction of cau se of end ogenou s sou rces

83

(C) ad m inistration of glu cose w ith insu lin (D) ad m inistration of acid ifying solutions (E) ad m inistration of calciu m glu conate 342. Regard ing a you ng ad u lt w ith Du chenne muscular d ystrophy w ho is to und ergo general an esth esia, all of th e follow in g are tru e EXCEPT (A) the p atient often has d elayed gastric em p tying (B) the p atient has an increased risk of m alignant hypertherm ia (C) the p atient can safely receive nond ep olarizing m u scle relaxants (D) the patient is usually a difficult intubation (E) ineffective cou gh from d im inished resp iratory m u scle strength can cau se retention of secretions post-operatively 343. Relatively rare d iseases that are m u ch m ore com m on in patients w ith the acquired im m unod eficiency synd rom e inclu d e all of the follow ing EXCEPT (A) p neu m onia d u e to Pneumocystis carinii (B) Ad d ison d isease (C) infection d u e to M ycobacterium avium com p lex (D) B-cell lym p hom a (E) Kap osi’s sarcom a 344. A 56-year-old w om an w ith a d iagnosis of the synd rom e of inappropriate secretion of antid iu retic horm one (SIADH ) requ ires a m astectom y for recently d iagnosed breast cancer. Find ings seen in patients w ith SIADH includ es all of the follow ing EXCEPT (A) (B) (C) (D) (E)

hyp ernatrem ia low seru m osm olality excessive renal secretion of sod iu m norm al renal fu nction absence of ed em a

84

6: Re na l, He pa tic, Endocrine, He ma tologic, a nd Me ta bolic S ys te ms

345. A 26-year-old m an w ith severe inflam m atory bow el d isease is being treated w ith com p lete bow el rest for several w eeks. A nutritionist recom m end s parenteral nutrition. All of the follow in g facts abou t p aren teral n u trition ad m inistered via a p erip heral vein are tru e EXCEPT that p erip heral nu trition (A) IV catheters shou ld be changed every three d ays (B) has a low incid ence of p hlebitis if the d extrose concentration is kept below 10% (C) u ses lip id as a sou rce of calories (D) u ses p rotein as a sou rce of calories (E) can be stop p ed d u ring p atient transp ort if it w ill be restarted w ithin an hou r 346. The blood su p ply to the liver is by tw o vessels, the hepatic artery and the hepatic portal vein. Which one of the follow ing statem ents regard ing hep atic circu lation is correct? (A) 60% of the blood su pp ly com es from the hepatic artery. (B) The p ortal vein provid es 90% of the oxygen su pply. (C) Portal vein blood has a higher oxygen satu ration than the hepatic artery. (D) The p ortal vein su p p lies the bu lk of the nu trients to the liver. (E) The hepatic artery is more important than the portal vein in transporting orally administered medications to the liver. 347. A 61-year-old w om an w ho had a cad averic kid ney transp lant seven years ago now p resents for knee rep lacem ent su rgery. All of the follow ing statem ents abou t im m u nosu p p ression are correct EXCEPT (A) im m u nosu p p ression m ay occu r w ith p rotein–calorie m alnu trition (B) renal transp lant recip ients have a higher incid ence of acu te rejection and im m u nologic graft loss than recip ients of any other solid organ (C) op ioid s ind u ce im m u nosu p p ression in p art by d ecreasing natu ral killer cell activity and antibod y prod u ction

(D) cyclosp orine u se after renal transp lantation m ay cau se hypertension, hyperlipid em ia, and accelerated atherosclerosis in renal transp lant p atients (E) the m ost com m on cau se of invasive fu ngal infections in im m u nocom p rom ised p atients is Candida sp ecies 348. A 59-year-old m an w ith recent cholangitis u nd ergoing an op en cholecystectom y u nd er general an esthesia d evelop s in traop erative hypotension u nresponsive to vasopressors and flu id s. There is no sign of hem orrhage. What d ru g shou ld you ad m inister at this p oint? (A) (B) (C) (D) (E)

hyd rocortisone 100 m g intravenou sly regu lar insulin 5 u nits intravenou sly glu cagon 1 m g intravenou sly levothyroxine 100 m cg intravenou sly 50 m L of 50% glu cose intravenou sly

349. When evalu ating renal fu nction in a 75-yearold healthy m an u nd ergoing p reop erative assessm ent for an elective orthop ed ic p roced u re, one m u st consid er that (A) p roteinu ria is alw ays p athological (B) a sp ecific gravity of 1.023 or greater d em onstrates good concentrating fu nction (C) BUN elevation is alw ays ind icative of renal d ysfu nction (D) less creatinine is p rod u ced by m u scu lar persons (E) by the eighth d ecad e of life, GFR is red u ced by 85% in otherw ise healthy ad u lts 350. A 40-year-old w om an w ith no history of d iabetes d evelops hyperglycemia d uring abd om inal su rgery. Mechanism s by w hich blood glucose can increase d u ring su rgery includ e all of the follow ing EXCEPT (A) intravenou s ad m inistration (B) secretion from the liver secondary to catecholamine-stimulated glycogenolysis (C) increased catecholam ine ou tp u t inhibiting glu cose u p take by insu lind epend ent tissues

Que s tions : 345–357

(D) increased catecholam ine ou tp u t stim u lating p ancreatic insu lin secretion (E) stress-ind u ced cortisol release from the ad renal cortex 351. All of the follow ing statements about a 47-yearold m ale patient w ith d iabetes insip id u s (DI) are correct EXCEPT (A) (B) (C) (D) (E)

the seru m sod iu m is high the osm olality of the seru m is high the u rine is concentrated thirst need not be p resent pituitary DI is treated with desmopressin

352. A 39-year-old man has been diagnosed with prim ary hyperald osteronism. H e is expected to have all of the following manifestations EXCEPT (A) excess secretion of horm one from the ad renal m ed ulla (B) hyp ertension (C) m etabolic alkalosis (D) hyp okalem ia (E) inability to concentrate u rine 353. A 49-year-old w om an w ith hyp erthyroid ism d u e to Graves d isease is having a thyroid ectom y. She (A) is likely to have an increased MAC (B) is likely to have a p alp able nod u le (C) shou ld be heavily sed ated in the p reop erative p eriod (D) shou ld not be treated w ith beta blockers periop eratively (E) shou ld be p retreated w ith iod id e one hour before su rgery 354. A 56-year-old m an w ith Gilbert synd rom e is sched uled for a bone marrow harvest for d onation to his sister. Im portant consid erations in the perioperative period for this patient should inclu d e (A) avoid ance of p reop erative sed ation (B) p reop erative transfu sion of fresh frozen plasm a (C) p reop erative ad m inistration of m u ltip le antiem etic m ed ications

85

(D) recognition of the benign etiology of the laboratory abnorm ality (E) liver biop sy for confirm ation of the d iagnosis 355. A 52-year-old w om an u nd ergoes a total thyroid ectom y for a toxic goiter. Up on arrival to the PACU her tem peratu re is 38°C; all other vital signs are norm al. Thirty minutes later you are called by the patient’s nu rse w ho tells you the patient is now in atrial fibrillation w ith a heart rate of 140. What m ed ication shou ld you ord er first? (A) (B) (C) (D) (E)

m etop rolol am iod arone fu rosem id e nitroglycerin d iltiazem

356. A 20-year-old African-Am erican w om an w ith sickle cell d isease d evelops severe knee pain after p elvic su rgery for a tu bal p regnancy. She tells you this pain is typical of her sickle cell crises and u su ally resp ond s w ell to op ioid s. The sickling of red blood cells in this p atient (A) is a reversible p rocess (B) occu rs w hen oxygenated hem oglobin m olecu les precipitate (C) im p airs the clotting cascad e (D) m ay cau se infarction in tissu es w ith a high oxygen extraction ratio (E) m ay be related to acu te sp lenic infarction given her age 357. A 57-year-old m an w ith cirrhosis d evelop s hep atorenal synd rom e. Regard ing the hep atorenal synd rom e, all of the follow ing are true EXCEPT (A) it is associated w ith kid neys that are norm al on biop sy (B) typ e I is an ind ication for u rgent liver transplantation (C) it is associated w ith oligu ria (D) it may be readily diagnosed by the abnormalities in urinary sediment analysis (E) the p rognosis is p oor

86

6: Re na l, He pa tic, Endocrine, He ma tologic, a nd Me ta bolic S ys te ms

358. A 19-year-old m ale p atient arrives at the hospital w ith a gunshot w ou nd through the liver and is brou ght im m ed iately to the op erating room hyp otensive and still bleed ing heavily. H e rem ains hypotensive follow ing fluid resu scitation w ith 3 L of lactated Ringer solu tion. Flu id resu scitation shou ld continu e u sing (A) u ncrossm atched O-negative p acked red blood cells (B) p latelets (C) 5% albu m in (D) fresh frozen p lasm a (E) cryop recip itate 359. A 36-year-old w om an w ith p olycythem ia vera (PV) p resents for a d iagnostic lap aroscop y as part of an infertility w ork-u p. All of the follow ing statem ents abou t PV are correct EXCEPT (A) PV is the m ost com m on of the chronic m yelop roliferative d isord ers (B) PV is m ost often d iagnosed by the incid ental find ing of a high hem oglobin and hem atocrit (C) chem otherap y is the u su al first line treatm ent for this cond ition (D) if u ntreated , erythrocytosis cau ses hyperviscosity that can lead to throm bosis (E) the goal of treatm ent in fem ale p atients is to m aintain the hem oglobin level d esflu rane > isoflu rane > halothane (B) d esflu rane > sevoflu rane > isoflu rane > halothane (C) isoflu rane > halothane > d esflu rane > sevoflu rane (D) halothane > isoflu rane > sevoflu rane > d esflu rane (E) halothane > sevoflu rane > d esflu rane > isoflu rane DIRECTION S: Use the follow ing figu re to answ er Qu estions 382-383:

A

R

e

s

p

o

n

s

e

B

97

(D) increases the efficacy of d ru g X (E) d ecreases the p otency of d ru g X 384. A 63-kg p atient w ith a history of glau com a w ho su ffered a second d egree bu rn of 10% of the total bod y su rface area in a m otor vehicle accid ent is u nd ergoing su rgery for intram ed u llary n ailin g of an op en tibia fractu re 18 h after the accid ent u nd er general end otracheal anesthesia. Ind u ction of anesthesia w as accom p lished by ad m inistration of fentanyl 2 m cg/ kg, p rop ofol 2.5 m g/ kg, and su ccinylcholine 1 m g/ kg intravenou sly. Antibiotic prop hylaxis consisted of gentam icin 5 m g/ kg in traven ou sly p rior to in cision . Prolon ged d u ration of neurom uscu lar blockad e after succinylcholine in this p atient w ou ld be m ost likely d u e to (A) p otentiation of the neu rom u scu lar blocking effects by gentam icin (B) the bu rn su ffered in the m otor vehicle crash (C) chronic treatm ent of glau com a w ith top ical isoflu orop hate (D) p rolonged im m obilization 385. Desflu rane

log dos e

382. If A and B are two different medications producing the same effect, then the figure shows that (A) A has higher efficacy than B (B) A has low er efficacy than B (C) A and B m u st act via d ifferent recep tors to prod u ce their effects (D) A is m ore p otent than B (E) A has a higher ED 50 than B 383. If A d ep icts the d ose-response relationship of d ru g X acting alone, and B d ep icts the d oseresponse relationship of d rug X in the presence of d ru g Y, then it can be said that d ru g Y (A) is a com p etitive antagonist of d ru g X (B) is a noncompetitive antagonist of drug X (C) m u st act via a d ifferent recep tor than d ru g X

(A) is highly lip id solu ble (B) has a MAC of 6% w hen ad m inistered w ith O 2 (C) is highly w ater solu ble (D) is not irritating to the airw ay (E) m etabolism releases significant qu antities of fluorid e ion 386. A patient sched uled for an em ergency proced u re gives a history of heroin abuse. A problem th at m u st be an ticip ated in th e p eriop erative p eriod is (A) d elayed gastric em p tying (B) p resence of left ventricu lar hyp ertrop hy (C) p rogressive sensory-m otor polyneurop athy (D) d ecreased need for anesthetics (E) increased sensitivity to catecholam ines

98

7: P ha rma cology

387. Local anesthetics have their effect at the (A) (B) (C) (D) (E)

p resynap tic nerve term inal p ostsynap tic nerve term inal GABA recep tor m em brane calciu m channel

388. The action of p rop ofol after injection is term inated by (A) its elim ination u nchanged by the kid neys (B) its biotransform ation by the liver (C) its being bou nd to p roteins (D) its red istribu tion (E) being taken u p in fatty tissu es 389. Methohexital (A) is m etabolized to a greater extent than thiopental (B) is converted to active m etabolites (C) has a longer term inal half-life than thiopental (D) cau ses histam ine release from m ast cells (E) is contraind icated in asthm a 390. The first-p ass effect refers to (A) the biotransform ation of a d ru g in its vehicle of ad m inistration (B) the change of a d ru g by enzym es in m u scle (C) biotransform ation of a d ru g as it passes throu gh the intestinal m ucosa and liver (D) the d ru g lost by urinary excretion (E) the d ru g lost by fecal excretion 391. An exp ected card iovascu lar change after ketam ine ad m inistration is (A) elevated d iastolic p ressu re, norm al systolic pressure (B) elevated d iastolic and systolic p ressu re (C) d ecreased d iastolic and systolic p ressu re (D) d ecreased d iastolic p ressu re, increased systolic p ressu re (E) no change in blood p ressu re

392. A sm all d ose of a nond ep olarizing m u scle relaxant given 3 m in before an intu bating d ose of su ccinylcholine (A) increases the d ose of su ccinylcholine requ ired (B) w ill not p revent the rise in intracranial pressu re (C) is u sefu l in p reventing arrhythm ias (D) d ou bles the tim e to recovery from neu rom u scu lar blockad e (E) p erm its faster intu bation 393. A d ru g is ad m inistered by bolus intravenou s injection. Ap p roxim ately w hat p ercentage of the d ru g rem ains after fou r half-lives have elap sed ? (A) (B) (C) (D) (E)

1% 4% 6% 10% 12%

394. Flecainid e is (A) (B) (C) (D)

an antiarrhythm ic d ru g in Class IC a lid ocaine analog a m u scle relaxant the d ru g of choice for local anesthesia reactions (E) ad m inistered only orally 395. The ad m inistration of anticholinesterase d rugs w ill (A) p rolong all neurom uscular blockad e (B) alw ays reverse nond epolarizing agents (C) shorten the block of a d epolarizing agent (D) reverse nond epolarizing agents if the p lasm a concentration of the d rug is low enou gh (E) reverse the action of a d epolarizing agent if only partial p aralysis is present 396. A patient is ad m itted for em ergency orthoped ic su rgery. Prelim inary d ata show a BUN valu e of 85 m g/ d L and seru m p otassiu m of

Que s tions : 387–403

6.0 mEq/ L. The least d esirable d rug for intubation w ou ld be (A) (B) (C) (D) (E)

rocu roniu m vecu roniu m cisatracu riu m m ivacu riu m p ancu roniu m

397. The m ost sensitive test to d eterm ine ad equ ate recovery from neu rom u scu lar blockad e is (A) (B) (C) (D)

five-second head lift five-second hand grip insp iratory force tactile resp onse to d ou ble-bu rst stim u lation (E) tactile resp onse to train-of-fou r

99

(D) has an increased sensitivity to nond ep olarizing relaxants (E) has an increased sensitivity to d ep olarizing relaxants 401. Labetalol (A) is a nonselective α-ad renergic recep tor blocker (B) has intrinsic sym p athom im etic activity at β 2-ad renergic recep tors (C) u nd ergoes significant p lacental transfer and shou ld not be u sed in p regnancy (D) has a potency ratio of β− to α− adrenergic blockad e of ap p roxim ately 10:1 (E) is a short acting, selective β 1-ad renergic recep tor blocker 402. Physostigm ine

398. A p atient w ho had been given vecu roniu m received neostigm ine, 3 m g, at the term ination of the su rgical proced u re. Six m inu tes later, a large am ou nt of w hite, frothy secretions w as noted in the end otracheal tu be. Vigorou s su ctioning w as requ ired to rem ove these secretions in ord er to ventilate the p atient. The treatm ent of choice for such secretions is (A) (B) (C) (D) (E)

atrop ine d igoxin m ore neostigm ine read m inistration of vecu roniu m u se of a ventilator

399. H eparin inhibits blood coagu lation (A) (B) (C) (D) (E)

by bind ing calciu m ions throu gh interactions w ith p rotam ine by activating antithrom bin III by activating p lasm in by activating von Willebrand factor

400. The p atient w ith m yasthenia gravis (A) has norm al reactions to m u scle relaxants (B) reacts abnorm ally to relaxants only w hen the cond ition is not w ell controlled (C) has d ecreased sensitivity to nond epolarizing relaxants

(A) m ay be u sed for the treatm ent of central anticholinergic synd rom e (B) d oes not cross the blood –brain barrier (C) is less effective than neostigm ine in treating em ergence d elirium (D) is the d ru g of choice for reversal of the sed ative effect of benzod iazepines (E) often p rod u ces an u ncom fortably d ry m ou th DIRECTIONS: Use the follow ing scenario to answ er Questions 403-408. Many patients are unaw are of the reasons for w hich they are taking their medications. When provid ed w ith a list of med ications by a patient, an anesthesiologist must be able to recognize the med ications and know the likely pathological states for w hich the med ications are ind icated . For Questions 403-408, a med ication is follow ed by five diseases or pathological states. Choose the ON E disease for w hich the med ication may be ind icated . 403. Lisinop ril (A) p anic d isord er (B) hyp ertension (C) p aroxysm al su p raventricu lar tachycard ia (D) p rem atu re ventricu lar contractions (E) hyp erthyroid ism

100

7: P ha rma cology

404. Am iod arone (A) (B) (C) (D) (E)

Wolff-Parkinson-White synd rom e hyp erthyroid ism angina p ectoris p u lseless electrical activity ventricu lar tachycard ia

405. Fexofenad ine (A) (B) (C) (D) (E)

(A) (B) (C) (D) (E)

DIRECTION S: Use the follow ing figu re to answ er Qu estions 410-411:

hay fever insom nia schizop hrenia chronic p ain inflam m atory bow el d isease

406. Bu d esonid e (A) (B) (C) (D) (E)

d eep venou s throm bosis control of asthm a clau d ication angina p ectoris atrial fibrillation

407. Dofetilid e (A) (B) (C) (D) (E)

m u scle sp asticity trigem inal neuralgia atrial fibrillation p erip heral ed em a insom nia

408. Allop urinol (A) (B) (C) (D) (E)

m etoclop ram id e p rochlorp erazine fam otid ine d exam ethasone ep hed rine

system ic lu p u s erythem atosu s gou t osteoarthritis rheu m atoid arthritis p soriatic arthritis

409. A 42-year-old fem ale w ith a BMI of 32 and a history of severe p ostop erative nau sea and vom iting (PON V) is sched u led for a lap aroscopic cholecystectom y for sym ptom atic cholelithiasis. You r anesthetic p lan inclu d es the ad m inistration of a 5-H T3 recep tor antagonist and d roperid ol for the prevention of PON V. If you w ere to ad d a third agent, the m ost u sefu l d ru g w ou ld be

A

B

410. The train-of-fou r stim u lu s is d ep icted in the figu re. Stim u lation is at 2 H z. The resp onse in A is norm al. By looking at B, you know that (A) a d ep olarizing block is p resent (B) the p atient is p artially p aralyzed (C) the p atient is p artially p aralyzed , bu t one w ou ld need a baseline record ing to know how m u ch (D) the train-of-fou r ratio is 50% (E) the p atient cou ld su stain a head lift 411. Th e train -of-fou r ratio for a d ep olarizin g block is (A) (B) (C) (D) (E)

variable 50% 60% 75% 100%

412. Prop ofol as com p ared to thiopental (A) is less likely to p rovoke bronchosp asm (B) if ad m inistered in equ ip otent d oses for ind u ction of anesthesia cau ses less red u ction in system ic blood p ressu re (C) cau ses ad renal su p p ression after p rolonged infu sion (D) has no effect on cerebral m etabolic rate (E) d oes not cau se excitatory m otor activity

Que s tions : 404–422

413. Verap am il (A) (B) (C) (D)

increases m yocard ial contractility is a β -ad renergic antagonist is p oorly bou nd to p lasm a p rotein m ay p rod u ce AV-block w hen com bined w ith volatile anesthetics (E) belongs to the class of d ihyd rop yrid ines 414. Lep iru d in and argatroban exert their anticoagulant effect by (A) (B) (C) (D)

vitam in K antagonism inhibition of p latelet aggregation glycop rotein IIb/ IIIa inhibition increasing the rate of the throm binantithrom bin reaction (E) d irect throm bin inhibition 415. If a p atient is taking a m onoam ine oxid ase (MAO) inhibitor, w hich one of the follow ing should be avoid ed ? (A) (B) (C) (D) (E)

local anesthetics halothane vecu roniu m m ep erid ine asp irin

416. The most common electrolyte alteration caused by the thiazid e d iu retics is (A) (B) (C) (D) (E)

hyp okalem ia hyp oglycem ia hyp erchlorem ia hyp ernatrem ia hyp eru ricem ia

417. N ifed ip ine (A) is qu ite effective in su p raventricu lar tachycard ia (B) is u sed for the treatm ent of ischem ic heart d isease (C) has a half-life of 30 m in (D) is a p erip heral vasoconstrictor (E) is an effective d ru g for ventricu lar tachycard ia

101

418. The interaction of p rotam ine and hep arin to term inate anticoagu lation is of (A) com p etition for bind ing sites (B) a chem ical interaction lead ing to an inactive com p ou nd (C) p H change (D) a conform ational change (E) p latelet stim u lation 419. Ep inep hrine cau ses a p rolongation of activity of local anesthetics by (A) (B) (C) (D) (E)

chem ical interaction d ecreasing absorp tion altered p rotein bind ing com p etition for bind ing sites altered m etabolism

420. The com bination of nitrous oxid e at 0.5 MAC p lu s isoflu rane 0.5 MAC is one of (A) (B) (C) (D) (E)

antagonism p otentiation ad d itive effect synergism no effect

421. Glycopyrrolate (A) is a qu aternary am ine (B) crosses the blood –brain barrier w ith ease (C) is associated w ith the central cholinergic synd rom e (D) is a cholinergic agonist (E) is a natu rally occu rring bellad onna alkaloid 422. Lid ocaine (A) (B) (C) (D)

is elim inated chiefly by the liver is effective orally is toxic at levels over 1 m cg/ m L toxicity is noted by the ap p earance of hem atu ria (E) is u sefu l in su p raventricu lar tachycard ia

102

7: P ha rma cology

423. The interaction of p henobarbital and p henytoin can be d escribed as one of (A) (B) (C) (D) (E)

chem ical interaction interaction at site of absorp tion altered p rotein bind ing com p etition for bind ing sites altered m etabolism

428. Metoclop ram id e (A) is a d op am inergic agonist (B) d ecreases gastric acid secretion (C) stim u lates m otility of the u p p er gastrointestinal tract (D) m ay lead to vom iting (E) lead s to an ileu s and increased sm all intestinal transit tim e

424. Mid azolam (A) is contraind icated in the child (B) is shorter-acting than thiop ental (C) is associated w ith less frequ ent venou s irritation than d iazep am (D) su p p resses ad renal cortical fu nction (E) has a high incid ence of histam ine release 425. The p atient w ho has recently abu sed cocaine (A) (B) (C) (D) (E)

w ill be calm and sed ated m ay be treated w ith p rop ranolol exhibits signs of sym p athetic blockad e w ill have brad ycard ia w ill have hyp otension

426. The m ethylxanthine grou p of d ru gs (A) inclu d es caffeine (B) have a strong β 1-ad renergic m im etic effect (C) stim u lates p rod u ction of p hosp hod iesterase (D) cau ses bronchoconstriction (E) lead s to a d ecrease in cyclic AMP 427. Su blingu al d ru g ad m inistration (A) lead s to low er d ru g levels com p ared to oral ad m inistration (B) is m ore effective for ionized d ru gs (C) circu m vents the first-p ass effect (D) lead s to rap id liver breakd ow n of the d rug (E) requ ires a m u ch larger d ose for effectiveness

429. A p atient is sched u led for su rgery w ho is a Jehovah's Witness. She ad am antly refu ses to receive blood p rod u cts. The p roced u re m ay requ ire volu m e rep lacem ent. A p rod u ct that m ay be u sed is (A) (B) (C) (D) (E)

5% albu m in w ashed red cells au tologou s blood hetastarch p latelets to d ecrease bleed ing, thu s m aking transfu sion u nnecessary

430. A 21-year-old female is emergently taken to the operating room for exploratory laparotomy after sustaining multiple injuries including a grad e 4 splenic rupture in a high-speed motor vehicle accident. Upon transfer to the operating room table, the patient is noted to have pulseless ventricular fibrillation, and cardiopulmonary resuscitation accord ing to ACLS gu id elines is initiated. Despite resuscitation according to protocol including the administration of epinephrine intravenou sly, the p atient rem ains in ventricular fibrillation. Which one of the following d rugs or procedures is recommended as an alternative to epinephrine in this setting? (A) (B) (C) (D) (E)

p henylep hrine 80 m cg IV p u sh ad enosine 6-12 m g IV p u sh vasop ressin 40 U IV p u sh sod iu m bicarbonate 50 m Eq IV p u sh overd rive p acing w ith isop roterenol

431. Ed rop honiu m , in a d ose of 1 m g/ kg (A) has a slow er onset tim e than neostigm ine (B) has a m u ch shorter d u ration than neostigm ine

Que s tions : 423–438

(C) has greater m u scarinic sid e effects than neostigm ine (D) has a faster onset and d ecreased d u ration than neostigm ine (E) shou ld be p reced ed by atrop ine 432. Sod iu m nitroprussid e is to be u sed for treatment of intraoperative hypertension. This d rug (A) cau ses venou s d ilatation only (B) w ill be need ed in increased d oses if the patient has been p reviously treated w ith propranolol (C) m ay cau se cyanid e toxicity in high d oses, evid enced by alkalosis and increasing d ru g d osage need ed to achieve the sam e resu lt (D) m ay cau se acid osis as a sign of toxicity (E) m ay cau se a toxicity that is evid enced by an acid osis that is resp onsive to sod iu m bicarbonate 433. Postoperative pain control w ith m ethad one (A) is lim ited by its short half-life (B) is m ore effective w ith oral ad m inistration (C) is u sed on an every-2-h regim en (D) m ay take 48 h to obtain a stable effect (E) d oes not d ep ress resp iration 434. Dop am ine (A) is a transm itter confined to the central nervous system (B) stim u lates d op am inergic recep tors only at an infu sion rate of 10 m cg/ kg/ m in (C) d ecreases renal blood flow (D) increases card iac ou tp u t by stim u lating β 1-ad renergic recep tors (E) d ecreases p u lm onary artery p ressu re 435. A p atient has u nd ergone a laparotom y for a bow el obstru ction. A rapid sequ ence ind uction w as p erform ed u sin g su ccin ylch olin e for m uscle relaxation, and pancuroniu m w as su bsequently given. N euromuscular blockad e w as

103

reversed w ith neostigm ine. A few m inu tes after extu bation , th e su rgical d ressin g is stained w ith blood and the su rgeon d ecid es that the w ou nd m u st be reexp lored . If another rap id sequ ence ind u ction is to be p erform ed , (A) su ccinylcholine w ill be ineffective (B) su ccinylcholine w ill lead to severe hyp erkalem ia (C) succinylcholine w ill be effective, how ever the time of onset w ill be d elayed (D) su ccinylcholine w ill be effective, how ever its d u ration w ill be p rolonged (E) su ccinylcholine w ill behave as it d id d u ring the first rap id sequ ence ind u ction in this patient 436. Dru g clearance (A) is solely a fu nction of volu m e of d istribu tion (B) is a fu nction of age (C) is ind ep end ent of p rotein bind ing of a d rug (D) is d ep end ent on d ru g concentration (E) m ay be d u e to elim ination of an u nchanged d rug 437. Pharm acotherap y w ith cim etid ine m ay im p air the m etabolism of w hich one of the follow ing d ru gs? (A) (B) (C) (D) (E)

atracu riu m d op am ine rem ifentanil lid ocaine su ccinylcholine

438. Glu cagon prod u ces all of the follow ing effects EXCEPT (A) increased insu lin secretion (B) inotrop ic card iac effects (C) relaxation of gastrointestinal sm ooth m u scle (D) increased hep atic glu coneogenesis (E) increased lip olysis in ad ip ose tissu e

104

7: P ha rma cology

439. Which one of the follow ing statements regard ing injectable benzodiazepines is most accurate? (A) They significantly ind u ce the synthesis of hepatic CYP isozym es. (B) Ad m inistration of benzod iazep ines is safe in p atients w ith acu te interm ittent p orphyria. (C) They shou ld be avoid ed in p atients w ith a history of MH . (D) They exert their therap eu tic effect by increasing chlorid e cond u ctance and hyperpolarizing m em branes. (E) Excessive sed ation follow ing IV ad m inistration of benzod iazep ines shou ld be antagonized w ith IV naloxone. DIRECTION S: Use the follow ing figu re to answ er Qu estions 440-442:

L / g m ] c n o C [

440. The figu re show s the blood concentration of a d rug as a fu nction of tim e after an intravenous bolu s of 400 m g. The volu m e of d istribution of this m ed ication in the central com partm ent is 5L 8L 10 L 12 L 15 L

441. The red istribu tion half-life for this d ru g is (A) (B) (C) (D) (E)

(A) (B) (C) (D) (E)

1h 2h 3h 4h 5h

443. A p atient w ho has had surgical ablation of the p itu itary gland is sched u led for su rgery. She is on d esm opressin tw ice d aily. The d rug (A) m ay be given intravenou sly d u ring the p roced ure (B) m ay lead to hyp otension (C) has a half-life of a few m inu tes in the circu lation (D) is equ ally effective in nep hrogenic d iabetes insip id u s (E) m ay increase blood loss at the tim e of surgery 444. H yd ralazine

Time (hr)

(A) (B) (C) (D) (E)

442. The term inal half-life for this d ru g is

5 m in 10 m in 15 m in 20 m in 30 m in

(A) is a vasod ilator d u e to its catecholam ine inhibitor p rop erties (B) m ay cau se brad ycard ia (C) m ay lead to a lu p u s-like synd rom e (D) lead s to d iu resis and sod iu m loss (E) has beneficial effects in p atients w ith angina 445. An age-related d ifference in d ru g resp onse in the eld erly is (A) increase in MAC (B) d ecreased rate of hep atic glu cu ronid ation of m orphine (C) low er requ ired ind u ction d ose of thiopental (D) shorter recovery tim e to norm al ventilatory resp onse after fentanyl (E) low er intu bating d ose of vecu roniu m 446. N onp articu late antacid ad m inistration (A) shou ld be given 3 h before su rgery (B) d ecreases gastric volu m e (C) m ay lead to p u lm onary d istress if asp iration occu rs

Que s tions : 439–453

(D) has a lag tim e of 1 hou r for effectiveness (E) is aim ed at raising the p H to at least 2.5 447. A d ru g that is a m ixed op ioid agon ist– antagonist is (A) (B) (C) (D) (E)

bu torp hanol naloxone bleom ycin m ethohexital m id azolam

448. Patients w ith alcohol abu se (A) w ill have increased anesthetic requ irem ents in the acute state of intoxication (B) w ill have red u ced anesthetic requ irem ents in the chronic abu se state (C) w ill d evelop tolerance to its CN S effects w ith chronic u sage (D) w ill d evelop tolerance to its resp iratory effects w ith chronic u sage (E) are m ore resistant to the toxic effects of local anesthetics 449. A 24-year-old p atient is u nd ergoing brachial p lexu s blockad e for op en red u ction and internal fixation of a forearm fractu re. After injection of 30 m L of local anesthetic, blood is asp irated th rou gh th e in jection n eed le. Assu m ing significant inad vertent intravascu lar injection, w hich one of the follow ing agents is least likely to cau se Local An esth etic System ic Toxicity (LAST)? (A) (B) (C) (D) (E)

tetracaine chlorop rocaine rop ivacaine lid ocaine m ep ivacaine

450. The pKa of m ep ivacaine is 7.6. At physiologic pH (7.4), w hat percentage of mepivacaine m olecu les are in the u ncharged form ? (A) 3.9% (B) 6.1% (C) 39%

105

(D) 61% (E) 100% 451. If a p atient p resenting for em ergent craniotom y h as in creased in tracran ial p ressu re, w hich one of the follow ing techniqu es is m ost likely to resu lt in a net d ecrease in intracranial p ressu re by virtu e of red u cing cerebral blood flow ? (A) A com bination of halogenated inhalational anesthetics w ith nitrou s oxid e (B) H yp erventilation com bined w ith a balanced anesthetic techniqu e inclu d ing the u se of opioid s (C) Ketam ine bolu s for ind u ction follow ed by continu ou s infu sion, com bined w ith a halogenated inhalational anesthetic (D) Ad m inistration of a d obu tam ine (E) Ad m inistration of p henylep hrine 452. In the d istribu tion p hase of an intravenou s d rug, (A) the d elivery of the d ru g to tissu es is ind ep end ent of blood flow (B) highly charged , lip id -insolu ble d ru gs d istribu te to the vessel-rich grou p of tissues because of poor u ptake by fat (C) d istribu tion to the vessel-p oor grou p of tissu es is facilitated by bind ing to p lasm a p roteins (D) the interstitial concentration of the d ru g is affected by the p H of the interstitial flu id 453. The load ing d ose of a d ru g (A) is calcu lated based on the central volu m e of d istribu tion (Vc) (B) is d esigned to achieve im m ed iate stead y-state plasm a concentration (C) is not affected by the bioavailability (F) of the d rug (D) m ay lead to u nd esirable effects

106

7: P ha rma cology

454. Which one of the follow ing statem ents abou t am iod arone is tru e ? (A) It is the d ru g of choice for the treatm ent of Wolff-Parkinson-White (WPW) synd rom e. (B) It is consid ered the m ost effective of the antiarrhythm ic d ru gs for the prevention of recu rrences of atrial fibrillation. (C) H yp otension d oes not occu r w ith intravenou s ad m inistration. (D) It shou ld be avoid ed in the p atient w ith refractory ventricu lar fibrillation and tachycard ia. (E) Dru g toxicity d u ring oral load ing regim es is com m on, and m ostly affects the kid neys. 455. A 78-year-old patient w ith past med ical history of chronic renal insu fficiency and congestive heart failu re is ad m itted to the intensive care u nit w ith sym ptom s of nausea, visu al d istu rbances, confu sion, and sinu s brad ycard ia. The p atient is u nable to give a reliable history inclu d ing m ed ication regim en. Based on the patients' presentation, the d ru g m ost likely to cau se these sym ptom s is (A) (B) (C) (D) (E)

am iod arone qu inid ine triazolam carved ilol d igoxin

456. The d ecrease in cerebral m etabolic rate cau sed by inhaled anesthetics is (A) m ore p ronou nced w ith isoflu rane as com pared to sevoflu rane (B) ind ep end ent of d ose (C) associated w ith a d ecrease in cerebral electrical activity (D) p resent d u ring seizu re activity (E) d u e to vasoconstriction and resu lting d ecreased cerebral blood flow 457. A p atient has been on oral steroid therap y for a d ermatologic problem for tw o years. She is to und ergo a cholecystectomy. In ord er to prevent second ary ad ren al insu fficien cy, the m ost

ap p rop riate ap p roach to p eriop erative steroid coverage w ou ld inclu d e w hich one of the follow ing? (A) hyd rocortisone 100 m g PO at the tim e of oral benzod iazep ine p rem ed ication (B) 50-75 m g hyd rocortisone IV at the tim e of ind u ction (C) 50-75 m g hyd rocortisone IV at the tim e of ind u ction, follow ed by a rapid tap er back to the u su al d ose (D) 25 m g hyd rocortisone IV at the tim e of ind u ction (E) 100-150 m g hyd rocortisone IV at the tim e of ind uction, follow ed by a rapid taper back to the u sual d ose 458. A 78-kg, 36-year-old p atient is u nd ergoing sap henou s vein strip p ing of her left low er extrem ity. After u neventfu l ind u ction of general an esth esia w ith p lacem en t of LMA, cefazolin 2 gm IV is being ad m inistered , w hen there is a su d d en increase in airw ay pressure, w heezing, tachycard ia, and d ecreased blood pressu re. In ad d ition to stop ping infu sion of cefazolin, an IV fluid bolu s, and ad m inistration of 100% oxygen, w hich one of the follow ing actions is m ost ap p rop riate? (A) ad m inistration of IV glu cocorticoid and phenylep hrine (B) ad m inistration of su bcu taneou s ep inep hrine and IV glu cocorticoid (C) ad m inistration of d ip henhyd ram ine and ep hed rine (D) ad m inistration of IV ep inep hrine and IV glu cocorticoid (E) ad m inistration of IV d op am ine and IV glu cocorticoid 459. A 37-year-old p atient w ith a history of chronic alcohol abu se is ad m itted to the hosp ital for p alp itation s. On exam in ation , the p atien t app ears w eak and m alnou rished , and is com p lain in g abou t m u scle cram p s. An ECG obtained is significant for p olym orp hic ventricular tachycard ia w ith QT prolongation. An electrolyte panel obtained from a venous blood d raw w ill m ost likely show w hich one of the follow ing electrolyte abnorm alities?

Que s tions : 454–467

(A) (B) (C) (D) (E)

hyp ochlorem ia hyp ocalcem ia hyp onatrem ia hyp erkalem ia hyp om agnesem ia

460. Opioid agonists prod u ce (A) d ilated p u p ils (B) nau sea and vom iting m ed iated throu gh the gastrointestinal tract (C) good am nesia (D) u nconsciou sness at high d oses 461. A d ru g suitable for the patient w ith hyperthyroid ism is (A) (B) (C) (D)

am iod arone p rop ranolol levothyroxine asp irin

462. In h alation al an esth etic-m ed iated coron ary “steal” (A) resu lts in a higher incid ence of p eriop erative ischem ia w hen isoflu rane is used in p atients w ith coronary artery d isease (B) involves the d iversion of blood flow aw ay from areas of fixed stenotic lesions (C) is a com m on p roblem u nd er d esflu rane anesthesia (D) is associated w ith increased p ostop erative m ortality (E) is a com m on p roblem u nd er sevoflu rane anesthesia 463. Corticosteroid s given for the treatm ent of asthm a (A) have an im m ed iate onset of action (B) are best given by the inhalation rou te to d ecrease onset tim e (C) have no u se in the anesthetized patient (D) shou ld involve only those d rugs w ith little m ineralocorticoid effect (E) are less effective in controlling inflam m ation as com pared to inhaled β 2-agonists

107

464. An anesthetic agent w ith a prolonged d u ration of action in patients w ith renal failu re is (A) p ancu roniu m for intu bation (B) fentanyl for analgesia (C) m aintenance of anesthesia w ith isoflu rane (D) p rop ofol for ind u ction (E) etom id ate for ind u ction 465. Ionization of a d ru g (A) is affected by volu m e of d istribu tion (B) is ind ep end ent of the p Ka of the d ru g (C) is d escribed by the Michaelis-Menten equ ation (D) d oes not affect the ability of the d ru g to cross m em branes (E) is a fu nction of p H of the flu id in w hich it is d issolved 466. A p h arm acologic d ifferen ce in th e obese patient as com p ared to the lean p atient is a(n) (A) d ecreased d egree of m etabolism of isoflu rane (B) d ecreased term inal half-life of thiop ental (C) increased volu m e of d istribu tion of m id azolam (D) p rolonged d u ration of action of su ccinylcholine (E) increased volu m e of d istribu tion of d igoxin 467. Granisetron exerts its therap eu tic effect via w hich one of the follow ing m echanism s? (A) blockad e of 5-H T3 recep tors in the chem oreceptor trigger zone (B) histam ine H 2-recep tor antagonism (C) blockad e of d op am inergic recep tors in the chem oreceptor trigger zone (D) histam ine H 1-recep tor antagonism (E) neu rokinin recep tor antagonism

108

7: P ha rma cology

468. A patient w ith acute interm ittent p orphyria (A) void s u rine containing u rop orp hyrins that are p athognom onic for the d isease (B) w ill likely su ffer a card iac arrest if ad m inistered thiopental for ind u ction of anesthesia (C) w ill have u rine negative for p orphobilinogen (D) m ay be given m orp hine safely 469. A patient w ho has been treated in the past w ith high d oses of cisp latin m ay have w hich one of the follow ing perm anent problem s? (A) (B) (C) (D) (E)

p u lm onary fibrosis chronic p ancreatitis p erip heral neu rop athy congestive heart failu re hep atic cirrhosis

470. A com parison of d issociative and inhalational anesthesia show s that (A) the relaxed , nonresp ond ing state is p resent in each (B) there is d ecreased m u scle tone in each (C) the p resence of m ovem ent is a sign of insu fficient d osage in each (D) the eyes m ay rem ain op en d u ring d issociative anesthesia (E) com p lete u nconsciou sness is a hallm ark featu re of d issociative anesthesia 471. A patient is su sp ected to have had an allergic reaction to an injection of lid ocaine obtained from a m u ltip le d ose vial. Which one of the follow ing statem ents is m ost accurate? (A) The patient will probably have a reaction if a second injection is made with lidocaine from a fresh, single-dose vial. (B) The p atient m ay be exhibiting a reaction to m ethylparaben. (C) The risk of an allergic reaction to p rocaine in the sam e ind ivid u al is low. (D) The p atient is p robably reacting to a bacterial contam inant p reviou sly introd u ced into the vial. (E) The reaction m ay be avoid ed by ad d ition of a vasoconstrictor.

472. Cap topril (A) activates angiotensin-converting enzym e (B) increases levels of angiotensin II (C) d ecreases venou s cap acitance (D) d ecreases arteriolar resistance (E) increases card iac contractility 473. Dru g antagonism (A) m ay occu r w hen tw o d ru gs chem ically com bine in the bod y (B) occu rs w hen tw o d ru gs affect a p hysiologic system in a sim ilar w ay (C) is consid ered com p etitive w hen a d ru g w ith affinity for a recep tor, and intrinsic efficacy com p etes w ith the agonist for the p rim ary bind ing site (D) m ay occu r w hen tw o d ru gs d isp lace the d ose-resp onse cu rve in the sam e d irection (E) w hen com p etitive, w ill d ecrease the m axim al response of the agonist on the d ose-resp onse cu rve DIRECTION S: Use the follow ing scenario to answ er Qu estions 474-476: A 46-year-old , 63-kg w om an enters the hospital for correction of strabism u s. She has been u sing echothiop hate iod id e d rop s for 2 years. She u nd ergoes ind u ction of anesthesia w ith p rop ofol 2.5 m g/ kg and su ccinylcholine 1 m g/ kg is given to facilitate intu bation. Anesthesia is m aintained w ith 70% N 2O in O 2 an d fen tan yl 1 m cg/ kg. Du ring the p roced u re she is m echanically ventilated , and no untow ard effects occu r d u ring the case. At the end of the su rgery, the p atient d oes not aw aken and d oes not m ove or breathe. 474. The m ost likely cau se of the ap p arent p rolonged anesthesia is (A) (B) (C) (D) (E)

fentanyl overd ose p rior u se of echothiop hate iod id e effects of su ccinylm onocholine effect of nitrou s oxid e p rop ofol overd ose

Que s tions : 468–482

475. A m aneuver that can be d one to ascertain the cause of the prolongation is (A) (B) (C) (D)

u se of a nerve stim u lator inhalation of carbon d ioxid e ad m inistration of flu m azenil ad m inistration of another d ose of su ccinylcholine

476. Ap p lication of a nerve stim u lator reveals that the patient has no tw itch or tetanic resp onse to nerve stim u lation. Op ioid antagonism w ith naloxone is attem pted w ithout response. The app rop riate intervention is (A) ad m inistration of p ralid oxim e (B) ad m in istration of a stim u lan t (e.g., ep inep hrine) (C) ad m inistration of neostigm ine (D) continu ed ventilation (E) ad m inistration of p hysostigm ine 477. Intram u scu lar injection (A) into the glu teu s m axim u s p erm its a m ore rap id onset than if the d ru g w ere injected into the d eltoid m u scle (B) p erm its a m ore rap id onset than after subcutaneou s injection (C) resu lts in the sam e rate of d ru g absorp tion betw een m ales and fem ales (D) cannot be used for nonaqueous solutions (E) avoids first-pass elimination in the lung prior to distribution to the rest of the body 478. When reversing nond ep olarizing neu rom u scu lar blockad e, (A) atrop ine shou ld be given at the sam e tim e as neostigm ine (B) glycop yrrolate shou ld be given at the sam e tim e as ed rop honiu m (C) neostigm ine shou ld be titrated u ntil reversal is com p lete, u p to a m axim u m d ose of 200 m cg/ kg (D) lack of fad e d u ring d ou ble-bu rst stim ulation is a good ind icator of sufficient m u scu lar function to perm it end otracheal extu bation (E) a TOF ratio = 0.7 ind icates com p lete recovery of p haryngeal fu nction

109

479. A 24-year-old female, G1 P0, w ith a history of CH F is p resenting for cesarean section at 36 w eeks gestation. Rapid sequence intubation facilitated by fentanyl, propofol, and succinylcholine is perform ed . Shortly after su ccessful intubation of the trachea, the patient suffers card iac arrest. The und erlying d isease most likely implied as the cause of the card iac arrest is (A) (B) (C) (D) (E)

m yasthenia gravis Du chenne m u scu lar d ystrop hy hyp okalem ic p eriod ic p aralysis Charcot-Marie-Tooth d isease m yotonic d ystrop hy

480. The neu rolep tic state is characterized by (A) (B) (C) (D) (E)

am nesia increased m otor activity ind ifference to the environm ent analgesia loss of consciousness

481. Which one of the follow ing agents has the highest p rop ensity for p rod u cing carbon m onoxid e d u ring inhalational anesthesia? (A) (B) (C) (D) (E)

isoflu rane halothane sevoflu rane d esflu rane enflu rane

482. Tw o d rugs are said to have an ad d itive effect w hen (A) one d ru g w ill accelerate the sp eed of onset of the other (B) one d ru g w ill p rolong the action of the other (C) both d ru gs are of the sam e chem ical fam ily (D) the com bined effect of the tw o d ru gs is the algebraic sum of the ind ivid ual actions (E) the combined effect of the two drugs exceeds the sum of each drug given alone

110

7: P ha rma cology

/ F

A

F

I

DIRECTION S: Use the follow ing figu re to answ er Qu estion 483:

Time (min)

483. The figu re show s the ratio of the concentrations of sevoflurane in alveolar gas to inspired gas as a fu nction of tim e. A factor that w ill increase the initial slop e is (A) p resence of nitrou s oxid e in insp ired gas (B) d ecreased insp ired sevoflu rane concentration (C) d ecreased m inu te ventilation (D) increased card iac ou tp u t (E) p u lm onary right-to-left shu nting 484. Eaton–Lam bert synd rom e is characterized by (A) d ecreased p rod u ction of acetylcholine by the nerve cell (B) d ecreased release of acetylcholine at the neurom uscu lar ju nction (C) nond ep olarizing neu rom u scu lar blockad e that is read ily reversed by neostigm ine (D) d ecreased sensitivity to d ep olarizing m u scle relaxants (E) increased su scep tibility to m alignant hypertherm ia 485. A patient is infected w ith Clostridium perfringens and has gas gangrene. Which one of the follow ing d ru g regim ens is consid ered first line for the treatm ent of this organism ? (A) p arenteral p enicillin G and p arenteral clind am ycin (B) p arenteral cefoxitin and p arenteral im ip enem

(C) oral levofloxacin and oral cep halexin (D) p arenteral gentam icin and p arenteral vancom ycin (E) oral penicillin V and parenteral amikacin 486. A 56-year-old , 85-kg p atient w ith a history of gastrod u od enal u lcers is presenting for elective resection of the sigm oid colon for chronic d iverticulitis. The patient is premed icated w ith 10 m g d iazep am orally, and u nd ergoes placement of an epid ural catheter in the mid thoracic area for p ostop erative p ain control. Ind u ction of anesthesia is p erform ed w ith p rop ofol 2 m g/ kg, fentanyl 2 m cg/ kg, and rocu roniu m 0.6 m g/ kg. Anesthesia is maintained w ith d esflu rane in nitrou s oxid e and oxygen. At the conclu sion of an u ncom p licated case, the p atient is noted to have prolonged em ergence from anesth esia, an d ap p ears oversed ated after tracheal extu bation, d esp ite fu ll recovery from neu rom u scu lar blockad e. Which one of the follow ing is the m ost likely cau se? (A) (B) (C) (D) (E)

ad m inistration of d esflu rane chronic therap y w ith cim etid ine chronic ingestion of ethanol fentanyl overd ose d iazep am overd ose

487. The m etabolism of nitrou s oxid e in hu m ans (A) (B) (C) (D)

yield s nitric oxid e as the m ain p rod u ct occu rs in the liver is a red u ctive p rocess is increased by p henobarbital pretreatm ent (E) occu rs in the lu ngs 488. When m ethylm ethacrylate (MMA) cem ent is u sed to secure the fem oral com ponent of a hip prosthesis in the fem oral canal, (A) hyp ertension is a com m on occu rrence (B) acu te vasod ilation and card iac collap se can occu r d ue to hypersensitivity reaction to MMA (C) card iovascu lar collap se and bronchoconstriction m ay occu r d ue to em bolization of fat p articles and d ebris from the intram ed u llary canal

Que s tions : 483–494

(D) the p atient shou ld have a central venou s catheter for the rem oval of an air em bolu s (E) the techniqu e of venting the canal can reliably p revent card iovascu lar com p rom ise d uring the proced u re 489. Ond ansetron (A) (B) (C) (D) (E)

facilitates gastric em p tying d ecreases gastroesop hageal reflu x is sed ating at higher d oses has antiem etic effects acts on d op am inergic recep tors in the area postrem a

DIRECTION S: Use the follow ing figu re to answ er Qu estion 490:

m

o

rr h

a

g

e

491. A recom m end ed single-agent p eriop erative p rop hylaxis regim en against a su rgical site infection for patient populations w ith low incid ence of m ethicillin-resistant Staphylococcus aureus (MRSA) is (A) (B) (C) (D) (E)

p enicillin G vancom ycin cefazolin am p icillin levofloxacin

492. Carbon m onoxid e (A) w hen bou nd to hem oglobin can be d ifferentiated from oxyhem oglobin by stand ard pu lse oxim etry (B) cau ses oxygen to be released less efficiently from hem oglobin in tissu es (C) d oes not p rod u ce toxicity in the p resence of an increased FIO 2 (D) com bines and d issociates from hem oglobin at a rate equal to that of oxygen (E) lead s to a right-shift of the oxygenhem oglobin d issociation curve

r

g ri

F

F

a

c

t

i

l

o

a

n

rm

o

o

f

N

d

o

s

e

i

n

C

N

S

H e

111

h

493. An active transm em brane transp ort m echanism is

t

Time

490. The figu re d ep icts the relative effect of p rop ofol in three p hysiologic states. From this w e can say that (A) the p otency of p rop ofol in the anxiou s patient is increased (B) the anxiou s p atient w ill requ ire a larger d ose (C) a p atient in shock w ill have a d ecreased effect from p ropofol (D) the bleed ing p atient has a faster d ecrease in brain concentration as a resu lt of the p rop ofol being lost w ith the blood (E) the anxiou s p atient w ill have loss of consciou sness sooner after IV bolu s, as com pared to the bleed ing patient

(A) (B) (C) (D)

transp ort via filtration channel m ed iated d iffu sion p assive nonionic d iffu sion transp ort of solu te by carrier p rotein d ow n an electrochem ical grad ient (E) satu rable 494. An active m etabolite of heroin is (A) (B) (C) (D) (E)

thebaine m orp hine hyd rom orp hone cod eine ap om orp hine

112

7: P ha rma cology

495. Low -m olecu lar-w eight hep arins (A) are p entasaccharid es (B) cau se a low er incid ence of hep arinind u ced throm bocytop enia than u nfractionated hep arin (C) are ad m inistered intravenou sly (D) all have equ ivalent antithrom botic effects (E) are able to brid ge antithrom bin to throm bin to the sam e extent as u nfractionated hep arin 496. A 42-year-old fem ale is ad m itted to the hosp ital approximately 18 h after supposed ly ingesting a large qu antity of an u nknow n d ru g in tablet form . The p atient ap p ears confu sed , tachypneic, com plains abou t nau sea and tinnitus, and is febrile at 39°C. Pu p ils are equ al bilaterally at 3 m m and reactive. Shortly after arrival, the patient starts to vom it. Blood gas analysis is significant for a m ixed picture consisting of respiratory alkalosis and m etabolic acid osis. The patients' symptoms are most consistent w ith d ru g overd ose from (A) (B) (C) (D) (E)

acetam inop hen asp irin oxycod one d ip henhyd ram ine lithiu m

497. Of the follow ing m etabolic reactions, w hich one is m ost likely to be catalyzed by cytochrom e P450? (A) (B) (C) (D) (E)

acetylation of hyd ralazine d em ethylation of ketam ine conju gation of m orp hine to glu cu ronid e oxid ative deamination of norepinephrine conju gation of acetam inop hen to su lfate

498. In a patient w ho is physically d epend ent on morphine, which one of the following is a symptom of the abstinence synd rome that begins within 12 h of the last dose of morphine?

(A) (B) (C) (D) (E)

brad ycard ia lacrim ation seizu res p ilom otor activity (“gooseflesh”) com a

499. A 72-year-old man w ith a history of congestive heart failu re presents for evalu ation of acu te renal insufficiency. On evaluation, the patient also complains about a loss of sense of taste and a chronic cou gh. Urinalysis is significant for proteinuria. He was started on a new antihypertensive med ication a few months ago. The d rug most likely implied in his symptoms is (A) (B) (C) (D) (E)

carved ilol hyd ralazine labetalol nifed ip ine cap top ril

500. The rap id ly p erfu sed (vessel-rich) tissu es (A) inclu d e the kid neys (B) receive an increasing p rop ortion of card iac ou tpu t w ith increasing age (C) receive the sam e p rop ortion of card iac outpu t in the neonate as the vessel-poor grou p (D) inclu d e m u scle (E) accou nt for 30% of total bod y w eight in ad u lts 501. Ketorolac 60 m g and m orp hine 10 m g p rod uce ap p roxim ately equ ivalen t m agn itu d es of w hich one of the follow ing effects after intram u scu lar injection? (A) (B) (C) (D) (E)

nau sea sed ation ventilatory d ep ression analgesia abd om inal p ain

502. The CN S resp onse to an ind u ction d ose of propofol w ou ld be m ost altered by w hich one of the follow ing cond itions?

Que s tions : 495–508

(A) (B) (C) (D) (E)

obesity hyp oalbu m inem ia ad vanced age renal failu re hyp ovolem ia d u e to blood loss

503. Which one of the follow ing statem ents regarding the CN S effects of halogenated volatile anesthetics is m ost accurate? (A) Isoflu rane, in contrast to halothane, abolishes autoregu lation (B) H alothane can p rod u ce an isoelectric EEG at clinically relevant d oses (C) Isoflu rane low ers the ischem ic threshold for EEG changes (D) Isoflu rane d ecreases cerebral blood flow (E) Sevoflu rane d ecreases ep ilep tic brain activity 504. Lam otrigine (A) is an atyp ical antip sychotic d ru g (B) has a p rolonged half-life w hen p entobarbital, carbam azep ine, or p henytoin are ad m inistered concom itantly (C) is not effective in absence ep ilep sy (D) is effective against a broad er sp ectru m of seizu res than phenytoin and carbam azepine (E) is not m etabolized in hu m ans 505. A 42-year-old p atient w ith alcoh ol ind u ced liver cirrhosis, Child -Pu gh class B, u nd erw ent op erative rep air of a fem u r fractu re su stained in a m otor veh icle accid en t u n d er gen eral anesth esia. There w ere no oth er associated in ju ries. In d u ction of an esth esia w as p erform ed w ith p rop ofol, fentanyl, and cisatracu riu m , and the anesthesia w as m aintained w ith sevoflu rane in oxygen and nitrou s oxid e. After u n even tfu l p ostop erative extu bation and recovery, the p atient is transferred to the floor. Three d ays later, the orthop ed ics team

113

con su lts you to evalu ate th e p atien t for jau nd ice, lethargy, and signs of encep halop athy. The p atient m ost likely su ffered intraop erative hep atocellu lar inju ry d u e to (A) im m u ne reaction associated w ith sevoflu rane ad m inistration (B) sevoflu rane-m ed iated d ecreased hep atic blood flow (C) hep atotoxic effects of p rop ofol (D) hep atotoxic effects of nitrou s oxid e (E) hep atotoxic effects of sevoflu rane 506. A d ru gs that m ay su p p ress the sym p tom s of opioid w ithd raw al is (A) (B) (C) (D) (E)

bu torp hanol chlorp rom azine nalbu p hine clonid ine flu m azenil

507. Em ergence sequ elae w ith ketam ine inclu d e d eliriu m and d ream ing that (A) are alw ays of an u np leasant natu re (B) are cau sed by GABA A antagonism of ketam ine (C) are not rem em bered because of the am nestic effects of ketam ine (D) can be m inim ized by giving a benzod iazep ine (E) occur equ ally across all age grou ps 508. Om ep razole (A) in typ ical d oses d im inishes the d aily acid prod u ction by app roxim ately 70% (B) inhibits the m etabolism of som e m ed ications by cytochrom e P450 (C) facilitates gastric em p tying (D) d ecreases the volu m e of gastric ju ice (E) is a histam ine H 2-recep tor antagonist

7: P ha rma cology

(A) (B) (C) (D) (E)

p rop ofol isoflu rane vecu roniu m nitrou s oxid e fentanyl

510. H istam ine H 2 antagonists ad m inistered in the im m ed iate p reop erative p eriod (A) facilitate gastric em p tying (B) shou ld be u sed in all p atients (C) increase the p H of flu id in the stom ach at the time of ad m inistration (D) p rotect against asp iration of gastric ju ice (E) d o not change the need for a cu ffed tu be 511. Which one of the follow ing d ru gs is consid ered u nsafe for the p atient know n to be su sceptible to m alignant hyp ertherm ia? (A) (B) (C) (D) (E)

nitrou s oxid e su ccinylcholine xenon ketam ine atracu riu m

512. An antihyp ertensive that interacts w ith the renin-angiotensin system is (A) (B) (C) (D) (E)

losartan p razosin nesiritid e m inoxid il hyd ralazine

m

e

s

C

o

n

ym

g

p

e

t

s

o

t

i

m

v

s

e

DIRECTION S: Use the follow ing figu re to answ er Qu estions 513-514:

l

u

o N

rm

al

e

v

o

E

r

o

k

B

t

509. A 42-year-old schoolteacher u nd erw ent transabd om inal hysterectom y for end om etrial carcinom a. H er past m ed ical history is significant for vitam in B12 d eficiency d iagnosed three years ago and is otherw ise u n rem arkable. Accord ing to the p atient, she is in excellent health because of her strictly vegan d iet. The case w as cond ucted und er general anesthesia w ith p rop ofol, fentanyl, and vecu roniu m for in d u ction, follow ed by isoflu ran e, nitrou s oxid e, and oxygen for maintenance of anesthesia. After four hours of surgery, the patient w as extubated and transferred to the PACU. On the third postoperative d ay, you are called to evalu ate the patient on the floor because she has d evelop ed signs of low er extrem ity p aresthesias and has an u nstead y gait. Which one of the anesthetics u sed for ind u ction and / or m aintenance of anesthesia could have a role in this patient's symptom s?

S

114

F Low-output symptoms

C D

A

LVEDP

513. Th e p osition on th e ven tricu lar fu n ction (Frank-Starling) cu rve for a p atient w ith systolic heart failure is d epicted as p oint A in the figu re. Ad m inistration of nicard ip ine w ou ld m ove the patient's ventricu lar fu nction (A) (B) (C) (D) (E)

from from from from from

A to A to A to A to A to

C B norm al D E

514. Ad m inistration of w hich one of the follow ing d rugs, or d ru g com binations, is m ost likely to m ove this p atient's ventricu lar fu nction from p oint A to p oint F? (A) (B) (C) (D) (E)

d obu tam ine d obu tam ine and fu rosem id e nitrop ru ssid e d obu tam ine and losartan d obu tam ine, losartan, and fu rosem id e

Que s tions : 509–523

515. Fosp rop ofol (A) has a rap id onset of action after IV ad m inistration (B) is a p rod ru g (C) has a high volu m e of d istribu tion (D) is associated w ith p ain on injection (E) is ad m inistered as a bolus of 2 m g/ kg for p roced u ral sed ation 516. Which one of the follow ing agents w ill lead to a d ecrease in p u lm onary vascu lar resistance throu gh interaction w ith the end othelin-A recep tor (ETA ) on vascu lar sm ooth m u scle? (A) (B) (C) (D) (E)

inhaled nitric oxid e ep op rostenol sild enafil trep rostinil bosentan

DIRECTION S (Qu estions 517-520): Each grou p of item s below consists of lettered head ings follow ed by a list of nu m bered p hrases or statem ents. For each nu m bered p hrase or statem ent, select the ON E lettered head ing or com p onent that is m ost closely associated w ith it and fill in the circle containing the corresp ond ing letter on the answ er sheet. Each lettered head ing or com p onent m ay be selected once, m ore than once, or not at all. (A) (B) (C) (D) (E) (F) (G) (H ) (I)

α 1-ad renocep tor α 2-ad renocep tor β 1-ad renocep tor β 2-ad renocep tor N a,K-ATPase PDE3 PDE4 PDE5 AT1

115

(J) AT2 (K) V1 For each m ed ication, select the recep tor or enzym e m ost responsible for its pharm acological effect. 517. antid iu retic horm one 518. inam rinone 519. sild enafil 520. valsartan DIRECTION S (Qu estions 521-523): Each group of item s below consists of lettered head ings follow ed by a list of nu m bered p hrases or statem ents. For each nu m bered p hrase or statem ent, select the ON E lettered head ing or com p onent that is m ost closely associated w ith it and fill in the circle containing the correspond ing letter on the answ er sheet. Each lettered head ing or com ponent m ay be selected once, m ore than once, or not at all. (A) (B) (C) (D) (E) (F) (G)

glom eru lu s p roxim al tu bu le d escend ing lim b of loop of H enle thin ascend ing lim b of loop of H enle thick ascend ing lim b of loop of H enle d istal convolu ted tu bu le collecting d u ct

For each d iu retic, select the p ortion of the nep hron m ost resp onsible for its pharm acological effect. 521. spironolactone 522. acetazolam id e 523. fu rosem id e

Answe rs a nd Expla na tions

366. (E) Use of heparin in a patient w ith heparin ind uced thrombocytopenia type II can lead to life-threatening thrombotic complications and is therefore not ind icated . Low -m olecu larw eight heparins can cross-react w ith heparin and should also be avoid ed . Warfarin should not be u sed u ntil the throm bocytop enia has resolved because it can cause venous limb gangrene or multicentric skin necrosis. Tirofiban is an antiplatelet drug that acts by inhibition of the glycoprotein IIb/ IIIa receptor. Fond aparinux is a synthetic five-saccharid e analog of a natural pentasaccharide sequence that is found in heparin and LMWH s and med iates their interaction with antithrombin. It has no intrinsic anticoagulant activity and instead it binds to antithrombin and accelerates the rate at w hich it inhibits various coagu lation proteases. Its potency is assessed w ith an anti Xa assay. Other suitable alternatives w ould be lepirud in, bivalirud in, or argatroban. (1:854, 859) 367. (B) Drug elimination may occur by metabolism or excretion and is u su ally com p lete after five half-lives. Polar com p ou nd s are u su ally excreted m ore efficiently than substances w ith high lip id solubility. An exam ple on how p harm acologic intervention can influence the rate of elim ination from the bod y is alkalinization or acid ification of the urine to hasten the excretion of som e d ru gs in the treatm ent of d ru g poisoning. Protein bind ing of a d ru g to blood and tissu es w ill affect the rate of elim ination from the bod y. (1:26-30) 368. (D ) Sevoflu rane is not know n to cau se hep atotoxicity and it is not flam m able. The agent prod u ces good m u scle relaxation , d oes n ot 116

p rod u ce tachycard ia and m ay be a p referable agent in p atients p rone to m yocard ial ischem ia. Sevoflu rane is m etabolized in the liver by the cytochrom e P450 system . (1:546) 369. (D ) Barbiturates do not have a clinically relevant analgesic effect. Thiop ental prod u ces a d ose-depend ent effect on the EEG. If given in small doses for sedation, median EEG frequency increases because the predominant EEG activity changes from α to β w aves. High d oses of the drug can cause burst suppression, characterized by δ w aves alternating w ith electrical inactivity. Methohexital m ay cau se abnorm al sp iking activity of the EEG, and may elicit seizures in patients w ith seizure disord er. Thiopental injection is follow ed by d ecreased blood pressure and increased heart rate. Coronary blood flow is increased . Stroke volume is d ecreased , as is card iac output. (5:694) 370. (B) Ketamine ad m inistration is associated w ith changes in card iovascu lar param eters sim ilar to those seen w ith sym p athetic stim u lation. Therefore, the heart rate is increased , and there is an increase in card iac ou tp u t. (1:539) 371. (C) The hyp otension that m ay be seen after atracuriu m ad m inistration is d u e to histam ine release, and m ay be exaggerated in a p atient w ith hyp ovolem ia. The extent of histam ine release is d ep end ent on the d ose and the speed of in jection . Atracu riu m d oes n ot cau se clin ically sign ifican t gan glion ic blockad e. Lau d anosine, a m etabolite of atracu riu m , can cause cerebral excitation and seizure activity in anim al m od els; how ever, this has not been a clinical problem in hum ans. (5:503)

Answe rs : 366–382

372. (C) Rocuronium is metabolized and eliminated by the liver. It can be ad m inistered intram u scularly and by continu ou s infu sion and d oes not cau se significant histam ine release or ganglionic blockad e. (1:262; 5:504) 373. (C) Drug metabolism is separated into tw o d istinct phases, and usually results in a d ecreased d uration of action because the d rug is rendered inactive, m ore w ater solu ble, m ore read ily excreted by tu bu lar secretion, and less likely to u nd ergo tu bu lar reabsorp tion. Reactions in p hase I are m ost likely to consist of oxid ation, red u ction, or hyd rolysis reactions, and generally resu lt in the loss of p harm acological activity. Phase II reaction s inclu d e conju gation reactions, aim ed at facilitating the excretion of inactive d ru g m etabolites. (1:27-8) 374. (E) Transd erm al scop olam ine is a m u scarinic antagonist that can cross the blood –brain barrier. It is not recom m end ed for the p rop hylaxis of em esis in child ren and can p rod uce behavioral sid e effects. It has, how ever, been show n to be u sefu l in ad ults if applied on the evening before, or 4 hou rs before the end of the op erative proced ure. (1:230; 5:1256) 375. (C) N aloxone is a p u re op ioid antagonist, m eaning that the d ru g has no opioid effect of its ow n. Bu torp hanol and levallorp han are both m ixed op ioid agon ist–an tagon ists. N eostigm ine and ed rop honiu m are cholinesterase inhibitors. (1:483-4) 376. (C) Milrinone is a vasod ilating and inotropic agent thou ght to fu nction throu gh inhibiting p h osp h od iesterase an d th ereby elevatin g cyclic-AMP levels. It is not a catecholam ine. (1:795) 377. (C) The toxicity associated w ith bu p ivacaine is d u e to blockad e of sod iu m channels. The d ru g is an am id e of long d u ration. It shou ld not be u sed in concentrations above 0.5% for epid ural anesthesia in pregnant patients. Toxicity m ay occu r after injection other than intravenously. (1:573; 5:1149)

117

378. (D ) Tetracaine is rarely u sed in p erip heral nerve blocks because of the large d oses that are typically required , its slow onset, and its potential for toxicity that is greater than chloroprocaine. Bu p ivacaine is m ore card iotoxic than equ ieffective d oses of lid ocaine. Clinical m anifestations of toxicity includ e severe ventricular arrhythm ias and m yocard ial d ep ression after inad vertent intravascu lar injection. The block of card iac sod ium channels is cu m u lative, and su bstantially m ore than w ou ld be pred icted by its local anesthetic p otency. Chlorop rocaine is very short-acting, and p robably not a good choice for the patient d escribed in this vignette w here p ostop erative analgesia from the brachial plexus blockad e w ould be d esirable. The d uration of action of m epivacaine is about 20% longer than that of lid ocaine. (1:573-4; 5:824) 379. (A) Etom id ate has anticonvu lsant activity, and it d ecreases intraocu lar p ressu re. There is no histam ine release and it d oes not have any analgesic effects. Etom id ate p rod u ces a higher incid ence of p ostop erative nau sea and vom iting than other intravenou s ind u ction agents. (5:696-7) 380. (A) The renal tu bu le has active transp ort p rocesses for the tu bu lar secretion of organic acid s, such as glucu ronid es, and organic bases. Since the glu cu ronid e d erivative is likely to be m uch m ore polar than the parent com pound , the rate of tubular reabsorption (w hich usually occu rs by passive nonionic d iffu sion) w ill be less. (1:26) 381. (D ) The correct ranking of the potencies of the volatile anesthetics, from m ost potent to least p otent, is halothane > isoflu rane > sevoflu rane > d esflu rane. (5:598) 382. (A) A has higher efficacy than B becau se the m axim u m response p rod uced by A is greater than the m axim u m response prod u ced by B. Becau se A and B are located at the sam e p lace along the X-axis, A and B are said to have the sam e p otency. The d ose at w hich the halfm aximal resp onse occu rs, the ED 50, is the same for both A and B. It is likely that A and B act via the sam e recep tor. (1:48-9)

118

7: P ha rma cology

383. (B) The effect of d ru g Y is to d ecrease the efficacy of X w ithout shifting the d ose-response relationship of X. This is the d efinition of a noncom p etitive antagonist. It is likely that X and Y act via the sam e receptor. (1:46-7) 384. (C) Ad m inistering succinylcholine to a p atient w ith a m assive bu rn m ay resu lt in an exaggerated hyperkalemic response that can cause card iac arrest. The recom m end ation is to avoid succinylcholine in these patients starting abou t 24 h after the bu rn. The p harm acokinetics of su ccinylcholine how ever w ill not be altered . Patients treated w ith inhibitors of cholinesterase (n eostigm in e, p yrid ostigm in e, isoflu orop hate) or w ho have atyp ical cholinesterase w ill exp erience a p rolonged su ccinylcholine effect. Am inoglycosid es can p rod u ce neu rom uscular blockad e by inhibition of acetylcholine release from the p reganglionic term inal, and to a lesser extent, by noncom p etitively blocking the recep tor. This effect how ever w ou ld be m ore likely in the p atient d escribed in the vignette had the p atient received a nond epolarizing m u scle relaxant d u ring the case. In ad d ition, the com bination of a m u scle relaxant w ith an am inoglycosid e increases the incid ence of m yopathies in critically ill p atients. (1:267; 5:1345, 1392) 385. (B) The MAC for d esflu rane is about 6% and it is irritating to airw ays. It is insolu ble in w ater and lipid s and is m etabolized to a lesser extent than other halogenated anesthetics. (1:545; 5:598) 386. (A) The know n heroin abu ser m ay have a fu ll stom ach d u e to the slow er em ptying tim e of the gastrointestinal tract, and m ay be highly tolerant to opioid s, necessitating higher d oses of anesthetics to achieve ad equ ate analgesia an d sed ation / h yp n osis. Left ven tricu lar hypertrophy can be exp ected in abu sers of hallu cinogen s su ch as p hen cyclid in e or LSD. Recent cocaine abu se can increase sensitivity to catecholam ines. The so called “glue-sniffer's neu rop athy” can be cau sed by inhalation of hexane-containing ad hesives. (5:326-7)

387. (D ) Local anesthetics act at the nerve m em bran e. Th e local anesthetics block sod iu m channels so that the membrane cannot cond uct nerve im pu lses. (1:565-7) 388. (D ) The action of p rop ofol is term inated by red istribu tion. The d ru g is bou nd to p rotein, and variou s concom itant d ru gs m ay interfere w ith bind ing. It is taken u p in fatty tissu es, bu t that is not the m echanism of term inating its effect. (5:691) 389. (A) Methohexital is m etabolized to a greater extent than thiop ental. The d ru g is m etabolized to hyd roxym ethohexital that is inactive. The term inal half-life is shorter than that of thiop ental. Methohexital d oes not release histam ine from m ast cells and is not contraind icated in asthm a. (1:533-5; 5:731) 390. (C) The first-pass effect is the biotransform ation of a d rug as it passes through the intestinal m u cosa and liver. In som e cases, m u ch of the p harm acological activity is lost at this point, leaving little of the d ru g to have its d esired effect. (1:20) 391. (B) Ketam in e ad m in istration is associated w ith increases in both systolic and d iastolic blood p ressu res. Th e m ech an ism for th e in creases is d irect cen tral n ervou s system stim u lation as w ell as th e liberation of catecholam ines. (1:539) 392. (A) The sm all blocking d ose of a nond ep olarizing m u scle relaxant given before an intu bating d ose of su ccinylcholine increases both the d ose of succinylcholine requ ired as w ell as the tim e to the onset of m uscle paralysis. The rise in intracranial p ressu re is attenu ated , bu t there is n o effect on su ccin ylch olin e-in d u ced arrh yth m ias. Th e tim e to recovery is n ot affected . (1:261; 5:501) 393. (C) After one half-life has elap sed , 50% of the d ru g rem ains; after tw o half-lives, 25%; after three half-lives, 12.5%; after fou r half-lives, 6.25%; and after five half-lives, 3.125%. (6:35)

Answe rs : 383–403

394. (A) Flecainid e is an antiarrhythm ic d ru g in Class IC. The d rug is not a m uscle relaxant and can be ad m inistered orally or parenterally. It is not the d ru g of choice for local anesthetic reactions. (1:840) 395. (D ) Ad m inistration of anticholinesterase d rugs w ill reverse the block of a nond ep olarizing agent only if the latter d ru g is in low enou gh concentration. Ad m inistration of an anticholinesterase w ill p rolong the block of su ccinylcholine by inhibiting its m etabolism . (1:266-7) 396. (E) Of the nond ep olarizing m u scle relaxants m entioned , the least d esirable agent w ould be p ancu roniu m d u e to its p rolonged term inal half-life in renal failu re. The interm ed iate– acting m u scle relaxants have a shorter d u ration of action even in the p resence of renal failu re. (5:504) 397. (D ) While all of the op tions are ind icators of recovery, an objective m easu rem ent of recovery from neurom u scu lar block is better than a clinical assessm ent. Double-bu rst stim u lation (DBS) w ith m echano-, electro-, or accelerom yography is the m ethod of choice. If this is u navailable, tactile resp onse to d ou ble-bu rst stim u lation (DBS) is p referable to tactile estim ation of the train-of-fou r ratio becau se it is easier to ap p reciate fad e w ith DBS. N orm al tid al volu m e is not su fficient to p rotect the airw ay; there m ust be enough force to generate a cou gh, and abou t 15 m L/ kg are need ed . H and grip is a clinical assessm ent and m u st be su stain ed to cor relate w ith recov er y. Insp iratory force is a good m easu rem ent and shou ld be at least 25 cm H 2O negative pressu re. The ad vantage of inspiratory force is that it can be assessed in the p atient w ho is still asleep and u nable to resp ond . (5:494-6) 398. (A) Anticholinesterase d ru gs have potent m uscarinic stimulating effects. These may be exhibited in the heart, bronchioles, or gu t. It is im portant to consid er each of these areas in the p articu lar p atient w hose relaxants are being

119

reversed . Atropine or glycopyrrolate shou ld be u sed w ith the reversal d ru g. Althou gh the m u scarinic m anifestations are not com m on, their occu rrence m ay be qu ite trou blesom e, su ch as the p rod uction of cop iou s respiratory secretions. (1:246-7) 399. (C) The antithrom bin III–hep arin com p lex inactivates thrombin and the other proteases of the intrinsic clotting p athw ay. (1:854) 400. (D ) The p atient w ith m yasthenia gravis has an altered sensitivity to relaxants. These patients are m ore sensitive to nond ep olarizing agents. This sensitivity is p resent w ith or w ithou t control of the d isease. These p atients m ay also be resistant to d epolarizing relaxants, particu larly those treated w ith pyrid ostigm ine. (5:143) 401. (B) Labetalol is a com p etitive antagonist at both, α 1-, and β-ad renoceptors. In ad d ition, the d ru g h as p artial agon ist activity at β 2ad renoceptors, and inhibits neu ronal uptake of n orep in ep hrine (a cocain e-like effect). The potency for β-blockad e is 5-10 fold greater than for α 1-blockad e. Labetalol is su itable for the treatm ent of hyp ertensive em ergencies, and m aintains card iac outpu t, w hile m aintaining, or slightly low ering, heart rate. Due to the poor lip id solu bility of labetalol there is very little p lacental transfer, and the d ru g is safe for use in the setting of p regnancy-ind u ced hyp ertensive crisis. (1:328-9) 402. (A) Flum azenil is the d rug of choice for the reversal of high d oses of benzod iazep ines. While physostigmine may be effective in reversing the sed ative effects of benzod iazepines, it is not consid ered the first-line agent. The d rug crosses the blood –brain barrier w ell since it is a tertiary amine. It is used in the treatment of central anticholinergic synd rom e and can cau se hypersalivation. (1:245-6, 461; 5:699, 1461) 403. (B) Lisinop ril is an inhibitor of angiotensinconverting enzym e u sed in the therap y of hyp ertension. (1:731-2)

120

7: P ha rma cology

404. (E) Am iod arone is a recom m end ed p harm acologic treatm ent for m ono- or polym orp hic ventricu lar tachycard ia in patients w ith stru ctural heart d isease. Sotalol is consid ered another first-line agent. The nonp harm acologic treatm ent of choice w ould be synchronized card ioversion. Am iod arone is also effective against m ost su praventricu lar arrhythm ias, but has a num ber of serious toxicities, som e of w hich are d u e to the d ru g's stru ctu ral sim ilarity to thyroid horm one. (1:43; 6:1893, 1911) 405. (A) Fexofenad ine is a second -generation histam ine H 1-receptor antagonist used to treat allergic cond itions su ch as hay fever. It d oes not ap preciably cross the blood -brain barrier and therefore lacks sed ative effects. (1:920) 406. (B) Bu d esonid e is an inhaled corticosteroid (ICS) u sed for the m anagem ent and control of asth m a. ICS are th e m ost effective antiinflam m atory agents u sed in asthm a therap y, red u cing inflam m atory cell nu m bers and their activation in the airw ays. (1:1051; 6:2111) 407. (C) Dofetilid e is a class III antiarrhythmic agent that is an action p otential-prolonging d ru g. It is effective in m aintaining sinu s rhythm in p atients w ith atrial fibrillation. (6:1883) 408. (B) Allopu rinol is an inhibitor of xanthine oxid ase that d ecreases u ric acid form ation. It is u sed in the therapy of gou t. (1:996) 409. (D ) The m ost u sefu l d rug to ad d to this regim en is d exam ethasone that has been show n to be effective either as a first-line agent, or in com bination w ith 5-H T3 receptor antagonists and / or d roperid ol. Metoclopram id e is a shortacting prokinetic agent m ainly u sed to am eliorate n au sea and vom itin g that accom p any gastroin testin al d ysm otility syn d rom es. Prochlorperazine shou ld not be ad m inistered since the p atient has alread y received a D 2 receptor blocker (d roperid ol). Fam otid ine is an H 2-recep tor antagonist and not ind icated for th e p reven tion or treatm en t of PON V. Ep hed rine is a non-catecholam ine α - an d β -ad renergic agonist and is a potent CN S stim ulant; it is not recommend ed for the prevention

or treatm ent of PON V. Ad d itional consid erations m ay inclu d e TIVA, as w ell as ad m inistration of a p reop erative tran sd erm al scop olam ine patch, as w ell as oral aprepitant for the highest risk patients. (1:233-4; 5:77-8) 410. (B) The p atient is p artially p aralyzed . The block that is p resent is not a d ep olarizing block. The train-of-fou r ratio is 75% and therefore the patient cou ld not sustain a head lift. The ad vantage of the train-of-fou r is that it is not necessary to have a control tracing, since it serves as its ow n control. (5:495-6) 411. (E) The train-of-fou r ratio for a d ep olarizing block is 100%, since there is no fad e. The fourth tw itch w ill be as high as the first. (5:500) 412. (A) Excitatory phenomena after propofol induction occur more frequently than with thiopental, however, bronchospasm appears to occur less frequently. In equipotent d oses, propofol produces a dose dependent decrease in blood pressu re that is m ore p ronou nced than w ith thiopental administration. Propofol and thiopental cause a similar decrease in cerebral metabolic rate. Adrenal suppression occurs after the administration of etomidate. (1:537; 5:689-90, 697) 413. (D ) Verap am il is a calciu m channel blocker. All d ru gs in this class are highly bou nd to protein. Calciu m channel blockers increase coronary blood flow, and d ecrease m yocard ial contractility. Verap am il, in com bination w ith inhalational agents m ay prod uce varying d egrees of AV-block. Structu rally, it is a phenylalkylamine com pound ; representatives of the d ihyd ropyrid in es in clu d e n ifed ip in e, am lod ip in e, nim od ipine, and nicard ip ine. (1:755-7; 5:757) 414. (E) Lep iru d in and argatroban are d irect throm bin inhibitors. (1:874) 415. (D) When meperidine is given to a patient on a monoamine oxidase inhibitor, hyperpyrexia and CNS excitation, which may be fatal, may result. Local anesthetics, halothane, vecuronium, and aspirin should not cause problems. (1:504)

Answe rs : 404–429

121

416. (A) H yp okalem ic, hyp ochlorem ic m etabolic alkalosis is one of the m ost com m on p roblem s in the p atient being treated w ith thiazid es. H yp eru ricem ia m ay occu r, bu t it is not as com m on as hypokalem ia. H yp onatrem ia and hyperglycem ia also m ay occu r. (1:689)

423. (E) The interaction of p henytoin and p henobarbital is one of altered m etabolism d u e to enzym e ind u ction. If a p atient has a stable d ru g level, it can be changed by starting another d ru g that m ay ind u ce the enzym e system that m etabolizes the first d rug. (1:592)

417. (B) N ifed ipine is effective for coronary vasospasm d ue to its vasod ilating activity. It has no antiarrhythm ic effect. Its half-life is abou t 4 h. (1:757-8)

424. (C) Mid azolam is associated w ith few er cases of venous irritation than d iazepam . The d ru g is not contraind icated in child ren; in fact, it is very useful in child ren. The d uration is longer than that of thiopental. There is no ad renal cortical d ep ression and no histam ine release. (1:466; 5:698-9)

418. (B) The bind ing of protam ine, a basic protein, and heparin, an acid ic protein, results in a com p ou nd w ith no anticoagu lation effect. There is no other m echanism involved . (1:858) 419. (B) Ep in ep h rin e cau ses vasocon striction , w hich lead s to d ecreased absorp tion of the local anesthetic, thu s p rolonging its effect. This action m ay also d elay m etabolism . Protein bind ing, chem ical interaction, and bind ing site com petition are not involved . (5:776) 420. (C) Th e com bin ation of n itrou s oxid e in 0.5 MAC con cen tration an d isoflu rane in 0.5 MAC concentration w ill lead to an anesthetic effect of 1 MAC. For all p ractical p u rp oses, nitrous oxid e and a halogenated agent are the only agen ts that are ad m inistered together, but if 2 halogenated agents cou ld be u sed together, their effect w ou ld be ad d itive. (1:50; 5:598) 421. (A) Glycop yrrolate is a qu aternary am ine that is an anticholinergic agent. It is a synthetic alkaloid , and not natu rally-occurring like atrop ine and scopolam ine. Since it is a qu aternary am ine, it d oes not cross the blood –brain barrier and is not associated w ith the central cholinergic synd rom e. (1:233; 5:976) 422. (A) Althou gh w ell absorbed from the gastrointestinal tract, lid ocaine is extensively m etabolized on its first p ass throu gh the liver. The sym ptom s of toxicity are u sually CN S stim ulation, and usu ally occu r at blood levels greater than 5 m cg/ m L. Lid ocaine is u sefu l in ventricu lar arrhythm ias. (1:572-3; 5:781)

425. (B) The p atient show ing signs of cocaine toxicity m ay be excited , anxiou s, and restless, and have hyp ertension and tachycard ia. (1:662; 5:324-5) 426. (A) The methylxanthine group of drugs includes caffeine. This group has strong β 2-adrenergiclike effects such as bronchodilation but does not act via β 2-ad renergic receptors. The mechanism of action is by phosphodiesterase inhibition that lead s to an increase in cyclic AMP. (1:663) 427. (C) Su blingu al ad m inistration avoid s the firstp ass effect and byp asses the liver. This rou te is not u sefu l w ith all d ru gs, only those that are u nionized (e.g., nitroglycerin). This rou te lead s to higher blood levels compared to oral ad ministration. (1:22) 428. (C) Metoclop ram id e stim u lates u p p er gastrointestinal m otility. It is an antiem etic and has no effect on acid secretion in the stom ach. Transit tim e is shorter and there is no ileu s, since m otility is stim u lated . (1:1325) 429. (D ) H etastarch is a synthetic colloid . The other op tions all inclu d e ad m inistration of blood prod ucts. Some people of this belief w ill accept blood that is au tologou s w hile others w ill require that the blood be in the circuit continuously (e.g., in a cell saver). It is im portant to u nd erstand the p articu lar p atient's requ irements before und ertaking the proced ure. (5:46)

122

7: P ha rma cology

430. (C) Vasop ressin is recom m end ed in the ACLS gu id elines as an alternative agent to ep inep hrine in cases of refractory ventricu lar fibrillation and p u lseless ventricu lar tachycard ia. Becau se of its potential ad verse effects, sod ium bicarbonate shou ld not be rou tinely ad m inistered d uring CPR. Phenylephrine, ad enosine, and isoproterenol are not recommend ed for the treatm ent of refractory ventricu lar fibrillation. (5:1433-4) 431. (E) Ed rophonium should be given w ith, or preced ed by, atropine. The d ru g has a fast onset bu t, in the d ose cited , a d u ration as long as neostigm ine. The m uscarinic effects are few er than w ith neostigm ine. (1:239-243, 549) 432. (D ) Sod iu m nitrop ru ssid e m ay lead to cyanid e toxicity that may be manifested as acid osis and a tolerance to the d ru g's effects. This d ru g affects both venou s and arterial vessels. The acid osis that is seen is treated by stop ping the d ru g and shou ld not be treated w ith sod iu m bicarbonate. The hyp otensive effect of sod iu m nitropru ssid e is p otentiated by prior treatm ent w ith β -ad renergic antagonists. (1:782-3; 5:746) 433. (D ) Postop erative p ain control w ith m ethad one requires some tim e to obtain stable levels. While single sm all d oses (< 10 m g) m ay have a d u ration of action less than an hou r d u e to red istribu tion, larger d oses are m u ch longeracting. Methad one is u su ally given in tw ice d aily d osage, and d oes d ep ress resp iration. The oral d ose is abou t 80-90% as effective as the sam e parenteral d ose. (5:718) 434. (D ) Dop am ine has d ifferent actions d ep end ing on the d osage: at infu sion rates greater than 10 m cg/ kg/ m in , th ere is stim u lation of α-ad renergic recep tors. At low er infu sion rates (approximately 1–3 mcg/ kg/ min), d opaminergic recep tors are stim u lated that lead s to an increase in renal blood flow. Dop am ine cau ses an in crease in p u lm on ary artery p ressu re, m aking it a p oor choice in the p atient w ith right heart failu re. Dop am ine is a transm itter in both the central and p erip heral nervou s system s. (1:355; 5:175)

435. (D ) The sequ ence that is p roposed m ay result in a p rolonged d u ration of su ccinylcholine action becau se the p rior ad ministration of neostigm ine w ill resu lt in p seu d ocholinesterase in h ibition an d d ecreased su ccin ylch olin e m etabolism . (5:500) 436. (E) Dru g clearance m ay occur by m etabolism to inactive or less active prod u cts or by elim ination of the u nchanged d ru g. Clearance is ind ep end ent of d ru g concentration, and is not a function of age; rather it is affected by volume of d istribu tion that often is d ecreasing w ith increasing age. Sim ilarly, the d egree of p rotein bind ing is an important d eterminant of the rate of d rug clearance. (1:33) 437. (D ) Cim etid ine m ay d ecrease the m etabolism of lid ocaine via inhibition of cytochrom e P450. The other d ru gs listed are not m etabolized by cytochrom e P450. (1:1314; 5:1033) 438. (A) Glu cagon inhibits insu lin secretion and increases p lasm a glu cose levels. (1:1237, 1240) 439. (D ) Injectable benzod iazep ines su ch as d iazepam, lorazepam and mid azolam d o not ind uce the synthesis of hepatic CYP isozym es. They are safe to ad minister in patients w ith a history of MH , how ever, should be avoid ed in patients w ith acute intermittent porphyria because they have been show n to ind u ce ALA synthase in anim al m od els. They exert their effect throu gh activation of the GABA A recep tor as d escribed in op tion D, and can be antagonized w ith flu m azenil. N aloxone is a com p etitive op ioid an tagon ist in d icated on ly for ven tilatory d ep ression d u e to op ioid overd ose. (1:1059; 5:697-9) 440. (A) At zero tim e, the blood concentration is 80 m g/ L. Volu m e of d istribu tion is the d ose of the d ru g d ivid ed by the concentration. Thu s, the volu m e of d istribu tion in the central com p artment im m ed iately after the injection of the d ru g is 400 m g/ 80 m g/ L or 5 L. (1:30-3) 441. (C) To obtain the red istribu tion half-life, the initial straight p ortion of the cu rve is extrap olated . At zero tim e, the blood concentration is

Answe rs : 430–450

80 m g/ L. At 1 h, the extrap olated line gives a blood concen tration of 5 m g/ L. The tim e required for the blood concentration to d rop from 80 m g/ L to 5 m g/ L is 4 half-lives: after one half-life, the blood concentration w ill be 40 m g/ L; after tw o half-lives, 20 m g/ L; after three half-lives, 10 m g/ L; and after fou r halflives, 5 m g/ L. Sin ce fou r h alf-lives occu rred in 1 h, then the redistribution half-life must be 0.25 h or 15 m in. (1:33) 442. (C) To obtain the term inal half-life, the final straight p ortion of the cu rve is extrap olated . At 10 h, the blood concentration is 1 m g/ L. The blood con cen tration is 2 m g/ L at 7 h , an d 4 mg/ L at 4 h. It is apparent that the blood concentration is d eclining by half every 3 h, thu s the term inal half-life m u st be 3 h. (1:33) 443. (A) Desm opressin is usu ally taken tw ice d aily by intranasal ad m inistration. In contrast to vasop ressin that is rap id ly inactivated , the d u ration of the effect of d esm op ressin is several hours. Desm op ressin m ay increase blood p ressure, but it d oes so to a m uch lesser d egree than vasopressin. Desm opressin increases the circu lating concentrations of factor VIII and von Willebrand factor, and has no effect in p atients w ith nephrogenic d iabetes insipid us. While there is not likely to be a need for intraop erative ad m inistration of d esm op ressin if the patient w as given a d ose by inhalation preop eratively, it m ay be given by intravenou s infu sion to d ecrease seru m osm olality and increase w ater retention. (1:708, 710) 444. (C) H yd ralazine m ay be associated w ith a lu p u s-like synd rom e. It is a d irect vasod ilator and m ay cause reflex tachycard ia that limits its u sefu lness in the p atient w ith angina. There m ay be retention of sod iu m and w ater that m ay requ ire d iuretics. (1:779-780) 445. (C) The eld erly generally requ ire a low er d ose of th iop en tal to in d u ce an esth esia. With increasing age, MAC for inhalational anesthetics d ecreases, and recovery of norm al ventilatory d rive after fentanyl ad m inistration is d elayed . In contrast, w ith increasing age there

123

is no alteration in th e in tu batin g d ose of vecuronium. The rate of hepatic synthetic reactions, such as the glucuronid ation of morphine, is also u nchanged in the eld erly, w hile the rate of hep atic oxid ative and red u ctive reactions d eclines w ith increasing age. (5:607, 680-1) 446. (E) The aim of antacid ad m inistration is to raise the p H . It shou ld be given 15-30 m in before su rgery. Antacid s w ill not d ecrease gastric volu m e bu t m ay actu ally increase it. N onp articu late antacid s shou ld not cau se any p roblem if asp irated . There is no lag tim e. (1:1315; 5:1159, 1657) 447. (A) Bu torp hanol is a m ixed op ioid agonist– antagonist. N aloxone is a pu re op ioid antagonist, m ethohexital a short-acting barbitu rate, m id azolam a benzod iazepine, and bleom ycin a d ru g used in cancer chem otherapy. (1:509-11) 448. (C) Patients w ho abu se alcohol m anifest tolerance to its CN S effects bu t not to its ventilatory effects. The acutely intoxicated person requ ires less anesthesia, w hereas the chronic alcoholic w ho is not acu tely intoxicated requ ires m ore. Becau se alcoholic liver d isease m ay lead to a d ecreased cap acity to m etabolize local anesthetics, local anesthetic toxicity m ay be m ore likely in the alcoholic. (5:321-3) 449. (B) Chlorop rocaine is the least toxic local anesthetic by virtu e of its rap id hyd rolysis after accid en tal in traven ou s in jection . (1:573-4; 5:781) 450. (C) The H en d erson–H asselbalch equ ation states that p H = pK + log [proton accep tor/ p roton d onor]. With a w eak base like m epivacaine, the u ncharged form is the p roton acceptor, an d 39% is p resen t in th is form at p hysiologic p H . Even w ithou t a calcu lator, an ap p roxim ation is p ossible. If the p H w ere equ al to th e p K of th e d ru g, equ ivalent am ou nts of the charged and u ncharged m oieties w ou ld be present. In this case, the p H is slightly low er than the p K, favoring slightly less than half of the d rug to be in the uncharged form . (5:769)

124

7: P ha rma cology

451. (B) At levels below 1.5 MAC, the net effect of halogenated hyd rocarbons is a d ecrease in cerebral blood flow. Opioid s, id eally as a continu ou s infu sion, should be ad m inistered concom itantly (balanced anesthetic), to d ecrease the am ou nt of volatile anesthetic requ ired . N itrou s oxid e and ketam ine are both likely to cau se an increase in CBF. The ad m inistration of both p henylep hrine and d obu tam ine can increase CBF. (5:873-4) 452. (D ) A w eak base is less ionized at higher p H valu es, thu s rend ering it m ore lip id -solu ble and m ore likely to d istribu te to the interstitial flu id . The converse is tru e of w eak acid s. Increased blood flow to a tissu e increases the d istribution of d rugs to that tissue. Distribution is d ecreased by bind ing to p lasm a p rotein and by d ecreased lip id solu bility. (1:30-3) 453. (D ) The loading d ose of a drug is the d ose given to achieve a faster blood concentration, and is calculated based on the volume of distribution at steady state (Vss), the desired plasma concentration (Cp), and the drugs bioavailability (F). While the load ing d ose d ecreases the tim e need ed to achieve a stead y state concentration, the d ru g concentration is initially higher because d rug d istribution after initial bolus is lim ited to the sm aller, central com p artm ent. With some d rugs, the larger d ose can have d etrim ental effects, e.g., a larger d ose of relaxant may lead to a longer time of paralysis or card iovascular effects. (1:37) 454. (B) Amiod arone is a structural analogue of thyroid hormone and exerts a variety of pharmacological effects. H ypotension after intravenous ad ministration of the d rug is d ue to vasod ilation and depression of myocardial performance. Ad enosine, verapam il, and p rocainamid e are some of the d rugs used for the treatment of card iac d ysrhythmias associated w ith the WPW synd rome; amiod arone is not ind icated . Drug toxicity d uring oral loading regimens is uncommon, and may includ e nausea. Long-term therapy can cause significant pulmonary fibrosis. The adverse effects of amiodarone are thought to be attributable to the drugs interaction w ith

nu clear thyroid horm one recep tors. (1:834; 5:764; 6:1839, 1911) 455. (E) Digoxin is an inotrop e and m ay cau se brad ycard ia. The m echanism involves inhibition of N a,K-ATPase. Calciu m w ill potentiate the effects of d igoxin. Digoxin toxicity can p resent as d escribed in the vignette, inclu d ing a variety of severe cond u ction d istu rbances. Sid e effects of carved ilol inclu d e brad yarrhythm ias, bu t the d ru g is u nlikely to cau se visu al d istu rbances. Qu inid ine can p rolong the QT-interval and cau se p olym orp hic ventricu lar tachycard ia. The ad verse effects of am iod arone includ e p u lm onary fibrosis, and an overd ose of benzod iazepines would cause respiratory depression. (1:803-4; 5:749, 753) 456. (C) All h alogen ated volatile an esth etics d ecrease cerebral m etabolic rate in a d osed epend ent m anner, and are associated w ith a d ecrease in cerebral electrical activity. They d iffer, how ever, in their potential to cause cerebral vasodilation that could raise ICP in patients w ith poor compliance. Sevoflurane causes less vasod ilation than either isoflurane or d esflurane. Cerebral metabolic rate rises d ramatically d uring seizure activity. (1:542-6; 5:874) 457. (C) Coverage of steroid therap y at the tim e of su rgery is im portant to avoid stress reactions and acu te ad renal insu fficiency. Rep lacem ent d oses are based on the anticip ated stress of the operative proced u re. Cholecystectom y is consid ered m od erately invasive, and the m ost ap p rop riate regim en is the one ou tlined in option C. Option D is appropriate for m inor p roced u res, w hile op tion E w ou ld be the recom m end ed regim en for cases such as card iac, or m ajor abd om inal su rgery. (5:157) 458. (D ) A p atient w ith the p resented sym p tom s m ay be having an anap hylactic reaction. In ad d ition to the interventions ou tlined in the vignette, blood pressure should be sup ported w ith epinephrine in 100-m cg increm ents that is the cornerstone of initial therapy. In addition, glucocorticoids and diphenhydramine should be administered as secondary treatment. The most

Answe rs : 451–467

com m on cau ses of anap hylactic reaction s d u ring anesthesia care inclu d e m u scle relaxants and antibiotics. (5:1484) 459. (E) The patient is m ost likely su ffering from m agnesium d eficiency that is com m on in m alnou rished alcoholic patients and often accom p an ied by h yp okalem ia. Th e arrh yth m ia d escribed is con sisten t w ith torsad es d es p ointes, w hile other find ings m ight inclu d e Trou sseau and Chvostek signs that are also fou n d in states of hyp ocalcem ia. (5:520-1; 6:1891) 460. (D ) The op ioid s u su ally p rod u ce u nconsciou sness at high d osage, bu t it is im portant to recognize that u nconsciou sness is not absolu te. The p u pils are constricted . Opioid s d o not p rod u ce reliable am nesia, and the nau sea and vom iting seen are m ed iated throu gh the central nervou s system in the chem orecep tor trigger zone. (1:488-490) 461. (B) The p atient w ith hyp erthyroid ism shou ld be ad equ ately p rep ared p reop eratively. This u su ally involves the preoperative ad m inistration of β-adrenergic antagonists to decrease the heart rate. Aspirin is u sually avoid ed becau se it m ay d isp lace thyroid horm on e from its p lasm a protein bind ing sites. Ad m inistration of exogenou s thyroid horm one shou ld be avoid ed , as should ad m inistration of am iod arone that is a stru ctu ral analogu e of thyroid hormone and can trigger thyroid storm. (1:834; 5:151-2) 462. (B) For coronary “steal” to occur, there m ust be a stenosed vessel supplying an area of myocard ium that is depend ent upon flow through collateral vessels. Isoflu ran e m ay th en cau se vasod ilation that d iverts blood aw ay from these collaterals, lead ing to ischem ia in the collateral-d ep end ent area of m yocard iu m . In sp ite of this theory, and the fact that m any persons have “steal-p rone” coronary anatom y, there is no evid ence that the use of isoflu rane is associated w ith an increase in p eriop erative ischem ia or m ortality. The p henom enon of

125

“steal” d oes not occu r w ith either d esflu rane or sevoflurane. (1:543; 5:612) 463. (D ) Corticorticoid s u sed for the treatm ent of asthm a shou ld be of the glu cocorticoid typ e since no m ineralocorticoid effect is d esired . The onset tim e is m easu red in hou rs. Giving the d rug by inhalation d oes not d ecrease the onset tim e bu t it d ecreases system ic toxicity becau se low er d oses may be u sed . Inhaled corticosteroid s are th e m ost effective an tiinflam m atory agents u sed in asthm a therap y. (5:131-2; 6:2110-1) 464. (A) The patient with renal failure usually does not need an altered dose of medications whose effects are term inated by red istribu tion . Pancuronium is primarily eliminated unchanged by the kidney. (1:274, 501) 465. (E) Ionization of a drug is important in the drug's function, since charged particles do not cross membranes well. The degree of ionization is a function of the pH of the solution and the pKa of the drug as given by the Henderson–Hasselbalch equ ation. The Michaelis-Menten equ ation describes transporter-mediated processes across biological membranes. (1:18-9, 95) 466. (C) Obese p atients have an increased volu m e of d istribu tion and an increased term inal halflife for highly lip id solu ble agents such as m id azolam and thiopental. The d egree of isoflurane metabolism is not significantly changed . Obese su bjects have higher levels of pseu d ocholinesterase activity; w hen a larger d ose of su ccinylcholine is given to an obese p atient, the d u ration of action is sim ilar to that seen w hen a sim ilar d ose (in m g/ kg) is given to a lean p atient. Even thou gh d igoxin is a highly lipop hilic d ru g, it has a com p arable volu m e of d istribu tion in obese an d lean in d ivid u als. (5:308-10) 467. (A) All of the options listed are targets for d ifferent classes of antiem etic agents. Granisetron is a 5-H T3 recep tor antagonist. (1:1342)

126

7: P ha rma cology

468. (D ) Acute intermittent porphyria is an inborn error of metabolism in w hich porphobilinogen cannot be converted to uroporphyrin. Therefore, the urinalysis is positive for porphobilinogen, w hich is pathognomonic, and negative for urop orp hyrins. Thiop ental ind u ces the enzym e δ-aminolevulinic acid synthetase that results in increased synthesis of p orp hobilinogen and therefore w ill w orsen acute interm ittent porphyria. This disease is associated with a constellation of sym p tom s and signs, the w orst of w hich are neurologic; how ever, cardiac arrest w ith thiopental is not among them. Morphine may be safely ad ministered . (5:141-2, 695) 469. (C) Both renal insu fficiency and p erip heral neu rop athy m ay persist after cou rses of therapy w ith cisplatin. Congestive heart failu re has been rep orted after high d oses of d oxoru bicin. Pu lm onary fibrosis is a serious comp lication of bleom ycin therap y, and hep atic cirrhosis m ay occu r after long-term therapy w ith m ethotrexate, for exam p le in p atients w ith p soriasis. (1:1689, 1694, 1714, 1718) 470. (D ) Dissociative anesthesia is noted by the presence of open eyes and occasional m u scle m ovem ents that are not signs of inad equ ate anesthesia. Mu scle tone m ay be increased in d issociative anesthesia, and the p atient is not necessarily u nconsciou s. (1:538; 5:1265) 471. (B) This patient is most likely having a reaction to the methylparaben preservative present in multiple-dose vials of lidocaine. Persons allergic to methylparaben are also likely to be allergic to local anesthetics of the ester typ e, su ch as p rocaine. Since tru e allergy to lid ocaine is extremely rare, a second dose of preservativefree lidocaine is very unlikely to cause a reaction. Bacterial contamination of a preservative-containing solution is also unlikely. Local anesthetic preparations containing a vasoconstrictor may also elicit allergic reactions due to sulfite added as an antioxidant. (1:571-2; 5:777) 472. (D ) Cap top ril inhibits conversion of angiotensin I to angiotensin II and thu s d ecreases arteriolar and venou s tone. (1:732-4)

473. (A) Dru g antagonism occu rs w hen tw o d ru gs have effects that tend to counteract one another. This m ay occu r by com bination, as in the case of heparin–p rotam ine interaction, or they m ay com p ete for the sam e recep tor site, as w ith m u scle relaxants and acetylcholine. The hallm ark featu re of com petitive antagonism is the lack of intrinsic efficacy of the antagonist. The d ose resp onse cu rve of the agonist w ill be shifted to the right, w ith no change in the m axim al resp onse. (1:46-8) 474. (B) Su ccinylcholine w ill have a p rolonged effect in a p erson d eficient in p seud ocholinesterase, either on the basis of a genetic d efect or because they have taken a cholinesterase inhibitor su ch as ech oth iop h ate th at is read ily absorbed from the eye. Su ccinylm onocholine is a metabolite of succinylcholine w ith minimal p otency and u nlikely to be the cau se of p rolonged p aralysis. While op ioid overd ose can cau se apnea and u nresponsiveness, the d ose of fentanyl ad m inistered to the p atient in the vignette is too sm all to explain her sym ptom s, and the sam e is tru e for the short-acting hyp notic propofol. Since 70% N 2O d oes not cau se ap nea, it is not likely to be associated w ith this p atient's current signs. (1:250; 5:500) 475. (A) In trying to d eterm ine the cause, a nerve stim ulator should be used to assess the level of m uscle relaxation. If no tw itches are elicited , a p rolonged effect of su ccinylcholine is d em onstrated . Ad m inistration of the benzod iazepine antagonist flu m azenil is not ind icated , since there is no m ention of benzod iazepine ad m inistration in the vignette. Ad m inistration of carbon d ioxid e by inhalation is technically d ifficu lt in m ost settings and w ill not p rovid e a d efinitive answ er. There is no reason to ad m inister another d ose of su ccinylcholine. (1:459; 5:500-1) 476. (D ) Once it has been established that there is no tw itch, the only course to take is to ventilate the patient as long as is necessary. Even w ith no m easu rable p seu d ocholinesterase activity, paralysis d ue to an intu bating d ose of su ccinylcholine shou ld w ear off w ithin an hou r or tw o d u e to renal elim ination of unm etabolized

Answe rs : 468–482

su ccinylcholine. N eostigm ine w ill not reverse the p aralysis if p seu d ocholinesterase d eficiency is the cau se. Pralid oxim e is unlikely to be effective in reactivating cholinesterase that is chronically inhibited by echothiop hate, and physostigmine is occasionally used in the treatm ent of cen tral anticholinergic syn d rom e. (1:86, 268; 5:500-1, 1461) 477. (B) The intram u scu lar rou te m ay be u sed for aqu eous or nonaqueou s solu tions and p erm its m ore rap id on set than after su bcu taneou s injection. Irritating su bstances that m ay cau se p ain if injected su bcu taneously m ay be tolerated by the intram u scu lar rou te. Absorp tion from the d eltoid or vastu s lateralis m u scles is m ore rap id than from the glu teu s m axim u s. The rate of absorption follow ing intramuscular ad ministration can d iffer significantly betw een m ales and fem ales, particu larly w ith injections into the glu teu s m axim u s that has been attribu ted to d ifferent d istribu tions of subcutaneous fat. Dru gs injected into the m u scle are su bject to first-p ass m etabolism in the lu ng; the only exception is the intra-arterial rou te. (1:22-3) 478. (D ) Because of their rapid onset times, atropine shou ld be given at the sam e tim e as ed rophon iu m in ord er to m in im ize sid e effects. Sim ilarly, glycop yrrolate shou ld be given at the sam e tim e as neostigm ine. The m axim u m recom m end ed d ose of neostigm ine is 75 m cg/ kg; this w ill p rod u ce m axim al inhibition of acetylcholinesterase and a larger d ose m ay actu ally cau se d ep olarizing neu rom u scu lar blockad e. The d isap p earance of fad e d u ring d ou ble-bu rst stim u lation ind icates su fficient recovery of m u scu lar fu nction to p erm it end otracheal extubation. A TOF ratio of 0.7 d oes not gu arantee com p lete recovery of p haryngeal function; even at TOF ratio of 0.9 some patients m ay have incom plete recovery of pharyngeal m uscles. (1:247, 267; 5:507) 479. (E) Myotonic d ystrop h y is a m em ber of a d iverse grou p of hered itary skeletal m u scle d isord ers. It is u su ally exacerbated d u ring p regnancy, and is associated w ith CH F and

127

restrictive lu ng d isease. Ad ministration of su ccinylcholine in these p atients can lead to hyp erkalem ic card iac arrest and to m u scle spasm so severe that ventilation may be impossible. N ond epolarizing neu rom u scular blocking agents w ou ld not p revent or treat the m uscle spasms in m yotonia becau se the pathologic lesion is at the m u scle m em brane level. Patients w ith m yasthenia gravis are resistant to the effects of su ccinylcholine d u e to the d ecreased nu m ber of acetylcholine recep tors. Du chenne typ e is the m ost com m on m u scu lar d ystrop hy. It is recessive sex-linked , and w ill affect m ales; how ever, fem ales m ay exhibit subclinical sym ptom s. Patients w ith hypokalem ic p eriod ic p aralysis m ay have increased serum potassium levels after ad m inistration of su ccinylcholine, how ever, card iac arrest is far less likely. Charcot-Marie-Tooth d isease is the m ost freq u en t p er ip h er al n eu rop ath y. Avoid ance of d ep olarizing N MB d ru gs is ad visable bu t again, u nlikely to cau se card iac arrest. (5:140-5) 480. (C) The neu rolep tic state is characterized by red u ced m otor activity and ind ifference to the su rround ings. N either am nesia nor analgesia are p rod u ced w hen a neu rolep tic agent is ad m inistered by itself. (1:417; 5:701) 481. (D ) All inhalational anesthetics can p rod u ce CO d u e to their interaction w ith strong bases in carbon d ioxid e absorben ts, p articu larly w hen the absorbent is d ry (su ch as w hen it has been flu shed w ith d ry gas d u ring an entire w eekend ). The relative p rop ensity for p rod u cing carbon m onoxid e is d esflu rane > enflu rane > isoflu ran e > sevoflu ran e > h aloth an e. Recom m end ations to d ecrease the risk of carbon monoxid e prod uction includ e low fresh gas flow s, u se of fresh absorbent, and the u se of sod a lim e instead of baralym e. (5:614) 482. (D ) Ad d itive effect refers to a com bined effect that is the algebraic su m of the ind ivid u al actions. The effect d escribed in op tion E w ould be synergistic. (1:77)

128

7: P ha rma cology

483. (A) An increased initial slop e of the cu rve ind icates an increased rate of sevoflu rane u ptake. An increased rate of u ptake w ill occu r if the insp ired concentration of sevoflu rane is increased , if nitrou s oxid e is ad d ed to the insp ired gas m ixtu re (the second gas effect), or if m in u te ven tilation is in creased . An increased cardiac output w ill cause a decreased rate of anesthetic u p take. Pu lm onary rightto-left shunting w ill lead to a d ecrease in alveolar u ptake. (5:603-5) 484. (B) Patients w ith the Eaton–Lambert syndrome have increased sensitivity to both d ep olarizing and nond epolarizing mu scle relaxants becau se of d ecreased release of acetylcholine at the neu rom u scular ju nction. The synd rom e d oes not resp ond to acetylcholinesterase inhibitors, and nond epolarizing neu rom uscular blockad e is d ifficu lt to reverse. As op p osed to som e patients w ith m uscu lar d ystrophies, the m yasthenic synd rom e is not associated w ith an increased risk of MH . (5:143-4) 485. (A) Clostridium perfrigens is a gram -p ositive rod . The d ru g of choice for gas gangrene is penicillin G that is often com bined w ith clind am ycin d ue to its ability to red u ce toxin exp ression. Pharm acologic therapy alw ays has to be com bined w ith ad equ ate d ebrid em en t of infected areas. Gas gangrene has to be treated w ith intravenou s antibiotics; none of the other com binations listed w ould be consid ered firstline regim ens. (5:1485, 1535) 486. (B) Cim etid ine inhibits the oxid ative m etabolism of m any d ru gs, inclu d ing d iazep am , by inhibiting cytochrom e P450. While oral d iazepam is an excellent oral preop erative sed ative for p atients u nd ergoing inp atient su rgery, the d ose m ay need to be red u ced in the setting of chronic cim etid ine therapy. Chronic ingestion of ethanol increases cytochrom e P450 levels. There is no inhibitory effect of nitrou s oxid e on cytochrom e P450. (5:699) 487. (C) H u man enzym es are not thou ght to be able to m etabolize nitrous oxid e. The red uction of nitrou s oxid e to nitrogen occu rs to a sm all

d egree in the gu t and is catalyzed by bacterial enzym es. N itrou s oxid e is alm ost com pletely elim inated , bu t not m etabolized , by the lu ngs. (1:546-7) 488. (C) While m ethylm ethacrylate can act as a d irect vasod ilator in vitro, plasm a concentrations ach ieved d u ring cem enting of joint rep lacem ents are 10-20 fold below th ose requ ired to cau se hyp otension. The m echanism of bone implantation syndrome is d escribed in op tion C, and can lead to acu te p u lm onary hyp ertension and right heart failu re. An air em bolu s of h em od yn am ic con sequ en ce is m u ch less likely, and cannot be effectively treated via aspiration of blood through the central venous catheter. Venting of the long bone d oes n ot reliably p reven t card iovascu lar com p rom ise. (5:1204) 489. (D ) Ond ansetron is a 5-H T3 receptor antagonist. It is an antiem etic and acts at the chem orecep tor trigger zone. It p rod u ces no sed ation and has few, if any, ad verse effects. It has no effect on gastrointestinal m otility. (1:1341) 490. (B) The sam e d ose of propofol is not approp riate for all p atients. An u nd erstand ing of the d iagram is im p ortant for the p rop er u se of intravenou s m ed ications. A p atient in shock w ill h ave an in creased effect of p rop ofol becau se the fraction of the card iac ou tp u t reach in g th e brain is h igh er in sh ock. Fu rtherm ore, the rate of red istribu tion from the brain is slow er becau se of d ecreased perfusion of other tissu es. The patient w ho is anxiou s and has an increased card iac ou tp u t w ill have faster red istribution, and the effect w ill be shorter. Another im portant factor to consid er in p rop ofol d osing is p atient age: the effect-site concentration of a given p rop ofol d ose w ill increase w ith increasing age. (5:680-3) 491. (C) Most strains of Staphylococcus aureus p rod u ce beta lactam ase that w ill rend er p enicillin G and am p icillin ineffective. Cefazolin and other first generation cep halosporins and vancom ycin are both effective d ru gs for singleagent p rop hylaxis of su ch a w ou nd infection;

Answe rs : 483–498

129

how ever, the latter is u su ally reserved for selected p atients at higher risk for MRSAinfection. Levofloxacin is cu rrently not recom m end ed as a first-line agent for su rgical site infection p rop hylaxis. (5:235-6)

p rod uce equivalent antithrom botic effects. At least half of the LMWH chains are too short to brid ge antithrom bin to throm bin; the sam e is tru e for the p entasaccharid e fond ap arinu x. (1:853-6)

492. (B) Carbon m on oxid e shifts the oxygen – hem oglobin d issociation cu rve to the left; thu s oxygen is bound m ore tightly and is less easily released in tissu es. Carbon m onoxid e com bines w ith hem oglobin at one-tenth the rate of oxygen, bu t d issociates from hem oglobin at only 1/ 2200 the rate of oxygen. The affinity of hem oglobin for carbon m onoxid e is therefore 220 times that of oxygen, and an increased FIO 2 p rovid es no protection from carbon monoxid e. The absorbance sp ectra of carboxyhemoglobin and oxyhem oglobin are sim ilar, and stand ard p u lse oxim eters cannot d istingu ish betw een the tw o. Oxygen saturation read ings can therefore be norm al, even in the p resence of lethal am ou nts of carboxyhem oglobin. (5:1336)

496. (B) Aspirin overd ose is a lead ing cau se of overd ose m orbid ity and m ortality. Peak p lasm a concentration m ay not be reached for 4-35 h after intoxication w ith the enteric-coated form . The d ru g cau ses the u ncou p ling of oxid ative p hosphorylation resu lting in increased carbon d ioxid e p rod u ction w ith resu lting hyp erventilation an d h yp erth erm ia. CN S sym p tom s inclu d e confu sion, d izziness, tinnitu s, d eliriu m , p sychosis, and in som e cases com a. Miosis and respiratory d epression are characteristic of intoxication w ith op ioid s. While lithiu m overd ose can cau se severe CN S sym p tom s an d com a, sym p tom s w ou ld likely becom e ap p arent sooner after ingestion. The sym ptom s of acu te acetam inophen poisoning are m ore insid iou s, p resenting as p red om inantly GI sym ptom s over the first tw o d ays, follow ed by acu te hep atic failu re in severe cases. (1:75, 450, 981, 984)

493. (E) Filtration occu rs w h en d ru gs d iffu se through aqueous pores in m em branes. Passive nonionic d iffu sion is the p rocess by w hich lip id -solu ble com p ou nd s d iffu se throu gh the lip oid al p ortions of m em branes. Active transp ort m echanism s u se carrier m olecules for certain ligands and are thu s subject to satu rability. Op tion D, w hile a m ed iated transport process, falls into the category of p assive transm em brane transp ort. (1:19, 674) 494. (B) H eroin is d iacetylm orphine. One or both of the acetyl grou ps m ay be rem oved via hyd rolysis to yield acetylm orp hine and m orp hine, resp ectively. (1:499-501) 495. (B) Low -m olecu lar-w eight hep arins (LMWH ) have a m ore pred ictable pharm acokinetic p rofile than stand ard hep arin that allow s w eightad justed subcutaneous ad ministration w ithout routine laboratory m onitoring. If monitoring is d eem ed necessary, anti-factor Xa activity can be d eterm in ed . Th e incid en ce of hep arin ind u ced throm bocytop enia is low er as com p ared to stand ard hep arin. Even if sim ilar anti-factor Xa activity is achieved w ith any of these agents, it cannot be assum ed that they

497. (C) Cytochrom e P450 p erform s m any oxid ative reactions, includ ing the oxid ative d em ethylation of ketam ine. While cytochrom e P450 is cap able of perform ing the oxid ative d eam ination of norepinephrine in vitro, the reaction is performed by monoam ine oxid ase in vivo. The acetylation of hyd ralazin e is catalyzed by N -acetyltransferase, the conju gation of m orp hine to glu cu ronid e is catalyzed by glucu ron yl tran sferase, an d th e con ju gation of acetam inop hen to su lfate is catalyzed by su lfate transferase. (1:127-8, 202) 498. (B) Sw eating and lacrim ation are tw o of the early signs of the abstinence synd rom e. Chills and p ilom otor activity are p rom inent effects a d ay or tw o after the last d ose of m orp hine. Seizu res and com a are not a part of the opioid w ithd raw al syn d rom e, bu t d o occu r w ith abru pt w ithd raw al of barbitu rates and benzod iazep ines. Brad ycard ia is one of the sym p tom s of w ithd raw al from cocaine. (1:660-2)

130

7: P ha rma cology

499. (E) All of these sym p tom s m ay occu r w ith ACE-inhibitor therap y. Cou gh is a com m on sid e effect, w hereas a loss of sense of taste is rare. Acu te renal insu fficiency is a risk in p ersons w ith renal artery stenosis or a history of CH F. Proteinu ria is less com m on, bu t is m ore likely to occu r in p ersons w ith u nd erlying renal im p airm ent. (1:735-6) 500. (A) The vessel-rich grou p in clu d es those organs that receive the bulk of circu lation. It inclu d es the heart, brain, kid neys, and liver. Mu scle is m ore p oorly p erfu sed . The vesselrich grou p of tissu es receives d ecreasing blood flow w ith increasing age, and rep resents abou t 10% of total bod y w eigh t in ad u lts, an d app roxim ately 22% in the neonate, com p ared to 13% in accou nting for the vessel-poor grou p in that age grou p . (5:254) 501. (D ) Ketorolac is a cyclooxygenase inhibitor that has high efficacy as an analgesic. Its m ajor ad verse effect is ep igastric d istress. It is m u ch less likely than m orp hine to cau se sed ation, ventilatory d epression, or nausea. (1:973, 986; 5:1304) 502. (E) The hyp ovolem ic p atient has a greater fraction of the card iac outpu t reaching the brain, and red istribu tion is less rap id d u e to the d ecreased blood flow to peripheral tissues. The im p act of blood loss on the effects of p rop ofol is d ram atic, w ith an estim ated 5-fold increase in d u ration of action, and a need for 80% d ose red uction in severe blood loss to provide effectsite concentrations equivalent to a person w ith norm al card iovascu lar p hysiology. The CN S resp onse m ay also be affected by the other factors listed , w hich alter the am ou nt of d ru g reaching the brain in the tim e im m ed iately follow ing injection, bu t not to the extent as option E. The patient w ith renal failu re is often h yp oalbu m in em ic; su ch p atien ts n eed a d ecreased d ose of ind u ction agent d u e to less protein bind ing that cau ses a higher unbou nd fraction to be available. Most, bu t not all eld erly p atients requ ire low er ind u ction d oses of intravenous anesthetics, although the mechanism is unclear. Drug d osing in obese patients should take into consid eration lean bod y m ass

for the initial bolu s to avoid excessive effectsite concentrations. (5:676-83) 503. (C) Un like haloth an e, isoflu ran e p rod u ces burst suppression of the EEG at 2 MAC. During carotid end arterectom y p erform ed u nd er isoflurane anesthesia, the cortical blood flow level at w hich EEG m anifestations of ischem ia are d etected is low er than that of halothane. N either agent abolishes autoregulation of cerebral blood flow. Isoflu rane, althou gh p rod u cing less increase in CBF than halothane, is still a cerebral vasod ilator. Sevoflu rane and enflurane have been rep orted to au gm ent ep ilep tic brain activity. (5:611) 504. (D ) Lam otrigine is an anticonvu lsant agent u sefu l in the treatm ent of partial and generalized tonic-clonic seizures. It is also effective for the treatm ent of absence seizu res. Its half-life is red u ced w hen p entobarbital, carbam azep ine, or phenytoin is ad m inistered concom itantly. The d rug is prim arily m etabolized by glucuronid ation as op posed to gabapentin that is not m etabolized by hu m ans. Lam otrigine also has a role in the treatm ent of variou s neu rop athic p ain synd rom es, bu t is consid ered second line d u e to significant sid e effects. (1:584, 599-600; 5:1562) 505. (B) Alterations in total hep atic blood flow d uring anesthesia and surgery can cause hepatocellu lar isch em ic in ju ry w ith resu ltin g hepatic failu re in p atients w ith stable, u nd erlying chronic liver d isease. Volatile anesthetics d ecrease hep atic blood flow, halothane to a greater d egree than sevoflu rane and isoflu rane. There is little effect of p rop ofol or nitrou s oxid e on hepatic blood flow. H alothane hepatitis is thou ght to be cau sed by an im m u ne resp onse to hep atic p roteins that becom e trifluoroacetylated as a consequence of halothane m etabolism . (1:537, 543; 5:193, 613-5) 506. (D ) Clonid ine is a centrally acting sym p athetic inhibitor and m ay su p p ress som e w ithd raw al sym ptom s. Chlorp rom azine is ineffective for this ind ication. Bu torp hanol and nalbu p hine are both op ioid agonist–antagonist m ed ications that are likely to precipitate w ithd raw al.

Answe rs : 499–515

Flu m azen il is a ben zod iazep in e recep tor antagonist. (1:468, 497, 661) 507. (D ) The emergence reactions that are seen w ith ketam in e, an N MDA recep tor an tagon ist, inclu d e d ream ing, hallu cinations, and d elirium. These effects are less com mon in child ren. While ketamine prod uces am nesia for intraop erative events, the am nesia, as opposed to profou nd analgesia, d oes not p ersist into the p ostop erative p eriod , an d m an y p atients rem em ber their d ream s or hallu cinations. The em ergence reactions can be m inim ized by the concu rrent ad m inistration of a benzod iazep ine. (1:539; 5:699-700) 508. (B) Om ep razole is an inhibitor of the H + ,K+ ATPase responsible for the synthesis of hyd rochloric acid in the stom ach. In typ ical d oses, it d ecreases the d aily acid p rod u ction by abou t 80-95%. In contrast, ranitid ine, as a representative of the H 2 antagonists, w ill d ecrease acid p rod uction by abou t 70%. Om ep razole has no effect on gastric em p tying or on the volu m e of gastric ju ice. It is m etabolized by cytochrom e P450 and inhibits the m etabolism of som e other m ed ications by this enzym e. (1:1311-2) 509. (D ) In p atients w ith p reexisting vitam in B12 d eficiency, even relatively short exp osu res to n itrou s oxid e can p rod u ce m egaloblastic changes and a neu rop athy. The neu rologic findings characteristic of vitamin B12 d eficiency are a bilateral p erip heral n eu rop ath y that affects p red om inantly the low er extrem ities as w ell as an u nstead y gait and d im inished d eep tend on reflexes. N itrous oxid e has been show n to irreversibly inactivate the enzym e m ethionine synthase that ultimately lead s to impaired synthesis of phosp holipid s, m yelin, and thym id in e th at is an essen tial DN A-base. Megaloblastic hem atop oiesis and su bacu te com bined d egeneration of the sp inal cord can ensu e. (1:546-7; 5:613) 510. (E) Pharm acologic p rep aration of the p atient d oes not p reclu d e good airw ay m anagem ent. H istam in e H 2-recep tor an tagon ists d o n ot p rotect against asp iration, d o not facilitate gastric em p tying, nor d o they have any effect

131

on flu id that is alread y present in the stom ach. If ad m inistered 2-3 h before ind u ction of anesthesia, they w ill increase gastric p H and by d ecreasing gastric acid secretion lead to low er resid u al gastric volu m es. There is no need to u se histam ine H 2-recep tor antagonists in all p atients. (1:1313; 5:1033-4) 511. (B) All volatile anesthetics and d ep olarizing m u scle relaxants (eg, su ccin ylcholine) are know n to be triggering agents in p atients su sceptible to m alignant hyp ertherm ia. All other m ed ications are consid ered safe. (5:615, 1497) 512. (A) Losartan is a com p etitive antagonist of angiotensin II. Prazosin is an α-ad renergic antagonist, nesiritid e a recom binant B-typ e natriu retic p ep tid e, and m inoxid il and hyd ralazine are arterial vasod ilators. (1:767; 5:747) 513. (A) Ad m inistration of a vasod ilator su ch as nicard ip ine w ou ld m ove the ventricular fu nction curve from A to C, resu lting in im proved ventricu lar fu nction w hile red u cing card iac filling p ressu res. An inotrop e, su ch as d obu tam ine w ou ld resu lt in a higher ventricu lar curve, from A to B, resulting in greater card iac w ork at the sam e ventricu lar filling p ressu res. Diu retics im p rove CH F sym p tom s by m oving filling p ressu res along the sam e ventricu lar fu nction cu rve from A to D. The com bination of inotrop es and vasod ilators w ould result in a shift of the Frank-Starling cu rve from A to E. (1:792) 514. (E) The com bination of inotrope, d iuretic, and vasod ilator w ou ld result in a shift of the FrankStarling cu rve from A to F. (1:792) 515. (B) Fosp rop ofol is a w ater-solu ble p rod ru g of p rop ofol sup p lied as a 3.5% aqu eou s solu tion and is not associated w ith pain on injection. Unlike p rop ofol the d ru g has a sm all volu m e of d istribu tion, and has a significant lag tim e of ap p roxim ately 3-4 m in after intravenou s ad m inistration before consciou sness is lost. The recom mend ed bolu s d ose is 6.5 mg/ kg. (1:536; 5:693)

132

7: P ha rma cology

516. (E) All of the agents m entioned w ill lead to a d ecrease in p u lm onary vascu lar resistance, each through a d ifferent m echanism . Inhaled nitric oxid e is u sed for the treatm ent of acu te p u lm onary hyp erten sion and acts throu gh stim u lation of soluble gu anylate cyclase w ith resu ltant vascu lar sm ooth m uscle relaxation. Intravenou s p rostacyclin (ep op rostenol, PGI2) ind u ces relaxation of smooth m u scle by stim u lating the p rod u ction of cyclic AMP and can also be ad m inistered via in halation. Other d ru gs in this category are inhaled ilop rost and su bcu taneous treprostinil. Sild enafil is a phosp hod iesterase typ e 5 inhibitor. These d ru gs have an acu te p u lm onary vasod ilator effect that is d u e to enhancem ent of nitric oxid em ed iated pulm onary vasod ilation. Bosentan is an orally active end othelin antagonist used for th e treatm en t of p r im ar y p u lm on ar y hypertension. (1:1059-60) 517. (K) Antid iu retic horm one, or arginine vasop ressin, is an agonist at the vasop ressin V1 recep tor. (1:704)

518. (F) Inam rinone is an inhibitor of p hosphod iesterase-3 (PDE3). (1:805) 519. (H) Sild enafil is an inhibitor of p hosp hod iesterase-5 (PDE5). (1:50) 520. (I) Valsartan is a com p etitive antagonist at the angiotensin-1 (AT1) recep tor. (1:736) 521. (G) Sp ironolactone is a com p etitive antagonist of ald osterone in the collecting d uct. (1:692-3) 522. (B) Acetazolam id e in an inhibitor of carbonic anhyd rase in the proxim al tu bu le. (1:677-8) 523. (E) Fu rosem id e is an inhibitor of the N a + -K+ 2Cl– sym port in the thick ascend ing limb of the loop of H enle. (1:682)

CHAPTER 8

Ge ne ral Ane s the s ia Que s tions DIRECTION S (Qu estions 524 throu gh 558): Each of the nu m bered item s or incom plete statem ents in this section is follow ed by answ ers or by com pletions of the statem ent. Select the ON E lettered answ er or com pletion that is BEST in each case.

at m ost 20 feet before need ing to rest. H e is sched u led to u nd ergo an exploratory laparotom y becau se of a sm all bow el obstruction. H e w ou ld be classified by the Am erican Society of Anesthesiologists as p hysical statu s

524. A patient w ith obstru ctive lung d isease has an altered anesthetic ind u ction w ith an insolu ble agent because of

(A) (B) (C) (D) (E)

(A) (B) (C) (D) (E)

d ecreased card iac ou tp u t increased p erfu sion increased P CO 2 u neven ventilation d ecreased m inu te volu m e

525. The term MAC refers to (A) the m ed ian anesthetic concentration (B) the anesthetic concentration that p revents m ovem ent after skin incision in 50% of patients (C) a m easu rem ent that is not affected by age (D) a m easu rem ent that is p ertinent only to volatile anesthetics (E) the m ean alveolar concentration 526. Signs of inad equate general anesthesia includ e all of the follow ing EXCEPT (A) (B) (C) (D) (E)

eyelid m ovem ent p u p illary constriction hyp erventilation sw eating lim b m ovem ent

527. A patient has chronic obstru ctive pu lm onary d isease requ iring the constant ad m inistration of oxygen. H e is d yspneic at rest and can w alk

III IIIE IVE V VE

528. The p erson w ho p erform ed the first p u blic d em onstration of d iethyl ether as a general anesthetic w as (A) (B) (C) (D) (E)

Karl Koller William Thom as Green Morton H orace Wells Charles Jackson John Collins Warren

529. The m ost com m on p ostop erative visu al com p lication follow ing general anesthesia is (A) (B) (C) (D) (E)

retinal d etachm ent retinal artery occlu sion ischem ic op tic neu rop athy vitreou s hem orrhage corneal abrasion

530. The correlation of anesthetic p otency w ith lip id solu bility is know n as the ru le of (A) (B) (C) (D) (E)

Fergu son Michaelis and Menten H end erson and H asselbalch Singer and N icholson Meyer and Overton 133

134

8: Ge ne ra l Ane s the s ia

531. One m L of d esflu rane liqu id occu p ies w hat volu m e at 1 atm pressure and 37°C if all of the liqu id is vap orized ? The id eal gas constant is 0.082 L-atm -°K−1-m ole −1, the sp ecific gravity of d esflu rane is 1.45, and its m olecular w eight is 168. (A) (B) (C) (D) (E)

219 m L 238 m L 243 m L 256 m L 276 m L

532. Th e correct ord er of solu bilities in blood , from greatest to least, among the volatile anesthetics is (A) halothane > isoflu rane > sevoflu rane > d esflu rane (B) sevoflu rane > isoflu rane > d esflu rane > halothane (C) d esflu rane > isoflu rane > sevoflu rane > halothane (D) d esflu rane > halothane > sevoflu rane > isoflu rane (E) sevoflu rane > halothane > d esflu rane > isoflu rane 533. The state of general anesthesia may be reversed by (A) the ad m inistration of a com p etitive antagonist (B) increasing the atm osp heric p ressu re (C) increasing the am bient tem p eratu re (D) d ecreasing the am bient tem p eratu re (E) the ad m inistration of any m ed ication that increases cerebral perfusion 534. A 30-year-old p atient is to have a cholecystectomy. The anesthesiologist d ecid es to use sevoflu rane in oxygen as the sole anesthetic agent, w ith n o oth er m ed ication s ad m in istered . Ap p roxim ately w hat concentration of sevoflu rane w ill be requ ired to prevent hem od ynam ic changes in response to surgical incision? (A) 2.1% (B) 3.4%

(C) 4.6% (D) 5.5% (E) 6.4% 535. If 2% isoflu rane in oxygen, flow ing at a rate of 3 L/ m in, is ad d ed to a circle system , w hat w ill the concentration of isoflu rane be after 6 m in? Assu m e com plete m ixing of gas in the system , and that excess gas is scavenged . The reservoir bag has a volu m e of 2 L, the carbon d ioxid e absorber has a volu m e of 3 L, and the connecting hose and valves have a volu m e of 1 L. (A) (B) (C) (D) (E)

1% 1.26% 1.73% 1.90% 1.96%

536. If the u p take of gaseou s anesthetic in L/ m in is x, and the p atient’s card iac ou tp u t su d d enly d ou bles, the rate of u ptake (A) cannot be calcu lated w ithou t fu rther inform ation (B) w ill becom e x/ 2 (C) w ill becom e 2x (D) w ill becom e 4x (E) w ill becom e x2 537. The likelihood of intraop erative aw areness u nd er general anesthesia is highest w ith the u se of (A) (B) (C) (D) (E)

inad equ ate benzod iazep ine d oses high-d ose op ioid s m u scle relaxants no p rem ed ication nitrou s oxid e as the sole gaseou s anesthetic

538. Stage 2 anesthesia can be characterized by all of the follow ing signs EXCEPT (A) (B) (C) (D) (E)

am nesia p u rp oseless m ovem ent hyp oventilation d isconju gate gaze increased airw ay reflexes

Que s tions : 531–545

539. Contraind ications to the d ischarge to hom e of a p atient w ho had a hernia repair u nd er general anesthesia inclu d e all of the follow ing EXCEPT (A) nau sea (B) inability to d rink liqu id s w ithou t vom iting (C) heart rate 50% higher than the p reop erative value (D) inability to w alk d u e to groin p ain (E) d isorientation to p erson and p lace 540. You are given the honor of p rovid ing the first anesth etic in a n ew rad iology room . Th e p atient is a 38-year-old m an w ith an arteriovenou s m alform ation of the thoracic spine that is cau sing severe pain bu t no neurologic d eficit. The rad iologist p lans to em bolize the lesion, and estim ates that the proced u re w ill requ ire ten hours. Which one of the follow ing is the LEAST im p ortant requ irem ent for the room in w hich this p roced u re w ill occu r? (A) Pip eline oxygen su p p ly (B) Pip eline nitrou s oxid e su p p ly (C) Ad equ ate sp ace to p lace an anesthesia m achine in p roxim ity to the p atient (D) Availability of su ction (E) Au xiliary lighting available to the anesthesiologist 541. A 22-year-old p atient is brou ght to the operating room for repair of a fractured femur. H e fell off a boat and rem ained in the w ater for a long tim e p rior to rescu e. H e is hypotherm ic w ith a tem p eratu re of 33°C. Other vital signs and laboratory valu es are norm al. It can be assu m ed that the MAC for isoflurane in this patient is approxim ately (A) 1% (B) 1.25% (C) 1.5%

135

(D) 1.75% (E) 2% 542. Which one of the follow ing volatile agents u nd ergoes the greatest d egree of biotransform ation? (A) (B) (C) (D) (E)

halothane isoflu rane d esflu rane sevoflu rane nitrou s oxid e

543. All of the follow ing factors d eterm ine alveolar tension of an inhaled anesthetic EXCEPT (A) (B) (C) (D) (E)

m inu te ventilation blood :gas p artition coefficient card iac ou tp u t insp ired concentration bod y tem p eratu re

544. The second gas effect (A) has its m axim u m effect early in an anesthetic (B) ap p lies only to anesthetic gases (C) ap p lies only to nitrou s oxid e (D) in volves tw o gases ad m in istered at sim ilar concentrations (E) m ay be resp onsible for d iffu sion hyp oxia 545. At the anesthetic level associated w ith the alveolar concentration MAC-aw ake, patients (A) d o not resp ond to sim p le com m and s (B) w ill not m ove in resp onse to a su rgical incision (C) are likely to rem em ber w hat is told to them (D) m ay m anifest signs of excitem ent (E) w ill likely be ap neic

136

8: Ge ne ra l Ane s the s ia

546. Du ring a su rgical p roced u re to rep air a trau m atized liver in a patient w ho w as in a m otor vehicle accid ent, the p atient requ ired 100% O 2 in ord er to m aintain an ad equ ate valu e for oxygen satu ration, and each tim e a volatile an esthetic w as given, th e blood p ressu re d rop p ed to an u naccep table valu e. A m ed ication that m ight prevent the occurrence of recall for intraop erative events in the absence of nitrous oxid e and a volatile agent is (A) (B) (C) (D) (E)

m orp hine fentanyl scop olam ine d rop erid ol d exm ed etom id ine

547. Du ring general anesthesia w ith isoflu rane, nitrou s oxid e, and cisatracu riu m , exp ected ocu lar effects inclu d e (A) m ild ly increased intraocu lar p ressu re in norm al ind ivid u als (B) ablation of the ocu locard iac reflex (C) m yd riasis (D) ocu lar akinesia (E) angle closu re glau com a in su scep tible p atients 548. A p atient has had a total laryngectom y in the d istant past. The patient now presents for m astectom y and axillary nod e d issection for the m anagem ent of breast cancer. A reasonable m ethod of m an aging this p atient’s airw ay d u ring general anesthesia is inserting a(n) (A) low -p ressu re cu ffed end otracheal tu be via the m ou th (B) nasotracheal RAE tu be (C) oral airw ay to p revent obstru ction (D) laryngeal m ask airw ay (E) reinforced , cu ffed end otracheal tu be into the tracheostom y stom a 549. N asotracheal intubation m ay be u sed safely in a patient w ho has (A) fractu res of the low er cervical sp ine and the ethm oid bone (B) a LeFort I fractu re of the m axilla

(C) a LeFort II fractu re of the m axilla (D) a LeFort III fractu re of the m axilla (E) a CSF leak requ iring rep air 550. A 24-year-old w om an is to have d iagnostic lap aroscop y as an ou tp atient. H er m ed ical history is significant only for sym ptomatic gastroesop hageal reflu x. She is 61 inches tall and w eighs 185 p ou nd s. Prior to the ind u ction of general anesthesia, she shou ld be p rem ed icated w ith all of the follow ing m ed ications EXCEPT (A) (B) (C) (D) (E)

m etoclop ram id e d rop erid ol glycop yrrolate cim etid ine ond ansetron

551. Am ong all p atients u nd ergoing general anesthesia, the overall risk of aw areness is approxim ately one case in how m any anesthetics? (A) (B) (C) (D) (E)

75 325 700 2,200 6,300

552. Mod erate sed ation is associated w ith all of the follow ing characteristics EXCEPT the p atient (A) follow s a sim p le com m and like m aking a fist (B) m ay need a jaw thru st to relieve airw ay obstru ction (C) says ”ou ch” (or its equ ivalent) w hen pinched (D) has a norm al m inu te ventilation (E) has a norm al blood p ressu re 553. If nitrou s oxid e is ad m inistered at a constant concentration, the u ptake into the blood stream in m illiliters p er m inu te w ill (A) (B) (C) (D) (E)

be constant increase w ith tim e d ecrease w ith tim e d ep end on tem p eratu re be ind ep end ent of concentration

Que s tions : 546–558

554. An 85-kg p atient w as brou ght u rgently to the operating room after sustaining m u ltiple gu nshot w ou nd s to the abd om en. Exp loratory lap arotom y revealed m u ltip le inju ries to both large and sm all bow el that w ere rep aired by resecting the d am aged segm ents and p erforming anastom oses. There w ere no vascu lar inju ries and the p rolonged su rgical p roced u re w as associated w ith little blood loss. What is the m ost reasonable infu sion rate of lactated Ringer solu tion intraop eratively in ord er to m aintain ad equate u rine ou tp ut in the absence of significant blood loss? (A) (B) (C) (D) (E)

175 m L/ h 400 m L/ h 750 m L/ h 1,400 m L/ h 2,200 m L/ h

555. Intraarterial injection of w hich one of these m ed ication s m ay lead to lim b-threaten in g ischem ia d istal to the arterial catheter? (A) (B) (C) (D) (E)

sod iu m nitrop ru ssid e m ethohexital bu p ivacaine verap am il p ap averine

556. Su p erficial throm bop hlebitis m ay follow the intravenou s injection of m any m ed ications. Which one of the follow ing m ed ications has the LOWEST incid ence of throm bophlebitis? (A) (B) (C) (D) (E)

m id azolam lorazep am d iazep am etom id ate d antrolene

137

557. Which one of the follow ing m ed ications is thou ght to be the safest d u ring the first trim ester of pregnancy? (A) (B) (C) (D) (E)

Pred nisone Diazep am Warfarin Phenytoin Thalid om id e

558. A young, previou sly healthy m an suffered an extensive cru sh inju ry to his low er extrem ities at w ork. Before being brou ght to the operating room for d ebrid em ent of d evitalized tissu e, he w as given lactated Ringer solu tion in the em ergency d ep artm ent and w as consid ered eu volem ic by the anesthesiologist at the beginn in g of the case. As th e su rgical p roced u re p rogressed , the su rgeon requ ested that 5% albu m in solu tion be given instead of ad d itional lactated Ringer solution. Such use w ould be exp ected to cau se (A) an u nequ ivocal d ecrease in m ortality (B) an u nequ ivocal increase in m ortality (C) less of an im p airm ent of coagu lation than 10% d extran 40 solu tion (D) less of an im p airm ent of coagu lation than 10% pentastarch solu tion (E) a low incid ence of allergic reaction as long as the blood typ es are m atched

Answe rs a nd Expla na tions

524. (D) The patient w ith chronic obstructive lung disease has a prolonged induction due to ventilation/ perfusion mismatching. The cardiac output is usually not decreased. The increased PCO2 does not d irectly affect the u ptake of the agent. Decreased minute volume is not a factor. (5:604) 525. (B) The term MAC refers to m inim u m alveolar concentration. It is d efined as the alveolar anesthetic concentration su fficient to p revent m ovem ent in response to su rgical incision in 50% of the subjects. It d ecreases w ith increasing age. MAC can be u sed to sp ecify the p otency of both volatile and gaseous anesthetics. It shou ld be ap p arent to the read er that since MAC refers to a m id point in a popu lation, the w ord ”m inim um ” is a m isnom er and in fact ”m ean” or “m ed ian” w ou ld have been m ore accu rate. (5:598, 607) 526. (B) Pu p illary d ilatation is one of the signs of light anesthesia, as are tachyp nea, sw eating, and somatic movement. Lack of eye movement and p u p illary constriction are tw o d eterm inants of ad equ ate d ep th. (5:608) 527. (C) This patient w ou ld be classified as physical statu s IV becau se he has an incapacitating system ic illness. Becau se the patient is to u nd ergo an em ergency proced u re, “E” is ad d ed to the p hysical statu s. (5:54) 528. (B) William Thom as Green Morton d em onstrated the general anesthetic effect of d iethyl ether at Massachu setts General H osp ital on October 16, 1846. John Collins Warren w as the su rgeon. Charles Jackson w as Morton’s chem istry p rofessor at H arvard Med ical School and 138

tau ght him abou t the chem ical p rop erties of ether. H orace Wells d emonstrated the effects of nitrou s oxid e at MGH a year before Morton’s d em onstration; Wells’ d em onstration w as consid ered a failure becau se the patient cried out w hen his tooth w as extracted , althou gh he later said that he felt no p ain and rem em bered nothing. Karl Koller w as the first person to use cocaine as a local anesthetic in su rgery. (5:2) 529. (E) While all of the listed visu al com plications have been rep orted follow ing general anesthesia, corneal abrasion is by far the most common. (5:370) 530. (E) The Meyer-Overton ru le states that anesthetic potency is proportional to lipid solubility, and this ru le is valid for the m ajority of gaseou s anesthetics. Ferguson’s rule states that anesthetic p otency is p rop ortional to therm od ynam ic activity (id eal solu bility). Singer and N icholson p rop osed the lip id bilayer hyp othesis of m em brane stru ctu re. (5:600) 531. (A) One m L of d esflu rane liqu id is 1.45 g or 0.00863 mole (1.45 g/ 168 g/ mole). Thus, by the id eal gas law, V = nRT/ P = (0.00863) × (0.082) × (273 + 37) = 0.219 L = 219 m L. (5:627) 532. (A) The correct ord er of solu bilities in blood , from greatest to least, am ong the volatile anesthetics is halothane > isoflu rane > sevoflu rane > d esflu rane. (5:598) 533. (B) The state of general anesthesia, prod u ced by gaseou s agents or barbitu rates, m ay be reversed by increasing the atm ospheric p ressu re. (5:588)

Answe rs : 524–544

534. (C) MAC is d efined in term s of the p revention of m ovem ent in resp onse to skin incision in 50% of patients. It is therefore the valu e for the ED50 and is abou t 2.1% for sevoflu rane. A m ore reasonable target for clinical anesthesia is the ED95 and that valu e is abou t 1.3-fold higher than the ED50. Blu nting the au tonom ic resp onse to skin incision requ ires abou t 1.7fold higher a concentration than p reventing m ovem ent in resp onse to incision. Therefore, a reasonable target in this case is 2.1% × 1.3 × 1.7 = 4.6% sevoflu rane. (5:598, 607-8) 535. (D ) The tim e constant of the circu it is its total volum e (6 L) d ivid ed by the fresh gas flow rate (3 L/ m in), or 2 m in. The concentration at any p oint in tim e is given by the follow ing exponential equ ation: C = C o (1 − e -(t/ τ)) w here Co is the concentration in the fresh gas (2%), τ is the time constant (2 min), and t is the time in question (6 min). Thus, after one time constant has elapsed , the concentration in the system is 63.2% of the fresh gas concentration or 1.26% isoflurane; after two time constants, 86.5% or 1.73% isoflurane; after three time constants, 95% or 1.9% isoflurane; and after four time constants, 98.2% or 1.96% isoflurane. (5:601) 536. (C) Up take by the blood of a gaseou s anesthetic from the lu ng is p roportional to card iac ou tp u t. Thu s, if the card iac ou tp u t d ou bles, u ptake w ill d ou ble. (5:601-2) 537. (C) The likelihood of recall is not correlated w ith the use, or the lack of use, of any anesthetic agent. H ow ever, the u se of m u scle relaxants, w hich may block the observation of movement as a sign of inad equate anesthesia, is the pharmacological risk factor of greatest importance for intraoperative aw areness. (5:609-10) 538. (C) The second stage of anesthesia is characterized by excitement, somatic movement, increased airw ay reflexes, d isconju gate gaze, hyp ertension, hyperventilation, loss of consciou sness, and am nesia. (5:606)

139

539. (A) N au sea w ithout vom iting is very com m on after general anesthesia and as an isolated symptom is not a contraind ication to d ischarge to hom e. Patients should be oriented and their vital signs shou ld be near their preop erative values. Their pain shou ld be und er reasonable control and they shou ld be able to tolerate flu id s w ithou t vom iting. (5:1278-84) 540. (B) This p atient requ ires a general anesthetic because of his pain and because of the expected d u ration of the proced ure. General anesthesia w ou ld m ost likely be p rovid ed by an anesthesia machine located near the patient. If the flow of nitrou s oxid e is set at 2 L/ m in, a single fu ll cylind er w ou ld last for abou t 13 h. Conversely, the ventilator m ight requ ire 10-20 L/ m in, meaning that an oxygen cylind er might last for only 30-60 min; a supply of w all oxygen is generally requ ired w henever a ventilator is to be u sed . The anesthesiologist also m u st have the ability to su ction the p atient’s airw ay and to see the patient and the anesthesia equ ipm ent, consid ering that m u ch of the proposed proced u re w ill take place w ith the room d arkened . (A SA Statement on N onoperating Room A nesthetizing Locations, www.asahq.org/for-members/standards-guidelines-and-statements.aspx) 541. (A) The effect of altered bod y tem p erature on MAC is to d ecrease MAC by approximately 5% for each 1°C d ecrease from norm al bod y tem p eratu re. (5:598, 607) 542. (A) N itrou s oxid e is not m etabolized , and isoflu ran e an d d esflu ran e u n d ergo m in im al m etabolism . Sevoflu rane is abou t 5% m etabolized w hile halothane is abou t 20% m etabolized . (5:598) 543. (E) Minu te ventilation, insp ired concentration, blood :gas p artition coefficient, and card iac ou tp u t all affect the alveolar tension of the anesthetic gas. Bod y tem perature has no effect. (5:600-2) 544. (A) The second gas effect occu rs w hen the ad m inistration of a high concentration of one gas increases the rise in alveolar concentration of another gas. This effect m ay app ly to any

140

8: Ge ne ra l Ane s the s ia

gas. The m axim u m effect in anesthesia is early in the cou rse of the anesthetic, and one of the gases m u st be cap able of being given in high concentration. Diffusion hypoxia m ay occu r at the end of an anesthetic w hen the d iffu sion of large volu m es of nitrou s oxid e from the p u lm onary circu lation to the alveoli d ilu tes the alveolar concentration of oxygen. (5:605) 545. (D ) MAC-aw ake is the alveolar concentration of an inhalational anesthetic at w hich 50% of the patients respond to com m and s. This valu e m ay be applied to patients as general anesthesia is being ind u ced , or as they are em erging from anesthesia, and is sim ilar to the alveolar concentrations associated w ith the excitem ent stage. This concentration is low er than that w hich w ill p revent m ovem ent in resp onse to incision (or MAC) or resu lt in ap nea. Patients are u su ally am nestic for events that occu r at MAC-aw ake. (5:607-8) 546. (C) Scopolam ine prod u ces anterograd e am nesia in m ost patients w ithou t ad versely affecting blood pressure and cardiac output. Opioid s (like m orp hine and fentanyl), bu tyrop henones (like d roperidol), and α 2-ad renoceptor agonists (like d exm ed etom id ine) are not associated w ith am nesia. (5:1374) 547. (D ) Isoflu rane d ecreases intraocu lar pressu re and cisatracu riu m rend ers the eye im m obile. Deep general anesthesia d oes not p revent the ocu locard iac reflex. Med ications that cau se m yd riasis m ay precip itate an episod e of angle closu re glau com a in su scep tible p atients. (5:608, 1210, 1220) 548. (E) A patient w ho has had a total laryngectomy u sually has a perm anent tracheostom y stom a in the neck. Su ch p atients have no connection to the airw ay via the oral rou te, thus oral or nasal intubation or the p lacem ent of a laryngeal m ask airw ay is im p ossible. The stom a m ay be intu bated and a reinforced tu be is a p op u lar choice. An alternative is to su p p ly supplemental oxygen via a tracheostomy mask w h ile th e p atien t breath es sp on tan eou sly d u ring total intravenous anesthesia. (5:127)

549. (B) LeFort II and III fractu res, fractu res of the ethm oid bone, and a CSF leak all increase the risk of the end otracheal tu be p enetrating into the brain d uring nasotracheal intu bation. This risk is not p resent if the p atient has a LeFort I fractu re. (5:1243-4) 550. (C) Patients w ith sym p tom atic reflu x shou ld be p rem ed icated w ith a histam ine H 2 antagonist (su ch as cim etid ine) and a gastrointestinal p rokinetic agent (su ch as m etoclopram id e) in ord er to d ecrease the volu m e and increase the pH of the gastric contents. This patient also has several risk factors for p ostop erative nau sea and vom iting, includ ing fem ale gend er, young age, obesity, and an em etogenic operative p roced u re. She should receive antiem etic p rop hylaxis such as w ith d roperid ol and ond ansetron. She has no ind ication for preop erative glycop yrrolate. (5:78, 80, 1034) 551. (C) In a com bination of stu d ies inclu d ing over 30,000 anesthetics, the incid ence of aw areness w as fou nd to be abou t 0.13%. (5:608) 552. (B) Th e p atien t u n d er m od erate sed ation shou ld maintain an unobstructed airw ay w ithou t assistance. Card ioresp iratory p aram eters shou ld be norm al or near norm al, and the p atient shou ld resp ond ap p rop riately to p ain or a com m and . (5:1261) 553. (C) The u p take w ill d ecrease over tim e as equilibrium is reached . The uptake is d epend ent on concentration, being greater w ith a higher concentration. (5:600-3) 554. (C) This p roced u re w ou ld be consid ered a “m ajor” su rgical trau m a in term s of intraoperative flu id replacem ent need s, and therefore intraop erative flu id rep lacem ent in the range of 6-12 m L/ kg/ h is likely to be ap propriate for this p atient. (5:534) 555. (B) The accid ental intraarterial injection of a barbitu rate solu tion is likely to lead to intense vasospasm that m ay put the limb at risk of loss u nless there is tim ely treatm ent w ith a vasod ilator su ch as p ap averine, sod iu m nitrop ru ssid e, or verapam il, also injected intraarterially.

Answe rs : 545–558

The accid ental intraarterial injection of a local anesthetic into an artery above the d iap hragm m ay result in retrograd e flow of the local anesthetic and su bsequ ent d elivery to the brain by a carotid or vertebral artery resu lting in a seizu re. (5:694) 556. (A) Of the injectable benzod iazep ines, m id azolam has the low est incid ence of throm bop hlebitis because it is the only one that d oes not contain p rop ylene glycol in the vehicle. Etom id ate solu tion also contains p rop ylene glycol. Dantrolene solution is irritating because it has a high p H and is hyp erosm olar. (5:697, 699, 1500) 557. (B) Old er d ata su ggested that d iazep am m ight have teratogenic effects, bu t that is no longer

141

thought to be the case. The other m ed ications listed are likely or certain teratogen s in hum ans. (5:247, 298, 1150) 558. (C) The u se of albu m in instead of crystalloid solu tion in the general p atient p op u lation is associated w ith neith er an increase, nor a d ecrease, in m ortality, and som ew hat controversial in the traum a p op ulation, particu larly in those p atients w ith trau m atic brain inju ry. There is little effect on coagu lation w ith albu m in or w ith p entastarch, how ever d extran solu tions d o im p air coagu lation. Albu m in solu tions are not associated w ith a particu lar blood typ e. (5:539-41)

This page intentionally left blank

CHAPTER 9

Re g io nal Ane s the s ia Que s tions DIRECTION S (Qu estions 559-600): Each of the num bered item s or incom plete statem ents in this section is follow ed by answ ers or by com p letions of the statem ent. Select the ON E lettered answ er or com pletion that is BEST in each case. 559. As the su p ervising anesthesiologist you are assigned to fou r op erating room s staffed by CRNA’s. Regional anesthesia is preferred in all cases; however, you are concerned about the use of regional anesthesia in the setting of anticoagulation. Of the follow ing clinical scenarios, which patient is most likely to sustain a bleeding complication from regional anesthesia? (A) A 46-year-old m ale w ho takes a baby aspirin d aily and u nd ergoes a sp inal anesthetic for a knee arthroscopy. (B) An elderly female with osteoporosis undergoing a total knee arthroplasty under epidural analgesia and receiving low-molecular-weight heparin once daily. (C) A 56-year-old fem ale w ho d iscontinu ed clopid ogrel ten d ays prior to having an epid u ral placed for p ostop erative pain control follow ing a right hem icolectom y. (D) A 23-year-old healthy parturient w ho consu m es d aily garlic sup plem ents u nd ergoing an elective cesarean section. 560. As the chief of anesthesiology in you r hosp ital, you are asked by the ad m inistration to d iscu ss a p rocess that is im p ortant in p rom oting a safe regional anesthesia p ractice. Which one of the follow ing top ics is the m ost im p ortant one to d iscuss? (A) Patient care team ed u cation and aw areness of lim b p rotection and neu rologic evalu ation.

(B) Details regard ing billing com p liance. (C) Know led ge of w hen to change the bolu s rate of a continu ou s intrathecal catheter. (D) Training of m ed ical stu d ents and resid ents in app rop riate use of liposom al local anesthetics. 561. A 71-year-old m an w ith osteoarthritis of the right hip is u nd ergoing a right total hip arthrop lasty. You d ecid e to p erform a sp inal anesthetic. While assessing the land m arks w hen choosing the interspace in w hich to perform the lum bar puncture, you m u st rem em ber that the caud al term ination of the ad u lt spinal cord is typ ically at (A) (B) (C) (D) (E)

T11-T12 L1-L2 L4-L5 L5-S1 S2-S3

562. A 92-year-old m an is u nd ergoing total knee arthrop lasty for severe p ainfu l osteoarthritis. After stru ggling in the p lacem ent of a lu m bar spinal anesthetic, you d ecid e to u se the Taylor ap p roach. What best characterizes the Taylor ap p roach to p erform ing sp inal anesthesia? (A) Uses im age guid ance to facilitate bony land m arks (B) Uses a m id line and steep angled ap p roach (C) Uses m icrocatheters to prod u ce long-lasting blocks (D) Uses a loss of resistance technique w ith air (E) Uses a param ed ian ap p roach at L5-S1

143

144

9: Re giona l Ane s the s ia

563. You are d iscu ssing a labor ep id u ral analgesia w ith a 25-year-old p artu rient. She also happens to be a physician w ho is cu riou s on w hat is the m ost p op u lar techniqu e for confirm ing correct need le location w hen p erform ing an epid ural block. You respond that the most popu lar technique is (A) (B) (C) (D) (E)

loss of resistance to either saline or air u ltrasou nd im aging rad iograp hic confirm ation p atient resp onse block qu ality

564. What is the prim ary barrier to d ru g absorp tion from the ep id u ral sp ace into the CSF? (A) (B) (C) (D) (E)

Arachnoid m ater Du ra m ater Pia m ater Sp inou s p rocess Ligam entu m flavu m

565. A 68-year-old m an w ith rheum atoid arthritis u nd ergoes a left total knee arthroplasty. You prefer to u se an isobaric solu tion over a hyp erbaric or hyp obaric solu tion becau se (A) an isobaric solu tion w ill rise in the CSF w hen the p atient is in the su p ine p osition (B) a hyp erbaric solu tion w ill sink in the CSF (C) an isobaric solu tion w ill settle in the m ost d ep end ent region of the thoracic spine in the sup ine position (D) an isobaric solu tion w ill rise less than a hyperbaric solu tion and potentially lead to less hypotension (E) an isobaric solu tion of 0.5% bu pivacaine is ap p roved by the FDA for sp inal anesthesia 566. A 25-year-old football p layer p resents for a righ t kn ee arth roscop y after su stain in g a m ed ial m eniscu s tear. H e is frightened of general anesthesia and w ants to rem ain aw ake th rou gh ou t su rgery. In you r d iscu ssion of spinal anesthesia in the am bu latory setting, the top ic of transient neu rological sym p tom s

(TN S) arises. Which local anesthetic is associated w ith the highest risk of su staining TN S? (A) (B) (C) (D) (E)

Tetracaine Bu p ivacaine Lid ocaine Chlorop rocaine Rop ivacaine

567. A p atient receives a lu m bar ep id u ral block w ith 2% lid ocaine for a vaginal hysterectom y. Three hou rs after su rgery the p atient rep orts a d ense m otor block. An MRI d em onstrates a lu m bar ep id u r al h em atom a. For th e best n eu rologic ou tcom e, it is recom m end ed that a su rgical evacu ation of the hem atom a occu rs w ithin (A) (B) (C) (D) (E)

2-4 h 4-6 h 6-8 h 10-12 h 16-24 h

568. A 54-year-old w om an w ith rheu m atoid arthritis is sched u led for total knee rep lacem ent. The anesthesiologist p lans to u se 0.5% p lain bu p ivacaine to w hich 25 m cg of clonid ine has been ad d ed for sp inal anesthesia. Intrathecal clonid ine (A) w ill shorten the d u ration of the sp inal block (B) can p rolong the sp inal m otor block (C) is contraind icated in p regnant p atients (D) p rovid es a reliable su rgical block as a sole agent (E) has not been ap p roved by the FDA for ep id u ral u se 569. A 42-year-old m an p resents for ACL rep air to you r am bu latory su rgery center. You d ecid e to perform a spinal anesthetic becau se the patient asks to w atch the su rgery. You have a slow su rgeon and contem p late ad d ing 100 m icrogram s of epinephrine to you r local anesthetic. What is the m ajor d raw back to ad d ing ep inep hrine to a local anesthetic for spinal anesthesia in an am bu latory p atient?

Que s tions : 563–575

(A) (B) (C) (D) (E)

Ind u ces vom iting Does not ad d clinical benefit Delays gastric em p tying Delays retu rn of blad d er fu nction Cau ses itching

570. A 27-year-old healthy fem ale p resents for a triple arthrod esis of the ankle. She p refers to have a sp inal anesthetic, bu t recalls feeling short of breath d u ring a previou s spinal anesthetic for cesarean section. You tell her that (A) gross p u lm onary fu nction is m aintained w ith sp inal anesthesia in healthy ad u lts (B) sp inal anesthesia never com p rom ises pu lm onary fu nction (C) sp inal anesthesia resu lts in severe p u lm onary com prom ise and general anesthesia is her best choice (D) sp inal anesthesia rou tinely blocks the phrenic nerve and she p robably had a phrenic nerve p alsy d u ring her p revious cesarean section (E) sp inal anesthesia rarely im p airs p u lm onary function and is the anesthetic of choice in patients w ith severe cystic fibrosis 571. A 25-year-old partu rient p resents for an elective cesarean section. After performing a spinal anesthetic w ith 13.5 m g of hyperbaric bu p ivacaine, the patient reports “feeling sick to her stom ach.” Subsequently you notice the blood p ressu re is 62/ 30. The best d escrip tion of the card iovascu lar response to a spinal anesthetic is that (A) no hem od ynam ic changes occu r (B) both arterial and venou s vasod ilation contribute to hypotension (C) it exacerbates d iastolic d ysfu nction (D) hyp otension only occu rs in p atients w ith stenotic valvu lar lesions (E) it w orsens regu rgitant valvu lar lesions 572. A 33-year-old male u nd ergoes a su bgluteal sciatic nerve block for op en rep air of an ankle fractu re. In p rep aring for the block, the p rim ary d ru g therap y that the anesthesiologist

145

m u st consid er u sing for local anesthetic system ic toxicity is (A) (B) (C) (D) (E)

lid ocaine ep inep hrine breyteliu m p henylep hrine 20% lip id em u lsion

573. What w as the first d ru g u sed to p rod u ce local anesthesia? (A) (B) (C) (D) (E)

Lid ocaine Procaine Cocaine Prilocaine Tetracaine

574. A 23-year-old partu rient sched uled for an elective cesarean section und ergoes a spinal anesthetic w ith 10.5 m g of hyp erbaric bu p ivacaine. Ten m inu tes follow ing the sp inal anesthetic, she com plains of nau sea. H er vital signs are a BP of 75/ 52, pu lse rate of 123 beats/ m in, and oxygen satu ration of 98%. The best pharm acologic treatm ent for hyp otension related to the sp inal anesthetic is (A) (B) (C) (D) (E)

atrop ine ep inep hrine p henylep hrine calciu m labetalol

575. Tw enty-fou r hou rs after p erform ing a sp inal anesthetic in a 27-year-old fem ale for a vaginal hysterectom y, you are consu lted to evalu ate her for a p resu m ed post-d u ral p u nctu re head ache. Regard ing the sym p tom s of a p ost-d u ral p u n ctu re head ach e, you w ou ld exp ect the head ache to (A) be p ositional in natu re w ith im p rovem ent w hen sitting (B) be p ositional in natu re w ith im p rovem ent lying d ow n (C) not be p ositional in natu re (D) localized to one eye (E) be associated w ith d elayed gastric em p tying

146

9: Re giona l Ane s the s ia

576. A 72-year-old fem ale p resents for a right thoracotom y for a right low er lobe resection for lung cancer. She consents to a thoracic epid ural cath eter for p ostop erative an algesia. Sh e u nd erstand s the risks that m ay resu lt and asks abou t the p rognosis sh ou ld she d evelop an ep id u ral hem atom a. The statem ent that best ch aracterizes th e p rognosis of an ep id u ral hem atom a is (A) a sp inal ep id u ral hem atom a is u niversally fatal (B) a sp inal ep id u ral hem atom a alw ays resu lts in qu ad rip legia (C) a sp inal ep id u ral hem atom a alw ays resu lts in p arap legia or a chronic p ain synd rom e (D) a sp inal ep id u ral hem atom a is not d iagnosable and thus has a very poor p rognosis (E) w ith p rom p t d iagnosis, treatm ent strategies are effective 577. A 36-year-old otherw ise healthy m ale w ith Crohn d isease p resents for op en hem icolectomy. You place a thoracic epid ural catheter for postoperative analgesia. What w ou ld be consid ered a positive heart rate response from a test d ose of 15 m cg of ep inep hrine? (A) (B) (C) (D)

H eart rate variability changes Systolic blood p ressu re d rop s Skin vasoconstriction noted H eart rate increases by 20 beats p er m inute (E) H eart rate increases by 10 beats p er m inute 578. A 21-year-old m ale p resents for ACL reconstruction of the left knee. The patient prefers to have a fem oral nerve block bu t is w orried becau se he has been told that he has an allergy to “N ovacaine”. Which statem ent best characterizes allergic reaction s related to local anesthetics? (A) Allergies d o not occu r related to local anesthetics. (B) Reactions are u su ally lim ited to system ic hives and u rticaria.

(C) Allergic reactions are m ore com m on after exposu re to ester com p ou nd s than am id es. (D) Allergic reactions can be avoid ed by p re-treatm ent w ith d iphenhyd ram ine. (E) Allergic reactions can effectively be treated w ith intravenou s lipid s. 579. A 50-year-old fem ale is sched u led to u nd ergo op en rep air of a d istal righ t rad iu s fractu re. Sh e p refers to have a su p raclavicu lar nerve block bu t is concerned abou t nerve inju ry. Specifically, she is concerned abou t the risk of intraneu ral injection. Which statem ent best characterizes the risk of intraneu ral injection of local anesthetics? (A) Ultrasou nd im aging su ggests that it occu rs frequ ently w ithou t lead ing to clinical injury. (B) N erve inju ries d o not occu r by this m echanism . (C) Intraneu ral injection m u st only be avoid ed in d iabetics. (D) Intraneu ral injections d o not occu r w hen u ltrasou nd is u sed . (E) Intraneu ral injections are only a concern in p erip heral nerves and not in p lexi. 580. A 28-year-old fem ale is abou t to u nd ergo left hand su rgery and you d iscu ss p erform ing a su praclavicu lar nerve block. She appears concerned and asks about the incid ence of pneu m othorax. You resp ond that w ith trad itional land m ark techniqu es, the incid ence of a p neu m othorax w hen p erform ing a supraclavicular block has been rep orted to be in the range of (A) (B) (C) (D) (E)

0.1 - 0.3% 0.2 - 1% 0.5 - 5% 1 - 10% 5 - 30%

Que s tions : 576–587

581. Maintaining the stand ard s of care d u ring regional anesthesia p ractice involves all of the follow ing EXCEPT (A) offering regional anesthesia to all patients (B) inform ed consent (C) m onitoring regional anesthesia p ractice (D) ap p rop riate and tim ely p ostop erative follow u p (E) m onitoring vital signs 582. A 22-year-old parturient requests a labor epidural. During your evaluation of the patient she exp lains that she has an allergy to one local anesthetic but cannot remember w hich one. To w hich one of the follow ing local anesthetics is a patient most likely to have an allergic reaction? (A) (B) (C) (D) (E)

tetracaine bu p ivacaine lid ocaine rop ivacaine p rilocaine

583. The rate of local anesthetic absorp tion from a p articu lar injection site, from highest to low est rate, is (A) ep id u ral, intercostal, brachial p lexu s, subcutaneou s tissu e, low er extrem ity (B) ep id u ral, intercostal, su bcu taneou s tissue, low er extrem ity, brachial plexu s (C) brachial p lexu s, ep id u ral, intercostal, subcutaneou s tissu e, low er extrem ity (D) intercostal, ep id u ral, brachial p lexu s, low er extrem ity, su bcu taneou s tissu e (E) subcutaneous tissue, intercostal, epidural, brachial p lexu s, low er extrem ity 584. You are providing informed consent to a patient about to undergo a popliteal sciatic block. She is concerned about the risk of long-term neurologic injury from a peripheral nerve block. You exp lain that the incid ence of late neu rologic d eficits after peripheral nerve blocks is (A) 1/ 10,000 (B) 4/ 10,000 (C) 1/ 100,000

147

(D) 4/ 100,000 (E) 4/ 1,000,000 585. During the performance of a left femoral nerve block utilizing traditional land mark techniques and nerve stim u lation, you obtain a p atellar twitch at 2.5 mA current and proceed to decrease the current w hile still maintaining a p atellar twitch. The highest stimulating current that reliably predict intraneural needle placement is (A) (B) (C) (D) (E)

0.2 m A 0.5 m A 1.0 m A 1.5 m A 2.0 m A

586. You are su p ervising a first-year anesthesia resid ent d u ring her regional anesthesia rotation. You instru ct the resid ent to use blu nt-tip need les. Blu nt-tip need les (A) are m ore likely to p enetrate neu ral tissue (B) are less d isru p tive to neu ral tissu e than sharp need les (C) are m ore d isru p tive to neu ral tissu e than sharp need les (D) have a higher incid ence of su ccessfu l block (E) are only p referred w hen they are greater than 15 gau ge 587. A 72-year-old fem ale w ith painfu l osteoarthritis u nd ergoes a right total knee arthrop lasty. She receives an u ltrasou nd -gu id ed fem oral nerve block. The follow ing d ay, the p atient rep orts inability to d orsiflex the right foot. The m ost likely reason w ou ld be (A) a p ersistent m otor block to the right fem oral nerve (B) neu rologic inju ry to the right com m on p eroneal nerve (C) a p ersistent m otor block to the right saphenou s nerve (D) neu rologic inju ry to the right tibial nerve (E) neu rologic inju ry to the su p erficial p eroneal nerve

148

9: Re giona l Ane s the s ia

588. Tw enty m inu tes after the p erform ance of a lu m bar p lexu s block w ith 40 m L of 0.5% bu p ivacaine for a total hip arthroplasty, the patient rep orts “ringing in the ears” and feeling “light head ed .” The p atient’s cond ition d eteriorates and ECG m onitoring show s p rogression to sinus brad ycard ia, ventricu lar fibrillation, and u ltim ately asystole. The m ost likely explanation for these sym ptom s is (A) (B) (C) (D) (E)

d irect intravenou s injection oversed ation w ith m id azolam d irect intra-arterial injection p rofou nd vasod ilatation vascu lar absorp tion of local anesthetic

589. Utilizing a transarterial axillary techniqu e, a brachial p lexu s block is p erform ed . After confirm ing negative asp iration of blood , 5 m L of 0.5% bu p ivacaine is ad m inistered . Su d d enly the patient becomes u nresponsive. Initial pharm acologic m anagem ent shou ld inclu d e the ad m inistration of (A) (B) (C) (D) (E)

20% lipid em u lsion ep inep hrine, 1 m g am iod arone, 150 m g atrop ine, 1 m g vasop ressin, 40 u nits

590. A 54-year-old fem ale w ith ad enocarcinom a of the lu ng presents for right thoracotom y. You plan to place a thoracic ep id ural catheter for postoperative analgesia. In the process of p rovid ing inform ed consent, the patient becom es w orried abou t the risk of an ep id u ral hem atom a. You explain that the incid ence of an epid u ral hem atom a follow ing the ad m inistration of a neuraxial anesthetic is estim ated to be less than (A) (B) (C) (D) (E)

1 in 1 in 1 in 1 in 1 in

10,000 50,000 150,000 400,000 1,000,000

591. A 64-year-old fem ale p resents for a right total kn ee arth rop lasty. A sp in al an esth etic w as p erform ed in the L3-L4 intersp ace w ith 0.5%

isobaric bu p ivacaine. Five m inu tes after the sp inal anesthetic, the patient is insensate at the level of the u m bilicu s. This closely ap p roxim ates w hich one of the follow ing d erm atom e levels? (A) (B) (C) (D) (E)

T4 T7 T10 L1 L3

592. A 42-year-old previously healthy ASA I patient und ergoes an aw ake fiberoptic intubation w ith topical anesthesia for a laparoscopic append ectom y. Up on extu bation she w as noticed to have a satu ration of 93%. While in the PACU his oxygen satu ration continu ed to d rop to 85% d esp ite being alert, taking large tid al volu m es, having a norm al chest x-ray, and not im p roving w hile breathing 100% oxygen via a non-rebreather m ask. You su sp ect m ethem oglobinemia. Which one of the follow ing topical anesthetics is m ost associated w ith m ethem oglobinem ia? (A) (B) (C) (D) (E)

Cocaine Tetracaine Benzocaine Lid ocaine Prop aracaine

593. A 39-year-old female presents for carpal tunnel release su rgery. You d iscu ss p erform ing an IV regional block (Bier block). All of the follow ing are im p ortant to consid er EXCEPT (A) thorough exsanguination of the extremity shou ld take place p rior to the injection of local anesthetic (B) p reservative-free lid ocaine is the m ost frequ ently u sed local anesthetic (C) bu p ivacaine is the d ru g of choice becau se it p rovid es a long d u ration block (D) a d ou ble tou rniqu et p erm its longer su rgical tim e than a single tou rniqu et (E) a tou rniqu et is m ore reliable w hen placed p roxim al to the elbow than at a m ore d istal location

Que s tions : 588–598

594. A 51-year-old p atient is sched uled to u nd ergo a right lu ng low er lobectom y for lung cancer. You d iscu ss the risks and benefits of thoracic epid u ral analgesia. You d iscu ss that a comm on m ed ical ind ication for p erform ing regional anesthesia is (A) increased p rofessional billing (B) less card iac m orbid ity in the p reviou sly healthy p atient (C) red u ction in p ostop erative w ou nd infections (D) red u ction in the ad m inistration of op ioid s in the im m ed iate periop erative period . (E) less oxygen ad m inistration DIRECTION S: Use the follow ing scenario to answ er Qu estions 595-596: You have been follow ing a 27-year-old form er nu rse in you r chronic pain clinic for com p lex regional p ain synd rom e of the left hand . She initially su stained a w ork related inju ry to her hand a year ago and has since exp erienced ongoing chronic pain. After conservative treatm ent w ith neu rop athic m ed ications she has had little relief. You d ecid e to offer her a regional nerve block as p art of a m u ltim od al app roach to treating her pain. 595. Which one of the follow ing nerve blocks w ould be m ost ap p rop riate? (A) (B) (C) (D) (E)

Axillary brachial p lexu s block Stellate ganglion block Celiac p lexu s block A cervical transforam inal injection An infraclavicu lar brachial p lexu s block

596. After a su ccessfu l nerve block, one w ou ld expect to see all of the follow ing signs EXCEPT (A) m iosis (B) nasal congestion (C) anhid rosis

149

(D) a d ecrease in tem p eratu re of the blocked lim b by at least 1°C (E) p tosis DIRECTION S (Questions 597-598): Each grou p of item s below consists of lettered head ings follow ed by a list of nu m bered p hrases or statem ents. For each nu m bered p hrase or statem ent, select the ON E lettered head ing or com p onent that is m ost closely associated w ith it. Each lettered head ing or com pon en t m ay be selected on ce, m ore th an on ce, or not at all. (A) Glossop haryngeal nerve (B) Internal branch of the su p erior laryngeal nerve (C) External branch of the superior laryngeal nerve (D) Recu rrent laryngeal nerve (E) Trigem inal nerve (F) Anterior ethm oid al nerve (G) Greater p alatine nerve For each p atient, select the ap p rop riate airw ay block. 597. A 23-year-old obese fem ale rep orts having a kn ow n d ifficu lt airw ay. Sh e rem em bered having an aw ake intu bation the last tim e she p resen ted for su rgery an d an esth esia. Sp ecifically she states having had a strong “gag reflex” w hen the fiberop tic scop e m ad e contact w ith her soft p alate. 598. Du rin g you r p erform an ce of an aw ake fiberop tic intu bation, you are able to ad vance the fiberop tic scop e ju st above the ep iglottis. Ju st as you are abou t to ad vance the scop e throu gh the vocal cord s, it m akes contact w ith the posterior su rface of the ep iglottis and the vocal cord s snap shut.

150

9: Re giona l Ane s the s ia

DIRECTION S (Qu estions 599-600): Each grou p of item s below consists of lettered head ings follow ed by a list of nu m bered p hrases or statem ents. For each nu m bered phrase or statem ent, select the ON E lettered head ing or com p onent that is m ost closely associated w ith it. Each lettered head ing or com p on en t m ay be selected on ce, m ore than on ce, or n ot at all. (A) (B) (C) (D) (E) (F) (G)

C6 C7 T1 T4 T7 T10 L4

For each p atient, select the vertebra d escribed in the p hysical exam ination.

599. A 25-year-old fem ale requ ests a labor ep id u ral for analgesia. In assessing the ap p rop riate land m arks, you p alp ate the anterior su p erior iliac sp ines and d raw a line connecting both iliac crests. 600. A 25-year-old m an p resents for a right total shou ld er replacem ent. You consent the patient for continu ou s cervical p aravertebral blocks u tilizing nerve stim u lation. In assessing the ap p rop riate top ographical land m arks you p alpate the most prominent spinous process in the neck.

Answe rs a nd Expla na tions

559. (B) Certain su bsets of p atients have been id entified as high risk. This includ es the scenario d escribed in B. Su ch a patient has a 1 in 3600 risk of having an ep id u ral hem atom a in a tenyear Sw ed ish observational stu d y. (5:784)

565. (D ) Isobaric m eans a baricity of 1.0, thus the d rug w ill neither rise nor sink. (5:795) 566. (C) Lid ocaine is the biggest risk factor for TN S. In p articu lar, 5% lid ocaine has the highest risk. (5:796)

560. (A) Extrem ities that have d ecreased sensation need to be p rotected from p ositional related inju ries. Sp ecific ed u cation and instru ctions to protect the extremities follow ing regional anesthesia shou ld be p rovid ed to care p ersonnel and the p atient. (5:785)

567. (C) For the best neu rologic ou tcom e, d ecom p ression of an ep id u ral hem atom a m u st occur im m ed iately an d p referably w ith in 6-8 h. Prognosis is w orsened if d ecompression occurs after 8 h. (5:860)

561. (B) The ad u lt sp inal cord typ ically term inates at L1-L2. The infant sp inal cord term inates arou nd L3. (5:786)

568. (B) Intrathecal clonid ine can p rolong both the motor and sensory block associated w ith spinal anesthesia. Intrathecal clonid ine cannot act as the sole anesthetic. (5:798)

562. (E) The Taylor approach begins w ith id entification of the posterior su perior iliac spine (PSIS). The skin entrance site is 1 cm m ed ial and 1 cm caud ad to the PSIS. The need le is then d irected app roxim ately 45 d egrees m ed ial and cep halad to enter the L5-S1 intersp ace. (5:792) 563. (A) When the need le enters the ep id u ral space, continu ou s or interm ittent gentle p ressu re p laced onto a syringe containing saline or air w ill result in the ability to inject into the epid u ral sp ace. This is know n as loss of resistance. (5:793) 564. (A) Local anesthetics and op ioid s m u st p enetrate through the arachnoid m ater to get to the CSF. The sp ecifics of u p take of a d ru g by the CSF d ep end also on d ru g characteristics su ch as lip id solubility. (5:794)

569. (D ) Intrathecal epinephrine can lead to urinary retention. Itching is thou ght to be a sid e effect of intrathecal opioid s. (5:799) 570. (A) Althou gh accessory m u scles m ay be im p acted by sp inal anesthesia, as long as the p hrenic nerve is not blocked , gross p ulm onary fu nction shou ld be m aintained . (5:800) 571. (B) Since both arterial and venous vasod ilation occu r, the recom m end ed treatm ents for the hyp otension accom p anying spinal anesthesia are the ad m inistration of intravenou s flu id s and α 1-ad renocep tor agonists. (5:801) 572. (E) Lip id em u lsion therap y has been d ocu m ented to reverse local anesthetic system ic toxicity in both hu m ans and anim als. It is consid ered the treatm ent of choice and shou ld be available w herever regional anesthesia is p erform ed . (5:780) 151

152

9: Re giona l Ane s the s ia

573. (C) It has lim ited u se in m od ern anesthesia practice becau se of its relatively high p otential for system ic toxicity and ad d iction liabilities. Cocaine is an effective topical anesthetic agent, and it prod u ces vasoconstriction at clinically u sefu l concentrations. It is often u sed to anesthetize and constrict the nasal m u cosa before nasotracheal intu bation. It is the only local anesthetic that inhibits the reu p take of catecholam ines in the central and p eripheral nervou s system s. (5:781) 574. (C) Most of the hypotension in a eu volemic patient is related to arterial and venous dilatation. Thus, administration of an α 1-adrenoceptor agonist like phenylephrine is the treatment of choice. Secondary treatment would be intravenous fluid administration. Atropine and/ or epinephrine w ou ld be used in a situ ation of profound bradycardia. (5:802) 575. (B) The classic post-d u ral pu ncture head ache (PDPH ) is p osition-related in that recu m bency helps to alleviate sym ptom s. Comm on sym p tom s are severe bi-frontal throbbing, nausea, vom iting, and m alaise. Treatm ents inclu d e flu id , caffein e, th eop h yllin e, an d ep id u ral blood p atch. (5:802) 576. (E) Although seriou s, prompt d iagnosis w ith im aging stud ies and treatm ent w ith surgical decompression can result in full recovery. (5:803) 577. (D ) A positive response shou ld be conclu d ed if any of the follow ing occu r: heart rate increase > 20 beats/ m in , systolic blood p ressu re increase > 15 m m H g, T w ave am p litu d e d ecrease by ≥ 25% on ECG. (5:805) 578. (C) The fu ll sp ectru m of reactions from itching to anaphylaxis has been reported. Allergies are m ore com m on w ith ester com p ou nd s as com pared to am id es. (5:786) 579. (A) Ultrasound im aging has p rovid ed new inform ation on the reality that conventional approaches to nerve localization often result in what appears to be intraneural injections. There is current debate as to what constitutes dangerous, versus safe, intraneural injections. (5:855)

580. (C) Trad itionally the su p raclavicu lar ap p roach had been avoid ed d u e to a high incid ence of p neu m othorax. This high incid ence of p neu m othorax led to the p op u larity of the axillary approach. With ultrasound guid ance, there has been renew ed interest in the su p raclavicu lar ap p roach. (5:833) 581. (A) Practitioners shou ld select ap p rop riate p atients for regional anesthesia consid ering absolu te as w ell as relative contraind ications. (5:848) 582. (A) Allergic reactions are m ore com m on after exposure to ester com pou nd s than am id e com p ou nd s. Tetracaine is the only ester com p ou nd listed . (5:852) 583. (D ) A nu m ber of factors influ ence the absorp tion rate of local anesthetics from tissu e. The m ost im p ortant factor is the site of injection. Absorp tion is m ore rap id in highly vascu lar tissu e and less so in p oorly p erfu sed tissu e. (5:852) 584. (B) A recent analysis of 7,000 p erip heral nerve blocks rep orted an incid ence of 0.04% of late neurologic d eficits. (5:855) 585. (A) A recent stu d y fou nd that a stim u lating cu rrent w ith 0.2 m A or less reliably p red icts in tran eu ral n eed le p lacem en t. H ow ever a threshold current greater than 0.2 m A d oes not reliably p reclu d e intraneu ral need le p lacem ent. (5:855) 586. (C) Blu nt tip need les are less likely to penetrate neu ral tissu e and therefore are the p referred need le typ e. H ow ever, if they d o p enetrate neural tissu e they are likely to be m ore d isru ptive to the neural tissu e. (5:855) 587. (B) The motor innervation of the common peroneal is via the d eep peroneal nerve that allow s for dorsiflexion of the foot. Given its superficial nature, this nerve may be injured in the perioperative p eriod after a total knee arthroplasty. The superficial peroneal nerve is a branch of the common peroneal nerve; how ever, it d oes not provid e motor innervation. (4:71-2)

Answe rs : 573–600

588. (E) The lu m bar p lexu s block is associated w ith an increased risk of local anesthetic toxicity d u e to the high vascu larity of the area and large volu m es of local anesthetic requ ired for the d esired sp read . This increases the vascu lar absorption of local anesthetic from the psoas m uscle. With a d irect intravascu lar injection, one w ou ld exp ect the m anifestation of toxicity to be im m ed iate up on injection. (5:840) 589. (A) With the ad m inistration of intravenou s lip id em u lsion, recent stu d ies d em onstrate im p roved hem od ynam ics and a su rvival benefit in anim al m od els of bu pivacaine toxicity. It has been suggested that lipid em u lsion acts as a lip id sin k bind in g local an esth etics an d rem oving them from card iac bind ing sites. Tw enty p ercent lip id em u lsion shou ld be ad m inistered early as a bolu s of 1.5 m L/ kg over one m inu te and follow ed 0.25 m L/ kg/ m in. (5:853-4) 590. (C) The true incid ence is u nknow n bu t an epid u ral hem atom a after neu raxial blockad e is rare and estim ated to occu r in few er than 1 in 150,000 cases. (5:859) 591. (C) The u m bilicu s closely ap p roxim ates the T10 d erm atom e level. The nipple line closely app roxim ates the T4 level. (4:237) 592. (C) Benzocaine is know n to cau se m ethem oglobinem ia. The anesthesiologist m u st be cau tious and vigilant to such an occurrence. (5:970) 593. (C) Bupivacaine is not recommend ed given the p otential for card iac toxicity. The d u ration of a Bier block is lim ited by tou rniqu et p ain. When u sing a d ouble tou rniqu et, the proxim al tourniqu et is inflated first. When the p atient com p lains of tou rniquet p ain, the d istal tou rniquet (that is w rap p ed arou nd an anesthetized area) is inflated follow ed by the proximal tourniquet being d eflated . (5:866) 594. (D ) Regional anesthesia is know n to d ecrease the need for op ioid s and therefore d ecrease opioid related m orbid ity. (5:785)

153

595. (B) Com p lex regional p ain synd rom e (CRPS) is sym p athetically-m ed iated p ain. A sym p athetic block of the u p p er extrem ity can be obtained by p erform ing a stellate ganglion block. The stellate ganglion is also know n as the cervicothoracic ganglion becau se it is a fu sion of the seventh cervical and first thoracic ganglion. (5:1579-81) 596. (D ) A sym p athetic block w ou ld resu lt in an increase in tem p eratu re to the block lim b by at least 1°C. This is a resu lt of the vasod ilation that occu rs to the extrem ity. H orner synd rom e as w ell as nasal congestion w ou ld also occu r. (5:1579-81) 597. (A) The glossopharyngeal nerve innervates the orop harynx, soft p alate, p osterior p ortion of the tongu e, and the p haryngeal surface of the ep iglottis. The p haryngeal nerve can be anesthetized by inh alin g n ebu lized lid ocain e, application of topical anesthesia, or d irect glossopharyngeal nerve blocks. (4:335) 598. (B) The internal branch of the su perior laryngeal nerve is a branch of the vagu s nerve and provid es sensory innervation to the base of the tongu e, p osterior surface of the epiglottis, aryep iglottic fold , and the arytenoid s. The external branch of the su p erior laryngeal nerve provid es m otor innervation to the cricothyroid m u scle. Besid es topical ad m inistration, bilateral su p erior laryngeal nerve block can be p erform ed by injecting local anesthetic at the level of the greater cornu of the hyoid bone. (4:336) 599. (G) The intercristal line reflects the sp inou s p rocess of L4. (5:810) 600. (B) C7 is the m ost p rom inent and therefore easily p alp ated sp inou s p rocess in the neck. (5:809)

This page intentionally left blank

CHAPTER 10

Prac tic e Te s t Que s tions DIRECTION S (Qu estions 1-150): Each of the nu m bered item s or incom p lete statem ents in this section is follow ed by answ ers or by com p letions of the statem ent. Select the ON E lettered answ er or com p letion that is BEST in each case. 1. The interaction of rocu roniu m and acetylcholine at the m yoneu ral ju nction is one of (A) (B) (C) (D) (E)

synergism com p etition for bind ing sites chem ical com bination alteration of m etabolism altered p rotein bind ing

2. As one m oves from the ap ex of the lu ng to the d epend ent portions (A) the alveoli becom e larger (B) the caliber of the air p assages becom es larger (C) p leu ral p ressu re d ecreases (D) com p liance becom es greater (E) ventilation of the alveoli becom es less d u e to d ecreased com pliance 3. A 70-year-old w om an w ith lon g-stan d in g typ e 2 d iabetes m ellitu s p resents for a p reop erative assessm ent p rior to fem oral-p op liteal byp ass su rgery. H er only m ed ication is exenatid e. H avin g never heard of th is d ru g, the

anesthesia resid ent looks it u p and find s that it is (A) a glu cagon-like p ep tid e-1 recep tor agonist (B) FDA app roved for treatm ent of type 1 and typ e 2 d iabetes m ellitu s (C) a m em ber of the bigu anid e class of oral hypoglycem ic agents like m etform in (D) a thiazolid ined ione like rosiglitazone (E) also u sed for p revention of heart d isease in typ e 2 d iabetics 4. The m ost im p ortant site of d ru g transform ation is usu ally the (A) (B) (C) (D) (E)

liver sp leen kid ney lu ngs blood stream

5. Esm olol is chosen for a tachycard ic p atient w ith liver cirrhosis. The 9-m inu te half-life of the d ru g stem s from (A) (B) (C) (D) (E)

p seu d ocholinesterase skeletal m u scle esterase red blood cell esterases p hosp hod iesterase alkaline p hosp hatase

155

156

10: P ra ctice Te s t

6. A 49-year-old m an w ith Ad d ison d isease presents for a herniorrhap hy. All of the follow ing statem ents abou t Ad d ison d isease are tru e EXCEPT (A) p atient’s w ith this cond ition u nd ergoing even m inor p roced u res requ ire stress d ose of steroid s intraop eratively and a long p ostop erative tap er (B) typ ical laboratory abnorm alities inclu d e hyponatrem ia and hyp erkalem ia (C) Ad d ison d isease is a typ e of p rim ary ad renal insufficiency cau sed by ad renal gland d estru ction (D) both skin hyp erp igm entation and vitiligo are signs of Ad d ison d isease (E) com m on sym p tom s in u ntreated Ad d ison d isease are d iarrhea and orthostatic hypotension 7. Bioavailability of a d ru g refers to the am ou nt of d rug that (A) (B) (C) (D) (E)

is ad m inistered intram u scu larly is ad m inistered orally reaches the liver is excreted by the kid ney reaches its site of action

8. A 21-year-old w om an w ith a long history of anorexia nervosa typ ically h as all of th e follow ing EXCEPT (A) osteop orosis (B) increased risk of intraop erative d ysrhythm ias (C) d elayed gastric em p tying (D) hyp otension (E) resting tachycard ia 9. In cases of u nequ ivocal intraop erative aw areness, the incid ence of p osttrau m atic stress d isord er m ay be app roxim ately (A) (B) (C) (D) (E)

0.1% 1% 10% 50% 99%

10. A 40-year-old w oman presents w ith a corrected QT interval (QTc) p rolonged to 500 m sec. She is (A) at increased risk for p eriop erative m orbid ity and m ortality related to card iac arrhythm ias (B) recom m end ed to receive am iod arone (C) likely to be hyp ocalcem ic (D) at risk for Wolf-Parkinson-White synd rom e (E) at risk to requ ire intraop erative p acem aker therapy 11. Renal clearance of a d rug (A) is u su ally of little im p ortance (B) has no relationship to creatinine clearance (C) is constant for a given d ru g (D) varies w ith p H , u rine flow rate, and renal blood flow (E) m ay exceed renal blood flow 12. A 50-year-old m an p lan ned for sh ou ld er su rgery is ju d ged to be in N ew York H eart Association fu nctional class III. H e has (A) (B) (C) (D)

m arked lim itation of p hysical activity sym p tom s at rest slight lim itation of p hysical activity inability to com fortably carry ou t any physical activity (E) no sym p tom s d u ring ord inary activity 13. All of the follow ing are tru e of closing volu m e EXCEPT that it is (A) m easu red by a single-breath nitrogen techniqu e (B) u sefu l in d eterm ining d isease of sm all airw ays (C) d ecreased at the extrem es of age (D) u nchanged in obesity (E) m easu red at p hase IV on the nitrogen w ashou t cu rve

Que s tions : 6–21

14. A patient has been given an injection of ketam ine in a d ose calcu lated to be su fficient for anesthesia. H is eyes rem ain op en, and there is slight nystagm u s and occasional pu rposeless m ovem ents. This is an ind ication that (A) the d ose is inad equ ate (B) m ore ketam ine shou ld be given to stop the m ovem ents (C) the d ose is excessive (D) the d ose is ad equ ate for anesthesia (E) the p atient is having a seizu re 15. Und er norm al p hysiologic cond itions cerebral blood flow is (A) (B) (C) (D) (E)

1 m L/ 100 g/ m in 10 m L/ 100 g/ m in 25 m L/ 100 g/ m in 50 m L/ 100 g/ m in 100 m L/ 100 g/ m in

16. A 50-year-old m an has a bicu spid aortic valve. Su rgical rep air of his ascend ing aorta is ind icated at a d iam eter of (A) (B) (C) (D) (E)

4 cm 4.5 cm 5 cm 5.5 cm 6 cm

17. The p erip heral chem orecep tors are (A) located in the m ed u lla oblongata (B) p oorly p erfu sed and , therefore, resp ond slow ly to changes in the oxygen content of the blood (C) resp onsible for the hyp oxic d rive to resp iration (D) influ enced by oxygen content rather than oxygen tension (E) not affected by increasing age 18. Which one of the follow ing agents shou ld be avoid ed in a p atient w ith hep arin-ind u ced throm bocytop enia type II?

(A) (B) (C) (D) (E)

157

d anaparoid lepirud in argatroban w arfarin benzod iazepines

19. Possible w ays to assu re proper placem ent of th e end otrach eal tu be in clu d e all of th e follow ing, EXCEPT (A) (B) (C) (D)

chest rad iograp h bronchoscop ic exam ination au scu ltation of the thorax continu ou s end -tid al carbon d ioxid e cap nography (E) fogging w ithin the end otracheal tu be 20. H yp oxic p u lm onary vasoconstriction (A) is not im p ortant in the intact hu m an being (B) is active only at high altitu d e (C) cau ses m ore blood flow to the base of the lu ng (D) cau ses higher d ead sp ace/ tid al volu m e ratio (VD / VT) than in the nonhyp oxic lu ng (E) d iverts blood flow from hyp oxic to nonhyp oxic lu ng areas 21. A 9-year-old boy su stained a 40% bu rn to the anterior p ortion of his bod y. On the tw entieth d ay after the bu rn, he is brou ght to the op erating room for a skin graft. Anesthesia is ind uced w ith thiopental, and succinylcholine is injected for relaxation. The ECG show s peaked T w aves follow ed by asystole. The m ost likely cau se of the arrhythm ia is (A) (B) (C) (D) (E)

an overd ose of thiop ental hyp oxia hyp erkalem ia electrocu tion ad m inistration of the w rong d ru g

158

10: P ra ctice Te s t

22. An anesthesia circu it is connected to a circle system and the com bined volu m e of both is 5 L. The fresh gas flow is 1 L/ m in. After d esflu rane 10% is tu rned on, how long w ill it take the concentration in the circu it to reach 8.5% d esflu rane? (A) (B) (C) (D) (E)

1 m in 5 m in 8.5 m in 10 m in 17 m in

23. A 20-year-old p atient w ith hem op hilia B (Christm as d isease) is p lanned for ap p end ectom y. The coagu lation d eficit is that of clotting factor (A) (B) (C) (D) (E)

II VII VIII IX XII

24. Prop ofol (A) (B) (C) (D) (E)

increases cerebral blood flow is very solu ble in aqu eou s solu tions cau ses catecholam ine release has little analgesic activity has a longer sleep tim e than thiop ental

25. A 50-year-old w om an w ith m etabolic synd rom e p resents for p reop erative assessm ent p rior to elective coronary artery byp ass su rgery. Tru e statem en ts abou t th e m etabolic synd rome inclu d e all of the follow ing EXCEPT (A) p atients w ith the m etabolic synd rom e are at increased risk for d evelop ing overt d iabetes (B) to m ake the d iagnosis, a p atient m u st have 3 of the follow ing 5 criteria: elevated w aist circu m ference, elevated triglycerid es, red u ced high-d ensity lip op rotein cholesterol, elevated blood pressu re, and elevated fasting blood su gar (C) p atients w ith the m etabolic synd rom e are at increased risk of cerebrovascu lar d isease

(D) p atients w ith the m etabolic synd rom e shou ld be treated prophylactically w ith m etform in (E) w om en w ith p olycystic ovarian synd rom e are at increased risk for d evelop ing m etabolic synd rom e 26. A d iscrepancy is noted in a patient bearing tw o arterial catheters. N orm ally the systolic blood pressu re is highest in the (A) (B) (C) (D) (E)

ascend ing aorta d escend ing aorta fem oral artery d orsalis p ed is artery p u lm onary artery

27. Recogn ition of hyp oxem ia in the recovery room (A) d ep end s on the d etection of cyanosis (B) d ep end s on the d etection of ap nea (C) d ep end s on the d etection of circu latory resp onses (D) is best accomplished with pulse oximetry (E) is d on e better w ith a tr an scu tan eou s oxygen m on itor th an w ith a p u lse oxim eter 28. A 36-year-old m ale w ho recently su ffered multip le trau m a inclu d ing therm al inju ries is to u nd ergo general end otracheal anesthesia for elective orthop ed ic su rgery. Which one of the follow ing conditions w ould most likely explain resistance to non-d ep olarizing neu rom u scu lar blockers d u ring the op erative p roced u re? (A) (B) (C) (D) (E)

resp iratory acid osis recent large bod y su rface area bu rn ad m inistration of a volatile anesthetic hyp otherm ia hyp erm agnesem ia

29. The H ering–Breu er reflex (A) is accentu ated in p re-term infants w hen com pared to fu ll-term infants (B) cau ses a d eep insp iration after a total exp iration in the laboratory anim al

159

Que s tions : 22–36

(C) cau ses a d eep insp iration after a cou gh (D) has a m ajor role in the control of ventilation (E) is stim u lated by barbitu rates 30. A 42-year-old African Am erican w om an w ith a mu ltinod ular goiter is d iagnosed w ith hyperthyroid ism . Clinical m anifestations of hyp erth yroid ism in clu d e all of th e follow in g EXCEPT (A) (B) (C) (D) (E)

sw eating fatigu e tachycard ia shivering hyp erm otility and d iarrhea

31. A find ing that may be u sed to d istinguish d eep sed ation from general anesthesia is (A) (B) (C) (D) (E)

ability to follow a sim p le com m and ad equ ate m inu te ventilation resp onse to a p ainfu l p inch u nobstru cted airw ay BIS valu e

DIRECTION S: Use the follow ing figu re to answ er Qu estions 32-33:

33. At the tim e p oint (3), a tetanic stim ulation w as ap p lied w ith the d ep icted resu lt. Follow ing this, one could state that (A) the d ru g is not a m u scle relaxant (B) the d ru g is a d ep olarizing m u scle relaxant (C) the d ru g is a nond ep olarizing m u scle relaxant (D) either the d ru g is a nond ep olarizing m uscle relaxant, or a d u al block is p resent (E) the tracing show n is d u e to artifact 34. A very fit am ateu r m ou ntaineer w ishes to clim b w ithout the u se of supplem ental oxygen or m ed ications. H e also w ishes to avoid letting h is arterial oxygen con cen tration d ecrease below 45 m m H g. Using the table below, calcu late the m axim u m height to w hich the m ou ntaineer shou ld ascend . Altitude (ft) Patm (mm Hg )

(A) (B) (C) (D) (E)

7,000 595

10,000 12,000 15,000 18,000 534

496

443

395

7,000 ft 10,000 ft 12,000 ft 15,000 ft 18,000 ft

35. The p rim ary d ifference betw een cisatracu riu m and atracu riu m is that cisatracu riu m is

1

2

3

32. The figu re show s the evoked tw itch resp onse follow ing an u nknow n d ru g. The segm ent (1) is the baseline before d ru g ad m inistration, and (2) is the resp onse three m inu tes after d ru g ad m inistration. From these tw o segm ents, you can say that the d rug (A) (B) (C) (D) (E)

is not a m u scle relaxant is a d ep olarizing m u scle relaxant is a nond ep olarizing m u scle relaxant cau sed com p lete p aralysis cau sed 90% d epression

(A) (B) (C) (D) (E)

a d ifferent isom er than atracu riu m op tically active su bstantially shorter-acting less likely to resu lt in histam ine release m etabolized to a greater d egree by nonspecific p lasm a esterases

36. The second heart sou nd coincid es w ith all of the follow ing EXCEPT (A) (B) (C) (D) (E)

closing of the aortic valve isom etric relaxation of m yocard ial fibers closu re of the m itral valve the T w ave of the electrocard iogram closu re of the p u lm onic valve

160

10: P ra ctice Te s t

37. In a carbon d ioxid e response curve, the m inute ventilation is plotted against variou s concentrations of carbon d ioxid e. Which one of the follow ing statem ents is TRUE abou t a carbon d ioxid e resp onse cu rve? (A) The p atient u nd er anesthesia w ill show the sam e effects regard less of the agent. (B) The slop e of the resp onse m ay change, bu t the p osition of the cu rve rem ains the sam e. (C) The p osition and slop e are the sam e in ad u lts and infants. (D) Increased w ork of breathing lead s to a steeper slope. (E) The slop e of the line m easu res the p atient’s sensitivity to carbon d ioxid e. 38. The blood su p p ly to the m otor tracts of the spinal cord is prim arily from the (A) p osterior sp inal artery (B) anterior sp inal artery (C) p enetrating branches of the rad icu lar arteries (D) artery of Ad am kiew icz (E) basilar artery 39. The inability to su stain contraction (fad e) d u ring tetanic contraction is d ue to (A) inability to transm it rap id electrical im p u lses (B) cellu lar loss of p otassiu m (C) d ep letion of cellu lar DN A (D) inability of record ing ap p aratu s to show all contractions (E) the inability of the end p late to release sufficient acetylcholine 40. A 39-year-old w om an being evalu ated for cataract su rgery has Von Willebrand d isease. This d isease (A) com m only first m anifests as abnorm al bleed ing after tooth extraction or tonsillectom y in child ren (B) u su ally affects only fem ales (C) is u su ally associated w ith a low p latelet cou nt

(D) is cau sed by factor VIII d eficiency (E) is one of the rarest of the congenital bleed ing d isord ers 41. Calciu m chlorid e is ad m inistered to a p atient receiv in g citrated p lasm a tr an sfu sion s. Calciu m ion (A) (B) (C) (D) (E)

d ecreases m yocard ial contractile force d ecreases d u ration of systole d ecreases vascu lar tone d ecreases ventricu lar au tom aticity enters the card iac cell, cau sing excitation

42. Prolonged su rgery on the spine in the prone position is occasionally follow ed by postoperative visual loss. The m ost com m on etiology of su ch visu al loss is (A) (B) (C) (D) (E)

central retinal artery occlu sion central retinal vein occlu sion cortical blind ness anterior ischem ic op tic neu rop athy p osterior ischem ic op tic neu rop athy

43. Transd erm al scopolam ine (A) is effective in the p revention of PON V if ad m inistered p reop eratively (B) is effective in the treatm ent of PON V w hen ad m inistered p ostop eratively (C) p atch shou ld be ap p lied to the d eltoid m u scle (D) is d evoid of any sid e effects (E) in a m u ltilayered ad hesive u nit has a d u ration of action of abou t 16 h 44. A p ed iatric p atient w eighing 14 kg is being ven tilated w ith a trad ition al an esth esia m achine ventilator u sing the follow ing set param eters: Tid al volu m e = 50 m L Ventilatory rate = 20/ m in I:E ratio = 1:2 Oxygen flow = 2.5 L/ m in N itrou s oxid e flow = 3.5 L/ m in

Que s tions : 37–50

The resu lting m inu te ventilation is ap proxim ately (A) (B) (C) (D) (E)

1 L/ m in 1.5 L/ m in 2 L/ m in 2.5 L/ m in 3 L/ m in

45. Com p ared to the you ng ad u lt, the eld erly p atient (A) has an u nchanged venou s ad m ixtu re d u ring general anesthesia (B) has a d ecrease in tru e p u lm onary shu nting d u ring general anesthesia (C) has an increase in total lu ng cap acity (D) has an increase in closing cap acity (E) has a d ecrease in closing cap acity 46. If su ccinylcholine is given to facilitate intu bation, and the patient is allow ed to recover fully from the d ep olarizing block, and then is given p ancu roniu m , one w ou ld expect (A) a low er than norm al am ou nt of pancuroniu m is need ed to reestablish neurom uscu lar blockad e (B) the p ancu roniu m to have a shorter d u ration of action as a resu lt of the su ccinylcholine (C) no change in the d u ration of action of pancuroniu m (D) no change in the d u ration of action of pancuroniu m bu t less intensity of relaxation (E) the d evelop m ent of a p hase II block 47. A fourth-year m ed ical stu d ent is on her anesthesia rotation. You are d iscu ssing local anesthetics for spinal anesthesia w ith her. The topic of baricity arises on w hich you exp lain that baricity is the (A) m illigram d ose of an equ ip otent d ru g (B) m inim u m concentration need ed to reach clinical effect (C) half-life of elim ination

161

(D) context sensitive half-tim e (E) ratio of the specific gravity of a d ru g to a reference specific gravity 48. When d esflu rane, bu t not sevoflu rane, in a carrier gas of 100% oxygen is p assed throu gh very d ry carbon d ioxid e absorbent, the p atient m ay be exp osed to a toxic concentration of (A) (B) (C) (D) (E)

ozone p hosgene carbon d ioxid e carbon m onoxid e flu orid e

49. Technical d ifficu lties in treating patients w ith m orbid obesity inclu d e all of the follow ing EXCEPT (A) sp u riou sly low blood p ressu re cu ff read ings (B) d ifficu lt venou s access (C) d ifficu lt intu bation (D) d ifficu lt airw ay m aintenance w ith m ask (E) d ifficu lty w ith nerve blocks DIRECTION S: Use the follow ing scenario to answ er Qu estions 50-52. Many p atients are u naw are of the reasons for w hich they are taking their m ed ications. When p rovid ed w ith a list of m ed ications by a p atient, an anesthesiologist m u st be able to recognize the m ed ications and know the likely p athological states for w hich the m ed ications are ind icated . For Qu estions 50-52, a m ed ication is follow ed by five d iseases or p athological states. Choose the ON E d isease for w hich the m ed ication m ay be ind icated . 50. Om eprazole (A) (B) (C) (D) (E)

p ep tic u lcer d isease d iabetic gastrop aresis vertigo u rinary tract infection typ e II d iabetes m ellitu s

162

10: P ra ctice Te s t

51. Acetazolam id e (A) (B) (C) (D) (E)

glau com a asthm a p ep tic u lcer d isease d eep venou s throm bosis atrial fibrillation

52. Glybu rid e (A) (B) (C) (D) (E)

m yasthenia gravis typ e II d iabetes m ellitu s angina p ectoris glau com a hyp ercalcem ia

53. Som atosensory-evoked potentials (SSEP) (A) are extingu ished by p rop ofol (B) are u naffected by d eep levels of isoflu rane anesthesia (C) are evalu ated from the stand p oint of am plitu d e only (D) are rend ered u nread able by neurom u scu lar blocking agents (E) are m od estly affected by nitrou s oxid e 54. A team of anesthesiologists has traveled to a third -w orld cou ntry to provid e anesthesia for their surgical colleagues w ho plan to perform plastic reconstru ctive p roced u res on child ren w ith cleft palate and similar craniofacial abnorm alities. The anesthesiologists have brou ght an am p le qu antity of sevoflu rane w ith them bu t find that there are no sevoflu rane vap orizers at the hosp ital w here they w ill w ork. In term s of the accu racy in d elivering sevoflu rane, the least d ifference betw een the set and d elivered concentrations of sevoflu rane w ill resu lt if sevoflu rane is d elivered via w hich one of these vaporizers? (A) (B) (C) (D) (E)

Desflu rane vap orizer Enflu rane vap orizer H alothane vap orizer Isoflu rane vap orizer Methoxyflu rane vap orizer

55. An 18-year-old m ale p resents for rep air of a right shou ld er rotator cu ff tear. H e agrees to an

interscalene block for p ostop erative analgesia. You d ecid e to avoid u sing the local anesthetic that is recognized as being the m ost card iotoxic. You therefore d ecid e to avoid (A) (B) (C) (D) (E)

bu p ivacaine rop ivacaine chlorop rocaine lid ocaine m ep ivacaine

56. All of the follow ing is tru e concerning the change in fu nctional resid u al cap acity (FRC) w ith bod y p osition, EXCEPT (A) FRC is m arked ly red u ced in the su p ine position (B) in head -d ow n, FRC is little d ifferent from su p ine (C) FRC is red u ced in sitting p osition w hen com p ared to su pine position (D) FRC is greater in lateral than in su p ine position (E) FRC is greater in p rone than in su p ine position 57. The insp ired p artial p ressu re of isoflu rane is low er than the p artial p ressu re of the anesthetic in the fresh gas because of all of the follow ing p rocesses EXCEPT (A) (B) (C) (D) (E)

absorp tion of the agent by ru bber hoses dilution of fresh gas in the circle absorber m etabolism of isoflu rane ad sorp tion of the agent by sod a lim e u p take of the agent by the p atient

58. All of the follow ing are factors that DECREASE the rate of onset of the effects of an inhalational anesthetic EXCEPT (A) low (as op p osed to high) fresh gas flow rate (B) low (as op p osed to high) m inu te ventilation (C) low (as op p osed to high) anesthetic concentration in inhaled gas m ixtu re (D) low (as op p osed to h igh ) blood :gas p artition coefficient (E) high (as op p osed to low ) card iac ou tp u t

Que s tions : 51–65

s n o p s

60. The nu m ber of calories requ ired to raise the tem perature of 1 g of a su bstance by 1°C is (A) (B) (C) (D) (E)

the heat of vap orization the sp ecific heat the critical tem p eratu re therm al cond u ctivity equ al for all su bstances

61. Your patient has d yspnea on m ild exertion and p erip heral ed em a. Mech an ism s by w hich patients w ith failing hearts compensate for low card iac ou tp u t inclu d e all of the follow ing EXCEPT (A) increased sym p athetic d rive to the heart (B) m yocard ial hyp ertrop hy (C) renal loss of salt and d ecreased blood volu m e (D) second ary hyp erald osteronism (E) p erip heral vasoconstriction 62. The p in ind ex safety system (PISS) (A) p revents attachment of gas-administering equipment to the wrong type of gas (B) p revents incorrect yoke-tank connections (C) consists of qu ick-connectors typ ically m ounted on the w all or hanging from the ceiling (D) is fou nd on the w all end , bu t not the m achine end , of gas hoses connected to anesthesia m achines (E) is fou nd on the m achine end , bu t not the w all end , of gas hoses connected to anesthesia m achines

B

e

A

e

(A) u su ally occu rs as a late sign (B) is alw ays associated w ith a rise of central venous pressu re (CVP) (C) w ill be d etected becau se of d istend ed neck vessels (D) m ay be d u e to airw ay obstru ction (E) u su ally occu rs w ith norm al breath sou nd s present

DIRECTION S: Use the follow ing figu re to answ er Questions 63-64:

R

59. Pu lm onary ed em a in the recovery room

163

log dos e

63. If A and B are tw o d ifferent m ed ications p rod u cing the sam e effect, then the figu re show s that (A) A has higher efficacy than B (B) A has low er efficacy than B (C) A and B m u st act via d ifferent recep tors to prod u ce their effects (D) A is m ore p otent than B (E) A has a higher ED 50 than B 64. If A d ep icts the d ose-response relationship of a d ru g X acting alone, and B d ep icts the d oseresponse relationship of d rug X in the presence of d ru g Y, then it can be said that d ru g Y (A) is a com p etitive antagonist of d ru g X (B) is a non-competitive antagonist of drug X (C) m u st act via a d ifferent recep tor than d rug X (D) d ecreases the efficacy of d ru g X (E) increases the p otency of d ru g X 65. A 63-year-old m ale p atient w ith end -stage renal d isease from long-stand ing hypertension p resents for p reop erative assessm ent p rior to elective rep air of an abd om inal aortic aneu rysm . Laboratory abnorm alities com m on in p atients w ith end -stage renal d isease inclu d e (A) (B) (C) (D) (E)

anem ia throm bocytosis hyp okalem ia hyp op hosp hatem ia leu kocytosis

164

10: P ra ctice Te s t

66. Fou r years after a heart transp lant, a p atient requ ires bow el resection. The transp lanted heart (A) has an abnorm al Frank–Starling m echanism (B) resp ond s as d oes an innervated heart to atrop ine (C) bears fu nctional α- and β -ad renoceptors (D) d oes not resp ond to isop roterenol (E) d oes not increase card iac ou tp u t w ith increased p reload 67. Cerebrosp inal flu id (CSF) flow s throu gh all of the follow ing EXCEPT the (A) (B) (C) (D) (E)

arachnoid villi cerebral ventricle lateral ventricle su barachnoid sp ace ep id u ral sp ace

68. Verap am il (A) is u sefu l in the treatm ent of su p raventricu lar tachycard ia (B) is con train d icated in p atien ts w ith asthm a (C) is u sefu l w hen com bined w ith p ropranolol (D) is a p otent vasoconstrictor (E) has no effect on the p acem aker cells 69. A resid ent is carrying a Tec 5 vaporizer from th e anesthesia w orkroom to th e op erating room and slip s on the freshly w ashed floor. Althou gh he d id not d rop it, he d id tip it on its sid e w hen he fell. After p rop erly m ounting the vap orizer on th e an esthesia m achin e, the proper proced u re to flush the vap orizer is to (A) leave the vap orizer setting at “off” and p ress the oxygen flu sh valve for 10 m in (B) leave the vap orizer setting at “off” and ad ju st the oxygen valve to d eliver oxygen at 10 L/ m in for 10 m in (C) tu rn the vap orizer setting to its m axim um valu e and p ress the oxygen flush valve for 10 m in

(D) tu rn the vap orizer setting to its m axim u m valu e and ad ju st the oxygen valve to d eliver oxygen at 10 L/ m in for 10 m in (E) retu rn the vap orizer to the m anu factu rer for service; d o not m ou nt it on the anesthesia m achine 70. A 39-year-old fem ale p resents for a vaginal hysterectom y. Ten m inu tes after p erform ing a sp inal anesthetic w ith 15 m illigram s of 0.5% hyp erbaric bu p ivacaine, you notice a slow ing of the heart rhythm on continu ou s EKG m onitoring. A short period of asystole then d evelop s at w hich tim e you ad m inister 10 m cg of ep inep hrine and p erform a brief p eriod of chest com pressions. The m ost im portant ind ep end ent p red ictor of brad ycard ia follow ing spinal anesthesia is (A) (B) (C) (D) (E)

hyp ertension hyp erlip id em ia p reviou s sp inal anesthesia p reexisting brad ycard ia ASA I statu s

71. A 45-year-old m ale w ith a history of hyp ertension that is controlled w ith a d iu retic is sched u led for a hern ia rep air. H e is com p letely asym p tom atic, bu t h is p lasm a p otassiu m concentration is 3.0 m Eq/ L. The ap p ropriate m anagem ent is to (A) cancel the p roced u re (B) start a p otassiu m infu sion and p roceed (C) p roceed w ith the p roced u re bu t not ad m inister p otassiu m (D) give 40 m Eq of p otassiu m by m ou th before starting (E) p roceed w ith the p roced u re only if it can be d one u nd er regional anesthesia 72. Intracranial p ressu re (ICP) can be m onitored w ith all of the follow ing EXCEPT (A) a catheter im p lanted d irectly into brain parenchym a (B) by a p ressu re transd u cing bolt in the subarachnoid space

Que s tions : 66–80

(C) by a p ressu re transd u cer in the ep id u ral sp ace (D) a ventricu lostom y catheter (E) a catheter in the su bd u ral sp ace 73. In a m ixtu re of gases (su ch as air) (A) all vap ors, bu t not all gases obey Dalton’s law (B) each gas contribu tes a p artial p ressu re that is proportional to its solu bility (C) each gas contribu tes a p artial p ressu re that is p rop ortional to its m olecu lar fraction (D) the total p ressu re is equ al to the su m of the gases m ultiplied by the nu m ber of gases (E) the total p ressu re is equ al to the su m of the gases d ivid ed by the nu m ber of gases 74. An effect of m etoclop ram id e is (A) (B) (C) (D) (E)

d op am ine recep tor agonism β -ad renergic agonism antagonistic effects at 5-H T3 recep tors m u scarinic cholinergic agonistic effects im p rovem ent of large bow el m otility

75. A 22-year-old m ale gu nshot victim requ ires urgent abd ominal surgery. H is record ind icates that he has biopsy-proven hepatic cirrhosis but no history of d rug or alcohol use, au toim m u ne d isease, or hepatitis. Possible hereditary causes for cirrhosis in this p atient includ e all of the follow ing EXCEPT (A) (B) (C) (D) (E)

hem ochrom atosis α 1-antitryp sin d eficiency Wilson d isease antithrom bin III d eficiency cystic fibrosis

76. H yp ovolem ia is su sp ected in a sep tic p atient in the ICU. All of the follow ing m ay be seen in hypovolem ia EXCEPT (A) increased heart rate (B) w id e p u lse p ressu re (C) d ecreased u rine volu m e

165

(D) flat neck veins (E) p ale m u cou s m em branes 77. A 42-year-old m an is sched u led for ap pend ectom y for acute append icitis. Four years ago, he had a m yocard ial infarction and w as treated w ith the insertion of intracoronary stents. H e u nd erw ent su ccessfu l card iac rehabilitation and now plays tennis regularly w ithout angina or shortness of breath. H e w ou ld be categorized as ASA Physical Statu s (A) (B) (C) (D) (E)

I II III IIIE IV

78. A 45-year-old m ale is to have a gallblad d er p roced u re. H e has heart d isease and is on am iod arone. This d ru g (A) (B) (C) (D) (E)

shou ld be stop p ed before su rgery has a half-life of 4 h is u sed for ventricu lar arrhythm ias is elim inated by the kid neys has no effects on β -ad renergic recep tors

79. Spinal cord injury is feared in patients requ irin g th oracoabd om in al aortic su rgery. Th e arteria rad icu laris m agna (of Ad am kiew icz) (A) is the largest of the rad icu lar arteries su pplying the spinal cord (B) norm ally feed s the anterop osterior sp inal artery system (C) anastom oses w ith the sp lenic artery (D) alw ays originates from the su p rarenal aorta (E) m ay be ligated w ith im p u nity 80. The latent heat of vap orization (A) is equ al for all liqu id s (B) is ind ep end ent of the am bient tem perature (C) varies w ith the tem p eratu re of the liqu id (D) is very low for solid s (E) for w ater is 1 calorie/ m L

166

10: P ra ctice Te s t

81. Carbon m onoxid e d iffu sion cap acity (DLCO) w ill d ecrease w ith (A) (B) (C) (D) (E)

sm oking w ithin 24 h of the test exercise p u lm onary fibrosis increased hem oglobin congestive heart failu re

82. A local anesthetic that inhibits the reu ptake of norepinephrine is (A) (B) (C) (D) (E)

p rocaine cocaine bu p ivacaine m ep ivacaine lid ocaine w ith ep inep hrine

83. A m id d le-aged m an w ith a know n pheochrom ocytom a is sched u led for elective abd om in al su rgery. Wh ich on e of th e follow in g statem ents abou t his p reop erative p harm acologic m anagem ent is correct? (A) α-ad renergic blockad e is institu ted at the sam e tim e as β -ad renergic blockad e (B) d iu retic antihyp ertensives are contraind icated (C) p reop erative d ru g therap y shou ld be started 12 h before su rgery (D) d osage is ad ju sted accord ing to the levels of u rinary catecholam ine m etabolites (E) calciu m channel blockers are contraind icated in this cond ition 84. A p atient w ith history of syncope is su sp ected of Wolff-Parkinson-White synd rom e. In this cond ition, the (A) heart rate is u sually 60 to 80 beats/ m in d uring sinus rhythm (B) PR interval is less than 0.12 sec d u ring sinu s rhythm (C) QRS d u ration is u su ally longer than 0.12 sec (D) the u p stroke of the QRS com p lex is often slu rred (E) ad enosine m ay increase heart rate

85. Atracu riu m (A) has both prejunctional and postjunctional effects (B) liberates histam ine at all d ose levels (C) liberates histam ine at all infu sion rates (D) breaks d ow n into lau d anosine that has caused seizures in hu m ans (E) cau ses increased blood p ressu re 86. A 70-year-old m an is receivin g an ticoagu lan t d ru gs to inhibit p latelet glycop rotein IIb/ IIIa recep tor. EXCEPT for the follow ing, the recep tor (A) (B) (C) (D) (E)

is activated by cyclic AMP p articip ates in aggregation of p latelets is inhibited by abcixim ab is inhibited by ep tifibatid e is activated by d iverse stim u li

87. Cerebrosp inal flu id p ressu re m ay be increased by (A) cou ghing (B) long exp iratory tim e d u ring p ositive pressu re ventilation (C) short insp iratory tim e d u ring p ositive pressu re ventilation (D) low exp iratory resistance (E) negative exp iratory p hase 88. Isoflu rane (A) is a p oor m u scle relaxant com p ared w ith halothane (B) has a vap or p ressu re of 175 m m H g at 20°C (C) stim u lates ventilation (D) has a MAC of app roxim ately 1.2% (E) shou ld be d elivered by sp ontaneou s ventilation 89. When inhaling from fu nctional resid u al cap acity (FRC) to m axim al insp iration, all of the follow ing are tru e, EXCEPT (A) p u lm onary vascu lar resistance increases (B) p leu ral p ressu re d ecreases (C) resid u al volu m e stays constant

Que s tions : 81–96

(D) basilar alveoli becom e larger than ap ical alveoli (E) p roxim al airw ay p ressu re is greater than alveolar p ressu re 90. Prop ofol has all of the follow ing p harm acological effects EXCEPT (A) (B) (C) (D) (E)

d ecreasing cerebral blood flow crossing the blood -brain barrier red u cing ICP d ecreasing cerebral m etabolic rate d ecreasing the latency of som atosensory evoked p otentials

91. Which one of the following statements regarding heparin is m ost accu rate? (A) It inhibits several step s in the intrinsic pathw ay of blood clotting. (B) Its d osage m ay be ad ju sted by estim ating the p atient’s clotting ability via the bleed ing tim e. (C) It shou ld not be injected su bcu taneou sly. (D) It interacts w ith d ru gs that inhibit the liver m icrosom al enzym e system . (E) It shou ld never be com bined in therap y w ith an oral anticoagu lant. 92. A patient undergoing a blood transfusion intraoperatively develops an acute hemolytic reaction. The diagnosis of such a reaction typically (A) reveals ABO incom p atibility to be an unlikely cau se (B) id entifies incom p atibility w ith a rare antigen after d etailed testing (C) first p resents as hem oglobinu ria if the patient is u nd er general anesthesia (D) show s that the severity of the hem olytic reaction is u nrelated to the volu m e of transfu sed blood (E) lead s to shock-ind u ced hep atic failu re that is the etiology of m ost of the associated m orbid ity 93. A cyanotic child is su sp ected of tetralogy of Fallot. The condition includes all of the following EXCEPT

(A) (B) (C) (D) (E)

167

atrial sep tal d efect (ASD) ventricular septal d efect (VSD) right ventricu lar hyp ertrop hy p ulm onary outflow obstru ction overrid ing aorta

94. The p ressu re of oxygen d elivered by the w all connectors in an op erating room is ap p roxim ately (A) (B) (C) (D) (E)

1900 p si 750 p si 50 p si 1 atm 760 m m H g

95. A 41-year-old p atient w ith a history of d ru g abu se is ad m itted to the em ergency d ep artm ent for acute onset, anterior chest pain that is rad iating into the back betw een the shou ld er blad es. Vital signs on arrival inclu d e a H R of 107, and a BP of 172/ 98. The p atient app ears p ale and d iap horetic. Em ergent TEE find ings inclu d e d ilated card iom yop athy and evid ence of acute aortic d issection; the patient is referred for em ergent card iac su rgery for aortic rep air. When p lanning this p atient’s anesthetic, caution has to be exercised w ith the ad m inistration of w hich one of the follow ing d ru gs? (A) (B) (C) (D) (E)

labetalol ep inep hrine m id azolam ep hed rine p henylep hrine

96. An ap p lication of the Bernou lli theorem is the m easu rem ent of (A) card iac ou tp u t w ith a therm od ilu tion p u lm onary artery catheter (B) m ean arterial p ressu re u sing an au tom ated noninvasive blood pressu re d evice (C) m inu te ventilation u sing a sp inning tu rbine (D) the p ressu re grad ient across a stenotic m itral valve u sing echocard iography (E) airw ay resistance u sing a sp irom eter

168

10: P ra ctice Te s t

97. A 45-year-old m an w ith hyp ertension and a 60-pack-year sm oking history is being evalu ated in the p reop erative clinic for an elective neu rosu rgical p roced u re to rem ove a benign spinal tu m or. All of the follow ing statem ents abou t exp osu re to tobacco in the p eriop erative period are correct EXCEPT (A) nicotine rep lacem ent therap y shou ld be started 1-2 w eeks before an attem p t at cessation (B) sm okers are m ore likely to exp erience w ou nd infections (C) p atients p resenting for su rgery are m ore likely to qu it sm oking w hen so ad vised than sm okers not having surgery (D) varenicline for sm oking cessation shou ld be started 1-2 w eeks before a quit attem pt (E) bu p rop ion is effective p harm acotherap y to assist w ith sm oking cessation in the p erioperative p eriod 98. Arterial oxygen tension can be im p roved by (A) transferring an obese p atient from the su pine to the Trend elenbu rg (B) d ecreasing insp ired FIO 2 (C) u sing calciu m channel blockers in p atients on one lu ng ventilation (D) u sing PEEP in the p resence of atelectasis (E) increasing the concentration of the volatile agent 99. A 19-year-old p atient p resents for em ergent craniotomy for evacuation of a subd ural hematom a follow ing a m otor vehicle accid ent. Which one of the follow ing anesthetics is m ost likely to increase cerebral blood flow ? (A) (B) (C) (D) (E)

ketam ine thiop ental sevoflu rane (1 MAC) m id azolam p rop ofol

100. A patient has an arterial catheter in place and the anesthesiologist can view the d isp layed

value of pulse pressure variability. This valu e m ay be correlated w ith (A) (B) (C) (D) (E)

the the the the the

p robability of card iac ischem ia d ep th of anesthesia d egree of hyp ovolem ia card iac ou tp u t com p liance of the lu ngs

101. A 37-year-old obese fem ale w ith a know n d ifficu lt airw ay presents for a laparoscopic cholecystectom y. You d iscu ss perform ing an aw ake fiberop tic intu bation w ith top ical anesthesia. Which local anesthetic is u sed exclu sively for topical pu rposes? (A) (B) (C) (D) (E)

Prilocaine Tetracaine Lid ocaine Rop ivacaine Benzocaine

102. A 40-year-old m ale p atient w ith acrom egaly p resenting for p rostate resection is likely to have all of the follow ing EXCEPT (A) (B) (C) (D) (E)

a d ifficu lt airw ay glu cose intolerance hyp ertension hyp erthyroid ism card iom egaly

103. Calciu m channel blockers (A) shou ld be stop p ed p rior to elective su rgery (B) have a p rofou nd negative inotrop ic effect w hen ad m inistered to a p atient w ho has been given a high d ose op ioid anesthetic (C) m ay p rolong atrioventricu lar cond u ction tim e w hen com bined w ith volatile anesthetics (D) m ay antagonize the effect of m u scle relaxants (E) can be u sed interchangeably d u e to id entical effect sites and m echanism s of action

Que s tions : 97–111

104. Which one of the follow ing is tru e regard ing the u pper airw ay? (A) The hyoid bone su sp end s the sternothyroid m uscle at the level of C7. (B) The anatom ical stru ctu re referred to as “Ad am ’s app le” is called cricoid cartilage. (C) The cricothyroid m em brane is su itable for em ergency airw ay access; it is located in the m id line at the level of C6. (D) To each sid e of the larynx and inferior to the aryepiglottic folds is the rima glottidis, w here the end otracheal tu be shou ld be placed d uring intu bation. (E) The laryngoscop e shou ld be p ositioned in the p iriform sinu s d u ring d irect laryngoscop y. 105. The irrigating flu id for a 55-year-old otherw ise h ealth y m an u n d ergoin g tr an su reth r al p rostatic resection (A) can cau se hyp otension and tachycard ia in the anesthetized p atient (B) shou ld be w ater (C) shou ld be isosm olar (D) shou ld be hyp ertonic saline (E) shou ld be a solu tion of a nonm etabolized solu te 106. H yp ercarbia occu rring u nd er anesthesia m ay be d u e to all of the follow ing, EXCEPT (A) (B) (C) (D) (E)

increased d ead sp ace ventilation exhau stion of sod a lim e p u lm onary em bolism hyp erventilation m alignant hyp ertherm ia

107. An anesthetic agent for w hich a sp ecific antagonist exists is (A) (B) (C) (D) (E)

etom id ate bu torp hanol thiop ental p rop ofol su ccinylcholine

169

DIRECTION S: Use the follow ing scenario to answ er Questions 108-109. A 55-year-old m ale is ad m itted w ith a history of hyp ertensive heart d isease and evid ence of acu te m yocard ial infarction. Vital signs are: blood p ressure 180/ 110, heart rate 124 p er m inu te, bod y tem peratu re 101°F, and respiratory rate 24 per m inu te. 108. Efforts to im p rove m yocard ial oxygenation shou ld includ e the follow ing EXCEPT (A) (B) (C) (D) (E)

d ecreasing arterial blood pressure d ecreasing bod y tem peratu re ad m inistration of intravenou s flu id slow ing heart rate ad m inistration of oxygen

109. Chest pain w orsens as the heart rate and blood pressure are reduced with metoprolol and nitroglycerin. Additional measures might include (A) an inhibitor of cyclic-GMP p hosp hod iesterase (B) am iod arone (C) antifibrinolytics (D) anticoagu lation (E) lid ocaine 110. A contraindication to the discharge to home of a previously healthy, ASA I patient, w ho had a hernia repair under general anesthesia would be (A) heart rate 20% higher than baseline (B) systolic blood p ressu re 20% low er than baseline (C) inability to void (D) oxygen satu ration of 93% on room air (E) requiring the assistance of another person to am bu late 111. Which one of the follow ing factors introd uces the greatest d egree of error in the measurement of hemoglobin saturation by pulse oximetry? (A) (B) (C) (D) (E)

H igh concentration of fetal hem oglobin H igh concentration of sickle hem oglobin H igh concentration of m ethem oglobin H igh concentration of d eoxyhem oglobin H yp erbiliru binem ia

170

10: P ra ctice Te s t

DIRECTION S: Use the follow ing scenario to answ er Qu estions 112-113: An op erating room is p rotected by an isolated p ow er system that is m onitored by a line isolation m onitor. Du ring lap aroscop ic cholecystectom y su rgery, the line isolation m onitor alarm s and the m eter ind icates 8 m A. 112. What action shou ld initially be taken? (A) Unp lu g the flu id w arm er becau se it d raw s the m ost current. (B) Unp lu g the su rgeon’s rad io becau se it is u nnecessary to the proced u re. (C) Tu rn off the flu orescent lights becau se they often interfere w ith the line isolation m onitor. (D) Unplug the last device that was connected to the circuit. (E) Unp lu g the anesthesia m onitors and operate them on battery p ow er. 113. If none of the m aneu vers taken by the OR staff rem oves th e alarm cond ition, w h at action shou ld be taken next? (A) Relocate the p atient to a d ifferent operating room w hile m aintaining anesthesia. (B) Convert to op en cholecystectom y and avoid cau tery. (C) Continu e the su rgery becau se 8 m A is a harm less cu rrent that is flow ing throu gh the patient. (D) Continu e the su rgery becau se a second , rare electrical fau lt w ill need to occu r in ord er to cau se electrical inju ry to the p atient. 114. A 51-year-old p atient is u nd ergoing popliteal block u nd er u ltrasou nd gu id ance for p ostop erative p ain control follow ing ankle su rgery. Mid azolam 2 m g intravenou sly has been ad m in istered in crem en tally for p roced u ral sed ation. Shortly after injection of 30 m L of rop ivacaine 0.5%, the p atient starts to com plain about visual d isturbances that are quickly follow ed by loss of consciou sness and resp iratory arrest. Vital signs at the tim e are significant for new onset hyp otension and card iac

arrhythm ia. Assu m ing that the p atient is exp eriencing sym p tom s of local anesthetic toxicity (LAST), w hich one of the follow ing interventions is m ost appropriate? (A) Ventilation w ith 100% O 2, p rop ofol bolu s, ad m inistration of lid ocaine and p henylep hrine (B) Ventilation w ith 100% O 2, m id azolam bolu s, ad m inistration of lid ocaine and vasopressin (C) Ventilation w ith 100% O 2, ad m inistration of d iazepam , a beta blocker, and ep inep hrine (D) Ventilation w ith 100% O 2, ad ministration of m id azolam follow ed by continu ous infusion of Intralipid 20%, ad ministration of lid ocaine and phenylephrine (E) Ventilation w ith 100% O 2, ad m inistration of 1.5 m L/ kg Intralipid 20% follow ed by continu ou s infu sion, ad m inistration of phenylephrine 115. The p rincip al d isad vantage of m ethohexital is (A) p oor w ater solu bility (B) d ose d ep end ent d ecrease in blood p ressu re greater th an th at p rod u ced by p rop ofol (C) low p H of solu tion (D) involu ntary m u scle m ovem ents (E) increase in ictal activity 116. Assu m ing a constant rate of carbon d ioxid e form ation, the relationship betw een alveolar ventilation (in a patient und er general anesthesia) and carbon d ioxid e and hyd rogen ion concentration is (A) carbon d ioxid e is d irectly p rop ortional to ventilation (B) d ou blin g ven tilation w ill lead to resp iratory alkalosis (C) red u cing ventilation to one fou rth of norm al w ill lead to m etabolic acid osis (D) interru p ting ventilation for five m inu tes w ill cau se, next to hyp oxem ia, p rofou nd resp iratory alkalosis (E) alveolar ventilation only resp ond s to end ogenou s CO 2

Que s tions : 112–123

117. Effects of fat em bolism inclu d e all of the follow ing, EXCEPT (A) (B) (C) (D) (E)

hyp oxia tachyp nea interstitial p u lm onary ed em a neu rological im p airm ent left-axis d eviation in the ECG

118. A 37-year-old woman presents for a brain biopsy u nd er m onitored anesthesia care. She has a family history of acute intermittent porphyria (AIP) and relates a history of several porphyric attacks over the last 20 years. All of the following statements about AIP are true EXCEPT (A) p rop ofol is safe in p atients w ith AIP (B) severe abd om inal p ain, m ental statu s changes and peripheral neuropathy characterize acu te porphyric attacks (C) it is related to a sp ecific enzym e d eficiency in the biosynthesis of flavincontaining enzym es (D) etom id ate shou ld be avoid ed in p atients w ith AIP (E) factors know n to p recip itate acu te p orphyric crisis inclu d e fasting, d ehyd ration, and infection 119. The p atient w ith m yasthenia gravis (A) has w eakness of m u scles innervated by cranial nerves (B) u su ally has d iap hragm atic w eakness (C) is very sensitive to su ccinylcholine (D) has focal sensory d eficits (E) often has associated carcinom as 120. The brain stem is supplied w ith blood from the (A) (B) (C) (D) (E)

m id d le cerebral artery cingu late artery basilar artery p osterior cerebral artery p osterior com m u nicating artery

121. When d onated blood is initially collected , it is m ixed w ith an anticoagulant and preservation solution containing glucose, citrate, phosphate, and ad enine. It has a shelf life of five w eeks.

171

If the unit of w hole blood thu s collected is fractionated into its com p onents, the shelf life of the p acked red cells m ay be extend ed to six w eeks by ad d ing ad d itional glu cose and (A) (B) (C) (D) (E)

ascorbic acid (Vitam in C) 2,3-DPG sod ium bicarbonate α-tocop herol (Vitam in E) ad enine

122. All of the follow ing statements are tru e regard ing sm oking, EXCEPT (A) sm oking is the strongest m od ifiable risk factor for card iovascu lar d isease (B) com p ared to non-sm okers, FEV1 tend s to d ecrease m ore in sm okers w ith increasing age (C) sm oking cessation before su rgery m ay red u ce airw ay hyp erreactivity (D) p ostop erative sm oking cessation m ay im p rove w ou nd healing (E) risk of p ostop erative nau sea and vom iting is higher in sm okers 123. Rop ivacaine (A) is su p p lied as a racem ic m ixtu re (B) is less likely than bu p ivacaine to cau se ventricu lar arrhythm ias after accid ental intravenou s injection (C) p rod u ces m otor blockad e of longer d u ration than sensory blockad e (D) has a low d egree of toxicity by virtu e of its rap id intravascu lar m etabolism (E) is an am inoester agent DIRECTION S: Use the follow ing case to answ er Qu estions 124-126: A 67-year-old w om an is brou ght to the operating room for an exploratory laparotom y for an intra-abd om inal abscess. After ind uction of anesthesia, a hot air w arm er is applied to the patient and bu t not initiated becau se the esop hageal tem p eratu re is m easu red at 37.8°C. The skin is w arm and w et to tou ch. Anesthesia is m aintained w ith isoflu rane in an air/ oxygen m ixtu re and m u scle relaxation is provid ed by vecu ronium . After opening the abd om en and w ashou t of the abscess cavity, the tem p eratu re d rop s to 36.5°C.

10: P ra ctice Te s t

c

e

n

t

r

a

t

i

o

n

B

n

therm al-regu lating m echanism p rocess m ed iated by β -ad renocep tors resp onse to sep sis and hyp erp yrexia resp onse to em otional stim u li resp onse to hyp oglycem ia

(A) is closely regulated by the hypothalam u s (B) is elevated if the p atient is p erip herally vasod ilated (C) cannot be increased by increased m etabolism (D) is best assessed by skin tem p eratu re (E) w ill tend to d rift tow ard am bient tem peratu re 126. Regu lation of bod y core tem peratu re involves all of the follow ing EXCEPT (A) (B) (C) (D) (E)

heat heat heat heat heat

p rod u ction p rod u ction d issip ation d issip ation d issip ation

by m u scle by the liver by shivering by the lu ngs by the skin

127. The d ifference betw een the alveolar p ressu re and the am bient pressure is the su m of (A) transp u lm onary p ressu re and chest w all transm u ral pressure (B) esop hageal balloon p ressu re and chest w all transm u ral p ressu re (C) chest w all transm u ral p ressu re and pu lm onary artery occlu sion p ressu re (D) p u lm onary artery occlu sion p ressu re and esophageal balloon pressu re (E) esop hageal balloon p ressu re and transpu lm onary pressure

l B

125. Core tem p eratu re in the p atient u nd er general anesthesia

o

o

d

(A) (B) (C) (D) (E)

DIRECTION S: Use the follow ing figu re to answ er Qu estions 128-129:

o

124. The p atient m ay sw eat as a resu lt of all of the follow ing p rocesses EXCEPT a

c

172

A 0

1

2

3 4 Time (days )

5

6

128. Dru gs A and B w ere given by intravenou s bolus every 24 h. From the graph, it can be conclud ed that (A) d ru g A m aintains p ersistent therap eu tic levels w hen ad m inistered every 24 h (B) d ru g B is cu m u lative w hen ad m inistered every 24 h (C) d ru g A exhibits cu m u lative p rop erties (D) d ru g B is com p letely d estroyed in less than 24 h (E) d ru g B has reached stead y state p lasm a concentration after 4 d ays 129. In ad d ition, the grap h show s that (A) d ru g A has a long half-life in the bod y (B) a large initial d ose of both d ru gs w as given (C) d ifferent d oses are given for initial d oses and m aintenance d oses (D) in the case of d ru g B, the bod y d oes not rem ove one d ose before another is ad m inistered (E) d u ration of action of the tw o d ru gs is id entical 130. All of the follow ing are tru e of airw ay resistance EXCEPT (A) d ecreases w ith m axim al insp iration (B) increases w ith p arasym p athetic stim ulation (C) increases w ith sym p athetic stim u lation (D) increases w ith acetylcholine inhalation (E) increases w ith inhalation of sm oke

Que s tions : 124–137

131. Mannitol d ecreases cerebral ed em a becau se it (A) blocks reu p take of sod iu m in the collecting system (B) d raw s w ater across an intact blood brain barrier to d ehyd rate the brain (C) red u ces red blood cell volu m e (D) is a p erip heral vasod ilator (E) can be given in kid ney failu re 132. Sickle cell form ation is facilitated by all of the follow ing EXCEPT (A) (B) (C) (D) (E)

173

135. Milrinone (A) (B) (C) (D) (E)

inhibits N a,K-ATPase causes vasod ilation is an end othelin receptor antagonist stim u lates phosphod iesterase has inotropic effects via β 1-ad renocep tor stim u lation

DIRECTION S: Use the follow ing figu re to answ er Qu estions 136-137:

hyp oxem ia sevoflu rane acid osis fever d ehyd ration

133. A 33-y ea r-old w om a n w ith a recen t on set (3 w eeks) of clinical hyperthyroid ism is ad mitted for repair of a tend on laceration. She has n ot taken m eth im azole for 4 d ays. At th is tim e the (A) p atient shou ld be essentially eu thyroid (B) p atient shou ld have general anesthesia if at all p ossible (C) therap y w ith m ethim azole m ay have rend ered the p atient hyp othyroid (D) p atient w ou ld p robably have brad ycard ia (E) p atient shou ld be consid ered as a highrisk hyp erthyroid p atient 134. Which one of the follow ing is a characteristic of general anesthetic agents? (A) All inhaled anesthetics p rod u ce bronchod ilation. (B) All general anesthetics are gases at bod y tem peratu re. (C) All general anesthetic effects m ay be exp lained by their ability to d isru p t m em brane lip id -p rotein interactions. (D) General anesthetic effects m ay be reversed by d ecreasing the am bient pressure. (E) N itrou s oxid e cau ses a d ose-d ep end ent inhibition of DN A synthesis.

136. As com p ared to the norm al Cu rve 2, all of the follow ing are TRUE about Cu rve 3 EXCEPT (A) (B) (C) (D) (E)

resid u al volu m e is elevated late exp iratory flow is red u ced vital cap acity is relatively norm al total lu ng cap acity is elevated it is characteristic of restrictive lu ng d isease

137. As com p ared to the norm al Cu rve 2, all of the follow ing are TRUE about Cu rve 1 EXCEPT (A) (B) (C) (D) (E)

resid u al volu m e is red u ced late exp iratory flow is red u ced vital cap acity is red u ced total lu ng cap acity is red u ced it is characteristic of restrictive lu ng d isease

174

10: P ra ctice Te s t

138. Thiop ental is contraind icated in (A) (B) (C) (D) (E)

p orp hyria congenita p orp hyria cu tanea tard a acu te interm ittent p orp hyria m yotonia chorea

139. Signs of hypoxemia includ e all of the follow ing EXCEPT (A) d ecreased ventilatory effort d u e to chem oreceptor stim ulation (B) increased heart rate d u e to sym p athetic stim u lation (C) cyanosis (D) d ecreased heart rate d u e to d irect effect of hypoxem ia on the heart (E) increased blood p ressu re d u e to sym p athetic stim ulation 140. Dop am ine (A) in a low d ose infu sion can cau se vasod ilation and inhibit release of norep inep hrine from sym pathetic nerves (B) as a low d ose infu sion is selective for the α 2-ad renocep tor (C) as a high d ose infu sion is beneficial in patients w ith p rim ary card iac contractile d ysfu nction (D) acts via nicotinic recep tors (E) is a β 1-selective inotrop e 141. Agents in anesthetic p ractice that can cau se bronchod ilatation inclu d e all of the follow ing, EXCEPT (A) (B) (C) (D) (E)

ketamine atrop ine isoflu rane sevoflu rane thiop ental

DIRECTION S (Qu estions 142-143): Each grou p of item s below consists of lettered head ings follow ed by a list of nu m bered p hrases or statem ents. For each nu m bered p hrase or statem ent, select the ON E lettered head ing or com p onent that is m ost closely associated w ith it. Each lettered head ing or com ponent m ay be selected once, m ore than once, or not at all. (A) (B) (C) (D) (E) (F) (G) (H )

Pneu m othorax Myocard ial infarction Local anesthetic toxicity Anap hylaxis H em id iap hragm atic p aresis Transient neu rologic inju ry Sp inal stenosis Total sp inal anesthesia

For each p atient rep orting severe shortness of breath, select the m ost com m on reason for his or her sym ptom s 142. An 18-year-old cheerlead er is sched u led to u nd ergo op en red u ction and internal fixation of her w rist after sustaining a d istal rad ial fractu re. One hour before the sched uled su rgical p roced u re she u nd ergoes a su p raclavicu lar brachial plexu s block w ith 20 m L of 0.5% bu pivacaine. One hour after returning to the PACU, she reports severe shortness of breath. 143. A 72-year-old p atien t w ith COPD u n d ergoes a left in terscalen e brach ial p lexu s block w ith 20 m L of 0.5% rop ivacain e. Fifteen m in u tes after the local anesthetic injection, the p atient rep orts severe sh ortn ess of breath . Th e m on itor d isp lays sin u s tach ycard ia w ith a h eart rate of 134.

Que s tions : 138–150

DIRECTION S (Qu estions 144-145): Each group of item s below consists of lettered head ings follow ed by a list of nu m bered p hrases or statem ents. For each num bered phrase or statem ent, select the ON E lettered head ing or com p onent that is m ost closely associated w ith it. Each lettered head ing or com p on ent m ay be selected once, m ore than on ce, or n ot at all. (A) (B) (C) (D) (E) (F)

T2 T4 T6 T7 T10 L1

For each p atient, select the ap p rop riate d erm atom e level corresp ond ing to the height of the sp inal anesthetic block. 144. A 21-year-old fem ale p resents for a cesarean section d u e to an arrest in labor. You perform a spinal anesthetic w ith 13.5 m g of hyp erbaric bupivacaine. When assessing the level of block you d eterm ine that the p atient is insensate to the level of the nipple line. 145. A 78-year-old m an w ith a history of coronary artery d isease p resents for transu rethral resection of the p rostate. You d ecid e to p erform spinal anesthetic in ord er to be able to m onitor the patient’s neu rologic statu s. Your goal is to achieve a level of blockad e correspond ing to the u m bilicus. DIRECTION S (Qu estions 146-150): Each group of item s below consists of lettered head ings follow ed by a list of nu m bered p hrases or statem ents. For each nu m bered phrase or statem ent, select the ON E lettered head ing or com p onent that is m ost closely associated w ith it. Each lettered head ing or com ponent m ay be selected once, m ore than once, or not at all.

175

(A) (B) (C) (D) (E) (F)

Acu te p ericard itis Chronic p u lm onary hyp ertension Chronic constrictive p ericard iop athy Myocard itis Pericard ial tam p onad e Post-infarction ventricu losep tal d efect (VSD) (G) Pu lm onary em bolism (H ) Rheu m atic fever (I) Sp ontaneou s p neu m othorax For each p atient w ith a card iac sym p tom or sign, select the m ost likely m echanism cau sing the sym ptom s or sign. 146. A 70-year-old m an ap p ears to be stabilizing w ell after a m yocard ial infarction. H e su d d enly d evelop s p rofou nd hyp otension and resp iratory d istress. An end otracheal tu be is p laced , and an intra-aortic balloon p u m p is initiated . 147. A 60-year-old w om an w ith insid iou s d yspnea on m ild exertion has a systolic heart m urm ur. 148. A you ng ad u lt com p lains of recent onset of chest pain and breathlessness. 149. A 58-year-old man und erw ent uneventful coronary artery bypass grafting, and is transferred to the intensive care unit. After 4 h of hemod ynamic stability, he suddenly develops profound hypotension, and equalization of invasive pressures is noted on initial evaluation. 150. A 60-year-old recipient of heart transplantation has poor card iac ou tp u t initially after the su rgery bu t im p roves d u rin g treatm en t w ith inhaled nitric oxid e and a p hosp hod iesterase inhibitor.

Answe rs a nd Expla na tions

1. (B) The interaction of rocu roniu m and acetylcholine at the m yoneu ral ju nction is one of com petition for bind ing sites. As the relaxant occu p ies the recep tors, acetylcholine cannot bind and have an effect. (1:60) 2. (D ) Com pliance increases as one m oves d ow n the lu ng. Depend ent alveoli are sm aller d ue to the w eight of the above tissu e. H ow ever, this also m eans that they have a higher cap acity for exp ansion. The shap e of the thorax and d iaphragm enhance this effect (5:959) 3. (A) (1:1255, 1263) 4. (A) The liver is the m ost im p ortant site of d ru g transform ation. The kid ney is responsible for the excretion of m any d ru gs, and the m etabolism of a few. There are som e d ru g m etabolism reactions that occu r in the lu ng and a few that occu r in the spleen. (1:28) 5. (C) Esm olol is inactivated by esterase activity in the cytosol of red cells. Pseud ocholinesterase is a p lasm a p rotein attacking su ccinylcholine and ester local anesthetics. Rem ifentanil is d egrad ed in skeletal m u scle. Alkaline p hosp h atase activates w ater-solu ble p rop ofol. (5:752-3) 6. (A) The type of med ical or surgical stress need s to be consid ered w hen ord ering glucocorticoid su p p lem entation in p atients w ith ad renal insu fficiency. Patients u nd ergoing m inor p roced ures such as herniorrhaphy or colonoscopy are sup p lem ented w ith 25 m g of hyd rocortisone or 5 m g of m ethylpred nisolone IV for the proced u re only. (5:157)

176

7. (E) Bioavailability is the am ou nt of d ru g that reaches its site of action in active form . (1:20) 8. (E) Patients w ith anorexia nervosa m ay have p rofou nd electrolyte d istu rbances increasing the risk of d ysrhythm ias. Osteop orosis m ay resu lt from m alnu trition. Orthostatic hyp otension, d elayed gastric em p tying and brad ycard ia are com m on. (6:637) 9. (D ) In one sm all stu d y, p atients w ith u nequ ivocal intraoperative aw areness had a 50% incid en ce of d evelop in g p osttrau m atic stress d isord er. (5:609) 10. (A) The congenital syndrome affects 1 in 5000 people and occurs because of cardiac ion channel m u tations. The acqu ired synd rom e is m ore common and is caused by medication side effects or electrolyte d isturbances. A prolonged QT interval increases the likelihood of torsades de pointes and ventricular fibrillation. (5:107-8) 11. (D ) Renal clearance of a d ru g in an ind ivid u al w ith norm al renal fu nction m ay vary from p atien t to p atien t and from tim e to tim e, d ep end ing on su ch variables as u rine p H and flow rate and renal blood flow. Excretion is correlated w ith renal blood flow. Renal elim ination is correlated w ith creatinine clearance, and one m ay use changes in creatinine clearance to gau ge how d oses m u st be altered . (1:28-30) 12. (A) Class I is asym p tom atic d u ring ord inary exertion. Class II is slightly lim ited becau se ord inary activity is sym ptom atic. Class III is m arked ly lim ited bu t asym p tom atic at rest. Class IV has sym p tom s at rest. (6:1818)

Que s tions : 1–25

13. (C) Closing volume is the lung volume at which small airways collapse. Closing volume is related to lung elasticity. Factors that change FRC (general anesthesia, obesity, and supine position) can cause bronchioles to close during each breath, but do not change closing volume, per se. Closing capacity is measured by a single breath from maximal exhalation of a tracer gas, or by nitrogen washout after using 100% O 2 as the tracer gas. Measurem ent of closing volume w ill d etect changes in the small airways before they become apparent by spirometry. (5:133, 256) 14. (D ) Trad itional signs of the anesthetic state are not seen w ith ketam ine ad m inistration. Purposeless m ovem ents are seen, and these d o not ind icate the n eed for m ore an esth etic. (5:699-701) 15. (D ) Global cerebral blood flow is 50 m L/ 100 g/ min u nd er normal physiological cond itions. Cortical areas rich in gray m atter receive m ore flow, 75 to 80 m L/ 100 g/ m in, w hereas su bcortical, w h ite m atter rich areas receive 20 m L/ 100 g/ m in. (5:872; 6:171-2) 16. (B) In Marfan synd rom e, collagen vascular d isease, fam ilial aortic d issection, bicu sp id aortic valve, ascend ing aortic repair is ind icated at a d iam eter of 4.5 cm . For other p atients the ind ication is a d iam eter of 5.5 cm, though 5.0 cm ind icates ascend ing aortic rep lacem ent if the aortic valve requ ires su rgery. (5:915) 17. (C) The perip heral chem oreceptors are p rimarily responsible for the hypoxic d rive of respiration. They are d isabled at very high oxygen ten sion s. Th e p erip h eral ch em orecep tors respond to changes in oxygen tension, not content, and are located in the carotid arteries. In the eld erly, d ysfu nction of the peripheral chem orecep tors can cau se a 50-60% red u ction in the ventilatory response to hypoxia and hypercap nia. (5:134, 251; 6:277) 18. (D ) Warfarin shou ld not be u sed u ntil the throm bocytopenia has resolved becau se it can cau se venou s lim b gangrene or m u lticentric skin necrosis. Danaparoid , lep iru d in, and argatroban are anticoagu lants that are ap proved

177

for p atients w ith hep arin-ind u ced throm bocytop enia. Benzod iazep ines have no clinical effect on the coagulation system . (5:214-5) 19. (E) The presence of CO 2 can d emonstrate respiratory gas exchange, bu t the correct p osition w ithin the tracheo-bronchial tree can only be d eterm ined by bronchoscop y, X-ray, or (less reliably) au scu ltation . Foggin g can ap p ear d u ring esop hageal intu bation. (5:467, 573) 20. (E) H yp oxic p u lm on ary vasocon striction causes d iversion of blood flow from hypoxic to non-hypoxic lung tissue. This causes a d ecrease in VD / VT and shu nting. (5:612) 21. (C) Su ccinylcholine in the su scep tible p atient causes the release of potassium from insid e the cell. The efflu x of p otassiu m , w hich is norm al at the m otor end p late, is seen over a m u ch w id er area of the m em brane in the bu rned p atient. This m ay result in a m assive outpouring of p otassiu m and card iac arrest. (5:1345) 22. (D ) The tim e constant of the system is the volume divided by the fresh gas flow, i.e., 5 L ÷ t 1 L/ m in , or 5 m in. Sin ce 1 − e τ = 1 – e –1 = 0.63, after one tim e constant the d esflu rane concentration w ill be 63% of the valu e set on the vap orizer. After tw o tim e constants, the p ercentage is 85%; after three tim e constants, 95%; after fou r tim e constants, 98%; and after five tim e constants, 99.3%. (5:601) 23. (D ) Factor VIII is d eficient in hem op hilia A, w hile factor IX is the H agem an factor. (5:208-9) 24. (D ) Prop ofol is an intravenou s agent w ith a fast onset and short tim e to aw akening. It is essentially insolu ble in w ater and there is very little analgesic effect. When com p ared to thiop ental, the sleep tim e is shorter. (5:688) 25. (D ) Metabolic synd rom e increases the risk of card iovascu lar d isease, cerebrovascu lar d isease, and d iabetes. App roxim ately one-half of p atients w ith p olycystic ovarian d isease are obese and m etabolic synd rom e is com m on. There is no role for p rop hylactic m ed ication. (5:165-6; 6:387)

178

10: P ra ctice Te s t

26. (D ) Pu lse p ressu re w id ens in sm all d istal vessels. (5:406)

rep resent either typ e of blockad e. Com p lete relaxation is not p resent. (5:494-6)

27. (D ) Analysis of arterial blood gases is the m ost reliable m ethod of d ocu m enting hyp oxem ia, bu t p u lse oxim etry gives accu rate, fast, and continu ou s resu lts and thu s is best as a m onitor. Detection of cyanosis and circu latory resp onses are all su bjective and m ay not be accu rate. Resid u al an esth etics m ay blu n t norm al responses to hyp oxia, and hyp oxem ia m ay be p resent w ithou t ap n ea. The p u lse oxim eter perm its the d etection of hyp oxem ia in less tim e than transcu taneou s oxygen m easu rem ents. In a p atient given su p p lem ental oxygen, p u lse oxim etry is less sensitive in d etecting hyp oventilation. (5:88)

33. (D ) The d ru g is a m u scle relaxant since tetanic stim u lation prod u ces fad e. The d ru g m ay be either a d epolarizing or nond epolarizing relaxant. If the cu m u lative d ose of a d ep olarizing relaxant is su fficient to cau se a p hase II block, fad e m ay be d em onstrated . (5:494-6, 500)

28. (B) Patients w ith bu rns over a large portion of their bod y surface area are resistant to nond epolarizing neu rom u scu lar blockad e. The other factors all p otentiate su ch blockad e. (5:502-3) 29. (A) The H ering-Breuer response is accentu ated in p reterm infants com p ared w ith fu ll-term infants and lead s to ap nea w ith lu ng inflation in p reterm infants. (5:255) 30. (D ) Sw eating and tachycard ia are com pensatory m echan ism s to rid the bod y of the increased heat being p rod u ced in the p atient w ith hyp erthyroid ism . Fatigu e is com m on. H yp erthyroid ism m ay p rod u ce hyp erm otility w ith resultant d iarrhea. Shivering is not a heatlosing m echanism . (6:314, 2923) 31. (C) A p atient w ho is d eep ly sed ated shou ld resp ond p u rp osefu lly to a p ainfu l p inch w hile a p atient u nd er general anesthesia shou ld not. The ability to follow a sim p le com m and , and m aintain ad equate m inu te ventilation and an u nobstructed airw ay are all characteristics of m od erate sed ation. The BIS values associated w ith d eep sed ation and general anesthesia overlap su bstantially and therefore cannot be u sed to d istingu ish the tw o states. (5:1261) 32. (E) From the d ata p resented in the tw o segm ents, the only statem ent that can be m ad e is that there is a 90% block. The block cou ld

34. (B) Th e m ou n tain eer ’s alveolar oxygen concentration is estim ated from the alveolar gas equ ation: P AO 2 = FIO 2 × (P atm − P H O ) − Pa CO 2 2

× (FIO 2 + (1 − FIO 2)/ RQ)) Assum ing a valu e of 0.8 for the resp iratory qu otient, the m ountaineer ’s alveolar oxygen concentration w ou ld be ap p roxim ately 54 m m H g at an altitu d e of 10,000 feet. Becau se he is fit, his alveolar-arterial oxygen d ifference is p robably less than 10 m m H g, so he shou ld avoid ascend ing above 10,000 feet to m aintain h is a r ter ia l o xy g en co n cen tr a tio n a b o v e 45 m m H g. (5:459) 35. (D ) Atracu riu m has fou r chiral centers and therefore there are 16 p ossible isom ers. The m arketed prod u ct contains a mixture of 10 isom ers, each w ith d ifferent p harmacokinetic and p harm acod ynam ics p rop erties. Cisatracu riu m is one of the isom ers am ong the ten contained in atracu riu m . Even at very large d oses, histam ine release ind u ced by cisatracu riu m is negligible. Atracu riu m is both m ore rap id in onset and shorter in d u ration than cisatracu riu m . (5:498, 503-4) 36. (C) At the tim e of the second heart sou nd , the m itral valve is op ening. The closing of the aortic and p u lm onic valves, isom etric relaxation, and the T w aves are coincid ent. (6:1826) 37. (E) The slope of the response curve is an ind ex of the patient’s sensitivity to carbon d ioxid e. The slop e and p osition m ay change. Increased w ork lead s to a flatter slop e. Anesthetics d iffer in their effect on the cu rve. (5:710)

Answe rs : 26–49

38. (B) The d escend ing m otor tracts are located in the ventral w hite matter in the spinal cord . The p rim ary blood su p p ly is from the anterior sp inal artery. (5:485; 6:3371) 39. (E) When a nond ep olarizing m u scle relaxant is p resent, acetylcholine cannot be m obilized in sufficient quantities to sustain contraction. It is not a resu lt of record ing inabilities or potassium . Cellu lar DN A is not a factor. (1:258-9) 40. (A) Von Willebrand d isease is one of the m ore com m on congenital bleed ing d isord ers and is often d iagnosed in child ren. The d efect is in von Willebrand factor and has an autosom ald om inant transm ission in m ost cases. There is an equ al m ale-to-female d istribution ratio. Von Willebrand factor participates in the process of p latelet ad hesiveness. Thu s, affected patients u su ally have a prolonged bleed ing tim e and a norm al platelet cou nt. (5:209-10; 6:971-2) 41. (E) Excitation of the card iac cell membrane and d ep olarization are accom p anied by calciu m entering the cell. Calciu m ions increase m yocard ial contractile force, p rolong d u ration of systole, increase vascu lar tone, and increase ventricular autom aticity. (1:57-8, 755-6) 42. (D ) All of the listed cau ses of blind ness have been associated w ith p rolonged sp ine su rgery in the p rone p osition, bu t anterior ischem ic optic neuropathy is the most common etiology. (5:895-6) 43. (A) The transd erm al therap eu tic system containing scopolam ine should be applied to the p ostau ricu lar m astoid region w here d ru g absorption is esp ecially efficient. The d ru g is m uch m ore effective in p revention of PON V th an in treatm en t of n au sea once it has occurred . The patch has a d uration of action of abou t 72 h; sid e effects inclu d e d ry m ou th, d row siness, and blu rred vision. (1:233-4, 923; 5:1288-9) 44. (E) In a typ ical anesthesia ventilator, d u ring insp iration, the p ressu re relief valve is closed so that the tidal volume d elivered to the patient

179

is the su m of the set tid al volu me plus the fresh gas flow that occu rs d u ring insp iration. In this case, the total fresh gas flow is 6,000 m L/ m in or 100 mL/ sec. With a ventilator rate of 20/ min, each breath lasts for 3 sec. With an I:E ratio of 1:2, the in sp iratory p h ase lasts for 1 sec. Therefore, the set tid al volu m e is au gm ented by fresh gas flow ing for 1 sec, or 100 m L/ breath. The d elivered tid al volu m e is therefore 150 m L and the m inu te ventilation is 3 L/ m in. (5:650) 45. (D ) With in creasin g age closin g cap acity increases and total lu ng cap acity d ecreases. While total venou s ad m ixtu re increases u nd er general anesthesia m ore in the eld erly d u e to increases in d iffu se V/ Q m ism atch, the tru e intrap u lm onary shu nt increases only slightly w ith age. (5:281) 46. (A) After a p atient has had and recovered from a d ose of su ccinylcholine, it takes a sm aller am ou nt of nond epolarizing d ru g than norm al to establish a neurom u scu lar block. This m ay be d ue to continu ed d esensitization of the end p late. A longer d u ration of the nond epolarization block is also expected . (1:267) 47. (E) The baricity of a local anesthetic w ou ld be the sp ecific gravity of the local anesthetic (nu m erator) d ivid ed by the sp ecific gravity of CSF (d enom inator). A d ru g w ith baricity greater than 1 is consid ered hyp erbaric. (5:794) 48. (D ) Desflu rane, bu t not sevoflu rane, m ay react w ith d ry sod a lim e to yield a p otentially toxic concentration of carbon m onoxid e. The reaction of trichloroethylene, an obsolete anesthetic agent, yield ed p hosgene. (5:614) 49. (A) Blood p ressu re read ings u su ally are falsely high d ue to the d ifficulty in obtaining a su itable cu ff. The cu ff blad d er length shou ld be at least 80%, and the w id th 40%, of the lim b circum ference. Venous and arterial access may be d ifficu lt. Intu bation m ay be d ifficu lt. It m ay be im p ossible to get a su itable m ask fit. N erve blocks m ay be d ifficu lt becau se of the p roblem in find ing land m arks. (5:312)

180

10: P ra ctice Te s t

50. (A) Om ep razole is an inhibitor of the p roton pu mp in the gastric mu cosa and d ecreases gastric acid secretion. It is u sed to treat p ep tic u lcer d isease. (1:1311) 51. (A) Acetazolam id e is an inhibitor of carbonic anhyd rase. It d ecreases the form ation of aqu eou s hu m or and low ers intraocu lar pressure in glau com a. (1:677-8) 52. (B) Glyburid e is an oral hyp oglycem ic agent u sed to treat non-insu lin-d epend ent (type II) d iabetes m ellitu s. (1:1255) 53. (E) SSEPs are affected by nitrou s oxid e to a sm all extent and to a lesser d egree than by volatile anesthetics. Deep levels of isoflu rane w ill d isru p t the tracings. They are not d isru p ted by low er d oses of p rop ofol. The tracings evalu ate latency and am p litu d e. (5:484-8) 54. (B) The vapor p ressu re of sevoflu rane at 20°C is 160 m m H g and that of enflu rane is 175 m m H g. Therefore, the least error in the d elivered concentration of sevoflu rane w ould resu lt by ad m inistering it w ith an enflu rane vap orizer. The sevoflu rane concentration w ou ld thu s be slightly less than that set on the vaporizer d ial. (5:627, 629) 55. (A) Of the options, bupivacaine is the most card iotoxic. Chlorop rocaine is the least card iotoxic d ru g given its rap id m etabolism by esterases in blood . This characteristic m akes it an attractive d ru g to ad m inister ep id u rally in obstetrical p atients. (5:779) 56. (C) All of the options are tru e, if positioning is d one properly, except C. This requires carefu l placem ent of rolls u nd er the bod y in the p rone position so that no pressure is p laced on the abd om en. There is an overall increase in ventilation w ith increased vital cap acity (VC) and FRC in the sitting p osition. (5:362) 57. (C) All of these effects act to increase the difference betw een the partial pressure of the agent in the fresh gas and the partial pressure in the

inspired gas except metabolism since isoflurane is essentially not metabolized . (5:598, 600-1) 58. (D) Low fresh gas flow rate, low minute ventilation, low anesthetic concentration in the fresh gas, and high cardiac output all decrease the rate of onset of an inhaled anesthetic. A low blood:gas partition coefficient, indicative of a low-solubility agent, increases the rate of onset. (5:600-2) 59. (D ) The p atien t w h o d evelop s p u lm onary ed em a in the recovery room u su ally d oes so w ithin the first hou r. Distend ed neck veins or increased CVP m ay be absent, bu t the p atient is u su ally w heezing. An occlu d ed end otracheal tu be m ay be the problem . (5:1288) 60. (B) The number of calories required to raise the tem peratu re of 1 g of a su bstance by 1°C is its sp ecific heat. The sp ecific heat varies for d ifferent su bstances. (5:626) 61. (C) The failing heart attem pts to com p ensate by salt retention and increasing blood volu m e. Myocard ial hyp ertrop hy occu rs, and there is an increased sym p athetic ou tflow to im p rove ou tput. Ventricu lar filling pressu re increases as the heart d ecom pensates. Vasoconstriction is p art of the neu rohu m oral resp onse to heart failu re. (5:176; 6:1906-7) 62. (B) The p in ind ex safety system consists of tw o p ins located on the yoke of the anesthesia m achine. The p osition of the pins is d ifferent for each m ed ical gas. This system p revents the w rong m ed ical gas tank from being hu ng on a p articu lar yoke. Althou gh it is fou nd on the anesthesia m achine, it is not at the site w here gas hoses are connected . (5:618) 63. (D ) A and B have the sam e efficacy becau se they p rod u ce the sam e m axim u m resp onse. Becau se A is located to the left of B on the X-axis, A is said to be m ore p otent than B; A w ill p rod u ce a p articu lar m agnitu d e of effect at a low er d ose than w ill B. The d ose at w hich the half-m axim al response occurs, the ED 50, is low er for A than for B. It is likely that A and B act via the sam e recep tor. (1:44-5)

Answe rs : 50–75

64. (A) The effect of d ru g Y is to shift the d ose– resp onse relationship of X to the right w ith no effect on the m axim u m resp onse to X. This is the d efinition of a com petitive antagonist. It is likely that X and Y act via the sam e recep tors. (1:46-8) 65. (A) Patients w ith end -stage renal d isease are anem ic becau se th e kid ney is the p rim ary sou rce of eryth rop oietin . Th ey h ave an increased bleed ing tim e d u e to p latelet d ysfu nction, bu t the p latelet cou nt is not necessarily abnorm al. Electrolyte abnormalities includ e h yp er kalem ia an d h yp erp h osp h atem ia. (6:2310-2) 66. (C) The transp lanted heart has intact α- and β -ad renocep tors. The Frank–Starling effect is intact. Atrop ine w ill not have any effect since there is neith er au tonom ic in nervation n or circu lating cholinergic agonist. (5:1093) 67. (E) CSF flow s through the cerebral ventricles and the su barachnoid sp ace before absorp tion in the arachnoid villi. There is no CSF in the epid u ral sp ace. (5:871-3; 6:3435) 68. (A) Verapamil is a calcium channel blocker that is u sefu l in the treatm ent of su p raventricu lar tachycard ia. It is u sefu l in the p atient w ith asthm a bu t shou ld not be com bined w ith prop ranolol, since it m ay cau se p rofound brad ycard ia. The d rug causes vasod ilatation. It slow s the rate in pacem aker cells. (1:757-9) 69. (D ) When the d ial on Tec 5 vap orizer is in the “off” position, no fresh gas flow s through the vap orizer. When the oxygen flu sh valve is p ressed , the d elivered oxygen has not p assed through the vap orizers. Therefore, w hen there is concern that liqu id anesthetic has entered the gas d elivery system , the vaporizer should be flushed w ith oxygen at 10 L/ m in for 10 m in w ith the vap orizer d ial set to its m axim u m valu e. (5:629) 70. (D ) Althou gh ASA I statu s is a pred ictor of brad ycard ia follow ing spinal anesthesia, the m ost important pred ictor is preexisting brad ycard ia. (5:801)

181

71. (C) There is a grow ing consensu s that the concern over hyp okalem ia is u nfou nd ed . If the likely cau se of the hyp okalem ia is know n and the patient is asymptom atic, there is no need to cancel the proced u re in an otherw ise healthy p atient. Intraop erative infu sions m ay cau se hyp erkalem ia, and rap id ad m inistration of p otassiu m m ay resu lt in asystole. It is not p ossible to replenish the intracellular potassium over a short period of tim e. (5:518) 72. (A) In tracran ial p ressu re is rou tin ely m on it o r e d b y t r a n s d u cin g a s m a ll ca t h et e r in trod u ced into the cerebral ventricle or subarachnoid space. Sim ilarly, a hollow bolt w ith p ressu re tran sd u cer attach ed or a boltm ou nted transd u cer can be p laced into the su bd u ral sp ace. Alternatively, ICP can be m easu red by placing a pressure transd u cer into the ep id u ral sp ace. A catheter im p lanted d irectly into brain parenchyma is not used ; the Cam ino Bolt u ses a fiberop tic cable p laced on the brain p arenchym a. (5:875; 6:2256) 73. (C) All gases and vap ors obey Dalton’s law. Each gas contribu tes to the total p ressu re in an am ou nt that is p rop ortional to its m olecu lar fraction. Dalton’s law states that each gas exerts the sam e p ressu re that it w ou ld if it alone occu p ied the container. Solu bility plays a role in d issolved gases. (5:626) 74. (C) Metoclopramid e has antagonistic effects on d op am ine D 2 as w ell as 5-H T3 recep tors. Its m ain pharm acologic action is 5-H T4 recep tor activation. The m ain ind ication for m etoclop ram id e is the am elioration of nau sea and vom iting that often accom p anies GI d ysm otility synd rom es. (1:1325) 75. (D ) H em ochrom atosis and Wilson d isease resu lt in cirrhosis d u e to the accu m ulation of excessive am ou nts of iron and cop p er, resp ectively, in the liver. α 1-antitryp sin d eficiency causes bullous emphysema as w ell as cirrhosis. Cirrhosis can occu r in p atients w ith cystic fibrosis. Antithrom bin III d eficiency is a d isord er of the clotting cascad e that results in excessive throm bosis. (6:2597)

182

10: P ra ctice Te s t

76. (B) In hyp ovolem ic states, the pulse pressu re is narrow ed . H eart rate is increased to m aintain card iac output, and the neck veins are flat. Urine volu me is d ecreased to preserve volu me. The m u cou s m em branes are p ale, reflecting low er blood flow to p erip heral areas. (5:1398) 77. (D ) Most clinicians w ou ld classify a p atient w ith a history of m yocard ial infarction as ASA physical statu s III at the m inim u m , regard less of the d egree of recovery from the MI. In this case, “E” is ad d ed to the p hysical statu s becau se it is an u rgent and u nsched u led p roced u re. (5:54) 78. (C) Am iod arone is u sefu l for both atrial and ventricu lar arrhythm ias. Am iod arone has a half-life of m any w eeks, therefore stop p ing it before su rgery w ou ld have little effect. It is eliminated through the liver. Autonomic effects of am iod aron e in clu d e a n on com p etitive β -ad renergic receptor blockad e. (1:834-7) 79. (A) There are several rad icu lar arteries that branch from intercostal and lu m bar arteries to anastom ose w ith the anterop osterior sp inal artery system to su p ply the spinal cord w ith blood . Th e arteria rad icu laris m agn a (of Ad am kiew icz) is the largest of these 4 to 10 rad icu lar branches. Its origin m ay be either su pra- or infrarenal. (5:1573) 80. (C) The latent heat of vap orization is the nu m ber of calories need ed to convert 1 g of liqu id into vap or at a constant tem p eratu re. This valu e is d ep end ent on the am bient tem peratu re. The cold er the liqu id , the m ore calories need ed to vap orize a given am ou nt of liqu id . (5:626) 81. (C) DLCO d ep end s on d iffu sion of CO across p erfu sed alveoli to be avid ly bound to hem oglobin. Smoking falsifies the test result through exogenou s CO u p take. Exercise, congestive heart failure, and increased hem oglobin prom ote CO u ptake, w hile fibrosis im ped es d iffusion an d p u lm on ary em bolism d ecreases effective gas transfer su rface. (5:953)

82. (B) Cocaine is the only local anesthetic that blocks the reu p take of norep inep hrine. The ad d ition of ep inep hrine to a local anesthetic solu tion d oes not affect norepinephrine reu ptake. (1:278) 83. (B) An α-ad renocep tor antagonist, su ch as p henoxybenzam ine or prazosin, is begu n one to tw o w eeks p rior to su rgery. Once blood pressure control is achieved , if tachycard ia persists, a β -ad renocep tor antagonist is ad d ed . Ad ministration of a β-ad renoceptor antagonist p rior to ad equate α-ad renergic blockad e m ay resu lt in w orsen in g of th e h yp erten sion . Becau se p atients w ith p heochrom ocytom a are hyp ovolem ic, d iuretics are contraind icated in the control of the hyp ertension. The α- and β -ad renocep tor antagonists are titrated to blood p ressu re and heart rate control, and d o not d ecrease the u rinary excretion of catecholam ine m etabolites. (5:158-9) 84. (D ) An accessory cond u ction p athw ay is evid enced by the w id e QRS com p lex and its slurred upstroke (d elta w ave). The patients are p rone to d evelop atrial fibrillation or p aroxysm al supraventricular tachycard ia. (6: 1889) 85. (A) Atracu riu m has both p re- and p ostju nctional effects. H istam ine release is m ore likely only w ith higher d oses (> 0.5 m g/ kg) injected rap id ly (in less than a m inu te). Lau d anosine has cau sed seizu res at high p lasm a levels in experimental animals, but this problem has not been observed in hu m ans. The effect of atracu riu m on blood p ressu re is u su ally m inor, bu t hyp otension m ay occur if a large d ose is given rap id ly. (5:503) 86. (A) The glycop rotein is a recep tor for fibrinogen or von Willebrand factor, p roteins w hich, in tu rn, p rovid e crosslinks to hold p latelets togeth er in an aggregate. Diverse p lateletactivating stimuli operate via the glycoprotein. Abcixim ab is an inhibitory m onoclonal antibod y, and ep tifibatid e is an inhibitor p ep tid e originally fou nd in snake venom . (6:991-2)

Answe rs : 76–86

87. (A) Cerebrospinal fluid pressure is increased w ith cou ghing. Cou ghing increases intrathoracic an d in traabd om in al p ressu re th at is transm itted to the cranial vau lt. Maneu vers th at d ecrease in trath oracic p ressu re w ill d ecrease intracranial p ressu re. (5:880-3) 88. (D ) Isoflu rane is a better m u scle relaxant w hen com pared to halothane. The vap or pressu re is 238 m m H g at 20°C. Isoflu rane d ep resses ventilation and shou ld be d elivered by assisted or controlled ventilation. The value for MAC is correct. Card iac output u nd er isoflurane anesthesia in clinically relevant d oses is w ell m aintained . (1:544; 5:598) 89. (D ) Basilar alveoli expand more than the apical ones, but in their end position are abou t the sam e size. Resid u al volu m e is ind epend ent of th e state of in sp iration . Pleu ral p ressu re d ecreases, balancing the increased elastic recoil of the lu ng. Pu lm onary vascu lar resistance is low est at FRC. Becau se of airw ay resistance, p roxim al airw ay p ressu re w ill alw ays be greater than alveolar p ressu re d u ring insp iration w hen flow is present. (5:470; 6:2084) 90. (E) Prop ofol is a p otent m etabolic su pp ressant. Since flow -metabolism coupling is maintained , there is a significant red u ction in cerebral blood flow, blood volu m e, and ICP. Prop ofol, as d o m ost anesthetic agents, increases the latency of SSEP’s. (5:688-92, 875) 91. (A) H ep arin increases the activity of antithrom bin III that neu tralizes the activated form s of factors II, IX, X, XI, XII, and XIII and of kallikrein. The effect of hep arin is estim ated by the activated p artial throm bop lastin time. It m ay be given su bcu taneou sly and it m ay be given w hile w aiting for the effect of an oral anticoagulant to occur. (1:853-5) 92. (C) Most hem olytic transfu sion reactions are cau sed by clerical errors in w hich ABOincom p atible red cells are transfu sed . Many of the com m on signs of a hem olytic transfu sion reaction (fever, chills, ch est p ain) m ay be m asked by general anesthesia, and hem oglobinu ria and coagu lop athy are the signs m ost

183

likely to be noted . The severity of the reaction is d irectly related to the volume of transfused cells, so immediate d iscontinuation of the transfusion is mandatory if a hemolytic reaction is su spected . Only a small percentage of hem olytic transfusion reactions are caused by the failure to d etect incompatibility d uring serologic testing. Shock-ind uced renal failu re and d issem inated intravascu lar coagu lation convey most of the morbid ity and mortality. (5:1441) 93. (A) The tetralogy of Fallot includ es a VSD, pulm onary ou tflow obstru ction, overrid ing of the aorta, and right ventricu lar hyp ertrophy. An ASD is not part of the com plex. (6:1926-7) 94. (C) The oxygen pressu re at the w all connector is abou t 50 p si. The p ressu re in fu ll E cylind ers containing oxygen or nitrou s oxid e is abou t 1900 p si or 750 p si, resp ectively. (5:619) 95. (B) The p atient d escribed in this vignette m ost likely su ffers from the consequ ences of chronic cocaine abu se. Cocaine cau ses the release of catecholam ines and p revents their reu p take, thus raising the risk for hypertension, m yocard ial ischem ia, and card iom yop athy, as w ell as aortic d issection am ong m any other com p lications. Exogenou sly ad m inistered catecholam ines given intraop eratively can have an exaggerated effect. β -ad renergic antagonists are given for cocaine-ind u ced arrhythm ias, how ever in the acu tely intoxicated p atient, com bined α-, and β -blockad e w ith labetalol m ay be p referable to selective β -blockad e, so as to avoid u nop posed α-effects. There is no reason to w ithhold op ioid s for analgesia, or benzod iazep ines for sed ation. Ep hed rine is an ind irect vasoconstrictor, enhancing release of norep inep hrine from sym p athetic neu rons. It m ay be in effective in treatin g h yp oten sion in the cocaine abu ser d u e to catecholam ine d ep letion; phenylephrine how ever w ill remain effective. (1:300; 5:324-5) 96. (D ) The Bernou lli theorem m ay be u sed to estim ate the p ressu re grad ient across a stenotic valve: ΔP = 4(V 22 − V 21)

184

10: P ra ctice Te s t

w here ∆P is the pressure grad ient, and V1 and V2 are the blood flow velocities on either sid e of the stenotic valve. (5:432) 97. (A) Exp osu re to tobacco either d irectly or th rou gh secon d -h an d sm oke in creases th e risk of m any p eriop erative com p lication s. Pharm acologic interventions to assist in sm oking cessation su ch as varenicline and bu p ropion should be started one to tw o w eeks before a qu it attem pt bu t nicotine replacem ent therapy is effective im m ed iately. (5:66-7) 98. (D ) The oxygen concentration in the blood is d eterm ined by the concentration in the alveoli, the efficiency of the lu ngs, and oxygen consumption. Decreasing shunting by using PEEP or reverse Trend elenbu rg p ositioning in the obese can red u ce alveolar to arterial oxygen tension d ifference. Calciu m channel blockers m ay d ecrease hyp oxic vasoconstriction and thus increase shunting. (5:312, 969) 99. (A) Ketam ine can lead to an increase in CBF by virtu e of increasing CRMO 2. At levels below 1.5 MAC, the net effect of halogenated hyd rocarbons is a d ecrease in cerebral blood flow. All other agents d ecrease CBF. (5:873-4) 100. (C) Pu lse p ressu re variability is a fu nction of the m axim u m and m inim u m valu es for pu lse pressure d uring a positive pressure breath. The higher the valu e, the greater the d egree of hyp ovolem ia, and the greater the exp ected resp onse to ad m inistered flu id . (5:425) 101. (E) Benzocaine is a local anesthetic com m only u sed as a topical analgesic, or in cou gh d rops. Benzocaine is the ethyl ester of p-aminobenzoic acid (PABA); it can be p rep ared from PABA and ethanol. It is a common cau se of methemoglobinem ia. It is available only as a top ical prep aration. (5:782) 102. (D ) Acromegaly is d u e to the hypersecretion of grow th horm one. H yp ertrop hy of skeletal and connective tissu e, esp ecially of the face and head , m ay m ake intu bation d ifficu lt. Diabetes m ellitu s and hypertension are also com m on. The p itu itary ad enom as u su ally resp onsible

for acromegaly generally d o not secrete TSH or cau se hyperthyroid ism . (5:159) 103. (C) Calciu m channel antagonists shou ld not be stop p ed in the p reop erative p eriod becau se of the p ossibility of w orsening ischem ia or hyp ertension. In p atients u nd ergoing inhalational an esthesia, th ey p rolon g atrioventricu lar con d u ction tim e, w h ile in p atien ts u n d ergoing a h igh d ose op ioid anesth etic, th ey h ave little effect on card iac ou tp u t. Calciu m ch an n el blockers also p oten tiate n eu rom u scu lar blocking agents. Unlike the β -blockers, effect sites an d m ech an ism s of action vary, and th ey cann ot be u sed in terchangeably. (5:754-7) 104. (C) The hyoid bone susp end s the thyroid cartilage by the thyrohyoid m em brane, at the level of C4. The Ad am s’s ap ple is on the thyroid cartilage. The cricothyroid m em brane at the level of C6 is an easily accessible, relatively avascular structure for cricothyrotomy or injection of local anesthetics for fiberoptic intubation. To each sid e of the larynx and inferior to the aryep iglottic fold s is the p iriform sinu s. The end otracheal tu be shou ld be p laced in the trachea, w hile the tip of the laryngoscop e should be placed in the glossoepiglottic reflection. (5:548, 563) 105. (C) The irrigating flu id s shou ld be nonhem olytic and isosm olar. The com p osition shou ld not be close to w ater, since w ater is hyposmotic. Electrolyte solu tion s sh ou ld n ot be u sed becau se they cond u ct electricity and therefore interfere w ith the electrocau tery. (5:1140) 106. (D ) Increased d ead sp ace ventilation im p ed es CO 2 elim ination. Exhau sted sod a lim e is not cap able of rem oving all CO 2 from the resp iratory circuit. A pulmonary embolus or increased VD / VT w ill cau se end -tid al CO 2 to fall w hile cau sing hypercarbia. The earliest signs of MH are hyp ercarbia, sinu s tachycard ia, and m asseter m u scle rigid ity. H yp erventilation cau ses hyp ocarbia. (5:460, 640, 1491) 107. (B) N aloxone m ay reverse the effects of opioid agonists su ch as fentanyl and bu torp hanol,

Answe rs : 87–115

and flu m azenil m ay reverse the effects of a benzod iazep ine su ch as m id azolam . There is no sp ecific antagonist for the hypnotic effects of p ropofol, thiopental, or etom id ate. The only sp ecific antagonist for steroid al m uscle relaxants is su gam m ad ex, a γ -cyclod extrin com p ou nd th at encap su lates rocu roniu m and vecuronium , thu s rem oving the d ru g from the neurom u scu lar ju nction. As of the end of 2012, it is not ap p roved for u se in the U.S. (1:1059; 5:507) 108. (C) This patient has the typical list of problems. A m ainstay of therapy is to lim it the m ism atch of m yocard ial oxygen su pply and d em and by low ering the p ressu re, tem p eratu re, and heart rate. Ad m inistration of oxygen w ill increase the su pply. (5:902-3) 109. (D ) A cyclic-GMP p hosp hod iesterase inhibitor has a positive inotropic effect and w ou ld low er system ic vascu lar resistance. It w ould only be u sed in a p atient w ith a low flow state d u e to low card iac ind ex or card iogenic shock. A balloon p u m p is exp ected to red u ce m yocard ial d em and and im p rove su p p ly. As a coronary throm bosis has probably occurred , fibrinolytics (not antifibrinolytics) and anticoagu lants m ay help. (5:97, 2015-20) 110. (C) Inability to void is not alw ays a contraind ication to d ischarge to hom e, bu t since inguinal hernia rep air has a high incid ence of urinary retention, su ch patients should void before d ischarge. H is heart rate, blood p ressu re, and oxygen saturation are w ithin reasonable limits. It is not u nu su al for a p erson w ho has ju st u nd ergone hernia repair to require assistance w ith am bu lation. (5:1278-84) 111. (C) Pulse oxim eters use d u al-w avelength spectroscop y to m easu re th e concen tration s of oxyhem oglobin and d eoxyhem oglobin. The p resence of a significant concentration of m ethem oglobin introd u ces su bstantial error into the m easurem ent. Interestingly, high concentrations of fetal or sickle hem oglobin or of biliru bin d o not ad versely affect the accu racy of the m easu rem ent. (5:462)

185

112. (D ) An alarm of the line isolation monitor most commonly immed iately follow s the connection of a faulty electrical d evice. Disconnecting that d evice from the circu it is the initial step and m ost likely w ill resolve the alarm cond ition. (5:376-8) 113. (D ) The meter on the line isolation monitor d isplays the maximum potential current that could flow throu gh the p atient if another fau lt w ere to occu r. Since the likelihood of su ch an occu rrence is rare in the short term , the op eration shou ld proceed . Of course the reason for the fau lt cond ition shou ld be investigated m ore thorou ghly after the operation is com p leted . (5:376-8) 114. (E) The p resenting sym p tom s of LAST are isolated CN S sym p tom s in abou t 45% of cases, com bined CN S and CV sym p tom s in abou t 44%, and isolated CV sym p tom s in 11% of cases; the latter occu r by interference w ith sod ium cond uctance. The card iac toxicity w ith bu p ivacaine is of longer d u ration as com p ared to other local anesthetics and therefore hard er to treat. Rapid recognition and immed iate sup p ortive care as d escribed in E are param ou nt. In som e instances, infu sion of lip id em u lsion m ay be beneficial by virtu e of neutralizing circu lating local anesthetic d ru g. It shou ld be ad m inistered as a bolu s, follow ed by continu ous infusion. In cases of LAST, propofol should be avoid ed in p atients w ith hyp otension, as shou ld vasopressin, β -blockers, calciu m channel blockers, and local anesthetics. CN S sym p tom s such as seizu re activity should be treated w ith injectable benzod iazep ines. (1:565, 571; 5:777-80) 115. (D ) The principal d isad vantages of m ethohexital are p ain on in jection and in volu n tary m uscle m ovem ents. The pH of the solu tion is high and it is w ater-solu ble. Propofol has card iod ep ressant effects far greater than those associated w ith ad ministration of barbiturates. Because it low ers the seizure threshold , methoh exital is a good choice for anesthesia in patients und ergoing electroconvulsive therapy. (1:535-7; 5:693-6)

186

10: P ra ctice Te s t

116. (B) Carbon d ioxid e is inversely proportional to ventilation, i.e., the higher the ventilation, the low er the CO 2. CO 2 w ill equ ilibrate w ith bicarbonate, acting as an acid . Red u cing ventilation lead s to resp iratory acid osis, increasing ven tilation lead s to resp iratory alkalosis. Ventilation w ill increase in resp onse to end ogenous or exogenou s CO 2. (5:461) 117. (E) Fat em boli cau se m echanical blockage, and then end othelial d am age from free fatty acid breakd ow n p rod u cts. Th e p u lm on ary and cerebral vascu lar bed s can be involved . The ECG m ay show right-axis d eviation or right bu nd le branch block. (5:1203) 118. (C) Porp hyrias are a grou p of rare inherited d isord ers in w hich a sp ecific enzym e d eficiency lead s to errors in the biosynthesis of hem e. Drugs that ind uce cytochrom e enzym e p rod u ction m ay cau se an acu te p orp h yric attack and shou ld be avoid ed . These inclu d e etom id ate and thiopental. (5:141-2, 695) 119. (A) The p atient w ith m yasthenia gravis has w eakness in the m u scles innervated by cranial nerves and often experiences ophthalm oplegia and p tosis. Althou gh the accessory m u scles of resp iration m ay also be w eak, d iap hragm atic w eakness is u ncom m on. These p atients are u p to 100 tim es more sensitive to nond epolarizing m u scle relaxants as com p ared to u naffected ind ivid u als, and resistant to the effects of su ccinylcholine. Myasthenia gravis is exclu sively a m otor d isease. The m yasthenic synd rom e (Eaton-Lam bert-synd rome), an acqu ired au toim m u ne d isord er of the neu rom u scu lar ju nction, is often associated w ith carcinom as. (5:142-3) 120. (C) The tw o vertebral arteries join to form the basilar artery. This u np aired m id line stru ctu re gives rise to m any branches and p erforating arteries that su p p ly the brain stem and the cerebellum . Transient ischemic attacks or strokes of the basilar arterial system resu lt in unconsciou sness from loss of reticu lar activating system fu nction. In ad d ition, m otor fu nction can be lost from ischemia of the cerebral pyram id s

that contain long motor tracts to the brain stem nu clei and sp inal cord . (5:872; 6:3286-7) 121. (E) Packed red cells are mixed w ith an ad d itive nu trient containing ad d itional glu cose and ad enine in ord er to extend the storage p eriod to 42 d ays. (5:1439) 122. (E) FEV1 d ecreases w ith increasing age, in sm okers even m ore than in non-sm okers. A p atient’s risk for PON V is increased in nonsm okers. (5:113, 133, 1255) 123. (B) Only the S-isom er of rop ivacaine is p resent in solu tions for clinical u se, in contrast to other optically active local anesthetics that are su pp lied as racem ic m ixtu res. The S-form app ears to be less toxic, m ore potent, and longer acting that the R form , or the racem ic m ixtu re. Rop ivacaine is m u ch less toxic than bu p ivacaine and prod uces m otor blockad e that is of shorter d uration than sensory blockad e. Like other am id e local anesthetics, rop ivacaine is m etabolized in the liver. (5:768, 772, 779, 781) 124. (B) Sw eating is an im p ortant tem p eratu re-regu latin g m ech anism that cools the bod y. It occu rs in resp onse to sym p athetic nervou s system -m ed iated stim u lation of the sw eat gland s by cholinergic neu rons and can resu lt from a num ber of p hysiological p ertu rbations that increase sym p athetic activity, inclu d ing hyp oglycem ia, em otional stress, and increased bod y tem p eratu re from infection. (5:1506-8; 6:165, 2883) 125. (E) In the p atient u nd er general anesthesia, core tem p eratu re w ill tend to d rift tow ard am bient tem p eratu re. It is im p ortant to take steps to both prevent heat loss and to actively treat hypotherm ia. The problem is especially com m on d uring su rgery involving open bod y cavities. H ypothalam ic regulation and m echan ism s to con serve an d gen erate h eat are obtund ed by general anesthesia. Skin temperature unreliably reflects core tem perature und er general anesthesia (5:1506-8)

Answe rs : 116–137

126. (C) Regu lation of bod y tem p eratu re is a com p lex m echanism involving heat prod u ction by m etabolism w ithin m u scles and liver and heat loss from skin and lu ngs. The cond u ct of anesthesia d isru pts this system by vasod ilation of the skin, p reventing shivering w ith m u scle relaxan ts, an d gen eral su p p ression of th e hypothalam ic fu nction. (5:1506-8; 6:165, 2883) 127. (A) Transthoracic p ressu re is the su m of transpu lm onary pressure and chest w all transmu ral p ressu re. Transp ulm onary p ressu re is the grad ient from alveoli to p leu ra, and transm u ral p ressu re is the grad ient from pleura to am bient. Esop hageal balloon p ressu re is a m easu re of p leu ral pressu re. Pu lm onary artery occlu sion pressure (or w ed ge pressure) is a m easure of left atrial pressu re. (5:470) 128. (B) In the grap h, the blood level of d ru g A d ecreases to nearly zero betw een d oses. Dru g B show s increasing levels over a p eriod of tim e, i.e., it accu m ulates. Dru g B never reaches stead y state concentrations on this grap h. (5:32-5) 129. (D ) The grap h show s that d ru g B is not com p letely elim inated before th e next d ose is given. The relative size of the d ose cannot be d eterm ined from the d iagram , bu t each of the d oses is the sam e size. The half-life of d ru g A is not long, and its d u ration of action is shorter than B. (5: 32-5) 130. (C) Resistan ce d ecreases w ith in sp iration becau se the airw ays d ilate. Parasym p athetic stimulation, acetylcholine, and smoke constrict the airw ay. (5:470) 131. (B) Mannitol is an osm otic d iu retic in th e kid ney that d oes not cross the blood brain barrier and d raw s w ater ou t of brain tissu e from w hich it is exclu d ed . Rap id ad m inistration can p rod uce transient peripheral vasod ilation and h yp oten sion. It red u ces m ean corp u scu lar volu m e sim ilar in m echanism to red u cing brain volu m e bu t this has no effect on cerebral ed em a. There is no d iu retic effect in renal failu re in w hich it is contraind icated . (5:880; 6: 340-50)

187

132. (B) Patients w ith sickle cell d isease shou ld be treated to p reven t p eriop erative h yp oxia, d ehyd ration, acid osis, and hypotherm ia, all of w hich can resu lt in a vasoocclu sive crisis. General anesthesia p er se d oes not overtly increase the risk of sickling. (5:206) 133. (E) The typ ical tim e it takes for m ethim azole to rend er a p atient eu thyroid is abou t 4-8 w eeks. This p atient w ou ld p robably still have tachycard ia. If a regional anesthetic is feasible, it m ight be safer for the inad equ ately treated hyp erthyroid patient. (5:151-2) 134. (E) General anesthesia m ay be reversed by increasing the am bient p ressu re, regard less of the agent u sed to prod u ce the anesthetic state. Som e of the general anesthetics that are not gases at bod y tem p eratu re inclu d e halothane, isoflu rane, and thiop ental. While the p ertu rbation of m em brane lip id -protein interactions is an attractive hyp othesis for general anesthetic action, m ost anesthetic agents have not been stu d ied in term s of this action. While isoflurane, sevoflu rane, and halothane are bronchod ilators, nitrou s oxid e has little effect on airw ay resistance, and d esflu rane m ay cau se bronchoconstriction. By irreversibly inhibiting m ethionine synthase, nitrou s oxid e inhibits m any biochem ical reactions, inclu d ing DN A synthesis. (5:588, 590-2, 598, 613) 135. (B) The inotrope m ilrinone is a selective phosp hod iesterase-3 inhibitor. It cau ses an increase in cyclic AMP and vasod ilatation, and has no effect on β 1-recep tors. Bosentan is an end othelin-antagonist used in the treatment of primary p ulm onary hyp ertension. Inhibition of N a,KATPase is the principal effect of d igoxin. (1:801, 805, 1059) 136. (E) This cu rve is characteristic of obstru ctive lu ng d isease, w ith a higher resid u al volu m e and scooping of the expiratory limb from small airw ay closure. (5: 997; 6:2089) 137. (B) This cu rve is characteristic of restrictive lu ng d isease, w ith sm aller volu m es and w ell p reserved flow s relative to volu m e. (5: 997; 6:2089)

188

10: P ra ctice Te s t

138. (C) Thiop ental is contraind icated in p atients w ith acu te interm ittent p orp hyria. The other porphyrias listed as op tions are not associated w ith enzym e ind u ction. (5:697) 139. (A) Chem orecep tor stim u lation lead s to an increased ventilatory effort. Cyanosis requ ires at least 5 g/ d L d eoxyhem oglobin in the blood to be ap p arent. Severe hypoxem ia can cau se brad ycard ia, in ad d ition to a rapid fall in blood pressu re and circu latory collap se. (5:249-51) 140. (A) Dop am ine is not selective for any single ad renergic recep tor; rather, the net effect of d op am ine is d ose-d ep end ent. Dop am inergic recep tors are activated by the low est d oses, w hile β -ad renergic, and then α-ad renergic, receptors are activated as the d ose is increased . Dop am ine in high d oses is a p oor choice for the p atient w ith p rim ary card iac contractile d ysfu nction becau se of p red om inant vasoconstriction, increased afterload , and resu lting w orsening of left ventricu lar p erform ance. (1:804) 141. (E) Ketam ine has sym p athom im etic activity th at can reverse bron ch osp asm . Atrop in e cau ses bronchod ilatation by its m u scarinic antagonism , blocking the action of the vagu s on bronchial sm ooth m uscle. All potent inhalation agents block the bronchial resp onse to m ed iators of bronchoconstriction, althou gh in sp ontaneou sly ventilating patients, the respiratory d epression w ill cau se low er lu ng volu m es. Therefore ventilation w ill occur at a less compliant range of the pressure–volume curve. Thiop ental has no significant effect on bronchial tone. (5:309, 612, 699, 977) 142. (A) Trad itionally su p raclavicu lar blocks carried a relatively high risk of a p neu m othorax, given the p roxim ity of the lu ng to the brachial p lexu s. This risk has been rep orted to be as high as 5% in some stud ies. Hemidiaphragmatic p aresis w ou ld be a less likely cau se in an 18-year-old athlete w ith no p rior p u lm onary p athology. (5:833)

143. (E) Interscalene blocks have trad itionally carried a 100% incid ence of hem id iap hragm atic p aresis from p hrenic nerve blockad e. Phrenic nerve blockad e may red uce one’s vital capacity by 40%. Dep end ing on the severity of lu ng d isease, patients w ith COPD m ay experience p rofound shortness of breath after an interscalene block. (5:833) 144. (B) The nipp le line correlates to 4th thoracic d erm atom e (T4). (4:198) 145. (E) Blad d er sensation is cond u cted by sym pathetic afferent fibers of the hypogastric plexus that originate from T11-L2. Therefore a T10 sensory level is required to avoid the sensation resu lting from blad d er d istension from irrigation flu id . The u m bilicu s correlates to a T10 level block. (4:198; 5:1141) 146. (F) Post-infarction VSD requ ires p rom p t su rgery bu t can be stabilized w ith the aid of a balloon p u m p . The IABP su p p orts d iastolic p erfu sion p ressu re and red u ces afterload . The latter action red u ces the left-to-right shu nt. (6:2235-6) 147. (H) Remote rheumatic fever can result in mitral stenosis. Antibiotic therap y of strep tococcal infections has red u ced the incid ence. (6:2752) 148. (I) Sp ontaneou s p neu m othorax of id iop athic etiology rarely cau ses tension p neu m othorax. (5:979-80) 149. (E) An arterial anastom osis su tu re line m ay be bleed ing. A ligated branch of a vein graft m ay have op ened . Su d d en shock after card iac su rgery can p rom p t em ergency op ening of the chest. In event of tam p onad e, op ening the chest m ay rap id ly im p rove hem od ynam ics u ntil control of bleed ing is achieved . (5:457-8) 150. (B) Left heart failu re before transp lantation tend s to increase p u lm onary vascu lar resistance. A healthy transplanted heart m ay suffer right-sid ed failu re in initially cop ing w ith the chronically insu lted lung. (5:1083-4)

PART II

Advanc e d To pic s in Ane s the s io lo g y

This page intentionally left blank

CHAPTER 11

Pharmac o lo g y Que s tions DIRECTION S (Qu estions 1-63): Each of the nu m bered item s or incom p lete statem ents in this section is follow ed by answ ers or by com p letions of the statem ent. Select the ON E lettered answ er or com p letion that is BEST in each case. 1. A child is brou ght to the em ergency d epartm ent for evalu ation of his d isoriented state. This began shortly after he ate som e berries in the gard en. H e is noted to be very w arm , flu shed , and to have d ilated p u p ils. The m ost ap prop riate action inclu d es ad m inistration of (A) (B) (C) (D) (E)

atrop ine asp irin d ip henhyd ram ine p hysostigm ine p henylep hrine

2. Flu oxetine is u sed for the treatm ent of (A) (B) (C) (D) (E)

influ enza m ajor d ep ressive d isord er Graves d isease Parkinson d isease H od gkin d isease

3. A 52-kg p atien t u n d ergoes an esth esia for rem oval of a bronchial carcinom a. H is anesthesia consists of p rop ofol, isoflu rane, and N 2O/ O 2, in ad d ition to 50 m g of rocu roniu m after intu bation. At the end of the p roced u re 3 h later, only one w eak tw itch is p alp able on train-of-fou r stim u lation , and the p atient rem ains ap neic. The m ost likely exp lanation for ap nea in this p atient is

(A) rocu roniu m overd ose (B) d elayed p u lm onary elim ination of nitrou s oxid e after lu ng su rgery (C) im p aired release of acetylcholine from the nerve term inal (D) hyp oxic p u lm onary vasoconstriction (E) resp iratory d ep ression from resid u al effects of p rop ofol 4. Dobu tam ine (A) (B) (C) (D)

is p rim arily an α-ad renocep tor agonist has p rim arily β 1-ad renocep tor effects cau ses d ecreased renal blood flow is associated w ith severe increases in heart rate (E) is a natu rally occu rring catecholam ine

5. A patient is scheduled for an elective cardioversion. A single intravenous injection of a shortacting agent is p lanned . Which one of the following agents is most likely to cause vomiting after the procedure when used in this patient? (A) (B) (C) (D) (E)

Ketam ine Thiop ental Etom id ate Methohexital Prop ofol

6. Esm olol is inactivated by (A) (B) (C) (D) (E)

m onoam ine oxid ase catechol O-m ethyltransferase erythrocyte esterase p lasm a p seu d ocholinesterase acetylcholinesterase

191

192

11: P ha rma cology

7. An 18-year-old patient w ith a history of asthma is brou ght to the op erating room for em ergent lap aroscop ic ap p end ectom y for p erforated ap p end ix. Shortly after ind u ction of general end otracheal anesthesia, the p atient exp eriences severe bronchosp asm . Which one of the follow ing d ru gs, ad m inistered as m onotherapy w ith a pressurized m etered -d ose inhaler via a sp acer cham ber throu gh the ventilator circu it, is m ost likely to resu lt in bronchod ilation in the acu te setting? (A) (B) (C) (D) (E)

Ip ratrop iu m brom id e Albu terol Theop hylline Terbu taline Salm eterol

8. A 55-year-old w om an sched u led to u nd ergo carotid end arterectom y has been sm oking heavily for over 40 years and w ishes to qu it. Which one of the follow ing m ed ications is best at alleviating the sym p tom s of nicotine w ithd raw al? (A) (B) (C) (D) (E)

Bu sp irone Alp razolam Bu p rop ion Zolp id em Citalop ram

9. Botu linu m toxin interferes w ith neu rom u scu lar transm ission by (A) (B) (C) (D) (E)

p reventing synthesis of acetylcholine p reventing breakd ow n of acetylcholine p reventing storage of acetylcholine p reventing release of acetylcholine blocking recep tors for acetylcholine

10. A 62-year-old m ale w ith a history of d iabetes m ellitu s, hyp ertension, and coronary artery disease is ad mitted to the hospital for unstable

angina. H e w as recently d iagnosed w ith hep arin-ind u ced throm bocytop enia typ e II and is n ow sch ed u led to u n d ergo off-p u m p coronary artery byp ass grafting for severe threevessel coron ary artery d isease. A su itable agen t for in traop er ativ e an ticoagu lation w ou ld be (A) (B) (C) (D) (E)

hep arin w arfarin argatroban fond ap arinu x clop id ogrel

DIRECTION S: Use the follow ing scenario to answ er Qu estions 11-12: A 61-year-old m ale w ith a past m ed ical history of d iverticu losis is ad m itted to the intensive care u nit after u nd ergoing an em ergent lap arotom y w ith sigm oid colectom y and creation of H artm an’s p ou ch for p erforated d iverticu litis. Accord ing to the su rgeon, there w as frank contam ination of the p eritoneal cavity. Despite broad -spectru m antibiotic coverage, the p atient d evelop s sym ptom s consistent w ith sep tic shock on the first p ostoperative d ay. 11. Desp ite aggressive flu id resu scitation an d blood p ressu re su p p ort w ith norep inep hrine, the p atient rem ains hypotensive. A 2-D echocard iogram is obtained that show s no w all m otion abnorm alities, a hyp erd ynam ic left v en tricle, an d a calcu lated card iac in d ex of 4.8 L/ m in/ m 2. Which one of the follow ing agents w ou ld be m ost usefu l in ad d ition to the n orep in ep h rin e for th e treatm en t of this p atient’s hyp otension? (A) (B) (C) (D) (E)

Milrinone Ep hed rine Phenylep hrine Dobu tam ine Vasop ressin

Que s tions : 7–17

12. On the second p ostop erative d ay, the patient continu es to requ ire high-d ose vasop ressor su p p ort w ith tw o m ed ications d esp ite ad equ ate flu id resu scitation. Low d ose hyd rocortisone therap y is started . Which one of the follow ing statem ents is tru e? (A) In p atients w ith sep tic shock, ad d ition of low d ose glu cocorticoid s m ay enh an ce th e vascu lar reactivity to vasoactive su bstances. (B) Glu cocorticoid s in the setting of sep tic shock w ill enhance the bactericid al effect of broad -spectru m antibiotics. (C) Ad d ition of glu cocorticoid s to the therapeu tic regim en w ill im p rove glycem ic control. (D) The m ajor action of hyd rocortisone in this setting is sod iu m retention w ith su bsequ en t im p rovem ent of volu m e status. (E) Ad m inistration of glu cocorticoid s w ill m ost likely cau se leukopenia in this patient. 13. A 51-year-old fem ale w ith an anxiety d isord er is ad m itted for elective su rgery. Becau se the p atient is extrem ely nervou s, lorazep am is given IV in rep eat, su bh yp n otic d oses to m anage her anxiety. Which one of the follow ing statem ents is m ost accu rate? (A) The p atient’s m otor fu nction is likely to be m ore im p aired than her cognitive fu nction. (B) Lorazep am is m ore rap id in onset as com p ared to m id azolam . (C) The p atient m ay exp erience extrap yram id al sid e effects. (D) To achieve anxiolysis, highly sed ating d oses of the d ru g have to be ad m inistered . (E) Ad m inistration of lorazep am w ill cau se retrograd e am nesia.

193

14. Milrinone (A) is a catecholam ine u sed for treatm ent of congestive heart failu re (B) is an antiarrhythm ic d ru g (C) cau ses p erip heral vasod ilation (D) inhibits N a + ,K+ -ATPase (E) has a longer elimination half-life than inam rinone 15. A patient w ho is taking lithiu m for treatm ent of bipolar d isord er is sched uled to have general anesthesia. The lithium (A) need not be consid ered in the anesthetic regim en (B) m ay affect both d ep olarizing and nond epolarizing m uscle relaxants (C) decreases the duration of nondepolarizing m uscle relaxants (D) shou ld be stop ped 2 w eeks before su rgery (E) m ay increase anesthetic requ irem ents 16. A d ru g th at is associated w ith p u lm onary toxicity is (A) (B) (C) (D) (E)

d oxoru bicin bleom ycin vincristine m ethotrexate l-asp araginase

17. A 48-year-old w om an takes a tricyclic antid ep ressant for d ep ression. This p atient (A) m ay have an increased nu m ber of arrhythm ias (B) should have halothane and pancuronium as d ru gs of choice (C) shou ld be cau tioned to stop the m ed ication before surgery (D) m ay becom e hyp otensive w ith ketam ine (E) m ay have short em ergence w ith thiopental

194

11: P ha rma cology

18. A 66-year-old m ale w ith a history of d iabetes m ellitu s, hyp ertension, and coronary artery d isease is to u nd ergo u rgent su rgery for an exp and ing 6.4 cm infrarenal abd om inal aortic aneu rysm . H e had a m yocard ial infarction tw elve m onths ago at w hich tim e he u nd erw ent p ercu taneou s coronary angiop lasty w ith p lacem ent of d ru g-elu ting stents to the left anterior d escend ing and right coronary arteries. Cu rrent m ed ications inclu d e clop id ogrel and asp irin that w ere started after the stent placem ent. With resp ect to the com bination of these tw o d ru gs and the intraoperative risk of bleed ing, it is tru e that (A) both d ru gs exert their antip latelet effect through the sam e m echanism (B) clop id ogrel d oes not ad d significantly to the risk of bleed ing posed by aspirin (C) if these d ru gs w ere stop p ed one d ay p rior to su rgery, the risk of bleed ing w ou ld be m inim ized (D) both d ru gs can be antagonized (E) clopid ogrel should be d iscontinu ed 5-10 d ays prior to surgery in elective cases w ere su rgical hem ostasis is need ed 19. Gabap entin (A) is ap p roved for th e m on oth erap y of p artial seizu res (B) is a benzod iazep ine (C) is m etabolized by the liver (D) is u sefu l for the treatm ent of m igraine, chronic pain, and bipolar d isord er (E) increases the p lasm a concentrations of carbam azepine and p henobarbital w hen u sed concom itantly 20. A heroin ad d ict injects six bags of heroin per d ay. After being involved in a m otorcycle accid ent resulting in a fractu re of the tibia, he is brou ght to the em ergency d ep artm ent. Which one of the follow ing analgesics is m ost likely to be effective and least likely to p recipitate an acu te w ithd raw al synd rom e or cau se significant toxicity? (A) Mep erid ine (B) N albu p hine

(C) Bu p renorp hine (D) Bu torp hanol (E) Fentanyl 21. Which one of the follow ing is a β-ad renoceptor antagonist? (A) (B) (C) (D) (E)

Isop roterenol Dobu tam ine N ad olol Albu terol Ritod rine

22. A 76-year-old m ale is u nd ergoing CABG for severe three vessel coronary artery d isease u nd er card iop u lm onary byp ass (CPB) w ith fu ll an ticoagu lation w ith h ep arin . After u ncom p licated w eaning from CPB, the su rgeon requ ests reversal of the anticoagu lant w ith protamine. Shortly after ad ministration of a ratio of 1 m g intravenou s p rotam ine to 1 m g hep arin, the p atient d evelop s su d d en onset of hem od ynam ic instability. Which one of the follow ing statem ents regard ing p rotam ine is m ost accurate? (A) Anap hylactic reactions occu r in abou t 10% of d iabetic p atients on p rotam inecon tain in g in su lin (N PH in su lin ) w h o are ad m inistered intravenou s protam ine. (B) Anap hylactic p rotam ine reaction in the general p op u lation consisting of p u lm onary hyp ertension, right ventricular d ysfu nction, and system ic hypotension is rare. (C) The p atient d escribed in this vignette received a p rotam ine overd ose. (D) Protam ine is equ ally effective in reversing the anticoagu lant activity of low m olecu lar w eight hep arin as com pared to u nfractionated hep arin. (E) Protam ine can be u sed as a sp ecific antid ote for fond ap arinux. 23. Dantrolene (A) has a half-life of abou t 36 h (B) red u ces concentrations of intracellu lar calcium

Que s tions : 18–29

(C) cau ses m arked card iac d ep ression (D) in the setting of m alignant hyp ertherm ia is d osed at 1 m g/ kg initial bolu s (E) cau ses nep hrotoxicity 24. A 76-year-old p atient w ith a history of N YH A stage IV heart failu re is ad m itted to the intensive care unit for the treatment of acute, d ecomp en sated h eart failu re. Th e p atien t is hyp otensive, tachycard ic, and in resp iratory d istress w ith increased w ork of breathing as w ell as h yp oxem ia d ocu m en ted on ABG. Which one of the follow ing is the m ost app rop riate therapeu tic agent? (A) (B) (C) (D) (E)

Losartan Fenold op am Minoxid il N icard ip ine N esiritid e

25. N ifed ip ine (A) m ay cause tachycardia as a compensatory effect (B) exerts its therap eu tic effect by slow ing AV-nod e cond u ction (C) achieves m axim u m p lasm a concentration m ore qu ickly w hen ad m inistered su blingual, as opp osed to the oral rou te (D) is a coronary vasoconstrictor (E) has p ositive inotrop ic effects 26. Doxoru bicin m ay p rod u ce a card iom yop athy that (A) (B) (C) (D)

is p resent only d u ring therap y is ind ep end ent of the d ose ad m inistered is not affected by rad iation therap y can be evalu ated w ith echocard iograp hy

27. A hom eless person bu ys an inexp ensive qu art of d enatu red alcohol (90% ethanol, 10% m ethanol) at a h ard w are store. H e m ixes a p in t of d enatured alcohol w ith a half-gallon of orange ju ice and d rinks the m ixtu re over a p eriod of an hou r. A short w hile later, he is fou nd u nconscious and brought to the emergency departm ent

195

of a nearby hospital. All of the follow ing statem ents abou t this p atient are tru e EXCEPT (A) he is likely to have been d ru nk p rior to d rinking the d enatu red alcohol becau se he d id not recognize the abnorm al taste of the m ethanol present in it (B) he is likely to have a severe m etabolic acidosis due to the effects of a metabolite of one of the alcohols he ingested (C) he is likely to be blind d u e to the effects of a m etabolite of one of the alcohols he ingested (D) his d ecreased level of consciou sness is likely d u e to the d irect effects of one of the alcohols he ingested (E) he is likely to benefit from hem od ialysis 28. Tachyphylaxis (A) can occu r w ith ad m inistration of p henylep hrine (B) d escribes the rap id ly d ecreasing effectiveness of certain d ru gs w ith rep eat ad m inistration (C) occu rs w ith continu ou s ad m inistration of norepinep hrine (D) is also know n as “ceiling effect’’ (E) can occu r d u e to ind u ction of enzym e system s 29. A m echanism that accu rately d escribes the effect of a d ru g u sed to p rod u ce d eliberate hyp otension is (A) sod iu m nitrop ru ssid e only d ilates resistance vessels (B) nicard ip ine cau ses coronary and p erip heral vasod ilatation (C) isoflu rane cau ses d ecreased card iac outpu t and peripheral vascular resistance (D) nitroglycerin p rim arily d ilates resistance vessels (E) esm olol w ill attain a d ecrease in blood p ressu re faster than a d ecrease in heart rate

196

11: P ha rma cology

30. A patient w ith a history of type 1 d iabetes m ellitu s, coronary artery d isease, and COPD is to receive β -blocker therap y as p art of an antihyp ertensive regim en. The m ost su itable agent for this ind ication is (A) (B) (C) (D) (E)

p rop ranolol labetalol atenolol carved ilol p ind olol

31. Which one of the follow ing agents u sed as an ad ju nct in the therap y of chronic p ain can cau se rebound hypertension after abrup t cessation of therap y? (A) (B) (C) (D) (E)

Top iram ate Clonid ine Milnacip ran Carbam azep ine Gabap entin

32. Mannitol (A) is effective at d ecreasing ICP in the absence of blood -brain barrier integrity (B) is u sefu l in the treatm ent of d ialysis d isequ ilibrium synd rom e (C) is a w eak acid (D) is alm ost com p letely reabsorbed from the renal tu bule (E) is effective at p rovid ing p eriop erative renal p rotection d u ring high-risk u rologic p roced u res 33. Based on the neurotransmitter aberrations that are thought to be responsible for the symptoms of Parkinson disease, which one of the following d ru g types w ou ld be therap eu tic in a patient w ith Parkinson d isease? (A) (B) (C) (D) (E)

Monoam ine oxid ase inhibitor Phenothiazine Bu tyrop henone Serotonin antagonist Serotonin reu p take inhibitor

34. The w ithd raw al synd rom e in a p erson p hysically d epend ent on w hich one of the follow ing d ru gs can be fatal? (A) (B) (C) (D) (E)

Am p hetam ine Phencyclid ine Diazep am H eroin Eszop iclone

35. Fenold op am (A) activates α- and β -ad renocep tors (B) cau ses a d ose related d ecrease in renal blood flow and glom eru lar filtration rate (C) is effective in p reventing rad iological contrast-ind uced nephrop athy (D) is a selective d op am ine D 1 recep tor agonist u sed to treat severe hypertension (E) cau ses a d ose-d ep end ent increase in heart rate and card iac contractility 36. Which one of the follow ing agents m ay be ad m inistered via the intravenous route to treat p ostp artu m hem orrhage? (A) (B) (C) (D) (E)

Oxytocin Prostagland in F2α Methylergonovine Progesterone Prostagland in E2

37. A 34-year-old w om an h as a h istory of prolonged paralysis after a short, elective operation, and she w as su bsequ ently show n to be homozygou s for the atypical form of the butyrylcholinesterase gene. H er 4-year-old son now need s anesthesia for bilateral m yringotom y and ear tu be p lacem ent. What is the p robability that her son w ill have a prolonged resp onse to su ccinylcholine? (A) (B) (C) (D)

25% 50% 100% Cannot be calcu lated w ithou t m ore inform ation

Que s tions : 30–44

38. A 22-year-old fem ale college stud ent is sched u led for placem ent of a tu nneled IV catheter for total p arenteral nu trition. She has a history of anorexia nervosa and is severely m alnou rished . It w ou ld be reasonable to avoid w hich one of the follow ing med ications in the perioperative p eriod ? (A) (B) (C) (D) (E)

Cefazolin Drop erid ol Su ccinylcholine Vancom ycin Sevoflu rane

39. Each of the follow ing characteristics app lied to an anesthesiologist m ight be su ggestive, bu t not d iagnostic, of self-ad m inistration of intravenou s opioid s EXCEPT (A) w earing a long sleeve t-shirt u nd erneath a scru b top (B) chronically show ing u p late for w ork (C) m iosis in a d ark room or m yd riasis in a bright room (D) requ esting m ore frequ ent breaks than other colleagu es (E) avoid ing changing into scru bs in the locker room 40. A 44-year-old w oman w ith a significant history of p rior postoperative nausea and vom iting is given ap rep itant p reop eratively before a d iagnostic laparoscopy as part of a m u ltid ru g antiem etic regim en . Ap rep itan t in teracts m ost strongly w ith w hich end ogenou s ligand ? (A) (B) (C) (D) (E)

β -end orp hin Dynorp hin A Su bstance P Proop iom elanocortin Leu -enkep halin

41. A person w hose ancestors cam e from w hich one of the follow ing geographical areas is most likely to need the low est d aily d ose of w arfarin to m aintain a therapeu tic value for IN R? (A) Su b-Saharan Africa (B) China

197

(C) Scand inavia (D) Sou thern Eu rope (E) Arabian peninsu la DIRECTION S: Use the follow ing table to answ er Qu estion 42: Che mic al clas s

pro te in bo und (%)

Dia ze pa m

We a k ba s e

98

Is opropa nol

Nonionic

As pirin

We a k a cid

80

Pa ncuronium

Qua te rna ry a mine

10

P he noba rbita l

We a k a cid

50

0

42. The intravenou s ad m inistration of sod iu m bicarbonate w ill increase the excretion rate of w hich one of the follow ing d ru gs to the greatest d egree? (A) (B) (C) (D) (E)

Diazep am Isop rop anol Asp irin Pancu roniu m Phenobarbital

43. An anesthesiologist is d ep loyed in the d esert of a Mid d le Eastern cou ntry. An artillery shell exp lod es in the m id st of his base, and an alarm is sou nd ed becau se a nerve gas has been d etected . H e is not w earing p rotective gear and shou ld therefore im m ed iately inject him self w ith (A) (B) (C) (D) (E)

d iazep am and atrop ine p hysostigm ine and d iazep am p hysostigm ine and p ralid oxim e p ralid oxim e and atrop ine p ralid oxim e and d iazep am

44. The benzod iazep ine that is m ost ap p rop riate as a bed tim e sed ative d u e to its p harm acokinetic prop erties is (A) (B) (C) (D) (E)

oxazep am nord azep am d iazep am flu razep am qu azep am

198

11: P ha rma cology

45. Stereosp ecific bind ing of an agonist to the µ -opioid receptor prod uces all of the follow ing effects EXCEPT (A) (B) (C) (D) (E)

eu p horia cou gh su p p ression sed ation constip ation analgesia

46. Cyclooxygenase (COX) is associated w ith p articip ating in nu m erou s p hysiological fu nctions. Which one of the follow ing effects is prim arily ascribable to COX-2 as op p osed to COX-1? (A) (B) (C) (D) (E)

Platelet ad hesiveness Inflam m ation Regu lation of renal blood flow Pain p ercep tion Protection of GI m u cosa against acid d am age

47. Metoprolol exerts its therap eu tic effect via (A) (B) (C) (D) (E)

β 1-ad renocep tor β 1-ad renocep tor β 2-ad renocep tor α 2-ad renocep tor α 1-ad renocep tor

agonism antagonism antagonism antagonism antagonism

48. Both d iazep am and bu sp irone (A) low er seizu re threshold (B) cau se sleep iness (C) d ecrease abnorm al behaviors in obsessive-com pu lsive d isord er (D) d isp lay cross-tolerance w ith ethanol (E) d ecrease anxiety 49. A p atient has been recently d iagnosed w ith bip olar d isord er. H e is also on d ialysis d u e to renal failu re resu lting from u ntreated u reteral reflu x. H is psychiatrist w ishes to begin chronic therapy w ith a m ed ication to prevent fu rther m anic episod es. Which one of the follow ing is the BEST choice in this patient? (A) Lithiu m carbonate (B) Citalop ram

(C) Carbam azep ine (D) Phenelzine (E) Bu p rop ion 50. Which one of the follow ing opioid s is ineffective as an analgesic in a significant fraction of th e p op u lation d u e to a gen etic p olym orp hism ? (A) (B) (C) (D) (E)

Cod eine H yd rom orphone Meperid ine Methad one Oxycod one

51. Soon after a patient is started on haloperid ol he begins com p laining of feeling stiff and having d ifficu lty in p erform ing rap id , fine m otor m ovem ents like typing at his com p uter keyboard . Which one of the follow ing med ications w ill MOST likely help these new -onset sym p tom s? (A) (B) (C) (D) (E)

Flu oxetine Diazep am Asp irin Benztrop ine Lithiu m

52. Which one of the follow ing characteristics of a su bstance m ake it less likely to be abused by hu m ans? (A) (B) (C) (D) (E)

Tolerance d evelop s to som e of its effects. It p otentiates d op am ine neu ral activity. It is cheap and easy to obtain. It has a rap id onset. It has a long d u ration.

53. A 47-year-old m an w ith long-stand ing typ e I d iabetes m ellitu s has severe peripheral vascular d isease and peripheral neuropathy. H is feet are chronically cold and blue and are extremely painful. In com mencing therapy for his painful feet, the least ap p rop riate m ed ication to try first is (A) oxcarbazep ine (B) oxycod one

Que s tions : 45–63

(C) gabap entin (D) am itrip tyline (E) carbam azep ine

199

58. Ad renergic α 2A 59. Ad renergic β 2

DIRECTION S (Qu estions 54-59): Each grou p of item s below consists of lettered head ings follow ed by a list of nu m bered p hrases or statem ents. For each nu m bered p hrase or statem ent, select the ON E lettered head ing or com p onent that is m ost closely associated w ith it. Each lettered head ing or com p on en t m ay be selected on ce, m ore than once, or not at all.

DIRECTION S (Qu estions 60-63): Each grou p of item s below consists of lettered head ings follow ed by a list of nu m bered p hrases or statem ents. For each n u m bered p h rase or statem ent, select th e ON E lettered head ing or com p onent that is m ost closely associated w ith it. Each lettered head ing or com ponent m ay be selected once, m ore than once, or not at all.

(A) Agonists d ecrease the heart rate by inhibiting ad enyl cyclase. (B) Agonists cau se vasoconstriction in arterioles by activating p hosp holip ase C and increasing the intracellu lar concentration of calcium . (C) Agonists relax bronchial sm ooth m u scle by activating gu anylyl cyclase. (D) Agonists increase ganglionic transm ission by activating phosp holipase C and increasing the concentration of inositol triphosphate. (E) Agonists cau se vasod ilation by inhibiting gu anylyl cyclase. (F) Agonists cau se vasod ilation by increasing end othelial nitric oxid e synthase. (G) Agonists relax bronchial sm ooth m u scle by activating ad enyl cyclase. (H ) Agonists inhibit transm ission in sym p athetic neurons by inhibiting ad enyl cyclase.

(A) Increases nu m bers of ad ip ocytes and increases u p take of fatty acid s by ad ipocytes (B) Blocks voltage-d ep end ent calciu m channels d ecreasing intracellu lar calciu m in pancreatic β cells (C) Stim u lates tyrosine kinase associated w ith insu lin recep tors in hep atic cells (D) Increases the activity of AMP-d ep end ent p rotein kinase (AMPK) increasing fatty acid oxid ation in hep atocytes (E) Inhibits N a + -K+ -ATPase thereby increasing intracellu lar p otassiu m in ad ip ocytes (F) Inhibits p hosp hod iesterase (PDE) thereby increasing insulin release from p ancreatic β cells (G) Agonist at the GLP-1 (glu cagon-like p ep tid e) recep tor (H ) Cau ses closu re of KATP channels in p ancreatic β cells increasing insu lin release

For each autonomic receptor, select the corresponding m echanism of action:

For each m ed ication u sed to treat d iabetes m ellitu s, select the correspond ing m echanism of action:

54. Muscarinic M 1 55. Mu scarinic M 2 56. Mu scarinic M 3 57. Ad renergic α 1A

60. Exenatid e 61. Rosiglitazone 62. Metform in 63. Rep aglinid e

Answe rs a nd Expla na tions

1. (D ) The patient has all the sym ptoms of central anticholinergic synd rom e that m ay follow the ingestion of jim son w eed that contains atropine. The typ ical sym ptom s of “d ry as a bone, blind as a bat, red as a beet, hot as a stove, and m ad as a hatter” are d u e to the central and perip heral antim u scarinic effects of atrop ine. Physostigm ine can cross the blood –brain barrier and w ill cou nteract both the central and perip heral effects. (1:252) 2. (B) Flu oxetine is an antid ep ressant u sed to treat persons w ith major d epressive, obsessivecom pulsive, and anxiety d isord ers. (1:405) 3. (C) The p atient m ay be exhibiting the Eaton– Lam bert synd rom e, an acqu ired au toim m u ne d isord er of the neu rom u scu lar ju nction often associated w ith carcinom as. These p atients are hyp ersensitive to the effects of both d ep olarizing and non-d ep olarizing m u scle relaxants. Propofol shou ld not be a factor in the apnea after a tim e period need ed to com p lete a thoracotom y, and neither shou ld be the fact that the p atient received nitrou s oxid e. H yp oxic p u lm onary vasoconstriction is a p u lm onary vascu lar m echanism aim ed at m inim izin g V/ Q m ism atch. (5:143, 612) 4. (B) Dobu tam ine is a synthetic catecholam ine and contains a chiral center. The preparation used in clinical practice contains a racemic mixtu re of (+ ) and (–) enantiom ers in equ al amounts and the apparent effect of the combination of the tw o isom ers is β 1-ad renocep tor stim u lation. Dobu tam ine increases card iac ou tp u t and contractility. At higher d oses,

200

α-ad renoceptor agonist effects may be seen. It causes increased renal blood flow. (1:290, 804-5) 5. (C) Etom id ate p rod u ces the highest incid ence of p ostop erative vom iting after short p roced u res, p rop ofol the low est. (5:697, 1255) 6. (C) Catecholam ines are inactivated by MAO and COMT. Pseu d ocholinesterase inactivates su ccinylcholine and m ivacuriu m, w hile acetylcholinesterase inactivates acetylcholine. (1:187, 200; 5:500-2) 7. (B) Ipratropium bromide is an inhaled anticholinergic that causes bronchodilation. While anticholinergics are effective in severe acute asthma, they are less effective than β 2-adrenoceptor agonists and are often part of a combination therapy regimen. While terbutaline is an effective bronchodilator, it is only marketed in oral and subcutaneou s form s in the US. Salm eterol is a long-acting β 2-adrenoceptor agonist with relatively slow onset, thus it is less suitable for therapy of the acute asthma attack. Theophylline is available in oral and parenteral preparations; h ow ever, it is less effective th an n ebu lized β 2-adrenoceptor agonists and does not increase their bronchodilator response. (1:291-3, 1043-5) 8. (C) The antid epressant bu propion has efficacy in d ecreasing the sym p tom s of nicotine w ithd raw al and im p roving abstinence rates in sm okers w ishing to quit. (1:407, 658) 9. (D ) Botu linu m toxin lead s to p aralysis by p reventing the release of acetylcholine from nerve term inals. (1:186)

Answe rs : 1–15

10. (C) Use of heparin in a patient w ith heparinind u ced throm bocytop enia typ e II can lead to life threatening throm botic com p lications and is therefore not ind icated . Warfarin is an oral anticoagu lant. Fond ap arinu x is an activated factor X inhibitor ap p roved for p rop hylaxis, and treatm ent of d eep vein throm bosis and p ulm onary em bolu s, and is ad m inistered subcu taneou sly. Clop id ogrel is an inhibitor of ADP-ind u ced p latelet aggregation ind icated for the second ary p revention of stroke and the red u ction of card iac events after p ercu taneou s coronary intervention. It is ad ministered orally. Argatroban is a d irect throm bin inhibitor w ith a relatively short half-life of 1-2 h that can be continu ou sly infu sed d u ring CABG su rgery. Other su itable alternatives w ould includ e lepiru d in, bivaliru d in, and d anap aroid . (1:855, 870; 5:211-2, 900) 11. (E) Arginine vasop ressin (AVP) concentrations in p atients w ith sep tic shock are inap p rop riately low, an d w h en ad m inistered exogenou sly, vasopressin can increase systolic blood p ressu re in this patient popu lation. The proposed mechanism is tw ofold : first, end ogenous AVP d epletion m ay be the result of excessive barorecep tor-m ed iated release from su stained h yp oten sion, and second , AVP ap p ears to restore vascu lar sensitivity to norep inephrine. Another agent that cou ld be ad d ed to restore blood p ressu re in sep tic shock is ep inep hrine instead of vasop ressin. Milrinone and d obu tam ine are not ind icated becau se this patient has good m yocard ial contractility. Ep hed rine is a m ixed -actin g sym p ath om im etic d ru g that ind irectly releases norep inep hrine and has som e d irect effects on β 2-ad renocep tors. It is n ot in d icated for con tin u ou s in fu sion in sh o ck states. Ph en ylep h r in e is a selectiv e α 1-ad renocep tor agonist and d oes not p rovid e ad d itional benefit. (1:278, 708; 5:1321) 12. (A) Ad m inistration of a low -d ose glucocorticoid to patients in septic shock has been show n to d ecrease the time to d iscontinuation of vasop ressor therapy in som e clinical trials, and is recom m end ed for p atients w ith sep tic shock

201

refractory to vasopressors and flu id therapy by the Su rviving Sep sis Cam p aign Gu id elines. One of the m ajor actions of glu cocorticoid s on the card iovascu lar system is to enhance the vascu lar reactivity to other vasoactive su bstances, for exam ple norepinephrine or phenylep hrine. Glu cocorticoid s d o not im p rove the bactericid al activity of broad -sp ectru m antibiotics, they w ill increase blood glu cose concentrations, and cau se a leu kocytosis d u e to increased release of polymorphonuclear leukocytes from the bone m arrow. Sod iu m retention is much m ore pronounced w ith ad ministration of m ineralocorticoid s; glu cocorticoid s p lay a p erm issive role in the renal excretion of free w ater. (1:1224, 1231; 5:1321) 13. (A) Lorazep am is a benzod iazepine and therefore d oes not cau se extrap yram id al effects as are seen w ith the p henothiazines. Mid azolam is m ore rap id in on set th an lorazep am . Benzod iazep ines u su ally p rod u ce a state of sed ation accom p anied by d ecreased anxiety and anterograd e, not retrograd e, am n esia. They can red u ce anxiety at d oses that are not highly sed ating. Cognition tend s to be less affected than m otor p erform ance, w hich is w hy m ost ind ivid u als w ho receive benzod iazep ines u nd erestim ate their d egree of im pairm ent. (1:465; 5:698) 14. (C) Milrinone and inam rinone are p hosp hod iesterase in h ibitors u sed for th e sh ort-term p arenteral inotrop ic su p p ort in severe heart failu re. They exhibit their therap eu tic effect throu gh inhibiting the breakd ow n of cyclic AMP an d have n o antiarrhythm ic activity. Becau se m ilrinone has a shorter half-life and few er sid e effects, it is a better choice than inam rinone for this ind ication. (1:805) 15. (B) Lithiu m m ay p rolong the effect of both d epolarizing and nond epolarizing relaxants. It is not necessary to stop the d ru g before ind u ction, bu t it is necessary to consid er the d rug w hen p lanning the anesthesia. In ad d ition to the effect on relaxants, the d ru g m ay lead to a red u ction of anesthetic requ irem ents. (1:267)

202

11: P ha rma cology

16. (B) Bleom ycin is associated w ith p u lm onary fibrosis, som etim es long after the d ru g has been stop p ed . Doxoru bicin p rod u ces card iac toxicity. Vincristine is associated w ith m yelosu ppression. Methotrexate toxicity is m anifest as imm u nosu ppression. l-asparaginase is associated w ith hep atic toxicity and allergic reactions. (1:1717-8) 17. (A) Arrhythmias are common in the patient on a tricyclic antidepressant and the combination of halothane, pancuronium, and a tricyclic antidepressant has been reported to be associated with severe intraoperative arrhythm ias. The d ru g should not be stopped prior to surgery, but the anesthesiologist must be aw are of its interactions w ith anesthetic agents. Ketam ine and catecholam ines m ay cau se an exaggerated hypertensive response in these patients. (1:411) 18. (E) Asp irin irreversibly blocks the p rod u ction of throm boxane A 2 in p latelets, w hereas clop id ogrel exerts its antip latelet effect by blocking the p latelet ADP receptor. Becau se of their d iscrete m echanism s, the effect of these d rugs is ad d itive or even synergistic that resu lts in a higher risk for excessive bleed ing d u ring su rgical p roced u res. There are no antagonists available for these m ed ications. Becau se of their long d u ration of action, stop p ing the d ru gs one d ay p rior to su rgery w ou ld not d ecrease the risk of bleed ing. Cu rrent recom mend ations are to stop clopid ogrel 5-10 d prior to su rgery in elective cases w here su rgical hem ostasis is need ed . Asp irin in vascu lar su rgical p atients shou ld be continu ed throu gh the periop erative period (5:115) 19. (D ) Gabapentin is app roved for the treatm ent of partial seizures in ad ults w hen used in ad d ition to other anticonvu lsant d ru gs. It is not m etabolized and is excreted u nchanged in the u rine. It is also u sed for the treatm en t of m igraine, chronic pain, and bip olar d isord er and concom itant u se d oes not increase the p lasm a concentrations of carbam azep ine or phenobarbital. (1:599) 20. (E) Of the med ications listed , only fentanyl and m ep erid ine are fu ll agonists at the µ -op ioid

recep tor. Becau se the p atient is likely to have significant tolerance requiring a large d ose of the agonist, m ep erid ine w ou ld not be a good choice becau se it has an active m etabolite that can cause CN S excitation resulting in seizures and inhibition of serotonin reu p take that can cau se serotonin synd rom e. N albu p hine and bu torp hanol have µ -op ioid antagonistic activity w hile bu p renorp hine is a µ -opioid partial agonist; each of these three m ed ications m ay p rod u ce w ith d raw al if given to an op ioid tolerant person. (5:715-6; 722-4) 21. (C) Of th e d ru gs listed , only n ad olol is a β -ad renocep tor antagonist. Isop roterenol is a β -ad renocep tor agonist and d obu tam ine is a β 1-ad renocep tor agonist. Albu terol and ritod rine are β 2-ad renocep tor agonists. (5:752) 22. (B) Protam ine is a natu rally occu rring p rotein salt (from salm on sem en). It cau ses anaphylactic reactions in only abou t 1% of p atients w ho have been sensitized to p rotam ine (e.g., N PH insulin), w hile true anaphylactic reaction in the general p opu lation as d escribed in op tion B is rare. H yp otension on ad m inistration is a frequ en t sid e effect th at can be m itigated if injected slow ly and into a p erip heral vein. The p atient in this vignette d id not receive an overd ose, as m ost p rotocols call for ad m inistration of p rotam ine in slight excess of 1 m g p er 1 m g (or 100 u n its) of h ep arin ad m in istered . Protam ine only bind s long heparin m olecu les, an d th erefore on ly p artially rev erses th e effects of low m olecu lar w eight hep arins, and has no effect on the very short m olecu les of fond ap arinu x. (1:858-9; 5:900) 23. (B) Dantrolene d ecreases intracellu lar calciu m and cau ses generalized m u scle w eakness. The h alf-life is abou t 9 h . H ep atotoxicity is com m on after p rolon ged th erap y. Card iac toxicity is not u su ally seen. The initial d ose of d antrolene in the setting of MH is 2.5 m g/ kg, id eally ad m in istered in to a large vein . It shou ld be ad m inistered for at least 24 h after the MH ep isod e u ntil all signs of the hyp erm etabolic state are resolved and the concentration of creatin in e kin ase is d ecreasin g. (1:286, 626; 5:1499)

Answe rs : 16–31

24. (E) The m ajor site of action for the hu m an recom binant brain natriuretic p ep tid e (nesiritid e) is the inner m ed u llary collecting d u ct (IMCD). It lead s to an increase in u rinary sod iu m excretion, d ecreases system ic and pu lm onary resistances and left ventricu lar filling p ressure, and results in a second ary increase in card iac outpu t. N esiritid e d ecreases d ysp nea in p atients w ith d ecom p ensated heart failu re and pu lm onary ed em a. The other d ru gs listed w ill d ecrease blood p ressu re, w ithou t increasing card iac contractility. (1:695-6, 5:746-8) 25. (A) N ifed ip ine d oes not affect cond u ction throu gh the AV-nod e at routine clinical d oses and is used as a coronary vasod ilator and antihyp ertensive. The d rug has no positive inotrop ic effect. The arterial d ilatation cau sed by nifed ip ine m ay cau se a reflex tachycard ia in som e p atients. Su blingu al ad m inistration of the d rug for control of acu te hypertension has largely been aban d on ed , an d su blin gu al ad m inistration w ill achieve target plasm a concentrations no faster than oral ad m inistration. (1:756-60, 777) 26. (D ) The card iom yop athy seen w ith d oxoru bicin m ay be seen long after the chem otherapy h as been d iscon tinu ed . This sid e effect is d ep end ent on the cu m u lative d ose of d oxoru bicin and m ay be evalu ated by echocard iograp hy. Patients w ho have received rad iation therapy to the med iastinum may have a greater likelihood of this effect. (1:1714-5) 27. (A) It is d ifficu lt to d ifferentiate m ethanol from ethanol by taste. Methanol is converted to form ic acid that is p articu larly toxic to the retina as w ell as being a m ajor cau se of the severe m etabolic acid osis that occu rs. Both m ethanol and ethanol are read ily rem oved by hem od ialysis. (1:631-2) 28. (B) Tachyp hylaxis, or acu te tolerance to a d ru g that d evelops rapid ly, is seen w ith ep hed rine becau se of d ep letion of norep inep hrine stores. Tachyphylaxis also occurs d ue to d esensitization of recep tor system s becau se of rep eat

203

stim u lation by agonists and is not d u e to ind u ction of enzym e system s. Rep eated or continu ous ad m inistration of norepinephrine or phenylephrine d oes not red uce their effectiveness. A ceiling effect occurs w hen a patient d oes not exhibit an increasing effect w ith an increasing d ose; an exam p le of a ceiling effect is the analgesia p rod u ced by the op ioid p artial agonist bu p renorphine. (1:68) 29. (B) Sod iu m nitrop ru ssid e d ilatates both the resistance and cap acitance vessels, w hereas nitroglycerin p rim arily d ilatates cap acitance vessels. N icard ip ine is a calciu m channel blocker that cau ses coronary and p erip heral vasod ilatation. Isoflu rane in clinically m eaningfu l d oses d ilates the p erip heral vascu latu re w ith ou t an y sign ifican t effect on card iac ou tp u t. The effects of esm olol on heart rate occu r faster than those on blood p ressu re. (1:543, 747-50, 757-8, 782-3; 5:753) 30. (C) With the excep tion of atenolol, all agents listed are non-selective β-adrenoceptor antagonists. This class of d rugs has the p otential to d elay recovery from hyp oglycem ia in p atients w ith typ e 1 d iabetes. Fu rtherm ore, there is a p otential for life-threatening bronchoconstriction w ith nonselective β -ad renoceptor blockad e, and selective agents su ch as atenolol are p referred . (1:315) 31. (B) Rebou nd hyp erten sion can occu r after abru pt cessation of β-ad renoceptor antagonists su ch as propranolol and centrally acting antihyp ertensives su ch as clonid ine. Mechanism s of action for top iram ate, an anticonvu lsant, includ e sod ium channel blockad e, potentiation of GABA-m ed iated inhibition, and a d ecrease in glu tam ate neu rotransm ission. Milnacip ran is a d u al norep inep hrine and serotonin reu p take inhibitor. Carbamazepine slow s the recovery rate of sod iu m ch an n els. Gabap en tin increases GABA release. All of these agents are u sed as ad ju ncts in the therap y of chronic pain bu t as op p osed to clonid ine, d o not cau se p otentially life-threatening hyp ertension u pon acu te cessation. (5:1555, 1557, 1560-1)

204

11: P ha rma cology

32. (B) Dialysis d isequilibrium synd rome is caused by too rap id rem oval of solu tes from the extracellu lar flu id by hem od ialysis resu ltin g in r ed u ct io n o f o sm o la lit y o f ext r a cellu la r flu id and associated CN S sym p tom s (headache, nau sea, CN S d ep ression, convu lsions). Mannitol is filtered by the glom eru lus and is negligibly reabsorbed . It is a nonelectrolyte that has alm ost no pharm acologic effects asid e from its osm otic activity after intravenou s injection. The d ru g’s effectiveness at d ecreasing elevated ICP d ep end s on p reservation of the integrity of the blood -brain barrier in a significant portion of the brain. There is no evid ence that ad m inistration of m annitol is of any benefit in surgical patients at high risk for renal d am age. (1:681-2; 5:1146, 1374) 33. (A) The basal ganglia are thou ght to contain inhibitory dopaminergic neurons and excitatory ch olin ergic neu ron s. In Parkin son d isease, there is a loss of d opam inergic neurons and relative excess in cholinergic neuronal activity. Dopamine agonists, acetylcholine antagonists, and monoamine oxid ase inhibitors, by virtue of decreasing dopamine degrad ation and increasing d op am inergic activity, are therap eu tic. Phenothiazines and butyrophenones have antidopaminergic effects and should be avoid ed in patients w ith Parkinson d isease. Drugs affecting serotonin transmission have little positive or negative effect on the d isease. (5:149) 34. (C) Withd raw al from barbitu rates and benzod iazep ines can resu lt in grand m al seizu res that m ay be fatal. Op ioid and am p hetam ine w ithd raw al, w hile u ncomfortable, are not consid ered to be life-threatening. Eszopiclone is a non-benzod iazep in e sed ative u sed for the long-term treatm ent of insom nia and sleep m aintenance. Unlike the benzod iazepines, no signs of tolerance or seriou s w ithd raw al are associated w ith its u se. Phencyclid ine (PCP, “angel d u st”) is a p syched elic agent that is u sed interm ittently by hum ans, therefore, tolerance and w ithd raw al synd rom es have not been observed . (1:411, 468, 665; 5:327; 6:3556) 35. (D ) Fenold op am is a selective d op am ine D 1 recep tor agonist that is ind icated for short

term , rap id red u ction of blood p ressu re in severe hyp ertension. The d ru g is d evoid of αand β -ad renocep tor and d op am ine D 2 activity and cau ses a d ose-related increase in renal blood flow and glom eru lar filtration rate. It d oes not increase heart rate or card iac contractility. While fenold op am has d em onstrated nep hrop rotective p rop erties in critically ill patients and those u nd ergoing card iac su rgery, it has not been show n to m itigate the severity of con trast-ind u ced n ep hrop athy to d ate. (5:747) 36. (A) Methylergonovine and PGF2 are ad m inistered intram u scu larly, and PGE2 is ad m inistered via the oral, rectal, or vaginal rou tes. Oxytocin is ad m inistered in slow intravenous increm ents; all of these d ru gs increase u terine contractions and m ay d ecrease p ostp artu m hem orrhage. Progesterone u su ally relaxes the u teru s. (5:1157) 37. (D ) The m other has certainly p assed onto her son one gene for the atyp ical form of butyrylcholinesterase, how ever w ithou t know ing the father ’s genotyp e, it is im p ossible to p red ict the son’s phenotypic response to su ccinylcholine. H e is, at least, a carrier for the atyp ical gene. If the father has no cop ies of the atyp ical gene, then the son w ill be p henotyp ically norm al. If the father is a carrier and has one copy, the son has a 50% chance of being affected w ith a p rolonged resp onse to su ccinylcholine. If the father is homozygous and has tw o copies of the atyp ical gene, then the son has a 100% chance of being affected . (5:79-80) 38. (B) Persons w ith anorexia nervosa and resu lting severe malnutrition are at increased risk for having prolonged QT and QTc intervals. Of the d rugs listed , d rop erid ol m ay increase the risk of torsad es d e pointes. (5:108, 220) 39. (B) The anesthesiologist w ho is d ep end ent on opioid s m ost typically arrives earlier to w ork than others so as to be able to obtain and d ivert op ioid m ed ication early in the d ay to avoid w ithd raw al and to requ est frequ ent breaks throu ghou t the d ay to self-inject. They often take measures to hide cutaneous manifestations

Que s tions : 32–52

205

of m u ltip le injection sites from colleagu es. Miosis in a d ark room , ind icating excessive op ioid effect, or m yd riasis in a bright room , ind icating im p end ing w ithd raw al, are also w orrisom e signs. (5:1626-8)

are L-isom ers. The D-isom ers of these m ed ications are d evoid of analgesic effects and d o not bind to the µ -op ioid recep tor, how ever they retain antitu ssive activity. Su ch a m ed ication is d extrom ethorp han. (1:491-5; 512-3)

40. (C) Aprepitant is an antagonist of su bstance P at the neu rokinin (N K1) recep tor. (1:1344)

46. (B) Inflam m ation is m ed iated in p art by p rostagland ins synthesized by COX-2. Pain and GI m ucosal integrity are associated w ith prostaglan d in s syn th esized by both COX-1 an d COX-2. Regu lation of renal blood flow and p latelet ad hesiveness is d ue to prostagland ins or throm boxanes, respectively, synthesized by COX-1. (1:849-51; 949-51)

41. (B) There is significant geograp hical heterogeneity in the VKORC1 phenotype that is responsible for the enzym e inhibited by w arfarin. Persons of black African d escent are m ost resistant to w arfarin and requ ire the highest d aily m aintenance d oses, w hile p ersons of Chinese d escent require the low est d aily d oses. (1:861-3) 42. (E) The rate of the u rinary excretion of w eak acid s is increased by increasing the pH of the u rine w ith sod iu m bicarbonate. Th e greater the fraction of a d rug bound to plasma protein, the less the renal clearance w ill be. (1:19, 24-5) 43. (D ) The em ergency m anagem ent of p oisoning w ith a n erve agen t (an irreversible in h ibitor of acetylcholinesterase) is the ad m inistration of atrop in e (to d ry p u lm on ary secretion s) an d p ralid oxim e (an acetylch olin esterase reactivat o r ). (1:248-9) 44. (A) A benzod iazep ine u sed as a bed tim e sed ative shou ld have a rap id onset after oral ad m inistration and a short d u ration that is not associated w ith excessive sed ation the follow ing d ay. That typ ically m eans that it has no active m etabolites. Oxazep am m eets these criteria. Diazep am is long-acting, w ith a half-life of about a d ay, and it has an active m etabolite, nord azep am , w ith a half-life of abou t 4 d . N ord azep am is m etabolized to oxazep am ; flu razep am , and qu azep am also h ave active m etabolites w ith half-lives greater than a d ay. (1:463-6) 45. (B) Althou gh all µ -op ioid agonists u sed as analgesics have antitu ssive activity, this effect is not d u e to stereosp ecific bind ing to the µ -op ioid recep tor. All su ch µ -op ioid agonists

47. (B) Metop rolol is a selective β 1-ad renocep tor antagonist. Other m em bers of this grou p are esm olol, atenolol, and acebutolol (5:311) 48. (E) Both d iazep am and bu spirone are antianxiety agents. Bu sp irone is neither a sed ative nor an anticonvulsant. Buspirone has no efficacy in p ersons w ith obsessive com pu lsive d isord er. (1:343-4, 349, 412-3) 49. (C) Of the m ed ications listed , both lithiu m carbonate and carbam azep ine have efficacy as m ood stabilizers in p ersons w ith bipolar d isord er. Lithiu m carbonate is contraind icated in p erson s w ith ren al failu re becau se of the increased risk of toxicity. (1:447) 50. (A) Cod eine is a p rod ru g and requ ires m etabolism by CYP2D6 to m orp hine for analgesic activity. Abou t 10% of p ersons of w estern Eu rop ean d escent are d eficient in CYP2D6. (5:42) 51. (D ) The symptoms are typical of the Parkinsonlike rigid ity the antid op am inergic antip sychotic agents m ay cause. Specific treatm ent is w ith a centrally-acting m u scarinic cholinergic antagonist like benztrop ine. (1:422, 437) 52. (E) Factors that increase the likelihood that a su bstance w ill be abu sed includ e rapid onset an d offset, easy availability, and low p rice, and that it p otentiates d op am inergic rew ard p athw ays in the brain. (1:650)

206

11: P ha rma cology

53. (B) Som e antid ep ressants and anticonvu lsants, such as the ones listed , are effective in d ecreasing neu rop athic p ain. In contrast, op ioid s are less effective and their u se in neu rop athic p ain is associated w ith significant toxicity as w ell as the d evelopm ent of tolerance requ iring d ose escalation. (1:490, 518)

57. (B) (1:203)

54. (D ) (1:192)

61. (A) (1:1260-1)

55. (A) (1:220)

62. (D ) (1:1258)

56. (F) (1:221)

63. (H) (1:1257)

58. (H) (1:203) 59. (G) (1:204) 60. (G) (1:1261-3)

CHAPTER 12

Re g io nal Ane s the s ia Que s tions DIRECTION S (Qu estions 64-93): Each of the num bered item s or incom p lete statem ents in this section is follow ed by answ ers or by com p letions of the statem ent. Select the ON E lettered answ er or com p letion that is BEST in each case. 64. A 73-year-old m an u nd ergoes a d escend ing thoracic aortic aneu rism repair. Follow ing the su rgical repair, the patient is u nable to m ove his legs. Perfu sion to the spinal cord appears to have been com p rom ised d u ring the rep air. Which one of the follow ing d oes N OT su p p ly blood to the sp inal cord ? (A) (B) (C) (D) (E)

Anterior sp inal artery Posterior sp inal arteries External carotid arteries Anterior rad icu lar arteries Vertebral arteries

65. A 52-year-old fem ale u nd ergoes a right total knee arthrop lasty. H er p reference is to have an ep id u ral catheter p laced for p ostop erative analgesia. When thread ing a catheter into the epid u ral space, the anterior bord er of the ep id u ral sp ace m ay be encou ntered . The tissu e creating the anterior bord er of the ep id u ral space is (A) (B) (C) (D) (E)

anterior longitu d inal ligam ent p osterior longitu d inal ligam ent ligam entu m flavu m d u ra transverse p rocess

(A) (B) (C) (D) (E)

Iliohyp ogastic Ilioingu inal Fem oral Genitofem oral Su ral

67. A 25-year-old fem ale u nd ergoing an elective cesarean section agrees to a sp inal anesthetic. When performing a spinal anesthetic, the anesthesiologist m u st be know led geable of the fact th at cerebral sp inal flu id (CSF) is located betw een w hich tw o tissu e layers? (A) (B) (C) (D) (E)

Pia and arachnoid Pia and d ura Pia and sp inal cord Du ra and arachnoid Du ra and spinal cord

68. A 44-year-old female undergoes a right infraclavicular block for a right ulnar nerve transposition surgery at the elbow. The musculocutaneous nerve appears to have been missed. The musculocutaneous nerve usually emerges from the (A) (B) (C) (D) (E)

lateral cord inferior cord C5 nerve root posterior d ivision axillary nerve

66. A lu m bar p lexu s block is p erform ed in a 72-year-old m an sched u led to und ergo a total hip arthroplasty. Which nerve d oes N OT originate from the lu m bar p lexu s? 207

208

12: Re giona l Ane s the s ia

69. A 22-year-old m ale p resen ts for left h an d reconstructive surgery. You d iscuss performing an axillary brachial p lexu s block. After p erform ing an axillary brachial p lexu s block for forearm surgery, the nerve that w ill most likely need to be blocked by a separate injection is the (A) (B) (C) (D) (E)

axillary nerve su p rascap u lar nerve u lnar nerve m u scu locu taneou s nerve m ed ian nerve

70. The norm al bony vertebral colu m n is m ad e u p of how m any total vertebrae? (A) (B) (C) (D) (E)

27 29 31 33 35

71. A 72-year-old man presents for a total should er arthrop lasty. H e u nd ergoes a continu ou s interscalene block utilizing a trad itional land m ark technique. Bone is contacted w ithin 2 cm of the skin that likely represents the (A) (B) (C) (D) (E)

first rib clavicle scap u la transverse p rocess vertebral bod y

DIRECTION S: Use the follow ing scenario to answ er Questions 72-74: An 18-year-old m ale su stained an op en right u lna forearm fractu re after slip p ing on ice. H e p resented for op en red u ction and internal fixation. H e u nd ergoes a p erip heral nerve block for surgical anesthesia w ith 20 m L of 0.5% bu p ivacaine. The follow ing u ltrasou nd im age is obtained :

72. After su ccessfu l com p letion of the block, all of the follow ing are exp ected to occu r on the ipsilateral sid e EXCEPT inability to (A) (B) (C) (D) (E)

flex the w rist shru g the shou ld er abd u ct the shou ld er flex the arm at the elbow extend the arm at the elbow

73. The p atient rep orts p ain d u ring skin incision on the u lnar sid e of his d istal forearm . The m ost likely reason is the failu re to (A) anesthetize the lateral antebrachial cu taneous nerve (B) ad vance the need le to the area rep resented by the "X" (C) anesthetize the u lnar nerve (D) anesthetize the m ed ial brachial cu taneou s nerve (E) ad vance the need le to the area rep resented by the arrow 74. If the need le is ad vanced to the area d ep icted by the arrow, w hich one of the follow ing m ay occu r? (A) (B) (C) (D) (E)

Inability to fu rther ad vance the need le A com p lete and d ense su rgical block A p neu m othorax An intravascu lar injection Su ccessfu l blockad e of the inferior tru nk of the brachial plexu s

Que s tions : 69–82

75. A 42-year-old m ale p resents w ith neck pain of 1-year d u ration. H e u nd ergoes a cervical ep id u ral steroid injection for w hat is p resu m ed to be herniation of the inner contents of the intervertebral d isk that resu lted in inflam m ation. The inner contents of the intervertebral d isk called is also called the (A) (B) (C) (D) (E)

annu lu s fibrosu s ligam entu m flavu m nu cleu s p u lp osu s p osterior longitu d inal ligam ent facet joint

76. A 41-year-old fem ale und ergoes right should er arthroscopy u tilizing an interscalene block as the su rgical anesthetic. Upon m aking a sm all skin incision on the cape of the shou ld er, the patient reports feeling the painfu l incision. The skin on the top of the should er is innervated by w hich nerve? (A) (B) (C) (D) (E)

Rad ial Su p raclavicu lar Axillary Med ian Ulnar

77. A 72-year-old m ale p resents for rep air of a d istal hu m eru s fracture. The patient prefers a regional anesthetic for su rgery and a brachial p lexus block is perform ed . The organization of the brachial plexu s starting proxim ally in the neck and traveling d istally to the axilla is (A) (B) (C) (D) (E)

tru nks, d ivisions, roots, cord s, branches d ivisions, tru nks, roots, cord s, branches branches, cord s, d ivisions, tru nks, roots roots, tru nks, cord s, d ivisions, branches roots, tru nks, d ivisions, cord s, branches

78. All of the follow ing nerves provid e exclu sively cu taneous innervation to the foot EXCEPT the (A) (B) (C) (D)

su ral nerve sap henou s nerve tibial nerve su p erficial p eroneal nerve

209

79. An 18-year-old fem ale sustained a closed right ankle fracture w hile skating. She presents five d ays later for open red uction and internal fixation of the ankle. She agrees to a regional anesth etic for su rgical anesth esia as w ell for p ostop erative analgesia. Which nerve innervates the skin over the m ed ial asp ect of the low er leg? (A) (B) (C) (D) (E)

Saphenous Sciatic Sural Com m on p eroneal Ilioingu inal

80. A 54-year-old m ale is sched u led to u nd ergo a right total hip arthrop lasty. You p erform a lu m bar p lexu s block as w ell as a glu teal sciatic block for p ostop erative analgesia. The sciatic nerve is com posed of the ventral ram i of (A) (B) (C) (D) (E)

L4 - S3 T12 - L4 T12 - S3 S1-S3 T10-S3

81. You are p erform in g an u ltrasou nd -gu id ed su p raclavicu lar block in a 15-year-old fem ale for left hand su rgery. You encou nter bone w ith need le ad van cem en t. You su sp ect th at th e need le has m ost likely encou ntered the (A) (B) (C) (D) (E)

clavicle second rib transverse p rocess of C6 vertebral bod y of C6 first rib

82. A 72-year-old fem ale u nd ergoes a transm etatarsal am putation of the left foot for peripheral vascular d isease. All of the follow ing are su itable regional anesthetics EXCEPT a(n) (A) (B) (C) (D) (E)

ankle block fem oral and p op liteal sciatic block sap henou s and p op liteal sciatic block fem oral and sap henou s nerve block sp inal block

210

12: Re giona l Ane s the s ia

83. You are obtain in g in form ed con sen t from 46-year-old law yer for a fem oral nerve block for a right ACL rep air. You exp lain that you w ill be u sing u ltrasou nd as the m eans of nerve localization. All of the follow ing are tru e w hen u sin g u ltrasou n d in region al an esth esia EXCEPT that it (A) d ecreases the incid ence of u nintentional vascu lar punctu res (B) d ecreases the nu m ber of need le p asses (C) d ecreases the volu m e of local anesthetic requ ired (D) d ecreases the onset tim e of a block (E) has not been show n to facilitate the p lacem ent of epid u ral blocks in d ifficu lt p atients DIRECTION S: Use the follow ing scenario to answ er Qu estions 84-85: A 25-year-old m ale u nd ergoes ACL rep air of the right knee u tilizing a ham string tend on. In the PACU, the patient rep orts intense pain and requ ests a p erip heral nerve block. Utilizing ultrasou nd for the nerve block, 25 m L of 0.5% rop ivacaine w as ad m inistered and the follow ing u ltrasound -guid ed im age w as obtained :

(C) extend the low er leg at the knee joint (D) flex the low er leg at the knee joint (E) flex the hip joint 85. After a su ccessfu l block, the p atient w ou ld be exp ected to have loss of sensation to the (A) (B) (C) (D) (E)

lateral asp ect of the right thigh m ed ial asp ect of the right low er leg lateral asp ect of the right low er leg p osterior asp ect of the thigh d orsu m of the foot

86. A brachial p lexu s block is p erform ed in a 26-year-old m an for op en red u ction and internal fixation of the left elbow. A su p raclavicu lar block is p erform ed to anesthetize the entire brachial p lexu s. The brachial p lexu s is p rim arily d erived from the nerve roots of (A) (B) (C) (D) (E)

C5-C8 C5-T1 C6-C8 C6-T2 C2-C4

87. A 66-year-old fem ale w ith severe COPD presents for left u p p er extrem ity su rgery. She prefers a regional anesthetic in ord er to avoid intraop erative intu bation and w ants to avoid the need for postoperative intubation. Which one of the follow ing u p p er extrem ity blocks shou ld be avoid ed becau se it typically resu lts in p hrenic nerve blockad e w ith resu lting ip silateral d iap hragm p aralysis? (A) (B) (C) (D) (E)

84. Du ring the u ltrasou nd -gu id ed block show n above, anesthetizing the nerve d ep icted by the w hite arrow w ou ld resu lt in inability to (A) d orsiflex the foot (B) p lantarflex the foot

Su p raclavicu lar Interscalelene Infraclavicu lar Axillary Su p rascap u lar

88. Du ring a barroom fight, a m an fractu red his 5th m etacarp al after throw ing a p u nch. The follow ing d ay he u nd erw ent a su rgical rep air of the 5th metacarpal. Which one of the follow ing nerve(s) w ou ld need to be anesthetized to ensu re ad equ ate p ostop erative analgesia?

Que s tions : 83–93

(A) (B) (C) (D) (E)

Ulnar nerve Ulnar and rad ial nerves Med ian and rad ial nerves Med ian, rad ial, and u lnar nerves Mu scu locu taneou s, m ed ian, rad ial, and u lnar nerves

89. A popliteal sciatic nerve block w as p erform ed for op en red u ction and internal fixation of the ankle. The p atient rep orted feeling the painfu l su rgical incision m ad e on the m ed ial asp ect of the ankle. The nerve m ost likely m issed w as (A) (B) (C) (D) (E)

su ral nerve sap henou s nerve d eep p eroneal nerve su p erficial p eroneal nerve tibial nerve

DIRECTION S (Qu estions 90-91): Each grou p of item s below consists of lettered head ings follow ed by a list of nu m bered p hrases or statem ents. For each nu m bered p hrase or statem ent, select the ON E lettered head ing or com p onent that is m ost closely associated w ith it. Each lettered head ing or com p on en t m ay be selected on ce, m ore th an on ce, or n ot at all. (A) (B) (C) (D) (E) (F) (G) (H )

Sciatic Fem oral Su p raclavicu lar Tibial Sap henou s Infraclavicu lar Su p erficial Cervical Plexu s Su p rascap u lar

For each p atient, select the best p erip heral nerve block. 90. A 62-year-old morbid ly obese patient sustained an open ankle fracture after falling. She und ergoes general anesthesia for open red uction and internal fixation. In the PACU she rep orts intense pain not alleviated w ith opioid s.

211

91. A 50-year-old gentlem an is sched u led for elective rotator cuff repair of the left shou ld er. You d ecid e to perform an interscalene nerve block; how ever, you are u nable to obtain an ad equ ate u ltrasou nd im age. For com p lete su rgical anesthesia, the m ost ap propriate alternative techniqu e is need ed . DIRECTION S (Qu estions 92-93): Each grou p of item s below consists of lettered head ings follow ed by a list of nu m bered p hrases or statem ents. For each nu m bered p hrase or statem ent, select the ON E lettered head ing or com p onent that is m ost closely associated w ith it. Each lettered head ing or com p on en t m ay be selected on ce, m ore th an on ce, or not at all. (A) (B) (C) (D) (E) (F) (G) (H )

Ulnar nerve block Mu scu locu taneou s nerve block Med ian nerve block Rad ial nerve block Obtu rator nerve block Su p rascap u lar nerve Block Med ial brachial cu taneou s nerve block Med ial antebrachial cu taneou s nerve block

For each p atient, select the best rescu e block. 92. A 26-year-old m ale su stain ed a left elbow fractu re. After the p erform ance of an infraclavicu lar block, you note that sensation is still intact on the ventral and lateral asp ects of the forearm . 93. A sup raclavicular nerve block is perform ed for surgical anesthesia of the hand . Up on testing the success of the block prior to su rgery you notice that there is a lack of sensation on the volar and lateral asp ect of the forearm bu t intact sensation on the m ed ial sid e.

Answe rs a nd Expla na tions

64. (C) The only artery in the list that d oes not su pply the spinal cord is the external carotid artery. (5:780) 65. (B) The posterior longitu d inal ligam ent creates the anterior board er of the ep id u ral sp ace. (5:789) 66. (E) Th e su ral n erve origin ates from th e com m on p eron eal an d tibial n erves. The com m on p eroneal nerve and the tibial nerve originate from the sciatic nerve (sacral plexus). (5:839) 67. (A) The CSF is mad e by the choroid plexu s and is fou nd in the su barachnoid sp ace. (5:786) 68. (A) The musculocutaneous nerve is a branch of the lateral cord . If the infraclavicu lar nerve block is p erform ed in a m ore d istal location w ith ultrasound guid ance, the musculocutaneous nerve m ay be m issed . The m usculocutaneou s nerve is m ore likely to be m issed w ith an axillary ap p roach and a sep arate injection is requ ired . (5:829) 69. (D ) The m u scu locu tan eou s n erve can be m issed w hen an axillary block is p erform ed becau se in m any cases it has branched aw ay from the neu rovascu lar bu nd le m ore p roxim ally. A separate injection is u sually necessary. (5:834) 70. (D ) There are 7 cervical, 12 thoracic, 5 lu m bar, 5 sacral, and 4 coccygeal vertebrae. (5:786) 71. (D ) The transverse p rocess w ou ld rep resent th e m ost su p erficial bony stru ctu re in th e 212

vicinity of a correctly p erform ed land m arkbased interscalene block. (5:830) 72. (B) This is an u ltrasou nd im age of a su p raclavicu lar block. After a su ccessfu l block, one w ou ld still be able to shru g the shou ld ers as the trapezius muscles are innervated by cranial nerve XI. (5:831-3) 73. (B) The m ed ial asp ect of the forearm is innervated by the m ed ial antebrachial cu taneou s nerve that is com p rised of nerve fibers from the C8-T1 nerve roots or low er tru nk of the brachial plexus. To consistently anesthetize these fibers the need le w ou ld need to be ad vanced closer to the area d esignated by the "X." This area has been term ed the "corner p ocket" that is form ed by the su bclavian artery and first rib. (5:831-3) 74. (C) The arrow d epicts the pleura of the lu ng. Puncturing the pleura may result in a pneumothorax. (5:831-3) 75. (C) The nu cleu s p u lp osu s rep resents the inner asp ect of the intervertebral d isk and the ou ter fibrou s ring is called the annu lu s fibrosu s. (5:786) 76. (B) The su p raclavicu lar nerves innervate the top of the shoulder and originates from the cervical plexus. The axillary nerve also innervates the skin of the shou ld er, bu t m ore d istally. (5:836) 77. (E) The organization of the brachial p lexu s begins w ith the roots of C5-T1, w hich then organize into three tru nks, follow ed by the

Answe rs : 64–90

anterior and p osterior d ivisions. Below the level of the clavicle the d ivisions organize into three cord s and then finally transition into the branches of the brachial p lexu s. (5:828-9) 78. (C) The sural, saphenous, and superficial peron eal nerves p rovid e exclu sively cu taneou s innervation to the foot. The tibial nerve, in ad d ition to p rovid ing cutaneou s innervation, also provid es innervation to the d eep er stru ctures of the foot. (5:845-7) 79. (A) The sap henou s nerve is a branch of the fem oral nerve and travels d ow n the m ed ial asp ect of the low er leg to innervate the ankle. (5:841) 80. (A) The sciatic nerve is com posed of the ventral ram i of L4-S3 from the sacral p lexu s. The ventral ram i of T12-L4 com p rise the lu m bar p lexus. (5:843) 81. (E) The first rib is easily im aged w hen p erform ing u ltrasou nd -gu id ed su p raclavicu lar blocks. It m ay p rovid e som e level of p rotection from a p ossible p neu m othorax w hen ad vancing a need le. (5:832) 82. (D ) Branches of the sciatic nerve and fem oral nerve need to be blocked for a transm etatarsal am p u tation. An ankle block w ill anesthetize the tibial, d eep peroneal, su perficial peroneal, and su ral nerves that are all branches of the sciatic nerve. In ad d ition the saphenous nerve, w hich is a branch of the fem oral nerve, is p art of a com plete ankle block. (5:840-7)

213

allow ing for flexion at the knee joint. The fem oral nerve d oes not innervate the hip flexors. (4:69-72) 85. (B) The cu taneou s innervation of the m ed ial asp ect of the low er leg is from the saphenous nerve that is a branch of the fem oral nerve. The cutaneou s innervation of the lateral asp ect of the thigh is from the lateral fem oral cutaneou s nerve and the posterior thigh from the posterior fem oral cu taneou s nerve. The sciatic nerve innervates the lateral asp ect of the low er leg and d orsu m of the foot. (5:842) 86. (B) The ventral ram i of the sp inal nerves from C5-T1 com p rise the brachial p lexu s. There is variable contribu tion from C4 and T2. The cervical plexus is comprised of the ventral ram i of C2, C3, and C4. (5:828) 87. (B) Typ ically the interscalene ap p roach resu lts in a nearly 100% incid ence of blockad e of the p hrenic nerve. The incid ence of p hrenic nerve blockad e from a su p raclavicu lar ap p roach has ranged from 3050%. (5:831) 88. (A) The u lnar nerve p rovid es d orsal and ventral innervation to the cu taneou s as w ell as intrinsic components of the 5th metacarpal and d igit. Ulnar nerve blockad e at the level of the forearm w ill p rovid e com p lete p ostop erative analgesia. (5:837)

83. (E) Ultrasou nd has been show n to facilitate n eed le p lacem en t in n eu raxial techn iqu es, esp ecially in p atients w ith d ifficu lt ep id u ral localization. (5:851)

89. (B) The saphenou s nerve provid es cutaneou s innervation to the m ed ial asp ect of the low er leg and ankle w ith a variable extension on the m ed ial asp ect of the foot. The innervation d istal to the knee joint is p red om inately from the sciatic nerve w ith the sap henou s nerve being the only contribu tion from the fem oral nerve. (5:841)

84. (C) This is an u ltrasou nd im age of the fem oral nerve after injection of local anesthetic. The fem oral nerve innervates the qu ad ricep s m u scles that are resp onsible for extend ing the low er leg at the knee joint. The sciatic nerve is resp onsible for p lantar flexion, d orsiflexion, eversion, and inversion of the foot. The sciatic n erv e in n erv ates th e h am strin g m u scles

90. (A) The p red om inate innervation to the ankle joint com es from the sciatic nerve and therefore a sciatic nerve block w ou ld be ap p rop riate. Althou gh the sap henou s nerve p rovid es som e innervation to the ankle joint, it has a m inor role. For p ostop erative analgesia, the sciatic block shou ld be the first block p erform ed and in m any instances a sap henou s

214

12: Re giona l Ane s the s ia

nerve block may not be necessary. H ow ever for com plete su rgical anesthesia, in ad d ition to a sciatic nerve block, a saphenous nerve block is requ ired . (5:843-5) 91. (C) Available evid ence su ggests that a su p raclavicular block can take the place of an interscalene block for anesthetizing the shou ld er joint. The su p rascap u lar nerve and axillary nerve innervate the shou ld er joint. The su prascap u lar nerve typ ically branches from the u pper tru nk of the brachial plexu s w hile the axillary nerve is a branch nerve of the brachial p lexu s. Since the su p raclavicu lar block is a block at the tru nk level of the p lexu s, both of these nerves w ou ld be anesthetized . (5:831-3) 92. (B) The ventral and lateral asp ect of the forearm is innervated by the lateral antebrachial cu taneou s nerve (LACN ). The LACN is an

extension of the m u scu locu tan eou s n erve. Therefore a m u scu locu taneou s nerve block w ou ld need to be p erform ed . (4:849) 93. (A) The m ed ial asp ect of the forearm is innervated by the m ed ial antebrachial cu taneou s nerve (MACN ). The nerve fibers of this nerve are d erived from the C8-T1 spinal nerves or the inferior tru nk of the brachial p lexu s. If sensation w ere intact on the m ed ial asp ect of the forearm , this w ou ld su ggest that these fibers w ere m issed w hen p erform ing the su p raclavicu lar block. The u lnar nerve, rad ial nerve, and m ed ian nerve innervate the hand . The u lnar nerve fibers are also d erived from the C8-T1 spinal nerves or inferior trunk. Therefore if sensation w ere intact on the m ed ial asp ect of the forearm , this w ould also su ggest that the u lnar nerve w ou ld also be intact. (5:837)

CHAPTER 13

Ane s the s ia fo r Cardio tho rac ic and Vas c ular S urg e ry Que s tions DIRECTION S (Qu estions 94-174): Each of the nu m bered item s or incom p lete statem ents in this section is follow ed by answ ers or by com p letions of the statem ent. Select the ON E lettered answ er or com p letion that is BEST in each case. 94. A 72-year-old p atient w ho und erw ent successfu l CABG is extu bated 2 hou rs after arrival in the ICU. The next m orning, the patient experiences the acute onset of sinus brad ycard ia w ith associated hyp otension. The next best step in m anagem ent is to (A) (B) (C) (D) (E)

initiate artificial card iac p acing start isop roterenol start ep inep hrine p erform a stat TTE ad m inister ad enosine

95. Afterload is red u ced w hile d iastolic p erfu sion p ressu re is increased by (A) (B) (C) (D) (E)

d op am ine ep inep hrine nitroglycerin nitrop ru ssid e intraaortic balloon cou nterp u lsation

97. Increased am p litu d e of v w aves in a central venou s pressure record ing ind icates (A) (B) (C) (D) (E)

ju nctional rhythm atrial fibrillation tricu sp id regu rgitation hyp ovolem ia heart block

98. At 17°C (A) (B) (C) (D)

EEG activity is u nchanged cellu lar integrity is lost cerebral blood flow increases cerebral m etabolic rate of oxygen consu m ption (CMRO 2) is less than 10% of norm otherm ic value (E) the brain sw itches to anaerobic m etabolism

99. Which one of the follow ing m od alities for artificial p acin g w ill resu lt in ven tricu lar, as opposed to atrial, pacing only? (A) (B) (C) (D)

Transvenou s end ocard ial lead s Ep icard ial lead s External noninvasive electrod es Esop hageal electrod es

96. Ischem ia from a right coronary artery lesion w ou ld m ost likely be evid ent on electrocard iograp hic lead (A) (B) (C) (D) (E)

I II VR AVL V5 215

216

13: Ane s the s ia for Ca rdiothora cic a nd Va s cula r S urge ry

100. A 62-year-old m ale w ith a p ast m ed ical history of alcohol abuse, smoking, and COPD is ad mitted to the hospital w ith severe substernal chest pain after an ep isod e of binge d rinking d u ring w hich he rep orted ly threw u p violently. H e is brou ght to the op erating room for em ergent exploration of the chest for su sp ected esop hageal rup tu re. H is physical exam ination is notable for signs and sym p tom s consistent w ith severe sep sis, tachyp nea, and Sp O 2 of 89% w hile breathing O 2 throu gh a nonrebreather facem ask. Airw ay exam is n otable for a Mallam pati III score and a fu ll beard . The surgeon inform s you that one-lu ng ventilation w ill be requ ired to facilitate esop hageal rep air. The anesthetic p lan most likely to resu lt in safe ind u ction of anesthesia and ad equ ate lu ng sep aration in ad d ition to rap id -sequ ence ind u ction inclu d es (A) p lacem ent of right-sid ed d ou ble-lu m en tube (B) p lacem ent of left-sid ed d ou ble-lu m en tube (C) p lacem ent of single-lu m en tu be follow ed by right m ainstem intu bation (D) p lacem ent of Univent tu be follow ed by selective bronchial blockad e (E) p lacem ent of single-lu m en tu be follow ed by left m ainstem intu bation 101. The best d rying effect before fiberop tic end oscopy is perform ed is achieved by the ad m inistration of (A) (B) (C) (D) (E)

neostigm ine p yrid ostigm ine ed rop honiu m atrop ine glycop yrrolate

102. A patient und ergoing right upp er lobectom y for lu ng cancer in the left lateral d ecu bitu s p osition is exp eriencin g h yp oxem ia w h ile receiving an FIO 2 of 1.0 d u ring one-lu ng anesthesia w ith a d ouble-lum en tube. Which one of the follow ing options is the m ost app rop riate next step in the treatm ent of hypoxem ia?

(A) Ap p lication of continu ou s p ositive airw ay p ressu re (CPAP) to the nond epend ent lu ng (B) Ap p lication of PEEP to the d ep end ent lu ng (C) Interm ittent reinflation of the nond epend ent lu ng (D) Reinstitu te tw o-lu ng ventilation (E) Ad m inistration of oxygen via su ction catheter to the collapsed lu ng 103. Du ring card iop u lm onary byp ass, p atients (A) need large d oses of m u scle relaxants at 25°C (B) requ ire neu rom u scu lar blockad e d u ring cooling and w arm ing (C) shou ld have Pa CO 2 m aintained at abou t 30 m m H g (D) shou ld have their lu ngs hyp erinflated (E) shou ld have continu ed p u lm onary ventilation 104. N itroglycerin causes vasod ilation that is m arked ly p otentiated by (A) (B) (C) (D) (E)

m etop rolol rem ifentanil labetalol m agnesiu m su lfate sild enafil

105. An 82-year-old p atient w ith a card iac rhythm management d evice (ICD/ pacemaker) in place requ ires su rgery that entails the u se of electrocautery. Which one of the follow ing statem ents is tru e? (A) The cutaneous electrode (skin pad ) of a unipolar electrocautery unit should be as close to the pulse generator as possible. (B) Electrocard iograp hic m onitoring is necessary only if the patient is being p aced . (C) Placing a magnet over the device can d eactivate the pacing function of an ICD. (D) The risk for interference is negligible if the pacer is a d em and unit. (E) Electrocau tery can electrically reset the p acem aker.

Que s tions : 100–111

DIRECTION S: Use the follow ing scenario to answ er Qu estions 106-107: A 42-year-old w om an is referred by a thoracic su rgeon from the clinic for m ed iastinoscopy. On evalu ation, the patient is sitting u p on the stretcher, and the respiratory p attern is notable for tachyp nea that accord ing to the p atient has been p resent for the p ast tw o w eeks. The p atient's voice sou nd s hoarse, and she tells you that she sleeps w ith tw o p illow s at night. 106. Based on the d escrip tion of sym p tom s, w hich one of the follow ing find ings is likely to be p resent on p hysical exam ? (A) Pain of the shou ld er and m ed ial asp ect of forearm (B) Ru bor of face, and arm veins fail to em pty on elevation (C) Low er extrem ity ed em a, and rales on au scu ltation (D) Pu rsed lip s, p rolonged exp iriu m , and bilateral w heezing on au scu ltation (E) Paraesthesias, tetany, and d izziness 107. Based on the p atients' p resentation, w hich one of the follow ing is the safest approach to ind uction of anesthesia? (A) Up p er extrem ity IV access and rap id sequ ence ind u ction w ith placem ent of a single-lu m en tu be (B) Up p er extrem ity IV access and rap id sequ ence ind u ction w ith placem ent of d ou ble-lu m en tube (C) Low er extrem ity IV access, stand ard IV ind u ction, m ask ventilation, p lacem ent of single-lu m en tu be (D) Low er extrem ity IV access, p lacem ent of arterial blood p ressu re m onitor, m aintenance of sp ontaneou s breathing, placem ent of single-lu m en tube (E) Up p er extrem ity IV access, p lacem ent of arterial blood p ressu re m onitor, m aintenance of sp ontaneou s breathing, placem ent of single-lu m en tube 108. The p atient w ith a p acem aker in p lace m ay d evelop com p eting rhythm s w hen a norm al sinu s rhythm is present and the unit has been converted to the asynchronou s m od e. If the

217

p acing stim u li fall on the T w ave of the p reviously cond u cted beats, (A) ventricu lar fibrillation w ill follow (B) there is little d anger, since the energy ou tput is low w ith cu rrent pu lse generators (C) ventricu lar fibrillation is less likely if hyp oxem ia is p resent (D) ventricu lar fibrillation is less likely w ith catecholam ine release (E) ventricu lar fibrillation is less likely w ith m yocard ial infarction 109. Intraaortic balloon cou nterp u lsation is a circu latory assist m ethod that (A) is u sed for p atients w ith aortic aneu rysm s (B) is u sed for p atients w ith aortic insu fficiency (C) cau ses an intraaortic balloon to be inflated d u ring systole (D) increases coronary blood flow (E) increases im p ed ance to the op ening of the left ventricle 110. The clam p ing of the thoracic aorta in aneu rysm rep air is follow ed by (A) (B) (C) (D) (E)

im m ed iate hypotension im m ed iate hypertension card iac stand still no change loss of blood pressu re in the right arm

111. A 72-year-old p atient is u nd ergoing thoracic aortic su rgery. When the p atient is extubated on the first postoperative d ay, he is noted to have p arap legia on neu rological exam . This com p lication of aortic su rgery is m ost com m only d u e to (A) p ressu re on the sp inal cord d u ring su rgery (B) long p eriod s of hyp otension (C) hypothermia associated with the surgery (D) sp inal cord ischem ia (E) loss of cerebrosp inal flu id

218

13: Ane s the s ia for Ca rdiothora cic a nd Va s cula r S urge ry

112. H yp ovolem ia m ay occu r d u ring abd om inal aneu rysm p roced u res as a resu lt of all of the follow ing EXCEPT (A) (B) (C) (D) (E)

blood loss inad equ ate flu id rep lacem ent u se of vasod ilators loss of flu id into the bow el exp ansion of the vascu lar bed d u ring occlu sion

113. The blood flow rate for an ad u lt on total card iopu lm onary byp ass is generally (A) (B) (C) (D) (E)

15 m L/ kg/ m in 35 m L/ kg/ m in 55 m L/ kg/ m in 85 m L/ kg/ m in 115 m L/ kg/ m in

114. When an ad u lt patient is on total card iopu lmonary bypass, (A) arterial p ressu re is generally m aintained above 50 m m H g (B) blood is p u m p ed from the venae cava and d rains by gravity into the aorta for circu lation (C) the level of blood in the venou s reservoir of the pu m p reflects the central venous pressu re of the patient (D) venou s p ressu re elevation is of no consequ ence (E) venou s retu rn to the p u m p is alw ays started before arterial infu sion 115. A 48-year-old fem ale w ith a history of 40 p ackyears of sm oking, hyp ertension, and hyp ercholesterolem ia is to u nd ergo right carotid end arterectom y for sym p tom atic, high-grad e stenosis. Which one of the follow ing statem ents regard ing the anesthetic care for carotid artery surgery in this p atient is tru e? (A) Blood p ressu re d u ring carotid occlu sion shou ld be m aintained at, or above the p atient's baseline. (B) The u se of a BIS m onitor w ill reliably d etect intraop erative cerebral ischem ia.

(C) The u se of volatile anesthetics w ou ld resu lt in su p erior brain p rotection as com pared to propofol. (D) The su rgical p roced u re shou ld be p reced ed by tests to show if carotid clam ping can be tolerated . (E) The u se of regional anesthesia w ou ld m inim ize the risk of perioperative stroke. 116. The m ost reliable m onitor for d etection of intraop erative m yocard ial ischem ia is (A) (B) (C) (D) (E)

creatine p hosp hokinase levels changes in the ST-T segm ent on ECG transesop hageal echocard iograp hy trop onin concentrations exhaled nitric oxid e

117. A 72-year-old m ale is p resenting for end ovascu lar stent graft rep air of a 6.8 cm d escend ing thoracic aortic aneu rysm . The p atient's p ast med ical history is significant for insulin d epend ent d iabetes, hypertension, hypercholesterolem ia, coronary artery d isease, and m yocard ial infarction. The p atient's p ast su rgical history is significan t for in fraren al aortic an eu rysm rep air. In ord er to d ecrease the risk of sp inal cord ischem ia, p art of the anesthetic m anagem ent shou ld be (A) the p reop erative p lacem ent of a lu m bar d rain (B) intraop erative EEG m onitoring (C) d eep hyp otherm ic circu latory arrest (DH CA) (D) d eliberate hyp otension (E) a "high-d ose narcotic" techniqu e 118. An 84-year-old fem ale w ith know n severe m itral stenosis is to u nd ergo u rgent left hip hem iarthrop lasty after su ffering a fem u r fractu re after a fall. The patient had a failed sp inal anesthetic in the p ast and is refu sing neu raxial anesthesia. The hemod ynamic goal after ind uction of anesthesia should includ e (A) increased heart rate (B) increased contractility (C) increased p reload

Que s tions : 112–124

(D) d ecreased contractility (E) increased afterload 119. In com p aring p atients u nd ergoing esop hagectom y against those u nd ergoing p u lm onary resection, it is generally TRUE that esop hagectom y patients (A) (B) (C) (D)

have better nu tritional statu s have less risk of asp iration have better p u lm onary fu nction are less likely to be hyp oxic d u ring single lu ng ventilation (E) are less likely to need p ostop erative ventilation 120. The hu m an larynx (A) lies at the level of the 1st through 4th cervical vertebrae (B) in the ad u lt is narrow est at the level of the cricoid cartilage (C) is innervated solely by the recu rrent laryngeal nerve (D) is p rotected anteriorly by the w id e exp anse of the cricoid cartilage (E) lies w ithin the thyroid cartilage

219

(C) ind u ction, p aralysis, intu bation, and laryngoscop y (D) p aralysis, intu bation, ind u ction, and laryngoscop y (E) to establish an airw ay before p aralysis or instrum entation 123. A p atient w ith esop hageal obstru ction is to have a general anesthetic for esop hagoscopy. H e has had a bariu m sw allow on the p reviou s d ay. One of the greatest d angers of the planned p roced u re is (A) (B) (C) (D) (E)

bleed ing hyp otension d ifficu lt intu bation asp iration arrhythm ia

DIRECTION S: Use the follow ing figu re to answ er Qu estions 124-127:

121. A p atient is u nd ergoing a m ed iastinoscop y w hen there is a su d d en loss of p u lse and p ressure w ave being m onitored at the right w rist. Th e m ed iastin oscop e is w ith d raw n , w ith resu m p tion of norm al vital signs. The m ost likely cau se of the p roblem is (A) (B) (C) (D) (E)

card iac arrest su p erior vena cava obstru ction air in the m ed iastinu m com p ression of the innom inate artery anesthetic overd ose

122. A 71-year-old m an is ad m itted w ith a com p laint of hoarseness and sore throat. On ind irect laryngoscop y, a su p raglottic m ass is noted w ith ed em a of the cord s. H e is sched u led for a d irect laryngoscop y u nd er general anesthesia. The ap p roach to this p roced u re shou ld be (A) kept sim ple, since it is a short proced u re (B) ind u ction, paralysis, and laryngoscopy

124. The figu re show s a view of a p atient w ith a d ou ble-lu m en tu be view ed from the head of the bed . To ventilate the right lu ng and d eflate the left lung, one shou ld (A) (B) (C) (D) (E)

clam p clam p clam p clam p clam p

at at at at at

1 and 6 and 5 and 5 and 1 and

u ncap at 3 u ncap at 3 u ncap at 3 u ncap at 4 6 and u ncap at 3

220

13: Ane s the s ia for Ca rdiothora cic a nd Va s cula r S urge ry

125. After the tube has been correctly p ositioned , a bronchoscop e is u sed . By op ening at 4 and looking d ow n the lu m en of 2, one shou ld see (A) the left u p p er lobe w ith a right-sid ed tube (B) the carina w ith a right-sid ed tu be (C) the trachea w ith a right-sid ed tu be (D) the carina w ith a left-sid ed tu be (E) the left u p p er lobe w ith a left-sid ed tu be 126. To lavage the right lu ng w hile ventilating the left lu ng, one w ou ld p erform all the follow ing EXCEPT (A) clam p at 6 (B) clam p at 2 (C) p ou r flu id into 4

128. The d evice d ep icted in the figu re is (A) (B) (C) (D)

a Univent bronchial blocker tu be a Fogarty catheter a Sengstaken-Blakem ore tu be an ind ep end ent (Arnd t) end obronchial blocker (E) an airw ay exchange catheter 129. Which one of the follow ing statem ents abou t the d evice d epicted is tru e?

(D) inflate the bronchial cu ff (E) inflate the tracheal cu ff 127. All of the follow ing are tru e abou t the d ou blelu m en tu be in the figu re EXCEPT (A) lu m en 1 is the bronchial lu m en in a leftsid ed tu be (B) the p ressu res are equ al at 1 and 2 w hen no clam ps are ap plied (C) CPAP to the right lu ng is ap p lied at 4 (D) clam p ing at 6 isolates the left lu ng from the anesthesia circu it (E) clam ping at 5 and u ncap p ing at 3 w ill allow the left lu ng to collapse DIRECTION S: Use the follow ing figu re to answ er Questions 128-129:

(A) It is u sefu l to obtain one-lu ng ventilation in p atients w ho are alread y intu bated or have a d ifficu lt airw ay. (B) Positive p ressu re ventilation has to be interru p ted for p rop er p lacem ent. (C) There is no need for fiberop tic bronchoscopy to verify correct positioning of the d evice. (D) It can only be u sed in p atients u nd ergoing right sid ed su rgery. (E) With this device, the endotracheal tube has to be exchanged at the end of surgery if the patient is to remain intubated.

Que s tions : 125–136

130. Du ring ap neic oxygenation (A) the tim e elap sed before d esatu ration occu rs is ind epend ent of the patient's pu lm onary statu s (B) all arrhythm ias are d u e to hyp oxem ia (C) the carbon d ioxid e tension is not im p ortant (D) the carbon d ioxid e level rises abou t 3-6 m m H g/ m in (E) p u lse oxim etry is not help fu l 131. The bronchial venous systems d rains into all of the follow ing vascu lar bed s EXCEPT (A) (B) (C) (D) (E)

Thebesian veins hem iazygos veins azygos veins p u lm onary veins m ed iastinal veins

132. A 58-year-old m ale w ith past m ed ical history significant for m od erate to severe aortic regu rgitation is u nd ergoing lap aroscop ic ventral h ern ia rep air u n d er gen eral an esth esia. Assu m ing that the p atient's blood p ressu re is norm al an d at baselin e, ad m inistration of w hich one of the follow ing d ru gs is likely to resu lt in im p roved forw ard flow ? (A) (B) (C) (D) (E)

N orep inep hrine Glycop yrrolate Esm olol Phenylep hrine Vasop ressin

133. The ad m inistration of fentanyl in large d oses (0.1 m g/ kg) generally resu lts in (A) (B) (C) (D)

increased p u lm onary vascu lar resistance d ecreased heart rate histam ine release m ore p rofou nd hyp otension than is seen w ith m orp hine (1 m g/ kg)

221

134. Which one of the follow ing interventions is m ost likely to resu lt in p reservation of renal fu nction d uring aortic aneurysm su rgery? (A) Ad m inistration of fu rosem id e (B) Keeping aortic cross clam p tim e less than 120 m in (C) Ad m inistration of d opam ine (D) End ovascular app roach to aortic aneurysm rep air (E) Ad m inistration of m annitol DIRECTION S: Use the follow ing scenario to answ er Qu estions 135-136: A 75-kg p atient is u nd ergoing elective rep air of an aortic aneu rysm . Tw enty m inu tes after incision the p atient d evelops tachycard ia, hyp otension, and su bsequ ent ST elevations in lead V, as w ell as a rise in the p u lm onary artery (PA) p ressu res. An infusion of vasop ressor is started to correct hyp otension. 135. In ad d ition to correcting the hyp otension, the m ost effective intervention for the treatm ent of elevated PA p ressu res in this scenario is (A) (B) (C) (D) (E)

infu sion of m ilrinone ad d ition of inhaled nitric oxid e infu sion of nitroglycerin start hyp erventilation increase in FIO 2 to 1.0

136. With respect to the patient's tachycard ia, the next best step in m anagem ent is (A) (B) (C) (D) (E)

observation only ad m inistration of m etop rolol ad m inistration of nicard ip ine ad m inistration of neostigm ine ad m inistration of glycop yrrolate

222

13: Ane s the s ia for Ca rdiothora cic a nd Va s cula r S urge ry

137. A 72-year-old patient u nd erw ent an uncomp licated CABG for severe three-vessel coronary artery d isease fou r h ou rs ago an d is now requ iring increasing levels of hem od ynam ic su p p ort. The p atient w as started on an ep inephrine infusion, currently at 2 mcg/ kg/ min. Vital signs are blood pressure (BP) 80/ 50, heart rate 130, p u lm onary artery p ressu re 50/ 25, central venous p ressu re 24, card iac ind ex 1.6, and the ECG show s variations in am plitu d e. A bed sid e transthoracic echocard iogram show s right atrial collap se and abnorm al ventricu lar septal m otion. Based on the patient's presentation, the next best step in m anagem ent is to (A) ad d norep inep hrine (B) stop ep inep hrine and start m ilrinone (C) retu rn im m ed iately to the op erating room for reexp loration of the chest (D) ad d nitroglycerin (E) stat CT of the chest w ith p u lm onary em bolism p rotocol 138. An anesthetic consid eration for Marfan synd rom e is (A) (B) (C) (D) (E)

atlanto-axial instability aortic stenosis p ossible d ifficu lt intu bation m itral regu rgitation ventricu lar sep tal d efect (VSD)

139. A 29-year-old male su ffered a motorcycle crash and is u nd ergoing intram ed u llary nailing of the left fem u r. The accid ent occu rred 24 h ago. Shortly after intram ed u llary ream ing, the patient d evelops tachycard ia, hypotension and hyp oxem ia. There are no ST-segm ent changes on the five-lead EKG. What is the m ost likely find ing on the p atient's transesop hageal echocard iogram ? (A) Flu id collection arou nd the heart w ithou t any d iastolic collapse (B) Regional w all m otion abnorm alities in the anterolateral w all of the left ventricle (C) Large color flow via the intra-atrial sep tu m

(D) Distend ed right atriu m and ventricle, collapsed left atriu m and ventricle (E) Large color flow via the intraventricu lar septum 140. When Pa CO 2 and p H are m anaged by the alpha-stat method d u ring hypothermic card iop u lm onary byp ass, (A) the corrected p H is 7.4 (B) ABG resu lts w ill be corrected to cu rrent p atient tem p eratu re (C) the u ncorrected Pa CO 2 is 40 m m H g (D) the corrected Pa CO 2 is 40 m m H g 141. A 77-year-old w om an w ith coronary artery d isease and significant aortic stenosis d evelops m yocard ial ischem ia shortly after ind uction of anesthesia. Sh e is being treated w ith beta blockers and nitroglycerin, and her blood pressu re is being su p p orted w ith an infu sion of norep inep hrine. Throu ghou t this event the p atient's oxygen satu ration is 100%. Th e p atient continu es to be ischem ic. What statem ent abou t the u se of an intraaortic balloon p u m p (IABP) in this p atient is tru e? (A) An IABP is not ind icated and the p atient shou ld be p ut on card iop ulm onary byp ass. (B) An IABP is contraind icated becau se the p atient has aortic stenosis. (C) An IABP shou ld be p laced im m ed iately. (D) The tip of the IABP balloon has to be placed just distal to the coronary arteries. (E) An IABP is contraind icated d u e to the risk of leg ischem ia. 142. A 65-year-old fem ale w ith a history of severe aortic stenosis requ ires u rgent lap aroscop ic cholecystectom y. An im portant hem od ynam ic goal d u ring anesthetic care is (A) (B) (C) (D) (E)

d ecreased afterload slow heart rate d ecreased p reload high heart rate d ecreased contractility

Que s tions : 137–147

143. Cannon w aves in the central venou s p ressu re tracing (A) are cau sed by atrial fibrillation (B) can be seen w ith atrioventricu lar nod al rhythm s (C) resu lt from left atrial contraction against the closed m itral valve (D) w ill resolve w ith ventricu lar p acing (E) rep resent an artifact cau sed by air in the pressu re transd u cer system 144. The right lu ng, in the u p right p osition (A) (B) (C) (D) (E)

is the sm aller of the tw o has a single fissu re has three lobes receives 45% of total lu ng blood flow is less frequ ently involved in asp iration com p ared to the left

145. A 74-year-old patient is to u nd ergo for rightsid ed thoracotom y for resection of lu ng cancer. The patient's past med ical history is significant for m yocard ial infarction fou r m onths ago at w hich tim e the p atient u nd erw ent the p lacem en t of tw o d ru g-elu ting stents an d w as started on antip latelet therapy w ith clopid ogrel. Accord ing to the p atient's card iologist, the patient is to continue clop id ogrel throu ghou t the periop erative p eriod . The su rgeon is requ esting an analgesic strategy that w ou ld p rovid e the patient w ith abou t 36 h of p ostop erative p ain control to facilitate extu bation and early m obilization. Which one of the follow ing is the m ost effective and safest m ethod of analgesia for this p atient? (A) p ostop erative right intercostal block (B) lu m bar ep id u ral catheter (C) p atient controlled analgesia w ith IV op ioid (D) thoracic ep id u ral catheter (E) right p aravertebral block via continu ou s catheter

223

146. A patient w ith severe COPD and severe pu lm onary hyp ertension is to u nd ergo bilateral lu n g tran sp lan t. Th e en -bloc d ou ble-lu n g transp lant, as com p ared to bilateral sequ ential single-lung transp lantation (A) has a d ecreased need for blood transfu sions (B) has a low er incid ence of ischem ia at the site of tracheal anastom osis (C) resu lts in a higher need for card iop u lm onary byp ass (D) is technically easier (E) d oes not requ ire card iac arrest 147. A 68-year-old p atient is to u nd ergo a Whip ple p roced u re for resection of a tu m or of the head of the p ancreas. The patient's past m ed ical history is significant for m yocard ial infarction and the placem ent of a bare metal stent, hyp ertension, hypercholesterolem ia, and COPD second ary to tobacco abu se. The p reop erative ECG is significant for a right bu nd le branch block, and the p reop erative TTE is significant for d iastolic d ysfu nction w ith an ejection fraction of 45% as w ell as mod erate to severe aortic regu rgitation. In ad d ition to arterial blood p ressure, a d ecision is m ad e for intraop erative m onitoring w ith a pu lm onary artery catheter for estimation of left ventricular preload . Based on this patient's presentation, left ventricu lar p reload as assessed by p u lm onary artery catheter m easu rem ents w ill likely be (A) u nd erestim ated becau se of aortic regu rgitation (B) overestim ated d ue to the right bund le branch block (C) u nd erestim ated d u e to d ecreased left ventricular com p liance (D) u nd erestim ated becau se the patient w ill be on p ositive p ressu re ventilation (E) accu rately reflected by LAP, LVEDP, and PAOP

224

13: Ane s the s ia for Ca rdiothora cic a nd Va s cula r S urge ry

DIRECTION S: Use the follow ing figu re to answ er Qu estions 148-149:

(A) (B) (C) (D) (E)

Stanford A DeBakey II Stanford B Craw ford I Craw ford II

149. The risk of sp inal cord ischem ia w ith su rgical rep air of this typ e of lesion is ap p roxim ately (A) (B) (C) (D) (E)

2% 10% 20% 30% 40%

150. An 82-year-old p atien t w ith an terior tw ovessel coronary artery d isease and u nstable an gin a is referred by th e card iologist for CABG. H er p ast m ed ical history is otherw ise significant for typ e I d iabetes, chronic renal in su fficien cy, p erip h eral vascu lar d isease, COPD, and atherosclerotic ascend ing aortic d isease. The card iac su rgeon d ecid es to p erform off-p u m p CABG to the LAD and the RCA. Consid ering this patient's com orbid ities and the p rop osed p roced u re, it is tru e that

148. A 58-year-old m ale com es to the em ergency d epartment w ith d iffuse chest pain that is rad iating into his back. H is p ast m ed ical history is significant for tobacco and alcohol abu se, hyp ertension, and hyp ercholesterolem ia, as w ell as p erip heral vascu lar d isease. A w orku p for acu te coronary synd rom e is negative, and the patient und ergoes CT imaging of chest and abd om en w ith IV contrast that reveals a thoracoabd om inal aneurysm that is d eem ed sym p tomatic and the patient is sched uled to und ergo u rgent su rgical repair. Based on the d raw ing, w hich one of the follow ing classifications of this patient's aneurysm is accu rate?

(A) m inim ally invasive coronary artery byp ass grafting w ou ld be a su itable alternative (B) the su rgeon w ill requ ire a stabilization d evice for the d istal anastom oses (C) cooling of the p atient is requ ired as p art of the proced u re (D) hem od ynam ic goals are easier to achieve as com p ared to CABG w ith card iopu lm onary byp ass (E) the u se of p rop ofol for m aintenance of anesthesia w ill result in equ ivalent m yocard ial p rotection as com pared to volatile anesthetics 151. The p atient p op u lation m ost likely to benefit from transcatheter aortic valve im p lantation is (A) child ren and you ng ad u lts w ith congenital, noncalcific aortic stenosis (B) ad u lt p atients w ith severe aortic regu rgitation

Que s tions : 148–157

(C) asymptomatic adult patients with calcific aortic stenosis and severe obstru ction (D) ad u lt p atients w ith sym p tom atic severe aortic stenosis d eem ed too high risk for su rgery (E) ad u lt p atients w ith sym p tom atic severe aortic stenosis w ithou t other associated com orbid ities 152. A 60-year-old fem ale w as ad m itted w ith a large goiter and a history of hoarseness. An incid ental find ing on the chest x-ray w as tracheal d eviation w ith questionable narrow ing of the tracheal lu m en. After ind u ction w as com plicated by a d ifficult intubation requiring m u ltip le attem p ts, the thyroid w as rem oved w ith som e d ifficu lty and at the end of the p roced u re the p atient w as breathing sp ontaneously. Im m ed iately after extubation, breathing w as labored and retraction w as noted . Cau ses of this may includ e all of the follow ing EXCEPT (A) bilateral recu rrent laryngeal nerve inju ry (B) laryngosp asm (C) tracheal collap se (D) bronchosp asm (E) thyrotoxicosis

225

noted in the su rgical field . A throm boelastogram show s d ecreased m axim u m am p litu d e. The ap p rop riate treatm ent inclu d es ad m inistration of w hich one of the follow ing? (A) (B) (C) (D) (E)

Protam ine Cryop recip itate Fresh frozen p lasm a Am inocap roic acid Platelets

155. Structu res that pass anteriorly to the trachea inclu d e all of the follow ing EXCEPT (A) (B) (C) (D) (E)

p u lm onary artery thyroid isthm u s innom inate artery aortic arch left brachiocep halic vein

156. Ind ications for one-lu ng ventilation inclu d e all of the follow ing EXCEPT (A) (B) (C) (D)

infection w ith p u ru lent secretions m assive p u lm onary hem orrhage bronchop leu ral fistu la u nilateral bronchop u lm onary lavage for alveolar p roteinosis (E) Ivor-Lew is esop hagectom y

153. H yp oxic p u lm onary vasoconstriction (A) occu rs w hen regional atelectasis m echanically obstru cts blood flow (B) is p rim arily triggered by alveolar carbon d ioxid e tension (C) lead s to d iversion of blood aw ay from poorly ventilated areas of the lu ng (D) is p otentiated by ad m inistration of nitrou s oxid e (E) is au gm ented by an increase in p u lm onary artery pressu re 154. At the conclu sion of an aortic aneu rysm rep air associated w ith significant blood loss, d iffu se bleed ing and the absence of clot form ation is

157. Du ring aw ake, closed chest ventilation in the lateral d ecu bitu s p osition, (A) the lu ng relationship s are the sam e as in the sem irecu m bent p osition, i.e., the ap ex is in zone 1 and the bases are in zone 3 (B) ventilation is highest at the ap ex (C) p erfu sion is greater in the nond ep end ent lu ng (D) com p liance is u nequ al in the tw o lu ngs (E) the nond ep end ent lu ng receives m ost of the tid al ventilation

226

13: Ane s the s ia for Ca rdiothora cic a nd Va s cula r S urge ry

158. A 35-year-old m an is ad m itted to the em ergency d ep artm ent follow ing an au tom obile accid ent. It is noted that there is a contu sion over the anterior thorax, he is tachypneic, and he has a scap h oid abd om en. Au scu ltation reveals p oor breath sou nd s on the left sid e. Chest x-ray show s a large air cavity in the left sid e of the thorax. Blood pressu re is 80/ 60, and heart rate is 120 p er m inu te. Diagnoses that m ust be consid ered includ e all of the follow ing EXCEPT (A) (B) (C) (D) (E)

ru p tu red sp leen p neu m othorax d iap hragm atic hernia card iac contu sion fat em bolism synd rom e

159. A 69-year-old w om an is sched u led for rou tine CABG. Since her preoperative card iac catheterization, her p latelet cou nt has d rop p ed from 312 to 252 × 103/ m m 3. A hep arin-ind u ced throm bocytop enia (H IT) im m u noassay has been ord ered and it resulted in a p ositive H IT antibod y. She has a large bru ise at the site of her catheterization. There are no signs of d eep ven ou s th rom bu s or any other throm botic events. You shou ld (A) d elay surgery for 2 m onths (B) p roceed w ith su rgery and u se a d irect throm bin inhibitor instead of heparin (C) p roceed w ith su rgery and u se heparin (D) u se only 50% of the regu lar hep arin d ose and proceed w ith su rgery (E) u se a com bination of w arfarin and low d ose heparin to achieve ad equ ate anticoagulation 160. A 73-year-old p atient is und ergoing em ergent three-vessel CABG for sym ptom atic left m ain coronary artery d isease. Upon com pletion of the su rgical p roced u re, the p atient d evelop s

the need for high d ose inotropic su pport, and the d ecision is m ad e to im plant a left ventricu lar assist d evice (LVAD) to facilitate w eaning from card iop u lm onary byp ass (CPB). After d iscontinu ation of CPB, the p atient is noted to be hyp otensive w ith low p u m p flow rates ap p arent on the LVAD d evice. Im m ed iate TEE exam ination d oes not show any evid ence of inflow cannu la obstru ction, bu t is significant for systolic collap se of the right atriu m and d iastolic collap se of the right ventricle. The ECG show s low QRS voltage. The m ost likely cau se for this patient's hypotension is (A) (B) (C) (D) (E)

graft failu re acu te hyp ovolem ia right ventricu lar failu re p u lm onary em bolu s p ericard ial tam p onad e

161. The ad vantages of u ltrasou nd -gu id ed central venous catheter placem ent, as com pared to the land m ark techniqu e, inclu d e all of the follow ing EXCEPT (A) p revention of arterial inju ry (B) d irect visu alization of the target vessel (C) d ecreased tim e requ ired for internal ju gu lar vein catheterization (D) d ecreased nu m ber of attem p ts requ ired (E) d ecreased overall com p lication rate DIRECTION S (Qu estions 162-164): Each grou p of item s below consists of lettered head ings follow ed by a list of nu m bered p hrases or statem ents. For each nu m bered p hrase or statem ent, select the ON E lettered head ing or com p onent that is m ost closely associated w ith it and fill in the circle containing the correspond ing letter on the answ er sheet. Each lettered head ing or com p onent m ay be selected once, m ore than once, or not at all.

Que s tions : 158–165

227

lettered head ing or com p onent that is m ost closely associated w ith it. Each lettered head ing or com p on en t m ay be selected on ce, m ore th an on ce, or not at all. (A) (B) (C) (D) (E) (F) (G) (H ) (I) (J) (K) (L) (M) (N ) (O) For each p atient, select the ap p rop riate d iagram of the heart sou nd s. 162. A 64-year-old patient w ith a history of rheu m atic fever presenting for elective aortic valve rep lacem ent. 163. A 21-year-old fem ale w ith Dow n synd rom e and echocard iograp hic evid ence of right ventricu lar overload . 164. A 70-year-old m ale w ith a 60 p ack-year history of sm oking and a history of pneum onectom y for lu ng cancer w ith m ild resp iratory insu fficiency and evid ence of right ventricu lar strain on echocard iography. DIRECTION S (Qu estions 165-174): Each grou p of item s below consists of lettered head ings follow ed by a list of nu m bered p hrases or statem ents. For each nu m bered phrase or statem ent, select the ON E

Mid -esop hageal fou r cham ber Mid -esop hageal tw o cham ber Mid -esop hageal long axis Transgastric tw o cham ber Transgastric m id -p ap illary short axis Mid -esop hageal aortic valve short axis Mid -esop hageal aortic valve long axis Mid -esop hageal bicaval Mid -esop hageal right ventricu lar inflow -ou tflow Deep transgastric long axis Up p er esop hageal aortic valve short axis Up p er esop hageal aortic valve long axis Transgastric long axis Mid -esop hageal ascend ing aortic short axis Mid -esop hageal ascend ing aortic long axis

For each p hotograp h of a transesop hageal echocard iogram , select the stand ard , tw o-d im ensional tom ographic view. 165.

228

13: Ane s the s ia for Ca rdiothora cic a nd Va s cula r S urge ry

166.

169.

167.

170.

168.

171.

Que s tions : 166–174

172.

173.

174.

229

Answe rs a nd Expla na tions

94. (A) The best next step in m anagem ent is to initiate artificial card iac p acing, either via pacer cables still in situ, or via transcutaneou s pad s. Ind ications for artificial pacing includ e, bu t are not lim ited to: SA nod e d ysfu nction w ith sym p tom atic brad ycard ia as d escribed in this case, junctional rhythm that can som etim es be corrected by means of overd rive pacing, symptom atic chronotropic incom p etence, and atrial fibrillation w ith brad ycard ia an d p au ses greater than 5 sec. In settings ou tsid e the card iac ICU, ad m in istration of ch ron otrop ic agents su ch as epinep hrine m ight be ind icated on a tem porary basis as a brid ge to pacer therapy. (6:1876) 95. (E) N o pharm acological alternative m eets both goals. (6:2235) 96. (B) The inferior w all of the left ventricle is su p plied by the right coronary artery and is m ost app arent in lead s II, III, and AVF. (6:1836) 97. (C) An incom p etent tricu sp id valve p erm its right ventricu lar p ressu re to be transm itted to the right atriu m , cau sing increased am plitu d e of the v w ave. Increased a w aves occu r in heart block and ju nctional rhythm , w hile a w aves are absent in atrial fibrillation. (5:409; 6:1905) 98. (D ) At 17°C, cerebral oxygen consum ption is reduced to about 8% of the normothermic value and , w hile metabolic activity is d ecreased , cellular integrity is maintained . This accounts for the brain's tolerance to mod est period s of card iac arrest during hypothermia. Both cerebral blood flow and electrical activity d ecrease d uring hypothermia. (6:166)

230

99. (C) The external noninvasive u nits are ven tricu lar p acing d evices. (1:896; 4:817; 5:1084; 6:72) 100. (D ) The patient d escribed in this vignette likely has m ed iastinitis from esop hageal ru p tu re, acu te lu ng injury, and very little, if any, tolerance for apnea d uring airw ay instru mentation. The safest choice am ong the op tions listed to secure the airw ay and be able to p rovid e single lu ng ventilation is rap id -sequ ence ind u ction, follow ed by p lacem ent of a Univent tu be. The Univent is a single lu m en tu be w ith bu ilt in bronchial blocker that p asses throu gh a sm all channel w ithin the w all of the end otracheal tube. Other choices w ould inclu d e the Arnd t, or Cohen, end obronchial blockers, that are gu id ed w ith fiberop tic bron ch oscop y, or a w ire-gu id ed m echanism resp ectively. If a d ou ble-lu m en tu be is chosen, a left-sid ed d ou blelu m en tu be is the tu be of choice, how ever u nlikely to be tolerated by this p atient d u e to baseline hyp oxem ia. Placem ent of a stand ard single-lu m en tu be is unlikely to resu lt in ad equ ate lu ng sep aration, thu s m aking su rgical exp osu re far m ore d ifficu lt. (5:967, 985) 101. (E) Of the anticholinergic agents, glycop yrrolate and scopolamine are better antisialagogues than atropine. The other m ed ications listed are cholinesterase inhibitors that have increased salivation as a m ajor sid e effect. (5:973) 102. (A) CPAP w ith oxygen to the collap sed lu ng w ill d ecrease shu nt and im p rove the hyp oxem ia. The rem aining choices B, C, and D are listed in the ord er the clinician shou ld p roceed if ap p lication of CPAP to the non-d ep end ent

Answe rs : 94–112

lu ng d oes not resolve the hyp oxem ia. Op tion E w ou ld likely be less effective, and m ight lead to overd istention of the op erative lu ng. (5:972) 103. (B) Shivering is blocked w ith m uscle relaxants d u ring w arm ing and cooling bu t d oes not occur in very cold m u scle. (5:908) 104. (E) N itroglycerin is metabolized to nitric oxid e, an activator of gu anylate cyclase in vascular smooth muscle. The enzyme prod uces intracellu lar cyclic-GMP. The cyclic-GMP is d egrad ed by a sp ecific p hosp hod iesterase (PDE-5) that is inhibited by sild enafil. Profou nd hyp otension can occur w hen a sublingual d ose of nitroglycerin is given w ithin several hou rs of an oral d ose of sild enafil. Intravenou s nitroglycerin m ust be slow ly titrated in that setting. (1:752; 6:2025) 105. (E) Magnets cau se m ost p acem akers to p ace asynchronously at a preset rate, how ever som e d evices m ay be d eactivated p erm anently and have to be rep rogram m ed . Magnets d o not affect the p acing rate of ICD's. Since new er d evices are very com p lex, the u se of m agnets shou ld be reserved for em ergency situations, and the d evices have to be interrogated after com p letion of su rgery. The ind ifferent p late shou ld be placed aw ay from the p acem aker or ICD. If the p acer is in d em and m od e, it m ay sense the cautery current as d epolarization and shut off. (5:63, 1281)

231

sp ontaneou s respirations d u e to the high risk of airw ay com pression and card iovascular collap se resu lting from the loss of m u scle tone associated w ith ad m inistration of m u scle relaxants and the su p ine p osition. In m any cases these patients requ ire fiberoptic intubation. Du e to the im p aired venou s d rainage from the SVC to the heart, intravenou s lines shou ld be p laced in the low er extrem ities. (5:993-4) 108. (B) With m od ern pacem aker u nits, the energy ou tpu t is so low that there is little d anger of fibrillation. H ow ever, ventricu lar fibrillation is m ore com m on in the patient w ith a high catecholam ine concentration, m yocard ial infarction, or hypoxem ia. (6:1899) 109. (D ) The intraaortic balloon pu m p is d eflated d u ring systole. It is rap id ly inflated d u ring d iastole, thereby increasing coronary blood flow. It d ecreases im p ed ance to left ventricle ejection. It is contraind icated in the p atient w ith aortic insu fficiency. (6:2235-6) 110. (B) Clam p ing of the thoracic aorta lead s to an im m ed iate in crease in blood p ressu re. Assum ing the heart can w ithstand the m arked ly increased afterload , there shou ld not be stand still or hypotension. Blood pressure read ings m ay be lost in the left arm , bu t the p u lse in the right arm shou ld be p resent, d ep end ing on the placem ent of the clam p relative to the take-off of the vessels to the arm s. (5:1025-7)

106. (B) This patient has sym p tom s consistent w ith su perior vena cava (SVC) synd rom e. In ad d ition to the sym ptom s d escribed in (B), these p atients m ay have m u ltiorgan involvem ent inclu d ing neu rologic, resp iratory, card iac, gastrointestinal, and renal sym ptom s. The other find ings d escribed here m ight be fou nd in a p atient w ith thoracic ou tlet synd rom e in (A), congestive heart failure in (C), acute asthm a or a COPD exacerbation in (D), and hyp erventilation synd rom e in (E). (5:993-4; 6:2185)

111. (D ) A com p lication of aortic su rgery is sp inal cord ischem ia d ue to com p rom ise of the rad icu lar arteries. These arteries are not constant, and the large artery to the sp ine, the artery of Ad amkiew itz, may be compromised lead ing to ischem ia. Pressu re on the cord can also lead to ischem ia. H yp otherm ia m ay be p rotective. Loss of cerebrosp inal flu id is not a factor. Long p eriod s of hypotension m ay lead to ischem ia. (5:1025-8)

107. (D ) Patients w ith SVC synd rom e shou ld be kept u p right to facilitate venou s d rainage of the u p p er vessels, and if a general an esthetic is requ ired be ind u ced w hile m aintaining

112. (E) The exp ansion of the vascu lar bed that occu rs follow s the release of the clam ps. It is d u e to a reactive vasod ilatation. Other factors, such as blood loss, inadequate fluid replacement,

232

13: Ane s the s ia for Ca rdiothora cic a nd Va s cula r S urge ry

vasoactive d ru gs, and extravasation of flu id , are all im p ortant. (5:1028-9) 113. (C) The flow rate on total byp ass can be varied to achieve the p erfu sion p ressu re that is d esired . This m u st be varied w ith the state of resistance. (5:899) 114. (A) When the patient is on bypass, the pressure is usu ally kep t above 50 m m H g. Low er pressu res can be tolerated , bu t there is no agreem ent on an y sp ecific p erfu sion p ressu re. (5:899) 115. (A) The overrid ing goals of anesthetic care for carotid end arterectom y (CEA) su rgery are hemod ynamic stability and prompt emergence from anesthesia at the conclu sion of the p roced u re to facilitate n eu rologic exam in ation . There is u niversal agreem ent that the p atient's blood p ressu re shou ld be m aintained at, or 20% higher, than the highest record ed aw ake blood p ressu re to m aintain ad equ ate collateral cerebral p erfu sion d u ring carotid clam p ing. Exp erience w ith processed EEG m onitors su ch as the BIS is still lacking, and the raw EEG provid es higher sensitivity for the d etection of intraop erative cerebral ischem ia. There is no conclusive evid ence in the literatu re su ggesting an ad vantage of volatile anesthetics over p rop ofol w ith regard to brain p rotection d u ring CEA. Preop erative clam p tests are not ind icated , and general anesthesia com p ared to regional anesthesia ap p ears to resu lt in id entical ou tcom es, w ith the p ossible excep tion of p atients w ith contralateral carotid occlu sion w ho m ight benefit from regional anesthesia. (5:1018-21) 116. (C) Wall motion abnormalities d etected by TEE gen erally p reced e isch em ic ST-ch an ges d etected on ECG. N one of the other op tions is su itable for the d etection of intraop erative ischem ia. (5:1867) 117. (A) Strategies to mitigate the risk of spinal cord isch em ia in th is h igh -risk p atien t in clu d e p lacem ent of a lu m bar d rain, as w ell as neurop h ysiologic m onitoring su ch as SSEP's or MEP's. Intraoperative EEG monitoring w ill not

p rovid e inform ation abou t sp inal cord ischem ia, and is consid ered the gold stand ard for monitoring the brain d uring CEA. Hypotension is to be avoid ed since it w ould increase the risk of ischem ia. DH CA is a techniqu e em ployed d u ring op en rep air or rep lacem ent of the thoracic aorta. Patients u nd ergoin g su ccessfu l stent graft repair of the thoracic aorta are routinely extubated at the end of the p roced ure, and a high-d ose op ioid techniqu e d oes not p rovid e sp inal cord p rotection. (5:915, 940, 1019) 118. (C) The hem od ynam ic goals for this p atient shou ld includ e a slow heart rate and augm entation of preload to ensure preservation of forw ard flow th rou gh th e sten osed valve. Tachycard ia and increase in afterload and contractility cou ld d ecrease card iac ou tp u t and increase m yocard ial oxygen d em and resp ectively. (5:910) 119. (C) Esop hagectom y patients are generally malnou rished . They are at risk for aspiration, since esop h ageal fu n ction is frequ en tly com p rom ised . Th eir p u lm on ary fu n ction is equ al betw een their lu ngs, and thu s they are m ore likely to have a significant shu nt d u ring onelung ventilation. The surgery is more extensive than m ost thoracotom ies, and the p atients m ore frail, so p ostop erative ventilation is com m on. (5:983) 120. (E) The larynx lies at the level of the 3rd --6th cervical vertebrae, and in the ad u lt, the narrow est p ortion is at the level of the vocal cord s. The larynx is innervated by the recu rrent laryngeal nerve and the su p erior laryngeal nerve. It lies w ithin the thyroid cartilage. The w id e m argin of the cricoid cartilage is p osterior. (5:548) 121. (D ) The innom inate artery passes anterior to the trachea and can be com p ressed by the m ed iastinoscope. The right su bclavian artery is a branch of the innom inate artery, w hile the left su bclavian artery com es off the aorta d irectly and d oes not cross in front of the trachea. Com pression of flow in the innom inate w ill also obstru ct flow to the right carotid

Answe rs : 113–131

artery, and m ay com p rom ise cerebral p erfu sion in patients w ith cerebrovascu lar d isease or an incom p lete circle of Willis. Air in the m ed iastinum shou ld have little hem od ynam ic effect. Card iac arrest, su p erior ven a cava obstru ction, and anesthetic overd ose can all cause p ulselessness, bu t rarely as sud d enly or reversibly. H aving a m onitor of p erfu sion on the other arm , such as an oxim eter probe, can h elp d ifferentiate inn om inate com p ression from other cau ses of circu latory com p rom ise. (5:988) 122. (E) In any p atient w ith hoarseness and a d ocu m ented su praglottic m ass, it is m and atory to establish the airw ay before p roceed ing w ith the anesthesia. Aw ake fiberop tic intu bation, aw ake tracheostom y (all w ith top ical anesthesia), or sp on tan eou s ven tilation w ith an inhaled anesthetic are the only safe m ethod s. (5:1234-5) 123. (D ) Asp iration is possible, since the esop hagu s m ay contain barium from the exam ination as w ell as oth er oral in take. Th e esop h agu s should be suctioned before ind uction, although the thick bariu m suspension m ay not be com p letely rem oved . (5:983) 124. (C) By clam ping at 5, gas cannot flow into the left lumen, and thus to the left lung. Uncapping at 3 allow s the gas in the left lu ng to escap e. Clam ping at 1 w ou ld p revent flow to the left sid e, bu t uncapping w ou ld cause a leak in the anesthetic circu it. Assu m ing good tu be position and isolation, the hand ed ness of the tube d oesn't m atter. (5:963-5) 125. (D ) Throu gh p osition 4, one is looking d ow n the right lum en. From the right lu m en, no left lu ng segm ental bronchi can be seen w ith any typ e of p rop erly p ositioned d ou ble-lu m en tu be. The right lu m en w ill be the tracheal lu m en on a left-sid ed tu be, in w hich case the carina can be seen. (5:965) 126. (B) All the step s are correct for ventilating via the left lu m en w hile p ou ring flu id into the right lu m en excep t that clam p ing at 2 w ill

233

block the flu id flow. The cu ffs shou ld be u p to ensu re lu ng isolation. (5:978) 127. (D ) Clam p ing at 6 isolates the right lu ng. (5:963-5) 128. (D ) The Arnd t bronchial blocker can be used in p atients w ho are alread y intubated , so there is no need to change the end otracheal to a d ouble-lu m en tu be shou ld one-lu ng ventilation be requ ired . It can also be u sed for p atients w ith a d ifficu lt airw ay and p atients w ith trau m a w ho require one-lu ng ventilation. The Univent tu be is a single lu m en end otracheal tu be w ith an integrated , m ovable bronchial blocker. The Sengstaken-Blakem ore tu be is an esop hageal d evice u sed for tem p orary control of intractable variceal bleed ing. (5:967) 129. (A) Positive p ressu re ventilation d oes not have to be interru p ted d u ring p lacem ent of the bronchial blocker becau se of the m u ltip ort ad ap ter that w ill accom m od ate the breathing circu it, a fiberop tic bronchoscop e and the end obronchial blocker. The d evice has to be p laced u nd er fiberop tic bronchoscop ic gu id ance and its u se is not lim ited to only the left or right lu ng. If the p atient continu es on m echanical ventilation u pon conclu sion of surgery, the end obronchial balloon is d eflated and the d evice rem oved w hile the end otracheal tu be rem ains in place. (5:967) 130. (D ) Th e carbon d ioxid e ten sion rises abou t 6 m m H g d u ring the first m inu te of ap neic oxygenation and abou t 3-6 m m H g p er m inu te thereafter. It w ill rise more quickly in ind ivid uals w ith a low FRC. All arrhythm ias are not d ue to hypoxem ia but m ay be d ue to increased catecholamines. These in turn may be increased d u e to increased carbon d ioxid e. Pu lse oxim etry has been of d efinite benefit in these proced u res. (5:551) 131. (A) The Thebesian veins are the sm allest veins in the heart and d rain d irectly into the ventricles, w hereas the rem aind er of the coronary arterial blood is d rained via the venou s system into the coronary sinu s and eventu ally into the

234

13: Ane s the s ia for Ca rdiothora cic a nd Va s cula r S urge ry

right atriu m . All other op tions are correct. (5:902, 951) 132. (B) The hem od ynam ic goal of anesthetic care is the red u ction of the regu rgitant fraction that can be achieved by augm entation of preload , mild ly increased heart rate (i.e., ad ministration of low -d ose glycopyrrolate), and avoid ance of increased afterload . (5:910) 133. (B) Ad m inistration of high-d ose fentanyl w ill resu lt in brad ycard ia, bu t blood p ressu re w ill rem ain relatively stable. Fentanyl, as op p osed to m orphine, d oes not release histam ine. The card iac ou tpu t w ill d ecrease to a m ild d egree. Morp hine-ind u ced changes are m ore p ronou nced . (5:711-2) 134. (D ) While som e of the p harm acological strategies m entioned have been show n to p reserve renal fu nction in anim al m od els, this has not been conclu sively show n in hu m ans. Am ong the strongest pred ictors of postoperative renal d ysfunction is aortic cross clam p tim e, particularly if the aorta is clam ped for m ore than 50 min. Of note, even the infrarenal location of the aortic cross clam p can cau se changes in renal blood flow that in tu rn can contribu te to p ostoperative renal d ysfu nction. The m ost significant p rogress in p revention of acu te kid ney inju ry d uring aortic surgery has com e from the w id esp read ad op tion of end ovascu lar techniqu es. (5:1027-8) 135. (C) The rise in PA p ressu re is d u e to left ventricu lar d ysfu nction resu lting in a rise in left ventricu lar end d iastolic p ressu re (LVEDP). The goal in this situ ation is to d ecrease LVEDP, w hich w ill translate into low er PA p ressu res, in ord er to im p rove m yocard ial p erfu sion. N itroglycerin in d u ces ven od ilation , th u s d ecreasing ventricular filling, transmural pressu re, and m yocard ial w ork. A sid e effect of nitroglycerin is hyp otension that is cou nterp rod u ctive and often treated sim u ltaneou sly w ith phenylephrine. While milrinone decreases PA p ressu re it lead s to an increase in m yocard ial oxygen con su m p tion . N itric oxid e d ecreases the PA p ressu re as w ell, bu t it m ight not have a significant effect on LVEDP. Since the

p atient in this vignette is neither hypoxic nor hypercarbic, increased ventilation and oxygenation are u nlikely to d ecrease PA p ressu re. (1:747-55) 136. (B) Tachycard ia in p atients w ith coron ary artery d isease is controlled w ith beta blockers. They d ecrease heart rate and m yocard ial contractility, resulting in better coronary perfusion and less oxygen d em and . Observation only is not ind icated . N icard ipine may low er the heart rate, bu t d oes not red u ce contractility. (1:326) 137. (C) This patient has the sym ptom s of im m ed iate p ostop erative card iac tam p onad e: hyp oten sion , tach ycard ia, equ alization an d elevation of PA d iastolic p ressu re and CVP, and low card iac ind ex. While norep inep hrine m ight help to su p p ort the blood p ressu re, it d elays d efinitive care. Milrinone w ould ad d to the hypotension, and so w ou ld nitroglycerin. Chest CT is not ind icated based on the hem od ynamic abnormalities and w ould delay definitive therap y. (5:458, 913; 6:1975) 138. (C) Marfan synd rom e is associated w ith aortic changes m aking d issection m ore com m on. H yp ertension shou ld be avoid ed . The higharched p alate m ay m ake intu bation m ore d ifficu lt. Aortic stenosis is not seen, bu t aortic insu fficiency is com m on. Atlanto-axial instability and / or ventricu lar sep tal d efect are seen w ith congenital synd rom es su ch as Dow n and achond rop lasia. (5:271-2) 139. (D ) The m ost likely d iagnosis is fat em bolism synd rom e. The typ ical TEE p ictu re for any large (p u lm onary) em bolu s is an overfilled right sid e of the heart, and significantly u nd erfilled left sid e. Flu id arou nd the heart is ind icating a p ericard ial effu sion or tam p onad e; this p atient is alread y 24 h p ost-trau m a and there is no collapse of the patient's heart cham bers that rend ers the d iagnosis of tam p onad e u nlikely. Regional w all m otion abnorm alities in d icate a p ossible ischem ic even t that is u nlikely d u e to this patient's age and the lack of ST-changes on a 5 lead EKG. Large color flow s across the sep ta on echocard iograp hy

Answe rs : 132–148

ind icate an ASD or a VSD that are u nlikely to ap pear acu tely in this situ ation. (5:1203) 140. (C) The other resp onses p ertain to the p H -stat m ethod . (5:532) 141. (C) Becau se p harm acologic attem p ts at reversin g isch em ia h ave failed , in sertion of an intraaortic balloon is ind icated . The treatm ent of m yocard ial ischem ia inclu d es correcting the hem od ynam ics (slow -norm al heart rate, highnorm al blood pressure, and low ering of the PA p ressu re) and treatm ent of severe anem ia and hypoxem ia. Since the event occu rred shortly after ind u ction, large blood loss and severe anem ia are u nlikely. Im m ed iate initiation of card iopu lm onary bypass in this situation is an option, bu t is im practical since the su rgery is ju st beginning. Absolu te contraind ications for p lacem ent of an IABP are aortic regu rgitation and aortic d issection. The p u m p can be p laced w ith the help of TEE, x-ray or even by ap p roxim ation, if TEE and x-ray are not available. The tip of the balloon has to be placed in the p roxim al thoracic aorta just d istal to the left subclavian artery. IABP therap y m ay cau se leg ischem ia, how ever in this situation the benefits of IABP (increased coronary blood flow ) outw eigh the p otential for leg ischem ia, for w hich the p atient need s careful p ostop erative observation. (6:2234-5; Kar B, et al., Circulation 2012; 125:1809-17). 142. (B) Slow heart rate and m aintenance of afterload are the hemod ynamic goals for the patient w ith severe aortic stenosis, w hile d ecreased afterload and higher heart rate are p referred in p atients w ith aortic and m itral regu rgitation. Decreased contractility is d esired in hyp ertrop hic obstructive card iom yop athy (H OCM) to avoid left ventricu lar ou tflow obstru ction . (5:910) 143. (B) Can non w aves ap p ear w hen the right atriu m contracts after closu re of the tricusp id valve, and is not d u e to artifact. The a-w ave is not p resent w ith atrial fibrillation and can be cau sed by ventricu lar p acing and resu lting AV-d issociation . Effectiv e restoration of AV-synchrony w ith atrial or AV-p acing w ill

235

restore the norm al CVP trace. Loss of the atrial com ponent of ventricu lar filling d uring ju nctional rhythm m ay be poorly tolerated . (5:409; 6:1823) 144. (C) The right lu ng is the larger of the tw o. It has three lobes (u pp er, m id d le, and low er) and tw o fissures (horizontal and oblique). Since it is the larger of the tw o, it receives 55% of total blood flow. Since the axis of the right mainstem bronchu s is m ore in line w ith the trachea, the right lu ng is m ore frequ ently involved in cases of asp iration as com p ared to the left. (5:951, 959, 1002) 145. (E) Becau se the p atient is on antip latelet therap y w ith clop id ogrel, neu raxial techniqu es are contraind icated . An intercostal block w ill only last from 4–8 h; an intrap leu ral catheter w ould be m ore su itable bu t is not listed as an op tion in the qu estion. IV PCA is u nlikely to p rovid e the sam e level of analgesia and is associated w ith greater sid e effects as comp ared to a p aravertebral catheter that is consid ered safe in p atients on antiplatelet therap y. (5:1007-8) 146. (C) With the excep tion of op tion C, all of the op tions listed are ad vantages of bilateral sequ ential single-lung transplantation that has the ad d itional ad vantage of not requiring full anticoagu lation. (5:1091) 147. (A) Left ventricu lar p reload in this p atient w ill likely be u nd erestim ated d u e to aortic regu rgitation, w hich lead s to a d iscrepancy betw een LAP and LVEDP, d u e to continu ed ventricu lar filling after mitral valve closure. A right bundle branch block and d ecreased pu lm onary vascu lar bed su ch as in COPD w ill also m ake u nd erestim ation of left ventricu lar p reload m ore likely, w hile p ositive p ressu re ventilation and d ecreased left ventricular compliance w ill lead to overestim ation of left ventricu lar p reload . (5:414-5) 148. (E) The figu re show s a thoracoabd om inal aneu rysm classified as Craw ford typ e II that involves the entire d escend ing thoracic aorta w ith extension across the d iap hragm throu gh the abd om inal aorta to the aortic bifu rcation.

236

13: Ane s the s ia for Ca rdiothora cic a nd Va s cula r S urge ry

Stanford A and B as w ell as BeBakey I through III are classifications of aortic d issections. (5:918; 6:2063)

p rod u ction of carbon d ioxid e and increased w ork of breathing, its onset w ou ld be m ore grad u al. (5:151, 575-6)

149. (C) The risk of spinal cord ischemia w ith either open or end ovascu lar rep air is approxim ately 20% for this typ e of lesion. (5:917)

153. (C) H yp oxic p u lm on ary vasocon striction (H PV) is a constriction of p u lm onary arteries in resp onse to alveolar hyp oxia. Atelectatic lu ngs have id entical d egrees of H PV to those ventilated w ith nitrogen, exclu d ing m echanical factors. H PV can occur in d enervated lungs (e.g., after tran sp lan tation ). H yp ocap n ia d ecreases H PV, bu t hyp ercap nia has no effect. H yp oxia is th e p rim ary trigger of H PV. Alkalosis and acid osis both d ecrease H PV, as d oes ad m inistration of nitrou s oxid e. The p u lm onary vasoconstrictor resp onse to hypoxia is d ecreased w ith increases in p u lm onary artery p ressu re, card iac ou tp u t, left atrial p ressu re, or central blood volu m e. (5:612-3, 967-70)

150. (B) While off-p u m p CABG is a su itable app roach for anterior, inferior, and lateral vessels, m inim ally-invasive CABG, u tilizing the internal thoracic (m am m ary) artery, is su itable for one- or tw o-vessel d isease on the left sid e of the heart. While the hem od ynam ic goals for patients und ergoing CABG w ith card iopu lmonary bypass, and those und ergoing off-p u m p CABG are the sam e, they are hard er to attain w ith the latter ap p roach d u e to the m obilization of the heart that m ay cau se arrhythm ia and hem od ynam ic instability, p articu larly w hen the d istal anastom oses are p erform ed . Myocard ial p rotection ap p ears to be better w ith the u se of volatile anesthetics as com pared to propofol, reflected in less m yocard ial injury in the first 24 h p ostop eratively. Patients u nd ergoing off-p u m p CABG p roced u res are kep t norm otherm ic. (5:907-8) 151. (D ) Cu rrent ind ications for transcatheter aortic valve im plantation inclu d e ad ult patients w ith severe aortic stenosis d eem ed too high risk for surgery. The proced ure is currently being evalu ated as an alternative to reop eration in patients w ith p rosthetic valve failure not d u e to paravalvular regurgitation. The patient popu lation d escribed in op tion A is u su ally referred for percu taneou s balloon aortic valvu lop lasty, w hile asym p tom atic p atients w ith severe obstruction are u sually follow ed carefu lly w ith serial echocard iogram s u ntil they m eet op erative criteria. (6:1941) 152. (E) N erve inju ry is com m on w ith d ifficu lt d issections. Laryngospasm m ay be present d ue to secretions or injury to the vocal cord s d uring d ifficu lt intu bation. Tracheal collap se m ay be present d u e to tracheom alacia. Bronchosp asm m ay be a reason for the d ysp nea d ue to airw ay sensitivity. While thyrotoxicosis can cau se resp iratory failu re secon d ary to in creased

154. (E) A throm boelastogram (TEG) m easures the clot strength over tim e. The am plitud e of the grap h refers to the clot strength at a given tim e d u ring the clot form ation. The m axim u m strength (or amp litu d e) correlates w ith platelet fu nction. Protam ine w ou ld be given to reverse the effects of heparin. To d ifferentiate betw een the effects of heparin ad m inistered intraoperatively and other causes of coagulopathy, tw o TEG sam p les need to be analyzed : one w ith, and one w ithout, heparinase. Plasma and cryop recip itate w ou ld be ind icated for slow onset and slow form ation of clotting on the TEG, resp ectively (p rolonged R-valu e and d im inished angle A). Am inocap roic acid is ad m inistered for states of hyperfibrinolysis. (5:194) 155. (A) The p u lm onary artery is cau d al to the carina. (5:988) 156. (E) The other options are indications for one-lung ventilation. In the setting of infection with purulent secretions, the goal is to avoid spillage and contamination of the contralateral lung. During surgery for bronchopleural fistula, the goal is to control the distribution of ventilation to the unaffected lu ng. Other p roced u res w ith high p riority for one-lung ventilation includ e thoracic aortic aneurysm repair, pneumonectomy, upper lobectom y and vid eo-assisted thoracoscop ic

Answe rs : 149–166

su rgery (VATS), however the indication is not absolute. (5:963, 1038) 157. (D ) In the lateral position, lu ng relationships change. While there are three d istinct zones as in the u p right p atient, these are d istribu ted along a vertical grad ient in the lateral position. Perfusion is therefore greater in the d epend ent lung that also receives most of the tid al ventilation. Com p liance d iffers betw een the lu ngs becau se the d ep end ent lu ng is at a low er FRC d u e to the w eight of the abd om inal contents and m ed iastin u m . The d ep end en t alveoli therefore are on the steep portion of the transp u lm onary p ressu re-alveolar volu m e cu rve that exp lains their greater share of the tid al ventilation (5:961-2) 158. (E) In the p atient w ith blu nt anterior chest inju ry, resp iratory d istress, and hyp otension, all of the d iagnoses listed w ith the excep tion of op tion E m u st be consid ered . Fat em bolism synd rom e is m ore likely to affect p atients w ith m ajor low er extrem ity or p elvic fractu res. (5:1351, 1362) 159. (C) Su rgery shou ld proceed w ith u sing heparin. It rem ains the first-line anticoagu lant for p atients und ergoing card iac su rgery and can be read ily reversed w ith p rotam ine at the conclusion of su rgery. The patient d escribed in this vignette d oes not fit the d iagnosis for H IT. The large hem atom a d escribed m ight exp lain the d rop in p latelets that is less than 50% from baselin e. An tibod y serocon version in th e absence of clinical signs d oes not confirm the d iagnosis of H IT. The p atient's su rgery shou ld not be d elayed nor should alternative m ethod s of anticoagu lation be pu rsu ed . Direct throm bin inhibitors su ch as argatroban and bivaliru d in cannot be reversed. Low er doses of heparin are not ind icated , and w ou ld increase the risk of clotting w ithin the card iopulm onary bypass m achine. (5:900) 160. (E) The m ost com m on causes for hypotension in the setting of low LVAD p u m p flow rates in ad d ition to inflow cannu la obstru ction are options B through E. The latter is m ost likely in this scenario given the TEE find ings as w ell as

237

the low voltage ECG. Ad d itional find ings on TEE evalu ation m ay inclu d e p ericard ial effu sion as w ell as equ alization of cham ber pressures on pulmonary artery catheter monitoring. Graft failure is less likely based on the scenario given. (5:925) 161. (A) The u se of u ltrasou nd technology, w hile p rovid ing several ad vantages, d oes not com p letely p revent arterial inju ry. A thorou gh u nd erstand ing of anatom y as w ell as form al training in u ltrasou nd techniqu e is param ou nt in avoid in g th ese typ es of com p lication s. (5:411) 162. (D ) Dep icted here is reversed or parad oxical sp litting of the second heart sound that can be fou nd in aortic stenosis, as w ell as LBBB. Exam ination find ings consistent w ith severe AS w ou ld inclu d e p arvu st et tard u s carotid u p strokes, a late-p eaking grad e 3 or greater m id systolic m u rm u r, as soft A 2, a su stained LV ap ical im p u lse, and an S4. (6:1827-8) 163. (B) The p atient has a congenital atrial sep tal d efect that is com m on in Dow n synd rom e, often first encou ntered in the ad u lt life, and m ore com m on in fem ale patients. The associated murmur is mid-systolic in nature. (6:1827-8, 1921) 164. (E) The p atient has p u lm onary hyp ertension after p neu m onectom y that w ill m anifest as narrow splitting of S2. Pre-existing im pairment of pu lm onary vascular com p liance associated w ith CH F, and cor p ulm onale m ay be exacerbated after extensive lu ng resection, lead ing to seriou s p u lm onary hyp ertension and rightsid ed heart failu re. (5:954; 6:1827-8) 165. (I) (Lauer R, M athew JP. Transesophageal tomographic views. In: M athew JP, et al., eds., Clinical M anual and Review of Transesophageal Echocardiography, 2nd ed., N ew York: M cGrawHill, 2010, Figure 5-27) 166. (G) (Lauer R, M athew JP. Transesophageal tomographic views. In: M athew JP, et al., eds., Clinical M anual and Review of Transesophageal

238

13: Ane s the s ia for Ca rdiothora cic a nd Va s cula r S urge ry

Echocardiography, 2nd ed., N ew York: M cGrawHill, 2010, Figure 5-20) 167. (D ) (Lauer R, M athew JP. Transesophageal tomographic views. In: M athew JP, et al., eds., Clinical M anual and Review of Transesophageal Echocardiography, 2nd ed., N ew York: M cGrawHill, 2010, Figure 5-14) 168. (E) (Lauer R, M athew JP. Transesophageal tomographic views. In: M athew JP, et al., eds., Clinical M anual and Review of Transesophageal Echocardiography, 2nd ed., New York: McGraw-Hill, 2010, Figure 5-11) 169. (C) (Lauer R, M athew JP. Transesophageal tomographic views. In: M athew JP, et al., eds., Clinical M anual and Review of Transesophageal Echocardiography, 2nd ed., New York: McGraw-Hill, 2010, Figure 5-9) 170. (A) (Lauer R, M athew JP. Transesophageal tomographic views. In: M athew JP, et al., eds., Clinical M anual and Review of Transesophageal Echocardiography, 2nd ed., New York: McGraw-Hill, 2010, Figure 5-7)

171. (H) (Lauer R, M athew JP. Transesophageal tomographic views. In: M athew JP, et al., eds., Clinical M anual and Review of Transesophageal Echocardiography, 2nd ed., N ew York: M cGrawHill, 2010, Figure 5-25) 172. (B) (Lauer R, M athew JP. Transesophageal tomographic views. In: M athew JP, et al., eds., Clinical M anual and Review of Transesophageal Echocardiography, 2nd ed., N ew York: M cGrawHill, 2010, Figure 5-8) 173. (L) (Lauer R, M athew JP. Transesophageal tomographic views. In: M athew JP, et al., eds., Clinical M anual and Review of Transesophageal Echocardiography, 2nd ed., N ew York: M cGrawHill, 2010, Figure 5-34) 174. (F) (Lauer R, M athew JP. Transesophageal tomographic views. In: M athew JP, et al., eds., Clinical M anual and Review of Transesophageal Echocardiography, 2nd ed., New York: McGraw-Hill, 2010, Figure 5-19)

CHAPTER 14

Ane s the s ia fo r Ne uro s urg e ry Que s tions DIRECTION S (Qu estions 175-227): Each of the num bered item s or incom plete statem ents in this section is follow ed by answ ers or by com pletions of the statem ent. Select the ON E lettered answ er or com pletion that is BEST in each case. 175. As the tem p eratu re of the brain d ecreases (A) MAC increases (B) au toregu lation of blood flow is lost (C) cerebral m etabolic rate d ecreases 6% to 7% p er d egree Celsiu s (D) cerebral Q 10 d ecreases (E) brain oxygen extraction increases 176. Of the m any factors affecting intracerebral blood flow, w hich one of the follow ing is a correct d escrip tion? (A) Vasom otor paralysis: vasoconstriction of vessels in or near ischem ic areas (B) Au toregu lation: ability of vessels to resp ond in a m anner consistent w ith m aintaining hom eostasis (C) Lu xu ry p erfu sion: m etabolic requ irem ents in excess of blood flow (D) Intracerebral steal: d ecrease of blood flow in norm al areas w ith increased flow to ischem ic areas (E) Inverse steal: d iversion of flow to norm al areas from ischem ic areas 177. Which one of these is the best agent to d ecrease cerebral oxygen requ irem ent? (A) A m u scle relaxant (B) A glu cose solu tion (C) An anticonvu lsant

(D) A barbitu rate (E) Oxygen by m ask 178. Use of su ccinylcholine to facilitate end otracheal intu bation in p atients w ith increased intracranial p ressu re is associated w ith (A) (B) (C) (D)

increased intracranial p ressu re no change in intracranial p ressu re incom p lete m u scle relaxation cond itions m ore satisfactory than those w ith the u se of p ancu roniu m (E) hyp erkalem ia DIRECTION S: Use the follow ing scenario to answ er Qu estions 179-180: A 45-year-old w oman is undergoing a coil embolization procedure to obliterate a basilar tip aneurysm. She suffers from mild hypertension for w hich she takes hyd rochlorothiazid e. She is anesthetized w ith 2% sevoflurane in oxygen and vecuronium for m uscle relaxation. The catheter approaches the aneu rysm throu gh the vertebral artery. On d eploying the coil, there is a sud d en increase in arterial blood pressure and global d epression of ST segments d etected by ECG. 179. The m ost likely d iagnosis is (A) hyp ertensive crisis (B) d issection of the vertebral artery and brain stem ischem ia (C) arterial air em bolism to the p osterior circulation and hind brain ischem ia (D) p ain from m anip u lation of the intra-arterial catheter (E) ru p tu re of the aneu rysm w ith su barachnoid hem orrhage

239

240

14: Ane s the s ia for Ne uros urge ry

180. Treatm ent shou ld inclu d e all of the follow ing EXCEPT (A) hyp erventilation (B) em ergent p lacem ent of an external ventricular d rainage catheter (C) p rop ofol bolu s follow ed by continu ou s infu sion (D) im m ed iate increase in sevoflu rane concentration to red uce arterial blood p ressu re (E) im m ed iate angiograp hy and continu ed p lacem ent of coils 181. In th e artificially ven tilated n eu rosu rgical patient, PEEP (A) shou ld be u sed rou tinely (B) shou ld be u sed only on selected p atients w ith the head of the p atient never elevated (C) has no effect on intracranial p ressu re (D) shou ld be w ithheld in all cases (E) shou ld be titrated against requ irem ents for oxygenation and neu rologic statu s 182. Treatm ent of the neu rosu rgical p atient w ith m annitol m ay be follow ed by all of the follow ing EXCEPT (A) (B) (C) (D) (E)

initial hyp ervolem ia d ecreased u rine volu m e hyp ovolem ia d ecreased central venou s p ressu re a d ecrease in arterial p ressu re

183. N itrou s oxid e shou ld be avoid ed in p atients w ith (A) (B) (C) (D) (E)

brain tu m or subarachnoid hem orrhage closed head injury p neu m ocephalus subd ural hem atom a

184. To obtain maximum benefit from hyperventilation d u ring a neu rosu rgical p roced u re, the Pa CO 2 shou ld be m aintained at

(A) (B) (C) (D) (E)

15 to 20 to 25 to 35 to 40 to

20 m m 25 m m 30 m m 40 m m 45 m m

Hg Hg Hg Hg Hg

185. Following closed head injury, systemic sequelae may includ e all of the follow ing EXCEPT (A) d issem inated intravascu lar coagu lation (B) d iabetes insip id u s (C) synd rom e of inap p rop riate secretion of antid iu retic horm one (D) hyp erglycem ia (E) hyp ocarbia 186. An intraop erative "w ake u p " test p erform ed d u ring su rgery on the spine (A) assesses integrity of the d orsal sp inal cord (B) is not necessary if som atosensory evoked p otentials are m onitored (C) assesses sensory fu nction of the u p p er extrem ity (D) is intend ed to assess recall (E) can be associated w ith venou s air em bolism 187. Electroconvu lsive therap y (ECT) (A) is relatively contraind icated in p atients w ith know n cerebral or aortic aneu rysm s (B) never p rod u ces a seizu re (C) is not contraind icated in p atients w ith intracranial m ass lesions (D) d oes n ot requ ire h em od yn am ic m on itorin g (E) cannot be p erform ed w ith m u scle relaxants 188. Attention m u st be given to the valu e of intracr an ial p ressu re on in d u ction becau se increased intracranial p ressu re m ay lead to (A) herniation of brain tissu e (B) increased cerebral blood flow

Que s tions : 180–194

(C) elevation in cerebral p erfu sion p ressu re (D) brain retraction (E) increased CSF volu m e DIRECTION S: Use the follow ing scenario to answ er Qu estions 189-192: A p atient u nd ergoing a craniotom y in the sitting position has both a rad ial artery and a right atrial p ressu re catheter in p lace. The external aud itory canal is 26 cm above the level of the right atriu m (5 cm below the m anu briu m ). The cranium is open and the brain exp osed . 189. With the arterial p ressu re transd u cers located at the level of the right atriu m , the m ean arterial blood pressu re is 90 m m H g and the central venou s pressu re is 5 m m H g. What is the cerebral p erfu sion pressu re? (A) (B) (C) (D) (E)

95 85 70 59 Cannot be d eterm ined d irectly

190. If the arterial catheter transd u cer w ere rep ositioned to the level of the external au d itory canal, then (A) the MAP w ou ld not requ ire correction to m easure perfusion pressu re at the base of the brain (B) the m easu red MAP w ou ld rem ain the sam e if the arm w ere not elevated (C) the sam e effect cou ld be accom p lished by elevating the arm to the level of the external au d itory canal (D) CPP w ou ld equ al m easured MAP–CVP (E) blood p ressu re d eterm ined w ith a cu ff on the upp er arm w ould be less than the m easu red p ressu re 191. When electronically "zeroing" the transd u cer system , the stopcock im m ed iately above the transd ucer d iaphragm is op ened to air and (A) the transd u cer shou ld be p ositioned at the point w here pressure is m easured (B) the p osition relative to the p atient is irrelevant

241

(C) the transd u cer shou ld be p ositioned at the level the catheter enters the rad ial artery (D) the arm m u st be p ositioned at the level of the right atrium (E) the transd u cer shou ld be re-zeroed w henever the p osition is changed 192. During the proced ure the arterial catheter fails; cuff pressures are m onitored . Cerebral perfusion pressu re (A) cannot be d eterm ined u nless the head is low ered to the level of the heart (B) can be d eterm ined only from a cu ff p ressu re d eterm ined at the rad ial artery (C) cannot be d eterm ined u nless lim b w ith the cuff is elevated to the level head (D) equ als the systolic blood p ressu re d eterm ined at the brachial artery irrespective of location (E) is d eterm ined from the m ean blood p ressu re corrected for the d ifference in height w here m easu red from the p osition of the external aud itory canal 193. Du ring a craniotom y, after the d u ra m ater is opened , the intracranial p ressu re (A) increases (B) equ als zero (C) changes d irectly p rop ortional to blood flow (D) d ecreases only if the head is elevated (E) is u nchanged 194. Ju gu lar venou s oxygen satu ration m onitoring (A) assesses global oxygen extraction from brain (B) requ ires bilateral p lacem ent to fu lly assess the brain (C) is u nchanged d u ring hyp erventilation (D) is highly sensitive to all cerebral ischem ia (E) is d irectly affected by card iac ou tp u t

242

14: Ane s the s ia for Ne uros urge ry

DIRECTION S: Use the follow ing scenario to answ er Qu estions 195-197: A 15-year-old girl had a sp inal fu sion w ith H arrington rod instru m entation. Motor and som atosensory evoked p otentials w ere obtained throughou t. On em ergence, the patient w as unable to m ove her left low er extrem ity. 195. The cau ses of this m ay inclu d e all of the follow ing EXCEPT (A) (B) (C) (D) (E)

overcorrection of the scoliotic cu rve cord com p ression d u e to hem atom a d irect su rgical d am age to the cord hyp otherm ia traction of the anterior sp inal artery

196. The best cou rse of action on d iscovery of this loss of fu nction is (A) extu bate the trachea, begin blood transfu sion (B) observe for 24 h (C) establish baseline neu rologic fu nction and observe for changes (D) im m ed iate im aging of sp ine by CT or MRI (E) initiate som atosensory evoked p otentials 197. Motor evoked p otentials for the left low er extrem ity w ere red u ced d u ring su rgery bu t im p roved w ith ind u ced hyp ertension. The app ropriate initial m aneu ver on d iscovery of the d eficit w ould be (A) avoid p ressors that cou ld increase vascu lar constriction (B) increase m ean arterial blood p ressu re w ith p ressors (C) ind u ce hyp ercarbia (D) ad m inister m annitol (E) ad m inister hyp ertonic saline infu sion 198. All of the follow ing are com p lications associated w ith the sitting p osition for cervical su rgery EXCEPT (A) sciatic and cranial nerve trau m a (B) p neu m ocephalus

(C) qu ad rip legia (D) airw ay ed em a (E) blind ness 199. Th e ad van tage of th e sittin g p osition for craniotom y is (A) (B) (C) (D) (E)

red u ced intraop erative blood loss easier p ositioning p reservation of cranial anatom y easy access to the airw ay hyp ertension

200. H yp ertonic saline ad m inistered to a p atient w ith elevated ICP (A) m ay lead to hyp erosm olar com a (B) rem oves w ater from the norm al brain tissue w hile increasing filling pressure (C) m ay lead to cerebral ed em a if the blood –brain barrier is im p aired (D) is effective in d oses of 0.25 g/ kg (E) is contraind icated in p atients w ith renal failu re 201. Wh en ad m in istered to th e n eu rosu rgical patient, d exam ethasone (A) w ill red u ce cerebral ed em a su rrou nd ing a brain tu m or (B) is effective becau se of its osm olar prop erty (C) is m ore effective in control of ed em a cau sed by trau m atic inju ry (D) is contraind icated in p atients w ith Ad d ison d isease (E) p rod u ces hyp oglycem ia 202. Ad m inistration of nitrou s oxid e 66% in oxygen (A) red uces intracranial pressu re (B) d ep resses responsiveness of cerebral blood flow to carbon d ioxid e (C) p rod uces cerebrovascular d ilatation (D) slow s EEG (E) increases m etabolic sup pression p rod u ced by p rop ofol

Que s tions : 195–210

203. Air em bolism m ay be a fatal com p lication d ep end ing u p on (A) (B) (C) (D)

the site of entry the am ou nt of air and rate of entry volu m e statu s p resence of a p rop erly p ositioned pu lm onary artery catheter (E) p atient p osition 204. Concerning m agnetic resonance im aging (A) m otion artifacts are rare (B) objects containing ferrom agnetic m aterial are p rop elled w ithin the m agnetic field (C) rou tine m onitoring is im p ossible (D) large p rosthetic m etal im p lants are com pletely contraind icated (E) au tom atic im p lanted card iac d efibrillators should be sw itched off before im aging 205. Sensitive m ethod s to d etect venou s air em bolism inclu d e all of the follow ing EXCEPT (A) (B) (C) (D) (E)

p record ial Dop p ler m ass sp ectrom etry cap nograp h electrocard iograp h transesop hageal echocard iograp hy

206. Concerning ind u ced hyp otherm ia all of the follow ing are tru e EXCEPT (A) cerebral m etabolism is d ecreased (B) cerebral vascu lar resistance increases (C) cerebral vascu latu re rem ains resp onsive to carbon d ioxid e (D) cerebral blood flow rem ains cou p led to m etabolism (E) m ore glu cose is requ ired by the brain for m etabolism 207. If surgery is to be performed on a patient in the sitting position

243

(A) the legs should be w rapped w ith elastic band ages (B) the legs should be positioned below the level of the heart (C) the p atient can only be p ositioned aw ake (D) the neck shou ld be hyperextend ed (E) the p atient shou ld be p ositioned as qu ickly as possible to avoid loss of m onitors 208. Morp hine as a p rem ed ication is ind icated to facilitate ind u ction in (A) infants (B) p atients w ith increased intracranial p ressu re (C) com atose p atients (D) very anxiou s p atients (E) p u lm onary hyp ertension 209. Du ring a cerebral aneu rysm clip obliteration, sod iu m nitropru ssid e is infused . The expected resu lts inclu d e (A) short d u ration of action w hen infu sion is term inated (B) brad ycard ia (C) alkalosis (D) elevated sod iu m thiosu lfate levels (E) m ethem oglobinem ia 210. When a p record ial Dop p ler u ltrasonic transd u cer is used to d etect air em bolu s, it (A) (B) (C) (D)

can d etect 0.5 m L of air fu nctions at 15 H z requ ires central venou s access is p ositioned over the p oint of m axim u m intensity (E) is less sensitive than cap nograp hy

244

14: Ane s the s ia for Ne uros urge ry

DIRECTIONS: Use the follow ing scenario to answ er Questions 211-213: A patient w ith a convexity meningioma and several month history of severe head aches has a ventriculostomy for intracranial pressure monitoring in place preoperatively. Ind uction of general anesthesia w ith oxygen and nitrous oxid e ad ministered to the patient lead to an increased intracranial pressure. 211. The increase in intracranial p ressu re m ay be m itigated by ad m inistration of (A) (B) (C) (D) (E)

isoflu rane vecu roniu m fentanyl ketam ine p rop ofol

212. The effect of agents or d ru gs to m od ify a resp onse of increased intracranial p ressu re (A) rem ains ind ep end ent of ind ivid u al and brain state (B) is consistent across situ ations (C) d ep end s on the su m m ation of influ ences on cerebrovascu lar tone (D) can be d eterm ined w ith im aging (E) is ind ep end ent of ventilation state 213. Which one of the follow ing w ill red u ce the increase in intracranial p ressu re? (A) Op en the ventricu lostom y to d rain (B) Positive p ressu re hyp oventilation (C) Red u cing blood p ressu re w ith nitroglycerin (D) Facilitate venou s d rainage (E) Low ering the transd u cer from the level of the external au d itory canal to the m id axillary line 214. Du ring su rgery for excision of an intrad u ral tu m or in the low er thoracic level, integrity of the sp inal cord m ay be confirm ed by (A) p erform ing an intraop erative "w ake-u p" to test m otor fu nction in low er extrem ities (B) m onitoring brainstem -evoked potentials

(C) m onitoring som atosensory and m otor evoked p otentials (D) m onitoring the train-of-fou r on all fou r lim bs (E) intraop erative MRI of the sp ine 215. Therap y for neu rogenic p u lm onary ed em a inclu d es all of the follow ing EXCEPT (A) (B) (C) (D) (E)

red u ce intracranial hyp ertension α-ad renergic antagonists su p p ortive resp iratory care central nervou s system d ep ressants naloxone 4 m g IV

216. Cervical spine instability should be consid ered in any p atients w ith (A) (B) (C) (D) (E)

ankylosing sp ond ylitis Dow n synd rom e Marfan synd rom e sp inal stenosis neu rofibrom atosis

DIRECTION S: Use the follow ing scenario to answ er Questions 217-219: A previously healthy 42-year-old w om an is ad m itted to the neu rological intensive care unit after su d d enly losing consciou sness w hile sitting at her d esk at w ork. A CT scan show ed a su barachnoid hem orrhage, and a cerebral angiogram revealed that the hem orrhage w as d u e to ru ptu re of an aneu rysm of the right m id d le cerebral artery. H er caregivers are concerned that she m ay d evelop vasospasm . 217. Detection of cerebral vasospasm includ es all of the follow ing EXCEPT (A) (B) (C) (D) (E)

transcranial Dop p ler assessm ent of m ental statu s ju gu lar bu lb venou s oxygen satu ration angiograp hy assessm ent of m otor fu nction

218. Cerebral vasosp asm is m ost likely to occu r after su barachnoid hem orrhage on d ays (A) 0-6 (B) 4-14 (C) 7-21

Que s tions : 211–224

(D) 12-20 (E) u p to one m onth 219. End ovascu lar treatm ent for cerebral vasosp asm after su barach n oid h em orrh age inclu d es all of the follow ing EXCEPT (A) (B) (C) (D) (E)

intraarterial injection of m annitol angiop lasty intravenou s infu sion of nicard ip ine ind u ced hyp ertension intracerebral stenting

220. A p atient retu rns to su rgery to treat a CSF leak after transsphenoid al resection of the pituitary. Ind uction of general anesthesia should includ e (A) inhalational ind u ction w ith nitrou s oxid e and sevoflu rane (B) head d ow n p ositioning to p revent CSF d rainage (C) rap id sequ ence end otracheal intu bation (D) p lacem ent of a ventricu lostom y to p revent CSF d rainage (E) p lacem ent of an nasogastric tu be to em p ty the stom ach of blood and CSF 221. Du rin g carotid en d arterectom y w ith EEG m onitoring, both hem isp heres d em onstrate profou nd slow ing of frequ ency and bu rst su p pression. The anesthetic techniqu e consists of continu ou s infusions of propofol and rem ifentanil. The m ost likely d iagnosis is (A) (B) (C) (D)

p rop ofol overd ose hyp otherm ia hyp otension inad equ ate p erfu sion to both hem ispheres (E) elevated ICP 222. Signs of venou s air em bolism inclu d e all of the follow ing EXCEPT (A) (B) (C) (D) (E)

arrhythm ia hyp ertension heart m urm u r bubbles at the operative site d ecreased end -expired carbon d ioxid e

245

DIRECTION S (Qu estions 223-225): Each group of item s below consists of lettered head ings follow ed by a list of nu m bered p hrases or statem ents. For each nu m bered p hrase or statem ent, select the ON E lettered head ing or com p onent that is m ost closely associated w ith it. Each lettered head ing or com pon ent m ay be selected once, m ore than on ce, or n ot at all. (A) (B) (C) (D) (E) (F) (G) (I)

Am yotrop hic lateral sclerosis Mu ltip le sclerosis Cau d a equ ina synd rom e Gu illain-Barre synd rom e Myasthenia gravis Mu scu lar d ystrop hy Fam ilial p eriod ic p aralysis Myasthenic synd rom e (Eaton-Lam bert synd rom e)

For each p atient w ith m u scle w eakness, select the m ost likely d isease p rocess. 223. A 64-year-old w om an has u nd ergone thoracotom y for lobectom y to resect sm all cell carcinom a of the lung. On em ergence she appears w eak. She cannot m aintain a su stained head lift and cannot generate su fficient tid al volu m es to be extu bated . Mu scle relaxation had been m aintained w ith vecu roniu m ; there w ere three sm all tw itches w ith fad e elicited w ith a blockad e m onitor set to train-of-fou r before ad m inistration of neostigm ine and glycopyrrolate. She rep orted a history of easy fatigu e w ith exertion p articu larly w h en clim bin g stairs. There w as no history of d iplopia or d ysp hagia. There w as no im p rovem ent w ith ad d itional neostigm ine. 224. A 47-year-old m an has a six-m onth history of p rogressive d ysarthria and d ysphagia, and is p resenting for a gastrostom y tube for w eight loss from inad equ ate nu trition. H e has d ifficulty managing oral secretions and has to sleep w ith the height of the bed at 45 d egrees because of obstru ctive sleep ap nea. H e d em onstrates no symp tom s of w eakness either w ith w alking or m ovem ent of hand s or arm s.

246

14: Ane s the s ia for Ne uros urge ry

225. A 27-year-old w om an p resents w ith su d d en onset of loss of bow el and blad d er fu nction and w eakness in both legs. She cannot stand or w alk bu t can sit. She has an elevated w hite blood cell cou nt w ith an abnorm al sm ear; w hite blood cell d ifferential is p red om inantly lym p hocytes w ith the p resence of m u ltip le im m atu re lym p hocytes and blast cell. She is afebrile and free of p ain. DIRECTION S (Qu estions 226-227): Each grou p of item s below consists of lettered head ings follow ed by a list of nu m bered p hrases or statem ents. For each nu m bered phrase or statem ent, select the ON E lettered head ing or com p onent that is m ost closely associated w ith it. Each lettered head ing or com p on en t m ay be selected on ce, m ore than once, or not at all. (A) (B) (C) (D) (E) (F) (G)

Atelectasis Pu lm onary em bolism Pneu m othorax Patent foram en ovale Asp iration N eu rogenic p u lm onary ed em a Sp inal shock

For each p atient w ith intraop erative cou ghing, select the appropriate d iagnosis. 226. A 65-year-old m an p resents for sp inal d ecom pression and bilateral foram inotom ies at m u ltiple levels for spinal stenosis extend ing from T8 to L4. H e has been incap acitated by pain

and leg w eakness, and has been largely bed rid d en for several w eeks before su rgery. H e has u nd ergone uneventfu l aw ake fiberoptic intubation after top ical anesthesia of the airw ay w ith lid ocaine w hile sitting u p right. H e has p ositioned him self p rone on the op erating room table and m oved his legs on com m and before in d u ction of gen eral an esth esia. Anesthesia is m aintained w ith rem ifentanil and nitrou s oxid e; no m u scle relaxants have been ad m inistered . During lam inectomy in the thoracic region he begins to cough. H e becomes hyp otensive and the SaO 2 falls from 98% to 89%. Positive p ressu re ventilation w ith 100% oxygen and high inflation p ressu re d o not im p rove oxygen satu ration. 227. A 16-year-old otherw ise healthy male is und ergoing H arrington rod placement to correct scoliosis. An esth esia con sists of con tin u ou s infu sions of p rop ofol and rem ifentanil; no m uscle relaxants have been ad ministered since ind u ction. Stand ard monitoring w ith noninvasive blood pressure, ECG, SaO 2, and esop hageal tem p eratu re p robe are ap p lied to the p atient. H e is p rone and both som atosensory an d m otor evoked p oten tials h ave been u nchanged since ind u ction. Im med iately after rod p lacem ent and d istraction of the sp ine, a m otor evoked potential is p erform ed and the p atient cou ghs several tim es. Du ring closure the su rgeon notices air bubbles in the arterial circu lation in the epid u ral space.

Answe rs a nd Expla na tions

175. (C) Cerebral m etabolic rate d ecreases w ith a fall in tem p eratu re. This d ecrease is qu antitated as the Q 10, or changes in m etabolic rate for a 10°C change in tem p eratu re. While the first Q 10 is 2.2, the second Q 10 is abou t 5; the d ifference betw een the first and second Q 10 valu es is thou ght to be d u e to cessation of brain electrical activity. Tem p eratu re has no effect either on au toregu lation or on oxygen extraction from blood . MAC d ecreases w ith a fall in tem p eratu re. (5:916, 1469-71; 6:166) 176. (B) The d efinition of au toregulation is correct. Vasom otor p aralysis involves vasod ilatation. Lu xu ry p erfu sion is p erfu sion in excess of requ irem en ts. In tracerebral steal in volves blood flow aw ay from ischem ic areas. Inverse steal involves d iversion of flow from norm al areas to ischem ic areas. (5:871-4; 6:2255) 177. (D ) The best agent for d ecreasing cerebral oxygen requirem ent is a barbitu rate. A m u scle relaxant m ay be u sefu l by p reventing the p atient from cou ghing and m oving. Phenytoin red u ces cerebral oxygen consu m p tion cau sed by seizu res; in the absence of seizu res it has little effect on cerebral m etabolic rate. (1: 535-6; 5:876,886; 6: 2257) 178. (A) Intracranial p ressu re m ay increase after su ccinylch oline. The in crease is attenu ated w ith p rior ad m inistration of a sm all d ose of a nond ep olarizing m u scle relaxant to p revent fascicu lation s. Both ven tilation to red u ce PaCO 2 and ad ministration of thiopental red uce the rise in ICP. Increased ICP d oes not pred isp ose to hyperkalem ia; seru m p otassiu m w ill

not rise above the u su al 0.5 to 1.0 m Eq/ L. (5:878) 179. (E) System ic hyp ertension and ECG changes often accom p any cerebral aneu rysm ru p tu re and resulting intracranial hypertension. This is a d read ed com p lication of end ovascu lar proced ures. Dissection or occlu sion of the vertebral artery can occu r bu t is m ore likely d u ring catheter placem ent. Arterial air em bolism that occlud es cerebral blood flow can occur but w ill not cau se hyp ertension. Vessels are insensate bu t p ain on m anip u lation is m ore com m on w ith d u ral vessels. (5:889-93) 180. (D ) Treatm en t of in trap roced u ral ru p tu re focu ses on im m ed iate d iagnosis w ith angiograp hy and obliteration of the aneu rysm to stop the hem orrhage. Placem ent of an external ventricu lar d rainage catheter facilitates red u ction in ICP by d rainage of CSF. Maneu vers to red u ce ICP are ap p rop riate inclu d ing hyp erven tilation and p rop ofol. H yp erten sion is reflexive and su p p orts cerebral circu lation d u rin g h igh ICP; in creasin g sevoflu ran e d ecreases perfusion pressure both by elevating ICP throu gh vasod ilation and red u cing blood p ressu re. (5:889-93) 181. (E) Althou gh PEEP m ay be necessary in the ventilation of the neu rosurgical p atient, its use shou ld not be rou tine. The level of PEEP shou ld be titrated to the need and effect. It is help fu l to m onitor intracranial p ressu re to ascertain possible d eleteriou s effects. The head shou ld be elevated . (5:873)

247

248

14: Ane s the s ia for Ne uros urge ry

182. (B) Urinary volu m e w ill increase w ith m annitol ad m inistration. At first the p atient m ay becom e hyp ervolem ic both from ad m inistration of the volu me of mannitol and the increase in intravascu lar volu m e from shifting of free w ater from the intracellu lar and extravascu lar spaces. H ypovolemia may d evelop after d iu resis w ith a fall in both venou s and arterial pressu re. (1: 681-2; 5:880) 183. (D ) While not an absolu te contraind ication to use, nitrous oxid e has been reported to increase cerebral blood flow ; this m ay be a consid eration in patients w ith increased ICP from head inju ry, intracranial hem orrhage, and tu m or. Since nitrous oxid e d iffuses into air-containing spaces, the presence of pneum ocephalus is a contraind ication to use because of the potential to increase ICP. (5:878, 882) 184. (C) The cerebral vasoconstrictive effect is d im inished by red u cing PaCO 2 below 25 m m H g. In ad d ition, at a PaCO 2 below 20 m m H g, the effect is self-d efeating w ith potential d evelop m ent of cerebral ischem ia. (5:87, 1468-9; 6:2255-7)

not ru le ou t a m otor d eficit, p rim arily a ventral cord function. N egative inspiratory force generated d u ring sp ontaneou s ventilation can cau se the com m on p roblem of venou s air em bolism . (5:489) 187. (A) In ord er to be effective in treating d epression, electroconvu lsive therapy m ust ind u ce a seizu re. Becau se of the rap id and u np red ictable changes in blood pressu re accom p anying the seizure, patients w ith aneu rysm s m ay not be cand id ates for this therap eu tic m od ality. In ad d ition, the increases in cerebral m etabolic activity and concomitant rise in blood flow can increase ICP and , therefore, m ay cause cerebral herniation in patients w ith d ecreased intracranial com pliance from m ass-occupying lesions. Therefore, these cond itions rep resent relative contraind ications to ECT. (1:266; 5:1267-8) 188. (A) If intracranial p ressu re increases on ind uction, global or areas of focal ischem ia m ay occur from herniation and / or d ecreased perfusion pressure. Brain retraction m ay cau se ischemia d u ring su rgery. H yd rocephalu s is a cau se of increased ICP, not a consequ ence. (5:871-3; 6:2255-7)

185. (E) There are m any systemic sequelae of closed head injury. Coagu lopathy and d issem inated intravascu lar coagu lation m ay resu lt from cerebral trau m a, p ossibly from the release of brain thrombop lastin into the systemic circu lation . Posterior p itu itary d ysfu n ction is com m on and m anifests as d istu rbances in antid iuretic hormone secretion. Hyperglycemia w ith nonketotic hyp erosm olar com a can also occur. These patients often su ffer from respiratory com p rom ise and are hyp oxem ic and hypercarbic. (5:1462-78; 6:2255-7)

189. (C) The cerebral p erfu sion p ressu re is m ean arterial blood p ressu re less the intracranial p ressu re (MAP – ICP). In this case the ICP is 0 since the craniu m is op en. Since the MAP is m easu red at the level of the right atriu m , the p ressu re w ould be less at the circle of Willis (located at the level of the external au d itory canal) becau se the head is elevated . Thu s the MAP at the base of the brain w ou ld be 70 m m H g; 90 m m H g − (26 cm ÷ 1.3 m m H g/ cm ) in ord er to correct for the height. (5:406-9, 882-3)

186. (E) Aw akening the p atient intraop eratively is a m eans to assess m otor fu nction in the low er and occasionally u p p er extrem ities after d istraction of the vertebral colu m n in sp ine su rgery. Com p rom ise of blood su p p ly m ay occu r from su rgical m anip u lation or straightening of the cord . The u se of som atosensory-evoked potentials assesses only sensory fu nction, p rim arily a d orsal spinal cord function, and d oes

190. (A) With the transd u cers p ositioned at the level of the circle of Willis, the m easu red p ressu re is the arterial p ressu re at the base of the brain. Position of the arm containing the arterial catheter w ith resp ect to the transd u cer w ill not alter the valu e of the m easu red p ressu re. Arterial p ressu re of any stru ctu re below the level of the transd u cer w ill be higher than the m easu red valu e. (5:406-9, 882-3)

Answe rs : 182–200

191. (B) Electronic zeroing m erely sets a p ressu re of 0 m m H g to the electrical ou tp ut of the transd u cer w hen the zeroing m aneu ver is p erform ed . Op ening the stop cock to am bient pressu re (air p ressure) exerts a pressu re on the transd u cer d iaphragm equ al to the height of the colum n of flu id betw een the opened stopcock and the d iap hragm . By convention, the stopcock opened is the one d irectly above the d iaphragm and the colu m n height is 1–2 cm . The p osition of the transd u cer relative to the patient w ill not affect the electronic zero and is irrelevant to the p roced u re (op ening the stop cock d isconnects the patient from the system ). Once the electronic zero is established , there is no need to re-zero the instru ment unless to correct for electronic d rift that can occu r w ith tim e. (5:406-9, 882-3) 192. (E) Arterial blood p ressu re can be d eterm ined by cu ff from any location. Cerebral p erfu sion pressu re is calcu lated from the m ean pressu re corrected for the d ifference in height from w here m easu red to the external au d itory canal by the equ ation given in the exp lanation to Qu estion 189 (5:406) 193. (B) Once the sku ll is op en and the d u ra m ater incised , the brain is no longer confined w ithin the craniu m , and intracranial pressu re is zero. (5:871-3) 194. (A) Ju gu lar bu lb ven ou s oxim etry d etects changes in brain oxygen extraction from blood . The catheter is positioned retrograd e in a single jugular bulb and reflects venous d rainage from p red om inantly a single hem isp here; if p ositioned extracranially, contam ination w ith noncerebral venou s blood d ilu tes effectiveness as a m onitor. Changes in oxygen d elivery, either from content or blood flow, are reflected in the p ercent satu ration of th e d rain ing venou s blood . Focal ischem ia m ay not be d etected . Card iac ou tp u t, u nless in extrem e shock, has no effect. (5:875) 195. (D ) H yp otherm ia is a system ic effect an d w ou ld not p resent as a focal d eficit. The d eficit d escribed m ay be the resu lt of any of the other

249

situ ations. Perip heral nerve d am age is an u nu sual com p lication. (5:484-9, 1200-1) 196. (D ) If the p atient aw akens w ith a d eficit that cannot be exp lained either im m ed iate im aging or exp loration of the su rgical site is ind icated for p otential cord com p ression from a h em atom a. Th e p atient shou ld be left intu bated p end ing a d ecision to facilitate sp eed of reop eration . Som atosen sory evoked p otentials d o n ot p rovid e m otor in form ation . (5:484-9, 1200-1) 197. (B) Traction on the anterior spinal artery prod u ced by straightening of the sp ine can cau se ischem ia in selected regions of the sp inal cord . Ind uced hypertension may im prove perfu sion. System ically ad m inistered p ressors d o not cau se vasoconstriction in the central nervou s system. Ed ema treated w ith mannitol or hypertonic saline is an u nlikely cause of inad equ ate p erfu sion in this setting. (5:484-9, 1200-1) 198. (E) Blind ness is a rare com p lication of the p rone position; the etiology is obscure but may involve increased intraocu lar p ressu re. With the sitting p osition, the sciatic nerve is at risk for com p ression from inad equ ate p ad d ing or traction inju ry from im p rop er p ositioning. Traction of cranial nerves, especially the abd u cens (VI), can result from cau d al d isplacem ent of the brain. Air trapped over the superior surface of the brain p rod u ces p neu m ocep halu s. Qu ad rip legia has been rep orted d u e to cord com pression from extrem e flexion of the neck. Sim ilarly, airw ay ed em a can occu r from obstruction of venous d rainage. (5:896) 199. (A) Intraoperative blood loss is red u ced in the sitting position d u e to increased venou s d rainage. The positioning is m ore d ifficu lt than w ith a su p ine position and access to the p atient and airw ay is lim ited . H yp otension is frequ ently a com plication. Anatom y is not preserved and cranial nerve traction is a problem . (5:365-6) 200. (B) H yp ertonic saline effectively red u ces ICP by rem ovin g w ater from n orm al tissu es. It m ay be ad m inistered in the setting of renal failu re. Unlike m annitol there is no p otential

250

14: Ane s the s ia for Ne uros urge ry

for hyp erosm olar com a if given in large d oses and d oes not cau se cerebral ed em a by d iffu sing intracellu larly if the blood –brain barrier is im p aired . Mannitol is effective in d oses as low as 0.25 g/ kg. (5:174, 511, 537; 6:2256-8, 2271) 201. (A) Dexam ethasone is effective in the treatm ent of cerebral ed em a in patients w ith brain tum ors. It has been show n to be less effective in the p atient w ith brain inju ry from acu te closed head trau m a. The m echanism is probably from a d ecrease in inflam m atory p rocesses cau sed by th e tu m or. There is no con tr a in d ica tion in A d d ison d isea se. H yp erglycem ia is a com m on s sid e effect. (1:1232; 5:881; 1126-7; 6:2256-8, 2271) 202. (C) Ind uction of anesthesia w ith nitrous oxid e as the sole agent has been show n to increase cerebral blood flow and intracranial pressure. Although nitrous oxid e prod uces cerebrovascular dilatation, the cerebral blood flow response to carbon dioxid e is preserved. These effects on cerebral hemodynamics are altered by the ad d ition of other anesthetic agents; barbiturates and benzod iazepines blunt the increase in intracranial pressure. (1:546-7; 5:873-4) 203. (B) Both the amount and rate of entry of air are important factors in determining consequences of venou s air em bolism . The site of entry is u nim p ortant. Althou gh a p u lm onary artery catheter m ay be u sefu l for monitoring, the port is too small to be helpful in the w ithd raw al of air. Volu me statu s has no effect on severity of response. While som e p ositions increase likelihood of em bolism , p atient p osition d u ring entrainment does not impact outcome. (5:881-2) 204. (B) All ferrom agnetic m aterial can create artifacts in the im age and can be d raw n into the m agnetic field . While som e m etal im p lants m ay shift position in the field and heat d u ring scanning, som e im p lants are safe for im aging. The patient must remain motionless for several m inutes d u ring scanning; m ovem ent artifacts from breath in g an d arterial p u lsation are com m on. Pacem akers and AICD's rem ain contraind icated for im aging. (5:896-7)

205. (D ) Arrhythm ias and card iovascular collap se are late signs of venou s air em bolism . The precord ial Dop pler in conju nction w ith a capnograp h or p u lm on ary artery cath eter w ill u su ally d etect air before p hysiologic consequ ences occu r. (5:881-2) 206. (E) Du ring hyp otherm ia, the brain requ ires less glu cose and oxygen becau se of red u ced m etabolic d em an d . Cerebral blood flow rem ains cou pled to m etabolism and d ecreases; cerebr al v ascu lar resistan ce in creases. Resp on siven ess to ch an ges in PaCO 2 is u nchanged . (5:1469-71; 6:2258-9) 207. (B) Positioning the p atient in the sitting p osition requ ires planning and is perform ed after ind u ction of anesthesia. In ord er to m inim ize venou s p ooling in the low er extrem ities, a potential cause of hypotension, the legs should be w rap p ed and p ositioned level w ith the heart. The head shou ld never be h yp erextend ed . Positioning shou ld be d one slow ly to avoid hyp otension. (5:365-6, 881-3) 208. (D ) Morp hine as a sed ating p rem ed ication is u sefu l in anxiou s p atients bu t can d ep ress ventilation. It is contraind icated in the very you n g, p atien ts w ith in creased in tracranial p ressu re, and com atose p atients. (1: 501-2; 5:703-5, 709-10, 713) 209. (A) Sod ium nitrop ru ssid e has a short d u ration of action. It m ay cause an acid em ia and is usu ally associated w ith tachycard ia. Cyanid e toxicity m ay also occu r; in traven ou s sod iu m thiosu lfate is the antid ote. (1:782-3; 6:1913; 5:746-7) 210. (A) Precord ial Dop p ler is one of the m ost sensitive m onitors of venou s air em bolism clinically available and can d etect air in qu antities as sm all as 0.5 m L. Th e u ltrason ic p robe, w hich fu nctions at 2.0 MH z, is p laced over the right sid e of the heart. Central venou s access can be u sed to w ithd raw entrained air from the right atrium; it is not required for monitoring. (5:874-5)

Answe rs : 201–220

211. (E) Propofol m ay block increases in intracran ial p ressu re p rod u ced by nitrou s oxid e. Isoflu rane and ketam ine are p otent cerebral vasod ilators and increase intracranial pressure. Vecuronium w ill have no effect on ICP. Opioids increase ICP by d ep ressing ventilation. (1: 536-7; 5:875-8) 212. (C) Dru g effects on intracranial p ressu re are not constant but vary am ong ind ivid u als, w ith ventilation, and the state of the brain at the tim e applied . The total effect on cerebrovascular tone w ill d eterm ine the effect on cerebral blood volu m e an d in tracran ial p ressu re. Im aging d oes not p red ict ICP. (5:871-3, 875-8; 6:2255-7) 213. (A) ICP can be low ered by opening the ventricu lostom y to rem ove som e CSF. While h yp erven tilation m ay red u ce ICP, sim p le m echanical ventilation w ithou t a change in Pa CO 2 w ill have no effect. N itroglycerin is a vasod ilator and in creases ICP. Moving the transd u cer w ill only change the valu e bu t the ICP rem ains the sam e. ICP is m easu red w ith the transd ucer at the level of the external aud itory can al. Obstru ctin g ven ou s d rain age increases ICP bu t sim p le facilitating d rainage w ill not overcom e the increase p rod u ced by nitrou s oxid e. (5:871-3; 6:2255-7) 214. (C) An intraop erative w ake-u p test d em onstrates upper and low er lim b m ovem ent on command and assures integrity of the ventral spinal cord containing the corticospinal tracts. Monitoring of somatosensory-evoked potentials tests the integrity of the d orsal portion of the spinal cord , w hereas motor evoked potential m onitoring d etects integrity of corticosp inal tracts. Brain stem-evoked potentials are useful on intracranial procedures involving the posterior fossa bu t have no valu es for monitoring spinal cord function. The train-of-four monitor only evaluates the function of a peripheral nerve and the neuromuscular junction. Intraoperative MRI is time consuming and im ages m ay not detect ischemic damage. (5:490) 215. (E) N eu rogenic p u lm onary ed em a is thou ght to involve massive sympathetic d ischarge from

251

inju red brain in resp onse to intracranial hypertension. It is treated by both red uction of intracran ial p ressu re an d su p p ortive care to m axim ize oxygen ation. N aloxone in high d oses can p rod u ce p u lm onary ed em a. (5: 886; 6: 3359) 216. (B) Patients w ith Dow n synd rom e and rheu m atoid arthritis are at high risk for atlantoaxial instability. Patients w ith ankylosing sp ond ylitis m ay have their cervical vertebrae fused in flexion. Su ch p atients m ay d evelop cervical spine instability after attem p ts to extend the neck d u ring intu bation. N eu rofibrom atosis m ay present as schw annom a w ithin the spinal canal but d oes not prod u ce cervical instability. (5: 137-47, 895, 1201; 6: 135) 217. (C) Cerebral vasosp asm is segm ental narrow ing of the cerebral vessels m ost com m only the large cond u it vessels at the base of the brain. Extrem e sp asm m ay im p air blood flow and cause ischem ia. The presence of sp asm can be m onitored by transcranial Dop p ler to d etect increased blood flow velocity and neurological exam for m ental statu s and m otor fu nction. Cerebral angiograp hy d etects loss of vessel caliber. Jugular bulb venous oxygen saturation is too insensitive to d etect sm all regions of ischem ia. (5:889-90) 218. (B) Cerebral vasospasm occurs most commonly on days 4 through 14 after subarachnoid hemorrhage w ith a p eak on d ay 7. (5:885-7; 6:2204, 2263) 219. (E) End ovascu lar treatm ent of cerebral vasosp asm after su barachnoid hem orrhage typ ically involves angiop lasty of the stenotic segm ents or intraarterial injection of vasod ilators inclu d ing papaverine or calciu m channel an tagon ists v era p am il or n icard ip in e. Intraarterial m annitol is u sed to treat larger vessel spasm d uring angiography. Intracerebral vessel stenting is unnecessary to restore flow. (1:757-60; 5:889-90) 220. (C) Active d rainage of CSF ind icates an open cond u it from the nose to the sella tu rcica. Positive pressu re ventilation carries the risk of

252

14: Ane s the s ia for Ne uros urge ry

passage of gas into the craniu m and should be avoid ed . N itrou s oxid e is avoid ed becau se of the risk of expand ing a pneum ocephalus often accom panying a CSF leak. Drainage of the CSF w ith a ventricu lostom y d oes not correct these issu es. A nasogastric tu be w ill obscu re the su rgical field and carries the p otential for p assage into the cranial cavity. (5:884-5) 221. (A) Bu rst su p p ression on EEG is p rod u ced m ost com m only by excessive d oses of hyp notic m ed ications inclu d ing p ropofol, barbitu rates, isoflu rane, or etom id ate. H yp otherm ia or hyp otension resu lting in inad equ ate brain perfu sion p rod u ce slow ing of the EEG or electrical silence. Sim ilarly, elevated ICP cou ld com prom ise cerebral perfu sion and slow the EEG. (1:333-46; 5:479, 611, 694, 696-8, 710) 222. (B) Air em bolism is associated w ith arrhythm ia, heart m u rm u r (w hen there is a large am ou nt of air), bubbles at the op erative site, and d ecreased end -tid al CO 2. H yp otension occu rs after a large volu m e of air is entrained . (5:881-2) 223. (I) Myasthenic synd rom e is associated w ith neop lasm s inclu d ing sm all cell carcinom a of the lung. Ad m inistration of neostigmine rarely resu lts in im p rovem ent of sym p tom s. Th e proxim al m u scle grou ps are more com m only affected , w hile bu lbar sym p toms from cranial nerve involvem en t are rare. (1:234; 5:958; 6:3482-3)

224. (A) Am yotrop hic lateral sclerosis can p resent w ith p red om inantly bu lbar sym p tom s of d ysarthria and d ysp hagia w hile m u scle strength in the extrem ities m ay be p reserved . Collap se of the p haryngeal m u scu lar d u ring sleep is com m on, as w ell as d ifficulty hand ling secretions. (5:139-40; 6:3345-7) 225. (C) Cau d a equ ina synd rom e is cau sed by com pression of the spinal nerves below the term ination of the cord . It can be prod u ced by m any d ifferen t etiologies in clu d in g lym p h om a. Bow el and blad d er sym p tom s are com m on. (5:1156; 6:2269) 226. (B) Coughing, even during general anesthesia, may be associated with pulmonary embolism. Inactivity associated with lower extremity weakness and back pain pred isposes to d eep vein thrombosis. (5:1257, 1288-9; 6: 104-5, 2170-7) 227. (D ) Du ring su rgery in the p rone p osition the w ou nd is above the heart and negative intrathoracic p ressu re p red isp oses to venou s air em bolism . Exp osu re of the trabecu lar bone d u ring instru m entation of the spine generates a rou te for the entrainm ent of air. Cou ghing or strong insp iration d u ring light anesthesia m ay p rod u ce negative intrathoracic p ressu re. The p resence of air w ithin the arterial system ind icated a p arad oxical em bolism via a p atent foram en ovale. (5:881-2; 6: 3275)

CHAPTER 15

Obs te tric Ane s the s ia Que s tions DIRECTION S (Qu estions 228-309): Each of the num bered item s or incom plete statem ents in this section is follow ed by answ ers or by com p letions of the statem ent. Select the ON E lettered answ er or com pletion that is BEST in each case. 228. A p artu rient p resents to labor and d elivery com p laining of d ecreased fetal m ovem ent at 34 w eeks gestational age. The m ost likely next step to evaluate the w ell-being of the fetus is to p erform a(n) (A) (B) (C) (D) (E)

oxytocin contraction test (OCT) biop hysical p rofile (BPP) non-stress test (N ST) u m bilical artery Dop p ler flow stu d y fetal scalp p H analysis

229. You receive a p age for an em ergent cesarean section in a 27-year-old p artu rient in labor w ith a fu nctioning ep id u ral for fetal d istress. You ru sh to the bed sid e, and after asp irating the epid u ral catheter, inject w hich one of the follow ing agents or com bination of agents w hile accom p anying the p atient to the op erating room ? (A) 2% lid ocaine w ith 1:200,000 ep inep hrine (B) 3% 2-chloroprocaine (C) 2% lid ocaine w ith 1:200,000 epinep hrine and bicarbonate

(D) 0.5% bu pivacaine w ith bicarbonate (E) 3% 2-chlorop rocaine w ith sod iu m bicarbonate 230. Absolu te contraind ications to m ajor cond u ction anesthesia in p artu rients inclu d e all of the follow ing cond itions EXCEPT (A) p reexisting neu rologic d isease of the spinal cord (B) p atient refu sal (C) infection at the site of need le insertion (D) hyp ovolem ic shock (E) severe coagu lop athy 231. As com p ared w ith region al anesthesia for cesarean section, general anesthesia results in a/ an/ the (A) app roxim ately 17-fold increased risk of m aternal d eath (B) higher incid ence of m aternal hyp otension (C) ap p roxim ately tw ofold increased risk of m aternal d eath (D) m ajority of recent m aternal d eaths related to failed intu bation rather than in the postop erative p eriod (E) low er incid ence of u terine atony

253

254

15: Obs te tric Ane s the s ia

232. A 30-year-old G1P1 p atient u nd erw ent an em ergen t cesarean section u n d er ep id u ral anesthesia. She received 4 m g ep id u ral m orp hine for pain control p rior to rem oval of the catheter. Fou r hou rs postpartum she begins to com p lain of incisional p ain. The m ost likely exp lanation for this is (A) the d ose of m orp hine is insu fficient for epid u ral ad m inistration (B) m orp hine has a high lip id solu bility resu lting in an exp ected analgesic p eriod of 1-4 h (C) 2-chlorop rocaine w as u sed for the u rgent cesarean section (D) 2% lid ocaine w ith bicarbonate w as u sed for the u rgent cesarean section (E) she received nalbu p hine for relief of pru ritu s 233. You receive a p hone call from an obstetrician w ho has just seen a patient with von Willebrand d isease (vWD) for her first prenatal visit. H e inqu ires as to the anesthetic-related im p lications of the patient's d isease d uring pregnancy and the u se of neu raxial anesthesia in this p atient. All of the follow ing responses are tru e of vWD EXCEPT (A) vWD is the m ost com m on inherited coagulopathy and affects 1-2% of the general p op u lation (B) evid ence-based recom m end ations for neuraxial anesthesia in the setting of vWD can be m ad e (C) sp ecific treatm ent strategies m ay be d eterm ined in consu ltation w ith a hem atologist (D) the treatm ent of vWD d ep end s on its severity and su btype (E) vWD is not necessarily a contraind ication to neuraxial anesthesia DIRECTION S: Use the follow ing scenario to answ er Qu estions 234–236: A 26-year-old G1P0 p atient p resen ts to th e labor an d d elivery d ep artm en t at 35 w eeks gestational age com p laining of a head ache and scotom ata. H er blood pressure is 150/ 100, pulse is 84, and her oxygen satu ration is 100% breathing

room air. Physical exam ination reveals m arked facial ed em a and a Mallam p ati 4 airw ay. H er cervix is 2 cm d ilated . Laboratory valu es inclu d e a p latelet cou nt of 90,000/ m m 3, a 24-hour u rine sp ecim en containing m ore than 5 g of p rotein, and norm al liver fu nction tests. Physical exam ination reveals ep igastric p ain. 234. All of the follow ing are consistent w ith a d iagnosis of severe p reeclam p sia EXCEPT (A) (B) (C) (D) (E)

p latelet cou nt of 90,000/ m m 3 head ache scotom ata d iastolic blood pressure of 100 m m H g ep igastric p ain

235. The obstetrician begins an infu sion of m agnesium sulfate and d ecid es to ind uce labor in the p atient given h er d iagn osis of severe p reeclamp sia. She then calls you to requ est an epid u ral. You inform the obstetrician that (A) severe p reeclam p sia is a contraind ication to epid u ral placem ent (B) the p latelet cou nt is an absolu te contraind ication to the placem ent of an ep id u ral (C) the ep id u ral shou ld be p laced later in active labor after a rep eat p latelet cou nt is obtained (D) you w ill obtain inform ed consent and place an ep id u ral (E) the p atient shou ld receive a sp inal to red u ce the risk of hem atom a 236. The m ost com m on cau se of m ortality in a p atient like this one is (A) (B) (C) (D)

card iac arrest renal failu re hep atic ru p tu re resp iratory arrest after inability to intu bate (E) cerebrovascu lar accid ent 237. A 26-year-old G2P0 patient requests an epid ural. The ep id u ral p lacem ent is u neventfu l. Tw enty minutes later you are called to the labor room becau se the p atient is com p laining of

Que s tions : 232–241

shortness of breath and is still in pain. A quick sensory exam reveals a much higher level on the right side of the patient w ith a patchy sensory block. There is no fetal d istress and the patient is oxygenating w ell. Your next step in the management of this patient should be to (A) p u ll the catheter back 1 cm and ad m inister another bolu s of local anesthetic solu tion (B) stop the ep id u ral infu sion, observe the patient, and ad m inister another bolus of local anesthetic solu tion w hen the sensory level reced es (C) stop the ep id u ral infu sion, observe the patient, rep lace the ep id u ral catheter, and ad m inister another bolu s of local anesthetic solu tion once the level has reced ed . (D) intu bate the p atient and p rovid e su p portive m easu res (E) ad m inister a bolu s of fentanyl throu gh the epid ural catheter to enhance the block 238. You are called by an obstetrician to see an H IVpositive p atient on her first p ostp artu m d ay. She und erw ent an uneventful vaginal d elivery u nd er ep id u ral anesthesia. She is now com p laining of a p ositional head ache and neck stiffness. You have a d iscu ssion about an epid u ral blood patch (EBP). You tell her (A) given her H IV statu s an EBP is contraind icated and you recom m end conservative treatm ent (B) an EBP has a su ccess rate of ap p roxim ately 70% after the first attem pt (C) if she w ere to have a w et tap in the fu tu re, evid ence-based m ed icine su pports the placem ent of a prophylactic EBP (D) severe com p lications of EBP are rare and m ay inclu d e transient brad ycard ia, facial p alsy, and arachnoid itis. (E) the absence of a d ocu m ented w et tap m akes the d iagnosis of postd ural pu ncture head ache (PDPH ) u nlikely in this patient

255

239. Regard ing placental physiology, w hich one of the follow ing d ru gs crosses the p lacenta in ap p reciable am ou nts? (A) (B) (C) (D) (E)

Atrop ine Glycop yrrolate Su ccinylcholine Vecu roniu m H ep arin

DIRECTION S: Use the follow ing scenario to answ er Qu estions 240-241: An 18-year-old G2P1 patient p resents for rep eat cesarean section. She is 59 inches tall and u nd erw ent a previou sly u neventfu l prim ary cesarean section. Shortly after the ad m inistration of sp inal anesthesia, the patient begins to com plain of shortness of breath. H er blood p ressu re is 80/ 40 and her heart rate is 48 bp m . The p atient tells you she is nau seated , short of breath, and her hand s are tingling. 240. Initial m an agem en t of th is p atient shou ld inclu d e all of the follow ing EXCEPT (A) ad m inistration of ep hed rine (B) asking the p atient to squ eeze you r hand s w ith hers (C) IV flu id bolu s ad m inistration (D) su p p lem ental ad m inistration of oxygen (E) p lacing the p atient in the Trend elenbu rg position 241. Within a few m inu tes the p atient becom es u nresp onsive, p rofou nd ly hyp otensive, and ap n eic. Ap p rop riate m an agem en t w ou ld inclu d e all of the follow ing EXCEPT (A) p ositive p ressu re ventilation via end otracheal intubation (B) left u terine d isp lacem ent (C) su p p ort of m aternal circu lation w ith IV flu id s and ep hed rine (D) ad m inistration of 100% oxygen (E) p rom p t ad m inistration of ep inep hrine to supp ort m aternal circulation

256

15: Obs te tric Ane s the s ia

242. A 32-year-old G1P0 p atient p resents to the labor and d elivery d ep artm ent in early labor. H er prenatal course is unremarkable except for a history of m u ltip le sclerosis (MS). True statem ents regard ing her care inclu d e all of the follow ing EXCEPT (A) an increase in bod y tem p eratu re of as little as 0.5°C can resu lt in an exacerbation or relapse (B) exaggerated resp onses to inhaled anesthetics can occu r d u e to autonom ic d ysfu nction (C) p regnancy is associated w ith an im p rovem ent in sym p tom s, bu t relap se can occu r postpartum (D) ep id u ral anesthesia is associated w ith greater risk of relap se than sp inal anesthesia (E) a hyp erkalem ic resp onse to su ccinylcholine can be seen in p atients w ith significant m u scle atrophy 243. A 30-year-old G2P1 fem ale p resen ts for a rep eat cesarean section at 40 w eeks gestational age. H er p renatal cou rse is com p licated by recurrent pregnancy loss and she is positive for the lu p u s anticoagu lant. She has been on unfractionated heparin 10,000 units subcutaneously tw ice a d ay throu ghou t her third trim ester. H er last d ose w as last evening (tw elve hou rs ago). True statem ents regard ing her care inclu d e (A) regional anesthesia is safe and no fu rther testing is necessary (B) regional anesthesia is contraind icated in this p atient (C) a general anesthetic is ind icated (D) a p artial throm bop lastin tim e and p latelet cou nt shou ld be obtained p rior to p erform ing a regional anesthetic (E) the p eak effect of su bcu taneou s hep arin is 4 h after su bcu taneou s injection DIRECTION S: Use th e follow in g scen ario to answ er Qu estions 244-245: A 27-year-old G2P1 fem ale p resents w ith a history of p rofou nd d ep ression at 34 weeks gestational age. She had a recent suicide attempt and her psychiatrist now recomm end s

electroconvu lsive therap y (ECT). You are asked to cou nsel the p atient regard ing the u se of ECT in p regnancy. 244. Which one of the follow ing statem ents is tru e abou t ECT d u ring p regnancy? (A) The Am erican Psychiatric Association recom m end s that ECT be d eferred u ntil the third trim ester of p regnancy. (B) Most of the p sychotrop ic m ed icines have a long history of safe use in p regnancy. (C) The overall incid ence of m iscarriage is higher w ith the u se of ECT than in the general p op u lation. (D) You r recom m end ation to the p atient can be based on p rosp ective, rand om ized controlled trials. (E) The anesthetic agents u sed for ECT have a long history of safe u se in p regnancy. 245. Ap p rop riate m easu res for p artu rients u nd ergoing ECT w ou ld inclu d e all of the follow ing EXCEPT (A) p reop erative obstetric consu ltation (B) m onitoring of the fetal heart rate before and after ECT (C) end otracheal intu bation (D) m onitoring of u terine contractions after ECT (E) left u terine d isp lacem ent after 14 w eeks gestational age 246. All of the follow ing d ru gs m ay cau se w orsening of m yasthenic sym p tom s EXCEPT (A) (B) (C) (D) (E)

oxytocin am inoglycosid e antibiotics m agnesiu m su lfate terbu taline carbop rost trom etham ine

247. A 27-year-old G2P1 fem ale p resents to you for consu ltation regard ing a vaginal birth after cesarean section (VBAC). H er first cesarean w as a low transverse incision for breech p resentation. You tell her that

Que s tions : 242–251

(A) VBAC is su ccessfu l in less than half of patients in w hom a low -transverse cesarean section w as m ad e d u ring previous cesarean d elivery (B) ep id u ral analgesia d oes not d elay the d iagnosis of u terine ru p tu re (C) ep id u ral analgesia d ecreases the likelihood of su ccessfu l VBAC (D) the risk of u terine ru p tu re is ap p roxim ately 2% (E) according to the American College of Obstetricians and Gynecologists (ACOG), anesthesia providers must be available within 30 min to provide emergency care for patients attempting VBAC 248. Which one of the following is the most common cause of severe postpartum hemorrhage? (A) (B) (C) (D) (E)

Lacerations Retained p lacental tissu e Coagu lop athy Uterine atony Uterine inversion

249. A 38-year-old G0 p atient op ts for in vitro fertilization (IVF). She has u nd ergone horm onal stim ulation and is about to receive human chorionic gonad otrop in (hCG) to ind u ce oocyte m aturation. She w ill p resent for transvaginal egg retrieval 36 h after ad m in istration of hu m an chorionic gonad otrop in (hCG). Which one of these anesthetic consid erations is tru e? (A) She w ill not need to fast if she is given consciou s sed ation. (B) Paracervical block m ay be u sed as the sole anesthetic. (C) Consciou s sed ation is the m ost com m only utilized anesthetic techniqu e. (D) Ad d ing 10 m cg of fentanyl to intrathecal lid ocaine solu tion im proves p ostop erative analgesia bu t increases tim e to am bu lation and d ischarge. (E) General anesthesia d ecreases the su ccessful fertilization rate of the retrieved oocytes.

257

DIRECTION S: Use the follow ing scenario to answ er Questions 250-251: A 38-year-old G1P0 female at 39 w eeks gestational age is being ind uced for preeclampsia. She is given a 4-g bolus of magnesium sulfate follow ed by an intravenous infusion at 2 g/ h. 250. Which one of the follow ing statem ents regard ing the use of magnesium sulfate in this patient is tru e? (A) Magnesiu m su lfate is elim inated p rim arily throu gh the liver. (B) Patellar reflexes are lost at serum magnesium levels of approximately 2 mg/ d L. (C) Magnesiu m su lfate m ay antagonize the effects of neu rom u scu lar blocking agents. (D) Therap eu tic blood concentrations are 4-8 m g/ d L. (E) Magnesiu m su lfate d oes not cross the placenta. 251. The p atient u nd ergoes an u neventfu l d elivery and is sent to the postp artu m floor on an infusion of m agnesiu m su lfate for tw enty-fou r hou rs. A nu rse d iscovers the p atient is having breathing d ifficu lty and you are p aged stat to the room . Up on arrival, you note that the p atient is leth argic. H er blood p ressu re is 110/ 60, heart rate is 70, and she is in severe resp iratory d istress. Man agem en t of th is p atient w ou ld inclu d e all of the follow ing EXCEPT (A) ad m inistration of ep inep hrine 1 m g intravenou sly (B) su p p ort of resp iration w ith bag-m ask and possible end otracheal intu bation (C) d iscontinu ing the infu sion of m agnesium su lfate (D) ad m inistration of calciu m glu conate 1 g intravenou sly (E) obtaining a seru m m agnesiu m level

258

15: Obs te tric Ane s the s ia

252. A 30-year-old G1P0 p atient presents for antenatal cou nseling regard ing her history of a m alignant hypertherm ia (MH ) episod e. Which one of the follow ing recom m end ations is tru e? (A) MH is inherited in an au tosom al recessive fashion. (B) There is a 25% chance that the fetu s w ill be MH -su scep tible. (C) Ester, bu t not am id e, local anesthetic agents are safe in MH -su scep tible patients. (D) H er child w ill need to be treated w ith non-triggering anesthetics in the fu tu re u nless the child u nd ergoes caffeinehalothane contractu re testing and the testing is negative. (E) Once labor begins, she w ill be given prophylactic d antrolene that d oes not cross the placenta. 253. An infant w ith a heart rate of 70, a w eak cry, m inim al m u scle flexion, a grim ace to oropharyngeal su ctioning and acrocyanosis at five m inutes w ould receive an Ap gar score of (A) (B) (C) (D) (E)

2 4 5 6 7

DIRECTION S: Use the follow ing scenario to answ er Qu estions 254-255: You are called to the trau m a room w here you encou nter a 31-year-old G2P1 p atient at 34 w eeks estim ated gestational age. She w as involved in a m otor vehicle accid ent and has su ffered m ultiple injuries. She is actively being resu scitated . Fetal heart tones are p resent. Shortly after your arrival, the p atient goes into card iac arrest d u ring the resu scitation. The obstetrical team is p resent in the trau m a room . 254. All of the follow ing statem ents regard ing the resu scitation of this p atient are tru e EXCEPT (A) left u terine d isp lacem ent shou ld be m aintained d u ring the resu scitation (B) chest com p ressions shou ld be p erform ed slightly above the center of the sternu m

(C) card ioversion has been u sed in all stages of pregnancy w ithou t significant com plications (D) vasop ressor agents m ay d ecrease blood flow to the u teru s (E) stand ard ACLS d ru g d osages shou ld be ad ju sted up w ard for p regnant p atients 255. It has been fou r m inu tes since resu scitative m easu res w ere institu ted . The patient rem ains asystolic. You shou ld now consid er (A) ad m inistering a bip hasic shock of 200 J (B) ad m inistering a m onop hasic shock of 360 J (C) asking the obstetrician to p erform a perim ortem cesarean section (D) ad m inistering a second d ose of vasopressin (E) calling the cod e 256. Well know n gastrointestinal changes that occur d u ring pregnancy includ e all of the follow ing EXCEPT (A) the com bination of esop hageal d isp lacem ent into the thorax and p rogestin resu lt in a low ering of the low er esop hageal sp hincter tone (B) gastric emptying of liquid and solid materials is not altered d uring pregnancy (C) a higher risk of gallblad d er d isease second ary to biliary stasis and an increased secretion of bile (D) gastric em p tying is slow ed d u ring labor (E) ep id u ral anesthesia u sing local anesthetics only d elays gastric em p tying 257. The m ost reliable m eans of d etecting u terine ru pture in this patient is (A) (B) (C) (D) (E)

m aternal tachycard ia u terine tend erness severe hem orrhage a non-reassu ring fetal heart rate p attern m aternal com p laint of p ain d u ring labor

Que s tions : 252–260

258. Regard ing p lacental transfer of d ru gs, w hich one of the follow ing favors increased m aternal to fetal d ru g transfer? (A) Molecu lar w eight less than 1,000 Daltons (B) H yd rop hilic su bstance (C) Charged su bstance (D) H igh bou nd d ru g fraction (E) Low er p ortion of u nionized d ru g in m aternal plasm a 259. In ord er for an ep id u ral to relieve the second stage of labor, the epid ural m ust cover w hich one of these d erm atom es? (A) (B) (C) (D) (E)

T10-L1 T11-T12 L2-L4 L3-L5 S2-S4

259

260. Well know n end ocrine changes that occu r d u ring pregnancy inclu d e all of the follow ing EXCEPT (A) the thyroid gland enlarges d u ring p regnancy (B) estrogen ind u ces an increase in thyroid -bind ing globu lin w hich resu lts in a relative hyp erthyroid state d u ring p regnancy (C) free T3 and free T4 concentrations rem ain u nchanged (D) p regnant p atients are insu lin resistant p rim arily d ue to p lacental p rod u ction of lactogen (E) cortisol levels are increased d u ring p regnancy DIRECTION S: Use the follow ing figu re to answ er Qu estion 261:

260

15: Obs te tric Ane s the s ia

261. You are finishing a rou tine p reop erative evalu ation on a patient for an elective rep eat cesarean section in the antepartu m assessm ent area w hen you view the follow ing fetal heart tracin g on a p atien t in th e n ext room w h o is being assessed for d ecreased fetal m ovem ent at 36 w eeks. She is contracting spontaneou sly. This is an exam p le of (A) (B) (C) (D) (E)

early d ecelerations a reactive non-stress test variable d ecelerations fetal brad ycard ia a p ositive oxytocin contraction test (OCT)

262. A variety of p h ysiologic chan ges in u terop lacen tal blood flow occu r d u rin g p regn an cy an d in clu d e w h ich on e of th e follow in g ch aracteristics? (A) The u teru s accou nts for rou ghly 12% of the card iac ou tp ut at term . (B) Uterop lacental blood flow is w id ely au toregu lated d u ring p regnancy. (C) Uterine blood flow increases d ram atically to ap proxim ately 200 to 300 m L/ m in at term . (D) The m ain su p p ly of blood to the u teru s is from the u terine arteries that arise from the aorta. (E) General anesthetic d oses typ ically u sed d ecrease u terine blood flow. 263. A 27-year-old G2P1 fem ale p resents for anesthetic consultation at 16 w eeks gestational age. H er prenatal cou rse is significant for a history of m ild m itral stenosis (MS). She is cu rrently asym p tom atic (N YH A Class 1 d isease). You tell her that all of the follow ing statem ents about her cond ition are tru e EXCEPT (A) approxim ately 25% of p atients w ith m itral stenosis first exp erience sym p tom s d u ring pregnancy (B) mitral stenosis is the most commonly encountered valvular lesion in pregnancy

(C) ep id u ral anesthesia is p referred for vaginal or cesarean d elivery (D) ep hed rine is the p ressor of choice to treat hypotension (E) m aternal exp u lsive efforts d u ring the second stage of labor are d iscou raged 264. Which one of the follow ing statem ents concerning the m anagem ent of a parturient w ith an u nanticip ated d ifficu lt airw ay is consistent w ith the ASA d ifficu lt airw ay algorithm ? (A) In an elective cesarean section, p roceed u nd er m ask ventilation if the patient has had nothing by m ou th. (B) N ever attem p t blind nasal intu bation becau se of the risk of bleed ing from engorged airw ays. (C) If able to ventilate w ith m ask and the fetu s is in seriou s d istress, no ad d itional steps should be taken to obtain a secu re airw ay and d elivery shou ld p roceed im m ed iately. (D) N ever attem p t cricothyrotom y in a p reeclam p tic p atient becau se of p otential coagu lopathy. (E) If u nable to ventilate or intu bate, attem p t to place an LMA w ith cricoid p ressure and if successful consid er proceed ing to cesarean section in the presence of fetal d istress. 265. Which one of the follow ing changes in lab valu es is expected d uring pregnancy? (A) (B) (C) (D) (E)

Decreased hem atocrit Increased P CO 2 Increased p H Increased creatinine Decreased factors VII, VIII, X, and fibrinogen

266. N onobstetric surgery d uring pregnancy occurs w ith an estim ated frequ ency of 0.3% to 2.2%. Anesthetic concerns regard ing nonobstetric su rgery d u ring p regnancy inclu d e all of the follow ing EXCEPT

Que s tions : 261–269

(A) rap id sequ ence ind u ction shou ld be p erform ed after 16 w eeks gestational age w hen general anesthesia is requ ired (B) regional anesthesia, w hen ap p rop riate, is a reasonable alternative (C) asp iration p rop hylaxis shou ld be accom plished w ith either a nonp articu late antacid and / or a histam ine H 2 antagonist com bined w ith a gastric p rokinetic agent (D) benzod iazep ines are contraind icated d u e to the increased incid ence of cleft p alate (E) left u terine d isp lacem ent for the p revention of aortocaval com pression is not necessary in the first trim ester DIRECTION S: Use the follow ing p hotograp h to answ er Questions 267-268:

261

u teru s is greater than expected size and her betahCG is m arked ly elevated (100,000 m IU/ m L). An u ltrasou nd is obtained and is show n above. 267. The m ost likely d iagnosis in this p atient is (A) (B) (C) (D) (E)

ectop ic p regnancy incom p lete sp ontaneou s abortion tw in gestation m olar p regnancy sep tic abortion

268. Proper anesthetic m anagem ent of this p atient w ou ld inclu d e all of the follow ing EXCEPT (A) blood p rod u cts shou ld be im m ed iately available (B) tw o large bore intravenou s catheters shou ld be placed (C) an intravenou s infu sion of oxytocin shou ld be started after evacu ation of the u teru s has been com pleted (D) close observation of the p atient p ost p roced u re for evid ence of hem orrhage or card iopu lm onary d istress (E) general anesthesia is p referred 269. A p artu rien t is d iagn osed w ith retain ed p lacenta after d elivery and requ ires m anu al exp loration of the u teru s. All of the follow ing are acceptable management strategies EXCEPT

A 27-year-old patient p resents at ten w eeks gestation w ith abnorm al uterine bleed ing, hyperem esis, and bilaterally enlarged ovarian cysts. On exam her

(A) (B) (C) (D) (E)

intravenou s analgesia ep id u ral analgesia sad d le block intravenou s nitroglycerine, 400 m cg ind u ction of general anesthesia w ith 1 m g/ kg ketam ine

262

15: Obs te tric Ane s the s ia

DIRECTION S: Use the follow ing figu re to answ er Qu estions 270-271:

270. The tracing in the grap h show s a p attern referred to as (A) (B) (C) (D) (E)

late d eceleration variable d eceleration early d eceleration m axim al d eceleration late acceleration

271. The typ e of heart rate tracing show n in the grap h is u su ally associated w ith (A) cord com p ression (B) p lacental insu fficiency (C) head com p ressions

(D) acu te fetal asp hyxia (E) tetanic contraction 272. You are asked to p lace an ep id u ral in a p atient w ho is laboring w ith a singleton in breech p resentation. All of the follow ing are tru e regard ing breech p resentation EXCEPT (A) a higher incid ence of congenital abnorm ality (B) a low er frequ ency of p rolap sed u m bilical cord (C) fetal head entrap m ent m ay necessitate the need for rapid ind u ction of general anesthesia

Que s tions : 270–279

(D) cau ses of breech p resentation inclu d e preterm d elivery, m u ltiple gestation, and uterine abnorm alities (E) it accounts for ap p roxim ately 3-4% of all pregnancies 273. Th e u su al blood loss associated w ith an u ncom p licated vaginal d elivery of tw ins is app roxim ately (A) (B) (C) (D) (E)

400 m L 600 m L 800 m L 1000 m L 1200 m L

274. A 27-year-old G1P0 patient is being treated for p reterm labor w ith terbu taline. An ad verse effect of terbutaline u sed for p reterm labor is (A) (B) (C) (D) (E)

card iac arrhythm ia hyp erkalem ia brad ycard ia hyp ertension hyp oglycem ia

275. N erve inju ry d u ring labor and d elivery can resu lt from all of the follow ing EXCEPT (A) com p ression of lu m bosacral tru nk by the head of fetus (B) peroneal nerve injury by lithotomy stirrup (C) ep id u ral hem atom a second ary to block (D) fem oral nerve com p ression by the lithotom y stirru p (E) chem ical contam ination of the su barachnoid sp ace 276. A 35-year-old heroin-ad d icted p artu rient in labor is requesting pain relief. Which one of the follow ing op tions is LEAST d esirable? (A) (B) (C) (D) (E)

Mep erid ine Continu ou s ep id u ral analgesia N itrou s oxid e Bu torp hanol Lu m bar sym p athetic block

277. Uterine ru p tu re is an obstetrical em ergency often necessitating general anesthesia. It is

263

associated w ith significant m aternal m orbid ity and increases neonatal m ortality by 60-fold . Cond itions associated w ith u terine ru p tu re inclu d e all of the follow ing EXCEPT (A) (B) (C) (D) (E)

p reviou s u terine su rgery trau m a grand m u ltip arity intrau terine m anip u lation tw in gestation

278. Well-know n changes that occu r in the central nervou s system of the partu rient includ e all of the follow ing cond itions EXCEPT (A) nerve fibers have increased sensitivity to local anesthetics (B) MAC for inhalational agents is d ecreased by ap proxim ately 40% (C) activation of the end orp hin system (D) no change in d ose requ irem ent for local anesthetic in the first trim ester (E) sp inal CSF volu m e is red u ced DIRECTIONS: Use the following scenario to answer Questions 279-280: You are called emergently to a delivery room where a patient has just undergone a natural childbirth. The nurse informs you that the patient is bleeding and asks for your assistance. The obstetrician states there is a uterine inversion. The patient is pale with a blood pressure of 60/ 40. Her pulse is 125. She is actively bleeding. She has a single 18-gauge IV that is infusing lactated Ringer solution containing 20 U/ L of oxytocin. The IV is running wide-open. 279. Initial step s in the m anagem ent of this p atient inclu d e all of the follow ing EXCEPT (A) obtain large-bore IV access (B) p lace the p atient in the Trend elenbu rg p osition and ad m inister vasop ressors as need ed (C) obtain a blood sam p le for CBC, typ e and cross m atch, and DIC screen (D) continu e infu sing the IV solu tion containing oxytocin (E) im m ed iate attem p t at rep lacem ent of the u teru s

264

15: Obs te tric Ane s the s ia

280. The obstetrician's initial attempts to replace the uterus are unsu ccessful. H e asks for your assistance. While preparing to m ove to the op eratin g room , w h ich on e of th e follow in g m aneu vers m ay be of m ost benefit? (A) (B) (C) (D) (E)

IV bolu s of 2 g of m agnesiu m su lfate IM injection of 5 m g of ritod rine IV injection of 0.2 m g of terbutaline IV bolu s of 50–100 m cg of nitroglycerine p lacem ent of a sad d le block

281. Prophylactic m easu res taken to prevent m aternal hypotension d u ring and follow ing sp inal anesthesia includ e all of the follow ing EXCEPT (A) ad m inistration of 500-1000 m L of crystalloid solu tion (B) left lateral d isp lacem ent of the u teru s (C) Trend elenbu rg p osition after sp inal injection (D) p lacem ent of the sp inal anesthetic w ith the patient in the lateral position (E) infu sion of a vasop ressor 282. An obstetrician asks you to stand by in case he need s to p erform an u rgent cesarean section. H is patient is 8 cm d ilated . The fetal heart rate pattern is nonreassu ring and there is no fetal resp onse to fetal scalp stim u lation. The p atient is com fortable w ith a w orking ep id u ral in p lace. H e states he is going to p erform fetal blood capillary pH testing. All of the follow ing statem ents are tru e regard ing fetal blood capillary p H testing EXCEPT (A) it is a m ethod that cannot be u sed to assess fetal w ell-being d u ring labor of a breech p resentation (B) a fetal scalp p H of 7.25 or higher is consid ered norm al (C) a fetal scalp pH less than 7.20 is ind icative of significant asphyxia and the need for im m ed iate d elivery (D) a fetal scalp pH of 7.20-7.24 is interm ed iate and requires close m onitoring and rep eat sam p ling (E) it requ ires ad equ ate d ilation of the cervix

283. The one find ing p resent in eclam p tic p atients and not in p reeclam ptics is (A) (B) (C) (D) (E)

hyp erreflexia d ecreased u terop lacental p erfu sion p resence of seizu re activity treatm ent w ith m agnesiu m su lfate general vasoconstriction

284. A change in the anatom y or physiology of the stom ach that is associated w ith pregnancy is (A) (B) (C) (D)

d ecreased acid secretion d ecreased gastric em p tying tim e d ow nw ard d isp lacem ent of the p yloru s incom p etence of the low er esop hageal sp hincter beginning in the first trim ester (E) ep id u ral analgesia u sing only local anesthetics slow s gastric em ptying 285. You are called to the p ostp artu m floor to p erform an ep id u ral blood p atch (EBP) on a p atient w ho had a w et tap tw o d ays p rior d u ring the p lacem ent of her labor ep id u ral. You exp lain to her that all of the follow ing are tru e of an epid ural blood patch EXCEPT (A) the success rate for a first EBP approaches 85% (B) p rop hylactic ep id u ral saline bolu s is effective in d ecreasing the incid ence of postd u ral p u nctu re head ache (PDPH ) (C) if the site of d u ral rent is u nknow n, the low erm ost intersp ace shou ld be u sed (D) 15–20 m L of asep tically obtained au tologou s blood is typ ically u sed for the EBP (E) 95% of p ostd u ral pu nctu re head aches last less than one w eek 286. You are called to the d elivery room and are asked to assist in the resu scitation of a new born term infant. The infant is not breathing and the heart rate is less than 100. Step s to take inclu d e all of the follow ing EXCEPT (A) p rovid e w arm th, clear the airw ay and d ry, stim u late, and reposition the infant (B) if the infant rem ains ap neic or the heart rate is less than 100, p rovid e positive pressu re ventilation

Que s tions : 280–291

(C) 100% oxygen shou ld be u sed d u ring assisted ventilation (D) d u ring the first assisted , breath p ositive pressu re at 20 cm H 2O should be m aintained for 4-5 sec at the end of insp iration to overcom e the su rface tension of the lungs and open up the alveoli (E) if m ask ventilation lasts m ore than 2-3 m in, the stom ach shou ld be em p tied w ith an orogastric tu be 287. A 27-year-old p arap legic p atient w ith a T6 lesion p resents to the labor and d elivery su ite in early labor. She is 4 cm d ilated and requ ests an ep id u ral. Which one of the follow ing statem ents regard ing her anesthetic management is tru e? (A) Sp inal anesthesia is contraind icated in this patient. (B) N eu raxial anesthesia is u nnecessary in this patient since the patient has no sensation below the T6 level. (C) The test d ose u sed after p lacem ent of an epid u ral w ill id entify u nintentional su barachnoid injection. (D) Patients w ith a sp inal cord inju ry are m ore prone to orthostatic hypotension that can contribu te to a d ecrease in u teroplacental p erfu sion. (E) If general anesthesia is necessary, a d ep olarizing agent may be u sed w ithou t reservation. 288. The local anesthetic that attains the low est fetal concentration relative to maternal concentration is (A) (B) (C) (D) (E)

lid ocaine rop ivacaine 2-chlorop rocaine m ep ivacaine bu p ivacaine

265

DIRECTION S: Use the follow ing scenario to answ er Qu estions 289-290. You are asked to evaluate a 30-year-old G4P3 fem ale w ho p resents at 37 w eeks gestation for her third cesarean section as w ell as tu bal ligation. H er prenatal course is com plicated by know n p lacenta p revia. She has had no antep artu m bleed ing. She cu rrently w eighs 250 p ou nd s and is 62 inches tall. H er airw ay exam reveals a Mallam p ati class 4 w ith norm al range of m otion. 289. During the anesthesia interview you advise the p atient that (A) (B) (C) (D) (E)

her risk of p lacenta accreta is 25% her risk of p lacenta accreta is 11% her risk of p lacenta accreta is over 50% sp inal anesthesia is contraind icated she m u st u nd ergo general anesthesia for her rep eat cesarean section

290. Du rin g you r p reop erative in terview, you ad vise her of all of the follow ing EXCEPT (A) tw o large-bore intravenou s catheters w ill be p laced (B) she w ill be typ ed and crossm atched for blood (C) com bined sp inal-ep id u ral (CSE) w ill be p erform ed (D) an obstetrical hysterectom y m ay be requ ired (E) the p roced u re w ill be p erform ed u nd er spinal anesthesia to avoid a potentially d ifficu lt airw ay 291. A neu rop athy m anifested as nu m bness, tingling, bu rning, or other p aresthesia that is p robably the m ost com m only en cou ntered neu rop athy related to child birth is (A) (B) (C) (D) (E)

com pression of the lu m bosacral trunk obturator nerve palsy fem oral nerve palsy sciatic nervy palsy lateral fem oral cu taneous nerve palsy

266

15: Obs te tric Ane s the s ia

292. You are called by a colleagu e w ho has tried m ultiple tim es to place an epid u ral in a laboring p atient w ho had corrective su rgery for scoliosis. She asks about the placement of a caud al anesthetic. You rep ly that all of the follow ing statem en ts are tru e abou t h er con d ition EXCEPT (A) cau d al anesthesia is an accep table alternative in this patient (B) id entification of the sacral hiatu s is necessary (C) a test d ose sim ilar to that u sed d u ring lu m bar ep id u ral p lacem ent is not necessary prior to ad m inistration of local anesthetic (D) accu rate p lacem ent of the cau d al need le is confirm ed by the "feel" of the need le p assing through the sacrococcygeal ligam ent that overlies the sacral hiatus betw een the sacral cornu a (E) the need le shou ld only be ad vanced 1 to 2 cm into the caud al canal 293. Patient-controlled analgesia (PCA) is a viable alternative for p ain m anagem ent d u ring labor in patients w ith a contraind ication to or unsuccessfu l attem p t at n eu raxial an algesia. Rem ifentanil PCA has m ore recently been u sed . Tru e statem ents regard ing rem ifentanil PCA includ e all of the follow ing EXCEPT (A) p lasm a concentrations of rem ifentanil in p regnant p atients are abou t half those fou nd in nonp regnant p atients (B) rem ifentanil rap id ly crossed the p lacenta (C) there is m ore nau sea and vom iting com p ared to m ep erid ine PCA (D) the context-sensitive half-tim e of 3.5 m in is not affected by the d u ration of infu sion (E) rem ifentanil is rap id ly m etabolized by p lasm a and tissu e esterases to an inactive m etabolite

294. In a p regnant w om an at term , you w ou ld expect an increase in all of the follow ing valu es EXCEPT (A) (B) (C) (D) (E)

fu nctional resid u al cap acity d ead sp ace tid al volu m e lu ng com p liance insp iratory reserve volu m e

295. A 30-year-old G3P2 fem ale p resents for an anesthesia consu ltation. H er p renatal cou rse is significant for a history of H SV-2 infection. All of the follow ing statem ents regard ing her care are true EXCEPT (A) p rim ary herp es infection is associated w ith virem ia (B) asym p tom atic shed d ing of the viru s m ay occu r in the genital tract (C) the p resence of either active lesions or p rod rom e are an ind ication for cesarean section (D) neu raxial anesthesia is contraind icated in a p atient w ith active recu rrent H SV-2 infection in labor (E) ep id u ral or intrathecal ad m inistration of m orp hine increases the risk of recu rrence of H SV-1, bu t not H SV-2, infection in obstetrical p atients 296. A 30-year-old G1P0 fem ale at 18 w eeks estim ated gestational age has just been d iagnosed w ith a p heochrom ocytom a and p resents for an anesthetic consu ltation. You tell her that all of the follow ing statem ents are tru e EXCEPT (A) early lap aroscop ic resection of the tum or is possible prior to the third trim ester (B) p heochrom ocytom a is associated w ith an increased incid ence of both fetal d eath and intrau terine grow th restriction (IUGR) (C) if su rgical resection is not accom p lished p rior to d elivery, spontaneou s vaginal d elivery is the p referred m ethod of d elivery

Que s tions : 292–300

(D) p henoxybenzam ine is the m ost com m only prescribed α-ad renocep tor antagonist used (E) nicard ip ine m ay be u sed intraop eratively to prevent hypertension d u ring resection of the tu m or 297. Tru e statem en ts regard ing fetal electronic m on itorin g d u rin g n on obstetric su rgery inclu d e all of the follow ing EXCEPT (A) the Am erican College of Obstetricians and Gynecologists (ACOG) acknow led ge there are no d ata to allow sp ecific recom m end ations for fetal heart rate (FH R) m onitoring for obstetric p atients u nd ergoing nonobstetric surgery (B) m onitoring shou ld be p erform ed before and after su rgery (C) intraop erative m onitoring is recom m end ed w hen possible, especially after 24 w eeks gestational age (D) a d ecrease in FH R variability d u ring general anesthesia is an ind ication of fetal com p rom ise and an ind ication for cesarean d elivery (E) an exp erienced obstetric p rovid er shou ld be present to m onitor and interpret the FH R tracing and u terine activity 298. Plasm a cholinesterase concentrations d u ring p regnancy are (A) (B) (C) (D)

highest at term u nchanged from norm al levels increased d ecreased resu lting in a clinically significant p rolongation of am id e typ e local anesthetics (E) d ecreased to a d egree not resu lting in a clinically significant prolongation of su ccinylcholine 299. You are confronted w ith a 28-year-old G1P0 fem ale w ith achond rop lasia. She p resents for an elective cesarean section at 39 w eeks gestational age. Tru e statem ents regard ing her

267

achond roplasia and obstetrical anesthetic managem ent inclu d e all of the follow ing EXCEPT (A) m arked lu m bar lord osis and scoliosis m ay cau se technical d ifficu lties d u ring attem p ts at neu raxial anesthesia (B) it m ay be d ifficu lt to estim ate the ap p rop riate d ose of local anesthetic for singleshot spinal anesthesia (C) sp inal m ay be p referred to ep id u ral injection of local anesthetic (D) achond rop lasia is inherited as an au tosom al d om inant m od e, although m ost cases occu r as a resu lt of sp ontaneou s m u tation (E) althou gh a higher incid ence of d ifficu lt intu bation has been rep orted in achond roplastic patients, m ost reports note no d ifficulty in airw ay m anagem ent. 300. A 27-year-old G1P0 w oman is ad mitted at term for an ex u tero intrap artu m therap y (EXIT) p roced u re. The surgeons p lan to excise a large em bryonic cervical tu m or. All of the follow ing are characteristics of the m aternal and fetal anesthetic care EXCEPT (A) p lacem ent of an ep id u ral catheter for p ostoperative m aternal p ain control (B) p u lm onary d enitrogenation, rap id sequence ind u ction, and end otracheal intu bation of the m other (C) fetal anesthesia p rovid ed by intram u scular injection of opioid and a paralytic agent either by u ltrasou nd gu id ance p rior to u terine incision or d irectly after hysterotom y and d elivery of the fetus (D) 1 to 1.5 MAC of volatile inhalational agent is required for the p roced u re (E) fetal tracheal intu bation after hysterotom y w ith m aintenance of fetoplacental circu lation u ntil the p roced ure is com pleted .

268

15: Obs te tric Ane s the s ia

C

a

r

d

i

a

c

o

u

t

p

u

t

DIRECTION S: Use the follow ing figu re to answ er Qu estion 301:

0

2

4

6

8

10

Ge s ta tion (months )

301. The figu re above show s the change in card iac ou tpu t w ith pregnancy. The d iscrepancy in the tw o lines is d u e prim arily to the effect (A) (B) (C) (D) (E)

on resp iration on u terine blood flow on venou s retu rn of p ressu re on the aorta on the central nervou s system

302. Ad ju vants su ch as ep inep hrine or bicarbonate are often u sed d u ring obstetrical ep id u ral anesthesia. Tru e statem ents regard ing the u se of these ad ju vants includ e all of the follow ing EXCEPT (A) ep inep hrine is m ore effective at p rolonging the action of short-acting local anesthetics than longer-acting agents (B) the ad d ition of sod iu m bicarbonate hastens the onset of the block by increasing the pH closer to the pKa of the local anesthetic (C) hyp otension occu rs less frequ ently w ith ep id u ral ad m inistration of an alkalinized local anesthetic (D) ep inep hrine has intrinsic analgesic effects via stim u lation of p re-synap tic α 2-ad renocep tors that contribu te to

greater reliability and intensity of the block (E) alkalization of bu p ivacaine m u st be perform ed carefu lly becau se the m argin betw een satisfactory alkalization and precip itation is narrow 303. Which one of the follow ing is the correct statem ent regard in g d ru g action and p lacen tal transfer in a partu rient? (A) Placental transfer is m inim al w ith m u scle relaxants. (B) Op ioid s d o not cross the p lacenta. (C) Inhalational anesthetics increase u terine m u scle tone. (D) N itrou s oxid e is contraind icated for cesarean section second ary to interference w ith vitam in B12 synthesis. (E) Thiop ental d oes not cross the p lacenta. 304. True statements regard ing total spinal anesthesia inclu d e all of the follow ing EXCEPT (A) it is a rare com p lication that can resu lt from both intrathecal or ep id u ral ad m inistration of local anesthetic (B) it cannot resu lt from su bd u ral ad m inistration of local anesthetic

Que s tions : 301–309

269

(C) supportive measures to provide oxygenation and prevent aspiration, including endotracheal intubation, may be necessary (D) p reventative m easu res m ay inclu d e w aiting for a p artial sp inal to w ear off and then ad m inistering a second sp inal in the case of elective cesarean section (E) conversion to general anesthesia m ay be necessary

307. A 30-year-old p artu rient at 32 w eeks gestational age presents to the labor and d elivery su ite com p laining of d ecreased fetal m ovem en t an d abd om in al p ain. Sh e ad m its to chronic cocaine u se throughout her p regnancy. H er blood p ressu re is 120/ 60. H er abd om en is firm . A non-stress test is non-reassu ring. An u ltrasou nd is p erform ed w hich reveals no abnorm al p lacental find ings.

305. Well-know n card iovascular changes that occur at term in pregnancy inclu d e w hich one of the follow ing p aram eters?

DIRECTION S (Qu estions 308-309): Each grou p of item s below consists of lettered head ings follow ed by a list of nu m bered p hrases or statem ents. For each nu m bered p hrase or statem ent, select the ON E lettered head ing or com p onent that is m ost closely associated w ith it. Each lettered head ing or com pon en t m ay be selected on ce, m ore th an on ce, or not at all.

(A) A 25% d ecrease in heart rate (B) A 20% increase in system ic vascu lar resistance (C) An increase in m yocard ial contractility (D) A d ecrease in left ventricu lar end d iastolic volu m e (E) A d ecrease in left ventricu lar end systolic volum e DIRECTION S (Qu estions 306-307): Each grou p of item s below consists of lettered head ings follow ed by a list of nu m bered p hrases or statem ents. For each nu m bered p hrase or statem ent, select the ON E lettered head ing or com p onent that is m ost closely associated w ith it. Each lettered head ing or com p on en t m ay be selected on ce, m ore than on ce, or n ot at all. (A) (B) (C) (D) (E) (F) (G)

Placenta p revia Placenta accreta Uterine inversion Ru p tu red u teru s Placental abru p tion Am niotic flu id em bolu s Severe p reeclam p sia

For each p atient, select the m ost likely etiology. 306. A 27-year-old primipara at term in labor at 8 cm ru p tu res her m em branes sp ontaneou sly, sits u p , and states, "My heart." She then becom es hypotensive and u nresponsive. She begins to h av e sign ifican t v agin al bleed in g sh ortly afterw ard s.

(A) (B) (C) (D) (E) (F) (G) (H )

Oxytocin Mifep ristone Carbop rost trom etham ine Vasop ressin Methysergid e Misop rostol Methylergonovine Methylp red nisolone

For each p atien t, select th e m ost ap p rop riate m ed ication . 308. You are called to a room w here a 27-year-old G6P6 female has just delivered via natural childbirth and is bleeding. The patient has a history of severe reactive airw ay d isease and u pon entering the room she is actively wheezing. The obstetrician states the uterus remains boggy and the patient continues to bleed despite the administration of oxytocin and methylergonovine. 309. In a p atient w ith excessive p ostp artu m bleed ing, this m ed ication is intend ed to be ad m inistered solely as an in tr am u scu lar or intramyometrial injection because it may cause severe hyp ertension w hen injected as an IV bolu s.

Answe rs a nd Expla na tions

228. (C) A non-stress test is the easiest test to perform w ith few contraind ications. The m other is p laced on an electronic fetal m onitor. A reassu ring non-stress test w ould d em onstrate tw o fetal heart rate accelerations w ithin a 20-m in period of at least 15 bp m for at least a 15-sec d u ration. A reassuring N ST has a high negative p red ictive valu e m eaning a low risk of fetal d eath w ithin one w eek of testing. A nonreassu ring N ST w ou ld requ ire ad d itional testing w ith either an OCT or BPP. (5:292-3) 229. (E) 2-Chlorop rocaine h as the fastest onset, esp ecially w hen com bined w ith sod iu m bicarbonate. It is also rap id ly m etabolized by both m other and fetu s resulting in little fetal accu m u lation. System ic toxicity is low m aking it one of the safest local anesthetics u sed in pregnancy. (1:579, 5:1157) 230. (A) All of the options are absolu te contraind ications except preexisting neurologic d isease that m ay be a relative contraind ication. (5: 1151) 231. (C) General anesthesia is associated w ith an increased risk of m aternal d eath. This is p artly d ue to the physiologic changes associated w ith p regnancy. The introd u ction of alternative airw ay d evices in the recent past as w ell as the d evelop m ent of the d ifficu lt airw ay algorithm has contributed to the d ecreased risk of m aternal d eath w ith general anesthesia as com pared to regional anesthesia from a 16.7- to a 1.7-fold increased risk. (5:1148, 1159) 232. (C) 2-ch lorop rocain e is know n to have an im p act on th e efficacy of n eu raxial op ioid s. 270

In this case, the m ost likely agent u sed for the em ergent cesarean section w ou ld be 2-chloroprocaine w ith or w ithou t bicarbonate d u e to its rap id onset and low system ic toxicity. (5: 1159) 233. (B) There are varying typ es of vWD, typ e 1 being the m ost m ild and com m on form seen in p regnancy. Most recom m end ations for neu raxial anesthesia are based on exp ert op inions (level of evid ence 5). N eu raxial anesthesia has been u sed safely in the p ast in p atients w ith vWD, how ever, m inim al "safe" factor levels rem ain u nd efined in the obstetrical popu lation and evid ence-based recom m end ations cannot be m ad e. (5:209-10; Choi S, et al. A nesth A nalg 2009; 109:648-60) 234. (D ) Blood p ressu re criteria for severe p reeclam p sia inclu d e a systolic blood p ressu re of greater than or equ al to 160 m m H g or a d iastolic blood pressu re of greater than or equ al to 110 m m H g. The rest w ould be consistent w ith severe p reeclam p sia. (5:294) 235. (D ) The p atient's airw ay exam is concerning. This is a patient w ho m ight very w ell not tolerate labor and require an emergent cesarean section. While it is controversial regard ing an absolu te platelet cou nt below w hich neu raxial anesthesia should be avoid ed , epid u ral analgesia has been su ccessfu lly p laced in p atients w ith p latelet cou nts below 100,000/ m m 3 if there is no other existing coagu lop athy. The p lacem ent of an epid u ral in this patient w ith a p otentially d ifficu lt airw ay is reasonable after inform ed consent has been obtained . In ad d ition, it m ay help to avoid a d ifficu lt airw ay scenario. Waiting until the patient is in active

Answe rs : 228–243

labor w ill only give m ore tim e for the p latelet count to d rop. While a spinal m ay d ecrease the incid ence of hem atom a form ation, the p atient is in labor and m ay requ ire rep eat sp inal anesthetics. This is not only im p ractical, but also d oes not help to avoid a d ifficu lt airw ay scenario shou ld em ergent cesarean section be required . (5:296) 236. (E) Magnesiu m su lfate is u sed in p reeclam p sia to p revent seizu res. Acu te treatm ent of elevated blood p ressu re, m ost com m only w ith hyd ralazine or labetalol, is u sed to p revent cerebrovascu lar com p lications cited as the m ost com m on cau se of m ortality in patients w ith severe p reeclam p sia. (2:987; 5:295) 237. (C) This m ost likely rep resents a su bd u ral p lacem ent of an epid ural catheter. The block d oes not follow the typical pattern and m ay be characterized as p atchy or extensive and is often higher than expected . It can be confu sed w ith subarachnoid injection of local anesthetic. Treatm ent consists of recognizing the subd ural p lacem ent, su pportive m easures, and replacem ent of the catheter. (5:860-1) 238. (D ) Postd u ral p u nctu re head ache (PDPH ) typ ically consists of a p ostu ral head ache in the frontal or occip ital areas. N au sea, vom iting, n eck stiffness, p h otop h obia, an d au d itory changes have all been rep orted . EBP is the d efinitive treatm ent for PDPH . EBP has been u sed su ccessfully in H IV-positive patients and is u nlikely to introd u ce H IV into the CN S as this occurs early in the clinical cou rse of the d isease. Su ccess rates are qu oted at ap p roxim ately 85% after a single blood p atch. The m ost frequent com p lication is back p ain, bu t rare com p lications inclu d ing facial p alsy, brad ycard ia, an d arach n oid itis h ave been rep orted . Prop hylactic EBP is controversial. (5:862-3) 239. (A) The anticholinergic agents atrop ine and scop olam ine d o read ily cross the placenta. In contrast, glycop yrrolate d oes not. Mu scle relaxants d o not read ily cross the p lacenta. (2:62-3)

271

240. (E) The p atient is exp eriencing a high sp inal block. Initial evalu ation and treatm ent w ou ld inclu d e assessing the level of the sp inal by having the p atient squ eeze her hand s and by d em onstrating her ability to speak in fu ll sentences. Vasopressors, fluid ad m inistration, and su p p lem en tal oxygen are all in d icated . Ep hed rine is a better choice than p henylep hrine given the brad ycard ia. Placing the p atient in the Trend elenbu rg p osition to treat the hyp otension m ay increase the height of the block and resu lt in a total sp inal. (5:860) 241. (C) The p atient has p rogressed to a total sp inal anesthetic. Im m ed iate airw ay and circulatory su p p ort are vital. Secu ring the airw ay via end otracheal tu be w ith 100% oxygen, left uterine d isp lacem ent, and p rom p t ad m inistration of ep inephrine are all critical to the care of this p atient. Ep hed rine w ou ld not be as efficaciou s as ep inep hrine at this p oint. (5:860) 242. (D ) Both sp inal and ep id u ral anesthesia has been u sed safely in p artu rients w ith MS. Som e evid ence su ggests there m ay be a higher incid en ce of relap se w ith sp in al an esth esia. Avoid ance of stress and hyp ertherm ia are essential. (5:139-41) 243. (D ) The gu id elines from the Am erican Society of Regional Anesthesia (ASRA) state that 5000 u nits of u nfractionated hep arin su bcu taneously tw ice a d ay d oes not increase the risk of ep id u ral/ sp inal hem atom a. If the p atient is on three tim es a d ay d osing or greater than 10,000 u nits/ d ay su bcu taneou sly, ASRA states the safety of neuraxial blockad e in these patients has not been established . ASRA recom m end s a p latelet cou nt be obtained in p atients on heparin for more than fou r d ays to rule out heparinind u ced throm bocytop enia (H IT). A norm al aPTT w ou ld also be p ru d ent prior to need le p lacem ent. The p eak effect of a su bcu taneou s d ose of heparin is tw o hou rs. (4:954-5; 5:1023; Horlocker T, et al. Reg A nesth Pain M ed 2012; 35: 64-101)

272

15: Obs te tric Ane s the s ia

244. (E) Data from case stu d ies show the incid ence of m iscarriage to be 1.6%. This is low er than the incid ence of m iscarriage in the general p op u lation. Many p sychotrop ic m ed ications are believed to increase the risk of birth d efects. ECT can be safely ad m inistered d u ring all trim esters of pregnancy. (2:351-2) 245. (E) The u teru s d oes not contribute to aortocaval com p ression u ntil ap p roxim ately 18-20 w eeks gestation . En d otrach eal in tu bation shou ld be consid ered after the first trim ester. Monitoring of the fetal heart rate and u terine co n t r a ct io n s sh o u ld b e co n sid er ed . Consu ltation w ith the p atient's obstetrician shou ld also be consid ered . (2:351-2) 246. (E) Many d ru gs can exacerbate the sym p tom s of m yasthenia gravis. Of the d ru gs listed above, carbop rost trom etham ine has not been show n to w orsen the symptoms. (2:1060; 5:996) 247. (B) VBAC is su ccessfu l in 60-80% of w om en in w hom a low -transverse incision w as m ad e d u ring p reviou s cesarean d elivery. The m ajor risk of VBAC is u terine ru p tu re that is estim ated to occu r in less than 1% of p atients u nd ergoing a trial of labor. ACOG states that p hysicians, anesthesia p rovid ers, and other p ersonnel m u st be im m ed iately available to provid e em ergency care for p atients attem p ting VBAC. (2:375-85)

u nd ertaken p rior to canceling the proced u re. Failu re to retrieve the oocytes can increase the risk of ovarian hyp erstim u lation synd rom e. In ad d ition, if the w ind ow for m axim al oocyte retrieval is missed (34-36 h after ad ministration of hCG), sp ontaneou s ovu lation w ill occu r w ith resu ltant loss of oocytes, invalid ating the significant effort and expense incurred d uring horm onal stim u lation. (2:306-13) 250. (D ) Magnesiu m su lfate is p rim arily elim inated by the kid neys. Preeclam p tic patients w ith significant renal im pairm ent m ay d evelop m agnesiu m toxicity m ore easily and shou ld be m onitored m ore closely (w ith serial p atellar reflex exam s an d seru m m easu rem en ts). Patellar reflexes are lost at seru m m agnesiu m levels of approximately 12 mg/ d L. Magnesium potentiates the effects of both d epolarizing and n on d ep olarizin g n eu rom u scu lar blockin g agents. Magnesiu m d oes cross the p lacenta and m ay cau se fetal respiratory com p rom ise. (2:769-71, 986)

248. (D ) Uterine atony is the most common cau se of severe postpartum hem orrhage accou nting for ap p roxim ately 80% or m ore of cases of p rimary (w ithin the first 24 h of d elivery) postpartu m h em orrh age. Desp ite p reven tative m easu res, postp artu m hem orrhage occurs in 4%-6% of pregnancies. (2:818-821)

251. (A) The p atient is exp eriencing m agnesiu m toxicity. This m ay be d ue to a m echanical m alfu nction of the infu sion p u m p or an error in the preparation of the infusion. Initial managem ent shou ld be d irected tow ard treating the resp iratory com p rom ise w ith either bag-m ask ventilation or obtaining a d efinitive airw ay w hen available. Calciu m glu conate shou ld be ad ministered as soon as possible in this patient w ith p resu m ed m agnesiu m overd ose. Th e m agnesiu m infusion shou ld be shut off im m ed iately and a m agnesiu m seru m level shou ld be obtained to confirm the d iagnosis. The p atient's card iovascular statu s is not com p rom ised and therefore epinep hrine is not ind icated at this tim e. (2:986; 5:521-2)

249. (C) Paracervical block m ay be u sed d u ring transvaginal oocyte retrieval, how ever, since this techniqu e incom p letely blocks sensation from the vaginal and ovarian p ain fibers, ad d itional analgesia w ill be requ ired . Consciou s sed ation, neu raxial anesthesia, and general anesthesia have all been u sed su ccessfu lly. Stand ard fasting gu id elines shou ld be u sed , h ow ever, carefu l con sid eration shou ld be

252. (D ) MH is an au tosom al d om inant d isease w ith incom p lete p enetrance. There are very few rep orts of the d evelop m ent of MH d u ring p regnancy. Dantrolene d oes cross the placenta and m ay be associated w ith fetal hyp otonia. There is also one rep ort of u terine atony after ad m inistration of d antrolene, althou gh this w as thou ght to be second ary to the effects of m an n itol. N eu raxial anesth esia sh ou ld be

Answe rs : 244–262

encou raged in all p artu rients w ith MH , how ever, stand ard prep arations shou ld be m ad e in case general anesthesia is requ ired . All local anesthetics are safe in MH patients. (2:1023-31) 253. (C) The Ap gar score is based on five p aram eters that are assessed at one and five m inu tes after birth. Fu rther assessm ents m ay be m ad e at five-m inu te intervals if the initial scores are low. Althou gh the u sefu lness of the Ap gar score is still being d ebated , it is u sed throu ghout the w orld to assess fetal w ellbeing at birth. (2:159-61) 254. (E) The d rug protocols u sed for ACLS in nonp regnant p atients shou ld also be u sed in p regnant p atients. A slightly higher placem ent of the hand s for com p ressions, left u terine d isp lacem ent, and early end otracheal intu bation are all ind icated in p regnant patients. While vasopressor agents m ay d ecrease blood flow to the u teru s, optim al care of the m other is the best therap y for the fetu s. The am ou nt of cu rrent that reaches the fetu s in card ioversion is negligible. (2:900-4) 255. (C) If resu scitative efforts are u nsu ccessfu l, there is som e evid ence to su pport the perform ance of a perim ortem cesarean section (hysterotom y). Evacu ation of the uterus allow s for relief of aortocaval com p ression and restoration of venous return to the heart and may aid e in m aternal resu scitative efforts. It m ay also allow for d elivery of a viable fetus d epend ing on the gestational age and tim e from onset of card iac arrest. Asystole is not a sh ockable rhythm . A second d ose of vasop ressin is not p art of the ACLS protocol. (2:900-4) 256. (E) Gastric em p tying is not altered d u ring pregnancy, bu t is slow ed d u ring labor. In ad d ition, the use of opioid s (intravenously or neuraxially) can also d elay gastric em p tying. Ep id u ral ad m inistration of only local anesthetic has been show n not to d elay gastric em ptying. (2:23-5) 257. (D ) Continu ou s fetal heart rate m onitoring rep resents the best w ay of d etecting u terine rupture. Pain, uterine tenderness, and maternal

273

tachycard ia have both low specificity and sensitivity as d iagnostic sym p tom s or signs of low er u terine segm ent ru p tu re. Most cases of low er u terine segm ent ru p tu re d o not lead to severe hem orrhage as the low er uterine scar is relatively avascu lar. (2:380-3) 258. (A) Factors affecting d ru g transfer across the placenta includ e d rug size, lipid solubility, protein bind ing, pKa, p H , and blood flow. It is the free u nbou nd d ru g fraction that m ore easily crosses the placenta. (2:62-4) 259. (E) The second stage of labor involves the d istention of the vaginal vau lt and p erineu m . Th ese im p u lses arise from th e p u d en d al n erv es th at are com p osed of low er sacral fibers (S2-S4). (2:223) 260. (B) Estrogen d oes ind u ce an increase in the p rod u ction of thyroid -bind ing globu lin resu lting in an increase in the total T3 and T4 concentrations. H ow ever, it is the u nbou nd free T3 and free T4 that are the active horm ones and these rem ain u nchanged d u ring p regnancy. The p atient thu s rem ains eu thyroid . (2:25-6) 261. (E) An OCT can be p erform ed w ith either sp ontaneou s contractions, ad m inistration of intravenou s oxytocin, or by d irect nip p le stim u lation. The d efinition of a positive oxytocin contraction test is three ad equ ate contractions in a ten-m inu te p eriod w ith rep etitive late d ecelerations. A negative OCT is associated w ith a 99% fetal su rvival w ithin one w eek. A p ositive OCT has been associated w ith an ad verse fetal outcome in approximately 40% of cases. This com bination of p ositive OCT and lack of fetal heart rate variability is esp ecially om inou s. (2:97-9) 262. (A) The nonpregnant uterine circu lation d em onstrates autoregulation. In contrast, the pregnant uterus has both placental and nonplacental circu lations. The u terop lacental circulation is a w id ely d ilated , low resistance system w here p erfu sion is largely pressu re-d ep end ent w ith little or no ability for autoregulation. Clinically, u terop lacental flow is d ep end ent u p on m atern al blood p ressu re. In con trast, a sm aller

274

15: Obs te tric Ane s the s ia

p ortion of uterine blood flow supplies the myom etrium and nonplacental end om etrium that may maintain autoregulatory responses in both the pregnant and nonpregnant state. The d oses of general anesthetics u sed clinically have minim al effects on uterine blood flow unless there is significant maternal hypotension. The uterine arteries arise from the internal iliac (hypogastric) arteries. (2:37-40; 5:292) 263. (D ) The goals of anesthetic m anagem ent for this patient shou ld inclu d e: (1) m aintenance of a slow heart rate and sinus rhythm; (2) aggressive treatment of acute atrial fibrillation; (3) avoid ance of aortocaval com p ression; (4) m aintenance of adequate venous return; (5) maintenance of ad equate system ic vascu lar resistance; and (6) prevention of p ain, hyp oxia, hypercarbia, and acid osis, all of w hich can exacerbate p u lm onary hypertension. Epid u ral use allow s for carefu l titration of local anesthetic and m ay help red uce the incid ence of hyp otension. The epid ural also d ecreases m aternal tachycard ia that m ay be poorly tolerated . Phenylephrine is preferred over ep hed rine to treat hypotension given the d irect chronotrop ic effect of ep hed rine. Vacu u m or low ou tlet forcep s d elivery is preferred to m aternal exp u lsive efforts d u ring the second stage of labor thus avoiding the d eleterious effects of the Valsalva m aneu ver. The postpartu m period is of p articular concern for these p atients as the su d d en increase in preload m ay flood the central circu lation and resu lt in th e d evelop m en t of p u lm on ary ed ema. Intensive monitoring should be continu ed for at least tw enty-fou r hou rs after d elivery of an y w om an w ith m itral sten osis. (2:891-4) 264. (E) The u n anticip ated d ifficu lt airw ay in obstetric anesthesia has the p otential for very serious consequ ences w ith tw o patients at risk. The m other's life has p riority, bu t every effort must be mad e to d eliver a viable infant. Airw ay assessm ent sh ou ld be reevalu ated p rior to cesarean section becau se significant changes can occu r d uring labor. In the presence of fetal d istress and the inability to intu bate and ventilate the patient, placem ent of an LMA w ith cricoid pressure and proceed ing to em ergency

cesarean section is an accep table alternative. (2:655-69) 265. (A) Pregnancy is associated w ith m any laboratory d eviations from "norm al." H em oglobin an d hem atocrit d ecrease, p latelets rem ain u nchanged or d ecrease, and m ost coagu lation factors increase, BUN and creatinine d ecrease, and both bicarbonate and P CO 2 d ecrease resulting in little change in p H . (2:120-3) 266. (D ) Benzod iazepines are no longer felt to cause facial cleft d efects and therefore m ay be u sed safely in pregnancy. Incompetence of the low er esop hageal sp hincter and anatom ic changes associated w ith p regnancy both increase the risk of asp iration p neu m onia. Althou gh it is u n clear at w h at stage of p regn an cy th is becom es significant, asp iration p rop hylaxis is p ru d en t in the p regnan t p atient. Regional anesthesia is a reasonable alternative to general anesthesia and p otentially lim its d ru g exp osu re d uring su rgery. The u terus d oes not becom e an intraabd om inal organ u ntil 12-13 w eeks gestation and therefore left u terine d isplacement is not necessary in the first trimester. (2:342-6; 5:1150) 267. (D ) Molar p regnancy occu rs in 1 of 1500 p regnancies in the United States. Clinically the p atient p resents w ith vaginal bleed ing after d elayed m enses. The absence of fetal card iac activity, a u teru s large for gestational age, and a m arked elevation of beta-hCG are all highly su ggestive of hyd atid iform m ole. The u ltrasou nd d em onstrates the presence of hyd ropic villi that is p athognom onic for m olar p regnancy. (2:329-32) 268. (C) Molar p regnancies tend to bleed p rofu sely. Oxytocin infusion is ad vocated either before or d u ring the evacu ation of the u teru s to help red u ce the am ou nt of blood loss. Acu te card iop u lm onary d istress has been observed in as m any as 27% of p atients after evacu ation. While there are m u ltip le cau ses, trop hoblastic em bolization is resp onsible in m ore than half the cases. General anesthesia is p referred over neu raxial anesthesia given the p otential for m assive hem orrhage. (2:329-32)

Answe rs : 263–278

269. (D ) Small d oses of nitroglycerine (50-100 mcg) have been shown to relax the uterus to allow for m anu al extraction of the p lacenta. Cau tion should be used with the use of regional anesthesia in the presence of significant postpartum bleed ing to avoid hypotension. Ketamine is a good induction drug in the presence of maternal bleed ing, but has been show n to prod uce a d ose-related increase in uterine tone. (2:822-23) 270. (C) The p attern rep resented is that of early d ecelerations, in w hich the heart rate d ecreases w ith the onset of contraction, reaches the low p oint w ith the peak of contraction, and then returns to baseline as the uterus relaxes. In contrast, late d ecelerations start after the contraction is u nd erw ay, and the low p oint occu rs after the contraction is over. Variable d ecelerations are variable in shap e and onset. (2:143-6; 5:292-3) 271. (C) The early d eceleration p attern is seen w ith fetal h ead com p ression , w h ich lead s to increased vagal tone. Cord com p ression lead s to variable d ecelerations, w hereas p lacental insu fficiency lead s to late d ecelerations. (2:1445; 5:293) 272. (B) There is a higher frequ ency of p rolap sed cord , esp ecially w ith a footling or com p lete breech p resentation, becau se the fetal head no longer occu p ies the low er u terine segm ent. A higher incid ence of congenital abnorm alities has been fou nd . The fetal head is the largest p resenting part and it is last to p resent, therefore, fetal head entrap m ent can occu r and is a life threatening com p lication of vaginal breech d elivery that m ay requ ire rap id ind u ction of general anesthesia or the u se of either su blingu al or intravenou s nitroglycerine. (2:779-786) 273. (D ) The u su al blood loss in an u ncom p licated vaginal d elivery of tw ins is ap p roxim ately 1000 m L. (2:23; 4:789) 274. (A) Tocolytic agents, su ch as ritod rin e an d terbu taline, relax u terine sm ooth m u scle by d irect stim u lation of β -ad renocep tors. These patients are at increased risk for fluid overload , tach ycard ia, d ysrh yth m ia, hyp erglycem ia,

275

hyp okalem ia, hyp otension, and p u lm onary ed em a. The etiology of p u lm onary ed em a is controversial and has been reported to be both card iac and noncard iac in origin. (2:765-769; 5:293-94) 275. (D ) Com p ression of the lu m bosacral tru nks by the fetal head is probably the m ost com m on cau se of p ostp artu m foot d rop . In correct p atient p ositioning can cau se p eroneal nerve inju ry second ary to the lithotom y stirru p s. Fem oral nerve com p ression can occu r second ary to excessive flexion of the hip in the lithotom y position. Epid ural hem atom a or chem ical contam ination of the subarachnoid sp ace fortunately are rare com plications. (2:701-21) 276. (D ) Op ioid agonist–antagonists m ay trigger w ithd raw al sym p tom s w hen ad m inistered to opioid -tolerant patients. (2:1142) 277. (E) Pred isposing cond itions for uterine rupture inclu d e p reviou s u terine su rgery, esp ecially vertical incisions and p rolonged intrau terine m anipu lation. Traum a and grand m u ltiparity have also been associated w ith u terine ru ptu re. Accord in g to th e Am erican College of Obstetricians and Gynecologists (ACOG), a p atient w ith a tw in gestation w ho is a cand id ate to u nd ergo a vaginal birth after cesarean section (VBAC) is at no greater risk of u terine ru pture than a singleton pregnancy. There has been concern abou t regional anesthesia m asking signs of ru p tu re, esp ecially in p atients attem p ting vaginal birth after cesarean. The exp erience seem s to su p p ort the safety of epid u ral anesthesia if continu ou s electronic m onitoring of u terine activity and the fetal heart rate is p erform ed . (2:817) 278. (D ) Anesthetic requ irem ents are d ecreased in p regnan cy, w ith 25% less local an esthetic n eed ed for region al an esth esia. An atom ic changes, such as d istend ed epid u ral veins or d ecreased volum e of CSF, increase sp read of local anesthetics. H ow ever, these changes are also seen in the first trim ester, w ell before significant m echanical changes have occu rred . It is p ostu lated th at p regn ancy-in d u ced horm onal changes in nerve tissu e sensitivity are

276

15: Obs te tric Ane s the s ia

also resp onsible for the altered sensitivity to local anesthetics. Increased p rogesterone concentrations m ay be responsible for the 25% to 40% red u ction in MAC to general anesthetics. End orp hin concentrations rise d u ring p regnancy thu s contribu ting to an elevated p ain threshold . (2:27-31) 279. (D ) All u terotonics shou ld be d iscontinu ed until after the u terus is rep laced . Uterine relaxation is n ecessary to rep lace the u teru s. Ad d itional help should be summoned as blood loss can be m assive resu lting in hyp ovolem ic shock and m aternal d eath. (Dayan S, et al., A nesth A nalg 1996; 82:1091-3) 280. (D ) IV ad m inistration of nitroglycerine has both a rapid onset (30-40 sec) and a short-lived effect (app roxim ately 1 m in) on u terine relaxation. H yp otension is m inim al at this d ose. Ritod rine, terbu taline, and m agnesiu m su lfate have all been u sed , but none are as rap id in onset and their effects may need to be reversed . A sad d le block w ou ld be im practical and contraind icated given the p rofou nd hypotension. (Dayan S, et al., A nesth A nalg 1996; 82:1091-3) 281. (C) Method s to prevent m aternal hypotension inclu d e flu id ad m inistration, lateral d isp lacem ent of the uterus, placing the patient on her sid e as opposed to the sitting position d u ring placem ent of the sp inal anesthetic, and infu sion of a vasop ressor. A head -d ow n tilt m ay resu lt in m ore cep halad sp read of a hyp erbaric local anesthetic solu tion. This w ill p otentially increase the level of the block and cau se m ore hyp otension. (2:531; 5:1155) 282. (A) Fetal blood cap illary testing m ay be u sed to assess fetal w ell-being d u ring labor. It requ ires ad equ ate cervical d ilation to allow for sampling of the presenting part. It may be used in both vertex and breech p resentations. If the resu lting p H is interm ed iate, continu ed close m onitoring and repeat sam pling w ithin thirty m inu tes is requ ired if d elivery has not been accom plished . (2:148) 283. (C) All the findings are present in preeclampsia and eclampsia, except the presence of seizure

activity. Eclampsia is defined as the new onset of seizures or unexplained coma during pregnancy or the postpartum period in a woman with signs and symptoms of preeclampsia and without a preexisting neurologic disorder (2:998) 284. (D ) The stom ach is d isp laced u p w ard and tow ard the left sid e of the d iap hragm and its axis is rotated 45 d egrees to the right. This d isp laces th e in traabd om in al p ortion of the esophagus into the thorax, thus red ucing low er esophageal sphincter tone. In ad d ition, progestins also contribu te to a relaxation of the low er esop hageal sp hincter tone. (2:23-5) 285. (B) Prop hylactic ep id u ral saline bolu s has not been show n to be effective in d ecreasing the incid ence of PDPH . MRI stud ies obtained after EBP have confirm ed a p red om inantly cep halad sp read of injected blood , therefore the low erm ost in tersp ace sh ou ld be u sed . Most anesthesia p rovid ers inject 10-20 m L of blood . Success rates of up to 85% have been cited after p erform ance of a single EBP w ith a 98% su ccess rate after a second EBP. N inety-five p ercent of postd u ral punctu re head aches last less than one w eek, although rarely symptoms may last m onths or even years. (2:682-95; 5:862-63) 286. (D ) Du ring the first assisted breath, p ositive p ressu re at 30-40 cm H 2O shou ld be m aintained for 4-5 sec at the end of insp iration to overcom e the surface tension of the lu ngs and op en the alveoli. Su bsequ ently, the m axim al p ressu re generated shou ld be betw een 20-30 cm H 2O. (2:164-70) 287. (D ) Loss of sympathetic tone below the level of the lesion d oes rend er the patient more susceptible to orthostatic hypotension. These patients often have low baseline blood pressure and this can be worsened by ad ministration of neuraxial or general anesthesia. Women w ith spinal cord lesions at or above T6 are at increased risk of au tonom ic hyp erreflexia and therefore neu raxial anesthesia is appropriate and ind icated in this patient. N oxious stim uli, blad d er d istention or uterine contractions can all cause malignant hypertension in a p atient w ith autonom ic hyp erreflexia. A stand ard test d ose of 3 m L

Answe rs : 279–296

1.5% lid ocaine w ill not id entify an accid ental su barachnoid injection as the patient is alread y p arap legic. The ad m inistration of succinylcholine to a p arap legic p atient m ay resu lt in a hyperkalem ic response. (2:1056-9) 288. (C) 2-chlorop rocaine is metabolized rap id ly by p lasm a and tissu e esterases, therefore it d oes not attain a high concentration in the fetus. The other agents also cross the p lacenta bu t are broken d ow n m ore slow ly resu lting in higher m aternal:fetal ratios. (2:248-9; 5:1157) 289. (C) The risk of p lacenta accreta is 1:2500 in the general p op u lation. This risk increases in the p resen ce of p lacen ta p revia an d fu rth er increases w ith each su bsequ ent history of p revious cesarean section. In this case, the risk of p lacenta accreta is ap proxim ately 60%. (5:296) 290. (E) This patient is at significant risk for p ostp artu m hem orrhage. She is also potentially at risk for a d ifficu lt airw ay. A CSE w ou ld allow for excellent anesthesia w ith the ability to extend the anesthetic shou ld it be w arranted . Large-bore IV access and the im m ed iate availability of blood is m an d atory. If p lacenta accreta is encou ntered , obstetrical hysterectom y m ay be requ ired . Althou gh the p roced u re cou ld be d one u nd er sp inal anesthesia, this m ethod w ou ld not allow for extension of the neu raxial block and cou ld necessitate general anesthesia in a patient w ith a su sp ected d ifficu lt airw ay. (5:296) 291. (E) Meralgia paresthetica is a neuropathy of the lateral femoral cutaneous nerve and is probably the m ost commonly encountered neuropathy related to childbirth. It may arise during pregnancy or intrapartum. The most likely cause is entrapment of the nerve as it passes around the anterior superior iliac spine beneath or through the inguinal ligament. (2:705-7) 292. (C) In this p atient w ith su rgically corrected scoliosis, epid ural analgesia m ay not be possible. Cau d al anesthesia m ay be m ore technically feasible and is an acceptable alternative. Unintentional d u ral p unctu re and intravascu lar can n u lation are p ossible w ith cau d al

277

an esth esia, esp ecially w h en th e n eed le is ad vanced fu rther into the cau d al canal. A test d ose is therefore recom m end ed . If a continu ou s cau d al is placed , there is possibly a higher risk of infection and larger volu m es of d ru gs are requ ired . (2:236-9, 432-3) 293. (C) PCA w ith m eperid ine, m orphine, or fentanyl have all been used for labor. More recently, remifentanil has been used because of its potential ad vantages resulting from its pharmacokinetic p rop erties that includ e rapid onset and rapid metabolism via plasma and tissue esterases. Remifentanil has been show n to cause less nausea and vomiting compared to other opioids su ch as m ep erid ine. Althou gh rem ifentanil readily crosses the placenta, it is rapidly redistributed and metabolized by the fetus. The context-sensitive half-tim e is u naltered by the d uration of ad ministration. (2:420-3; 5:713) 294. (A) FRC d ecreases at term . There is an increase in d ead sp ace, tid al volu m e, lu ng com p liance, and inspiratory reserve volume at term. (2:19-21; 5:290-1) 295. (D ) Recurrent genital herpes infection d oes not contraind icate the ad m inistration of neu raxial anesthesia. Unlike p rim ary herp es infection, m aternal antibod ies are present w ith second ary infection and therefore virem ia is u nlikely. There are insu fficient d ata to allow a d efinitive recom m end ation regard ing neu raxial anesthesia in patients w ith p rim ary infection. Patients w ith a history of H SV-2 are often p laced on p rop hylactic antiviral m ed ication in hop es of p reventing a recu rrent ou tbreak as this necessitates the perform ance of a cesarean section in ord er to d ecrease the incid ence of neonatal herp es infection. (2:806-7; 5:713) 296. (C) Pheochrom ocytom a is rare d u ring p regnancy w ith an estim ated incid ence of less than 0.2 p er 10,000 p regnancies. Patients p resent w ith a variety of p aroxysm al sym p tom s inclu d ing head ache, sw eating, and p alp itations. H yp ertension and orthostatic hypotension are common. Definitive therapy is surgical resection of the tu m or. Patients are p laced on α- and β -ad renocep tor antagonists p rior to

278

15: Obs te tric Ane s the s ia

resection. The second trim ester is the id eal tim e for lap aroscop ic or op en resection . Alternatives includ e cesarean section at term w ith concu rrent tu m or resection, cesarean at term w ith open or lap aroscop ic resection 2–8 w eeks later, or vaginal d elivery w ith lap aroscop ic resection 6 w eeks later. To p revent increased abd om inal p ressu re on the tu m or d u ring labor, cesarean section is the p referred m ethod of d elivery in patients w ith unresected pheochrom ocytom a. (2:929-34; 5:1124-6) 297. (D ) General anesthetic agents read ily cross the placenta and affect the fetus. In ad d ition, FH R variability d oes not becom e consistently reactive u ntil app roxim ately 27 to 28 w eeks gestation al age. Th erefore, a d ecrease in FH R variability alone is not necessarily a cause for concern. H ow ever, sustained fetal tachycard ia, brad ycard ia, or recu rrent FH R d ecelerations d o su ggest fetal com p rom ise. (2:1161) 298. (E) Plasm a cholinesterase levels are typically d ecreased by 25% d u ring p regnancy. The d ecreased concentrations d o not resu lt in clinically significant effects on ester-typ e local anesthetics or su ccinylcholine in the d oses generally u sed . (2:338-9) 299. (C) Achond rop lasia occu rs w ith a p revalence of 1 in 26,000 live births. Most cases d o occu r from a sp ontaneou s m u tation. N eu raxial anesthesia m ay be d ifficu lt becau se of the associated changes involving the sp inal colu m n. These p atients m ay have d ecreased cervical range of m otion and a higher incid ence of d ifficu lt intu bation has been rep orted in achond rop lastic p atients. Althou gh step s shou ld be taken to deal w ith a potentially difficult airw ay, m ost rep orts show no d ifficu lty w ith airw ay m anagem ent. It can be very d ifficu lt to pred ict the correct d ose of local anesthetic to be u sed d uring neuraxial anesthesia and these patients are at increased risk for either inad equate or high/ total spinal. It has been su ggested that epid u ral anesthesia be used as this allow s the anesthesiologist to titrate the d ose of local anesthetic to the d esired level of anesthesia. (2:1048-49)

300. (D ) EXIT p roced u res are u sed for a variety of situ ations inclu d ing thoracotom y for cystic ad enom atoid m alform ation, transition from p lacental gas exchange to ECMO for expected p u lm onary insu fficiency, excision of giant cerv ical teratom a, an d laryn geal atresia. Anesthetic techniqu e inclu d es the p lacem ent of an epid ural catheter for postoperative analgesia. The sam e p rincip les ap p ly to ind u ction of general anesthesia for an EXIT proced u re as those for cesarean d elivery. H ow ever, su fficient tim e m u st be allow ed after ind u ction of anesthesia to achieve high-end tid al concentrations of volatile agent (2–3 MAC) p rior to surgery to ensu re u terine relaxation and fetal anesthesia. Fetal anesthesia can be su p p lem ented w ith op ioid and / or p aralytic agent ad m inistered via ultrasou nd gu id ance p rior to hysterotom y or d irectly after d elivery of the fetu s. Intu bation is p erform ed after d elivery of the fetal head and shoulders, but the fetus is not ventilated an d fetop lacental circu lation is m aintained . After the su rgical p roced u re is com p leted , the fetu s is given su rfactant if ind icated through the end otracheal tube and ventilated . Once the fetus' oxygenation is ad equate, the um bilical cord is cu t and the infant is given to the neonatology team . Once the infant is d elivered , u terin e relaxation is rap id ly reversed , the u teru s is closed , and the p atient is extu bated at the end of the p roced u re. The ep id u ral is then u sed for p ostop erative p ain m anagem ent. (2:123-31) 301. (C) There is an increase in card iac output as p regnancy progresses u ntil abou t the eighth month, at which time the increase is attenuated. When studying cardiac output in the pregnant patient, one must know whether the study was made with the patient in the supine or lateral position. In the supine position, cardiac output will be decreased because of the weight of the u teru s p rim arily on the inferior vena cava although compression of the aorta also occurs. Up to 15% of term parturients will experience brad ycard ia and a substantial d rop in blood pressure when supine resulting in supine hypotension syndrome. A 10-15 degree left lateral tilt is recom m en d ed in p artu rien ts beyon d 17 to 20 weeks gestational age. (2:18, 531)

Answe rs : 297–309

302. (C) H yp otension occu rs m ore frequ ently w ith epid u ral ad m inistration of an alkalinized local anesthetic likely d u e to the m ore rap id onset of sym pathetic blockad e. (2:271-4) 303. (A) Many drugs administered to the parturient cross the placenta and have neonatal effects ranging from fetal heart rate changes to neonatal d epression. Opioid s are the most commonly used agents during labor and may produce neonatal d epression d epending upon the total dose and time interval from ad ministration to d elivery of the fetu s. Inhalational agents rap id ly cross the placenta and cause uterine relaxation. Muscle relaxants have minimal transfer. (2:62-6) 304. (B) Total spinal anesthesia is a very serious and rare com plication of intrathecal, ep id u ral, or su bd u ral ad m inistration of local anesthetic. Several m echanism s have been p rop osed after failed ep id u ral inclu d ing exp ansion of the ep id u ral sp ace resu lting in com p ression of the sp inal canal and fu rther cep halad sp read of intrathecal anesthetic, rap id transfer of anesthetic from the ep id u ral sp ace throu gh the d u ral hole, and su fficient coverage of the neu ral roots that d ecreases the d ose requ irem en ts of su bsequ en t sp in al an esth esia. Su p portive m easu res inclu d ing vasop ressors and flu id s shou ld be institu ted . End otracheal intu bation an d gen eral anesthesia m ay be requ ired . (2:462-3) 305. (C) There is an increase in left ventricular end d iastolic volu m e and no change in left ventricu lar end -systolic volu m e resu lting in an increased ejection fraction. Card iac contractility is increased , SVR d ecreases, and heart rate increases. (2:16-9; 5:290) 306. (F) Am niotic flu id em bolism occu rs w hen fetal tissu e gets into the m aternal circu lation. It is rep orted to occu r d u rin g tu m u ltu ou s labor.

279

It is now thou ght to be second ary to a m assive au toim m u ne resp onse to the fetal tissue rather that a tru e em bolus to the pu lm onary artery. Patients exp erience card iovascu lar collap se and becom e coagu lop athic. Im m ed iate d elivery of the fetus along w ith aggressive resu scita tio n a n d extr a co r p orea l m em b r a n e oxygenation m ay be ind icated . Despite these efforts, the m ortality rate rem ains high and intact su rvival rem ains low. (Gist R, et al., A nesth A nalg 2009; 108:1599-1602) 307. (E) Placental abru p tion occu rs w ith p rem atu re sep aration of the p lacenta from the u terine w all. Mu ltip le risk factors exist in clu d in g sm oking, cocaine u se, trau m a, hypertension, p reeclam psia, d iabetes, m ultiple p regnancies, and ad vanced m aternal age. Classic find ings inclu d e abd om inal p ain, bleed ing (som etim es concealed ), uterine irritability, and tend erness. Concealed bleed ing m ay not be seen by u ltrasou nd in u p to 50% of cases. (5:296) 308. (F) Althou gh carbop rost trom etham ine is the p referred prostagland in treatm ent for refractory u terine atony, it is relatively contraind icated in patients w ith reactive airw ay d isease. Misop rostol is a p rostagland in E1 analogu e that m ay be u sed in p lace of carboprost trom etham ine in p atients w ith reactive airw ay d isease or p u lm onary hyp ertension. The recom m end ed d ose is 800-1000 m cg per rectum for cases u nresp onsive to other u terotonic agents. (2:818-22; 5:1155-7) 309. (G) Ergot alkaloid s are a class of d rugs u sed to treat u terine atony. Ergonovine and m ethylergonovine are the tw o m ost com m only u sed ergot alkaloid s u sed to treat u terine atony. In cases of life-threatening hem orrhage, a slow intravenou s injection m ay be consid ered in an attem p t to treat both p ostp artu m hem orrhage and hyp otension. (2:821)

This page intentionally left blank

CHAPTER 16

Pe diatric Ane s the s ia Que s tions DIRECTION S (Qu estions 310-396): Each of the num bered item s or incom plete statem ents in this section is follow ed by answ ers or by com p letions of the statem ent. Select the ON E lettered answ er or com pletion that is BEST in each case. 310. Preoperative evaluation of a 4-year-old boy for m yringotom y and p lacem ent of tym p anostom y tubes is concerning for possible d ifficulty w ith airw ay m an agem en t. Ph ysical exam reveals m icrognathia, glossop tosis, and cleft p alate. What is the m ost likely d iagnosis? (A) (B) (C) (D) (E)

Beckw ith Synd rom e Gold enhar Synd rom e Pierre-Robin Synd rom e Treacher Collins Synd rom e Trisom y 21

311. A 2-year-old child is ru shed to the trau m a room for a lap arotom y after su staining m u ltip le inju ries in a m otor vehicle accid ent. The child arrives intu bated and pharm acologically p aralyzed . The child is tachycard ic, m ild ly hypotensive, and hyp otherm ic. All of the follow ing m ay be consequ ences of u nintend ed , intraop erative hyp otherm ia EXCEPT (A) (B) (C) (D) (E)

d ecreased oxygen consu m p tion increased m etabolic rate system ic hyp otension p u lm onary hyp ertension hyp oglycem ia

312. A new born infant born at 33 w eeks gestational age p resents for gastroschisis rep air. In p rep aring m ed ications for this p atient, all of the follow ing are consid erations that w ill affect you r m ed ication selection EXCEPT

(A) increased ventilatory d ep ression from m aternally ad m inistered op ioid s (B) im m atu re blood brain barrier (C) low er p rotein availability for d ru g bind ing (D) increased sensitivity to CN S toxicity of lid ocaine (E) im m atu re enzym e system s for d ru g m etabolism 313. A 3-w eek-old infant, born at 38 w eeks gestational age, w eighing 4 kg, presents for a Lad d ’s p roced ure. In ord er to requ est the ap propriate am ou nt of blood , you w ou ld like to know the allow able blood loss for this p atient. What is the blood volu m e of this patient? (A) (B) (C) (D) (E)

160 m L 200 m L 400 m L 280 m L 320 m L

314. A 6-year-old boy p resents to the hold ing area for elective rep air of an ingu inal hernia. H e is a m ild asthm atic and has not taken any d aily asthm a m ed ications for 5 w eeks. H e is cu rrently w heezing throughou t on physical exam. You inform the m other that you w ill (A) p roceed w ith su rgery w ithou t treatm ent (B) p ostp one u ntil tom orrow after 2 d oses of m ontelu kast (C) refer the p atient to his p ed iatrician for evalu ation and treatm ent (D) p roceed w ith su rgery after albu terol (E) p erform su rgery w ith sed ation and analgesia w ith local infiltration

281

282

16: Pe dia tric Ane s the s ia

315. A 2-year-old child has suffered extensive burns to his head , neck, and torso. What percentage of his bod y has been affected ? (A) (B) (C) (D) (E)

30% of total su rface 35% of total su rface 40% of total su rface 45% of total su rface 50% of total su rface

area area area area area

316. A neonate presents w ith respiratory d istress, a scaphoid abd om en, and absent breath sou nd s on the left sid e of the chest. The incid ence of this congenital lesion is 1 in 2000-5000 live births. Which one of the follow ing statem ents is tru e? (A) Mortality in infants w ith this lesion is 50-70%. (B) 70% of all lesions involve the foram en of Bochd alek. (C) 30% of infants w ith this lesion have an accom p anying congenital urologic abnorm ality. (D) 30% of infants w ith this lesion have an accom p anying congenital card iac lesion. (E) Ap p roxim ately 5% of infants w ith this lesion p resent w ith sym p tom s of bow el obstru ction. 317. A 3-m on th-old African -Am erican baby is sched u led for elective rep air of an ingu inal hernia. H e has an old er brother w ith sickle cell anemia, but he has not had any d iagnostic tests for sickle cell anem ia. H is hem atocrit is 30%. This baby (A) alm ost certainly has sickle cell anem ia (B) should receive a preoperative transfusion (C) shou ld u nd ergo a screening test for H bS p rior to anesthesia (D) m ay u nd ergo anesthesia safely w ithou t fu rther testing (E) has a 50% chance of having sickle cell anem ia 318. A 10-d ay-old infant, born at 27 w eeks gestational age, w eighing 1,100 g at birth, is noted to h ave clin ical sign s an d sym p tom s of

p eriton itis an d in testin al obstru ction . Th e p atien t h as had in creasin g oxygen requ irements over the past 48 h, and has been becoming p rogressively m ore tachyp neic. The p atient is also th rom bocytop en ic, an d h as d evelop ed m etabolic an d resp iratory acid osis. Wh ich one of the follow ing statem ents is tru e of this p atient’s likely d iagnosis? (A) It is an anom aly fou nd p red om inantly in p rem atu re infants. (B) Um bilical artery catheterization shou ld be p erform ed in ord er to m onitor hem atologic and m etabolic abnorm alities. (C) The m ortality is abou t 50%. (D) Card iovascu lar collap se u su ally occu rs early in the cou rse of the illness. (E) Metabolic abnorm alities inclu d e hypoglycem ia resu lting from intestinal m alabsorp tion. 319. A 4-year-old child requ ires p ostop erative intu bation and sed ation in the PICU. It is exp ected that the child w ill rem ain intu bated for m ore than 1-2 d . Which one of the follow ing is the least d esirable m ed ication to u se for sed ation for this p eriod of tim e? (A) (B) (C) (D) (E)

Mid azolam Morp hine Ketam ine Prop ofol Dexm ed etom id ine

320. A 4-d ay-old , fu ll term neonate p resents for rep air of im p erforate anu s. The baby has no other abnorm alities. All of the follow ing are true regard ing transitional circu lation in this p atient EXCEPT (A) the p atient’s p u lm onary vascu lar resistance has d ecreased relative to the pressu res in u tero (B) the p atient’s p ressu res on the left sid e of the heart have increased relative to the pressu res in u tero (C) com p letion of closu re of the d u ctu s arteriosu s requ ires ad equ ate arterial m u scle tissu e

Que s tions : 315–326

(D) m echanical closu re of the d u ctu s arteriosu s has occurred in this patient (E) events d u ring anesthesia m ay cau se a retu rn to fetal circu lation 321. A 1-m onth-old infant, born at 35 w eeks gestational age, presents for inguinal hernia repair. The p atient no longer requ ires oxygen and no longer d em onstrates ep isod es of ap nea and brad ycard ia. A sp inal anesthetic is p erform ed w ith tetracaine and then the p atient is p ositioned for su rgery. Du ring p ositioning the p atient’s legs are in ad verten tly raised u p above the p atient’s torso. What is the m ost likely clinical sign that w ill be seen in the p atient? (A) (B) (C) (D) (E)

A d ecrease in oxygen satu ration Agitation or irritability H yp otension An increase in heart rate Loss of consciou sness

322. A 2-year-old child (w eight 13 kg) is sched uled for circu m cision. The m ost su itable d ose of local anesthetic for a d orsal p enile block is (A) bu p ivacaine 0.25% 8 m L (B) lid ocaine 1% 8 m L (C) lid ocaine 1.5% w ith ep inep hrine 1:200,000 8 m L (D) bu pivacaine 0.25% 15 m L (E) bu pivacaine 0.125% 15 m L 323. Preoperative assessm ent of a healthy ped iatric p atient show s the p atient to be norm otensive w ith a blood p ressu re of 82/ 54. What is the likely age of this p atient? (A) (B) (C) (D) (E)

fu ll term neonate 4 m onths 8 m onths 12 m onths 18 m onths

283

324. A 3-year-old healthy child p resents p reop eratively for elective repair of an u m bilical hernia. The m other inform s you that the child had 4 ounces of apple juice 2 h ago. You recommend (A) (B) (C) (D) (E)

to to to to to

cancel su rgery d elay su rgery by 2 h d elay su rgery by 4 h d elay su rgery by 6 h p roceed w ith su rgery now

325. In the d elivery room , after birth, an infant is noted at 1 m in to be blu e, m otionless and unresp onsive, w ith m inim al resp iratory effort, and heart rate of 70 bp m . At 5 m in after birth, the infant is centrally p ink w ith blu e extrem ities, d em onstrates som e flexion at the hip s and slight grim acing to stim ulation, still has m inim al resp iratory effort, and a heart rate of 80 bpm . Initial resu scitation shou ld includ e all of the follow ing EXCEPT (A) (B) (C) (D) (E)

oxygen rad iant heat intu bation glu cose bicarbonate

326. A new born has Apgar scores of 4 and 5 at 1 and 5 m in, resp ectively, and requ ires resu scitation, inclu d ing intu bation, after birth. Which one of the follow ing is the m ost significant factor associated w ith lack of closu re of the d u ctus arteriosu s in this p atient? (A) (B) (C) (D) (E)

Increased Pa CO 2 Decreased Pa CO 2 Increased Pa O 2 Decreased Pa O 2 Increased p u lm onary artery p ressu re

284

16: Pe dia tric Ane s the s ia

327. A 5-m onth-old infant is anesthetized for correction of an eye cond ition. Im m ed iately after intu bation, bilateral breath sou nd s and chest excu rsion are noted and there is 100% oxygen saturation w ith an FIO 2 of 0.5. After positioning for su rgery, the oxygen satu ration is noted to have d ropped to 94%, no other changes having been m ad e. The m ost likely cau se for this fall in oxygen satu ration is (A) a kinked end otracheal tu be (B) bronchosp asm (C) m igration of the end otracheal tu be into the right m ainstem bronchu s (D) insp issated secretions p lu gging the tu be (E) anesthesia m achine failu re 328. A 20-m onth-old child p resents for rem oval of a sw allow ed foreign bod y. The child is crying intermittently, and is reluctant to separate from his father for any length of tim e. The child is d rooling significantly and ap p ears u nable to sw allow his secretions, bu t is not in resp iratory d istress. H e com plains of a sore throat. Which one of the follow ing is the m ost ap propriate prem ed ication and route for this patient? (A) (B) (C) (D) (E)

Oral m id azolam N asal m id azolam Rectal m id azolam Oral fentanyl Oral ketam ine

329. A neonate is noted to have a m u rm u r. The p atient is not cyanotic, bu t is noted to have d ysp nea, tachyp nea, and d iap horesis w ith eating. What is the m ost likely congenital card iac d efect? (A) (B) (C) (D) (E)

Atrial sep tal d efect Ventricu lar sep tal d efect Tetralogy of fallot Coarctation of the aorta Transp osition of the great arteries

330. An infant is u nd ergoing general anesthesia for pylorom yotom y. A rap id sequ ence ind u ction w as performed w ith propofol and succinylcholine. Anesthesia w as maintained w ith sevoflu rane in oxygen and air. After ind u ction the

p atient w as noted to be tachycard ic. Du ring the su rgery, the p atient is noted to becom e m ore tachycard ic w ith a rise in end tid al CO 2, and an increase in tem p eratu re as m easu red w ith nasop haryngeal tem p eratu re p robe. You are concerned that this cou ld be m alignant h yp ertherm ia. Of th e follow in g sign s an d symptoms, w hich one is the most common first sign of m alignant hypertherm ia? (A) (B) (C) (D) (E)

Cyanosis Dark-colored u rine H yp ercarbia Arrhythm ia H ot circle absorber

331. An 11-m onth-old , 10-kg infant p resents for hyp osp ad ias rep air. The child is otherw ise healthy. An LMA is placed after ind uction and the patient is then ventilated w ith tidal volumes of 80 mL. What percentage of that volu me is respiratory dead space in this patient? (A) (B) (C) (D) (E)

5% 10% 20% 30% 40%

332. You r institu tion is evalu ating variou s ped iatric w arm ing d evices in ord er to choose the m ost effective for use in your operating room. Which one of the follow ing statem ents is tru e of the available ped iatric w arm ing d evices? (A) When u sing rad iant w arm ers, core tem p eratu re shou ld be m easu red to p revent skin burns. (B) Circu lating w ater blankets are not very u sefu l in child ren sm aller than 10 kg d u e to the d ecreasing ratio of bod y surface area to bod y m ass. (C) A w arm air m attress is the m ost u sefu l d evice to keep a child w arm . (D) Use of heat m oistu re exchangers is an efficient w ay to increase a child ’s bod y tem peratu re. (E) Circu lating w ater blankets w ork by convection and therefore, they shou ld not be d irectly in contact w ith the skin.

Que s tions : 327–338

333. A healthy 5-month-old presents for repair of an u m bilical hernia. Ind u ction of anesthesia is u neventful and su rgical preparation and d rap ing are com p leted . At incision it is noted that the patient’s tem perature is 34.9°C. The patient had a norm al tem peratu re in the preoperative area. The m ost im p ortant factor in the operating room contribu ting to the p atient’s cu rrent tem peratu re is w hich one of the follow ing? (A) Bod y tem p eratu re on arrival to the op erating room (B) Room tem p eratu re (C) Lack of a w arm ing blanket (D) Use of cold flu id s (E) Tem p eratu re of p rep solu tions 334. A neonate p resents for rep air of an ingu inal hernia. The p lan is for a sp inal anesthetic. The p arents are agreeable to a sp inal bu t are concerned abou t inju ry to the infant’s spinal cord . You exp lain to the p arents the techniqu e for spinal in a neonate and how to m inim ize the chance of inju ry to the sp inal cord , em phasizing that the conu s m ed u llaris is located at w h ich on e of th e follow in g levels in th e neonate? (A) (B) (C) (D) (E)

First lu m bar vertebra Second lu m bar vertebra Third lu m bar vertebra Fou rth lu m bar vertebra Fifth lu m bar vertebra

335. A 4-year-old p reviou sly healthy child presents for repair of a femur fracture. The plan is for an inhalational ind u ction follow ed by p lacem ent of a peripheral IV and then intu bation. Which one of the follow ing is the m ost app ropriate size u ncuffed end otracheal tube for this child ? (A) (B) (C) (D) (E)

3.5 m m 4.0 m m 4.5 m m 5.0 m m 5.5 m m

336. An 18-m onth-old child presents for repair of tetralogy of Fallot. Du ring the proced ure, prior to card iop ulm onary bypass, the child ’s blood

285

p ressu re is noted to d ecrease over several m inu tes from 88/ 57 to 76/ 41. All of the follow ing are tru e EXCEPT (A) the p atient’s Sp O 2 w ill likely d ecrease w ith this d ecrease in blood p ressu re (B) p henylephrine is a usefu l m ed ication to increase blood p ressu re in this p atient (C) epinephrine is a usefu l m ed ication to increase blood p ressu re in this p atient (D) if the blood pressure continu es to fall, the surgeon shou ld be asked to apply p ressure to the aorta. (E) ephed rine is not a usefu l m ed ication to increase blood p ressu re in this p atient 337. A child is ad m itted w ith an incarcerated ingu inal hernia. The child has nasal congestion and d ischarge, w ith a p rod uctive cou gh. The child is afebrile and lu ng sound s are clear to au scu ltation bilaterally. Which one of the follow ing is the m ost app rop riate w ay to p roceed ? (A) The su rgery shou ld be cancelled . (B) The su rgery shou ld be allow ed to p roceed , bu t the child shou ld not be intu bated . (C) The child should be started on antibiotics, and the surgery should proceed. (D) The su rgery shou ld p roceed w ith carefu l m onitoring. (E) The p atient shou ld be op erated on only u nd er spinal anesthesia. 338. A neonate presents for repair of m yelom eningocele. The neonate w as born fu ll term . All of the follow ing are tru e regard ing anesthetic technique and surgery for this patient EXCEPT (A) su p ine p osition shou ld be avoid ed and therefore the patient should be intubated in the lateral p osition (B) su ccinylcholine m ay be u sed safely in this patient (C) extu bation at the conclu sion of the su rgery is d esirable (D) sp inal anesthesia is a p ossible techniqu e for this p atient (E) this p atient w ill likely need a ventricu lop eritoneal shu nt

286

16: Pe dia tric Ane s the s ia

339. A 6-year-old , 24-kg child p resents for a cyst rem oval. The child has been N PO since m id night and it is now 0800. What is this child ’s app roxim ate flu id requirem ent for the fasting d eficit? (A) (B) (C) (D) (E)

190 m L 240 m L 380 m L 510 m L 580 m L

340. A 12-year-old p atient w ith sickle cell d isease p resents for an exp loratory lap aroscopy and possible laparotom y. The p atient is otherw ise healthy and has not had a vaso-occlu sive crisis in several m onths. Prep aration of this p atient for su rgery shou ld inclu d e all of the follow ing EXCEPT (A) transfu se to a hem oglobin level of 15 g/ d L (B) treat infection (C) m aintain good hyd ration (D) p rovid e good p u lm onary care (E) avoid stasis of blood flow 341. A 6-w eek-old baby born at 34 w eeks gestation presents in the hold ing area for elective rep air of an inguinal hernia. The parents believe that they w ill be taking their child home tod ay after surgery. You inform them (A) they m ay take their child hom e tod ay (B) the child m ay have to stay for several hours (C) the su rgery w ill be p ostp oned u ntil the child reaches 60 w eeks p ostconcep tu al age (D) the child w ill be ad m itted for 23 h of ap nea m onitoring (E) the child w ill need ap nea m onitoring at hom e tonight 342. A 7-year-old , 35-kg girl is sched u led for excision of a large intraabd om inal m ass. H er starting hem atocrit is 36% and the m inim ally accep table h em atocrit is 24%. H ow m u ch blood cou ld the patient lose before transfu sion is necessary?

(A) (B) (C) (D) (E)

250 m L 450 m L 650 m L 950 m L 1100 m L

343. A 3-year-old child presents emergently for repair of an incarcerated inguinal hernia. The patient w as recently diagnosed w ith hypothyroid ism and started treatment less than one week ago. All of the following may be encountered in this patient while undergoing anesthesia EXCEPT (A) (B) (C) (D) (E)

hyp otherm ia hyp oventilation sensitivity to op ioid s sm all m ou th and large tongu e hyp erkinetic m yocard iu m

344. A 7-year-old p atient w ith Dow n synd rom e is ad m itted for d ental extractions. Ad d itional m ed ical h istory in clu d es w ell-con trolled asthm a. The patient had m yringotom y and ear tubes in the p ast w ithout problem s. In p rovid ing anesthesia for this patient, w hich one of the follow ing is tru e? (A) Atrop ine shou ld be avoid ed . (B) Op ioid s shou ld be avoid ed . (C) Preop erative neck m obility shou ld be d ocu m ented . (D) Intu bation shou ld be avoid ed . (E) N eu rom u scu lar blockers shou ld be avoid ed . 345. A 5-year-old boy is ad m itted w ith an op en eye second ary to severe globe laceration. H e had eaten 1 h before his accid ent. General anesthesia is requ ired for the rep air. The intu bation shou ld be accom plished (A) by an aw ake intu bation (B) after injection of 100 m g of su ccinylcholine (C) after ad m inistration of rocu roniu m follow ed by su ccinylcholine (D) after vecu roniu m ad m inistration (E) after inhalation ind u ction w ith sevoflurane

Que s tions : 339–350

346. A 1-d ay-old child presents w ith coughing and choking at his first feed . Follow ing investigation, a d iagnosis of tracheoesophageal fistu la (TEF) is m ad e. Which one of the follow ing statem ents is tru e? (A) Esop hageal atresia is associated w ith tracheoesop hageal fistu la in 10% of cases. (B) Air leak throu gh the fistu la is m inim ized w ith paralysis. (C) Postop erative intu bation is necessary to protect the airw ay from asp iration. (D) Su m p su ction is m aintained in the esop hageal p ou ch to lessen the risk of aspiration. (E) 10% of infants w ith TEF have the associated anom alies of VATER synd rom e. 347. N onanesthetized new borns and infants rely on nonshivering therm ogenesis to help m aintain bod y temperature. All of the follow ing are true of nonshivering therm ogenesis EXCEPT (A) it refers to the increased m etabolism of brow n fat (B) brow n fat is highly vascu larized and contains an abund ance of m itochond ria (C) brow n fat m etabolism resu lts in u p to 25% of the cardiac output being diverted through the brow n fat (D) brow n fat com prises 25% of the infant’s total bod y w eight 348. A neonate d evelop s respiratory d istress soon after birth. A chest rad iograph d em onstrates hyperinflation of the left lung, w ith herniation across the m id line and m ed iastinal shift, and atelectasis of the right lu ng. Which one of the follow ing statem ents is tru e? (A) The right low er lobe is m ost com m only affected in patients w ith this congenital m alform ation. (B) N eonates w ith this congenital m alform ation u su ally p resent w ith card iovascu lar collapse d u e to m ed iastinal shift.

287

(C) This congenital m alform ation coexists w ith congenital heart d isease in abou t 50% of cases. (D) N eonates w ith this congenital m alform ation shou ld be treated w ith assisted ventilation as soon as possible in ord er to im prove gas exchange. (E) Differential d iagnosis of this congenital m alform ation inclu d es congenital cystic lesions and congenital d iap hragm atic hernia. 349. A fu ll term neonate is sched u led for ingu inal hernia rep air. The anesthetic p lan is for spinal anesthesia. Which one of the follow ing statem ents is tru e of sp inal anesthesia in the neonate? (A) It is su itable as the sole technique of anesthesia for p roced u res lasting 2 h or m ore. (B) The apex of the conu s m ed ullaris is u su ally at L2-L3. (C) Epinephrine should never be ad d ed to local anesthetics. (D) Tetracaine 0.4 m g/ kg is a su itable d ose for su barachnoid block. 350. A new born, born at 31 w eeks gestational age, d evelop s nasal flaring, chest retractions, and gru nting soon after birth. Chest rad iograp h d em onstrates d iffu se atelectasis. The m ost likely d iagnosis is (A) (B) (C) (D) (E)

bronchop u lm onary d ysp lasia p atent d u ctu s arteriosu s congenital lobar em p hysem a hyaline m em brane d isease tracheoesop hageal fistu la

288

16: Pe dia tric Ane s the s ia

351. A 32-year-old w om an w ith recently d iagnosed preeclam psia d elivers a baby at 39 w eeks gestational age. Du ring labor and d elivery, it w as noted that th e am n iotic flu id w as heavily stained w ith m econium . Apgar scores for this neonate are 7 and 8 at 1 and 5 min, respectively. Which one of the follow ing statem ents is true? (A) This neonate w ill m ost likely d evelop resp iratory d ifficu lties in the first few d ays of life. (B) This neonate has a very high risk for d eveloping rad iographic evid ence of p neu m othorax. (C) Meconiu m is best rem oved by su ction via an end otracheal tu be. (D) Absence of m econiu m in the m ou th and p harynx preclu d es the presence of m econiu m in the trachea. (E) Meconiu m is an ind ication of fetal d istress. 352. A neonate, w ho w as born at 36 w eeks gestational age w ith gastroschisis and has u nd ergone staged red u ction of the d efect, p resents for final closu re in the op erating room . When planning for fluid management for this patient, all of the follow ing are im p ortant consid erations for this p atient EXCEPT (A) m atu ration of renal fu nction is m ore rap id in fu ll term com p ared to p reterm infants (B) the glom eru lar filtration rate of this p atient is less than 20% of the ad u lt value (C) the glom eru lar filtration rate d oes not affect the neonate’s ability to hand le free w ater (D) the glom eru lar filtration rate reaches the ad u lt valu e by abou t 1-2 years of age (E) p otassiu m excretion is m u ch less efficient in neonates com pared to ad u lts 353. A 4-year-old child is brou ght to the em ergency d ep artm ent at 1 a.m . She w as p u t to bed in app arently good health, bu t aw oke fou r hou rs later crying and having d ifficu lty breathing. Physical exam ination reveals that the child is flu shed , d rooling, sitting u p right, and has

severe inspiratory strid or. Which one of the follow ing statem ents is tru e? (A) The m ost likely d iagnosis is acu te laryngotracheobronchitis. (B) A p ossible d iagnosis is crou p . (C) Rectal tem p eratu re shou ld be checked . (D) The child shou ld be taken straight to the op erating room for intu bation/ em ergency tracheostom y. (E) It is im p ortant to p lace an IV p rior to ind u ction in this p atient. 354. A n eon ate u nd ergoes in halation in d u ction w ith 7% sevoflu rane. A p erip heral IV is p laced and the p atient is then intu bated . Soon after ind u ction and intu bation the p atient is noted to be m ild ly hyp otensive. The hyp otension m ay be exp lained by the fact that the card iac ou tp ut (A) m ay d ecrease significantly becau se of d ecreases in stroke volu m e (B) is not very sensitive to changes in afterload (C) is relatively insensitive to volu m e load ing (D) is reflected by a rightw ard shift of the card iac fu nction curve as com p ared to the ad ult (E) m ay d ecrease significantly d u e to d ecreases in heart rate 355. A full term neonate presents for bilateral inguinal hernia repair. General anesthesia w ith an end otracheal tu be is planned . All of the follow ing are tru e of this p atient’s airw ay EXCEPT (A) a 3.0-m m end otracheal tu be is a su itable first choice for this p atient (B) pressu res of 30 cm H 2O are u su ally requ ired for ad equ ate IPPV (C) this p atient’s glottis is located at the level of C2 (D) p ositive p ressu re ventilation shou ld be cond u cted at a rate of abou t 30-60 breaths p er m inu te (E) the narrow est p art of this p atient’s airw ay is at the cricoid ring

Que s tions : 351–361

356. A new born presents for repair of om phalocele. All of the follow ing are tru e of this d efect EXCEPT (A) it is a central m id line d efect (B) it is a congenital d efect originating in the first trim ester of pregnancy (C) it is u su ally associated w ith infection and loss of extracellu lar flu id (D) it is associated w ith a high incid ence of congenital abnorm alities (E) the herniated bow el is covered by the am nion 357. A new born presents for repair of m yelom eningocele. Du ring m aintenance of anesthesia, end tid al isoflu rane is 0.9%. A colleagu e offering you a break com m ents on the end tid al isoflu rane and exp resses concern that the p atient is not receiving ad equ ate anesthesia. You explain that the concentration is ad equ ate for the patient for all of the follow ing reasons EXCEPT (A) neonates have system (B) neonates have barrier (C) neonates have levels (D) neonates have β -end orp hin (E) neonates have

an im m atu re nervou s an im m atu re blood –brain elevated p rogesterone elevated blood levels of im m atu re liver fu nction

358. A 10-m onth-old w ith sep tic arthritis p resents for p lacem ent of a central venou s catheter for a cou rse of antibiotics. Th e p atien t h as a p eripheral IV in p lace. The patient is preoxygenated , u nd ergoes IV ind u ction follow ed by intu bation. Du ring intu bation it is noted that the Sp O 2 d ecreases from 100% to 86%. The reason for this rap id d esatu ration com p ared w ith ad u lts is d u e to w h ich on e of th e follow ing? (A) H igh resp iratory rate (B) H igh oxygen consu m p tion

289

(C) Sm all end otracheal tu be (D) Sm all exp iratory reserve volu m e (E) Sm all tid al volu m e (TV) DIRECTION S: Use the follow ing scenario to answ er Qu estions 359-360: A 2-year-old child presents for rep air of a ventricu lar sep tal d efect. Preop erative evalu ation reveals congestive heart failure, failu re to thrive, and a pu lm onary:system ic flow ratio greater than 2:1. 359. Which one of the follow ing is the m ost ap prop riate ind u ction techniqu e? (A) H alothane d ialed to 6% on the vaporizer in 100% oxygen. (B) Sevoflu rane d ialed to 6% on the vaporizer in 100% oxygen (C) Propofol 5 mg/ kg with fentanyl 2 mcg/ kg (D) Ketam ine 2 m g/ kg w ith fentanyl 2 m cg/ kg 360. If sh u n t reversal occu rs in traop eratively, w hich one of the follow ing is an ap p rop riate treatm ent? (A) (B) (C) (D)

Ketam ine α-ad renocep tor agonists 50% oxygen and 50% nitrou s oxid e H igh inhaled concentrations of volatile anesthetics

361. In a fu ll term , 4-kg neonate, w hich one of the follow ing is tru e regard ing bod y flu id ? (A) Total bod y w ater (TBW) constitu tes ap p roxim ately 3 kg of this patient’s w eight. (B) Extracellu lar flu id (ECF) accou nts for ap p roxim ately 2.5 kg of this patient’s w eight. (C) Ad ip ose tissu e accou nts for less than 1 kg of this p atient’s w eight. (D) Intracellu lar flu id accou nts for ap p roxim ately 2.5 kg of this p atient’s w eight.

290

16: Pe dia tric Ane s the s ia

362. Fetal circu lation is characterized by right to left shu nting across the d uctu s arteriosu s and the foram en ovale. Fetal circu lation p rogresses to transitional circulation w ith birth that is characterized by w hich one of the follow ing? (A) Com p lete closu re of the d u ctu s arteriosu s (B) Com p lete closu re of the foram en ovale (C) Decreased p u lm onary vascu lar resistance (D) Decreased system ic vascu lar resistance 363. A 3-w eek-old neonate, born at 32 w eeks gestational age presents for exploratory laparotom y for likely bow el obstruction. Du ring the proced ure vecuronium is used for muscle relaxation. After the initial d ose of vecu roniu m given at ind u ction, the p atient w as noted to be breathing sp ontaneou sly d u ring p rep aration and d rap ing for su rgery. An ad d itional d ose of vecuronium w as given prior to the start of su rgery, and it w as noted that the p atient d id not recover from this d ose as rap id ly as from the in itial d ose. Wh ich on e of th e follow in g exp lains the reason for the p rolonged recovery from the second d ose of m u scle relaxant? (A) (B) (C) (D) (E)

Blood brain barrier Volu m e of d istribu tion Albu m in concentrations Metabolism α 1-acid glycop rotein concentrations

364. A 5-w eek-old , 4-kg infant p resents for op en p ylorom yotom y. Electrolyte abn orm alities w ere corrected preoperatively. The patient w as born fu ll term an d is oth erw ise h ealth y. General anesthesia is ind u ced w ith p ropofol 15 m g and rocu roniu m 6 m g. N o op ioid is ad m inistered and the su rgeon injects local anesthesia at the incision site. At the conclu sion of the case, neostigm ine 0.1 m g and glycopyrrolate 0.02 mg are ad ministered . The patient is aw ake, bu t ap p ears w eak and w ith sp ontaneous breathing has tid al volum es of 6-8 m L. All of the follow ing cou ld explain this patient’s persistent w eakness EXCEPT

(A) m u scu latu re is p oorly d evelop ed (B) m u scle m ass is less (C) the m yoneu ral ju nction is not w ell d eveloped (D) total bod y w ater is greater (E) inad equ ate d ose of neostigm ine 365. At the conclu sion of a gastroschisis rep air, arran gem en ts are m ad e to tran sp ort th e p atient, intu bated , to the ICU. The su rgeon requ ests that the p atient rem ains p aralyzed and that you continu e w ith controlled ventilation. Which one of the follow ing is the best circu it to u se for transp ort of this p atient to the ICU? (A) (B) (C) (D) (E)

Map leson Map leson Map leson Map leson Map leson

A B C D E

366. Du ring resu scitation of a new born w ith an Ap gar score of 2 at 2 m in, sod iu m bicarbonate is ad m inistered . All of the follow ing are potential sid e effects of sod ium bicarbonate EXCEPT (A) m etabolic alkalosis (B) hyp ernatrem ia (C) hep atic necrosis if given throu gh a venous catheter w hose tip is in the liver (D) hyp erglycem ia (E) hyp erosm olality 367. You are called to help evalu ate a neonate w ho has p resented to the em ergency d ep artm ent w ith resp iratory d istress. The child is breathing >50 breaths per m in and has sternal and subcostal retractions. All of the follow ing contribu te to increased w ork of breathing in this neonate EXCEPT (A) (B) (C) (D) (E)

overcom ing elastic forces overcom ing resistive forces lam inar air flow increased resp iratory rate rad iu s of the infant’s airw ay

Que s tions : 362–373

368. Du ring anesthesia for a p reviou sly healthy infant having a spica cast p laced , you note that the patient period ically has m ild d esaturations that respond w ell to recruitm ent m aneu vers. Which one of the follow ing best exp lains w hy infants are p rone to airw ay collap se and atelectasis? (A) Closing cap acity is greater than tional resid ual cap acity. (B) Closing cap acity is greater than volum e. (C) Closing cap acity increases w ith increased age. (D) Closing cap acity is greater than tory reserve volum e. (E) Closing cap acity is greater than cap acity.

fu ncresid u al

exp iravital

369. An infant p resents for em ergent rep air of coarctation of the aorta. The infant w as born fu ll term , had no p roblem s after d elivery and w ent home on d ay 2 of life. All of the follow ing are associated w ith coarctation of the aorta EXCEPT (A) (B) (C) (D) (E)

ventricu lar sep tal d efect bicu sp id aortic valve Tu rner synd rom e m itral valve abnorm alities p u lm onic stenosis

370. A 4-year-old child w ith a history of u p p er airw ay obstru ction u nd ergoes tonsillectom y and ad enoid ectom y. The child is otherw ise h ealth y. Th e su rgery an d an esth esia are u neventfu l and the patient is extubated easily and w ithou t com p lication in the op erating room at the conclu sion of su rgery. In recovery, the patient is noted to have sym ptom s consistent w ith postintu bation laryngeal ed em a. The treatm ent for this should inclu d e all of the follow ing EXCEPT (A) inhalation of m ist (B) p arenteral glu cocorticoid s (C) nebu lized racem ic ep inep hrine

291

(D) sed ation (E) head u p p osition 371. A 4-w eek-old , 4-kg child p resents w ith several d ays of nonbiliou s, p rojectile vom iting and inability to tolerate feed s. The child w as born full term, had no problems after birth and w ent hom e on d ay 2 of life. The p atient has had d ecreased u rine ou tpu t and a m etabolic panel d em onstrates m u ltip le m etabolic d erangem ents. The next step in m anagem ent is (A) p roceed to the op erating room for em ergent rep air of the lesion (B) start an IV and give 20 mL/ kg of lactated Ringer solu tion, then p roceed to the operating room for repair of the lesion (C) ad m inister an IV solu tion containing bicarbonate ion (D) ad m it the p atient to the floor, start IV hyd ration, and check a m etabolic panel several hou rs later (E) ad m inister an antiem etic su ch as ond ansetron DIRECTION S: Use the follow ing scenario to answ er Questions 372-373: A 4-year-old child presents to the emergency d epartment w ith acute onset of a high fever, sore throat, stridor, dysphagia, and drooling. The patient is sitting up and appears anxious. 372. The m ost likely d iagnosis is (A) (B) (C) (D)

laryngotracheobronchitis ep iglottitis asp irated foreign bod y laryngeal p ap illom atosis

373. Treatm en t in clu d es all of th e follow in g EXCEPT (A) (B) (C) (D) (E)

hu m id ified oxygen glu cocorticoid s antibiotics intu bation hyd ration

292

16: Pe dia tric Ane s the s ia

374. A 14-year-old girl p resents to the em ergency d epartm ent w ith w heezing and hem optysis. A history reveals solvent abuse. All of the follow ing are possible physiological d erangements in this patient d u e to solvent abu se EXCEPT (A) (B) (C) (D) (E)

hep atic d ysfu nction p erip heral neu rop athy renal d ysfu nction neu trop enia m ethem oglobinem ia

375. A 20-m onth-old child w ith a ventricu lar sep tal d efect presents w ith d ysp nea, tachypnea, and d ecreased activity tolerance. The p atient is tachycard ic w ith Sp O 2 of 90-93%. All of the follow ing are m echanism s that m ay exp lain this patient’s presentation EXCEPT (A) d ecreased p u lm onary vascu lar resistance (B) obstru ction of the left m ain stem bronchus by the left atriu m (C) com p ression of bronchi by d istend ed pu lm onary vessels (D) increased interstitial and alveolar lu ng w ater (E) elevated left atrial p ressu re 376. A neonate w ith a d iagnosis of Pierre Robin synd rom e presents for anesthesia. Which one of the follow ing p reop erative evalu ations is ind icated in this p atient in the setting of this synd rom e? (A) (B) (C) (D) (E)

Card iac echo Chest x-ray Renal u ltrasou nd MRI of the sp ine Lu m bar p u nctu re

377. An infant born at 29 w eeks is brou ght to the operating room for laser treatm ent for retinop athy of prem aturity. The infant w as intu bated for the first tw o w eeks of life. Which one of the follow ing statem ents abou t retinop athy of prem aturity (ROP) is tru e?

(A) ROP is related to incom p lete vascu larization of the retina at birth. (B) ROP is d irectly related to the FIO 2. (C) ROP occu rs in 90% of extrem ely low birth w eight infants. (D) ROP occu rs only after exp osu re to hyp eroxem ia for at least 24 h. (E) ROP d oes not occu r in fu ll term infants. 378. A 5-year-old patient exhibits symptom s of su bglottic ed em a in the recovery area. The p atient has a history of m ild asthm a, abd om inal p ain, chronic constip ation, and is obese. The patient u nd erw ent an u pper end oscopy and colonoscop y u nd er general anesthesia. The p atient w as intu bated after three attem p ts and the rem aind er of the p roced u re and anesthesia w as u neventfu l. All of the follow ing statements about subglottic ed ema are true EXCEPT (A) it is m ore com m on in p ed iatric than ad u lt p atients (B) it m ay be p revented w ith u se of steroid cream on the end otracheal tu be (C) it m ay be p revented by ensu ring a leak arou nd the end otracheal tu be at 3.0 m Eq/ L

DIRECTION S: Use the follow ing scenario to answ er Qu estions 415-416: A 52-year-old m ale, involved in an MVA, is brou ght em ergently to the op erating room for exp loratory lap arotom y second ary to the presence of abd om inal free air and flu id on CT scan. H is p ast m ed ical history is significant for alcoholic cirrhosis, Child ’s class B. Preoperative coagulation stu d ies reveal a p latelet cou nt of 82,000/ m m 3,

p rothrom bin tim e of 18 sec (control 13 sec), and p artial throm bop lastin tim e of 62 sec (control 27 sec). 415. H is ap p arent coagu lop athy from liver d isease can be reversed w ith a combination of all of the follow ing EXCEPT (A) (B) (C) (D) (E)

p latelets fresh frozen p lasm a cryop recip itate factor VIII vitam in K

416. Du ring ind u ction and m aintenance of general anesthesia in this p atient, it is im p ortant to rem em ber that this p atient m ay exhibit a(n) (A) (B) (C) (D)

d ecreased resp onse to catecholam ines d ecreased volu m e of d istribu tion d ecreased clearance of fentanyl increased hep atic blood flow d u ring su rgery (E) increased p rotein bind ing DIRECTION S: Use the follow ing scenario to answ er Qu estions 417-418: A 64-year-old fem ale w ith a history of hypertension and benign ventricu lar arrhythm ias is sched u led for extracorporeal shock w ave lithotrip sy (ESWL) u nd er general anesthesia. You r recom m end ation to the u rologist is that he u se synchronized ESWL. 417. The m ost accu rate statem ent abou t synchronized ESWL is that (A) it generates higher intensity w aves than non-synchronized ESWL (B) shock w aves shou ld be tim ed 20 m sec before the R w ave (C) it requ ires d eep er levels of analgesia (D) it d oes not interfere w ith p acem akers (E) it shou ld be tim ed to corresp ond to the ventricu lar refractory period 418. You ’re told that the new lithotrip tor recently p u rchased by the hosp ital has not yet been ap p roved for u se. Instead , the u rologist w ill need to u se the old er m od el Dornier lithotriptor requ iring a w ater bath. You are now

Que s tions : 412–423

concerned abou t w hich one of the follow ing p hysiologic and hem od ynam ic changes that m ay occur once the p atient is positioned and the proced u re is u nd erw ay? (A) Transient hyp ertension follow ed by hypotension (B) Decrease in venou s retu rn (C) Rise in system ic vascu lar resistance (D) Increased fu nctional resid u al cap acity (E) Brad ycard ia DIRECTION S: Use the follow ing scenario to answ er Qu estions 419-420:An 82-year-old , 100-kg m ale w ith a history of hyp ertension and COPD is und ergoing a transu rethral resection of the prostate u nd er sp inal anesthesia at a T8 level. Thirty m inu tes into the p roced u re he com p lains that he is nau seated , and having trou ble catching his breath. You note his blood pressure has increased from 100/ 68 m m H g to 152/ 94 m m H g, his heart rate has d ecreased from 74 to 56 bp m , and his ECG show s ST elevation in lead II. H e is becom ing restless and trying to sit u p . 419. The management of this patient should includ e all of the follow ing EXCEPT (A) inform the su rgeon (B) check seru m sod iu m , seru m osm olality and hem oglobin (C) start an infu sion of a hyp ertonic saline solu tion (D) ad m inister intravenou s fu rosem id e (E) stop the p roced u re

311

DIRECTION S: Use the follow ing scenario to answ er Qu estions 421-422: A 62-year-old w om an w ith a history of severe gastroesop hageal reflu x is sched u led for a lap aroscop ic fu nd op lication. Past m ed ical history is significant only for a 55 p ack-year history of sm oking. After an u neventfu l ind u ction and intu bation, she is anesthetized w ith isoflurane, oxygen, air, and fentanyl. After insertion of the trocar throu gh the abd om inal w all, she w as p laced in steep Trend elenbu rg position, carbon d ioxid e w as insu fflated , and the p roced u re begu n. Three hou rs later, as the su rgeon is p rep aring to rem ove the trocar, the p atient becom es p rogressively m ore hyp otensive, w ith a concom itant rise in p eak insp iratory p ressu re and d rop in oxygen satu ration. 421. The m ost likely cau se of the hyp otension is (A) (B) (C) (D) (E)

com p ression of the inferior vena cava p atient p osition carbon d ioxid e em bolism p neu m op eritoneu m tension p neu m othorax

422. After increasing inspired oxygen concentration, the priority should be to (A) flatten the table (B) p lace the p atient into the left lateral p osition (C) ad m inister ep inep hrine (D) au scu ltate the chest (E) ask the su rgeon to d ecom p ress the p neu m op eritoneu m

420. As you ad m inister oxygen, d raw blood for labs, and begin m anagem ent of the p atient, you note that he is becom ing lethargic. H is p lasm a [N a + ] is 119 m Eq/ L. The p referred m aneuver to increase the p lasm a [N a + ] to 130 m Eq/ L is

423. An oth erw ise health y 36-year-old m ale is sched uled for an elective excision of vocal cord p olyp u tilizing CO 2 laser u nd er general anesthesia. On exam p atient is 5’11”, 79 kg, w ith a normal airw ay exam. The most effective means to avoid an airw ay fire in this case is

(A) correcting his total bod y sod iu m d eficit of 380 m Eq (B) infu sing isotonic saline to increase [N a + ] 1-1.5 m Eq/ L/ h (C) infusing 0.5 liters of 3% saline over 2 h (D) ad m inistering a potassium -sparing d iu retic

(A) m inim ize insp ired oxygen concentration (B) w rap the end otracheal tu be cu ff w ith m etal tape (C) u tilize an ap neic oxygenation techniqu e (D) fill the cu ff w ith a saline-blu e d ye m ix (E) ad d nitrou s oxid e

312

17: Ane s the s ia for Mis ce lla ne ous P roce dure s

424. A 43-year-old p reviou sly healthy m ale p resents for elective resection of a new ly d iagnosed p heochrom ocytom a. H e began his p henoxybenzam ine 10 d ago. All of the follow ing statem ents are accu rate abou t phenoxybenzam ine EXCEPT (A) d osing shou ld be institu ted before β -ad renocep tor blockad e (B) reversal d ep end s on synthesis of α-ad renocep tors (C) d osage shou ld be ad ju sted accord ing to the levels of u rinary catecholam ine m etabolites (D) it cau ses orthostatic hyp otension and reflex tachycard ia (E) it is a non-com p etitive, non-selective alpha blocker that covalently bind s to α-ad renocep tors 425. An 82-year-old patient is sched uled to und ergo hip arthroplasty w ith a cem ented p rosthesis to rep air a hip fractu re second ary to a mechanical fall. The p atient is anesthetized w ith a sp inal anesthetic, in ad d ition to a prop ofol infu sion for sed ation. Shortly after insertion of the prosthesis, the patient’s blood pressure falls from 128/ 85 to 84/ 40, H R increases from 73 to 108 bp m , Sp O 2 falls from 96% to 78%, and you notice a new right bu nd le branch block on ECG m onitoring. The m ost likely cau se of these changes is (A) m yocard ial ischem ia (B) u se of cem ent containing m ethylm ethacrylate (C) hyp oventilation second ary to sed ation (D) increased intram ed u llary p ressu re (E) hyp ovolem ia 426. A 43-year-old otherw ise healthy male, involved in a m otorcycle accid ent on his w ay to w ork, p resents to the op erating room w ith a p osteriorly d isplaced open tibial fracture. H e is schedu led for em ergent vascular exp loration of the calf w ou nd d u e to d im inished d istal p u lses, w ith su bsequ ent ORIF of the fractu re. The m ost app rop riate analgesic techniqu e is

(A) sciatic nerve block w ith 1.5% m ep ivacaine (B) p atient controlled analgesia w ith m orphine (C) ep id u ral analgesia w ith 0.5% bup ivacaine and fentanyl infu sion (D) com bined sciatic and fem oral nerve block w ith 0.5% bu p ivacaine (E) sciatic nerve block w ith 0.75% rop ivacaine 427. A 24-year-old m ale w as brou ght to the operating room intu bated after su ffering a closed h ead in ju ry from a fall w hile h ikin g. H e p resented for an u rgent ORIF of an op en com m inu ted d istal hu m eru s fractu re. An u p p er extrem ity tou rniqu et w as inflated contin u ously at 100 mm H g above the patient’s systolic blood p ressu re, and rem ained inflated for 2.5 h d u ring the case. The least likely effect from the u se of the tou rniqu et is (A) (B) (C) (D)

u lnar nerve p alsy fibrinolysis w ith lim b rep erfu sion m icrovascu lar throm bosis increased p reload u p on tou rniqu et inflation (E) d ecrease in core tem p eratu re after tourniqu et release 428. A 38-year-old m ale w h o had requ ired a p rolonged extrication after a m otor vehicle accid ent is brou ght to the op erating room for exp loratory lap arotom y for su sp ected ru p tu red sp leen as w ell as fixation of m u ltip le low er extrem ity fractu res. After ind u ction of general anesthesia and end otracheal intu bation, an esop h ageal tem p eratu re p robe is inserted show ing an initial tem p eratu re read ing of 32°C. All of the follow ing m ay be encou ntered EXCEPT (A) m etabolic acid osis (B) im p airm ent of the intrinsic clotting cascad e (C) card iac d ysrhythm ias (D) p latelet d ysfu nction (E) p rofou nd p erip heral vasod ilation

Que s tions : 424–433

DIRECTION S: Use the follow ing scenario to answ er Qu estions 429-430: A 22-year-old 5’10”, 110 kg otherw ise healthy m ale w as brou ght to the em ergency d ep artm ent after being stru ck in the eye w ith a baseball. H is exam revealed an op en globe inju ry for w hich he is now brou ght em ergently to the op erating room . H e states that he ate lu nch ju st p rior to the start of the ball gam e. On exam he has a large neck, 3-fingerbread th thyrom ental d istance, and a Mallam pati Class 3 airw ay. 429. The safest anesthetic techniqu e for this p atient is (A) retrobu lbar block w ith intravenou s sed ation (B) general anesthesia w ith laryngeal m ask airw ay (C) p eribu lbar block w ith intravenou s sed ation (D) general anesthesia w ith aw ake fiberop tic intu bation (E) general anesthesia w ith rap id sequ ence ind u ction 430. The best intu bating cond itions for this p atient w ill be achieved u sing (A) p rop ofol, fentanyl, and su ccinylcholine 1.5 m g/ kg (B) p rop ofol, lid ocaine, and rocu roniu m 1.2 m g/ kg (C) p rop ofol, fentanyl, and rocu roniu m 0.6 m g/ kg (D) prop ofol, fentanyl, rocu roniu m 0.5 m g, su ccinylcholine 1.5 m g/ kg 431. You are asked to begin a case for you r colleagu e. You introd u ce you rself to the p atient, and note that he is a 63-year-old m ale sched u led for rad ical p rostatectom y for p rostate cancer. H e is a very good historian, and tells you that his only m ed ical p roblem is coronary artery d isease, for w hich he u nd erw ent stent placem ent 2 years ago. H is only m ed ication is a beta blocker that he takes d aily. When asked to sign the anesthesia consent, the p atient inform s you that he is a Jehovah’s Witness, and that he refu ses all blood therapy. H e tells you that the su rgeon is aw are, and agrees. After

313

extensive d iscu ssion w ith the p atient in w hich you d iscu ss the variou s op tions as w ell as p ossible com plications from refu sal of such therap y, he still refu ses. You are u ncom fortable caring for this patient. Your next step should be to (A) p roceed w ith the case, as you are ethically obligated to care for this p atient (B) sp eak w ith his health care p roxy in an effort to obtain consent from him / her (C) call a consu lt from the hosp ital ethics com m ittee (D) refu se to care for the p atient, bu t obtain inform ed refu sal (E) call the hosp ital attorney to get a cou rt ord er to give him blood if his life is at stake 432. A 55-year-old m an has u nd ergone an u neventful knee arthroscopy und er general anesthesia. H is on ly significant p reop erative m ed ical p roblem is osteoarthritis. The criterion LEAST im p ortant in d eterm ining ap p rop riateness for d ischarge hom e from the Phase II postanesthesia care unit is (A) (B) (C) (D) (E)

p ain score ability to void oxygenation blood p ressu re p ostop erative nau sea and vom iting

433. A 78-year-old m ale is u nd ergoing rep air of a d etached retina u nd er general end otracheal anesthesia. The su rgeon inform s the anesthesiologist that he plans to inject a bubble of sulfur hexaflu orid e into the vitreal cavity to tam p onad e the retina. As the retina is re-attached , the anesthesiologist shou ld (A) d iscontinue nitrou s oxid e (B) d iscontinue all fluorinated anesthetic gases (C) ensu re that all extraocular m u scles are m axim ally p aralyzed via the ad m inistration of a m uscle relaxant (D) elevate the head of the bed (E) hyp erventilate the patient

314

17: Ane s the s ia for Mis ce lla ne ous P roce dure s

434. A 44-year-old p atient u nd erw ent a 4-h operative proced ure for em ergency repair of a liver laceration and sp lenectom y follow ing blu nt abd om inal trau m a as a resu lt of a m otor vehicle accid ent. In the PACU the patient is noted to have w orsening m etabolic acid osis, p oor u rine outpu t, and a tense and d istend ed abd om en. The su rgical team is qu estioning the p ossibility of abd om inal com partm ent synd rom e. Definitive d iagnosis is m ad e by (A) m easu rem ent of card iac ou tp u t (B) serial m easu rem ent of glom eru lar filtration rate (C) m easu rem ent of p ressu re w ithin the blad d er (D) m easu rem ent of sim u ltaneou s PCWP and CVP valu es (E) serial m easu rem ents of abd om inal girth 435. A 56-year-old , 84-kg patient is sched u led for ou tp atient knee arthroscop y for rem oval of loose intraarticular bod ies. The patient has a 10 year history of severe obstru ctive sleep ap nea for w hich he u ses CPAP nightly. The single factor that w ould increase the safety of postoperative d ischarge of this p atient is (A) a su rgical tim e of less than one hou r (B) p lacem ent of fem oral nerve block to red u ce p ostop erative p ain (C) u se of CPAP by the p atient at hom e (D) ad equ ate m anagem ent of p ostop erative pain w ith ibu p rofen 436. A 74-year-old p atient is sched u led for an elective right hem icolectom y. Du ring her p reoperative evaluation, she tells you r colleagu e that she w as recently d iagnosed w ith open angle glau com a for w hich she w as p rescribed tim olol eye d rop s that she u ses faithfu lly. She w ill requ ire a general anesthetic w ith a neu rom u scu lar blocking agent. You r colleague’s norm al p ractice is to reverse neu rom u scu lar blockad e w ith atrop ine and neostigm ine. Given this p atient’s history, you r colleague shou ld (A) reverse w ith a com bination of glycop yrrolate and neostigm ine (B) m ake no change in his p ractice

(C) avoid all nond ep olarizing neu rom u scu lar blocking agents (D) u se a short acting agent to allow sp ontaneou s return of neu rom u scu lar fu nction (E) reverse w ith a com bination of atrop ine and ed rophoniu m 437. A patient is brou ght to the operating room for rep air of an op en fractu re su stained from a fall from a w ind ow d uring a house fire. The patient w as intu bated at the scene and ventilated w ith 100% oxygen via a bag-m ask-valve d evice d u ring transport to the hospital. The m ost reliable m eth od for d eterm inin g w h ether th e p atient has carbon m onoxid e p oisoning w hile being ventilated w ith 100% O 2 is (A) (B) (C) (D) (E)

rou tine arterial blood gas analysis p u lse oxim etry cap nom etry arterial carboxyhem oglobin level cap nograp hy

438. You are asked to evalu ate an 84-year-old patient sched uled for a right hemicolectomy. In com p arison to a 44-year-old patient u nd ergoing a sim ilar p roced u re, you w ou ld exp ect to see a d ecrease in w hich one of the follow ing p hysiologic p aram eters in the 84-year-old ? (A) (B) (C) (D) (E)

Ventilation-p erfu sion m ism atch Closing volu m e Vital cap acity Alveolar d ead sp ace Resid u al lu ng volu m e

439. A 76-year-old fem ale d river of a m otor vehicle hit by a tru ck is brou ght to the op erating room for em ergent sp lenectom y and stabilization of a p elvic fractu re. She arrives intu bated , w ith a blood p ressu re of 80/ 40 and heart rate of 124. The patient’s p ast m ed ical history is unknow n. Initial labs reveal a seru m p otassiu m concentration of 7.1 m Eq/ L. The seru m p otassiu m concentration w ill N OT be d ecreased by (A) an intravenou s bolu s of calciu m chlorid e (B) hyp erventilation (C) infu sion of glu cose and insu lin

Que s tions : 434–445

(D) albu terol ad m inistered by nebu lizer (E) an intravenou s bolu s of sod iu m bicarbonate 440. A 62-year-old m ale w ith liver cirrhosis, cu rrently on the liver transp lant w aiting list, is u nd ergoing an em ergency evacuation of su bd ural hem atom a after a fall. The neurosurgeon is having d ifficu lty controlling bleed ing. The LEAST likely cau se of the bleed ing is (A) (B) (C) (D) (E)

d issem inated intravascu lar coagu lation sp lenic sequ estration of p latelets d efective fibrin clot form ation d ecreased synthesis of factor VIII red u ced fibrinogen concentrations

441. A 68-year-old m an w ho takes levod op a for management of Parkinson d isease is sched uled to u nd ergo an ingu inal hernia rep air. The patient has a history of typ e II d iabetes as w ell as gastroesophageal reflux. The most appropriate p eriop erative an esth etic m an agem en t inclu d es (A) d rop erid ol as an antiem etic (B) om ission of levod op a on the m orning of su rgery (C) m etoclop ram id e for red u ction of gastric volu m e (D) sp inal anesthesia w ith bu p ivacaine (E) m orp hine for p ostop erative analgesia 442. A 16-year-old m ale, hit by a car w hile crossing the street, is brou ght em ergently to the operatin g room for exp loratory lap arotom y for a p resu m ed liver laceration, as w ell as extensive orthoped ic inju ries. You d iscu ss w ith his parents the p otential need to transfu se blood and blood p rod u cts. When his m other asks w hat the greatest risk of transfu sion is, you tell her (A) m istransfu sion d u e to hu m an clerical error (B) transfu sion-related acu te lu ng inju ry (C) hep atitis C (D) hu m an im m u nod eficiency viru s (E) anap hylaxis

315

443. A 68-year-old w om an w ith 40% second and third d egree burns requ ires bu rn d ebrid em ent tw ice d aily. As she is being m echanically ventilated in the ICU, the su rgical team has asked that the d ebrid ement be d one at the bed sid e u nd er general anesthesia. The LEAST app ropriate agent for a brief general anesthetic in this p atient is (A) (B) (C) (D) (E)

etom id ate ketam ine p rop ofol m id azolam m ethohexital

444. A 63-year-old m an w ith cirrhosis is sched u led for a transju gu lar intrahep atic p ortosystem ic shu nt (TIPS) p lacem ent for m anagem ent of severe portal hypertension w ith ascites. When lying flat he is short of breath that w orsens w hen he sits u p . H e has a room air Sp O 2 of 83%. The only p hysiologic d erangem ent N OT seen in this p atient w ould be (A) im p aired hyp oxic p u lm onary vasoconstriction (B) elevated alveolar-arterial oxygen grad ient (C) intrap u lm onary shu nting (D) d ecreased nitric oxid e concentrations 445. An 88-year-old p atient, w ho tw o w eeks ago u nd erw ent aortic valve replacem ent, is noted d u ring a postoperative clinic visit to have an unstable sternu m and pu ru lent d rainage of his w ou nd . Desp ite having had breakfast, he is sched uled for an em ergent exploration of the w ou nd . When p lanning this p atient’s anesthetic m anagem ent, you recall that in the eld erly (A) MAC is increased for isoflu rane (B) m etoclop ram id e is a u sefu l ad ju nct to p rom ote gastric em p tying (C) the effective d ose for neu rom u scu lar blockad e is d ecreased (D) therm oregu lation is im p aired (E) glom eru lar filtration rate is increased

316

17: Ane s the s ia for Mis ce lla ne ous P roce dure s

446. A 32-year-old w om an su stains an injury to the left recu rrent laryngeal nerve d u ring a near total thyroid ectom y. Which one of the follow ing is the m ost likely p ostop erative find ing? (A) (B) (C) (D) (E)

Ad d u ction of the left vocal cord at rest Ap honia Laryngeal ed em a Strid or Paralysis of the left cricothyroid m u scle

447. You are p lanning the anesthetic m anagem ent of an 82-year-old m an u nd ergoing a carotid end arterectom y. H e has a history of p oorly controlled hyp ertension, as w ell as chronic renal failu re requ iring d ialysis. The LEAST appropriate m ed ication to m anage intraop erative and p ostop erative hypertension w ould be (A) (B) (C) (D) (E)

esm olol hyd ralazine fenold op am nitroglycerine nitrop ru ssid e

448. A 30-year-old p atient p resents for em ergency append ectom y. H e states that he w as recently d iagnosed w ith hypothyroid ism bu t has been u nable to afford the m ed ication p rescribed . The m ost likely m anifestation of his d isease in the perioperative period w ou ld be (A) card iac arrhythm ias w ith ketam ine ad m inistration (B) d ecreased ventilatory resp onse to hypoxia (C) increased MAC of inhalational anesthetics (D) d ecreased sensitivity to m id azolam (E) hyp otension 449. A 46-year-old m ale is brought to the op erating room for irrigation and d ebrid em ent of an open tibial fracture. H e had been trapped in his car after slid ing off the road and hitting a tree d u ring an ice storm . H e rem ained alert and hem od ynam ically stable at the scene d u ring a p rolonged extrication. Evalu ation in the em ergency d ep artm ent reveals only an op en tibial p lateau fractu re as w ell as a ”seatbelt” bru ise

on his anterior chest. H e is hypotherm ic w ith a tem p eratu re of 34°C. All other vital signs, as w ell as EKG and laboratory valu es, w ere norm al. MAC for isoflu rane in this p atient is ap p roxim ately (A) (B) (C) (D) (E)

0.8% 1% 1.25% 1.5% 1.75%

DIRECTION S: Use the follow ing scenario to answ er Questions 450-451: A 22-year-old m ale is brought emergently to the op erating room for exp loration of m ultiple abd ominal stab w ound s. H e w as intubated at the scene d u e to loss of consciousness and resp iratory com promise. BP on arrival to the operating room is 70/ 40, H R is 130 bpm , and oxygen satu ration is 92%. Blood pressure respond s minim ally to volum e resuscitation or vasopressors. Volatile anesthetic causes severe hypotension. Oxygen saturation d rops each time the FIO 2 is d ecreased below 1.0. 450. The m ost ap p ropriate m ed ication to p revent intraop erative recall in this p atient is (A) (B) (C) (D) (E)

rem ifentanil scop olam ine m id azolam d rop erid ol rocu roniu m

451. You consid er u tilizing a BIS m onitor to m easu re d epth of anesthesia. The only factor that d oes not affect the accu racy of BIS m onitoring is (A) (B) (C) (D) (E)

m u scle activity hyp oglycem ia p rop ofol electrical artifact ketam ine

DIRECTION S: Each grou p of item s below consists of lettered head ings follow ed by a list of nu m bered p hrases or statem ents. For each nu m bered p hrase or statem ent, select the ON E lettered head ing or com p onent that is m ost closely associated w ith it. Each

Que s tions : 446–456

lettered head ing or com p onent m ay be selected once, m ore than once, or not at all. (A) (B) (C) (D) (E)

N ear m iss H azard ou s cond ition Sentinel event Latent error Preventable ad verse event

(A) (B) (C) (D) (E) (F) (G) (H )

317

Plagu e Botu lism Sm allp ox Bru cellosis Anthrax Ebola Q fever Tu larem ia

For each situ ation, choose the ap p rop riate p atient safety concern.

For each p atient su sp ected to be a victim of biologic terrorism , select the m ost likely biological agent.

452. An anesthesiologist new to the institu tion is asked to p rovid e anesthesia for a 1-year-old requ iring an u rgent MRI. It has been several years since she has cared for a ped iatric patient, and she has never before p rovid ed anesthesia in an MRI environm ent.

454. A 46-year-old m an p resented w ith fever, fatigu e, and shortness of breath 5 d after su sp ected exposu re. Chest x-ray reveals m ed iastinal w id ening and bilateral p leu ral effu sions. The patient w as ad mitted to a regular ICU bed , and com bination therap y begu n w ith cip rofloxacin and clind am ycin.

453. An 18-year-old p atient w as ad m itted to the emergency d epartment after a bicycle accid ent. The Glasgow com a score at the scene w as 12 that d im inished to 7 in the em ergency d epartm ent. A CT scan of the head revealed an epid u ral hem atom a com pressing the left parietal lobe. The p atient w as brou ght em ergently to the operating room for evacuation of the hematom a. Prior to ind u ction of anesthesia, it w as noted that his left p u p il w as now fixed and d ilated . The patient w as qu ickly prepp ed and d rap ed , and a right craniotom y w as p erform ed . DIRECTION S: Each grou p of item s below consists of lettered head ings follow ed by a list of num bered p hrases or statem ents. For each nu m bered p hrase or statem ent, select the ON E lettered head ing or com p onent that is m ost closely associated w ith it. Each lettered head ing or com p onent m ay be selected once, m ore than once, or not at all.

455. Tw elve d ays after su sp ected exp osu re, a 28-year-old m an p resents w ith recent onset of fever, vom iting, backache, and a rash, w hich initially began on his face and extrem ities. Exam reveals vesicles and p u stu les on his face and extrem ities, and a m acu lop apu lar rash on his trunk, w ith lesions at the same stage at each location . H e is im m ed iately p laced in a negative-p ressu re room on strict isolation p recau tions u ntil resolution and scabbing of the lesions. 456. Tw enty-four hours after su spected exposu re, a 56-year-old w om an d evelops symptom s of d ry m ou th, d ouble vision, d ifficu lty sp eaking, and sw allow ing. Becau se the d iagnosis w as m ad e early, she w as given a single d ose of an equ ine antitoxin.

Answe rs a nd Expla na tions

397. (D ) Ap p lication of p ositive p ressu re ventilation d uring preoxygenation d ecreases atelectasis form ation and im proves oxygenation. Four VC breaths w ith 100% oxygen w ithin 30 sec have been su ggested as su perior to 3 m in of 100% p reoxygen ation in obese p atien ts. Preoxygenation in the head -u p or sitting p osition is m ore effective and significantly extend s the tolerance to apnea in obese patients w hen com p ared w ith the su p ine p osition. Placing the patient in the ”ram ped ” position, w ith the head , neck, u p p er bod y, and shou ld ers elevated w ith fold ed tow els or blankets, raises the chest to a point w here an imaginary horizontal line can be d raw n from the sternal notch to the external ear to better im p rove laryngoscop y and intu bation, not to im prove the effectiveness of preoxygenation. (5:312) 398. (E) Rhabd om yolysis has been d ocu m ented in m orbid ly obese p atients u nd ergoing p rolonged p roced u res; the m ain risk factor is p rolon ged d u ration of su rgery. Elevation s in seru m creatinine and CPK levels unexplained by other reasons and com p laints of bu ttock, hip , or shou ld er p ain in the p ostop erative p eriod should raise the su sp icion of rhabd om yolysis. Measu rem ent of seru m CPK p reand p ostop eratively aid s in early d iagnosis and treatm ent. Early and aggressive volu m e rep lacem ent is m and atory in p atients w ith rhabd omyolysis; alkaline fluid s m ay be beneficial in p reventing tu bu lar necrosis and cast form ation (5:310, 2306)

318

399. (C) The Child -Pu gh classification has been u sed to assess p rognosis in cirrhosis and to p rovid e stand ard criteria for listing liver transp lantation. Scoring is based u p on the objective criteria of seru m biliru bin, seru m albu m in, and p rothrom bin tim e as exp ressed by IN R, as w ell as u p on su bjective criteria of d egree of ascites and severity of encephalopathy. This system for assessing the need for liver transplantation has been replaced by the mod el for end -stage liver d isease (MELD). The score is calcu lated u sing three objective variables on ly: seru m biliru bin , p roth rom bin tim e expressed as INR, and serum creatinine. MELD scores of < 10, 10-14, and > 14 rou ghly corresp ond to Child -Pu gh classes A, B, and C, resp ectively. (5: 186-8, 2526) 400. (B) Patients w ho have u nd ergone p reviou s gastric byp ass su rgery m ay exhibit signs and sym p tom s of m etabolic and nu tritional abnorm alities related to m alabsorp tion. Com m on d eficiencies inclu d e vitam in B12, iron, calciu m , and folic acid . A collective form of postop erative polyneu ropathy know n as acu te postgastric red uction surgery (APGARS) neu ropathy cou ld result. Patients w ith APGARS neuropathy p resent w ith p rotracted p ostop erative vom iting, hyp oreflexia, and m u scu lar w eakness. Differential d iagnoses of this d isord er includ e thiamine d eficiency (Wernicke encephalop athy, beriberi), vitam in B12 d eficiency, and Guillain-Barre synd rome. Vitamin A is required for norm al vision, as w ell as iron u tilization

Answe rs : 397–406

and hu m oral and T-cell m ed iated im m u nity. Deficiency d u e to m alabsorp tion m ay cau se n igh t blin d n ess, as w ell as com p rom ise im m u ne d efenses to infection. It d oes not, how ever, resu lt in neu rop athy. (5:310, 600-1, 2260) 401. (C) N eck circu m ference has been id entified as the single biggest p red ictor of p roblem atic intu bation in morbid ly obese p atients. A larger neck circum ference is associated w ith the m ale sex, higher Mallam p ati score, grad e 3 view s at laryngoscop y, and obstru ctive sleep ap nea. Intu bation d ifficu lty correlates better w ith increased age, m ale sex, tem p orom and ibu lar joint p athology, Mallam p ati classes 3 and 4, history of OSA, and abnormal upper teeth than w ith BMI. (5:311) 402. (A) Prop ofol’s total clearance and volu m e of d istribu tion at stead y state correlate w ith total bod y w eight; how ever, its negative card iovascular effects com bined w ith the negative physiologic effects of obesity on the card iovascu lar system su ggest ind u ction d osing shou ld be based on lean bod y w eight. Su ccinylcholine shou ld be d osed on total bod y w eight d ue to the larger extracellu lar flu id com partm ent and linear increase in pseud ocholinesterase activity seen in obese p atients. Su fentanil and m id azolam each have an increased volu m e of d istribu tion and a prolonged term inal half-life in obese patients, but an unaffected plasm a clearance, so that ind u ction d oses shou ld be based u pon total bod y w eight. (5:308-10) 403. (D ) Arterial oxygenation d uring laparoscopy in m orbid ly obese p atients is affected m ainly by bod y w eight and not bod y p osition, p neu m op eritoneu m , or m od e of ventilation, and oxygenation is not significantly im proved by increasing either the resp iratory rate or tid al volum e. PEEP is the only ventilatory p aram eter that has consistently been show n to improve respiratory function in obese subjects, although

319

it m ay d ecrease venou s retu rn, card iac ou tp u t, and su bsequ ent oxygen d elivery (5:314) 404. (B) Fu n ction al resid u al cap acity w ill be d ecreased to ap p roxim ately 65% of the valu e seen in a n on obese in d ivid u al. Resid u al volum e, total lung capacity, forced vital capacity, and the forced exp iratory volu m e in 1 sec are m inim ally d ecreased in com p arison w ith a nonobese ind ivid ual. (5:2093) 405. (D ) Postoperative hem orrhage can resu lt in a rap id ly exp and ing hem atom a that can d irectly compress the trachea lead ing to signs of airw ay com pression. Chronic com pression by a large goiter can lead to tracheom alacia and su bsequ ent tracheal collap se. Unilateral recu rrent laryngeal nerve inju ry can cau se u nilateral vocal cord paralysis that m anifests as hoarsen ess, breath lessness, glottic in com p etence, p oor cough, and aspiration. Bilateral recu rrent laryngeal nerve inju ry p rod u ces bilateral vocal cord paralysis that w ill lead to airw ay obstru ction and strid or; signs and sym p tom s, how ever, are n oted alm ost im m ed iately after extu bation . H yp ocalcem ia m ay d ev elop acu tely second ary to inad verten t inju ry or rem oval of p arathyroid gland s d u ring su rgery. The hallm ark of hyp ocalcem ia is neu rom u scu lar irritability (m u scle w eakness, nu m bness, tingling). If severe, it m ay lead to cau se m ental statu s changes, hyp otension, and laryngosp asm . (5:156, 520, 1121-2, 1246) 406. (C) Graves d isease is m arked by su ppression of TSH an d elevated con cen tration s of u n bou n d T4. Th yroid fu n ction tests are review ed 3-4 w eeks after starting treatm ent w ith antithyroid d ru gs, and the d ose is titrated based u p on u nbou nd T4 concentrations. Most p atients d o not achieve a euthyroid state u ntil treated for 6-8 w eeks. TSH concentrations rem ain su p p ressed for several m onths and therefore d o not provid e a sensitive ind ex of treatm ent response. (5: 2925-6)

320

17: Ane s the s ia for Mis ce lla ne ous P roce dure s

407. (B) Thyroid storm is an acute life-threatening form of hyp erthyroid ism . Sym p tom s inclu d e nau sea, vom iting, d iarrhea, agitation, confu sion, d iap horesis, tem p eratu re greater than 38.5°C, tachycard ia out of proportion to temperature, and arrhythmias. Multiple events may precipitate it, includ ing acute illness (infection, CVA, trauma), surgery, and radioiod ine treatm ent of patients w ith partially or u ntreated hyperthyroid ism. Initial management includ es large doses of methimazole that block hormone synthesis and partially inhibit peripheral conversion of T4 to T3; glucocorticoid s that block conversion of T4 to T3 in the periphery and act as supportive therapy of possible ad renal insufficiency; β-adrenoceptor antagonists preferably propranolol (as it alone d ecreases conversion), to block card iovascular effects; cholestyramine that bind s thyroid horm one in the gu t and enhances GI clearance; su p p ortive m easu res (antip yretics, cooling, volu m e repletion); and management of precipitating illness. Salicylates are contraind icated as antip yretics as they d ecrease bind ing of thyroxine to thyroxine binding globulin, thereby aggravating the disease. (1:1135, 1147-51; 5:115, 1118, 2927; Connery LE, Coursin DB, A nesthesiol Clin N orth A mer 2004; 22:526-8) 408. (A) Reperfu sion is the period of greatest hemod ynam ic instability d u ring liver transp lantation. Release of preservative solu tion, clot, air, d ebris, and acid em ic blood (released from reperfused splanchnic circulation) into the pulm onary vascu latu re m ay cau se severe pulm onary hyp ertension and resu ltant right heart failure. The resultant increase in central venous p ressu re (CVP) m ay op en an occu lt p atent foram en ovale, w ith risk of p arad oxical em bolu s. Elevated CVP m ay also com prom ise graft rep erfu sion and contribu te to early graft failu re. Pu lm onary hyp ertension and right heart failu re w ill resu lt in a d ecrease in system ic pressu re and coronary p erfu sion p ressure d ue to inad equate left ventricular preload . Althou gh p u lm onary vascu lar resistance w ill often increase, system ic vascu lar resistance

w ill d ecrease. (5:1075-6; Steadman, RH, A nesthesiol Clin N orth A mer 2004; 22:687-711) 409. (C) Seru m sod iu m is often d ecreased in these p atients second ary to the form ation of ascites, renal failu re, and the u se of d iu retics. Baseline p otassium concentrations m ay vary, d epend ing u p on the u se of p otassiu m -sp aring or p otassium -w asting d iu retics to control ascites, as w ell as p atient’s u nd erlying renal fu nction. Desp ite that, hyp erkalem ia is com m on intraoperatively d u e to the natu re of the proced u re itself. Sp lanchnic ischem ia, the transfu sion of large volu m es of blood p rod u cts, m etabolic acid osis d u e to hyp otension from d ep letion of circu lating volu m e, and the preservative solu tion that is rich in potassium , all contribute to a significant rise in serum p otassiu m . Seru m ionized calcium tend s to d ecrease d ue to vigorous ad m inistration of citrated blood prod u cts. Citrate is a chelating agent that bind s to calciu m . H yp oglycem ia m ay be seen w hen the liver, the p rim ary site of glu coneogenesis, is rem oved d u ring the anhep atic p hase, p articu larly w hen the p hase is p rolonged . Exogenou s sou rces of glu cose tend to m itigate this possibility, how ever. Dextrose containing IV flu id s (d ru g-carrier infu sions), as w ell as the u se of m ethylpred nisolone, often result in hyperglycem ia. (1:853, 5:1073-5) 410. (D) Cisatracurium, rocuronium, and vecuronium d o not d ep end on the kid ney for elim ination, have m inim al hem od ynam ic effect, and have been u sed su ccessfu lly in p atients w ith m arginal or no renal fu nction. Becau se cisatracu riu m is m etabolized in the p lasm a by Michael elim in ation , (often er ron eou sly term ed H ofm ann elim ination), its d u ration of action is not p rolonged in renal failu re. Althou gh the liver is th e p rim ary m etabolic site for rocu roniu m and vecu roniu m , the d u ration of blockad e m ay be p rolonged w ith these d ru gs if large d oses are u sed , d u e to accu m u lation of m etabolites that are excreted by the kid ney. Pancu roniu m shou ld not be u sed in kid ney transplant recipients becau se it d ep end s p rim arily on the kid ney for elim ination, causing

Answe rs : 407–415

a p rolonged neu rom u scu lar block. N either d esflu rane nor isoflu rane have nep hrotoxic p rop erties. N 2O m ay also be u sed , as it has m inim al sid e effects, no renal toxicity, and rap id elim ination. Sevoflu rane is rarely u sed for renal transp lantation d u e to concerns of flu orid e and com p ou nd A toxicity. Althou gh m ost hu m an stu d ies have not d em onstrated d eleteriou s effects of sevoflu ran e on th e kid ney, it has been d eem ed p referable to u se one of the know n safe alternatives. The pharm acokinetics and pharm acod ynam ics of fentanyl, su fentanil, alfentanil, and rem ifentanil are not significantly altered by kid ney d isease, and have been successfu lly u sed d u ring renal transplantation (1:543-7; 5:1098) 411. (C) The most prevalent gastrointestinal d isease com p licating card iovascu lar su rgery is isch em ic colitis; the incid ence in p atients requ iring em ergent aortic rep air trip les from that of elective aortic rep air, abou t 5-9%. Mesenteric ischem ia is typically seen in p atients over 70 years old . It is categorized accord ing to etiology: non-occlu sive m esenteric ischem ia, arterioocclusive m esenteric ischem ia, or m esenteric venous throm bosis. N on-occlu sive ischem ia is d u e to m esenteric arteriolar vasosp asm in resp onse to a severe p hysiologic stress su ch as d ehyd ration or hyp otension. The hallm ark of m esenteric ischem ia is severe acu te, nonrem itting abd om inal pain com pletely out of proportion to p h ysical fin d in gs, u n like oth er intrabd om inal p athologies that often p rovid es signs and sym ptom s includ ing abd om inal d istention and / or peritoneal signs. (5:288, 2510-1) 412. (D ) Managem ent of p atients w ith p resu m ed m esenteric ischem ia includ es flu id resuscitation to cou nteract flu id sequ estration w ithin bow el w all, optim ization of oxygen d elivery, broad -sp ectru m antibiotics, and su p p ort of card iac ou tp u t w ith in otrop ic agen ts. Vasoconstrictors shou ld be avoid ed ; instead , d obu tam ine, a p u re β -ad renocep tor agonist, is the preferred agent. (5:2512) 413. (D ) Irrigation flu id s w ith glycine are often u sed in TURP. The am ino acid is an inhibitory

321

neu rotransm itter in the cortex and retina, and is most probably the causative agent of postoperative blind ness and seizures in som e TURP patients. Patients exhibit pupillary reflexes that are often slu ggish or absent, u nlike cortical blind ness, in w hich reflexes are m aintained . This su ggests that the m echanism of blind ness in these p atients is d irect inhibition of retinal potential transmission. Blind ness resolves w ith d ecreasing blood concentrations of glycine. Blind ness is bilateral, as op p osed to TIA, in w hich visu al d istu rbances are m ost com m only u nilateral. Elevated concentrations of am m onia, a byp rod uct of glycine m etabolism , cause sym ptom s of lethargy, m uscle w eakness, and encep halopathy. The absorption of sorbitol can resu lt in the d evelop m ent of hyp erglycem ia and lactic acid osis related to the m etabolism of sorbitol (5:895, 1140, 2526) 414. (A) Su ccinylcholine d epolarizes m u scle cells, causing an efflux of K+ throu gh acetylcholine recep tors. It w ill cau se a transient rise in [K+ ] in n orm al in d ivid u als of 0.3-0.5 m Eq/ L. Succinylcholine may cause exaggerated hyperkalemic response in patients w ith cond itions of m uscle m em brane d egeneration (e.g., trau m a, bu rns, p rim ary m u scle d isord ers) or neu ral d enervation (e.g., stroke, m u ltip le sclerosis, Guillain-Barre synd rom e, spinal cord injuries). Patients w ith renal failu re m ay have a higher baseline seru m [K+ ] d u e to im p aired p otassium excretion, bu t the rise in [K+ ] is no d ifferent. (5:273, 501, 517) 415. (D ) Causes of coagulopathy in patients w ith ad vanced liver d isease includ e: (1) Vitam in K deficiency; (2) impaired synthesis of coagulation factors synthesized in liver (all factors except von Willebrand Factor VIII); and (3) splenic sequestration of platelets second ary to hypersplenism. These patients may also have decreased synthesis of coagulation inhibitors, and may not be able to clear activated coagulation factors or fibrin split products. Coagulopathy may therefore be treated with a combination of vitamin K, FFP, cryoprecipitate, and platelets. (5:193-4, 215)

322

17: Ane s the s ia for Mis ce lla ne ous P roce dure s

416. (A) Patients w ith end -stage liver d isease have m u ltip le hem od ynam ic ch anges, inclu d ing d ecreased SVR, increased card iac ou tp ut, and d ecreased β -ad renoceptor function w ith resultant d ecrease in resp onse to catecholam ines. Patients w ith severe liver d isease h ave an increased volu m e of d istribu tion second ary to an expand ed extracellu lar volum e space. As a resu lt, clearances of m orp hine and alfentanil are red u ced in cirrhosis, w hereas fentanyl and su fentanil are not greatly affected . Because cirrhosis of the liver m arked ly d ecreases total hep atic blood flow as a resu lt of fibrosis at the p ortal triad , p atients w ith cirrhosis can be expected to have increased sensitivity to highly extracted d ru gs. H ep atic blood flow d ecreases d u ring regional and general anesthesia second ary to d irect and ind irect effects of anesthetic agents, ventilatory m od e, and typ e of p roced u re. Protein bind ing is d ecreased second ary to a d ecrease in albu m in that increases the u nbou nd fraction of ionized d ru gs. (1:24; 5:186, 720, 1051) 417. (E) Shock w aves have the p otential to trigger ventricu lar arrhythm ias w hen they coincid e w ith the rep olarization p eriod of the card iac cycle. For this reason, ECG synchronization and shock d elivery should be d u ring the ventricu lar refractory p eriod , 20 m sec after the R w ave. Shock w aves from either gated (syn ch ron ized ) or u n gated (u n syn ch ron ized ) ESWL m ay inhibit or rep rogram card iac p acem akers. Synchronization, or lack thereof, has no correlation w ith in tensity of w ave, or am ou nt of analgesia requ ired . (5:1141-2) 418. (C) The Dornier H M3 lithotrip tor is an old er electrohyd rau lic u nit, requ iring a w ater bath and generating high intensity w aves requ iring d eep er analgesia. Im m ersion in a w ater bath initially resu lts in transient vasod ilation and hyp otension, follow ed by rise in arterial blood p ressu re as venou s blood red istribu tes cen trally from the hyd rostatic pressure of w ater on the low er extremities and abd omen. As SVR rises, card iac ou tp u t m ay d ecrease d u e to hyd rostatic pressu re on the low er extrem ities and th e abd om en . Th e increase in venou s retu rn m ay shift blood to the intrathoracic

vessels, precipitating congestive heart failu re in susceptible ind ivid uals. FRC is d ecreased by 30-60%, p red isp osing som e p atients to hypoxem ia. Water bath im m ersion has no d irect effect on heart rate. (5:1141-2) 419. (C) Du ring resection of the p rostate, venou s sinuses are opened, and large amounts of irrigation fluid can be absorbed into the system ic circu lation, cau sing TURP synd rom e. The synd rome has multiple manifestations characterized by flu id overload , hyp oosm olality, hyp onatrem ia, and neu rologic d istu rbances, with onset as early as 15 min into the procedure. Sym p tom s m ay inclu d e nau sea, confu sion, hypertension, reflex brad ycard ia from volume overload , ECG changes from myocard ial ischemia, desaturation, seizure, visual changes, and coma. When TURP synd rome is suspected , the proced ure should be stopped ; serum sod ium, potassium, and osmolality should be measured, as shou ld hem oglobin, as a m easu re of flu id absorp tion . Presu m ed p u lm onary ed em a should be treated w ith d iuresis. H yponatremia d oes not need to be treated aggressively w hen it is not accompanied by hypoosmolality, or in the absence of neurologic symptoms. (5:1139-41) 420. (B) Rapid correction of hyponatremia may trigger d em yelination of p ontine or extrap ontine neu rons, lead ing to neu rologic d ysfu nction that m ay inclu d e quad riplegia, pseu d obulbar p alsy, seizu res, com a, and even d eath. For this reason, seru m sod iu m is increased slow ly, and hypertonic saline shou ld be u sed only in the p resen ce of life-threaten in g m an ifestation s su ch as com a and seizu res. Otherw ise, sod iu m concentrations can be increased by ad m inistration of norm al saline in com bination w ith a loop d iu retic or m annitol. Sod iu m correction shou ld never exceed 1-1.5 m Eq/ L/ h. N a + d eficit = TBW × (d esired [N a + ] − cu rren t [N a + ]). Total-bod y w ater of infants you nger than 1 year of age is 75-80% of bod y w eight, w hereas that of ad u lt m ales is 60% and that of females is 55%. Deficit = 100 × 0.6 × (130− 119) = 60 × 11 = 660 m Eq. It is com m on to correct only half the sod ium d eficit. (1:31; 5:515, 1139-41)

Answe rs : 416–425

421. (E) Differen tial d iagn osis of h yp oten sion d u ring a lap aroscop ic p roced u re inclu d es carbon d ioxid e em bolu s, hem orrhage, com p ression of th e vena cava from increased intraabd om inal p ressu re, and p neu m othorax. Relatively sm all CO 2 em boli are not u ncom m on and have been d etected by TEE, w hile massive CO 2 embolus is a rare and catastrophic event. H em orrhage w ou ld not increase PIP, w hile com p ression of the vena cava w ou ld be m ore likely d u ring active insu fflation of the abd om en w ith d evelop m ent of a significant p neu m op eritoneu m . Over tim e, how ever, a p neu m othorax m ay d evelop, w ith associated hypoxem ia, hyp otension, and elevated airw ay p ressu res. The incid ence of extrap eritoneal insu fflation of CO 2 has been rep orted to range from as little as 0.4%-2% to 20%-64%, and is m ore likely d u ring lengthy proced u res or proced ures on the low er esophagu s. (5:1041-2) 422. (D ) Wh en a p n eu m oth orax is su sp ected , aggressive investigation (au scu ltation, chest rad iograp h) and m anagem ent (e.g., chest tu be for tension p neu m othorax or conversion to an op en p roced u re) sh ou ld be u n d ertaken . (5:1041-2) 423. (C) If the laser beam p enetrates the end otracheal tube, the oxygen rich environment w ithin the tu be can cause an intense flam e. Utilizing a tu beless techniqu e (sp ontaneou s ventilation, ap neic oxygenation, or jet ventilation) w ill d im inish the risk by d ecreasing the concentrated oxygen rich environm ent w hile rem oving a p otentially flam m able object from the airw ay. Com bu stion of PVC yield s hyd rogen chlorid e gas that is acid ic, highly toxic, and irritating. Althou gh an end otracheal tu be m ay be w rap p ed w ith metallic tap e to d ecrease the likelihood that it w ill catch fire, there is no w ay to w rap the tube cuff w ith m etallic tape. Filling the cu ff w ith tinted saline w ill alert one to a cuff ru pture, bu t w ill not prevent fire. N itrous oxid e w ill su pport com bu stion. (5:1236-9) 424. (C) α-ad renoceptor blockad e d ose is increased every 3-4 d u ntil eith er no sym p tom s of

323

catecholamine excess are evid ent or the patient com plains of sid e effects from postural hypotension and / or a stu ffy nose, w ith a final d osage range of 40 to 100 mg/ d. β-ad renoceptor agon ists are ad m inistered for a p ersistent tachycard ia and for control of other peripheral β -ad renergic effects of catecholam ine excess. These d ru gs shou ld never be given before α -ad ren ocep tor blockad e becau se seriou s hyp ertensive sequ elae m ay resu lt. Ad equ ate volum e expansion after institu tion of the d ru g m ay take as long as 2 to 3 w eeks. Thu s p atients p resenting for su rgery w ho h ave been on p henoxybenzam ine for a shorter tim e period shou ld have hypovolem ia corrected preoperatively. (5:158, 1124) 425. (D ) The m ain com p onent of cem ent is m ethylm ethacrylate (MMA) that has been linked to a clinical scenario consisting of hyp otension, bronchoconstriction, hyp oxia, card iac arrest, and su d d en d eath. It w as at one tim e believed to be d u e to a hyp ersensitivity reaction to the MMA, resu lting in acu te vasod ilatation and card iac collapse. H ow ever, plasm a concentrations in vivo d uring u se of MMA have been fou nd to be 10-20-fold below the concentrations requ ired to cau se clinically significant vasod ilatation and hyp otension. The actu al cause is increased intram ed u llary canal p ressu re w ith resultant em bolization of fat particles and d ebris into the m ed u llary venou s p lexu s d u ring long bone m anip u lation, ream ing, and cem enting. Intram ed u llary p ressu re p eaks are as high as 680 m m H g in hu m ans w ith cement use, compared to peaks below 100 mm Hg with the use of non-cemented implants. Clinical signs are sim ilar to those fou nd in PE or fat em bolism : fever, tachycard ia, hypotension , h yp oxem ia, an d , in sp on tan eou sly breathing p atients, d ysp nea, and tachyp nea. Other signs of fat em boli also m ay be seen on ECG, inclu d ing right-axis d eviation or right bu nd le branch block, reflecting increased p u lm onary artery p ressu re and intrap u lm onary shunt, p otentially lead ing to right ventricu lar failu re and card iac arrest. (5:1204)

324

17: Ane s the s ia for Mis ce lla ne ous P roce dure s

426. (B) In choosing the anesthetic techniqu e for cases that m ay be associated w ith com p artm ent synd rom e, the anesthesiologist m u st avoid any p ostop erative techniqu e, su ch as ep id u rals or p erip heral nerve blocks th at w ou ld d elay d iagnosis of the synd rom e. If these techniques m ust be used in the setting of p otential com p artm ent synd rom e, consid eration should be given to continuously monitorin g com p artm en t p ressu res. Th e effect of techniques su ch as spinals and epid urals that cau se sym p athectom y (thus vasod ilatation) is u nclear. The increased blood flow to com partm ents m ay cau se a fu rther increase in com partm ent pressu res. (1:576-7, 579-80; 5:1205) 427. (A) The com p lications of tou rniqu et u se are m an y. In flation of th e tou rn iqu et cau ses increases in p reload and afterload ; d eflation cau ses d ecreases in p reload an d afterload ; rep erfu sion cau ses a d ecrease in core tem p erature of u p to 1°C, an increases in O 2 consu m p tion, and increases in the p artial p ressu re of CO 2 w ith resu ltant increase in cerebral blood flow, as w ell as m etabolic and resp iratory acid osis. Lim b ischem ia can cause ischem ic capillaries lead ing to d iffu se cap illary leak u p on rep erfu sion; tissu e p ressu re lead s to p latelet aggregation and cap illary obstru ction, lead ing to release of inflam m atory m ed iators that cause m icrovascu lar throm bosis. Tissu e acid osis causes release of tissue plasm inogen activator, cau sing a brief p eriod of fibrinolysis. Pressu re from the tou rniqu et can cau se significant nerve inju ry, esp ecially in the u pp er extrem ity. Rad ial nerve p alsy is th e m ost com m on nerve inju ry. (5:1202-3) 428. (E) Su ch p atients have p rofou nd p erip heral vasoconstriction ind uced by the thermoregulatory resp onses in an effort to conserve heat. The activation of arteriovenou s shu nts in the periphery d ecreases the effectiveness of app lying heat to the extrem ities and the su rface of the bod y in w arming the patient. Cold-ind uced hyp ertension is accom panied by an increase in circulating catecholam ines that au gm ent card iac irritability and lead to ventricular arrhythmias. Platelet function is impaired in even mild d egrees of p er iop er ativ e h yp oth er m ia,

p otentially related to a d efect in the release of throm boxane A 2. Som e im p airm ent of clotting factor fu nction is also noted . (5:1505-7, 1510) 429. (E) Repair of an open globe is a su rgical proced u re th at is n ot con sid ered am en able to regional anesthesia. The goals of general anesthesia for ophthalmic surgery includ e a smooth ind u ction w ith a stable intraocu lar p ressu re, avoid ance or treatm ent of the ocu locard iac reflex, m aintenance of a m otionless field , a sm ooth em ergence, and avoid ance of p ostop erative nau sea an d vom iting. This can be accom plished in p art by red u cing the need for opioid analgesia throu gh the u se of a bu lbar block in conju nction w ith general anesthesia. Efforts to qu ickly obtain good intu bating cond itions and prevention of coughing and straining are im p ortant for red u cing the risk of extru sion of eye contents, th ereby m akin g aw ake fiberop tic in tu bation a su bop tim al choice. Protection of the airw ay, especially in a p atient w ith a fu ll stom ach, lim its the u se of an LMA. (5:1222) 430. (D ) Althou gh historic concerns exist abou t the safety of su ccinylcholine in the presence of an open globe, several retrospective stud ies show no link. Althou gh su ccinylcholine d oes p rod u ce a tem porary increase (7-10 m in) in intraocular pressure (6-12 mm H g), it is far less than that p rod uced by cou ghing, straining (40 m m H g), or d irect p ressu re on the eye. Efforts to qu ickly obtain good intu bating cond itions and prevent coughing and straining are much more im p ortant for red u cing the risk of extru sion of eye contents than the avoid ance of su ccinylcholine. If the intu bation is ju d ged to be d ifficult but the eye is viable, su ccinylcholine (after p retreatm ent w ith a sm all d ose of a nond epolarizing m u scle relaxant,) is an ap p rop riate op tion. If the p atient is likely to be an easy intubation, a short- to intermed iate-acting nond ep olarizing agent, ad m inistered in greaterthan-norm al d oses, su ch as 1 to 1.2 m g/ kg of rocu roniu m , has p roven effective. The u se of p rop ofol has been show n to im prove intubating cond itions if rocu roniu m is u sed . IV lid ocaine and opioid s m ay also red u ce the risk of straining and coughing. (5:122-3)

Answe rs : 426–436

431. (D ) Anesthesiologists shou ld obtain inform ed refusal w hen patients refu se recommend ations or request a relevantly suboptim al techniqu e. The concep t u nd erlying inform ed refu sal is that these patients need to be m ore extensively informed about risks, benefits, and alternatives w h en th ey d esire in ad visable tech n iqu es. Anesthesiologists are not ethically obligated to p rovid e care for these patients in nonem ergent situ ations, although they m ay w ish to assist in find ing a w illing colleagu e. (5:46) 432. (B) Every PACU shou ld have d efinite gu id elines for d ischarge criteria that shou ld accou nt for the p reop erative statu s of the p atient and th e exp ected p ostop er ativ e m or bid ity. Generalized gu id elines for PACU d ischarge inclu d e baseline m ental statu s, stable vital signs w ithin accep table lim its, and achievem ent of certain criteria, p ossibly throu gh a clinical scorin g system . Cu rren t p ractice, end orsed by the ASA, d oes not requ ire d elay of d ischarge until void ing; the patient’s risk for failu re to void m u st be assessed in stead . Patients at high risk for u rinary retention second ary to neu raxial blockad e, colorectal or genitou rinary su rgery, or m ed ical history, m ay be kep t u ntil void ing, or if blad d er catheterization is necessary. (5:1253-4, 1256) 433. (A) N itrou s oxid e is a very insolu ble gas in blood , but is 117 times more soluble than sulfur hexaflu orid e. N 2O enters the intraocu lar gas bu bble m ore rap id ly that SF6 can exit. If N 2O ad m inistration continu es after injection of SF6 gas into the vitreal cavity, the bu bble can exp and u p to three tim es its volu m e, increasing intraocu lar p ressu re. If N 2O is then d iscontinu ed , the IOP can d rop qu ickly, thereby causing re-d etachm ent of the retina. Therefore N 2O shou ld be d iscontinu ed at least 20 m in before the injection of gas. (5:1223)

325

434. (C) Abd om inal com partm ent synd rom e often cau ses d ecreased card iac outpu t and increases in both PCWP and CVP. These changes are nonspecific and m ay be d u e to other cau ses. In traabd om in al p ressu re is m on itored by instilling 50 m L of saline into the blad d er, then m easu ring the pressu re. Pressu res >20 m m H g in com bination w ith organ failu re are consistent w ith a d iagnosis of abd om inal com partm ent synd rom e, althou gh pressu res as low as 12 m m H g have been associated w ith system ic inflam m atory resp onse synd rom e and m u ltiorgan failu re. Although increasing abd om inal d istension is also often present, serial measurem ents of abd om inal girth d o not provid e an ind ication of intraabd ominal pressure. (5:1363) 435. (D ) The p atient w ith obstru ctive sleep ap nea is at increased risk for opioid -ind uced apnea. The ability to m anage the p atient’s p ain w ithou t op ioid s m ay p erm it the p atient to have safe am bu latory su rgery. A p eriop erative n erve block d oes n ot necessarily increase safety becau se the p atient m ight requ ire op ioid s w hen the block regresses. A short surgical tim e d oes not necessarily m inim ize p ostop erative p ain. CPAP m ay p revent obstru ction-related ap neic ep isod es, bu t it d oes not necessarily affect the m agnitu d e of op ioid -ind u ced ventilatory d ep ression in these p atients. (5:710, 1257-8) 436. (B) Topical ad m inistration of an anticholinergic like atrop ine into the eye can raise intraocu lar p ressu re (IOP) in a p atien t w ith glaucoma, particu larly d angerou s in those persons p red isposed to the relatively rare narrow angle glau com a. System ic ad m inistration of atrop ine to these p atients m ay p recip itate a first attack in u nrecognized cases. It d oes not, how ever, resu lt in intraocu lar concentrations of atropine ad equ ate to cause m yd riasis in a p atient w ith op en-angled glau com a, p articu larly w hen treated ap p rop riately. (1:228-9)

326

17: Ane s the s ia for Mis ce lla ne ous P roce dure s

437. (D ) The d iagnosis of CO poisoning is m ad e by m easu ring the carboxyhem oglobin concentration in arterial blood , expressed as a percentage satu ration of hem oglobin. Pu lse oxim etry cannot d istinguish betw een these tw o forms of hem oglobin as the absorbance sp ectru m of both are sim ilar, lead ing to norm al oxim etry read ings in the p resence of very high am ou nts of carboxyhemoglobin in blood . The Pa O 2 in an arterial blood gas sample measures the amount of oxygen d issolved in blood and d oes not ind icate th e qu antity of oxygen bou nd to hem oglobin (satu ration), lead ing to a norm al Pa O 2 even w ith high concentrations of carboxyhem oglobin. The oxygen satu ration is a calcu lated valu e only. (5:1336-7) 438. (C) Vital capacity is significantly and progressively com p rom ised by increases in thoracic rigid ity and loss of ventilatory m u scle p ow er. Loss of lu ng elastic recoil is the p rim ary anatomic mechanism by w hich aging d egrad es the efficiency of pu lm onary gas exchange. Becau se the changes in elasticity are nonu niform , they d isrupt the normal matching of ventilation and p erfu sion w ithin the lu ngs, increasing both shu nting and p hysiologic d ead sp ace. Sm all airw ay patency, norm ally m aintained by elastic recoil, is compromised , and closing capacity in creases. Resid u al lu n g volu m e in creases becau se intrinsic lu ng elastic recoil is p rogressively red u ced . (5:277-82) 439. (A) Em ergency treatm ent is aim ed at qu ickly stabilizing the m yocard iu m by raisin g th e action potential threshold and red u cing excitability w ithout changing the resting membrane potential. This can be accom p lished by ad m inistering 0.5-1 g of calciu m chlorid e. Althou gh it p rotects the heart from the effects of elevated seru m potassiu m , intravenous calcium has no effect on the seru m potassiu m concentration. H yperventilation, as w ell as the ad ministration of sod iu m bicarbonate, nebulized albu terol, or insu lin and glu cose in com bination, w ill all aid in d ecreasing the serum p otassium concentration by d riving p otassiu m ion intracellu larly. In creasin g the availability of insu lin en h an ces the activity of the N a,K-ATPase p u m p in skeletal m u scle, as d oes the u se of

β 2-ad renoceptor agonists. Raising the system ic p H w ith sod iu m bicarbonate resu lts in hyd rogen ion release from cells that is accom p anied by p otassiu m ion m ovem ent into cells. The p otassiu m -low ering action of sod iu m bicarbonate is m ost p rom inent in p atients w ith m etabolic acid osis. (5:518, 6:357-9) 440. (D ) Coagu lation d istu rbances are com m on in p atients w ith chronic liver d isease. The p rim ary reasons inclu d e a d ecreased prod u ction of coagulation factors prod uced in the liver (all excep t factor VIII); and a congestive sp lenom egaly, or h yp ersp lenism , resu lting from p ortal hypertension. Throm bocytopenia is d ue to sp lenic sequ estration of p latelets that can resu lt in p latelet concentration of 70,000/ m m 3 or less. The d evelop m ent of DIC, a red u ction in fibrinogen, and d ysfibrinogenem ia second ary to im paired fibrin p olym erization, are all com m only seen in ad vanced liver d isease. (5:193; 6:980-1) 441. (D ) Regional anesthesia m ay offer significant ad vantages in p atients w ith Parkinson d isease by allow ing an earlier retu rn to oral intake, eliminating the u se of neuromu scular blocking d ru gs and the risks of general anesthesia. Bu tyrop henones su ch as d rop erid ol can low er the seizure threshold and cause extrapyram id al sym ptom s. Withd raw al of Parkinson d isease m ed ications in the p eriop erative period can cause significant w orsening of their respiratory symp toms caused by u pper airw ay d ysfu nction d u e to d isease involvem ent of the intrinsic laryngeal muscles. Morphine has been rep orted to red u ce d yskinesia at low d oses bu t to ind uce akinesia at greater d oses. (5:701, 876; N icholson G, et al., Br J A naesth 2002; 89: 904-16) 442. (B) Tran sfu sion -related acu te lu n g in ju ry (TRALI) is the lead ing cau se of transfu sionrelated m ortality. Estim ated TRALI incid ence is once for every 3,000 to 70,000 plasm a-rich com p onents transfu sed and 1 per 50,000 u nits of low plasma volume com ponents transfused . H u m an error accou nts for the second highest in cid en ce of ad verse tran sfu sion effects, w h ereas th e tran sfu sion -associated viral infectiou s risk is qu ite low, w ith the risk of

Answe rs : 437–447

hep atitis C being 1:1,935,000 and that of the h u m an im m u n od eficien cy v iru s bein g 1:2,135,000. Anap hylaxis and hem olytic transfu sion reactions are both p otentially fatal com p lications. (5:149, 203-4) 443. (A) Etom id ate inhibits the enzym e resp onsible for p erform ing the 11β -hyd roxylation reaction in cortisol synthesis. A single ind u ction d ose of 0.3 m g/ kg inhibits cortisol synthesis and the norm al resp onse to ad renocorticotrop ic horm one for u p to 12 h. Because of the long d u ration of ad renal su p p ression associated w ith etom id ate infu sions, continu ou s ad m inistration has not been as extensively stud ied . There rem ains conflict abou t the issu e of the red u ction of cortisol after ind uction d oses of etom id ate, bu t it has been show n that infu sions of several d ays’ d uration in ventilated intensive care u n it p atien ts w ere associated w ith increased m ortality. Given that the d u ration of su ppression of cortisol synthesis by etom id ate is d ep end ent on the cu m u lative d ose, u se of the agent in rep eated proced u ral sed ation in a critically ill patient is of concern. (5:687, 696-7) 444. (D) Hepatopulmonary syndrome is defined by the presence of hepatic d ysfunction or portal hyp ertension, an elevated alveolar-arterial oxygen gradient, and intrapulmonary vasodilatation. Unlike portopulmonary hypertension, w hich is associated w ith pulmonary vascular vasoconstriction, hepatopulmonary synd rome resu lts from vasod ilatation and p u lm onary vascular remodeling. Patients may present with d igital clu bbing, sp id er angiom ata, arterial hyp oxemia, and d yspnea that w orsens u pon moving from a recumbent to an upright position (orthod eoxia and platypnea). Intrapulmonary vasodilation may reflect true anatomical shunt, physiologic shunt, and precapillary or capillary d ilation, leading to alterations in oxygen diffusion. N itric oxid e is elevated in the exhaled breath of patients with hepatopulmonary synd rome. (5:194, 1051-5; 6:2601) 445. (D ) Decreasing lean tissu e mass in the eld erly red u ces the capacity for bod y heat prod u ction, and impairm ent of thermoregu latory vasoconstriction p laces them at increased risk for

327

in ad verten t in traop erative h yp oth erm ia. Intraoperative core temperatu re d ecreases at a rate tw ice as great as that observed in you ng ad u lts u nd er comp arable cond itions, and the tim e need ed for sp ontaneou s p ostop erative rew arm ing increases in d irect p rop ortion to patient age. There exists an age-related increase in pharmacod ynamic sensitivity to anesthetic agents, yet the cau se remains u nknow n. With aging, relative m inim u m alveolar concentration values for the new er inhalational agents d ecline by approximately 30%. Decreases in the eld erly brain neu rotransm itter reserves are manifested by an increased sensitivity to d ru gs that m ight p recipitate extrap yram id al sym p tom s or anticholinergic synd rom e, inclu d ing m etoclop ram id e. Althou gh the eld erly have red u ced skeletal m u scle m ass, d issem inated neurogenic atrophy at the neuromuscular junction allow s proliferation of extrajunctional cholinorecep tors. The m ed ian effective d ose and stead y-state plasma concentration required for half-m axim al neu rom u scu lar blocking effect (med ian effective concentration) rem ain virtually u nchan ged , or m ay actu ally in crease slightly, in the eld erly patient. Onset of blockad e may be d elayed , and d u ration prolonged , how ever. Renal plasm a flow, glom eru lar filtration rate (GFR), and creatinine clearance all d ecline significantly. (1:1326, 5:279-83) 446. (A) Inju ry to the recu rrent laryngeal nerve (RLN ) p revents abd u ction of the ip silateral vocal cord that becom es fixed in a p aram ed ian p osition becau se of the u nop posed action of the cricothyroid m uscle. It com m only results in hoarseness. Bilateral nerve inju ry cou ld resu lt in ap p osition of the vocal cord s and insp iratory obstru ction w ith associated strid or and ap honia. Laryngeal ed em a w ou ld be seen w ith venou s and lym p hatic obstru ction second ary to a hem atom a, not inju ry to the RLN . (5:1006, 1268, 1286) 447. (E) N itrop ru ssid e is rap id ly m etabolized to cyanid e and thiocyanate, and accu m u lation of these m etabolites can lead to cyanid e or thiocyanate toxicity d u ring prolonged ad m inistration, especially in patients w ith renal failure. (5:1403)

328

17: Ane s the s ia for Mis ce lla ne ous P roce dure s

448. (B) Anesthetic im p lications of u ntreated hyp othyroid ism inclu d e an im p aired ventilatory resp onse to both hyp oxia and hyp ercarbia. Atrial d ysrhythm ias, inclu d ing atrial fibrillation, are seen in hyperthyroidism. Conventional anesthetic w isd om hold s that thyroid statu s d oes not alter the m inim um alveolar concentration. Data from anim al stud ies u sing old er volatile anesthetics sup ports that assu m ption, bu t there are no d ata relative to the new er p otent inhalation agents. Sim ilarly, conventional w isd om hold s that hyp othyroid ism increases the sensitivity to sed ative, analgesic, and anesthetic m ed ications, bu t evid ence is lim ited to old er stu d ies d u e to the lack of new er stud ies on current agents. H ypertension, p articu larly d iastolic hyp erten sion, is also com m only noted in hypothyroid ism . (5:1117) 449. (B) A d irect pharm acod ynam ic effect of hyp otherm ia is its im p act on m inim u m alveolar concentration (MAC). There is an ap p roxim ately 5% red u ction in MAC per d egree d rop in core bod y tem p eratu re. (5:1510, 1653) 450. (B) Scop olam ine is a m u scarin ic recep tor antagonist w ith greater p erm eation across the blood brain barrier. In therap eu tic d oses it cau ses CN S d epression m anifested as d row siness and am nesia. The am nestic effects of mid azolam (w hen given in the u su al d oses for prem ed ication) are variable bu t usually short lived , and they shou ld not be relied on to p rev en t recall of in tr a op er a tiv e ev en ts. Dexm ed etom id ine p rod u ces intense sed ation, although it d oes not reliably p rod uce am nesia, hyp nosis, or general anesthesia. N either d roperid ol nor mu scle relaxants prod u ce am nesia. (1:227-30; 5:610, 698, 701, 1374) 451. (C) The BIS is a proprietary algorithm (Aspect Med ical System s, N ew ton, MA) that generates a linear d im ensionless nu m ber ranging from 0 to 100 that d ecreases in proportion to increased anesthetic d ep th. Evid ence show s that it m ay be effective at red u cing the incid ence of intraoperative aw areness w hen inhaled anesthetics are u sed to p rod u ce hyp nosis. Confou nd ing factors, w hich can alter BIS, inclu d e electrical artifact from electrocau tery d evices, p hysio-

logic alterations su ch as hyp oglycem ia, low voltage EEG cau sed by genetic variation or d ru gs, n eu rologic abn orm alities su ch as Alzheimer d isease, and EMG (mu scle). Finally, the valu e of BIS m onitoring has been valid ated only for p rop ofol and volatile anesthetics. When ketam ine and N 2O are present, the relationship betw een BIS valu e and p ercep tive aw areness can be significantly altered (5:610) 452. (D ) Jam es T. Reason is often cited for his concep tu al thinking abou t system failu re and hu man error. The process of accid ent evolution is w id ely referred to as the ”Sw iss cheese” m od el. The ”Sw iss cheese” m od el illu strates that accid ents are typically the result of a series of events that includ e precursors that trigger or allow the chain of events that resu lt in the final (active) ad verse event. Reason term ed these p recu rsors latent errors. Latent errors have the p oten tial for initiatin g or p rop agating an evolving accid ent. Exam p les are failu re to m aintain equ ipment or replace obsolete equip m ent, selection of low qu ality su p p ly item s, p oor sched u ling p ractices that p rom ote haste or fatigu e, and case sched u ling and staffing m od els that allow assignm ent of relatively inexp erienced clinicians to u nfam iliar cases or high-risk p atients. Latent errors rarely lead to an im m ed iate accid ent, bu t are seen as a ”lu rking enem y,” aw aiting the circu m stances that w ill com bine to p rod u ce an u nexp ected catastrop hic ou tcom e, often in w ays that are u nu sual and u npred ictable. (5:18) 453. (C) As d efined by the Joint Com m ission, a sentin el even t is ”an u nexp ected occu rrence involving d eath or seriou s p hysical or p sychological injury, or the risk thereof. Serious injury specifically includ es loss of lim b or function.” JCAH O sentinel events that potentially involve anesthesia care inclu d e events that resu lt in an u nanticipated d eath or m ajor perm anent loss of fu nction and are u nrelated to the natu ral cou rse of the p atient’s illness or u nd erlying cond ition, su rgery on the w rong p atient or bod y p art regard less of m agnitu d e of p roced u re, u nintend ed retention of a foreign bod y after su rgery or other p roced u re, hem olytic transfusion reactions becau se of m ajor blood

Answe rs : 448–456

grou p incom p atibilities, an d u nanticip ated d eath of a fu ll-term infant. (5:352-2) 454. (E) Althou gh there are three form s of anthrax (gastrointestinal, cu taneou s, and inhalational), it is the inhalational form that is m ost likely to be the cau se of d eath in a bioterrorist attack. The early sym p tom s inclu d e a viral p rod rom e w ith fever, fatigu e, nau sea and vom iting, and shortness of breath. CXR reveals characteristic find ings of m ed iastinal w id ening and p leu ral effu sions, as w ell as infiltrates. Sym ptom s tend to appear w ithin 4-6 d of exposure. Patients are not contagiou s, and therefore d o not need to be isolated . Th erap y m u st be started qu ickly, w ith p enicillin, d oxycycline, and cip rofloxacin p roven to be effective. Patients w ith active d isease are often started on the above agents, in com bination w ith either clind am ycin or rifam p in. (6:1769-72) 455. (C) Alm ost 50% of the U.S. pop u lation is su sceptible to the sm allpox viru s, as im m unization p rogram s w ere stop p ed in the United States in 1972. Primary and second ary viremias occur w ithin the first few d ays after exposu re. Tw elve to fou rteen d ays after first exp osu re, p atients d evelop high fever, vom iting, head ache, and backache. A m acu lop ap u lar rash form s, beginning on the face and extrem ities

329

and extend ing to the tru nk, w ith lesions in the sam e d evelop m ental stage w ithin the sam e location. The lesions evolve into vesicles and p u stu les. Becau se it is h igh ly in fectiou s, p atients m ust be kept in strict isolation u ntil the pustules resolve and become scabs. (6:1772, 1774-5) 456. (B) Althou gh botu linu m toxin is not a live m icroorganism , it is one of the m ost p otent toxins ever d escribed , w ith estim ates that 1 g of toxin w ou ld be enou gh to kill 1 m illion p eop le if ap p rop riately d isp ersed . On ce absorbed into the blood stream , it bind s to the neuronal cell m em brane, enters the cell, and cleaves one of the p roteins requ ired for intracellu lar bind ing of the synaptic vesicle to the cell m em brane, thu s p reventing release of the n eu rotransm itter to th e ad jacent cell. Patients d evelop cranial nerve palsies follow ed by a d escen d in g flaccid p aralysis. Most p atients d evelop d ip lop ia, d ry mou th, d ysphagia and d ysarthria, and extrem ity w eakness. Recovery requ ires regeneration of new m otor n eu ron synap ses th at m ay take w eeks to m onths. Treatm ent is m ainly supportive, but if d iagnosed early enou gh, ad m inistration of a hep tavalent equ ine antitoxin m ay d im inish symptoms and d ecrease severity of the d isease. (6:1776-8)

This page intentionally left blank

CHAPTER 18

Critic al Care Me dic ine Que s tions DIRECTION S (Qu estions 457-530): Each of the num bered item s or incom plete statem ents in this section is follow ed by answ ers or by com p letions of the statem ent. Select the ON E lettered answ er or com pletion that is BEST in each case. 457. A p reviou sly healthy 18-year-old m ale p resents to the em ergency d ep artm ent w ith three d ays of d ysp nea and facial sw elling. H e also reports a cough and hoarseness. H is symptoms w orsen w ith lyin g d ow n . H is chest x-ray reveals a w id ened m ed iastinu m . Which one of the follow ing is the m ost likely d iagnosis? (A) (B) (C) (D) (E)

Angioed em a Ep iglottitis Lym p hom a Pneu m onia Tracheom alacia

458. An 81-year-old m ale has been in the hosp ital for treatm ent of a u rinary tract infection and d eliriu m . Du ring the night he got ou t of bed u nassisted and sustained a fall. A su bsequ ent evalu ation revealed a C3 vertebral fractu re. H e w ent to the op erating room for rep air of his fractu re. Postop eratively he w as kep t intu bated for concerns abou t airw ay ed em a. The p lan is to reassess his ability to be extubated in 24 h. In the interim , he requ ires sed ation for safety. H is vital signs are T 37.1°C, H R 58, BP 122/ 78, Sp O 2 100%. Which choice of sed ative agent is m ost ap prop riate in this situ ation? (A) Dexm ed etom id ine (B) Etom id ate

(C) Mid azolam (D) Prop ofol 459. A 66-year-old m ale is in the intensive care unit recovering from a ru p tu red abd om inal aortic aneu rysm rep air. H is p ostop erative cou rse has been com p licated by acu te kid ney inju ry and an N STEMI. H e rem ain s in tu bated an d m echanically ventilated . Fou r d ays into his cou rse, his respiratory function w orsens, and h e is d iagn osed w ith ventilator-associated pneumonia. Sputum culture grow s Haemophilus influenza. Ad m inistration of w hich one of the follow ing m ed ications m ight have p revented this p neu m onia? (A) (B) (C) (D)

Oral chlorhexid ine System ic clind am ycin System ic d exam ethasone System ic om ep razole

460. Which one of the follow ing statem ents abou t organ d onation after card iac d eath (DCD) is m ost accu rate? (A) Requ ires d eclaration of brain d eath by a neu rologist (B) Cau ses longer w arm ischem ic tim e than d onation after brain d eath (C) Does not requ ire p atient/ fam ily consent (D) Does not involve extu bation of the p atient (E) Cannot involve ICU p hysician in end -of-life care of the p atient

331

332

18: Critica l Ca re Me dicine

DIRECTION S: Use the follow ing figu re to answ er Qu estion 461:

461. For the ventilatory mode d epicted in the figure, w hich statem ent is correct? (A) The end of insp iration is d eterm ined by d ecrease to a set fraction of p eak inspiratory flow. (B) Minu te ventilation is ind ep end ent of p atient effort. (C) Pressu re is the d ep end ent variable. (D) A bronchop leu ral fistu la w ill not affect insp iratory tim e. (E) Mand atory and sp ontaneou s breaths are d epicted . DIRECTION S: Use the follow ing scenario to answ er Qu estions 462-463: A 78-year-old m an is ad m itted to the ICU follow ing open repair of an abd om inal aortic aneu rysm . H e has a history of coronary artery d isease treated w ith a d ru g-elu ting stent four years ago. H is m ed ications includ e asp irin, m etoprolol,

p ravastatin, and lisinop ril. A p reop erative p harm acologic rad ionuclid e m yocard ial stress test revealed no evid ence of p erfu sion d efects. H is vital signs on arrival to the ICU are T 36.2°C, H R 68, BP 100/ 60, RR 12, oxygen saturation 99% on facemask oxygen. Pain score is 3/ 10. One hour later, his heart rate falls to 30 and blood pressure to 65/ 38; he is awake, alert, and has no com p laints. 462. Which one of the follow ing is the m ost ap prop riate first intervention? (A) (B) (C) (D)

Transvenou s p acing End otracheal intu bation Card iop u lm onary resu scitation Atrop ine

463. A few m inu tes later, the blood p ressu re is 180/ 110. An electrocard iogram is obtained and is show n in the figu re below :

Que s tions : 461–465

I

a VR

V1

II

a VL

V2

III

a VF

V3

333

V4

V5

V6

V1

Ap propriate interventions shou ld inclu d e (A) stat echocard iogram , d obu tam ine, blood transfu sion (B) throm bolysis, p u lm onary artery catheter, d obutam ine (C) esm olol, nitroglycerin, transfer to card iac catheterization su ite (D) stat CT angiogram , esm olol, card iac su rgery consu lt (E) nitrop ru ssid e, intravenou s hep arin, transfer to card iac catheterization suite DIRECTION S: Use the follow ing scenario to answ er Qu estions 464-469: A 77-year-old m ale is being treated for ascend ing cholangitis resulting in gram negative bacterem ia. H e is intubated , sed ated , and m echanically ventilated . A pulm onary artery catheter is placed five hou rs after p resentation to assist w ith managem ent of his hem od ynam ics and resu scitation. H is vital signs are T 38.7°C, H R 91, BP 76/ 43 and Sp O 2 90%. CVP is 4 mm Hg. CO is 5 L/ min.

His ABG is pH 7.29, Pa CO 2 37, Pa O 2 61. His hemoglobin is 9 g/ d L. 464. Which one of the follow ing w ould prod u ce the greatest increase in oxygen d elivery? (A) An increase in arterial oxygen satu ration to 100% (B) An increase in heart rate to 110 beats/ min (C) An increase in hem oglobin to 10 g/ d L (D) An increase in stroke volu m e to 80 m L/ beat 465. The cross sectional area of w hich one of the follow ing is the p rim ary d eterm inant of system ic vascu lar resistance (SVR)? (A) (B) (C) (D) (E)

Arteries Arterioles Cap illaries Venu les Veins

334

18: Critica l Ca re Me dicine

466. Which one of the follow ing correctly d escribes arteriolar an d vascu lar sm ooth m u scle recep tors? (A) α 1- and β 2-ad renocep tors m ed iate vasoconstriction (B) α 1-ad renocep tors m ed iate vasoconstriction w hile β 2-ad renocep tors m ed iate vasod ilation (C) α 1-ad renocep tors m ed iate vasod ilation w hile β 2-ad renocep tors m ed iate vasoconstriction (D) α 1- and β 2-ad renocep tors m ed iate vasod ilation 467. This p atient w ou ld be exp ected to have w hich one of the follow ing? (A) (B) (C) (D) (E)

Decreased angiotensin form ation Decreased renin release Decreased vasop ressin levels Increased ald osterone release Increased sod iu m excretion

468. Which one of the follow ing provid es an ind ex of total bod y tissu e perfusion? (A) (B) (C) (D)

Cerebral oxim etry Central venou s oxygen satu ration Mixed venou s oxygen satu ration Pu lse oximetry

469. This p atient w ou ld be exp ected to have w hich one of the follow ing m etabolic abnorm alities? (A) A d ecrease in hep atic lip ogenesis (B) A d ecrease in seru m glu cose concentrations (C) A d ecrease in seru m triglycerid e concentrations (D) An increase in p ancreatic insu lin release (E) An increase in p rotein catabolism 470. A 47-year-old male is being treated for strep tococcal pneum onia in the ICU. H e is intu bated and sedated for respiratory failure, and has had persistent fevers and an elevated w hite blood

cou nt. Diagnostic thoracentesis for a p arap neu m onic effu sion revealed that the effusion w as an em p yem a. As a resu lt, he is sched u led to have a therapeu tic thoracentesis in the interventional rad iology su ite. The p atient has been on volum e control ventilation w ith the follow ing settings: TV 450, RR 20, PEEP 14, FIO 2 0.6. Prior to transp ort to the interventional rad iology suite, you are called to the bed sid e. The p atient has had w orsening oxygenation w ith d ecreased oxygen saturation w hen he is turned or placed flat. H e has requ ired FIO 2 1.0 several tim es in the last several hou rs. Chest x-ray show s increasing size of the pleu ral effusion. Which one of the follow ing is the most app rop riate therap y? (A) Arrange for op en d rainage of the effusion in the op erating room (B) Arrange for p ercu taneou s d rainage of the effu sion at the bed sid e (C) Continu e w ith the sched u led proced u re (D) Delay thoracentesis u ntil the patient is m ore stable 471. A 52-year-old m ale com p lains of nau sea and a head ache at w ork. H e su bsequ ently has an acu te d ecline in w akefu lness. H is cow orkers bring him to the em ergency d ep artm ent. On evalu ation, he op ens his eyes to painfu l stim u li, makes incomprehensible sou nd s and localizes to p ain. Select the p atient’s GCS from the list below. (A) (B) (C) (D) (E)

6 7 8 9 10

472. A 47-year-old fem ale w as ad m itted to the ICU 18 h ago w ith p an creatitis an d SIRS. Sh e requ ired intu bation for increased w ork of breathing and large volu m e IV flu id resu scitation for hyp ovolem ia. H er clinical p ictu re has w orsened over the last several hou rs. She has p oor u rine ou tp u t, is hyp otensive and has

Que s tions : 466–475

d ecreased m inu te ventilation d u e to high airw ay p ressu res. A chest x-ray show s m ild bilateral p u lm onary ed em a. Which one of the follow in g is th e m ost likely cau se of th e p atient’s sym ptom s? (A) (B) (C) (D) (E)

Abd om inal com p artm ent synd rom e Card iom yop athy H yp ovolem ia Infection Pu lm onary em bolism

473. A 23-year-old fem ale p resents to the labor triage u nit w ith p rem atu re ru p tu re of m embranes at 34 w eeks and 6 d ays gestational age. Sh e rep orts that h er p regnancy has been u ncom p licated except for a DVT d iagnosed in her first trim ester. She is cu rrently taking p renatal vitam ins and enoxaparin (1 m g/ kg BID). H er vital signs are T 37.1°C, H R 71, BP 112/ 68, RR 22, Sp O 2 98%. A CBC, p rothrom bin tim e, partial throm bop lastin tim e, and com p rehensive m etabolic p anel d one as part of her evaluation are u nrem arkable excep t for a p latelet cou nt of 57,000/ m m 3. Fetal heart tones are reassu ring. What is the m ost likely cause of her laboratory abnorm ality? (A) Vitam in B12 d eficiency (B) Dissem inated intravascu lar coagu lation (DIC) (C) H ep arin ind u ced throm bocytop enia (H IT) (D) H ELLP synd rom e (E) Placental abru p tion DIRECTION S: Use the follow ing scenario to answ er Qu estions 474-478: A 41-year-old m ale m otorcyclist is stru ck by a car. H e is u nconsciou s at the scene and intu bated en rou te. On arrival to the em ergency d ep artm ent trau m a bay, his systolic blood p ressu re is 60 m m H g. Bilateral chest tu bes are p laced w ithou t an im provem ent in his hem od ynam ics. A chest x-ray, pelvic x-ray, long bone film s and a FAST exam

335

are rap id ly obtained . The exam s are p ositive for m u ltip le left sid ed rib fractu res, p u lm onary contu sions, an open-book pelvic fracture, a left fem u r fractu re, and flu id in the hep atorenal fossa. Transfu sion of tw o u nits O-negative packed red blood cells is initiated , and the p atient is brou ght em ergently to the operating room . A laparotom y is p erform ed and the patient is fou nd to have p rofu se bleed ing from a ru p tu red liver as w ell as an exp and ing retrop eritoneal hem atom a. Over the cou rse of the su rgery, the p atient receives 25 u nits of PRBCs, 15 u nits of FFP, and 4 single d onor u nits of p latelets. H is cu rrent vital signs are T 35.7°C, BP 81/ 42, H R 127, Sp O 2 95% on FIO 2 0.7. Laboratory valu es are rem arkable for H b 8.9 g/ d L, platelets 103,000/ m m 3, IN R 1.7, fibrinogen 95 m g/ d L, ABG p H 7.23, Pa CO 2 43 m m H g, Pa O 2 64 m m H g, K 4.4 m Eq/ L, creatinine 1.6 m g/ d L, and ionized calciu m 0.81 m m ol/ L. A TEE-p robe is inserted . There is no p ericard ial effu sion. Both ventricles appear w ell filled w ith norm al size, bu t the systolic fu nction of both ventricles ap pears red u ced . 474. Which one of the follow ing is m ost likely to im p rove the p atient’s hem od ynam ic p ictu re? (A) (B) (C) (D) (E)

Ad m inistration of bicarbonate Ad m inistration of calciu m Increased PEEP Transfu sion of p acked red blood cells Warming the patient to > 36°C

475. The p atient begins to bleed from his IV line sites and his nose. Which one of the follow ing w ou ld be most likely to im p rove the p atient’s clotting ability? (A) (B) (C) (D) (E)

Ad m inistration of cryop recip itate Ad m inistration of d esm op ressin Ad m inistration of fresh frozen p lasm a Ad m inistration of p latelets Warming the p atient to 37°C

336

18: Critica l Ca re Me dicine

476. The p atient u nd ergoes d am age control lap arotom y and em bolization of bleed ing p elvic vessels. H e is brou ght to the ICU for further stabilization. Over the next 6 h, the patient’s vasop ressor requ irem ent im p roves, and his tran sfu sion requ irem en t resolves. H e is w eaned from 32 to 7 m cg/ m in of norep inep hrine. Du ring that tim e fram e, how ever, he d evelops oligu ria. H is laboratory d ata show s: N a 143 m Eq/ L, K 5.9 m Eq/ L, BUN 20 m g/ d L, Cr 1.8 m g/ d L, u rine osm olality 332 m Osm / L H 2O, urine sod iu m 43 m Eq/ L, urine creatinine 18 m g/ m L. Urinalysis is negative for leu kocytes or red blood cells. What is the m ost likely cause of this patient’s oligu ria? (A) (B) (C) (D) (E)

Acu te tu bu lar necrosis Glom eru lonep hritis Interstitial nep hritis Prerenal azotem ia Urinary obstru ction

477. Th ere are p eaked T w aves on th e ECG. Intravenou s calciu m is ad m inistered . What is the role of calciu m in the treatm ent of hyperkalem ia? Calciu m (A) antagonizes gastrointestinal absorp tion of potassium (B) enhances renal excretion of p otassiu m (C) facilitates red istribu tion of p otassiu m into cells (D) increases card iac m yocyte excitability (E) raises the card iac action p otential threshold 478. Tw o d ays later the p atient returns to the operating room for fixation of a tibial fractu re. Du ring the p roced u re, he is fou nd to have compartm ent synd rom e of his low er leg, and a fasciotom y is p erform ed . Postop eratively his CPK is 9,562 IU/ L. Which of the follow ing laboratory abnorm alities are m ost likely? (A) Elevated calciu m , d ecreased p otassiu m , d ecreased phosp horus (B) Elevated calciu m , increased p otassiu m , d ecreased phosp horus (C) Decreased calciu m , d ecreased p otassiu m , increased p hosp horu s

(D) Decreased calcium, increased potassium, increased p hosp horu s (E) Decreased calcium, increased potassium, d ecreased phosphorus 479. A 26-year-old fem ale p resents to her oncologist’s office w ith w orsening d ysp nea and chest p ain. She is on w arfarin for a history of DVT d iagnosed six m onths ago, and she is being treated for breast cancer w ith chem otherapy. She is otherw ise healthy. Physical exam reveals T 37.1°C, H R 122, BP 80/ 40, and p erip heral ed em a. She is ad m itted urgently to the hospital. H er IN R is 3.2. ECG show s sinu s tachycard ia w ith low QRS voltage and no ST changes. Which one of the follow ing is m ost likely to lead to a d iagnosis? (A) (B) (C) (D) (E)

Card iac enzym es Chest x-ray D -d im er Echocard iograp hy Urinalysis

480. A 35-year-old , p reviou sly healthy fem ale is brou ght to the em ergency d ep artm ent after a seizure. She has no m em ory of the event. She currently has no complaints. H er vital signs are T 37°C, H R 82, BP 126/ 75, RR 17, Sp O 2 98%. H er neu rologic exam ination is norm al, inclu d ing a GCS of 15. H er optic d isks are sharp. Skin exam ination reveals a su spiciou s lesion on her right thigh. CBC and comprehensive metabolic p anel (CMP) are u n rem arkable. H ead CT show s a frontal m ass consistent w ith m etastasis. What is the m ost appropriate next step in m anagem ent? (A) (B) (C) (D) (E)

CSF d iversion Dexam ethasone Intu bation and hyp erventilation Mannitol Decom p ressive craniectom y

481. Which one of the follow ing measures is recomm end ed d u ring central venous catheter insertion, based on evid ence supporting a red uction in the rate of central line-associated blood stream infection?

Que s tions : 476–482

(A) Sterile head to toe d rap ing of p atient, chlorhexid ine skin preparation, subclavian site (B) Fu ll barrier p recau tions, hand w ashing prior to line insertion, p rophylactic antibiotic ad m inistration (C) Sterile head to toe d rap ing of p atient, sterile rew ire of catheter every 7 d , hand w ashing p rior to line insertion (D) Chlorhexid ine skin p rep aration, rem oval of unnecessary catheters, avoid ance of internal ju gu lar site (E) H and w ashing p rior to line insertion, head to abd om en d rap ing of patient, rem oval of su bclavian site catheters after 7 d 482. A 4-year-old boy w ith history of p rem atu re birth, asthm a, and d evelopmental d elay is hosp italized in the PICU w ith resp iratory failu re d u e to resp iratory syncytial viru s (RSV). H e has been intu bated , sed ated , and m echanically ventilated for 6 d . Vital signs are T 37.5°C, H R 92, BP 80/ 40. CVP is 6 m m H g. H e is ventilated in assist control p ressu re control m od e at the follow ing settings: FIO 2 0.7, PC 25 cm H 2O, PEEP 10, RR 30. On exam , he is u nresp onsive, has soft bilateral rales, and his u rine is d ark. ABG show s pH 7.18, Pa CO 2 56 m m H g, Pa O 2 65 m m H g. Laboratory values are N a 146 m Eq/ L, K 5.4 m Eq/ L, Cl 110 m Eq/ L, BUN 28, Cr 1.9, CPK 6500 IU/ L. Chest rad iograph show s d iffu se lobar infiltrates and low lu ng volu m es. Which is the m ost ap p rop riate next step in m anagem ent? (A) Transition from p ressu re control to p ressu re supp ort ventilation (B) Perform bed sid e echocard iogram (C) Initiate neu rom u scu lar blockad e (D) Discontinu e p rop ofol (E) Begin continu ou s venovenou s hem ofiltration DIRECTION S: Use the follow ing scenario to answ er Qu estions 483-486: A 59-year-old m an is ad m itted to the ICU follow ing a hepatic resection com plicated

337

by hem orrhage. Su rgery w as aborted , the abd om en p acked w ith laparotom y sp onges, and he w as transp orted intu bated , sed ated , and p harm acologically p aralyzed to the ICU. Vital signs are T 34.5°C, H R 127, BP 80/ 45, RR 20, Sp O 2 90% on FIO 2 0.8. H e is oligu ric w ith cool extrem ities; abd om inal exam is significant for increasing d istention. H e is placed on m echanical ventilation in the m od e d epicted in the follow ing figu re:

338

18: Critica l Ca re Me dicine

483. Follow ing tw o hou rs of continu ou s resu scitation w ith blood prod ucts, w hich one of the follow ing ventilatory alarm s is m ost likely? (A) Increased p eak airw ay p ressu re over set lim it (B) Ap nea > 30 sec (C) Low m inu te ventilation (D) H igh tid al volu m e (E) Excessive insp iratory tim e 484. An arterial blood gas d raw n on this p atient show s pH 7.18, Pa CO 2 55 m m H g, Pa O 2 60 m m H g, base d eficit 9. The p atient’s d isord er is best d escribed as (A) resp iratory acid osis (B) mixed respiratory acidosis and metabolic alkalosis (C) m etabolic acid osis (D) mixed respiratory acidosis and metabolic acid osis 485. The figu re below d ep icts the p atient’s arterial w aveform .

486. Based on the findings from the patient’s arterial waveform, the next management should be (A) (B) (C) (D) (E)

inotrop e ad m inistration vasop ressor ad m inistration p ericard iocentesis p u lm onary artery catheterization flu id ad m inistration

487. A 44-year-old female presents to the emergency department with one hour of severe headache and mental status changes. Initial CT scan shows subarachnoid hemorrhage (SAH) that is likely d ue to aneurysmal rupture. H er initial Fisher Grade is 2. She is intubated in the emergency department due to concerns that she is unable to effectively manage her secretions. A subsequent repeat head CT shows that she has progressed to Fisher Grade 3. Which one of the following is the most likely mechanism for this change? (A) (B) (C) (D)

Aneu rysm al rebleed ing Elevated intracranial p ressu re Seizu res Vasosp asm

488. Synchronized interm ittent m and atory ventilation (SIMV) is associated w ith w hich beneficial effect as com p ared to controlled ventilation?

The follow ing equ ation is d erived from the m easu rem ents d epicted in the figure: Δ1 − Δ 2 Δ1 + Δ2

× 100%

2 The valu e d eterm ined by the equ ation is: (A) (B) (C) (D) (E)

stroke volu me variation p u lse p ressure variation d elta d ow n resp iratory variation systolic p ressu re variation

(A) (B) (C) (D)

Resting of the d iap hragm Decreased w ork of breathing Im p roved V/ Q m atching Absence of p atient-ventilator d yssynchrony (E) Increased m ean intrathoracic p ressu re 489. A 41-year-old woman underwent resection of an acoustic neuroma under total intravenous anesthesia (TIVA). The surgical procedure was notable for 12-h duration with 500 mL blood loss. She is admitted to the ICU postoperatively, is extubated, breathing comfortably, and is neurologically intact. Vital signs are norm al. An ABG reveals pH 7.30, Pa CO 2 42 mm Hg, Pa O 2 150 mm Hg on supplemental oxygen, base d eficit 4. A m etabolic panel show s N a 143 m Eq/ L, K 3 mEq/ L, Cl 115 mEq/ L, HCO 3 20 mEq/ L. Which one of the following is the most likely explanation for the patient’s acid -base disturbance?

Que s tions : 483–492

(A) Crystalloid resu scitation flu id ad m inistered d u ring operation (B) Loop d iu retic ad m inistered to red u ce brain sw elling (C) TIVA anesthetic agent (D) H yp ovolem ia d u e to u nd erresu scitation (E) N itrop ru ssid e treatm ent of intraop erative hypertension 490. Com p ared w ith norep inep hrine in the treatm ent of septic shock, d opam ine (A) (B) (C) (D)

d im inishes m ortality d im inishes incid ence of renal failu re has greater p otency cau ses m ore tachyd ysrhythm ias

DIRECTION S: Use the follow ing scenario to answ er Qu estion 491: A 19-year-old previou sly healthy m an p resents after crashing his m oped into a stone w all. H e w as fou nd w ithou t a helm et, ejected from the m oped w ith GCS 4 (E1V1M2). Vital signs are T 35.8°C, H R 156, BP 79/ 45. H e is being m ask ventilated at rate 12 breaths p er m inu te; Sp O 2 99% w ith bag m ask ventilation at FIO 2 1.0. A noncontrast brain CT is obtained w ith find ings d ep icted below :

339

491. Follow ing end otracheal intu bation, w hich one of the follow ing should be employed for immed iate m anagem ent of this p atient? (A) Mechanical ventilation to Pa CO 2 26 m m H g, 23.4% saline bolu s, fu rosem id e bolu s, head of bed elevation to 30 d egrees (B) Mechanical ventilation to Pa CO 2 35 m m H g, ventricu lostom y, 23.4% saline bolu s, p henylep hrine infu sion to MAP > 80 mm Hg (C) Mechanical ventilation to Pa CO 2 18 m m H g, Trend elenberg p osition, em ergent neu rosurgical d ecom pression, volu m e resu scitation w ith 0.9% saline (D) Mechanical ventilation to Pa CO 2 35 m m H g, m ethylp red nisolone bolu s, volu m e resu scitation w ith 0.9% saline, em ergent neu rosurgical d ecom pression (E) Mechanical ventilation to Pa CO 2 35 m m H g, m annitol bolu s, volu m e resu scitation w ith 5% albu m in, em ergent neurosu rgical d ecom p ression DIRECTIONS: Use the follow ing scenario to answ er Questions 492-494: A 74-year-old male presents to the emergency d epartment from his nursing home w ith new onset confusion. H is vital signs are T 38.5°C, H R 94, BP 92/ 61, RR 24, Sp O 2 96%. H is w hite blood count is 14,000/ mm 3. Blood, urine and CSF cultures are obtained . A urinary catheter is placed yield ing scant d ark urine. Gram stain of the urine specimen reveals many gram-negative rod s. After volume resuscitation, the patient’s heart rate and blood pressure are 87 and 104/ 65, respectively. A central venous line is placed , and the CVP is 12 mm H g. Urine output remains low at 15 mL/ h for 2 h. 492. Th is p atien t w ou ld m ost accu rately be d escribed as having: (A) sep ticem ia (B) system ic inflam m atory resp onse synd rom e (SIRS) (C) sep sis (D) severe sep sis (E) sep tic shock

340

18: Critica l Ca re Me dicine

493. Which one of the follow ing w ould be the m ost app rop riate next step in m anagem ent? (A) (B) (C) (D) (E)

Flu sh the u rinary catheter Perform a head CT Give a bolu s of IV flu id s Perform a renal u ltrasou nd Start antibiotics

494. Which one of the follow ing resuscitation flu id s has been associated w ith an increased incid ence of acu te renal failu re in critically ill patients? (A) (B) (C) (D) (E)

Albu m in H yd roxyethyl starch N orm al saline Fresh frozen p lasm a Lactated Ringer solu tion

DIRECTION S: Use the follow ing scenario to answ er Qu estions 495-498: A 44-year-old m an w ith BMI = 40 kg/ m 2 is now on p ostoperative d ay 1 follow ing elective ventral hernia rep air. The rapid resp onse team is called to assess him for possible ICU transfer. H is nurse reports that 3 h prior, he w as sleep y, easily arou sable, hem od ynam ically stable bu t com p laining of 7/ 10 incisional p ain that w as treated w ith intravenou s m orp hine d elivered by PCA. One hou r ago, he becam e d isoriented and agitated , pu lling ou t his intravenou s access and u rinary catheter. At that time, lorazepam 5 m g IM w as ad m inistered and new intravenou s access established . N ow he is very som nolent. Vital signs show T 37.6°C, H R 92, BP 98/ 60, RR 7, Sp O 2 85% on su p plem ental oxygen via nonrebreather m ask. 495. Which one of the follow ing is the m ost likely find ing on p reop erative p u lm onary fu nction tests (PFTs) in this patient? (A) Increased resid u al volu m e (RV) (B) Decreased d iffu sing cap acity (DLCO) (C) Decreased forced exp iratory volu m e in first second / forced vital cap acity (FEV1/ FVC) ratio (D) Increased total lu ng cap acity (TLC) (E) Decreased exp iratory reserve volu m e (ERV)

496. The p atient’s w ife volu nteers a p reop erative history of snoring at night, d aytim e sleepiness, and a referral for a sleep stu d y that the patient has not yet u nd ergone. Which one of the follow ing com bination of cond itions is the m ost likely exp lanation for hyp oxem ia in the PACU in this p atient? (A) Atelectasis and anem ia (B) Intracard iac shu nt and increased oxygen consu m p tion (C) Atelectasis and hyp oventilation (D) H yp oventilation and hyp erm etabolism (E) Intrap u lm onary shu nt and d ecreased d iffusing cap acity 497. Which of the follow ing are most appropriate in initial m anagem ent of this p atient? (A) Ad m inister flu m azenil, p rovid e noninvasive positive pressu re ventilation (N IPPV), p osition head of bed at 45 d egrees (B) Position head of bed at 45 d egrees, p lace nasogastric tu be, d raw ABG (C) Ad m inister naloxone, p osition head of bed at 45 d egrees, d iscontinu e su p p lem ental oxygen (D) Ad m inister naloxone, p rovid e noninvasive positive pressure ventilation (N IPPV), p osition head of bed at 45 d egrees (E) Change to hyd rom orp hone analgesia, p erform end otracheal intu bation, and transfer to ICU 498. The p atient fails to im p rove from the initial m anagem ent strategy. H e is transferred to the ICU. Follow ing u neventfu l end otracheal intu bation u sing etom id ate, he becom es severely hypotensive, and u nresp onsive to fluid resuscitation. Vital signs are T 37°C, H R 110, BP 75/ 45, RR 18, Sp O 2 88% on FIO 2 1.0. ABG reveals p H 7.40, Pa CO 2 60 m m H g, Pa O 2 59 m m H g, base excess 14. CXR show s no evid ence of atelectasis or in filtrate. ECG reveals sinu s rhythm, R axis d eviation, and evid ence of right ventricu lar strain. Which one of the follow ing w ou ld be the m ost likely find ings from bed sid e echocard iogram ?

Que s tions : 493–502

(A) N orm al right ventricu lar size and fu nction, d ilated left atriu m , sm all and hypercontractile left ventricle (B) N orm al right ventricle size and fu nction, left ventricu lar ap ical ballooning (C) Dilated and hyp ertrop hied right ventricle, p arad oxical intraventricu lar septal m otion, sm all left ventricle (D) Large p ericard ial effu sion (E) N orm al right ventricu lar size and fu nction, d ilated , hypocontractile left ventricle w ith significant m itral regu rgitation DIRECTION S: Use the follow ing scenario to answ er Qu estions 499-501: A 58-year-old m an w ith COPD and d iabetes m ellitu s is ad m itted to the ICU w ith the d iagnosis of necrotizing fasciitis of the perineu m . H is vital signs are significant for T 39.1°C, H R 125, BP 79/ 52, RR 34, Sp O 2 88% on facem ask oxygen. 499. Which one of the follow ing is the explanation for a fall in m ean arterial p ressu re follow ing end otracheal intu bation and m echanical ventilation? (A) Catecholam ine release associated w ith d irect laryngoscop y (B) H istam ine release ind u ced by ketam ine ad m inistration (C) Red u ced venou s retu rn associated w ith p ositive pressure ventilation (D) Decreased w ork of breathing cau sing a w orsened lactic acid osis (E) Increased left ventricu lar afterload associated w ith p ositive pressu re ventilation 500. Mechanical ventilation of the patient is complicated by p atien t-ven tilator d yssyn chrony. What is the m ost likely cau se? (A) Rap id insp iratory flow rise tim e at the initiation of a breath (B) Decelerating flow p attern w ith volu m e targeted ventilation

341

(C) Excessive sed ation w ith p rop ofol (D) Au to p ositive end -exp iratory p ressu re (E) Ventilator insp iratory tim e m atched to p atient insp iratory tim e 501. Du ring p ressu re assist/ control m od e ventilation, the end otracheal tube becom es p artially obstru cted by secretions. Which one of the follow ing is m ost likely to resu lt? (A) (B) (C) (D) (E)

Increased tid al volu m e Increased p lateau p ressu re Ap nea Decreased p eak insp iratory p ressu re Decreased m inu te ventilation

DIRECTION S: Use the follow ing scenario to answ er Questions 502-503: A 65-year-old m an is ad m itted to the ICU follow ing percu taneous nep hrostom y tube p lacem ent to treat u reteral obstru ction. Im m ed iately follow ing the p roced u re, he d evelop ed fever and hyp otension. Vital signs u p on arrival to the ICU are T 39.2°C, H R 130, BP 80/ 40, RR 38. Oxygen saturation is u nobtainable. CVP is 3 m m H g. H e is aw ake and oriented and com plains of 5/ 10 p ain at the tu be site. H e has a Grad e II/ VI systolic ejection m u rm u r at the left sternal bord er, cool extrem ities and p oor p erip heral p u lses. Urine from the nep hrostom y tu be is cloud y yellow. Laboratory valu es show N a 138 m Eq/ L, K 4.6 m Eq/ L, Cl 108 m Eq/ L, BUN 32 mg/ dL, Cr 2.2 mg/ dL, WBC 22,000/ mm 3, Hb 13 g/ dL, p latelet cou nt 198,000/ m m 3. 502. Which one of the follow ing shou ld be p erform ed next? (A) Flu id resuscitate to CVP 15 m m H g (B) Transfu se 1 u nit packed red blood cells (C) Place p u lm onary artery catheter to m easu re Sv O 2 (D) Flu id resu scitate to CVP 10 m m H g (E) Op erative exp loration for bleed ing

342

18: Critica l Ca re Me dicine

503. Tw o hou rs later, the follow ing vital signs are record ed : T 38°C, H R 118, BP 105/ 60, RR 32. CVP is 12 m m H g, an d ScvO 2 is 55%. N orep inep hrine is infu sing at 10 m cg/ m in to su pport the blood pressure. Which one of the follow ing is the next m ost ap p rop riate step ? (A) (B) (C) (D) (E)

Ad m inister m ilrinone Increase norep inep hrine d ose Ad m inister d obu tam ine Ad m inister vasop ressin Ad m inister ep inep hrine

DIRECTION S: Use the follow ing scenario to answ er Qu estions 504-505: A 34-year-old w om an w ith system ic lu pu s erythem atosu s p resents w ith d ysp nea and chest pain. Vital signs are T 37.5°C, H R 128, BP 85/ 40, RR 36, Sp O 2 95% on nasal cannula oxygen. A bed sid e TTE is p erform ed , w ith a su bcostal im age d ep icted below :

505. Which one of the follow ing treatm ents is m ost ap p rop riate? (A) (B) (C) (D) (E)

Thoracentesis Throm bolysis Pericard iocentesis Diu resis Inotrop ic su p p ort

DIRECTION S: Use the follow ing scenario to answ er Qu estions 506-507: A 28-year-old w om an w ith a history of Rou x-en-Y gastric byp ass presents w ith nau sea, vom iting, and abd om inal p ain. Im aging su ggests a sm all bow el obstruction. On ind u ction of anesthesia for exp loratory lap arotom y, she has an observed asp iration of p articu late con ten ts. Follow ing rap id intu bation, her vital signs are T 37.2°C, H R 136, BP 80/ 40, Sp O 2 91% on m echanical ventilation w ith FIO 2 1.0. Physical exam reveals d iffu se exp iratory w heezing and rhonchi. A nasogastric tu be d rains copious feculent contents. 506. What is the next most appropriate intervention? (A) (B) (C) (D) (E)

Bronchoalveolar lavage Prone ventilation Chest CT Bronchoscop y w ith p articu late rem oval H yp erbaric oxygen

507. Which one of the follow ing is the m ost ap prop riate p harm acologic intervention?

504. Which one of the follow ing find ings is m ost likely on p hysical exam ination? (A) Right ventricu lar heave along left sternal bord er (B) Dim inished right sid ed breath sou nd s (C) Diastolic heart m u rm u r (D) Palp able liver ed ge (E) Pu lsus parad oxu s > 12 m m H g

(A) (B) (C) (D) (E)

Sod iu m bicarbonate Intravenou s hep arin Antibiotics Methylp red nisolone Inhaled nitric oxid e

DIRECTION S: Use the follow ing scenario to answ er Qu estions 508-513: You are respond ing to an overhead page rep orting a cod e in the PACU. On arrival, you find a 47-year-old m ale w ho has ju st u nd ergone a Whip ple p roced u re. H e had been d oing w ell, bu t as the nu rses w ere prep aring his transfer to the ICU for fu rther m onitoring, he su d d enly becam e

Que s tions : 503–513

u nresponsive. They called a cod e, initiated CPR, and began reconnecting him to the m onitors. You arrive as they are reconnecting the arterial line. 508. Critical m yocard ial blood flow is associated w ith an aortic d iastolic blood p ressu re greater than w hich one of the follow ing? (A) (B) (C) (D) (E)

15 m m 20 m m 30 m m 40 m m 50 m m

Hg Hg Hg Hg Hg

509. Which one of the follow ing d escribes the attributes of effective chest compressions in ad ults? (A) Rate tim e (B) Rate tim e (C) Rate tim e (D) Rate tim e (E) Rate tim e

80, 2 cm d ep th, 25% com p ression 80, 5 cm d ep th, 50% com p ression 100, 2 cm d ep th, 25% com p ression 100, 5 cm d ep th, 50% com pression 120, 2 cm d ep th, 25% com p ression

510. The p atient is p laced back on telem etry. The rhythm is ventricu lar fibrillation (VF). What is the most appropriate next step in management? (A) (B) (C) (D) (E)

Ad m inister am iod arone Ad m inister ep inep hrine Biphasic card ioversion at 150 J Biphasic d efibrillation at 200 J End otracheal intu bation

511. The resp iratory therap ist su ggests end -tid al CO 2 m onitoring. Du ring CPR, end -tid al CO 2 correlates w ith w hich one of the follow ing? (A) (B) (C) (D) (E)

Arterial CO 2 Card iac ou tp u t Minu te ventilation Arterial systolic blood p ressu re Venou s CO 2

343

512. Patients w ho are intubated and receiving CPR shou ld be ventilated at w hich one of the follow ing rates? (A) (B) (C) (D) (E)

< 4 breaths/ m in 4-6 breaths/ m in 6-8 breaths/ m in 8-10 breaths/ m in 10-12 breaths/ m in

513. If a p atient fails to regain consciou sness after return of spontaneous circulation follow ing VF arrest, w hich one of the follow ing is the m ost ap p rop riate next step in m anagem ent? (A) Ad m inister m annitol (B) Begin therap eu tic hyp otherm ia, if no contraind ications (C) N otify the fam ily of the p atient’s p oor p rognosis (D) Provid e 100% FIO 2 (E) Sched u le a card iac catheterization to be p erform ed after the p atient regains consciou sness DIRECTION S: Each grou p of item s below consists of lettered head ings follow ed by a list of num bered phrases or statem ents. For each nu m bered p hrase or statement, select the ON E lettered head ing or com ponent that is m ost closely associated w ith it. Each lettered head ing or com p onent m ay be selected once, m ore than once, or not at all. (A) Synd rom e of inap p rop riate antid iu retic horm one (SIADH ) (B) H ep atorenal synd rom e (C) Congestive heart failu re (D) Cerebral salt w asting (E) H yp ovolem ia (F) N ep hrogenic d iabetes insip id u s (G) Central d iabetes insip id u s For each of the follow ing clinical scenarios, select the m ost likely d iagnosis.

344

18: Critica l Ca re Me dicine

514. A 48-year-old w om an p resented 7 d ago w ith nausea, vom iting, and the w orst head ache of her life. She und erw ent a neurosu rgical proced u re, and is now obtu nd ed w ith a Glasgow Com a Score (GCS) of 4 (E1VTM2). H er vital signs are T 37°C, H R 99, BP 120/ 80, RR 14. CVP is 2 m m H g. Other relevant laboratory valu es in clu d e seru m sod iu m 134 m Eq/ L, u rin e sod iu m 43 m Eq/ L, p lasm a osm olality 270 mOsm/ kg, and urine osmolality 350 mOsm/ kg. 515. A 67-year-old m an is hospitalized in the ICU follow ing colon resection for p erforated sigm oid d iverticu litis. H e is in tu bated an d m echanically ventilated . H is vital signs are T 36.8°C, H R 88, BP 94/ 52, RR 16, Sp O 2 92%, FIO 2 0.6. Systolic p ressu re variation from a rad ial arterial line is 4 mm H g. H e has pitting perip heral ed em a. H e has seru m sod ium 133 m Eq/ L, u rine sod iu m 10 m Eq/ L, p lasm a osm olality 270 m Osm / kg, an d u rin e osm olality 600 m Osm / kg. 516. A 26-year-old man w ith a history of schizophrenia has su ffered traum atic brain inju ry after jumping from a height of 50 feet. H e has GCS 3(E1VTM1), no pupillary light reflexes, no corneal responses, and no gag or cough response. On apnea testing, arterial pH = 7.28 after 8 min w ith no observed resp iratory effort. Several hours later, urine output is > 400 mL/ h for 3 h, d im inishing to 50 m L/ h w hen intravenou s vasopressin is ad ministered . Laboratory values are serum sod ium 158 mEq/ L, urine sod ium > 20 mEq/ L, plasma osmolality 320 mOsm/ kg, and urine osmolality 100 mOsm/ kg. 517. A 56-year-old man with a history of non-alcoholic steatohepatitis (NASH) is hospitalized in the ICU following total colectomy to treat toxic megacolon. He is intubated and mechanically ventilated. His vital signs are T 37.5°C, HR 110, BP 90/ 52, RR 22, Sp O 2 100%, FIO 2 0.4. Pulse pressure variation from a radial arterial line is 18%. He has a serum sodium 130 mEq/ L, urine sodium 10 mEq/ L, plasma osmolality 290 mOsm/ kg, and urine osmolality 50 mOsm/ kg.

518. A 66-year-old w om an w ith a history of hep atitis C infection is transferred to the ICU on postop erative d ay 1 follow in g lap aroscop ic cholecystectom y. H er vital signs are T 37.2°C, H R 125, BP 85/ 42, RR 18, Sp O 2 94% on facem ask oxygen. Urine ou tp u t has been 5-20 m L/ h for the p ast 12 h. She has received 1.5 L norm al saline resu scitation over the p ast 4 h w ithout increase in urine output or blood pressu re. H er exam is notable for abd om inal d istention w ith shifting d u llness. Laboratory valu es are n otable for seru m sod iu m 128 m Eq/ L, u rine sod iu m 5 m Eq/ L, plasm a osm olality 285 m Osm / kg, and u rine osm olality 400 m Osm / kg. DIRECTION S: Each grou p of item s below consists of lettered head ings follow ed by a list of num bered p hrases or statem ents. For each nu m bered phrase or statem ent, select the ON E lettered head ing or com p onent that is m ost closely associated w ith it. Each lettered head ing or com p onent m ay be selected once, m ore than once, or not at all. (A) (B) (C) (D) (E) (F)

Anap hylactic shock Card iogenic shock H yp erd ynam ic sep tic shock H yp ovolem ic shock N eu rogenic shock Obstru ctive shock d u e to p u lm onary em bolism

For each p atient w ith shock, select the m ost likely m echanism cau sing the sym ptom s. 519. A 46-year-old male is brought to the emergency department by paramedics. He was found down in his home after sustaining a 45% total body surface area burn. He was unresponsive at the scene and was intubated en route. His vital signs are T 36.1°C, HR 126, BP 81/ 42, RR 21, Sp O 2 97%. A bedside hematocrit is 52%. A central line and an arterial line are placed to assist with care. His CVP is 6 mm Hg. Arterial w aveform analysis su ggests a d ecreased card iac ou tpu t. Central venous oxygen saturation is 47%.

Que s tions : 514–524

520. A 26-year-old fem ale is in the p ostp artu m u nit 12 h after d elivering a healthy 39-w eek m ale via elective rep eat cesarean section u nd er sp inal anesthesia. H er p ast m ed ical history inclu d es asthm a, GERD and chronic low back p ain . H er initial p ostop erative cou rse w as u neventfu l. She w as ambu latory and had good oral intake. Over the p ast several hou rs, how ever, she has become increasingly fatigu ed and lethargic. H er vital signs are T 37.1°C, H R 118, BP 79/ 51, RR 22, Sp O 2 94% on room air. H er extremities are cool and ed ematous. H er hemoglobin is 9 g/ d L. An intravenou s flu id bolu s d id not improve her vital signs. Central venous and arterial lines are p laced to assist w ith care. CVP is 15 m m H g. Arterial w aveform analysis su ggests a d ecreased card iac ou tp u t. Sh e begins to com p lain of d ysp nea that w orsens w ith lying flat. 521. A 62-year-old female is in a rehabilitation facility recovering from a right total hip rep lacement performed 3 w eeks ago. H er past med ical history is significant for obesity, hypertension, hyp ercholesterolem ia, h yp oth yroid ism and osteoarthritis. While w orking w ith the physical therapist, she d evelops d yspnea and chest p ain. Em ergency m ed ical p ersonnel arrive, find ing her heart rate 105 bp m and blood p ressu re 89/ 59 m m H g. She requ ires a nonrebreath er oxygen m ask to m ain tain h er oxygen satu ration > 90%. ECG d one en rou te to th e h osp ital sh ow s sin u s tach ycard ia w ithou t acu te ST changes. 522. A 22-year-old m ale w ith a history of m eningom yelocele is sched u led for correction of an Arnold -Chiari malformation. After an uneventfu l ind u ction of anesthesia, arterial and central catheters are placed . The patient is then positioned , prep ped and d rap ed . One hour after incision, the p atient d evelop s hyp otension, hyp oxem ia and increased airw ay p ressu res. The CVP is 1 m m H g. 523. A 56-year-old m ale is brou ght to the em ergen cy d ep artm en t by h is broth er d u e to

345

p rogressive som nolence over the p ast 2 d . The p atient is too lethargic to give a history bu t his brother reports that he avoid s m ed ical care. H e is know n to have a p enicillin allergy since child hood . H e had an ankle fracture repaired in the p ast. H e sm okes 2 p acks/ d and d rinks a p int of w hiskey d aily. H is vital signs are T 37.9°C, H R 113, BP 89/ 40, RR 26, Sp O 2 88% on 4 L/ min by nasal cannu la. There are rhonchi in the right lu ng. H is feet are w arm w ith bou nd ing p u lses. 524. A 16-year-old m ale is brou ght to the em ergency d ep artm ent by p aram ed ics. H e w as the u nrestrained d river in a high sp eed m otor vehicle accid ent. H e w as u nresponsive at the scene and requ ired intubation. On arrival, his vital signs are T 36.2°C, H R 61, BP 82/ 43, Sp O 2 100% w hile being ventilated w ith an Am bu bag. H is GCS is 4 (E2VTM1). H is initial chest x-ray and long bone film s d o not show any traum atic inju ries. H is FAST scan is negative. H is feet are w arm . DIRECTION S: Each grou p of item s below consists of lettered head ings follow ed by a list of nu m bered p hrases or statem ents. For each nu m bered phrase or statement, select the ON E lettered head ing or com p onent that is m ost closely associated w ith it. Each lettered head ing or com p onent m ay be selected once, m ore than once, or not at all. (A) (B) (C) (D) (E) (F) (G) (H ) (I) (J)

Su barachnoid hem orrhage H yp ercarbia H yp ertensive encep halop athy Mid d le cerebral artery infarction Sep tic shock Meningitis Alcohol intoxication Ep id u ral hem atom a Concu ssion Postseizu re state

For each p atient w ith com a, select the m ost likely d iagnosis.

346

18: Critica l Ca re Me dicine

525. A 79-year-old w oman w ith COPD and abd om inal p ain u nd erw ent exp loratory lap aroscop y u nd er spinal anesthesia. In the PACU, she is u nresp onsive w ith no focal or lateralizing signs. T 36.5°C, H R 110, BP 160/ 90, RR 6, Sp O 2 94% on sup p lem ental oxygen.

had run out of her med ications one week ago. T 35.2°C, HR 88 irregular, BP 170/ 90, RR 16, Sp O 2 95% on room air. Laboratory values are notable for sod iu m 145 m Eq/ L, glu cose 130 m g/ d L, BUN 40 m g/ d L, creatinine 1.8 m g/ d L, WBC 8,000/ mm 3, hemoglobin 12 g/ d L, INR 0.9.

526. A 22-year-old man presented to the em ergency d ep artm ent follow ing crashing his m otorcycle into a u tility p ole. When em ergency m ed ical p ersonnel resp on d ed , h e rep orted th at he struck his helm eted head into the pole w ithou t loss of consciou sness. Vital signs at the scene w ere T 37.2°C, H R 130, BP 160/ 98, RR 22, Sp O 2 98% on room air, GCS 15 (E4V5M6). On arrival to the hospital, he is com atose w ith a left tem poroparietal scalp laceration.

529. A 43-year-old w om an p resents w ith a history of su barachnoid hem orrhage 3 w eeks p rior d u e to ru ptu red cerebral aneurysm . She w as treated w ith coil em bolization of the aneurysm an d su bsequ en tly d evelop ed obstru ctive hyd rocephalus that has been treated since w ith an external ventricu lar d rain. She w as neu rologically intact and recovering w ell u ntil 8 h ago w hen her nu rse noticed increasing d row siness, p rogressing to obtu nd ation and com a. Vital signs are T 39.2°C, H R 120, BP 110/ 70, RR 30, Sp O 2 100% on room air. She has no focal or lateralizing neu rologic find ings. Laboratory values show sod ium 145 mEq/ L, WBC 27,000/ m m 3, hem oglobin 9.2 g/ d L, p latelet cou nt 110,000/ m m 3, IN R 1.0.

527. A-54-year old m an w ith a history of glioblastom a is brou ght to the em ergency room by fam ily w ho fou nd him u nresp onsive at hom e on the floor. Vital signs are T 37.8°C, H R 98, BP 150/ 80, RR 28, Sp O 2 98% on facem ask oxygen. There are no visible trau m atic inju ries, bu t his clothing is satu rated w ith u rine. H ead CT reveals no significant change from three w eeks ago w here a 2 cm lesion is seen in the right tem p oral lobe w ith m in im al su rrou nd in g ed em a. 528. An 80-year-old w oman w ith a history of prosthetic aortic valve and severe carotid stenosis w ho is p rescribed w arfarin anticoagu lation presents after being found on her floor at home u nresponsive. H er d au ghter reports that she

530. A 21-year-old m an has a history of bone m arrow transp lant 2 w eeks ago p resents w ith com a w ithout focal or lateralizing neu rologic find ings. Vital signs show T 38.8°C, H R 100, BP 80/ 40, RR 32, Sp O 2 96% on room air. Exam is notable for alop ecia, rigors, and cool extrem ities. Laboratory valu es reveal sod iu m 140 mEq/ L, WBC 800/ mm 3 w ith d ifferential count 50% n eu trop h ils, 30% lym p h ocytes, 10% m onocytes, 6% eosinop hils, and 3% basop hils.

Answe rs a nd Expla na tions

457. (C) This previously healthy young patient has d evelop ed sym p tom s referable to both the chest and the head / neck. Epiglottitis, pneumonia, or tracheom alacia could result in d yspnea, cou gh, and p erhap s h oarseness, bu t they w ou ld be u nlikely to cau se facial sw elling. Angioed em a cou ld accou nt for this constellation of sym p tom s, bu t a three-d ay cou rse w ou ld be u nu su al. This p atient has sym p tom s of su perior vena cava (SVC) synd rom e, a clinical d iagnosis based on sym p tom atology. Most cases of SVC synd rom e are caused by m alignancies su ch as lu ng cancer, lym p hom a or m etastases, but benign cau ses (strictures from intravascu lar d evices, thyrom egaly, and aortic aneu rysm ) also exist. SVC syn d rom e in a you ng m an w ith a m ed iastinal m ass is m ost com m only d ue to lym p hom a or a m ed iastinal germ cell tu m or. Patients w ith SVC synd rom e com m only present w ith facial/ neck sw elling, d ysp nea, and cou gh. H oarseness, head ache, congestion, hem optysis, d ysphagia, pain and syncope m ay also be seen. Sym p tom s w orsen w ith a head d ow n p osition. (6:265-6, 2266-7, 2711-3) 458. (D ) Although nonpharmacologic interventions (su ch as a qu iet environm ent, soothing m u sic, or a fam iliar fam ily m em ber at the bed sid e) m ay red u ce the n eed for sed atives, m any m echanically ventilated patients requ ire sed ation for safety. The choice of sed ative agent shou ld be tailored to the p atient, keep ing in m ind the p atient’s p hysiology and the sid e effect p rofiles of the sed ative m ed ications. Benzod iazep ines can w orsen confu sion and d eliriu m . As a resu lt, their u se shou ld be lim ited to cases w here sid e effects of other sed a-

tives are u naccep table or w here alcohol or benzod iazep ine w ithd raw al m ay be contribu ting to confu sion. Etom id ate su p p resses ad renocortical function. It is not appropriate for u se as an infu sion. Sid e effects of p rop ofol inclu d e hyp otension d u e to vasod ilation, resp iratory d ep ression , an d h yp erlip id em ia. Prop ofol infu sion synd rom e is a rare, bu t p otentially fatal sid e effect of p rop ofol ad m inistration. Dexm ed etom id ine is an α 2-ad renocep tor agonist. Ad m inistration cau ses sed ation and analgesia w ith little resp iratory d ep ression. Sid e effects of d exm ed etom id ine inclu d e hypotension and brad ycard ia. Dexm ed etom id ine is rarely u sed for long-term sed ation (> 1 d ay) d u e to cost. In this p atient w ith brad ycard ia and an accep table blood p ressu re, p rop ofol w ou ld be the m ost ap p rop riate sed ative. Pain shou ld also be treated , as u ntreated p ain is associated w ith an increased in cid ence of d eliriu m and w orse ou tcomes. (1:536-8, 548-9; 6:201) 459. (A) Intensive care u nit p atients are at risk for m u ltiple com plications d ue to their critical illn ess. Prop hylactic interventions have been d evelop ed to red u ce m orbid ity from stress u lcers, ventilator associated pneu m onia (VAP) and d eep venou s throm bosis. Oral d econtam ination w ith chlorhexid ine is recom m end ed to red u ce the incid ence of VAP. Other su ggested interventions inclu d e head of bed elevation and d aily assessm ent of read iness to w ean. Clind am ycin is an antibiotic that is active against gram p ositive and anaerobic bacteria. An infection w ith Haemophilus w ou ld not be treated by clind am ycin. Dexam ethasone is a glu cocorticoid . It has no role in the treatm ent 347

348

18: Critica l Ca re Me dicine

of VAP. Om eprazole (a proton pum p inhibitor) is a reasonable choice for stress u lcer p rop hylaxis, bu t p roton p u m p inhibitors are associated w ith in creased rates of p n eu m onia. (1:1434-5, 1755; 6:1114, 2448) 460. (B) Donation after card iac d eath (DCD) is an accepted method of organ d onation that occurs after w ithd raw al of life su p p ort in a p atient w ho is exp ected to d ie w ithou t su ch su p p ort. Recent renew ed interest in DCD has been sp urred by the m ism atch betw een the nu m ber of patients aw aiting organ transplantation and insu fficient nu m ber of d onors from the braind ead p rocu rem ent p ath. DCD d onors are u su ally critically ill from irreversible neu rologic illness, neu rom u scu lar d isease, or high sp inal cord inju ry. Usually the patient and / or fam ily, in collaboration w ith the treating ICU p hysicians, arrive at a d ecision to w ithd raw care in ord er to m inim ize su ffering and allow the patient to d ie. Only in this context (after the d ecision to w ithd raw care has been m ad e) shou ld the p atient or fam ily be ap p roached regard ing the p ossibility of organ d onation. If consent is rend ered by the com petent p atient, or absent this by the fam ily, DCD d onation m ay proceed . In a controlled setting (either the ICU or the op erating room ), life su p p ort is w ithd raw n inclu d ing rem oval of the end otracheal tu be. The p atient’s sym p tom s of p ain, anxiety, and d ysp nea are treated appropriately to p rovid e com fort d u ring d eath. If asystole occu rs w ithin a set tim e interval (usu ally less th an 2 h ), th e p rocu rem en t team w aits th e p rescribed tim e (u su ally 2-5 m in) after card iac d eath and then begins organ procurement. The d ying p rocess and w ait p eriod follow ing asystole subject the organs to a longer w arm ischem ic tim e than brain d ead d onors, bu t thu s far graft ou tcomes from DCD d onors app ear comparable to brain d ead d onors w hen ap propriate selection criteria are satisfied . Even hearts have been rep orted to have been su ccessfu lly transplanted follow ing DCD d onation. (Fanelli V, et al. Curr Opin A naesthesiol 2010; 23:406-10) 461. (A) Pressu re su p p ort ven tilation (PSV) is d epicted in the figure as evid enced by the negative pressu re d eflection before each insp ira-

tion that triggers the d elivered insp iratory flow. End of insp iration in PSV is d eterm ined by a d ecline to a certain p ercentage of p eak insp iratory flow (u su ally 25%, althou gh this m ay be ad ju sted ). Th e figu re show s only p atient triggered insp irations w ithou t ad d itional m and atory breaths. PSV m ay be problem atic in the setting of a large bronchop leu ral fistula since flow through the d efect may never d rop , thereby resu lting in continu ou s insp iration w ithout termination. Minute ventilation in PSV is d eterm ined by patient effort; a higher resp iratory rate w ill resu lt in a higher MV if tid al volum e rem ains app roxim ately constant. (5:1407-9; 6:2212) 462. (D ) In this setting, the brad ycard ia is cau sing asym p tom atic hyp otension. Atrop ine is an ap p rop riate intervention to increase the heart rate rap id ly. Transvenou s p acing requ ires central venou s access and subsequent placem ent of a w ire. This cannot be p erform ed qu ickly enou gh for this situ ation. Card iop u lm onary resu scitation (CPR) is not the ap p rop riate first intervention in this p atient w ho has a p u lse, althou gh CPR m ay be requ ired if the patient’s cond ition d eteriorates or he is u nresponsive to less invasive treatm ents. Sim ilarly, end otracheal intu bation is not necessary at this tim e. (1:227-8, 5:1433; 6:2244) 463. (C) The ECG d emonstrates an acute ST elevation MI. Likely etiologies inclu d e coronary artery plaqu e ru pture or in-stent throm bosis. The primary goal of therapy is rapid coronary rep erfu sion, p referably u sing catheter-based therapies d irected at the involved coronary artery. Mod ern interventions m ay inclu d e throm bus extraction, balloon angiop lasty, or coronary stenting u sing either bare m etal or d rug-eluting stents. Determination of the best intervention d epend s upon coronary anatomic findings and relative risk of potent anticoagulants ind ucing bleed ing at the recent surgical site. Better outcomes from acute MI are achieved w hen the ”d oor to balloon” time is less than 90 m in. Thu s, stat card iology consu ltation and rapid mobilization of the patient to the catheterization suite are key. Pharmacologic measures to red u ce m yocard ial oxygen consu m p tion

Answe rs : 460–468

shou ld be sim u ltaneou sly em p loyed . Thu s, tachycard ia shou ld be treated w ith a betablocker, in this case a short-acting intravenous agent such as esmolol. Myocard ial w all tension should be red uced by treating hypertension. Nitroglycerin is indicated for treatment in acute coronary syndrome (ACS) to d iminish ischemic pain (if present), treat hypertension, reduce pulmonary congestion, and, when implicated, treat coronary vasosp asm . N itroglycerin has not been demonstrated, how ever, to diminish mortality due to ACS. Other pharmacologic therap ies that shou ld be rap id ly ad m inistered include antiplatelet agents (aspirin ± a thienopyrid ine) d epend ing upon the estimated risk of surgical bleed ing from d ual agents as w ell as the sp ecific coronary artery intervention. A statin should be prescribed for patients not currently taking one, and an ACE-inhibitor consid ered for p atients w hose hem od ynam ics w ill tolerate the med ication. Systemic thrombolytics for ACS are contraind icated for patients w ho have u nd ergone recent su rgery (35°C. There is little evidence that bicarbonate administration improves hem od ynam ics or vasopressor response, and bicarbon ate h as m an y n egative effects. Bicarbonate is typically reserved for severe metabolic acid osis (pH < 7.2). Bicarbonate should not be given if the p atient w ill be unable to increase minute ventilation in response to the expected rise in Pa CO 2 prod uced by bicarbonate ad ministration. Although increasing PEEP may improve oxygenation, it is unlikely to improve the hemodynamic picture is a patient w ith an Sp O 2 of 95% and no evidence of volume overload on TEE. This patient has no evidence of hypovolem ia and a hem oglobin of 8.9 g/ d L. Transfusion of PRBC is unlikely to improve his hemodynamics. (5:1074, 1451-2; 6:166, 2222, 2230) 475. (A) A fibrinogen level < 100 m g/ d L can lead to a sign ifican t coagu lop ath y. Fresh frozen p lasm a con tain s fibrin ogen an d w ou ld im p rove this p atient’s coagu lop athy, bu t cryoprecip itate is the blood com p onent containing th e h igh est con cen tration of fibrin ogen . Cryop recip itate w ou ld be m ost likely to

351

resolve a coagu lop athy d u e to fibrinogen d eficiency. Platelet transfu sion is u nlikely to im p rove clotting in a p atient w ith a p latelet cou n t of >100,000/ m m 3. H yp oth er m iaind u ced coagu lop athy w ou ld be exp ected at tem p eratu res below 35°C. DDAVP is u sed to treat bleed ing d u e to som e su btyp es of von Willebrand d isease or severe kid ney d isease. (1:712-4; 5:209, 1443, 1446, 1451; 6:2316) 476. (A) The m ortality associated w ith renal failu re in critically ill p atients is 23-64%. This patient is at risk for p rerenal, p ostrenal, and intrinsic cau ses of renal failure. Urinary obstru ction is a less likely cau se in a p atient still m aking u rine, bu t flu shing or rep lacing the u rinary catheter p lus im aging could ru le ou t this im p ortant etiology of renal failu re. This p atient has a FEN A = 3%, a BUN / creatinine ratio < 20:1, a u rine N a > 40 m Eq/ L, and a u rine osm olality < 350 m Osm ol/ L H 2O. This set of laboratory values m akes an intrinsic cau se of renal failu re m ore likely than a p rerenal cau se. Urinalysis w ithou t w hite or red blood cells (w hich m ight im p licate glomeru lonep hritis or acu te interstitial nephritis) plu s the clinical circu m stance of p rolonged hyp otension m ake acu te tu bu lar n ecrosis the m ost likely etiology for th is p atient’s acu te kid ney inju ry. (5:1365-6, 1395; 6:334-8) 477. (E) There are fou r p arts to the treatm ent of hyperkalemia. First, stabilize the irritable myocard iu m w ith calciu m . Second , d rive p otassiu m intracellu larly. Insu lin + glu cose or inhaled β 2 agonists m ay be u sed for this p u rp ose. These are tem porizing m easu res. Third , eliminate potassium from the bod y. Potassiumw asting d iu retics, renal rep lacem ent therap y or cation exchange resins m ay red uce the total bod y p otassiu m load . Fou rth, treat the u nd erlying cau se of hyp erkalem ia. (1:1291; 5:1074; 6:355-9) 478. (D ) Rh abd om yolysis is a cau se of ren al failu re and electrolyte abn orm alities in critically ill p atien ts. Exp ected laboratory abn orm alities in clu d e h yp erkalem ia an d h yp erp h osp h atem ia d u e to release of in tracellu lar p otassiu m . Metastatic tissu e d ep osition of

352

18: Critica l Ca re Me dicine

calciu m p h osp h ate resu lts in hyp ocalcem ia. (5:148-9; 6:356, 362, 2303, 3089) 479. (D ) This p atient p resents w ith d ysp nea, chest pain, and shock. The d ifferential d iagnosis for this constellation of sym p tom s and find ings w ou ld be broad , inclu d ing p neu m onia, em p yem a, pu lm onary em bolism (PE), p neu m othor ax, m yocard ial isch em ia, m yocard itis, pericard itis/ pericard ial effusion w ith tam ponad e, esop hageal or gastric p erforation, pancreatitis, cholangitis, and aortic d issection. A norm al tem p erature, an IN R > 3, and an ECG w ithou t acu te ST changes m akes infection, PE, or m yocard ial ischem ia u nlikely. The patient’s history of breast cancer m akes m alignant pericard ial d isease m ore likely. Echocard iography is the test m ost likely to id entify p ericard ial effu sion/ tam p on ad e as th e sou rce of th is patient’s shock. (6:102-7, 2267-8) 480. (B) The d ifferential d iagnosis for new -onset seizu res in an ad u lt includ es trau m a, alcohol w ithd raw al, illicit d ru g u se, cerebrovascu lar d isease, brain tu m or, m etabolic d isord ers, and neu rod egenerative d iseases. This p atient w as fou nd to have a frontal m ass on head CT. Patients w ith seizu res d u e to sym p tom atic brain m etastases shou ld be treated w ith anticonvu lsant therap y (to increase the seizu re threshold ) and d exam ethasone (to red u ce tum or related ed em a). CSF d iversion, intu bation and h yp erventilation , m ann itol, an d d ecom p ressive craniectom y w ou ld be treatments for increased intracranial pressure (ICP). This patient is u nlikely to have elevated ICP, as she has a norm al neu rologic exam ination and sharp optic d isks. (1:1755; 6:2271-2, 3258) 481. (A) A sentinel article published in 2006 d emonstrated that a bund le of the follow ing five evid ence based -p roced u res cou ld red u ce the in cid en ce of cen tral lin e-associated blood stream infections (CLABSI) in a sustained fashion: hand w ashing, u se of fu ll-barrier p recau tions d u ring line insertion, chlorhexid ine skin p rep aration, avoid ance of fem oral site, and rem oval of u nnecessary central lines. While other m easu res m ay be u sefu l at p reventing CLABSI, the former bundle was shown effective

across m ore than 100 ICU’s and has been w id ely ad op ted in critical care u nits. (5:226, 234, 1319; 6:1114, 1116) 482. (D ) This p atient likely has p rop ofol infu sion synd rom e, a rare bu t highly fatal and poorly u nd erstood com p lication of p rop ofol. It has been m ost com m only d escribed in critically ill child ren w ho are sed ated on high d oses of prop ofol for a p rolonged p eriod , althou gh the exact d ose and d u ration of p ropofol exposure to p rod u ce th e syn d rom e is u n kn ow n . Characteristic featu res inclu d e rhabd om yolysis, m etabolic acid osis, and w orsening hypotension d espite pressor/ inotropic support. The course typically proceed s to card iac, renal, and other organ failu re before eventu al d eath. Card iac effects m ay m anifest as d ysrhythm ias, includ ing brad ycardia. The pathophysiology is believed to involve p rop ofol im p airm ent of free fatty acid u tilization by the m itochond ria lead ing to failu re of the electron transp ort chain, low cellu lar energy prod uction, and cell d eath. Treatm ent is d iscontinu ation of p rop ofol, and su p p ort of organ failu re. (1:537; 5:693) 483. (C) The figu re d ep icts p ressu re control ventilation (PC). In this m od e, the clinician sets a pressure limit, respiratory rate, and inspiratory tim e. Inspiratory flow occu rs in an exponentially d escend ing w aveform and the pressu re applied to the airw ay is constant. Tid al volu me is the d epend ent variable such that d im inished p u lm onary com p liance, as in the case of this p atient w ith increasing abd om inal d istention, resu lts in d im in ish ed m in u te ven tilation . (5:1406-9; 6:2211-2) 484. (D ) The d isord er is characterized as a m ixed resp iratory acid osis and m etabolic acid osis. The p H for a sim p le acu te resp iratory acid osis w ith a Pa CO 2 = 55 w ou ld be 7.28. This p atient’s pH is lower, suggesting a mixed respiratory and metabolic acid osis. In ad d ition, the base d eficit (or negative base excess) in the setting of ongoing bleed ing likely rep resents u nm easu red anions d ue to lactic acid . (5:525-31; 6:364-6) 485. (B) The d ifference betw een the tw o d ep icted values is pulse pressure variation, the difference

Answe rs : 479–488

betw een m axim al and m inim al p u lse p ressu re valu es d u ring one m echanical p ositive p ressu re breath d ivid ed by the m ean of these tw o valu es. It is only one of the available d ynam ic ind ices that u tilize arterial w aveform contou r d u ring p ositive p ressu re m echanical ventilation as a p red ictor of card iovascu lar resp onse to flu id ad m inistration. Another ind ex that h as been stu d ied is systolic p ressu re variation, the largest d ifference betw een the systolic p ressu re d u rin g on e breath . Systolic p ressu re variation m ay be fu rther characterized into ”d elta u p ,” the d ifference betw een baseline systolic p ressu re d u ring ap nea and the highest systolic p ressu re occu rring im m ed iately after d elivery of a p ositive p ressu re breath , an d ”d elta d ow n ,” th e d ifferen ce betw een baselin e systolic p ressu re d u rin g ap nea and the low est systolic p ressu re occu rring follow ing the increased systolic p ressu re after one m echanical p ositive p ressu re breath. A hyp ovolem ic p atient w ill have a m ore exaggerated arterial w aveform resp onse to p ositive p ressu re ven tilation becau se ven ou s retu rn is m ore im p aired , m ore lu n g u n its behave as West Zone 1 (w here airw ay p ressu re increases RV afterload ), and the ventricles are m ore sensitive to p reload changes w h en op eratin g on th e steep p art of the Starling curve (such as is generally present w ith hyp ovolem ia). (5:425-6; 6:2199; M ichard, F. A nesthesiology 2005; 103:419-28) 486. (E) The pulse p ressu re variation in this patient is greater than 12% that su ggests that the p atient is flu id resp onsive, i.e., flu id ad m inistration is exp ected to result in an increase in stroke volum e. Whether or not flu id should be ad m inistered d ep end s on the clinical context and the risks of flu id ad m inistration in the ind ivid u al p atient. In this case, the p atient has ongoing bleed ing, hypotension, and evid ence of abd om inal com p artm ent synd rom e (tense abd om en, oligu ria, and d im inished p u lm onary com p liance). Flu id ad m inistration is reasonable u ntil m ore d efinitive treatm ent of abd om inal com p artm ent synd rom e (e.g., su rgical d ecom p ression) is p erform ed . Su p p ort of the blood p ressu re w ith a vasop ressor w hile flu id resu scitation is u nd ertaken is reasonable

353

if necessary to p revent p recip itou s d ecline in p erfu sion, but vasop ressors shou ld be view ed as only tem p orizing in treatm ent of hyp ovolem ia. (5:424-8; 6:425, 1394) 487. (A) The Fisher Grad ing Scale d escribes the am ou nt of intracranial blood seen on head CT scan after su barachnoid hem orrhage (SAH ). Progression from Fisher Grad e 2 to Fisher Grad e 3 d escribes an increase in su barachnoid blood , likely d u e to rebleed ing. Rebleed ing is a p oor p rognostic sign in p atients w ith su barachnoid hemorrhage d ue to aneurysmal rupture. Risk of rebleed ing is highest in the first 24 h (u p to 8%), and the associated m ortality is greater than 50%. H yp ertension is a risk factor for rebleed ing. Even transient hyp ertension d u ring intu bation can be d evastating. As a resu lt, h yp ertension d u rin g laryn goscop y shou ld be avoid ed and aggressively treated . Elevated ICP, seizu res, and vasosp asm are other com plications of SAH , bu t they w ou ld not cau se an increase in intracranial hem orrhage. Vasospasm is usually not app arent u ntil >72 h after the hem orrhage. (5:885-7) 488. (C) SIMV is a ventilatory m od e that em ploys a com bination of spontaneou s breaths and ventilator d elivered breaths. It m ay be either volum e or pressu re targeted . The clinician sets a rate and target on the ventilator and the ventilator aim s to d eliver the breath synchronized to patient inspiratory effort, i.e., the ventilator w aits a set p eriod of second s and , if the p atient triggers, the inspiration is tim ed to coord inate. If after the p rogram med w ait p eriod , no breath is d etected , then a m and atory breath is d elivered . Sp ontaneou s breaths are not su p p orted by the ventilator in this m od e. Ad vantages inclu d e exercise of the d iap hragm and , w ith d iap hragm atic m otion, better V/ Q m atching to West zone III lu ng. Ad d itionally, SIMV m ay d ecrease m ean intrathoracic p ressu re, thereby red u cing RV afterload and im proving card iac ou tpu t. One d isad vantage to SIMV, how ever, is a d emonstrated increased w ork of breathing, p articu larly as the set rate on the ventilator is d im in ish ed . In creased w ork of breath in g d u ring SIMV w eaning is also associated w ith

354

18: Critica l Ca re Me dicine

p atient-ven tilator d yssyn ch rony. (5:1410-1; 6:2211-2) 489. (A) The p atient’s acid -base d isord er is a m ild m etabolic acid osis w ithou t an increased anion gap . The m ost likely d iagnosis is intraop erative resu scitation w ith 0.9% N aCl (norm al saline) intravenou s solu tion that is com m only used d uring neu rosurgical proced ures becau se it is slightly hyp ertonic com p ared to p lasm a and th eoretically m ay p rovid e ben efit in d im inishing brain ed em a. H ow ever, ad m inistration of large qu antities of norm al saline cau ses a hyp erchlorem ic m etabolic acid osis w ith norm al anion gap as a resu lt of d ilu tional acid osis. The clinical significance of this acid base d isord er rem ains to be elu cid ated , bu t likely d oes not carry as p oor a p rognosis as lactic acid osis. Loop d iu retic ad m inistration gen erally cau ses a m etabolic ”contraction ” alkalosis. Propofol infusion synd rome and cyanid e toxicity d u e to nitropru ssid e both cau se an elevated anion gap m etabolic acid osis d u e to lactic acid osis. (1:796; 5:508, 528, 535-6) 490. (D) Dopamine is an adrenoceptor agonist that acts at α, β 1, and dopaminergic receptors. It is a recommended first-line agent for the treatment of fluid -resuscitated patients in septic shock. Although it has theoretical benefits of splanchnic and renal circulatory dilatation, it has not been shown to produce superior outcomes when compared to norepinephrine. Studies suggest, however, a higher incid ence of tachycard ia and dysrhythmia with dopamine compared to norepinephrine. Norepinephrine is a more potent adrenergic agonist than dopamine. (1:288; 5:1373) 491. (B) This p atient is su ffering from trau m atic brain inju ry (TBI) w ith CT evid ence of su barachnoid , ep id u ral, and intrap arench ym al hem orrhage. There is m ass effect on the left lateral ventricle, and left to right m id line shift, find ings that su ggest elevated intracranial p ressu re (ICP). Em ergen t treatm ent goals inclu d e avoid ance of fu rther elevation of ICP that m ight precipitate herniation, correction of hyp oten sion an d h yp oxem ia, an d m ed ical m anagem ent of elevated ICP until m ore d efinitive su rgical m anagem ent can be em p loyed .

Priorities for m anagem ent inclu d e end otracheal intubation, carefu lly perform ed to avoid further exacerbation of hypotension, hypercarbia, or h yp oxia. H yp erven tilation m ay be em p loyed for brief p eriod s of tim e u ntil d efinitive su rgical m anagem ent occu rs, bu t significant cerebral vasoconstriction, w hich occu rs w ith Pa CO 2 < 30 m m H g, m ay in fact w orsen cerebral hyp op erfu sion, lead ing to p oor outcom es. Optim al ventilation is to m aintenance of Pa CO 2 35-40 mm H g. H emod ynamic support for th is p atient sh ou ld in clu d e crystalloid volu m e resu scitation , avoid ing hyp oton ic solu tions su ch as lactated Ringer solution that m ay w orsen brain ed ema and ICP. 5% album in shou ld be avoid ed in the resu scitation of p atients w ith trau m atic SAH since it is associated w ith p oor ou tcom es. Blood p ressu re should be su pported w ith vasopressors (phenylep hrine, norep inep hrine) to cerebral p erfu sion p ressu re > 60 m m H g. H yp ertonic saline in 23.4% solution, administered over 10-30 min, can be used to urgently red uce elevated ICP. H ead of bed elevation is im p ortant to facilitate venou s d rainage from the brain. While CPP = MAP − ICP und er conditions of elevated intracranial p ressu re, u n d er norm al cond ition s, CPP = MAP − CVP. Thu s, head of bed elevation should be em ployed , and Trend elenburg position avoid ed in patients at risk for intracranial hyp ertension. Unlike vasogenic ed em a d u e to tu m ors, glu cocorticoid s p lay no role in treatm ent of trau m atic ICP elevation. In this case, neurosu rgical treatm ent options includ e ventriculostom y to d rain CSF and red u ce ICP, and craniectom y w ith rem oval of bone flap to p rovid e d ecom p ression of the sw ollen brain parenchyma. (1:682; 5:537, 1360-1; 6:2221) 492. (D ) Sep ticem ia is the p resence of m icrobes or their toxins in blood . SIRS is p resent w hen there are tw o or m ore of the follow ing: oral tem p eratu re > 38°C or < 36°C, resp iratory rate > 24 breaths/ m in, heart rate > 90 beats/ m in, white blood cell count > 12,000 or < 4,000/ mm 3, or > 10% bands. SIRS may have a non-infectiou s etiology. Sep sis is SIRS w ith a p roven or su sp ected m icrobial etiology. Severe sep sis is d efined as sep sis w ith one or m ore signs of organ d ysfunction. Septic shock is d efined as

Answe rs : 489–497

sepsis p lus hyp otension/ need for vasopressor sup port d espite ad equ ate flu id resu scitation. This p atient has fever, leu kocytosis, an infectiou s sou rce, and neu rologic and renal d ysfu n ction , m akin g severe sep sis th e m ost ap prop riate d iagnosis. (6:2223) 493. (E) This p atient has oligu ria d esp ite an ad equ ate blood p ressu re and CVP. Althou gh it w ou ld be ap p rop riate to initiate a w ork-u p for his oligu ria (inclu d ing flu shing the u rinary catheter, u rine stu d ies, and renal u ltrasound ), the m ost appropriate next step in m anagement is to start an tibiotic treatm en t. An tibiotics w ou ld ad d ress the u nd erlying cau se of his sepsis w ith organ d ysfunction and should be initiated w ithout d elay. It is appropriate to perform a neu rologic evalu ation and consid er the p ossibility of an intracranial p rocess in this confu sed patient, how ever, if d eliriu m is the cause of his m ental status changes, then neuroim aging m ay not be help fu l. (6:196-200, 579) 494. (B) Volu m e resu scitation is an integral p art of shock m anagem ent. In ad d ition to a variety of d ifferent crystalloid s, gelatins, d extrans, albu m in, starches, and blood com p onents have been u sed to expand the intravascu lar volu me. Debate continu es over the m ost ap p rop riate resu scitation flu id w ith d ifferences in blood volu m e effect, ed em a form ation, antiinflam m atory effect, cost and risk of anap hylaxis, coagu lop athy, renal failu re, and pru ritis d om inating the d ebate. H yd roxyethyl starch solutions have been associated w ith an increased risk of acu te ren al failu re in critically ill p atients. (6:1399-1400) 495. (E) Decreased FRC and ERV are th e m ost com m on PFT abnorm alities in obese patients. Chest w all com p liance is d ecreased in obese p atients d u e to ad ipose tissu e in the chest w all and abd om en. Dim inished ou tw ard recoil of the chest resu lts in d im inished FRC and ERV. Dim inished FRC can resu lt in lu ng volu m es less th an closin g cap acity at norm al tid al volum e lead ing to atelectasis. These changes are exacerbated in the p erioperative period by d iap hragm atic d ysfu nction, p ain w ith d im inish ed cou gh an d secretion clearan ce, an d

355

hyp oventilation d u e to sed ative m ed ications. RV is u su ally u nchanged . TLC m ay be near normal in obesity, but d iminishes w ith increasin g BMI. FEV1/ FVC is norm al since both valu es are d im inished relatively equ ally in obesity. Diffusing capacity is usually preserved u nless pulm onary hypertension is present in w hich case this valu e is red u ced . (5:302-3; 6:2093) 496. (C) Atelectasis and hyp oventilation are the m ost com m on cau ses of hyp oxem ia in the PACU. Obesity and abd om inal su rgery w ill both increase th e risk. Su p in e p osition ing intraop eratively, p ostop erative d iap hragm atic d ysfu nction, and p ainfu l resp iration p rom ote atelectasis. Op ioid an algesics m ay w orsen hypoventilation. H yp oventilation and p rovision of su pplemental oxygen postoperatively further contribute to atelectasis. The w ife’s history suggests that this patient has a high likelihood of u nd iagnosed obstructive sleep apnea (OSA) based on the follow ing 4 (of 8) risk factors id entified in the ”STOP-BAN G” score: snoring, d aytim e sleep iness, BMI > 35, and m ale gend er. The rem aining risks assessed for in STOP-BAN G are observed ap neic p eriod s d u ring sleep , high blood p ressu re, age > 50, and neck circu m ference > 40 cm . Resid u al anesthetic agents and p ostop erative op ioid s contribu te to u p p er airw ay obstru ction in p atients w ith OSA d ue to impaired pharyngeal d ilator fu nction that cau ses p haryngeal collap se. (5:304-5; 6:2184) 497. (D ) These three m easu res are reasonable first m anagem ent step s to see if the p atient’s hypoventilation and hypoxemia can be quickly reversed , p otentially avoid ing need for ICU m anagem ent. The opioid antagonist naloxone in sm all, incremental d oses (40 m cg IV) may be u sed to attem pt to reverse resp iratory d epression w ithou t fully reversing analgesic effect. Flu m azenil is a reversal agent for benzod iazep in es, bu t, u n like n aloxon e, p rim arily reverses the CN S d ep ressant effects w ith no or incom p lete reversal of the resp iratory d ep ressant effects of benzod iazep ines. In this case, therefore, w here reversal of respiratory d epression is the priority, naloxone should be the first

356

18: Critica l Ca re Me dicine

choice agent. A trial of N IPPV m ay be helpfu l to reverse hyp oventilation if the patient w ill tolerate the tight-fitting m ask and has intact protective airw ay reflexes. H ead of bed p ositioning at 45 d egrees, by u tilizing gravity to red u ce im pingement of abd ominal contents on the d iap hragm , m ay im prove lu ng and chest w all com p liance, d im in ish atelectasis, an d improve ventilation. Ad d itionally, this position p rovid es su p erior p reoxygenation of obese patients com pared to su pine w hen p rep aring for anesthetic ind u ction and it m ay be u sefu l if the p atient d eteriorates, requ iring end otracheal intubation. An ABG m ay be perform ed after initial step s are taken to correct the p atient’s hyp oxem ia. Sim ilarly, nasogastric tube d ecom pression m ay be perform ed later if gastric d istention is d eem ed an im p ortant contributor to d iminished pulm onary com pliance. H yd rom orp hone carries sim ilar resp iratory d ep ressant risks as m orp hine and shou ld be avoid ed at p resent. (1:511, 656; 5:312, 314-5, 699, 1287) 498. (C) The patient likely has cor pu lm onale (p ulm onary hyp ertension and right ventricu lar hyp ertrop hy and d ilation). This is cou ld be attribu table to obesity-hyp oventilation syn d rom e (OH S) that affects app roxim ately 10% of m orbid ly obese patients. OH S occurs in the setting of chronic hypoventilation that lead s to pu lm onary hypertension and RV failu re. The acu te d eterioration occu rring follow ing end otracheal intu bation is second ary to acu tely d ecreased RV p reload and increased RV afterload associated w ith p ositive p ressu re ventilation. Find ings on TTE includ e evid ence of RV hyp ertrophy and d ilation, elevated estim ated systolic pu lm onary artery pressure, parad oxical m otion of the intraventricu lar septum d u e to RV overload , and a sm all LV d u e to d isp lacem ent of the intraventricu lar sep tu m by the d ilated RV. Althou gh LV failu re is the m ost com m on cause of cor p ulm onale, if this w ere the case in this patient, one w ou ld expect to see rad iographic evid ence of p u lm onary ed em a in this clinical setting. (6:1913-15) 499. (C) Th is p atien t h as sep tic shock, a form of d istribu tive sh ock. In itiation of m ech an ical

ventilation in this setting m ay be expected to w orsen hypotension d u e to 1) red u ced w ork of breathing d im inishing end ogenou s catecholam ine levels; 2) positive p ressu re ventilation red u cing venou s return; and 3) sed ative d ru gs causing vasod ilation or d epressed m yocard ial con tractility. Positive p ressu re ven tilation in creases righ t ven tricu lar afterload , bu t red u ces left ventricu lar afterload , p otentially im p roving left ventricu lar stroke volu m e in p atients w ith LV d ysfunction. Ketam ine d oes not cau se histam ine release (1:539; 6:2199) 500. (D ) Au to p ositive end -exp iratory p ressu re (”au to-PEEP”) is an alveolar p ressu re above the airw ay op ening pressu re. Causes inclu d e flow limitation, dynamic hyperventilation, and a high minu te ventilation. Patients w ith COPD often have au to-PEEP that m ay im pair triggering of the ventilator and m ay cau se p atientventilator dyssynchrony. (5:1411, 1413, 1416, 1419) 501. (E) In p ressu re assist/ control ventilation, tid al volu m e varies based on the peak pressu re setting. Secretions that obstru ct the end otracheal tu be w ill resu lt in a higher p eak insp iratory p ressu re, thereby d im inishing tid al volu m e. Assu m in g n o ch an ge in resp iratory rate, m inute ventilation w ill d ecrease. (5:1407) 502. (D) This patient has sepsis and , as is common in the presentation of sepsis, he is hypovolemic. To treat the hypotension, he should first be volume resu scitated , in this case u sing crystalloid to target a CVP of 8-12 mm H g. There is evid ence that outcom e from septic shock is im proved with ”early goal-directed therapy” that employs a step w ise fashion of rap id interventions to address the hypoperfused state. Current recommend ations by the Surviving Sepsis Campaign includ e initial fluid resuscitation w ith crystalloid to a minimum dose of 30 mL/ kg and continued fluid challenge if there is improvement in d ynamic or static hemod ynamic parameters. H ydroxyethyl starches should not be used to resuscitate septic patients because these produ cts are associated w ith an increased risk of kidney injury in this setting. Other recommend ations includ e that norepinephrine be the initial vasopressor choice w ith epinephrine ad d ed

Answe rs : 498–506

if the MAP goal of 65 mm Hg cannot be achieved w ith norepinephrine alone. Inotropic support should be initiated w ith d obutamine trial if the MAP goal has been achieved but there are ongoing signs of hypoperfusion. Blood transfusion is recommended to target a hemoglobin concentration of 7-9 g/ d L in clinical circu m stances w here active hemorrhage or myocard ial ischemia is not present. It is important to note that at the time of publication of this book, there are several multinational, multicenter trials ongoing to d etermine the ad equacy of these goals and their impact on patient outcomes. Note that these recommend ations may change during the publication cycle of this book. Current recom mendations may be viewed at the website www. survivingsepsis.org. Current goals of resuscitation in septic shock include a mean arterial pressure (MAP) > 65 mm Hg, urine output > 0.5 mL/ kg/ h, and Scv O 2 > 70%. (5:543; 6:2228-31) 503. (C) Cu rrent Su rviving Sepsis Campaign gu id elines su ggest that in this case, w here the MAP goal has been achieved , bu t there are ongoing signs of hyp operfu sion (low Scv O 2), a trial of inotropic su pp ort w ith d obu tam ine, up to 20 m cg/ kg/ m in, should be ad d ed . Treatm ent of im p aired contractility d u e to sep tic shock m ay be ind ivid u alized to the p atient bu t a few general p rincip les ap p ly. Increasing the norep inephrine d ose is not recom m end ed since the MAP goal has been achieved in this p atient. Milrinone is a phosphod iesterase inhibitor that inhibits d egrad ation of cAMP resu ltin g in im p roved m yocard ial contractility and relaxation; it also red u ces system ic and pu lm onary vascu lar resistance, and thu s d ecreases ventricu lar afterload . It is a usefu l d rug for treatm en t of heart failu re bu t m ay exacerbate hypotension in septic shock. Dobutam ine is a β 1 an d β 2 agon ist (β 1 >> β 2) th at in creases m y o card ial con tractility an d th erefore increases card iac ou tp u t. Tachycard ia from d obu tam ine is less com m on than w ith som e other ad renergic agents, althou gh tachyd ysrhythm ias m ay be a sid e effect that requ ires d ose red u ction or d iscontinu ation of the d ru g. Many patients in septic shock w ill require both a vasopressor (e.g., norep inephrine) for α 1m ed iated vasocon strictive effects, an d an

357

in otrope (e.g., d obu tam ine) to enhance card iac contractility. Vasop ressin acts at p erip heral vasop ressin (V1) recep tors to cau se vasoconstriction. It is effective d esp ite acid osis and m ay be usefu l in septic shock. It may be also be beneficial in rep lenishing vasop ressin levels in the relative vasop ressin d eficien t state of sepsis. Generally, vasopressin may be ad d ed as a vasopressor w hen p atients are receiving high d oses of stand ard α-agonists w ithou t accep table resp onse of blood pressu re or to d im inish the d ose of norepinephrine required to achieve the MAP goal. H ow ever, one stu d y (VASST) su ggested that the best benefit of vasopressin in the care of patients w ith septic shock may be in those w ith less severe shock. Ep inep hrine is an effective vasop ressor and inotrop e bu t is associated w ith significant risk of tachyd ysrhythm ias and , p erhap s, gu t ischem ia. For these reasons, it is not consid ered a first line therapy (as is norepinephrine) for treatm ent of sep tic shock. (1:301, 322, 805; 5:1373; 6:1373, 2:2229-30) 504. (E) The figu re show s a large p ericard ial effu sion w ith right atrial and right ventricu lar compression. Pulsus parad oxus > 12 mm H g is an extrem ely sensitive ind icator of p ericard ial tam p onad e in the p resence of p ericard ial effu sion. Tam p onad e is consistent w ith the p atient’s sym ptom s, vital signs, and echocard iograp hic find ings. (6:1830) 505. (C) Im m ed iate p ericard iocen tesis is th e treatm en t for life-th reaten in g p ericard ial tam p onad e. (6:2268) 506. (D ) Asp iration m ay have no, m ild , or severe effects d ep end ing on the qu antity and character of the asp irated m aterial, p atient resp onse, an d u n d erlyin g m ed ical con d ition . In th is case, asp iration of fecu lent m aterial m ay be an ticip ated to p rovoke severe lu n g in ju ry w ith system ic effects (e.g., h yp oten sion ). Since the asp irated contents are solid , bronchoscop y shou ld be p erform ed to rem ove the p articu late. Bronchoalveolar lavage, how ever, is not recom m end ed after asp iration. Prone ventilation m ay be beneficial in p atients w ith ARDS, althou gh a d efinitive ou tcom e benefit

358

18: Critica l Ca re Me dicine

has not been p roven. Chest CT is not likely to p rovid e u sefu l d iagn ostic in form ation to change m anagem ent and the transfer to the rad iologic su ite m ay be h azard ou s to th e p atien t w ith h er cu rren t card iop u lm on ary in stability. Th ere is n o role for h yp erbaric oxygen in the treatm ent of asp iration. (5:1288; 6:1334, 2137, 2141, 2205) 507. (C) Unlike patients w ho aspirate liqu id gastric contents, this p atient shou ld be treated w ith antibiotics d u e to the exp ected bacterial concentration of the small bow el contents. Sod ium bicarbonate ad m inistration in this p atient m ay w orsen acid osis if the lu ng inju ry is severe enough to cau se ventilation im p airm ent. This is d u e to the conversion of sod iu m bicarbonate to CO 2 that is eliminated via the lungs. If severe acid osis requ iring treatm ent exists in a p atient w ith severe lu ng inju ry or ARDS, consid eration shou ld be given to u sing TH AM (trom etham ine), a hyd rogen ion accep tor that functions as a buffering agent w ithout prod ucing CO 2. (5:532, 1074, 1288) 508. (D ) Critical m yocard ial blood flow is associated w ith an aortic d iastolic blood p ressu re > 40 m m H g. CPR is u nlikely to result in return of sp ontaneou s of circu lation if a d iastolic blood pressu re > 40 m m H g is not achieved d u ring chest com pressions. (5:1423, 1427) 509. (D ) Maintaining high qu ality, u ninterru p ted chest compressions is critical to the success of CPR. Current ad ult ACLS guid elines call for chest com pressions that d epress the sternu m 2 inches (5 cm) w ith 50% compression time at a rate of 100 compressions/ min. Interruptions in chest compressions should be minimized and should never be greater than 10 sec long. (5:1423, 1428) 510. (D ) The only consistently su ccessfu l treatm ent of ventricu lar fibrillation (VF) is d efibrillation, and the earlier d efibrillation occurs, the m ore likely the p atient is to su rvive. As a resu lt, the first p riority for a p atient in ventricu lar fibrillation shou ld be d efibrillation if it can be accom p lished w ithin 5 m in of arrest. If greater than 5 min has lapsed since the patient arrested ,

2-3 m in of chest com p ressions should be p erform ed p rior to d efibrillation. This w ill red u ce the m yocard ial oxygen d ebt and increase the chances of su ccessfu l d efibrillation. Intubation shou ld not d elay d efibrillation. Current energy recom m end ations for d efibrillation are 360 J for a m onop hasic d efibrillator. For a bip hasic d efibrillator, the app rop riate energy is d evice specific, bu t 200 J is recomm end ed if the d evice specific setting is u nknow n. A patient in ventricular fibrillation cannot be card ioverted as there is no organized rhythm. Epinephrine and amiod arone are reserved for after d efibrillation h as been attem p ted . Ch est com p ression s shou ld be resum ed im m ed iately after a d efibrillation attem p t (w ith ou t p au sin g for a p u lse/ rhythm check). (5:1430-1, 1433) 511. (B) The low flow cond itions of CPR resu lt in low p u lm onary blood flow and a su bstantial increase in d ead sp ace d u e to lack of p erfu sion to nond epend ent alveoli. This increased d ead space w ill d ilu te the alveolar CO 2 of perfu sed lung units resulting in a low end -tid al CO 2 that correlates poorly w ith arterial CO 2. When flow increases (d u e to im p roved CPR), there is an increase in end -tid al CO 2 d u e to a d ecrease in d ead sp ace (m ore lu ng u nits are p erfu sed ). As a resu lt, end -tid al CO 2 correlates w ell w ith card iac output d uring CPR. Current ACLS gu id elines recom m end end -tid al CO 2 m onitoring as a noninvasive m eans of m onitoring the ad equacy of chest com pressions. An end -tid al CO 2 < 10 m m H g is unlikely to resu lt in su ccessful resu scitation. An end -tid al CO 2 in this range is a m arker of the need to im p rove the d elivered chest com pressions. An increase in end -tid al CO 2 can also act as an early sign of retu rn of sp ontaneou s circu lation. (5:1423, 1427-8) 512. (D ) Cu rrent ACLS gu id elines recom m end 6-8 breaths/ m in (2 sec p au se for 2 breaths after every 30 com p ressions) for p atients receiving CPR w ithou t an ad vanced airw ay and 8-10 breaths/ m in (w ithou t interru p ting chest com p ressions) for patients receiving CPR w ith an ad vanced airw ay. H yp erventilation inhibits effective chest com p ressions and shou ld be avoid ed . (5:1426)

Answe rs : 507–519

513. (B) Post-arrest care shou ld focu s on stabilizing the patient, on treating precipitating causes for the arrest, and on beginning therapeu tic hypotherm ia in approp riate p atients. Tw o stu d ies have d em onstrated im proved neu rologic ou tcom es w hen su rvivors w ho rem ain com atose after retu rn of sp ontaneous circu lation receive 12-24 h of therapeutic hypothermia. H yperoxia and oxygen free rad icals w orsen brain reperfu sion inju ry. As a result, FIO 2 shou ld be titrated to the low est level that allow s m aintenance of an oxygen saturation > 94%. If ischem ia is susp ected , card iac cath eterization sh ou ld be sched u led p rom p tly. It is not necessary to d elay catheterization d ue to institution of therap eu tic hyp otherm ia. N eu rologic p rognosis is d ifficu lt to pred ict in the first 72 h p ost-arrest, and no clinical signs can reliably p red ict p oor neurologic outcome in the first 24 h post-arrest. There is no role for m annitol, a treatm ent for increased intracranial p ressu re, in im m ed iate p ost-arrest m anagem ent. (5:1423, 1435) 514. (D ) Cerebral salt w asting is a form of hyp ovolem ic hyp onatrem ia. Laboratory find ings are notable for low seru m sod ium (< 135 m Eq/ L), low p lasm a osm olality, u rine sod iu m > 40 mEq/ L, high urine osmolality (>100 mOsm/ kg). The m echanism of salt loss via the kid neys is poorly u nd erstood . Cerebral salt w asting m ost com m only occu rs in p atients w ith su barachnoid hem orrhage. (6:345) 515. (C) Congestive heart failu re is associated w ith hyp ervolem ic hyponatrem ia d ue to neu rohum oral activation. Low card iac ou tp u t w ith a red u ced effective circu latory volu m e to the kid neys cau ses renal sod iu m retention, bu t a p rop ortionately greater increase in total bod y w ater th at lead s to h yp on atrem ia. Urin e sod ium is low as the renal excretion of sod ium is lim ited . In this circu m stance, systolic p ressu re variation < 5 m m H g is consistent w ith flu id nonresp onsiveness. (5:89, 425; 6:345) 516. (G) Central d iabetes insip id u s (DI) m ay be seen in traum atic brain injury d u e to im paired ADH release from a d am aged hyp othalam u s. It is a comm on featu re seen in patients w ho are brain d ead , characterized by an absence of

359

brain stem reflexes, and absence of resp iration on ap nea testing. Characteristic laboratory featu res are hyp ernatrem ia, increased seru m osm olality, and an inappropriately d ilute urine (low u rin e osm olality). Withou t treatm ent w ith vasop ressin or d esm op ressin (DDAVP), p atients are at risk for severe hyp ovolem ia w ith associated h em od yn am ic in stability. (5:516; 6:340) 517. (E) This patient has evid ence of hypovolem ia based on hyp onatrem ia, norm al p lasm a osm olality, low u rine sod iu m and , m ost sensitively, a p u lse p ressu re variation > 12%. Pulse pressure variation (PPV) may be used as a d eterm inant of flu id resp onsiveness in p atients on positive pressu re ventilation w ho have an arterial catheter monitoring blood p ressu re. PPV is m easu red d u ring a single p ositive p ressu re breath, and is equ al to the d ifference betw een m axim al and m inim al p u lse p ressu re valu es d ivid ed by the m ean of these m axim u m and m inim um values. It is usually d escribed as a p ercentage, and has the best d iscrim inative ability for p red ictin g flu id resp on siven ess am ong cu rrent resp iratory variability-based ind ices (e.g., systolic p ressu re variation, d elta d ow n, stroke volu m e variation). (5:425; 6:345) 518. (B) H epatorenal synd rom e is a com plication of hep atic cirrhosis w ith associated significant ascites. The p athop hysiology involves severe renal vasoconstriction that p rod u ces oligu ria and an elevated creatinine d esp ite total bod y volu m e overload . H yp onatrem ia and u rine sod iu m < 10 m Eq/ L are characteristic. When severe, prognosis for recovery is poor w ithou t liver transp lantation. (5:1052; 6:2306, 2601) 519. (D ) Severe bu rn inju ry (especially w hen TBSA bu rned exceed s 25%) resu lts in significant tissu e traum a, a generalized capillary leak synd rom e, and hyp ovolem ic shock. Althou gh bu rn inju ry can also be com p licated by m yocard ial d ep ression, an elevated hem atocrit acts as a m arker for hyp ovolem ia and suggests the need for significant resuscitation. A low central venous pressu re, card iac outpu t, and venou s oxygen satu ration is consistent w ith hyp ovolem ic shock. (5:1334-5; 6:2219)

360

18: Critica l Ca re Me dicine

520. (B) The d ifferential d iagnosis for p ostp artu m hyp otension inclu d es infection, hyp ovolem ia, pu lmonary or venou s air embolism, card iomyopathy, and end ocrine abnorm alities. A central venou s p ressu re of 15 m m H g and a hem oglobin concentration of 9 g/ d L m ake hyp ovolem ia or u nd erresu scitated sep tic shock less likely. A patient w ith hypotension, tachycardia, cool ed em atou s extrem ities, and an elevated CVP cou ld have an air em bolism or a card iomyop athy. An embolism cau sing this d egree of hyp otension, how ever, w ould be exp ected to have a greater effect on oxygenation. As a resu lt, p ostp artu m card iom yop athy is a m ore likely cause of the patient’s sym ptom s. (6:1961, 2171-2) 521. (F) The d ifferential d iagnosis for chest p ain is broad , bu t the m ost likely d iagnoses in a patient w ith acu te onset of chest pain and d yspnea are acute coronary synd rom e (ACS) and p u lm onary em bolism (PE). An ECG w ithou t ST changes d oes not sup port the d iagnosis of ACS. This patient’s clinical pictu re is consistent w ith PE. Th e in cid en ce of DVT after h ip rep lacem ent su rgery in p atients w ho d o not receive throm bop rop hylaxis is estim ated to be as high as 80-90% (w ith a 2% incid ence of fatal PE). A large em bolism bu rd en is requ ired to cau se hypotension. As a result, a p atient w ith PE causing hypotension w ould be expected to have a large A-a grad ient and d ifficu lt to maintain oxygenation. Sinus tachycard ia is the most frequ en t ECG abn orm ality in PE. (5:1204; 6:102-7, 2170-2, 2177) 522. (A) Child ren w ith sp ina bifid a have a 28-67% incid ence of latex allergy. Intraop erative exp osu re to su rgical gloves can lead to anaphylaxis d u e to m u cosal absorp tion of latex p roteins. Card iovascu lar collap se and resp iratory d istress are the m ost com m on signs of anap hylaxis in anesthetized p atients. Bronchosp asm resu lts in in creased airw ay p ressu res an d red uced ventilation. Decreased minute ventilation and pu lm onary ed em a (d u e to capillary leak) lead to hyp oxem ia. There is vasod ilation and cap illary leak, resu lting in hyp otension w ith a d ecrease in both p reload and afterload .

CVP w ou ld be exp ected to be low. (5:1478-89; 6:2709-11) 523. (C) This p atient has evid ence of vasod ilatory shock w ith hyp otension and w arm , w ellp erfu sed extrem ities. This p atient w ill fu lfill the clinical d efinition of septic shock, SIRS plu s a p roven or su sp ected m icrobial etiology plus hypotension, if he rem ains hyp otensive after resu scitation. H is clinical p ictu re is consistent w ith asp iration p neu m onia. (6:1334-5, 2218-22, 2223) 524. (E) The d ifferential d iagnosis for shock in traum a includ es hemorrhage, SIRS, pericard ial tam p onad e, tension p neu m othorax, m yocard ial contu sion, and neu rogenic shock. The patient could also have shock that pred ated the trauma, perhaps causing the motor vehicle collision, fall, or other trau m atic m echanism . This you ng p atient has a negative initial trau m a evalu ation, w arm feet and a low heart rate. Althou gh brad ycard ia can occu r in the setting of hem orrhage, it is far m ore com m on in neu rogenic shock. The lack of m otor resp onse, d espite eye opening to verbal stim uli, supports the d iagnosis of high cervical sp inal cord inju ry. H igh sp inal cord inju ry resu lts in neu rogenic shock d u e to arteriolar and venod ilation from in terru p tion of sym p ath etic vasom otor input. (6:2218-22) 525. (B) H ypercarbia is the d iagnosis in this w oman w ith chronic COPD w ho u nd erw ent a su rgical p roced u re involving carbon d ioxid e insufflation of the peritoneum . Abd om inal d istention has cau sed basilar atelectasis, w orsened V/ Q m ism atch, and an increased fraction of d ead space ventilation. Sed ative med ications ad ministered d u ring the anesthetic have exacerbated hypoventilation and baseline hypercarbia. This in combination w ith carbon dioxid e absorption from the su rgical p roced u re has resu lted in severe acute su perim posed on chronic hypercarbia w ith the neu rologic find ing of com a. (5:2252) 526. (H) The clinical p resentation of head trau m a w ith a p eriod of lu cid ity then abru p t onset of

Answe rs : 520–530

361

coma is suggestive of epid ural hematom a, perhaps d u e to sku ll fracture w ith inju ry to the m id d le m eningeal artery. Treatm ent shou ld consist of stat CT of the head follow ed by em ergen t n eu rosu rgical d ecom p ression . (6:2254, 3379)

resultant brainstem compression, may prod uce com a. Depend ing u pon the size of the infarct, involvem ent of d om inan t or nond om in ant hem isphere, and neurologic p rognosis, su rgical intervention m ay be offered for d ecompression. (6:2252)

527. (J) A p ostseizu re state m ay be a transient cau se of com a. The etiology is thought to be d u e to release of toxic m etabolites d u ring the seizu re or to tem p orary exhau stion of brain energy su p p lies. EEG p erform ed d u ring the com a m ay reveal a generalized slow ing sim ilar to toxic m etabolic encephalopathy. (6:2249)

529. (F) The clinical p ictu re, notable for high fever and severe leu kocytosis, is m ost consistent w ith m eningitis from an ind w elling foreign bod y (external ventricu lar d rain). Treatm ent includ es immediate broad spectrum antimicrobials and rem oval of the d rain. (6:2252)

528. (D ) A large infarct of a cerebral hem isphere, su ch as a m id d le cerebral artery stroke w ith

530. (E) Sep tic shock or other severe infections can p rod uce com a and is the m ost likely d iagnosis in this im m u nosu p p ressed p atient. (6:2252)

This page intentionally left blank

CHAPTER 19

Ac ute and Chro nic Pain Que s tions DIRECTION S (Qu estions 531-600): Each of the num bered item s or incom plete statem ents in this section is follow ed by answ ers or by com p letions of the statem ent. Select the ON E lettered answ er or com pletion that is BEST in each case. 531. A-δ fibers (A) are u nm yelinated (B) are low -threshold m echanorecep tors (C) increase their firing as the intensity of the stim u lu s increases (D) d o not resp ond to noxiou s stim u li (E) are thick nerves 532. In a p atient w ith skeletal m etastases, bisp hosp honates have all of the follow ing effects EXCEPT (A) inhibiting the recru itm ent and fu nction of osteoclasts (B) inhibition of osteoblasts (C) they have their greatest effect in breast cancer and m ultiple m yelom a (D) they have an acu te p ain-relieving effect 533. In the d orsal horn of the sp inal cord , (A) cells from lam ina I and II p roject to the hypothalam u s (B) stim u lation of lam ina I and II p rod u ces pain (C) lam ina I and II are fou nd in the thoracic segm ent of the spinal cord only (D) d ischarge from lam ina I and II d ecreases as a noxiou s stim ulu s increases (E) w id e d ynam ic range (WDR) neu rons are located p red om inantly in lam ina I and II

534. Which one of the follow ing is tru e regard ing seizures as one of the m ultiple sid e effects from the u se of opioid s? (A) Morp hine and related op ioid s can cau se seizu re activity w hen m od erate d oses are given. (B) Seizu re activity is m ore likely w ith m eperid ine, especially in the eld erly and w ith renal d ysfu nction. (C) Seizu re activity is m ed iated throu gh stim u lation of N -m ethyl-D-aspartate (N MDA) recep tors. (D) N aloxone is very effective in treating seizu res prod u ced by m orphine and related d ru gs inclu d ing m ep erid ine. (E) Seizu re activity is m ost likely related to the fact that opioid s stim u late the prod u ction of γ -am inobu tyric acid (GABA). 535. Wind u p is a phenom enon that occurs d u e to constant inpu t of C-fiber activity to the spinal cord . This phenom enon d efines (A) red u ction in excitability of sp inal neu rons in the d orsal root ganglion (B) increase in excitability of sp inal neu rons in the d orsal root ganglion (C) red u ction in excitability of sp inal neu rons in the d orsal horn (D) increase in excitability of sp inal neu rons in the d orsal horn

363

364

19: Acute a nd Chronic Pa in

536. The cricoid cartilage corresp ond s w ith w hich vertebra? (A) (B) (C) (D) (E)

C1 C3 C5 C6 C8

537. The lu m bar facet joints are oriented (A) (B) (C) (D) (E)

in a coronal p lane in a sagittal p lane 45° off the sagittal p lane 20° off the coronal p lane 20° off the sagittal p lane

538. Vertebroplasty is ind icated for all of the following cond itions EXCEPT (A) m u ltip le m yelom a (B) chronic com p ression fractu res of vertebral bod y (C) osteolytic m etastatic tu m ors (D) facet arthrop athy 539. Ziconotid e, found in snail venom , acts prim arily on w hich typ e of calciu m channel? (A) (B) (C) (D) (E)

N -type T-type L-type P-typ e Q-type

540. Viscera are su p p lied by sym p athetic nerves that contribute to pain generation and transm ission. These nerves release all of the follow ing chem ical su bstances EXCEPT (A) (B) (C) (D)

norep inep hrine histam ine serotonin ep inep hrine

541. Pretreatm ent w ith an N MDA antagonist prior to inflam m ation has been show n to (A) enhance central sensitization (B) attenu ate central sensitization

(C) have no effect on central sensitization (D) enhance p erip heral sensitization (E) attenu ate p erip heral sensitization 542. Ind ications for lu m bar sym p athetic blockad e inclu d e all of the follow ing EXCEPT (A) (B) (C) (D) (E)

acu te herp es zoster p hantom lim b p ain com p lex regional p ain synd rom e lu m bar facet synd rom e vascu lar insu fficiency

543. N MDA receptor blockad e in the spinal cord (A) causes inhibition of pain m od u lation (B) causes m od u lation of p ain transm ission (C) d oes not have a role in pain transm ission (D) causes red uction in pain transm ission 544. One im p ortant characteristic of m ethad one that has to be consid ered w hen prescribing it on an ou tpatient basis is (A) there is u su ally a low chance for interactions on patients taking m u ltiple m ed ications (B) w ithd raw al sym p tom s are as severe as w ith m orp hine (C) it is rarely u sed in op ioid ad d iction (D) sed ation and resp iratory d ep ression can ou tlast the analgesic action (E) it allow s rap id titration 545. With the u se of intrap leu ral catheters for p ostoperative pain, all of the follow ing statem ents are tru e EXCEPT (A) the u se of p arenteral op ioid s is contraind icated (B) p neu m othorax can occu r (C) the u su al d ose is 20 to 30 m L 0.5% bu p ivacaine (D) the m echanism of action is ”u nilateral intercostal nerve block”

Que s tions : 536–551

546. A 35-year-old w om an com es to you r clinic com plaining of pelvic pain. Which one of the follow ing is im p ortant to consid er d u ring her evalu ation? (A) End om etriosis is the m ost com m on cau se of pelvic p ain in w om en. (B) End om etriosis m ost likely d oes not have an inflam m atory com p onent. (C) End om etriosis has been show n to be prim arily d ep end ent on the blood concentration of progesterone. (D) An inflam m atory p rocess w ou ld be su p ported by find ings of a d ecrease of interleu kin-8 in testing of p eritoneal flu id . 547. Which one of the follow ing statem ents regard ing fibrom yalgia is tru e? (A) Tw o central criteria for fibrom yalgia are chronic w id espread pain (CWP) d efined as pain in all fou r quad rants of the bod y and the axial skeleton for at least tw o years, and the find ing of p ain by 25-kg pressu re on d igital p alpation of at least 11 of the 18 d efined tend er p oints. (B) It is generally agreed that abnorm al CN S m echanism s are resp onsible for all of the sym ptom s of fibrom yalgia. (C) There are both p rim ary and second ary fibrom yalgia synd rom es. (D) Fibrom yalgia sym p tom s generally resolve if a rheu m atic p rocess is id entified and treated ap p rop riately. (E) Most fibrom yalgia p atients are m ale. 548. A recom m end ed p ractice in the m anagem ent of patients w ith cancer pain is (A) less concern regard ing sid e effects than in p atients w ith nonm alignant p ain (B) d osing analgesics only on an as need ed basis (C) consid ering the u se of ad ju vant d ru gs (D) avoid ing op ioid d ru gs d u e to the p otential for d evelop m ent of tolerance

365

549. An 85-year-old w om an com es into you r clinic w ith chronic p ain over her left breast for m ore than 1 year. The sym p tom s began after she broke ou t in a rash in the sam e d istribu tion. Which one of the follow ing statem ents is tru e? (A) Zoster reactivation is alw ays accom p anied by a rash. (B) Zoster reactivation m ay occu r tw o to three tim es in a healthy ind ivid ual. (C) Post herp etic neu ralgia (PH N ) is p ain that p ersists for m ore than 120 d ays. (D) The incid ence of PH N is exp ected to rem ain stable in the fu tu re. (E) PH N shou ld be treated w ith an antiviral agent like ganciclovir. 550. A 10-year-old boy w ith a d iagnosis of sickle cell d isease com es into your clinic. Which one of the follow ing statem ents is tru e regard ing his cond ition? (A) A vaso-occlusive crisis com m only involves the back, legs, and eyes. (B) Acu te p ain in patients w ith sickle cell d isease is cau sed by ischem ic tissu e inju ry resu lting from the occlu sion of m acrovascular bed s by sickled erythrocytes d uring an acu te crisis. (C) When a vaso-occlu sive crisis lasts longer than 7 d ays, it is im p ortant to search for other cau ses of bone pain. (D) Patients w ith sickle cell d isease have a low er incid ence of vaso-occlu sive d isease than p atients w ith β -thalassem ia. 551. Com p lex region al p ain syn d rom e typ e II (CRPS II) d iffers from CRPS I becau se in CRPS II there is (A) (B) (C) (D) (E)

allod ynia m ovem ent d isord er su d om otor and vasom otor changes evid ence of m ajor nerve d am age severe sw elling

366

19: Acute a nd Chronic Pa in

552. What is the prim ary m echanism by w hich op ioid s prod u ce analgesia? (A) Cou p ling of op ioid recep tors to sod iu m and p otassiu m ion channels, thereby inhibiting neu rotransm itter release (p resynaptic), and inhibiting neuronal firing (p ostsynap tically) (B) Cou p ling of op ioid recep tor to p otassium and calciu m channels, thereby inhibiting neurotransm itter release (p resynap tic), and inhibiting neu ronal firing (p ostsynaptically) (C) Cou p ling of op ioid recep tors to sod iu m and calciu m channels, thereby inhibiting neu rotransm itter release (presynaptic), and inhibiting neu ronal firing (p ostsynap tically) (D) Cou p ling of op ioid recep tors to p otassiu m and calciu m channels, thereby inhibiting neu ronal firing (p resynap tically), and inhibiting neurotransm itter release (p ostsynap tically) (E) Cou p ling of op ioid recep tors to sod iu m and calciu m channels, thereby inhibiting neu ronal firing (p resynap tically), and inhibiting neu rotransm itter release (p ostsynap tically) 553. The d efinition of p ain that is end orsed by the International Association for the Stu d y of Pain is, ”Pain is an u np leasant sensory and em otional exp erience associated w ith actu al or potential tissue d am age, or d escribed in term s of such d am age.” There are a host of p hysiologic m echanism s by w hich inju ries lead to nociceptive responses and u ltim ately to pain. H ow ever, not all nocicep tive signals are p erceived as p ain and not every p ain sensation originates from nociception. All of the follow ing statements regard ing pain are true EXCEPT (A) tw o typ es of p ain recep tors are p rim arily activated by nocicep tive inp u t. These inclu d e low -threshold nociceptors that are connected to fast p ain-cond ucting A-δ fibers, and highthreshold nocicep tors that cond uct im p u lses in slow (u nm yelinated ) C fibers

(B) neu rotransm itters (e.g., glu tam ate and substance P) are able to m od ulate postsynaptic resp onses w ith fu rther transm ission to su p rasp inal sites (thalam u s, anterior cingu late cortex, insu lar cortex, and som atosensory cortex) via ascend ing p athw ays (C) p rolonged or strong activity of d orsal horn neu rons cau sed by rep eated or su stained noxiou s stim ulation m ay subsequ ently lead to increased neu ronal resp onsiveness or central sensitization (D) windup refers to a mechanism present in the peripheral nervou s system in w hich rep etitive noxiou s stim u lation resu lts in a slow tem p oral su m m ation that is exp erienced in hu m ans as increased p ain (E) su bstance P is an im p ortant nocicep tive neurotransm itter. It low ers the threshold of synaptic excitability, resu lting in the u nm asking of norm ally silent interspinal synap ses and the sensitization of second -ord er spinal neu rons 554. A 32-year-old female d evelops severe stabbing, ”like an ice p ick,” p ain at the base of tongu e after an infratem p oral neu rosu rgical p roced u re. Pain com es in paroxysm s and last a few second s and is triggered by sw allow ing, yaw ning, and coughing. This patient m ost likely has (A) (B) (C) (D) (E)

trigem inal neu ralgia genicu late neu ralgia glossop haryngeal neu ralgia m igraine w ith atyp ical au ra clu ster head ache

555. An ad vantage of tran scu taneou s electrical nerve stim u lation (TEN S) for p ostop erative p ain is (A) no p atient instru ction is need ed (B) the absence of op ioid -ind u ced sid e effects (C) it can be u sed by p atients w ith pacem akers (D) it d oes not requ ire any action on the part of the p atient

Que s tions : 552–563

556. A 31-year-old w om an p resents to you r office w ith m arked p ain and sw elling in her ankle 6 w eeks after an op en red u ction and internal fixation w ith casting. On exam ination, the ankle is w arm and erythem atou s. Ligh tly tou ching the ankle w ith a cotton sw ab evokes severe, lancinating pain. You su spect CRPS I. As a synd rom e, CRPS is d iagnosed by (A) (B) (C) (D) (E)

lu m bar sym p athetic block p hentolam ine infu sion test trip le p hase isotop e bone scan erythrocyte sed im entation rate history and p hysical exam ination

557. Which one of the follow ing statem ents concerning central pain is tru e? (A) Sp inal cord inju ry is the lead ing cau se of central pain in the United States. (B) Lesions involving sp inothalam ocortical pathw ays are necessary and su fficient to cau se central pain. (C) Central p ain is a com m on sequ ela follow ing neu rosu rgical p roced u res. (D) Motor cortex stim u lation is an effective m eans to treat central p ain. (E) The m ost typ ical p resentation of central pain is a spontaneou s, bu rning sensation over the entire bod y contralateral to the lesion site. 558. Ep id u ral steroid injections m ay be effective in w hich one of the follow ing cond itions? (A) H erniated nu cleu s p u lp osu s w ithou t neurologic d eficit (B) Ankylosing sp ond ylitis (C) Fibrom yalgia (D) Fu nctional low back p ain 559. A 42-year-old m an u nd erw ent a celiac plexu s block proced ure w ith 20 mL of 50% alcohol. All of the follow ing listed cond itions are complications of this intervention EXCEPT (A) genitofem oral neu ralgia (B) hyp ertension (C) d iarrhea

367

(D) p aralysis (E) infection 560. A p atient w ho received 1 m L of 0.25% bu pivacaine after negative asp iration follow ing a selective cervical nerve root injection becam e agitated and then d eveloped generalized tonicclonic m ovem ents. Which one of the follow ing is the m ost likely exp lanation? (A) H igh sp inal anesthetic from accid ental intrathecal injection (B) Anxiety attack from p ain d u ring injection (C) Vertebral artery injection of local anesthetic (D) Injection into sp inal cord (E) H yp oxia 561. Which one of the follow ing is tru e regard ing oxcarbazepine? (A) It has m ore ad verse effects than carbam azep ine. (B) It is a sod iu m channel blocker. (C) A d ose ad ju stm ent is u nnecessary in a p atient w ith renal insufficiency. (D) Its m ost frequ ent ad verse effects is w eight loss and d izziness. 562. Sp inal cord stim u lation (SCS) has been u sed w ith su ccess for the treatm ent of all of the follow ing cond itions EXCEPT (A) (B) (C) (D) (E)

failed back su rgery synd rom e com p lex regional p ain synd rom e angina p erip heral vascu lar d isease d iffu se chronic p ain synd rom es

563. A p atient w ith obstru ctive lu ng d isease d evelop s com p lex regional pain synd rom e involving the right arm after an inju ry. Treatm ent of the synd rom e m ay involve all of the follow ing EXCEPT (A) (B) (C) (D)

stellate ganglion block lu m bar sym p athetic block su rgical sym p athectom y TEN S

368

19: Acute a nd Chronic Pa in

564. As com p ared w ith som atic pain, all of the follow ing are tru e abou t visceral p ain, EXCEPT (A) it m ay follow the d istribu tion of a som atic nerve (B) it is d u ll and vagu e (C) it is often p eriod ic and bu ild s to p eaks (D) it is often associated w ith nau sea and vom iting (E) it is p oorly localized 565. The follow ing statem ents are tru e regard ing preem ptive analgesia, EXCEPT (A) p reem p tive analgesia is help fu l in red u cing p ostop erative p ain in p art by red u cing the p henom enon of central sensitization (B) early p ostop erative p ain is not a significant pred ictor of long-term pain (C) local anesthetics, op ioid s, and COX inhibitors can be u sed for p reem p tive analgesia (D) p reem p tive analgesia m ay have the p otential to p revent the d evelop m ent of chronic p ain states (E) p reem p tive analgesia is thou ght to red u ce neu rop lastic changes in the sp inal cord 566. All of the follow ing statem ents abou t p ostherpetic neu ralgia (PH N ) are correct, EXCEPT (A) a m id thoracic d erm atom e is one of the m ost com m on sites for PH N (B) m en are affected m ore often than w om en in a ratio of 3:2 (C) the op hthalm ic d ivision of the trigem inal nerve is one of the m ost com m on sites for PH N (D) PH N m ay occu r in any d erm atom e (E) PH N has an incid ence of 9% to 14.3% 567. Which of th e follow in g statem en ts is tru e regard ing acetam inop hen toxicity? (A) Acetam inophen is nontoxic u ntil u nd ergoing m etabolism . (B) Oral glu tathione is the antid ote of choice.

(C) The antid ote m u st be started w ithin 6 h of ingestion in ord er to be effective. (D) Vasod ilatory β 2-ad renocep tor agonists p otentiate the antid ote efficacy. 568. Which one of the follow ing is tru e regard ing resp iratory d ep ression related to the u se of opioid s? (A) Op ioid agonists, p artial agonists, and agonist/ antagonists p rod u ce the sam e d egree of respiratory d epression. (B) Op ioid s p rod u ce a leftw ard shift of the CO 2 resp onsiveness cu rve. (C) Dep ression of resp iration is p rod u ced by a d ecrease in resp iratory rate, w ith a constant m inu te volu m e. (D) N aloxone p artially reverses the op ioid ind u ced resp iratory d ep ression. (E) The ap neic threshold is d ecreased . 569. Of the tw o principal neurolytic agents, alcohol and p henol, (A) alcohol has the greater tend ency to p rod u ce neu ritis (B) alcohol is u sed in a 6% concentration (C) p henol has the m ore rap id onset of action (D) p henol is u sed in a 50% concentration 570. Pain theories cu rrently being proposed postu late a (A) straight stim u lu s-to-nervou s system p ath (B) system involving large fibers only (C) system w ith inhibition exerted by sm all fibers (D) system involving excitatory and inhibitory inpu t w ith resu lting sensation 571. Clu ster head aches are characterized by (A) lancinating u nilateral head ache that is comm only triggered by stress factors (B) p ain that is strictly u nilateral and au tonomic sym p tom s that occu r ip silateral to the pain

Que s tions : 564–578

(C) slow onset w ith p rogressive w orsening of the pain over several hou rs w ith an attack u sually lasting 3 to 4 d ays (D) the com m on u se of m elatonin as therap y for the acute attack (E) a higher incid ence in eld erly p atients 572. Which one of the follow ing characterizes spontaneou s intracranial hypotension (SIH )? (A) It is the sam e entity as p ost–d u ral pu ncture head ache (PDPH ). (B) The head ache is consistently u nilateral. (C) Orthostatic head ache is p athognom onic. (D) Patients com p lain of bitem p oral head ache. (E) To confirm the d iagnosis, it is requ ired that cerebrospinal flu id (CSF) opening pressu res are below 60 m m H 2O. 573. Ad verse effects of ep id u rally ad m inistered glu cocorticoid s inclu d e all of the follow ing, EXCEPT (A) (B) (C) (D) (E)

Cu shing synd rom e osteop orosis avascu lar bone necrosis hyp oglycem ia su p p ression of the hyp othalamicpitu itary axis

574. All of the follow ing are tru e abou t chronic pain in the spinal cord injury (SCI) patient, EXCEPT (A) ap p roxim ately tw o-third s of all SCI patients suffer from chronic pain (B) ap p roxim ately one-third of SCI p atients w ith p ain have severe p ain (C) p ain in SCI p atients m ay lead to severe d epression and even suicid e (D) becau se of the overw helm ingly significant im pairm ent of other im p ortant fu nctions, p ain is only a m inor consid eration in an SCI p atient (E) p ain in SCI interferes w ith rehabilitation and activities of d aily living (ADLs)

369

575. Which one of the follow ing is tru e in relation to com plex regional pain synd rom e (CRPS)? (A) Males are m ore com m only affected than fem ales. (B) CRPS II is m ore com m on than CRPS I. (C) Three-p hase bone scan show ing u nilateral p eriarticu lar u p take is m and atory to confirm CRPS d iagnosis. (D) The d iagnosis of CRPS is m ainly clinical. (E) The m ean age group is betw een 15 and 25 years. 576. In the neu rolytic treatm ent of cancer p ain, all of the follow ing are true statem ents EXCEPT (A) alcohol injections arou nd a p erip heral nerve m ay p rod u ce an u ncom fortable neu ritis. (B) bow el and blad d er fu nction is alm ost alw ays p reserved . (C) su ch treatm ent shou ld be u sed only in those w ith term inal d isease. (D) absence of sensation m ay be p erceived by the p atient to be w orse than the p ain. 577. Ad vantages of intrathecal d rug-d elivery are all of the follow ing EXCEPT (A) the first-pass effect can be avoid ed (B) intrathecal m orphine is 300 tim es as effective as oral m orp hine for equivalent analgesia (C) the num ber of CN S-associated sid e effects can be red u ced (D) the blood brain barrier d oes not interfere w ith the CN S u p take of the d ru g (E) there is no system ic absorption of d ru gs ad m inistered via the intrathecal rou te 578. Which one of the follow ing opioid s d oes not p rod uce d ose-d epend ant brad ycard ia? (A) (B) (C) (D) (E)

Morp hine Fentanyl Mep erid ine Su fentanil Alfentanil

370

19: Acute a nd Chronic Pa in

579. The stellate ganglion is located betw een (A) (B) (C) (D) (E)

C6-C7 C7-T1 C5-C7 C5-C6 T1-T2

580. Which one of the follow ing statem ents is false regard ing tram ad ol? (A) (B) (C) (D)

It has op ioid characteristics. There is a d ose lim it of 400 m g/ d . It is a centrally acting analgesic. There is no effect on norep inep hrine or serotonin neurotransm ission. (E) The d ru g inhibits the reu p take of norep inep hrine and serotonin. 581. The p aroxysm al hem icranias are rare benign head ache d isord ers that may typically be associated w ith all of the follow ing EXCEPT (A) (B) (C) (D) (E)

conju nctival injection rhinorrhea p tosis eyelid ed em a m onocu lar blind ness

582. All of the follow ing statements regarding the theory and use of acupuncture are true EXCEPT (A) ”qi” is the life force or energy that flow s throu gh the bod y (B) ”qi” influ ences ou r health at physical, m ental, em otional, and sp iritu al levels (C) any excess or d eficiency of ”qi” w ill contribu te to ou r health problem s (D) blockage of ”qi” m ay cau se p ain (E) acup unctu re shou ld not be offered as p art of com prehensive pain m anagem ent u ntil m ore clinical trials proving its efficacy have been com p leted 583. The gate control theory is one p ostu lated m echanism of action for spinal cord stim ulators (SCS). Which one of the follow ing is the m ost accu rate app lication of SCS to this postu lated m echanism of action?

(A) Activation of large-d iam eter afferents thereby ”closing the gate” (B) Activation of large-d iam eter afferents thereby ”opening the gate” (C) Activation of sm all-d iam eter afferents thereby ”closing the gate” (D) Activation of sm all-d iam eter afferents thereby ”opening the gate” (E) Activation of both large- and sm alld iam eter afferents equ ally 584. The d u ration of asp irin effect is related to the tu rnover rate of COX in d ifferent target tissues becau se asp irin (A) com p etitively inhibits the active sites of COX enzym es (B) nonirreversibly inhibits COX activity (C) irreversibly inhibits COX activity (D) noncom p etitively inhibits the active sites of COX enzym es (E) acetylates COX-1 585. Which one of the follow ing best fits the pharm acologic m echanism s of action of trad itional nonsteroid al antiinflam m atory d ru gs? (A) (B) (C) (D) (E)

Inhibition of p hosp holip ase A2 Inhibition of COX-2 Inhibition of lip oxygenase Inhibition of arachid onic acid Inhibition of p rostagland in G/ H synthase enzym es

586. The follow ing statem ents are tru e abou t m ethad one EXCEPT it (A) (B) (C) (D)

has a highly variable oral bioavailability is a low cost m ed ication has no know n active m etabolites has N -m ethyl-D-asp artate (N MDA) recep tor agonist p rop erties (E) has high lip id solu bility 587. Ketam ine and m em antine are N MDA receptor (A) allosteric regu lators (B) agonists

Que s tions : 579–596

(C) inverse agonists (D) antagonists 588. Which one of the follow ing is the correct statem ent regard ing the pharm acologic properties of nonselective COX inhibitors? (A) They read ily cross the blood –brain barrier. (B) Their chem ical stru ctu re consists of aromatic rings connected to basic functional grou p s. (C) They act m ainly in the p erip hery. (D) They have a high renal clearance. (E) They are not m etabolized by the liver.

371

(D) Free nerve end ings on A-d elta and C fibers (E) Golgi–Mazzoni end ings 593. All of the follow ing statem ents are tru e abou t m yofascial p ain EXCEPT (A) it is d erm atom al in d istribu tion (B) it shou ld be treated early in the cou rse of the d isease (C) injection of local anesthetic m ay p rovid e relief (D) the p ain can occu r in the back, neck, and should ers (E) it can som etim es be relieved by sim p ly need ling the affected area

589. Occipital neuralgia involves (A) the greater occip ital nerve (B) the cervical p lexu s (C) a p ain d istribu tion confined to the occipital area (D) the scap u lar nerve (E) trop hic lesions of the sku ll 590. A tricyclic antid ep ressant in the second ary am ine class is (A) (B) (C) (D) (E)

im ip ram ine nortrip tyline d oxep in am itrip tyline trazod one

591. Inhibitory substances that are believed to m odu late the transm ission of nocicep tive signals in the d orsal horn of the spinal cord includ e all of the follow ing EXCEPT (A) (B) (C) (D) (E)

su bstance P β -end orp hin d op am ine epinephrine ad enosine

592. Which one of the follow ing is a nocicep tor? (A) Meissner ’s corpu scles (B) Pacinian corpuscles (C) Merkel’s d isks

594. All of the follow ing are m ajor anatom ic stru ctu res in the transm ission and relay of nociceptive inform ation EXCEPT (A) (B) (C) (D) (E)

sp inothalam ic tract locu s coeru leu s thalam u s reticu lar form ation sensory cortex

595. Gabapentin (A) is stru ctu rally unrelated to GABA (B) acts d irectly at GABA-bind ing site in the CN S (C) inhibits voltage-d epend ent calcium channels (D) is the d ru g of choice for fibrom yalgia 596. Ad van tages of p atient-con trolled analgesia (PCA) inclu d e all of the follow ing EXCEPT (A) high p atient satisfaction (B) elim ination of p ainfu l injections (C) no need to ad ju st d osing p aram eters w ith increasing age (D) m ore consistent levels of analgesia (E) the ability of the p atient to titrate p ain relief to p ainfu l p roced u res su ch as chest physical therapy

372

19: Acute a nd Chronic Pa in

597. The effectiveness of a neu rolytic agent is d epend ent on all of the follow ing EXCEPT (A) (B) (C) (D) (E)

location of the injection concentration histology of the nerve volu m e need le size

598. If a p atient u nd ergoing thoracotom y receives intercostal blocks w ith bu p ivacaine, his p ostoperative period w ill (A) be little d ifferent from controls (B) show m arked im p rovem ent in resp iratory fu nction over controls (C) show little d ifference in vital cap acity bu t m arked p ain relief (D) be m arked by hyp erventilation (E) be m arked by increased incid ence of atelectasis

599. The McGill Pain Qu estionnaire (A) (B) (C) (D) (E)

consists of three m ajor m easu res w as d evelop ed by McGill is not w id ely u sed is a single-d im ensional p ain scale d oes not ask abou t the location of p ain

600. Which of the follow ing is a tru e statem ent concerning the u se of epid u ral m orp hine? (A) A bip hasic resp iratory d ep ression p attern can d evelop, w ith the initial phase w ithin 30 m in of the bolu s d ose and a second p hase 2-4 h later. (B) Sp inal m orp hine solu tions m u st be p reservative-free w hile epid u ral m orp hine solu tions m ay contain p reservatives. (C) Patients shou ld be closely m onitored for 48 hou rs after the ad m inistration of ep id u ral m orp hine. (D) Patients m ay am bu late w ith assistance after an injection of ep id u ral m orp hine.

Answe rs a nd Expla na tions

531. (C) A-δ fibers are thin, myelinated fibers, hence they have a faster cond u ction velocity than C fibers. They are high threshold m echanoreceptors. They are associated w ith sharp p ain, tem p eratu re, cold , and p ressu re sensations. (7:13-4) 532. (B) Bisp hosp honates d ecrease resorp tion of bone d irectly, by inhibiting the recru itm ent and fu nction of osteoclasts, and ind irectly by stim u lating osteoblasts. In patients w ith bony m etastases, they are the stand ard therapy for hypercalcem ia after rehyd ration, and have the greatest effect in p atients w ith breast cancer and m u ltip le m yelom a. Bisphosp honates also have an acu te p ain-relieving effect that is thou ght to be d erived from the red u ction of variou s pain-prod u cing su bstances. (1:1294-6) 533. (B) The Rexed laminae is a complex of 10 layers of grey m atter located in the sp inal cord . They are labeled as I to X. Lam inae I to VI are in the d orsal horn and VII to IX are in the ventral horn. Lam ina X bord ers the central canal of the spinal cord . Lam ina I is also know n as the posterom arginal nu cleus. The neu rons in lam ina I receive inp u t m ainly from the Lissau er tract. They relay p ain and tem p eratu re sensation. Lam ina II is know n as the su bstantia gelatinosa. The neu rons contain κ- and µ -op ioid recep tors. C fibers term inate in the su bstantia gelatinosa. Lam inae I and II are fou nd along the entire spinal cord . The neurons in lam ina I p roject to the thalamus. WDR neu rons are concentrated in lam ina V. (7:17-9)

534. (B) Extrem ely high d oses of m orp hine and related op ioid s can p rod u ce seizu res, p resu m ably by inhibiting the release of GABA (at the synaptic level). N ormeperid ine, a metabolite of m ep erid ine, is p rone to p rod u cing seizu res and tend s to accu m u late in patients w ith renal d ysfu nction and in the eld erly. N aloxone m ay n ot effectively treat seizu res p rod u ced by m eperid ine. (1:494; 5:1302) 535. (D ) Wind up refers to the progressive increase in the m agnitu d e of C-fiber evoked resp onses of d orsal horn neurons prod uced by rep etitive activation of C-fibers. N euronal events lead ing to w ind up also prod u ce som e of the classical characteristics of central sensitization inclu d ing expansion of receptive field s and enhanced resp onses to C bu t not A-δ fiber stim u lation. (7:22) 536. (D ) The carotid tu bercle (Chassaignac tu bercle) lies 2.5 cm lateral to the cricoid cartilage. It is a p art of the transverse p rocess of the C6 vertebra and can be easily p alp ated . The carotid tubercle is an im p ortant land m ark for stellate ganglion blocks. (5:254) 537. (C) The cervical facet joints are oriented in a coronal p lane to allow for extension, flexion, and lateral bend ing. The thoracic facets are oriented ap proxim ately 20° off the coronal p lane. The lu m bar facet joints are oriented 45° off the sagittal plane. (7:285)

373

374

19: Acute a nd Chronic Pa in

538. (D ) Vertebrop lasty is best u sed for acute vertebral fractu re w here bone cem ent is p ercu taneously injected into a fractured vertebra in ord er to stabilize it. Altern atively, kyp h op lasty involves p lacem ent of a balloon into a collap sed vertebra, follow ed by injection of bone cem ent to stabilize the fracture. It is not clear if one proced ure has an ad vantage over the other. Both procedures may produce almost immediate pain relief. They are ind icated for p ainfu l com pression fractu res becau se of osteop orosis and m etastatic tu m ors. (5:1610) 539. (A) The nonop ioid analgesic ziconotid e has been d eveloped as a new treatment for patients w ith severe chronic p ain w ho are intolerant of and / or refractory to other analgesic therap ies. Ziconotid e is the synthetic equ ivalent of a 25-am ino-acid p olybasic p ep tid e fou nd in the venom of the m arine snail Conus magus. In rod ents, ziconotid e acts by bind ing to neuronal N -typ e voltage-sensitive calciu m channels, thereby blocking neu rotransm ission from prim ary nocicep tive afferents. Ziconotid e p rod u ces potent antinociceptive effects in anim al m od els and its efficacy has been d em onstrated in hu m an stu d ies. (1:514) 540. (D ) In the viscera, sympathetic nerve terminals, mast cells, and epithelial cells, including enterochrom affin cells in the gastrointestinal tract, release a variety of bioactive substances, includ ing norepinephrine, histamine, serotonin, ad enosine trip hosp hate (ATP), glu tam ate, nerve grow th factor (N GF), and tryp tase. Resid ent leu kocytes and m acrop hages attracted to an area of insult collectively contribute prod ucts of cyclooxygenase and lip oxygenase, inclu d ing prostagland in I2, prostagland in E2, hyd roxyeicosatetraenoic acid s (H ETEs), and hyd roperoxyeicosatetraenoic (H PETEs), and a variety of cytokines, reactive oxygen species, and grow th factors. Som e of these chem icals can d irectly activate visceral afferent terminals (e.g., serotonin, ATP, and glu tam ate), w hereas others probably play only a sensitizing role (e.g., prostagland ins, N GF, and tryptase). (1:514)

541. (B) Pretreatment w ith an N MDA antagonist attenuates the central sensitization from inflammation. (7:24) 542. (D ) Lu m bar sym p athetic blockad e has been show n to be effective in all of the p ain synd rom es listed excep t lu m bar facet synd rom e. H eat, COX inhibitors, and facet injections m ay be u sefu l. (1:1445-8; 5:1486) 543. (B) NMDA receptor activation causes increased pain transmission w hereas its blockad e attenuates p ain transm ission. There are fou r recep tor typ es for glu tam ate and asp artate in the som atosensory system . The class of receptors best activated by N MDA is term ed the N MDA recep tor. The N MDA recep tor is u su ally consid ered as recru ited only by intense and / or prolonged somatosensory stim uli. This characteristic is d u e to the N MDA recep tor ’s w ellknow n m agnesiu m block that is only relieved by p rolon ged d ep olarization of th e cell m em brane. (7:24) 544. (D ) Methad one, u nlike morphine, is metabolized through N-d emethylation by the hepatic cytochrome P450 enzyme system whose activity can vary widely in different people. Methadone should be administered with caution in patients receiving multiple medications, especially antivirals and antibiotics. Methad one’s withdrawal sym ptom s tend to be less severe than m orp hine’s; this and its long d u ration of action, good oral bioavailability, and high p otency mad e it the maintenance d rug or d etoxification treatm en t of ch oice in op ioid ad d iction . Methadone has biphasic elimination. A long terminal phase (ranging from 30-60 h) producing sedation and respiratory depression can outlast the analgesia that correlates with the redistribution phase (6-8 h). This biphasic pattern explains why methadone is used once a day for opioid maintenance therapy and every 4-8 h for analgesia. Rapid titration is not possible making this d ru g m ore u sefu l for chronic p ain. (5:329-30; 704; 718)

Answe rs : 538–548

545. (A) The u se of an intrap leu ral catheter can be effective for u nilateral p ostop erative p ain su ch as that follow ing cholecystectom y. A p neu m othorax can occu r d u ring catheter p lacem ent. Since only local anesthetic is injected , the use of p arenteral op ioid s is not contraind icated . (1:1327; 5:1829) 546. (A) End om etriosis is the m ost com m on cau se of chronic pelvic p ain in w om en. It is characterized by the p resence of uterine end om etrial tissu e ou tsid e the uterus, m ost com m only in the pelvic cavity. The d isord er m ainly affects w om en of rep rod u ctive age. End om etriosis has been d escribed as a pelvic inflam m atory p rocess w ith altered fu nction of im m u ne cells and an increased nu m ber of activated m acrop h ages in the p eritoneal environm ent that secrete various local prod u cts su ch as grow th factors an d cytokin es. En d om etriosis is estrogen-d epend ent and trad itional treatments have aim ed to d ecrease p rod u ction of estrogens su ch as estrad iol. H ow ever, the exact m echanism by w hich estrogens promote end om etriosis is u nclear and su ppression of estrogens has variable effects. End ometriotic lesions them selves secrete proinflam m atory cytokines su ch as interleu kin-8 (IL-8) that recru it m acrop h ages and T-cells to the p eritoneu m and m ed iate inflam m atory responses. (6:387-8) 547. (C) The tw o op erational criteria are chronic w id espread pain (CWP) defined as pain in all four quadrants of the body and the axial skeleton for at least three months, and the finding of pain by 4-kg pressure on d igital palpation of at least 11 of 18 d efined tender points. The exact pathogenesis of fibromyalgia has not been elucid ated yet, but accord ing to the currently held view a variety of biological, psychological, and social factors play a role in the manifestation of the d isord er. Am ong other things, inflam m atory, traumatic, and immunological processes;

375

static p roblem s; end ocrine d isord ers; and d epression and anxiety d isorders and stress factors are thought to trigger the synd rome. A d ysfunction of the central affective and/ or sensory pain memory may possibly be at w ork in the d ifferent illnesses mentioned above that then results in fibromyalgia pain. In principle, fibromyalgia can be categorized as primary or second ary fibromyalgia. In primary fibromyalgia, w hich is much more common than the secondary type, even the most careful work-up will not reveal any definitive organic factors triggering the syndrome. With secondary fibromyalgia, on the other hand , the und erlying d isease, such as inflammatory rheumatic processes or collagenosis can be d iagnosed w ith relative ease. Symptoms associated w ith fibromyalgia often d o not d isappear when the rheumatic processes have su bsid ed , suggesting that som e central mechanisms may be responsible for the persistence of generalized pain and hyperalgesia, possibly d ue to a d isord er of the central affective pain memory and / or the memory of sensory pain or else to latent peripheral immunological processes. It is precisely this coexistence of pain and hyperalgesia in second ary fibrom yalgia associated w ith system ic inflam m atory rheumatic d iseases that proves that pain and sensitivity to pain cannot be separated strictly in fibromyalgia. (6:2849-52) 548. (C) Most p u blished gu id elines for the treatm ent of cancer p ain inclu d e the concep t of d osing m ed ications, includ ing opioid s, to ad equ ately treat pain on an around the clock basis. Respiratory d epression is u nu sual in opioid tolerant p atients. The u se of ad ju vant d ru gs includ ing antid epressants and anticonvulsants m ay be u sefu l. Sid e effects su ch as nau sea and constipation shou ld be anticipated and treated . (1:1451; 5:2770)

376

19: Acute a nd Chronic Pa in

549. (C) Reactivation of the varicella-zoster virus can cause d ermatomal pain w ithout a rash in a process termed ”zoster sine herpete.” This d iagnosis cannot be m ad e on the basis of clinical presentation alone and w ould require evid ence of concurrent viral reactivation. Zoster reactivation typically occurs only once for an individ u al. Atyp ical m anifestations that occu r in immunocompromised patients includ e a prolonged course, recurrent lesions, and involvem ent of m u ltip le d erm atom es. Diagnostic laboratory tests are recommend ed when herpes simplex must be ruled out (e.g., recurrent rash or sacral lesions) and for patients w ith atypical lesions. Until recently, these d efinitions have been arbitrary, but the results of recent research now provid e support for the valid ity of d istingu ishing betw een three p hases of p ain in affected and ad jacent d ermatomes: (1) herpes zoster acu te pain (also termed acu te herpetic neuralgia), defined as pain that occurs within 30 days after rash onset; (2) subacute herpetic neuralgia, defined as pain that persists beyond the acute phase but that resolves before the d iagnosis of PH N can be mad e; and (3) PH N, d efined as pain that persists 120 d ays or more after rash onset. It can also be pred icted that the number of adults developing herpes zoster in the United States may increase as a consequence of reduced opportunities for subclinical immune boosting resulting from near-universal varicella vaccination of child ren. Recent d ata show ing an increase in herpes zoster in the United States are consistent w ith this pred iction. An increase in the incidence of herpes zoster could be offset by zoster vaccination, bu t the extent to w hich widespread herpes zoster vaccination will occur is presently unknow n. (7:417-9) 550. (C) A vaso-occlu sive crisis m ost com m only involves the back, legs, knees, arms, chest, and abd om en. The p ain generally affects tw o or more sites. Bone pain tend s to be bilateral and sym m etric. Recurrent crises in an ind ivid ual p atient u su ally have the sam e d istribu tion. Acute pain in patients w ith sickle cell d isease is caused by ischemic tissue injury resulting from the occlusion of microvascular bed s by sickled erythrocytes d uring an acute crisis. Acute bone pain from microvascular occlusion is a common

reason for em ergency d ep artm ent visits and hospitalizations in patients w ith sickle cell d isease. Obstru ction of blood flow resu lts in regional hyp oxem ia and acid osis, creating a recu rrent p attern of fu rther sickling, tissu e injury, and pain. The severe pain is believed to be cau sed by increased intram ed u llary p ressure, especially w ithin the juxtaarticular areas of long bones, second ary to an acute inflammatory response to vascular necrosis of the bone marrow by sickled erythrocytes. The pain may also occur because of involvement of the periosteum or periarticular soft tissue of the joints. When a vaso-occlusive crisis lasts longer than 7 d ays, it is important to search for other causes of bone pain, such as osteomyelitis, avascular necrosis, or compression d eformities. When a recu rrent bone crisis lasts for w eeks, an exchange transfusion may be required to abort the cycle. Patients w ith sickle cell d isease have a higher incid ence of vaso-occlu sive d isease than patients w ith β-thalassemia. (6:854-6) 551. (D ) CRPS I and CRPS II are clinically ind istingu ish able. Th e on ly d ifferen ce is th at in CRPS II there is evid ence of m ajor nerve d am age. (7:408) 552. (B) Opioid receptors are cou pled to G proteins that affect p rotein p hosp horylation via a secon d m essen ger, th ereby alterin g th e con d u cta n ce of p ota ssiu m a n d ca lciu m ion ch an nels. These are believed to be the m ain m echanism s by w hich end ogenou s and exogenou s op ioid s p rod u ce analgesia. The opening of p otassiu m channels inhibits the release of neu rotransm itters, inclu d ing su bstance P and glu tam ate, if the recep tors are p resynap tic. Opioid s also inhibit neu ronal firing by hyp erpolarization of the cell if the recep tors are p ostsynaptic on the neurons. (1:490-1) 553. (D ) Tw o types of pain receptors are prim arily activated by nocicep tive inp u t. These inclu d e low -threshold nocicep tors that are connected to fast cond u cting A-δ pain fibers, and highthreshold nociceptors that cond uct im pulses in slow, unm yelinated C fibers. Within the d orsal horn of the sp inal cord , these pain fibers synap se w ith sp inal neu rons. N eu rotransm itters

Answe rs : 549–555

(e.g., glu tam ate and su bstance P) are able to m od u late the postsynaptic responses w ith fu rther transm ission to su p rasp inal sites (thalam us, anterior cingulate cortex, insu lar cortex, and som atosensory cortex) via the ascend ing p athw ays. The sim p lest form of p lasticity in the nervou s system is that rep eated noxiou s stimulation may lead to habituation (d ecreased response) or sensitization (increased response). Prolonged or strong activity of d orsal horn neurons caused by repeated or sustained noxiou s stim u lation m ay su bsequ ently lead to increased neuronal responsiveness or central sensitization. N europlasticity and su bsequent CN S sensitization includ e altered function of chemical, electrophysiological, and pharmacological systems. These changes cause exaggerated perception of painful stimuli (hyperalgesia), a p ercep tion of innocu ou s stim u li as painfu l (allod ynia), and m ay be involved in the generation of referred p ain an d hyp eralgesia across m u ltip le sp inal segm ents. While the exact m echanism by w hich the sp inal cord becom es sensitized or in a ”hyp erexcitable” state cu rrently rem ains som ew hat u nknow n, some contribu ting factors have been proposed . Wind u p refers to a central spinal m echanism in w hich rep etitive noxiou s stim u lation resu lts in a slow tem p oral su m m ation that is exp erienced in hu m ans as increased p ain. In 1965, anim al experim ents show ed for the first tim e that repetitive C-fiber stim u lation cou ld result in a p rogressive in crease of electrical d ischarges from the second -ord er neu ron in the sp inal cord . This mechanism of pain am plification in the sp inal cord is related to tem poral su m m ation of second pain or w ind up . Second p ain, w hich is of d u ll qu ality and strongly related to chronic p ain states, is transm itted throu gh unm yelinated C fibers to d orsal horn n ocicep tive n eu ron s. Du rin g the C-fiber ’s tran sm itted stim u li, N MDA recep tors of second -ord er neu rons becom e activated . It is w ell know n that N MDA activation induces calciu m entry into the d orsal horn neu rons. Calciu m entry into sensory neu rons in the d orsal horn ind u ces activation of nitric oxid e (N O) synthase, lead ing to the synthesis of N O.

377

N O can affect the nocicep tor term inals and enhance the release of sensory neu rop ep tid es (in p articu lar, su bstance P) from p resynap tic neurons, therefore contributing to the d evelop m ent of hyp eralgesia and m aintenance of central sensitization. Su bstance P is an im p ortant n ocicep tive neu rotransm itter. It low ers the threshold of synaptic excitability, resu lting in the u nm asking of norm ally silent interspinal synapses and the sensitization of second -ord er sp inal neu rons. Fu rtherm ore, su bstance P can extend for long d istances in the spinal cord and sensitize d orsal horn neu rons at a d istance from the initial inp u t locu s. This resu lts in an expansion of receptive field s and the activation of w id e d ynam ic neu rons by nonnocicep tive afferent im p u lses. (7:13-24) 554. (C) Glossopharyngeal neuralgia is a d isord er characterized by intense pain in the tonsils, mid d le ear, and back of the tongue. The pain can be interm ittent or relatively p ersistent. Sw allow ing, chew ing, talking, sneezing, or eating spicy food s may trigger the d isord er. It is often the result of compression of the 9th nerve (glossopharyngeal) or 10th nerve (vagus), but in some cases, no cause is evid ent. Skull base surgery or su rgeries in the infratem poral region may result in d amage or irritation of the glossopharyngeal nerve. Conservative treatm ent inclu d es u sing anticonvu lsants. In refractory cases, glossopharyngeal nerve block m ay be helpful. Rad iofrequency lesioning or neurolytic treatment should be reserved for resistant cases or ones associated w ith head and neck cancer. Surgical d ecompression should be reserved for nonrespond ers and resistant cases. (7:248) 555. (B) Transcutaneous electrical nerve stimulation (TEN S) p rovid es a ”tingling” sensation that is not u su ally p erceived as p ain. Patients shou ld be instru cted in the u se of the stim u lator. It has the ad vantages of allow ing the patient to participate in his or her therap y and the lack of opioid -ind uced sid e effects. The presence of a card iac pacemaker is consid ered a relative contraind ication. (1:1424; 7:2777)

378

19: Acute a nd Chronic Pa in

556. (E) In the early 1990s, a p anel of exp erts reached a consensu s that the term s ”reflex sym p athetic d ystrop hy” and ”causalgia” had lost their u tility as clinical d iagnoses and su ggested a new nom enclatu re be ad op ted . The new term s d esignated for these cond itions are ”CRPS typ es I and II”. Accord ing to the new d iagnostic criteria, CRPS need not be m aintained by sym p athetic m echanism s. A threep hase isotop e bone scan is often p ositive in CRPS, but a normal bone scan d oes not exclude the d iagnosis. Erythrocyte sed im entation rate is a nonsp ecific test that is p ositive in m any painful cond itions inclu d ing infection, inflam m atory arthritid es, and inflam m atory m yopathies. As a synd rom e, CRPS is d iagnosed by history and physical exam ination. For CRPS I, the d iagnostic criteria inclu d e (1) an initiating noxiou s event; (2) sp ontaneou s p ain and / or allod ynia occu rring ou tsid e the territory of a single peripheral nerve that is/ are d isproportionate to the inciting event; (3) there is or has been evid ence of ed em a, cu taneou s p erfu sion abnormalities, or abnormal sud om otor activity in the region of p ain since the inciting event; and (4) the d iagnosis is exclu d ed by the existence of any cond ition that w ou ld otherw ise account for the degree of pain and dysfunction. (7:410-1) 557. (D ) Ow ing to its high incid ence, stroke is the lead ing cau se of central p ain in the ind u strialized w orld . The chance of d evelop ing central pain follow ing spinal cord injury is higher than after stroke (30%-50% vs 8%), but the overall nu m ber of stroke patients w ith central p ain is higher. Syringom yelia is the d isord er w ith the highest incid ence of central p ain (60%-80%). Accord ing to neurosu rgical stu d ies cond u cted by V. Cassinari and C.A. Pagni in the 1960s, inju ry to sp inothalam ocortical p athw ays is necessary bu t not su fficient to cau se central pain. The reason w hy som e p atients d evelop central pain bu t others w ith id entical inju ries d o not is u nknow n. Central p ain m ay occu r after neu rosu rgical p roced u res and intracranial hem orrhage, bu t these are u nu sual occu rren ces. There are n ow several p rosp ective stud ies show ing motor cortex stimulation to be an effective treatm ent for central pain. There is

no typical presentation for central p ain. While sp ontaneou s p ain is alm ost u niversal, allod ynia also affects a m ajority of central p ain p atients. The tim e lag betw een the inju ry and onset of pain, and the location of central pain are extrem ely variable. (7:394-400) 558. (A) An ind ication for ep id u ral steroid injections is nerve root irritation and accom panying inflam m ation. The u se of ep id u ral steroid s in low back p ain is an area of con troversy. H ow ever, m ost experts agree that a trial of epid u ral steroid s m ay be ind icated in rad icu lar p ain of less than 12 m onths d u ration. Ep id u ral steroid injections have been fou nd to be ineffective in the relief of fibrom yalgia, ankylosing sp ond ylitis, and fu nction al low -back p ain. (1:1443, 1448) 559. (B) Celiac plexus block is both a d iagnostic and therap eu tic tool to help in m anaging u p p er abd ominal pain arising from viscera. Pancreatic cancer is the lead ing d iagnosis requ iring neurolytic celiac p lexu s block; other cond itions m ay includ e visceral p ain arising from m alignancies of the liver or GI tract. The p roced ure is p erform ed either u nd er flu oroscop ic gu id ance or CT scan, thou gh blind ap p roaches have also been d escribed . Both single transaortic as w ell bilateral need le ap p roaches have been d escribed . Com p lications inclu d e d iarrhea, hyp otension, genitofem oral neu ralgia, infection, bleed ing, d am age to su rrou nd ing stru ctu res, and rarely paralysis. All com plications m entioned above m ay occu r excep t hypertension. (7:703) 560. (C) Selective cervical nerve root injection m ay be indicated for diagnosis and treatment of cervical rad icu lopathy. Com plications other than infection, bleed ing, and nerve d am age inclu d e intravascu lar u p take into the vertebral artery or rad icular arteries resulting in seizure, stroke, or paraplegia. Intraspinal spread into the epid u ral sp ace or intrathecal spread is also possible, resu ltin g in h igh sp inal anesthesia. Dam age to th e sp in al cord has also been rep orted w ith injection into the sp inal cord . Consid ering the life-threatening com p lications, selective cervical nerve root block should

Answe rs : 556–569

only be perform ed by physicians w ell versed in this techniqu e. (5:853-4; 7:741-3) 561. (B) Oxcarbazep ine is an analogu e of carbam azep ine w ith a keto grou p at the carbon-10 position. It is roughly 50% protein bound in the p lasm a. The d ose shou ld be d ecreased at least by half if the patient has significant renal insufficiency. The m ost frequ ent ad verse effects exp erienced inclu d e d izziness and vertigo, w eight gain and ed em a, GI symptoms, fatigue, and allergic-type reactions. Allergic cross-sensitivity to carbam azep ine occu rs in abou t 25% of p atients and m ay be severe. (5:1560) 562. (E) SCS has been u tilized by clinicians for a variety of chronic pain issues. Although a large bod y of w ork has been p u blished , p recise m echanism s of action of SCS rem ain elu sive. An im al stu d ies su ggest th at SCS triggers release of seroton in , su bstan ce P, an d γ -am inobu tyric acid (GABA) w ithin the sp inal cord d orsal horn. Pain resp ond s best to SCS w hen it is w ell localized , such as rad icular pain in the u p p er or low er extrem ities. Diffu se p ain m ay be im p ossible to treat w ith this m od ality becau se of the inability to get effective coverage w ith stimulation using SCS. (5:1595; Kumar K, et al., Pain 2007; 132:179-88; Deer TR, et al., Curr Pain Heache Rep 2009; 131:18-23; Kemler M A , et al., J N eurosurg 2008; 108:292-8) 563. (B) All of the options listed are im portant for the p atient w ith com p lex regional p ain syn d rom e. In the p atient w ith obstru ctive lu ng d isease, one m ay w ant to avoid the stellate ganglion approach, since there is a possibility of pneumothorax that could be life-threatening. A lu m bar sym p athetic block w ou ld not be u sed for p ain in the u pper extrem ity. (1:1446) 564. (A) The follow ing are the u su al featu res of som atic pain: w ell localized , sharp and d efinite, often constant (som etim es period ic); it is rarely associated w ith nau sea excep t w hen it is d eep som atic pain w ith bone involvement; and it m ay follow the d istribu tion of a som atic n erve. In contrast, visceral p ain: is p oorly localized , d iffu se, d u ll, and vagu e; it is often

379

p eriod ic and bu ild s to peaks (som etim es constant); and it is often associated w ith nausea and vom iting. (5:1601-2) 565. (B) It has been d em onstrated that early postoperative p ain is a significant p red ictor of longterm pain. The rest of the answ ers are correct. (5:1299) 566. (B) PH N affects w om en m ore often than m en, in a ratio of ap p roxim ately 3:2. The rest of the answ ers are correct. (5:417-9) 567. (A) The liver receives the major insult from acetaminophen toxicity. When very large doses of acetam inophen exceed the liver ’s capacity to conjugate it to glu cu ronic acid or sulfate, it u nd ergoes m etabolism by CYP2E1 to a toxic m etabolite that is scavenged by glu tathione. Hepatic necrosis ensues when the supply of glutathione is exhausted . Exogenous ad ministration of the glutathione analogue N-acetylcysteine scavenges the reactive metabolite and decreases the degree of cell death. N-acetylcysteine should be started within 36 h for greatest effectiveness. (1:983-4; 5:191) 568. (E) Opioid s prod uce a d ose-d ependant respiratory d epression by acting d irectly on the respiratory centers in the brainstem . Partial agonist and agonist-antagonist op ioid s are less likely to cau se severe respiratory d epression, as are the selective κ-op ioid agonists. Therap eu tic d oses of m orphine d ecrease minute ventilation by d ecreasing resp iratory rate (as op p ose to tid al volu m e). Opioid s d ep ress the ventilatory resp on se to carbon d ioxid e; th e carbon d ioxid e–resp onse cu rve show s a d ecreased slope and rightw ard shift. The ap neic threshold is d ecreased and the increase in ventilatory resp onse to hypoxem ia is blu nted by opioid s. N aloxone can effectively and fu lly reverse the resp iratory d ep ression from op ioid s. (5:710) 569. (A) Alcohol has a faster onset of action bu t a greater tend ency to p rod u ce neu ritis. It is u sed in concentrations of 50% to 100%. Phenol is u sed in concentrations of 5% to 20%. (1:1453)

380

19: Acute a nd Chronic Pa in

570. (D ) Cu rrent p ain theories involve the sm all and large nerves and a path through the d orsal root. Both inhibitory and excitatory nerves are involved . (1:1403; 5:2730; 7:2346) 571. (B) The first statem ent better d escribes trigem inal neu ralgia. Clu ster head ache affects m ore m ales than fem ales w ith a 5:1 ratio and can begin at any age. Attacks are severe, stabbing, screw ing, u nilateral p ain, occasionally p reced ed by premonitory symptoms, w ith sud d en onset and rapid crescend o. Therap eu tic interventions for the acu te attack inclu d e oxygen, triptans, d ihyd roergotam ine, ketorolac, chlorprom azine, or intranasal lid ocaine, cocaine, or cap saicin. Melatonin has been fou nd to be m od erately effective as a preventive treatm ent in ep isod ic an d ch ron ic clu ster h ead ach e. (7:212-4) 572. (C) PDPH and SIH are tw o d istinct clinical entities w ith sim ilar presentations. The head ache is alw ays bilateral, located in the occip ital and / or frontal area. Althou gh low CSF p ressu re is often noted , it is not necessary to confirm the d iagnosis. (7:242) 573. (D ) The major theoretical com plications of glucocorticoid ad m inistration inclu d e su p p ression of p itu itary-ad renal axis, hyp ercorticism , Cu shing synd rom e, osteop orosis, avascu lar necrosis of bone, steroid m yopathy, ep id u ral lip om atosis, w eight gain, flu id retention, and hyp erglycem ia. (7:658-9) 574. (D ) Chronic p ain is a m ajor com p lication of SCI. Ep id em iologic stu d ies in d icate th at ap p roxim ately tw o-third s of all SCI p atients suffer from chronic pain out of w hich one-third have severe pain. Pain interferes w ith rehabilitation, d aily activities, qu ality of life, and m ay have significant influ ence on m ood lead ing to d epression and even su icid e. (7:394-5) 575. (D ) Th e d iagn osis of CRPS I an d II follow s th e IASP clin ical criteria. Bon e scin tigrap h y m ay be a valu able tool to ru le ou t oth er con d itions. CRPS I is m ore com m on than CRPS II an d th e fem ale to m ale ratio is from 2:1 to 4:1. (7:405-14)

576. (B) In treating chronic pain, one m u st rem em ber that the p atient’s p ercep tion of the p ain m ay not be as bad as the loss of bow el and u rinary control and the absence of sensation. This m u st be thorou ghly d iscu ssed w ith the p atient prior to p roceed ing. Most pain specialists believe that neu rolytic blocks shou ld be reserved for the p atien t w ith a sh ort life exp ectancy. (1:1453-4) 577. (E) The p rem ise behind intrathecal d ru g d elivery is that by d irectly d ep ositing d ru gs into the CSF, the first-pass effect is avoid ed . Intrathecal m orp hine is 300 tim es m ore effective than oral m orp hine on a d ose basis. From sp inal to epid u ral m orp hine the conversion is in the ratio of 1:10. From epid u ral to IV m orp hine the conversion is in the ratio of 1:10. From IV to oral m orp hine the conversion is in the ratio of 1:3, hence 10 × 10 × 3 = 300. By the d irect action of th e m ed ication , th e n u m ber of CN Sassociated sid e effects can be red u ced . There is how ever, system ic u ptake of d ru gs ad m inistered via the intrathecal route su ch as opioid s, bu t in d oses too low to resu lt in analgesia. This is d ifferent from ad m inistration of opioid s into the epid u ral space, w here in the case of fentanyl for exam p le, an algesia is m ed iated throu gh system ic u p take and su p rasp inal effects. (1:20-3; 5:798) 578. (C) H igh d oses of any op ioid red u ce sym p athetic ou tp u t allow ing the p arasym p athetic ou tp u t to p red om in ate. Th e h eart rate d ecreases by stim u lation of the vagal center, especially w ith high d oses. Meperid ine has an atrop ine-life effect and m ay elevate the heart rate after IV ad m inistration. (5:711) 579. (B) The stellate ganglion is formed by the fusion of the inferior cervical ganglion resting over the anterior tubercle of C7 and first thoracic ganglion resting over the first rib. (7:696-7) 580. (D ) Tram ad ol hyd rochlorid e is a centrallyacting analgesic that is thou ght to p rovid e analgesia via at least tw o m echanism s: som e analgesia m ay be d erived from the relatively w eak interaction of tramad ol w ith the µ -opioid recep tor. The second and m ajor m echanism ,

Answe rs : 570–587

w hich is thought to accou nt for at least 70% of tram ad ol’s analgesic activity, is via inhibiting the reu ptake of norepinephrine and serotonin. (5:718) 581. (E) Paroxysm al hem icranias m ay be chronic (CPH , e.g., d aily) or ep isod ic (EPH , e.g., d iscrete head ache period s separated by period s of rem ission) characterized by severe, excru ciating, throbbing, boring, or p u lsatile p ain affecting the orbital, su p raorbital, and tem p oral regions. The pain tend s to be associated w ith at least one of the follow ing signs or symptoms ip silateral to the p ainfu l sid e: conju nctival injection, nasal congestion, lacrim ation, ptosis, rhin orrh ea, or eyelid ed em a. Attacks m ay occur at any time, occasionally w aking patients from sou nd sleep and tend to last for 2-25 m in (althou gh they m ay linger for hou rs). The p atient generally has 1 to 40 attacks p er d ay. Loss of sight is not associated w ith these head aches. (7:221; 256) 582. (E) Accord ing to trad itional Chinese m ed icine, ”qi” (p ronou nced ”chee”) is the life force or energy that flow s throu gh all living things. ”Qi” affects ou r bod y at physical, mental, emotional, and sp iritu al levels. Any im balance (d eficiency or excess) or blockage of ”qi” m ay resu lt in d isease or p ain. Acup unctu re treats d isord ers by influ encing the flow of ”qi,” thu s restoring the norm al balance of organ system s. Accord ing to a 1998 N IH consensus panel, acu p u nctu re m ay be includ ed in a com p rehensive m anagem ent p rogram for the treatm ent of a variety of cond itions su ch as head ache, fibrom yalgia, lateral epicond ylitis (”tennis elbow ”), m yofascial p ain, low back p ain, and osteoarthritis. (5:1569) 583. (A) Ronald Melzack and Patrick Wall p u blished the land m ark gate control theory in the jou rn al Science in 1965. Accord ing to this theory, large and sm all fibers p roject to the su bstantia gelatinosa. The su bstantia gelatinosa exerts an inhibitory effect on afferent fibers. Large fibers increase the inhibitory effect, ”close the gate,” and d ecrease the afferent pain signal. Small fibers d ecrease the inhibitory effect, ”op en the gate,” and increase the

381

afferent p ain signal. This gate control theory is com m only cited as the m echanism of action of SCS, bu t a 2002 review conclu d es that other m echanism s m ust also play a role. (Oakley JC, Prager JP, Spine 2002; 27:2574-83; M elzack R, Wall PD, Science 1965; 150:3699) 584. (C) Asp irin covalently acetylates COX-1 and COX-2, irreversibly inhibiting COX activity. This m akes the d u ration of asp irin’s effects related to the tu rnover rate of COX in d ifferent target tissu es. Other COX inhibitors com p etitively in h ibit th e active site of COX en zym es th at relate th eir d u ration m ore d irectly to the time course of drug d isposition. (1:964) 585. (E) Traditional nonsteroidal antiinflammatory drugs inhibit the prostaglandin G/ H synthase enzymes, colloquially known as COX, therefore inhibiting the synthesis of prostaglandin E, prostacyclin, and throm boxane. They inhibit the activity of not only COX-2 but also COX-1. Glucocorticoids inhibit phospholipase A2. (1:959) 586. (D ) Methad one has a variable oral bioavailability betw een 41% and 99% and , therefore, shou ld be started w ith extra cau tion (low initial d ose and slow su bsequ ent increases). Methad one d iffers from all other opioid s by its n on com p etitive an tagon ist activity at the N MDA recep tors. Activation of N MDA recep tors has been show n to p lay a role in d evelopm en t of toleran ce to analgesic effects of opioid s, as w ell as in the pathologic sensory states, such as neuropathic pain, inflam m atory p ain, isch em ic p ain, allod yn ia, an d sp inal states of hyp ersensitivity. (1:506-7) 587. (D ) Both are clinically u sed N MDA recep tor antagonists that prod u ce analgesia. Clinically available com p ound s that are d em onstrated to h ave N MDA recep tor-blockin g p rop erties in clu d e ketam in e, d extrom eth orp h an , an d m em antine. Dextrom ethorphan, for exam ple, is effective in the treatm ent of p ainfu l d iabetic neuropathy and not effective in p ostherpetic neu ralgia and central p ain. N MDA recep tor blockers may therefore offer new options in the treatm ent of pain. (1:417; 1778)

382

19: Acute a nd Chronic Pa in

588. (C) The COX inhibitors are w eak organic acid s, consisting of one or tw o arom atic rings connected to an acid ic fu nctional grou p . They d o not cross the blood –brain barrier, are 95% to 99% bou nd to albu m in, are extensively m etabolized by the liver, and have low renal clearance (< 10%). They act m ainly in the perip hery, bu t they m ay have a central effect. COX-2 ind u ction w ithin the sp inal cord m ay p lay an im p ortant role in central sensitization. The acu te antihyp eralgesic action of COX inhibitors has been show n to be m ed iated by the inhibition of constitutive spinal COX-2 that has been fou nd to be u p regu lated in resp onse to inflam m ation and other stressors. (5:1302-4) 589. (A) Occip ital neu ralgia involves the greater occipital nerve and lead s to a chronic head ache that m ay extend to the shou ld er or forw ard to the area arou nd the eye. It m ay be d ue to com pression of the occipital nerve w ithin the sku ll. The scap u lar nerve is not involved nor is the cervical plexu s. Occipital nerve block m ay be u sefu l in d iagnosis and treatm ent. (1:721) 590. (B) The tricyclic antid epressants (TCA’s) can be d ivid ed into tertiary am ines and their d em ethylated second ary am ine d erivatives. Exam ples of tertiary TCA’s inclu d e am itrip tyline, im ip ramine, clomipramine, and d oxepin. Examples of second ary TCA’s inclu d e nortrip tyline, d esipram ine, and protrip tyline. Trazod one is an atyp ical antid epressant. (1:404) 591. (A) All of choices listed are thought to be inhibitory m od u lators in the d orsal horn of the spinal cord w ith the exception of substance P. Substance P is found in the synaptic vesicles of u nm yelinated C fibers. It has been show n to aggravate pain. (1:1404)

be elicited by p alp ating the m u scles near the affected area. A m ainstay of treatm ent is the injection of local anesthetic into the ”trigger p oint.” (1:1445; 5:2772) 594. (B) N ociceptive information travels via the spinothalam ic tract. The cell bod y of the p rim ary afferent is located in the d orsal horn. From there the im pulse travels via the spinothalamic tract. Branches m ay synap se in areas of the brain stem , inclu d ing the p eriaqu ed u ctal gray and the nu cleu s rap he m agnu s. The thalam u s and sensory cortex are also im p ortant in the p ain p athw ay. (1:1404; 5:2706) 595. (C) Gabap entin has a chem ical stru cture sim ilar to GABA. It seem s not to act d irectly at the GABA-bind ing site in the CN S, how ever. The m echanism of action is still u nclear. It m ay enhance the release or activity of GABA and seem s to inhibit voltage-d ep end ent sod iu m channels. (1:599) 596. (C) The d em and d ose, lockou t interval, and one-hour limit should be ind ivid ually ad justed accord ing to the p atient’s p hysiologic statu s and requ irem ent for analgesia. (5:2734; 7:2330) 597. (E) The size of the need le u sed to d ep osit the neu rolytic agent is not im p ortant. What is im p ortant are the location of the injection (p roxim ity to the nerve), the concentration of the neu rolytic agent, the volu m e of the agent, and the histology of the nerve. Sm aller nerves are easier to block than larger nerves. (1:1453; 5:2776)

592. (D ) Cutaneous nociceptors are defined by the fiber type and the type of stimuli to which they respond. Merkel’s disks and Meissner ’s corpuscles are touch receptors. Pacinian corpuscles and Golgi-Mazzoni endings sense pressure. (1:1435; 7:2346)

598. (B) There is som e controversy concerning the u sefu lness of intraoperative intercostal blocks. Som e reports d id not d em onstrate any d ifference in postoperative ventilation. Most cu rrent au thors have fou nd that the p ostop erative cou rse is easier and shorter if blocks are d one at the tim e of the thoracotom y. Most u se bup ivacaine. There is still a d anger of a total spinal block and of ad m inistration of a toxic d ose of the local anesthetic d rug. (1:730; 5:2743)

593. (A) Sym p tom s of m yofascial p ain inclu d e p ain in a nond ermatomal d istribu tion. The p ain can

599. (A) The McGill Pain Qu estionnaire w as d evelop ed in 1975 by Ronald Melzack at McGill

Answe rs : 588–600

University in Canad a. It consists of three m ajor m easu res: pain rating ind ex, total nu m ber of w ord s chosen, and the p resent p ain intensity. This is a m u ltid im ensional scale for m easu rem ent of pain. The qu estionnaire tries to assess the three components of pain postulated by the gate theory: the sensory, the affective, and the evalu ative d im ensions. (7:73) 600. (C) The u se of hyd rop hilic op ioid s like m orphine in the epid ural space prod uces a biphasic

383

respiratory depression pattern. One portion of the initial bolu s is absorbed system ically, accou nting for the initial p hase that u su ally occurs w ithin 2 h of the bolus d ose. The second phase occurs 6-12 h later ow ing to the slow rostral spread of the remaining d rug as it reaches the brainstem. All m ed ications injected neuraxially shou ld be p reservative-free. Ep id u ral m orp hine d oes not cau se low er extrem ity w eakness. (1:513-4)

This page intentionally left blank

CHAPTER 20

Prac tic e Te s t Que s tions DIRECTION S (Qu estions 1-150): Each of the nu m bered item s or incom p lete statem ents in this section is follow ed by answ ers or by com p letions of the statem ent. Select the ON E lettered answ er or com pletion that is BEST in each case. 1. A 58-year-old m ale is u nd ergoing op en rep air of a 6.8 cm in fraren al aortic an eu rysm . Preoperative CT-angiogram d oes not show any evid ence of aortic occlu sion. Clam p ing of the d istal aorta in this p atient w ill m ost likely be follow ed by (A) (B) (C) (D) (E)

increased card iac ou tp u t d ecreased arterial blood p ressu re d ecreased system ic vascu lar resistance increased stroke volu m e stable heart rate

2. Du ring su rgery for strabism u s in an otherw ise h ealthy 3-year-old boy, you n ote that the p atient is likely d em onstrating the ocu locard iac reflex and you inform the surgeon. All of the follow ing are associated w ith the oculocard iac reflex EXCEPT (A) (B) (C) (D) (E)

sinu s tachycard ia sinu s brad ycard ia atrioventricu lar block fatigability of the reflex regional anesthesia

3. A 68-year-old fem ale is ad mitted to the ICU for m anagem ent of p eritonitis from a perforated d iverticu lu m d em onstrated on CT scan of the abd om en. H er hem od ynam ic evalu ation is consistent w ith a p ictu re of sep tic shock.

Which one of the follow ing statem ents regard ing the u se of hetastarch for volu m e exp ansion is m ost accu rate? (A) H etastarch is less likely than crystalloid solutions to cause renal insufficiency. (B) H etastarch is a synthetic colloid . (C) H etastarch is free of allergenic p otential. (D) H etastarch, w hen u sed in d ose >20 m L/ kg d oes not interfere w ith coagu lation. (E) H etastarch is m etabolized in the liver to glu cose m onom ers. DIRECTION S: Use the follow ing scenario to answ er Questions 4-5: A 27-year-old G1P0 p atient p resents at 39 w eeks gestational age for ind u ction of labor. H er p renatal cou rse w as com p licated by a history of m yasthenia gravis (MG). She is cu rrently being treated w ith pyrid ostigm ine. 4. True statem ents regard ing m yasthenia gravis in p regnancy inclu d e all of the follow ing EXCEPT (A) sm ooth m u scle and card iac m u scle are not affected by the d isease (B) the d isease affects tw ice as m any w om en as m en (C) the cou rse of MG in p regnancy varies w ith one-third show ing im p rovem ent of sym ptom s, one-third show ing w orsening of sym p tom s, and one-third show ing no change (D) there is an association betw een MG and other au toim m u ne d iseases (E) the first stage of labor is p rolonged in p atients w ith MG

385

386

20: P ra ctice Te s t

5. The anesthetic m anagem ent of this p atient inclu d es all of the follow ing EXCEPT (A) op ioid s shou ld be u sed cau tiou sly in this patient (B) neu raxial anesthesia is the p referred m ethod for pain m anagem ent d u ring labor (C) p atients w ith severe bu lbar involvem ent or resp iratory com p rom ise m ay require general anesthesia for cesarean section (D) su ccinylcholine shou ld be avoid ed d u ring em ergent cesarean section (E) sm all d oses of neostigm ine m ay be u sed for reversal of neu rom u scu lar blockad e 6. A 2-year-old child w eighing 12 kg is having a rep air of an u m bilical hernia and a right ingu inal hernia. The su rgeon asks you how m any milliliters of 0.5% bupivacaine may be u sed for local infiltration at the incision sites. Which one of the follow ing is the m ost ap propriate d ose for this p atient? (A) (B) (C) (D) (E)

3 mL 6 mL 12 m L 18 m L 24 m L

7. An 18-year-old m ale takes atenolol d aily to p revent card iac arrhyth m ias follow in g his d iagnosis of congenital long QT synd rom e. H e is sched u led for ou tp atient arthroscop ic su rgery of the knee. H e also has a history of severe rash follow ing ad m inistration of beta lactam antibiotics. Ap p ropriate p erioperative antim icrobial therapy w ou ld be w ith (A) (B) (C) (D) (E)

clind am ycin azithrom ycin levofloxacin gentam icin tetracycline

DIRECTION S: Use the follow ing figu re to answ er Question 8:

8. The figu re d ep icts w hich m od e of m echanical ventilation? (A) (B) (C) (D) (E)

Pressu re su p p ort Pressu re control Volu m e control, constant w aveform Volu m e control, d ecelerating w aveform Airw ay p ressu re release ventilation

9. The Pa CO 2 of a p atient on card iop u lm onary byp ass

Que s tions : 5–15

(A) is d eterm ined by the oxygen concentration of the fresh gas (B) is generally ad ju sted throu gh changes in fresh gas flow rate (C) is generally ad ju sted throu gh p u lm onary ventilation (D) shou ld be m aintained at less than 30 m m H g (E) is d eterm ined by the typ e of oxygenator 10. A 5-year-old child p resents for general anesthesia for closu re of a severe scalp laceration. H e ate a sand w ich 2 h before his accid ent. H e shou ld (A) have a rap id sequ ence ind u ction w ith propofol and su ccinylcholine (B) have a nasogastric tu be p assed to rem ove gastric contents before ind u ction (C) not be op erated on for 6 h (D) have vom iting ind u ced (E) be allow ed to aw aken w ith the end otracheal tu be in p lace at the end of the proced u re 11. You are seeing a p ed iatric p atient p reop eratively prior to su rgery for correction of scoliosis. In ord er to be p rop erly p rep ared for p ossible intraoperative blood loss and to p lan for appropriate monitoring, you ask the fam ily w hat type of scoliosis the patient has. All of the follow ing types of scoliosis are associated w ith large exp ected am ou n ts of in traop erative blood loss d u ring scoliosis correction EXCEPT (A) (B) (C) (D) (E)

Du chenne m u scu lar d ystrop hy sp inal m u scu lar atrop hy arthrogryp osis Marfan synd rom e id iop athic infantile scoliosis

12. Well know n resp iratory p hysiologic changes that occur at term in pregnancy inclu d e all of the follow ing EXCEPT (A) a 40% rise in resp iratory rate (B) a 45% increase in tid al volu m e (C) an increase in d ead space of 45%

387

(D) a d ecrease in resp iratory reserve volu m e of 15% (E) a com bined resp iratory alkalosis and com pensatory m etabolic acid osis 13. Anesthesia for carotid end arterectom y generally involves all of the follow ing EXCEPT (A) hyp ercap nia (B) norm al or slightly increased arterial oxygen tension (C) norm al or slightly increased arterial p ressu re (D) system ic hep arinization (E) norm otherm ia 14. The p rim ary m echanism of sp inal op ioid analgesia is via (A) activation of p resynap tic op ioid recep tors (B) activation of p ostsynap tic op ioid and GABA receptors (C) activation of op ioid recep tors on the m id brain (D) activation of op ioid recep tors on the rostral ventrom ed ial m ed u lla (E) inhibition of sm all sp inal interneu rons 15. A 3-m onth-old , fu ll term infant, w eighing 5 kg p resents for ventricu lop eritoneal shunt p lacement. The patient is asymp tom atic at this tim e. In review ing labs for this patient, you note that the hem atocrit is 30. Which one of the follow ing statem ents is tru e? (A) The hem atocrit has reached its low est p ost-d elivery valu e. (B) The p atient is anem ic and shou ld receive a blood transfu sion p rior to surgery. (C) The p atient is anem ic and su rgery should be postponed u ntil the hem atocrit rises. (D) The hem atocrit is likely low d u e to d ilu tion from IV flu id infu sions. (E) The p atient is anem ic and shou ld be started on iron sup plem entation.

388

20: P ra ctice Te s t

16. A 48-year-old , 111 kg female presents for surgical excision of a pheochromocytom a, for w hich sh e h as alread y received a 14-d cou rse of phenoxybenzam ine. The p atient is sched u led for a com bined ep id u ral and general anesthetic. The agent that should be avoid ed d uring m aintenance of anesthesia is (A) (B) (C) (D) (E)

nitrou s oxid e p rop ofol d esflu rane isoflu rane sevoflu rane

17. Wh en ven tilatin g th e p atien t w ith a h ead inju ry, all of the follow ing statem ents are tru e EXCEPT (A) the p atient shou ld be kep t su p ine (B) p rolonged hyp erventilation has d im inished efficacy in red u cing ICP (C) PEEP m ay be ap p rop riate (D) hyp oxia and hyp ercarbia shou ld be avoid ed (E) cou ghing shou ld be m inim ized 18. Esm olol (A) is a β 1-ad renocep tor agonist (B) has a half-life of 4 h (C) is contraind icated in the p atient w ith AV block (D) is m ore likely than p rop ranolol to cau se bronchosp asm (E) is m etabolized in the liver 19. In the ad u lt, the tracheobronchial tree (A) d ivid es at an u neven angle, m aking foreign bod ies m ore ap t to go to the left sid e (B) d ivid es into right and left bronchi, the left bronchu s being narrow er and longer (C) d oes not m ove w ith resp iration (D) is lined w ith squ am ou s ep itheliu m (E) is p rotected by circu lar cartilaginou s rings throu ghou t

20. Which one of the follow ing is m ost likely to resu lt in an u nd erestim ation of the anion gap in critically ill p atients? (A) (B) (C) (D) (E)

H yp erchlorem ia Lactic acid osis H yp erp hosp hatem ia H yp oalbu m inem ia H yp ocap nia

21. Mannitol m ay lead to subd ural hem atom a by (A) (B) (C) (D) (E)

increasing cerebral ed em a interfering w ith the clotting m echanism s lead ing to d isru p tion of cortical veins p rod u cing hyp ertension d irect p assage throu gh the vein w all

22. A p atient is referred to you w ith facial pain. Which one of the follow ing statem ents is tru e? (A) The pain of glossopharyngeal neu ralgia is very sim ilar to that of trigem inal neu ralgia bu t affects the anterior tw o-third s of the tongu e, tonsils, and pharynx. (B) Giant cell arteritis is a vascu litic cond ition that can lead to visu al loss bu t has never been reported in a case of stroke. (C) Cervical carotid artery d issection m ost com m only presents w ith neck, head , or facial p ain. (D) Pu re facial pain is rarely associated w ith sinusitis alone. 23. A 135 kg, 48-year-old fem ale is sched u led for a lap aroscop ic cholecystectom y u nd er general an esth esia. The factor m ost sign ifican t in affecting intraop erative arterial oxygenation is (A) (B) (C) (D) (E)

bod y p osition m od e of ventilation bod y w eight p resence of p neu m op eritoneu m lu ng com p liance

24. Well know n hem atologic changes that occur in p regn ancy inclu d e each of th e follow in g EXCEPT

Que s tions : 16–29

(A) m aternal blood volu m e increases by app roxim ately 45% at term (B) p lasm a cholinesterase levels d ecrease by abou t 25% resulting in a clinically relevant p rolongation of paralysis after a single d ose of succinylcholine (C) p lasm a albu m in levels d rop d u ring pregnancy (D) the concentrations of m ost coagu lation factors increase d u ring p regnancy thu s rep resenting a relative hyp ercoagu lable state (E) red blood cell volu m e increases second ary to increased prod u ction of erythrop oietin as w ell as the erythropoietin-like effects of p rogesterone, p rolactin, and p lacental lactogen DIRECTION S: Use the follow ing scenario to answ er Questions 25-29: A 65-year-old m ale has a right hilar mass w ith postobstru ctive pneu m onia. 25. In evalu atin g th is p atien t’s su itability for p neu m onectom y, w h ich on e of th ese tests w ou ld be the LEAST u sefu l? (A) (B) (C) (D) (E)

Pa CO 2 Pa O 2 FEV1 FVC Ventilation/ p erfu sion scan (V/ Q scan)

26. The p atient is hyp oxic on p reop erative evalu ation. Which therap y is LEAST likely to be effective? (A) (B) (C) (D) (E)

Lateral p osition Chest p hysical therap y Sod iu m nitrop ru ssid e Su p p lem ental oxygen Pneu m onectom y

389

27. The best m ethod to p rovid e good su rgical exp osu re for a left low er lobectom y is (A) u sing a large single-lu m en end otracheal tu be (B) u sing an end otracheal tu be w ith a ballooned catheter (Univent) p laced on the operative sid e (C) u sing a right sid ed d ou ble-lu m en tu be (D) u sing a left sid ed d ou ble-lu m en tu be (E) u sing an LMA 28. After ind u ction of gen eral an esthesia, th e p atient is intu bated w ith a left-sid ed d ou blelu m en tu be. When the circu it is connected to the d ou ble-lum en tu be, it is possible to ventilate the left lu ng, bu t not the right lu ng. Possible cau ses inclu d e all EXCEPT (A) the tu be is ad vanced too far (B) the right m ainstem bronchu s is blocked by tu m or (C) the right lu m en is blocked by secretions (D) the tu be is not ad vanced far enou gh (E) the tu be is in the w rong sid e 29. The p atient u nd erw ent u neventfu l right p neu m onectom y and rem ains intu bated at the end of the proced u re. On transferring the patient from the operating room table to the bed for transfer to the intensive care u nit, there is a sud d en d ecrease in blood pressure. Pu lses are faint. Appropriate maneuvers includ e all of the follow ing EXCEPT (A) p lacing the patient laterally on his left sid e (B) ap plying suction to the chest tu be (C) im m ed iately reopening the w ou nd and exp loring the su rgical field (D) p rovid ing oxygen and m anaging the airw ay (E) su pp orting card iac fu nction w ith an inotropic agent

390

20: P ra ctice Te s t

30. A 32-year-old G1P0 p atient p resents for an anesthesia consu lt at 20 w eeks gestational age. She has a history of tonic-clonic seizu res that are w ell-controlled on p henytoin. You tell her all of the follow ing statements about her cond ition are tru e EXCEPT (A) app roxim ately 0.5% of all partu rients have a chronic seizu re d isord er (B) single-agent anticonvu lsant therap y is p referred over com bination therap y (C) ap p roxim ately one-third of p atients w ill experience an increase in seizure frequency d u ring pregnancy (D) there is no contraind ication to neu raxial anesthesia (E) higher estrogen concentrations increase the seizu re threshold d u ring p regnancy 31. The agent w ith the highest ratio of β-adrenergic agonist to α-ad renergic agonist activity is (A) (B) (C) (D) (E)

isop roterenol d obu tam ine ep inep hrine norep inep hrine p henylep hrine

DIRECTION S: Use the follow ing scenario to answ er Qu estions 32-36: A 33-year-old m u ltip arou s fem ale ju st d elivered healthy tw in girls vaginally. After d elivery she has p oor uterine tone, resu lting in postpartu m hem orrhage. H er m ental statu s and blood pressu re d ecline, and you are called to assist w ith managem ent. You intu bate the p atient, p lace intravenou s access, and resu scitate the p atient w ith flu id s and blood p rod u cts w hile the obstetricians control the bleed ing. Once stabilized , she is brought to the ICU for further care. Over the course of tw o hou rs, she d evelop s w orsening oxygenation. H er vital signs are T 37.3°C, H R 93, BP 113/ 64, Sp O 2 92% on FIO 2 1.0. She is 5 feet, 2 inches and 76 kg. H er hem oglobin is 8 m g/ d L. Chest x-ray show s bilateral d iffu se infiltrates. A bed sid e echocard iogram is u nrem arkable. 32. Which one of the follow ing is the m ost ap p ropriate ventilator setting?

(A) Volu m e FIO 2 1.0 (B) Volum e FIO 2 1.0 (C) Volu m e FIO 2 1.0 (D) Volu m e FIO 2 1.0 (E) Volu m e FIO 2 1.0

control TV 250, RR 20, PEEP 5, control TV 300, RR 20, PEEP 5, control TV 300, RR 20, PEEP 12, control TV 440, RR 20, PEEP 5, control TV 440, RR 20, PEEP 12,

33. Six hou rs later, she is hem od ynam ically stable an d has requ ired no fu rther tran sfu sion s. Rep eat hem oglobin is 7.8 g/ d L. IN R is 1.6. Platelet count is 86,000/ m m 3. Which one of the follow ing is the m ost ap p ropriate therap eu tic intervention? (A) N o transfu sion (B) Transfu se one u nit of p acked red blood cells (C) Transfu se tw o u nits of packed red blood cells (D) Transfu se one u nit of fresh frozen p lasm a (E) Transfu se a single d onor u nit of p latelets 34. Tw enty fou r hours later, the patient has had a d ram atic im p rovem en t in h er p u lm on ary statu s, an d she is extu bated su ccessfu lly. Which one of the follow ing is the m ost likely cau se of her respiratory failure? (A) (B) (C) (D)

Am niotic flu id em bolism Pneu m onia Pu lm onary contu sion TRALI (transfu sion related acu te lu ng inju ry) (E) Volu m e overload 35. Mechanical ventilation w ith PEEP resu lts in w hich one of the follow ing? (A) Decreased p reload and left ventricu lar afterload (B) Decreased p reload and increased left ventricu lar afterload (C) Increased p reload and d ecreased left ventricu lar afterload

Que s tions : 30–42

(D) Increased p reload and left ventricu lar afterload (E) Increased p reload and right ventricu lar afterload 36. Which one of the follow ing has been show n to red u ce the n u m ber of d ays th at a p atient requ ires m echanical ventilation? (A) (B) (C) (D) (E)

Daily recru itm ent m aneu vers Daily sp ontaneou s breathing trials Prop hylactic antibiotics Sched u led su ctioning Volu m e control ventilation

37. The atyp ical antip sychotics like clozap ine and olanzap ine d iffer from antip sychotic agents like halop erid ol and chlorp rom azine in that they (A) cau se less extrap yram id al effects (B) d o not p rod u ce hyp otension (C) d o not cau se w eight gain and increased appetite (D) have less anticholinergic effects (E) have not been associated w ith a risk of new -onset type 2 d iabetes 38. General anesthesia as com p ared to regional anesthesia in p artu rients is associated w ith all of the follow ing EXCEPT (A) (B) (C) (D) (E)

less hyp otension less card iovascu lar instability less u terine relaxation m ore rap id ind u ction better control of the airw ay

39. A 2-year-old child is brou ght to the em ergency d ep artm ent w ith a 6-d history of increasing cou gh and w heezing. Sym p tom s have not resp on d ed to bron ch od ilator th erap y an d broad -sp ectru m antibiotics. On exam ination, the child is agitated , tachyp neic, and has a heart rate of 150 bp m . There are d ecreased breath sou nd s in the right low er zone of the

391

lu ng field s. This clinical p resentation is m ost consistent w ith (A) (B) (C) (D) (E)

right low er lobe pneu m onia exacerbation of und erlying asthm a inhaled foreign bod y acute laryngotracheobronchitis rup tu red congenital bullus

40. The blood flow d u ring total card iop u lm onary byp ass (A) (B) (C) (D) (E)

is not ad ju stable is virtu ally nonp u lsatile p rovid es a p u lsatile p ressu re is a p u lsatile flow m im ics norm al flow in all resp ects

41. In neu rosu rgical p atients, d extrose containing solu tions (A) (B) (C) (D) (E)

are the flu id s of choice m ay cau se excessive d iu resis m ay exacerbate hyp erglycem ia m ay p rod u ce brain ed em a lead to w ater retention

42. All of th e follow in g statem en ts regard in g head aches are tru e EXCEPT (A) the International H ead ache Society’s d iagnostic criteria for cervicogenic head aches inclu d es u nilaterality of sym p tom s and relief of pain by d iagnostic anesthetic blocks (B) m igraine w ith au ra is m ore com m on than m igraine w ithou t aura (C) in chronic tension-typ e head aches, the average head ache frequ ency is equ al to or greater than 15 d p er m onth (D) clu ster head aches are m ore p revalent in m en than in w om en (E) tricyclic antid ep ressants are a m ainstay of treatm ent for both m igraine and tension-type head aches

392

20: P ra ctice Te s t

43. Dexm ed etom id ine (A) is a hyp notic agent u sed for the ind u ction of general anesthesia (B) can m arked ly red u ce the MAC of inhalational anesthetics (C) w hen given as an IV bolu s of 2 m g/ kg w ill resu lt in an initial d ecrease in blood p ressu re and increase in heart rate (D) w ill resu lt in ad renal su p p ression w hen infu sed continu ou sly for p ostop erative sed ation (E) cau ses m ore resp iratory d ep ression than opioid s and benzod iazepines 44. Preeclam psia is associated w ith w hich one of the follow ing find ings? (A) (B) (C) (D) (E)

H yp ovolem ia Low seru m creatinine Low hem atocrit Decreased seru m u ric acid H yp otension

45. Carbon d ioxid e is som etim es ad d ed to the fresh gas su p p ly of the oxygenator d u ring hyp otherm ic card iopu lm onary bypass (A) to d ecrease p u lm onary vascu lar resistance (B) to increase the affinity of hem oglobin for oxygen (C) to d ilate coronary arteries (D) to m aintain the corrected Pa CO 2 at 40 m m H g 46. A relative, but not absolute, contraind ication to an ep id u ral steroid injections is (A) p reexisting neu rologic d isord er (e.g., m ultiple sclerosis) (B) sep sis (C) therap eu tic anticoagu lation (D) localized infection at injection site (E) p atient refu sal 47. You are giving a talk to a grou p of p regnant women during a birthing class about the options of analgesia d uring labor. You specifically d iscuss epidural analgesia. In your discussion, you

pull out a d iagram of the lumbar spine and you specifically state that a catheter w ill be placed (A) (B) (C) (D) (E)

below the d u ra betw een the d u ral and the arachnoid in the intrathecal sp ace betw een the d u ra and the p ia betw een the ligam entu m flavu m and the d u ra

48. A 74-year-old m an is und ergoing a transu rethral resection of prostate w ith sorbitol irrigation u nd er sp inal anesthesia. Ap p roxim ately 1 h after the beginning of the resection, he begins to complain of d ifficulty breathing. H e becom es progressively m ore confused , tachycard ic, and hypertensive. The seru m sod iu m concentration is 114 m Eq/ L. While inform ing the su rgeon and asking him to com p lete the proced ure exped itiou sly, the m ost appropriate next step w ou ld be to (A) ad m inister labetalol (B) ask the su rgeon to change the irrigating solution to norm al saline (C) ad m inister fu rosem id e (D) ind u ce general end otracheal anesthesia (E) ad m inister 3% sod iu m chlorid e 49. As the neu rosu rgeon m anip u lates tissu e in the p osterior fossa, there is su d d en brad ycard ia. The anesthetist shou ld (A) (B) (C) (D) (E)

low er the head ad m inister lid ocaine inform the neu rosu rgeon tu rn off all volatile anesthetics ad m inister naloxone to reverse effects of opioid s

50. Prostagland in E1 (A) is a p otent vasoconstrictor (B) shou ld be given by bolu s (C) shou ld alw ays be stop p ed before ind u cing anesthesia (D) is u sefu l in closing a p atent d u ctu s arteriosu s (E) m ay cau se hyp otension

Que s tions : 43–56

51. Post-herpetic neu ralgia (A) is com m on in child ren and ad olescents (B) is best treated w ith op ioid s (C) never resp ond s to local ap p lication of cou nterirritants (D) u su ally resp ond s to tricyclic antid epressants (E) is a d ifficu lt synd rom e to treat and su ccess is lim ited 52. A 61-year-old m ale w ith Marfan synd rom e is referred for su rgical rep air of a 6.4 cm ascend ing aortic aneu rysm that w ill inclu d e rep lacem ent of the aortic valve. The su rgeon inform s you that the repair w ill requ ire d eep hyp othermic card iac arrest w ith an expected d uration of over 30 m in. In ad d ition to effective cooling of the patient to a core tem peratu re of 25°C, the best strategy for the p revention of cerebral ischem ia d u ring su rgery is w hich one of the follow ing? (A) Ice p acks arou nd the p atient’s head (B) Selective antegrad e cerebral p erfu sion w ith cold , oxygenated blood into a single aortic arch branch vessel (C) Ad m inistration of a barbitu rate (D) Retrograd e cerebral p erfu sion w ith cold , oxygenated blood into the su perior vena cava (SVC) (E) Ad m inistration of high-d ose glu cocorticoid s DIRECTION S: Use the follow ing scenario to answ er Qu estions 53-55: A 24-year-old fem ale p ed estrian w as struck by a car 3 d ago. She w as brou ght to the em ergency d ep artm ent by em ergency m ed ical p ersonnel, w here she w as intu bated for confu sion and com bativen ess. H er in itial trau m a evalu ation revealed a p ositive FAST scan. She w as taken em ergently to the op erating room w here she u nd erw ent a sp lenectom y for a ru p tu red sp leen. She su bsequ ently u nd erw ent a m ore com p lete trau m a evalu ation that revealed a T10 bu rst fracture and an acetabu lar fractu re. She has been intu bated w hile aw aiting rep air of her fractu res. You are called to the bed sid e becau se the patient has d evelop ed fever, tachycard ia, hyp oxia, and increased secretions. Chest x-ray show s a new pu lm onary infiltrate. She

393

has rhonchi bu t no other changes in her p hysical exam ination. 53. Which one of the follow ing is the m ost ap p rop riate next step in m anagem ent? (A) (B) (C) (D) (E)

Ad m inister an antip yretic Ad m inister p ost-sp lenectom y vaccines Begin chest p hysiotherap y Obtain cu ltu res Start antibiotics

54. Which one of the follow ing is the m ost ap p rop riate p harm acotherap y? (A) (B) (C) (D) (E)

Ceftriaxone Levofloxacin Linezolid + ertap enem Vancom ycin + ceftriaxone Vancom ycin + levofloxacin + cefep im e

55. Forty-eight hou rs later, the p atient is afebrile w ith d ecreased secretions and im p roved oxygenation. Sp u tu m cu ltu re obtained before initiation of antibiotics revealed Haemophilus influenzae. What is the m ost appropriate next step in m anagem ent? (A) (B) (C) (D) (E)

Change antibiotics to ceftriaxone only Change antibiotics to clind am ycin only Change antibiotics to vancom ycin only Continu e cu rrent antibiotics Discontinu e all antibiotics

56. All of the follow ing statem ents are tru e concern in g som atosen sory evoked p oten tials EXCEPT (A) w aveform p eaks are d escribed in term s of am plitud e, latency, and polarity (B) am plitud e of evoked potentials is greater than those of the EEG (C) inju ry is m anifested as an increase in latency and / or d ecrease in am p litu d e (D) brain stem p otentials are m ore resistant to anesthetic influ ences than cortical p otentials (E) volatile anesthetics p rod u ce d ose-d ep end ent alterations in evoked p otentials

394

20: P ra ctice Te s t

57. You are called to a d elivery room w here the obstetrician has d ecid ed to p erform a low ou tlet forceps d elivery in a patient. All of the follow ing techniqu es are ap p rop riate EXCEPT (A) (B) (C) (D) (E)

bilateral p u d end al block p aracervical block su barachnoid block cau d al block ep id u ral block

58. Terbu taline (A) is an α-ad renergic agonist (B) is a selective β 2-ad renergic agonist (C) cau ses m ore tachycard ia than isop roterenol (D) shou ld be avoid ed in p atients w ith heart d isease (E) cau ses hyp erkalem ia

59. The one best block for p ain second ary to p ancreatic cancer is a(n) (A) (B) (C) (D) (E)

stellate ganglion block Bier block block of the hyp ogastric p lexu s celiac p lexu s block intrathecal neu rolysis

60. Pulm onary vascu lar resistance is increased by (A) (B) (C) (D) (E)

sevoflu rane d esflu rane isoflu rane nitrou s oxid e oxygen

DIRECTION S: Use the follow ing figu re to answ er Qu estions 61-62:

Que s tions : 57–67

61. The tracing in the grap h show s a p attern referred to as (A) (B) (C) (D) (E)

early d eceleration late d eceleration saltatory d eceleration variable d eceleration sinu soid al d eceleration

62. This typ e of fetal heart rate p attern is u su ally associated w ith (A) (B) (C) (D) (E)

u terop lacental insu fficiency head com p ression cord com p ression severe fetal asp hyxia p rem atu rity

63. Com p lication s of sp in al cord stim u lation inclu d e all of the follow ing EXCEPT (A) lead m igration (B) increased risk of d evelop m ent of chord om a (C) lead breakage (D) bleed ing 64. A 58-year-old otherw ise healthy m an is und ergoing repair of an Achilles tendon rupture. The orth op ed ic su rgeon asked that the low er extrem ity tou rniqu et rem ain inflated throu gh the entire 2-h p roced u re. Up on release of the tourniqu et, you w ill expect to see (A) (B) (C) (D) (E)

d ecreased end -tid al CO 2 m etabolic alkalosis increased blood p ressu re d ecreased bod y tem p eratu re increase in anesthetic requ irem ents

65. A 72-year-old m ale w ith Parkinson d isease on levod op a is sched u led for su rgery. The anesthetic plan should inclu d e (A) stopp ing levod op a for 24 h before ind u ction (B) avoid ance of p henothiazines (C) u se of neu rolep tanesthesia as the techniqu e

395

(D) anticip ated need for larger than u su al d oses of pressors (E) u se of high concentrations of a volatile agent in lieu of a nond ep olarizing m u scle relaxant 66. Acu te treatm ent of cerebral ed em a inclu d es all of the follow ing EXCEPT (A) (B) (C) (D) (E)

osm otic d iu retics loop d iu retics glu cocorticoid s su rgical d ecom p ression d rainage of CSF

67. You are called to the labor and d elivery suite to evalu ate a p atient w ho has ju st d elivered . The obstetrician states the uterus is atonic. Each of the follow ing m ay be u sed to treat the u terine atony EXCEPT (A) (B) (C) (D) (E)

u terine m assage intram uscular m ethylergonovine intrau terine p rostagland in F2α intravenou s m ethylergonovine intravenou s oxytocin

DIRECTION S: Use the follow ing figu re to answ er Qu estion 68:

396

20: P ra ctice Te s t

68. The flow / volu m e cu rve show s (A) norm al insp iratory and exp iratory volum e cu rves (B) fixed intrathoracic or extrathoracic obstruction (C) variable intrathoracic obstru ction (D) variable extrathoracic obstru ction 69. A 6-m onth-old infant, born fu ll term , presents for elective inguinal hernia repair. The infant is otherw ise healthy, and in no acu te d istress. Which one of the follow ing are norm al heart rate, blood p ressu re and O 2 consu m p tion for this patient? (A) H R 120, BP 5 m L/ kg (B) H R 140, BP 10 m L/ kg (C) H R 140, BP 5 m L/ kg (D) H R 120, BP 15 m L/ kg (E) H R 120, BP 10 m L/ kg

90/ 60, O 2 consu m p tion 90/ 60, O 2 consu m p tion 70/ 40, O 2 consu m p tion 90/ 60, O 2 consu m p tion 110/ 70, O 2 consu m p tion

70. Diagnostic criteria for cervicogenic head ache by the International H ead ache Society and the International Association for the Stu d y of Pain (IASP) inclu d e all of the follow ing EXCEPT (A) u nilateral head ache (B) relief of acu te attacks by blocking the V2 branch of the trigem inal nerve w ith local anesthetic (C) aggravation of the head ache w ith neck m ovem ents (D) d ecreased range of neck m otion 71. Upon d oing a m orning p ostop erative check on a p atient w ho u nd erw ent a total thyroid ectom y for a longstand ing mu ltinod ular goiter the previou s afternoon, you notice he is som ew hat strid orou s, w ith sternal retractions. H e asks w hy the anesthesia has m ad e his fingers and toes tingle. On evalu ation you note (A) a p ositive Chvostek sign (B) d im inished reflexes

(C) a negative Trou sseau sign (D) ed em a of his u p p er extrem ities (E) incisional sw elling 72. α-ad renocep tors in the ad renergic nervou s system (A) (B) (C) (D) (E)

are stim u lated by isop roterenol are blocked by m etop rolol cau se vasoconstriction cau se bronchial d ilatation are fou nd on ad ip ocytes

73. Patients w ith acu te su barachnoid hem orrhage m ay d em onstrate all of the follow ing EXCEPT (A) ST segm ent elevation consistent w ith m yocard ial ischem ia (B) card iac arrhythm ia (C) elevated card iac enzym es (D) d ep ressed ventricu lar fu nction (E) p u lm onary hyp ertension 74. All of the follow ing are ind icative of fetal w ellbeing EXCEPT (A) (B) (C) (D)

good long-term variability biop hysical p rofile score of 8/ 10 reactive non-stress test a p ositive oxytocin contraction stress test (E) fetal scalp p H of 7.27 75. A health y 29-year-old w om an d elivers a healthy fu ll term neonate. There have been no problem s or com p lications d u ring p regnancy or labor and d elivery. The neonate’s 1- and 5-m in Apgar scores w ere 8 and 9, respectively. Both m other and infant are d oing w ell. All of the follow ing w ould be expected to occur in the first 24 h of life of this neonate EXCEPT (A) fu nctional closu re of the foram en ovale (B) fu nctional closu re of the d u ctu s arteriosu s (C) d ecrease in afterload on the left ventricle (D) increase in p u lm onary blood flow (E) large increase in volu m e load on left ventricle

Que s tions : 68–83

76. In term s of com p lex regional p ain synd rom e (CRPS), w hich one of the follow ing statem ents is tru e? (A) The incid ence of CRPS is 20% after brain lesion. (B) Extrem ities affected by a brain inju ry are at higher risk of d evelop ing CRPS than u naffected extrem ities. (C) CRPS follow ing sp inal cord inju ry is frequ ent. (D) Low er extrem ities are m ore com m only affected than u pp er extrem ities. 77. A patient is sched uled for a CO 2 laser ablation of a laryngeal tum or. The risk of airw ay fire is d ecreased by (A) (B) (C) (D) (E)

a fresh gas flow less than 2 L/ m in u se of a clear PVC end otracheal tu be u se of a p etroleu m -based lu bricant su bstitu tion of heliu m for N 2O u se of the laser in continu ou s m od e

78. In p reeclam p tic or eclam p tic p atients (A) regional anesthesia is contraind icated (B) hyp oreflexia is com m on (C) flu id restriction is necessary becau se of the presence of pu lm onary ed em a in a m ajority of patients (D) resistance to vasop ressors is com m on (E) significant coagu lop athies m ay occu r 79. An 81-year-old w om an slipp ed on a piece of ice and in the p rocess su stained an op en right d istal u lna fractu re in her outstretched hand . All of the follow ing are reasonable regional anesthetic blocks for su rgeries d istal to the elbow EXCEPT a(n) (A) (B) (C) (D)

axillary block su p raclavicu lar block infraclavicu lar block interscalene block

80. A 6-year-old p atient is u nd ergoing a craniotomy in the prone position for removal of a posterior fossa tu m or. You are concerned abou t

397

venous air em bolism and hem od ynam ic instability d u ring this su rgery so you are u sing m u ltip le m onitors d u ring this su rgery in ad d ition to the stand ard ASA m onitors. These inclu d e an arterial catheter, end -tid al nitrogen, and esop hageal stethoscop e. Of the follow ing m onitors, w hich one is the m ost sensitive ind icator of a venou s air em bolism ? (A) (B) (C) (D) (E)

End -tid al nitrogen Blood p ressu re End -tid al carbon d ioxid e Change in ECG Esop hageal stethoscop e

81. A 1-m onth-old infant p resents for exp loratory lap arotom y for sym p tom s of bow el obstru ction. The initial tem peratu re after ind u ction is noted to be 35.6°C. At the tim e of incision, the tem p eratu re is noted to be 34.9°C. All of the follow ing are m echanism s by w hich an infant u nd er anesthesia loses bod y heat EXCEPT (A) (B) (C) (D) (E)

the m etabolism of brow n fat breathing d ry gases cond u ction to cold su rrou nd ings cold skin p rep aration solu tions exp osu re of abd om inal contents

82. Which one of the follow ing is a possible ad vantage of acu p u nctu re over other m ethod s of p ain therapy? (A) Safety, since the p hysiology of the p atient is not d istu rbed (B) A tonic or regu latory effect on the bod y (C) Long-lasting resid u al effects (D) An anti-inflam m atory effect 83. A 72-year-old p atient w ith a history of closed angle glau com a p resents for cataract su rgery. In ord er to p revent an increase in intraocu lar p ressu re, one shou ld avoid (A) (B) (C) (D) (E)

hyp otherm ia hyp ercarbia intravenou s acetazolam id e hyp otension top ical tim olol

398

20: P ra ctice Te s t

84. A 68-year-old m ale is sched uled to have end ovascu lar stent graft rep air of a 7.4 cm d escend ing thoracic aortic aneu rysm . The p atient’s past m ed ical history is significant for insu lin d ep end ent d iabetes, hyp ertension, hyp ercholesterolem ia, and m yocard ial infarction. The patient’s p ast su rgical history is significant for infrarenal aortic aneu rysm rep air. In ord er to d ecrease the risk of sp inal cord ischem ia, the patient w ill und ergo p reop erative p lacem ent of a lum bar d rain for CSF d rainage. Which one of the follow ing ad d itional interventions is m ost appropriate to d ecrease the risk of sp inal cord ischem ia in this patient? (A) Deep hyp otherm ic circu latory arrest (DH CA) (B) Pu lse d ose glu cocorticoid therap y (C) H igh-d ose op ioid anesthesia techniqu e (D) Deliberate m ild to m od erate system ic hypotherm ia (E) Deliberate m ild to m od erate hypotension 85. β 2-ad renocep tors in the au tonom ic nervou s system (A) (B) (C) (D) (E)

are stim u lated by isop roterenol are antagonized by esm olol are activated by clonid ine cau se vasoconstriction are the p rincip al p reju nctional recep tors that inhibit sym pathetic neu rotransm itter release

DIRECTION S: Use the follow ing scenario to answ er Qu estions 86-87: A 67-year-old w om an w ith COPD is ad m itted to the ICU w ith p neu m onia and resp iratory failu re several d ays follow ing vid eo-assisted thoracoscopic lu ng resection of squam ou s cell carcinom a. H er height is 63 inches, w eight 80 kg (id eal bod y w eight 52 kg). Vital signs are T 38.2°C, H R 98, BP 110/ 72, RR 36, Sp O 2 96% w hile m echanically ventilated w ith FIO 2 1.0. Spu tu m cu ltures reveal abu nd ant grow th of Streptococcus pneumoniae. 86. Which one of the follow ing ventilator settings is m ost likely beneficial in p reventing ventilator ind u ced lung inju ry?

(A) (B) (C) (D) (E)

Tid al volu m e 520 m L PEEP equ al to 0 cm H 2O Insp iratory tim e < 1 sec Plateau p ressu re < 30 cm H 2O Recru itm ent m aneu ver w ith CPAP = 40 cm H 2O

87. After 24 h of mechanical ventilation, a spontan eou s breathin g trial (SBT) is con d u cted . Which one of the follow ing com binations of criteria d uring the SBT suggest that extubation shou ld be consid ered ? (A) RR 36, Sp O 2 95%, H R 88 (B) RR 18, Sp O 2 91%, H R 90 (C) Patient requ esting to be extu bated , SBP 200 m m H g, H R 150 (D) RR 32, Sp O 2 96%, patient rep orting shortness of breath and anxiety (E) RR 12, Sp O 2 89%, H R 87 88. In creased n eon atal d ep ression h as been observed after nonurgent cesarean d elivery in w hich one of the follow ing circu m stances? (A) General anesthesia com p ared to regional anesthesia (B) An elap sed tim e of 8 m in betw een ind u ction and d elivery (C) Use of a volatile agent (D) An elap sed tim e of 4 m in betw een uterine incision and d elivery (E) ep id u ral versu s sp inal anesthesia 89. Con cer n in g in d u ced neu rosu rgery

h y p oten sion

in

(A) ind u ced vasod ilation m ay facilitate p erfu sion d u ring ischem ia (B) is best provided with phenoxybenzamine (C) m ay be required for brief p eriod s (D) m ay d ecrease p hysiologic d ead sp ace (E) is relatively contraind icated for any length of time d u ring intracranial su rgery DIRECTION S: Use the follow ing scenario to answ er Qu estions 90-91: A-22 year-old m ale p resents for left should er arthroscopy w ith a rotator cuff repair after

Que s tions : 84–95

su staining a basketball in ju ry. The follow ing u ltrasou nd -gu id ed nerve block w as perform ed for p ostop erative analgesia.

399

active as u su al. Lu ngs are clear to au scu ltation bilaterally. You inform the p arents that you w ill (A) p roceed w ith su rgery tod ay (B) p rescribe antibiotics for the p atient (C) ad m it to the floor p ostop eratively for overnight m onitoring (D) p ostpone su rgery for 2-4 w eeks (E) ad m inister an albu terol nebu lizer p reop eratively and p ostop eratively as need ed 93. An ad vantage of selective COX-2 inhibitors over nonselective COX inhibitors is (A) (B) (C) (D)

90. The nerve that w ill m ost likely be sp ared is the (A) (B) (C) (D) (E)

p hrenic nerve m u scu locu taneou s nerve u lnar nerve su p rascap u lar nerve rad ial nerve

91. Thirty m inu tes after the block is p erformed the p atient is noted to have sensation on the cap e of the ip silateral shou ld er. The m ost likely reason for this is failu re to (A) (B) (C) (D) (E)

block the axillary nerve block the su p erficial cervical p lexu s anesthetize the su p rascap u lar nerve anesthetize the m u scu locu taneou s nerve perform a T1 p aravertebral block

92. A 5-year-old boy presents for elective ou tp atient rep air of an ingu inal hernia. H is parents inform you that he has a ru nny nose w ith yellow nasal d ischarge and a m ild occasional w et cou gh. They d eny any fevers. H e has been eating and d rinking norm ally and has been

p rotective renal effects p rotective card iovascu lar effects inhibit p rod u ction of throm boxane A2 few er GI sid e effects

94. The m ed ication occasionally used as a bed time sed ative that is associated w ith CN S excitation a few hou rs after taking it is (A) (B) (C) (D) (E)

d iphenhyd ram ine ethanol p entobarbital eszopiclone m elatonin

95. What is the meaning of the term ”left dominant” on a card iac catheterization report? (A) It refers to the blood su p p ly of the sinu s nod e by the left circu m flex artery. (B) It ind icates that the p atient has left ventricu lar hyp ertrop hy. (C) It ind icates that the entire left ventricle is su p p lied by the left coronary artery. (D) It ind icates that the p osterior d escend ing artery is su p p lied by the left circu m flex artery. (E) It ind icates a large left anterior d escend ing artery.

400

20: P ra ctice Te s t

96. Com p lex regional p ain synd rom e is characterized by all of the follow ing EXCEPT it (A) m ay resu lt from a gu nshot w ou nd (B) is generally less severe in p ersons w ith u nd erlying anxiety or d epression (C) can resp ond to sym p athectom y (D) is often accom p anied by d ystrop hic changes of bone 97. Seizu res (A) p rod u ce cerebral alkalosis (B) are term inated by isoflu rane anesthesia at 1 MAC (C) generate bu rst activity on EEG (D) red u ce regional cerebral blood flow and m etabolism (E) are initiated by m ost general anesthetics 98. A 5-w eek-old infant p resents for p ylorom yotom y. The anesthetic p lan inclu d es a rap id sequ ence ind u ction and intu bation. Several steps in positioning and equ ipm ent selection are taken to aid in secu ring the airw ay as rap id ly as p ossible. All of the follow ing are tru e of the infant’s airw ay relative to an ad u lt’s airw ay EXCEPT (A) the infant has a m ore cep halad -p laced larynx (B) the infant’s ep iglottis is long and om ega shaped (C) the infant’s vocal cord s are slanted d ow n and anteriorly (D) the narrow est area of the infant’s airw ay is at the rim a glottid is (E) the infant’s tongu e is larger relative to the rest of the airw ay 99. A 30-year-old G4P0 p atient w ith a history of antip hosp h olip id synd rom e an d recu rren t p regnancy loss p resents at 20 w eeks gestational age for an anesthesia consult. She is cu rrently on asp irin and a prop hylactic d ose of low m olecu lar w eight hep arin (LMWH ) once d aily. She asks abou t her d isease and p regnancy. You tell her that

(A) ep id u ral anesthesia is contraind icated second ary to the increased risk of bleed ing w ith lu p u s anticoagu lant p resent (B) she m u st w ait 24 h from the last d ose of LMWH to receive neuraxial anesthesia (C) m ost infants of w om en w ith antip hosp holip id synd rom e have an increased rate of neonatal or child hood com plications (D) lu p u s anticoagu lant and anticard iolip in antibod y are associated w ith both venous and arterial throm botic events (E) w om en d iagnosed w ith antip hosp holip id synd rom e on the basis of a history of recu rrent p regnancy loss and evid ence of antiphospholipid antibod y, bu t w ithou t prior throm botic events, often su ffer throm botic events d u ring pregnancy 100. Arrh ythm ia is associated w ith all of th e follow ing EXCEPT (A) (B) (C) (D)

orbital d ecom p ression tentorial m anip u lation sp inal cord rhizotom y arterial venou s m alform ation em bolization (E) carotid artery balloon angiop lasty 101. Preeclam psia is characterized by all of the follow ing EXCEPT (A) (B) (C) (D) (E)

intravascu lar volu m e d ep letion p roteinu ria occu rrence anytim e tim e p regnancy no change in p lacental p erfu sion d ecreased p rod u ction of renin

102. An active agent u sefu l in the p erform ance of a neu rolytic block is (A) (B) (C) (D)

40% p otassiu m hyd roxid e 10% glycerin 50% alcohol 100% p henol

Que s tions : 96–108

103. A m ajor sid e effect of valproic acid is (A) (B) (C) (D) (E)

d ecreased renal fu nction elevation in liver enzym es p otentiation of m u scle relaxants arrhythm ias anem ia

DIRECTION S: Use the follow ing scenario to answ er Qu estions 104-106: A 42-year-old fem ale w ith a longstand ing history of typ e I d iabetes and end stage renal d isease requ iring d ialysis has received the call that an ap propriate kid ney is now available for transp lant. She is sched u led for im m ed iate transplant, and arrives, qu ite anxious, d irectly from d ialysis to the p reop erative area. She has been N PO for 4 h. Preoperative labs inclu d e H gb of 8.4 g/ d L, glu cose 183 m g/ d L, and [K+ ] 5.3 m Eq/ L. H er vital signs are BP 112/ 62, H R 104 bpm , Sp O 2 97%. The anesthetic p lan is for general anesthesia. 104. The anesthetic agent best avoid ed in this patient is (A) (B) (C) (D) (E)

etom id ate N 2O su ccinylcholine sevoflu rane vecu roniu m

105. The su rgeon is concerned abou t the risk of a p ostop erative ep id u ral hem atom a, and asks that you use parenteral op ioid s for postoperative analgesia rather than p lace an ep id u ral catheter. As he com pletes the p roced u re, the su rgeon expresses concern abou t the fu nctioning of the transp lanted kid ney. The op ioid that w ou ld be affected the m ost by this p atient’s renal d ysfu nction is (A) (B) (C) (D)

fentanyl hyd rom orp hone m orp hine su fentanil

106. Which one of the follow ing m aneu vers is the m ost effective m eans to p revent acu te tu bu lar

401

necrosis and thereby facilitate im m ed iate graft fu nction in this p atient? (A) Volu m e exp ansion w ith saline, albu m in and / or blood (B) Volu m e exp ansion p lu s m annitol (C) Ad d ition of cyclosp orine (D) Volu m e exp ansion p lu s fu rosem id e 107. A 27-year-old G1P0 p atient p resents for an anesthesia consu lt at 20 w eeks gestational age. H er p renatal cou rse is notable for a history of corrected id iopathic scoliosis. Tru e statem ents regard ing her d isease in p regnancy inclu d e all of the follow ing EXCEPT (A) p regnant w om en w ith corrected id iop athic scoliosis tolerate pregnancy, labor, and d elivery w ell (B) insertion of an ep id u ral need le in the fu sed area m ay not be p ossible (C) the sp read of injected local anesthetic m ay be altered resu lting in an increased incid ence of inad equ ate analgesia after ep id u ral p lacem ent (D) 80% of p atients u nd ergo fu sion to the low est lu m bar levels, lim iting the p otential for neu raxial anesthesia (E) there is a higher incid ence of d u ral p u ncture 108. A 72-year-old fem ale p atient is u nd ergoing right p neu m onectom y for lu ng cancer. After starting one-lung ventilation w ith oxygen and isoflurane, a tid al volume of 500 m L, and a rate of 10 breaths p er m inu te, the p atient’s oxygen saturation starts to fall as the surgeon is opening the chest. The m ost effective m eans of im p roving the oxygen satu ration is (A) increasing oxygen flow to the anesthesia circu it (B) ad d ing 10 cm H 2O PEEP to the ventilated lung (C) clam p ing the right p u lm onary artery (D) starting intravenou s nitroglycerin (E) increasing the concentration of isoflu rane

402

20: P ra ctice Te s t

109. A 35-year-old G1P0 patient w ith a history of chronic hypertension w ho takes lisinopril presents for her first p renatal visit at 12 w eeks gestational age. The obstetrician calls and asks you r op inion regard ing her m ed ication. You ad vise the obstetrician that w hich one of the follow ing d ru gs has the best safety p rofile in pregnancy? (A) (B) (C) (D) (E)

Labetalol N ifed ip ine Methyld op a Fu rosem id e Cap top ril

110. Managem ent of the airw ay d uring ind u ction of general anesthesia in a p atient in a halo brace for a nond isp laced u nstable fractu re of C6 incurred in a high-speed motor vehicle accid ent includ es all of the follow ing EXCEPT (A) assessm ent for inju ries of the face (B) aw ake fiberop tic intu bation (C) ad equ ate anesthesia of the trachea to p revent cou ghing (D) rem oval of the cervical brace for intu bation (E) nasal or oral rou te of intu bation 111. An eld erly gentlem an p resenting to the p ain clinic com plains of right-sid ed thoracic pain. H e d eveloped a very painful rash that started abou t 6 m onths ago. The lesions are now gone, bu t he states that the p ain is p ersistent. A beneficial therap y m od ality is (A) high-d ose asp irin (B) TEN S (transcu taneou s electrical nerve stim ulation) (C) tricyclic antid ep ressants (D) op ioid s 112. A 3-year-old , p reviou sly healthy child , p resents for an u rgent left sid ed VATS p roced u re and chest tu be placement for a parapneu monic effu sion. The child is on 3 liters of oxygen by nasal cannu la w ith Sp O 2 of 92-95%. She is tachypneic, bu t d oes not appear to be in d istress an d is h em od yn am ically stable.

Which one of the follow ing is tru e regard ing m anagem ent of ventilation in this patient? (A) Use of a d ou ble lu m en tu be w ou ld be the best w ay to provid e lu ng isolation in this case. (B) Becau se the blocker tu be is attached to the m ain end otracheal tu be in a Univent tube, there is greater chance of d isplacem ent of the blocker than w hen other blockers are u sed . (C) Lu ng isolation is not likely to be necessary for this proced ure. (D) An arterial catheter w ill be necessary to gu id e ventilation m anagem ent. (E) If a bronchial blocker is u sed to p rovid e lu ng isolation for this p roced u re, a larger end otracheal tu be w ill be requ ired in ord er to allow the p assage of the fiberoptic bronchoscope and the bronchial blocker. 113. The neu ral p athw ays that are resp onsible for the transm ission of p ain d u ring the first and second stages of labor are (A) (B) (C) (D) (E)

T6 – T12 and S1 – S4 T8 – L2 and S1 – S3 T10 – L1 and S2 – S4 T10 – L5 T12 – L3 and S2 – S5

114. A 38-year-old w om an p resents w ith p alp itations and is found to be in atrial fibrillation. She is otherw ise healthy and active. An echocard iogram is p erform ed that reveals a congen ital card iac lesion . Th e m ost likely d iagnosis in this patient is (A) (B) (C) (D) (E)

ventricu lar sep tal d efect atrial sep tal d efect coarctation of the aorta p atent d u ctu s arteriosu s p u lm onary stenosis

DIRECTION S: Use the follow ing scenario to answ er Qu estions 115-116: A 65-kg p atient presents for elective CABG. Ind uction of anesthesia is facilitated w ith fentanyl 15 m cg/ kg, m id azolam 150 m cg/ kg, p rop ofol 0.8 m g/ kg, and rocu roniu m 0.8 m g/ kg.

Que s tions : 109–121

Anesthesia is m aintained w ith isoflu rane 0.7 MAC in oxygen and air, and initial m inu te ventilation is set at 4 L/ m in. Du ring the p rebypass p eriod the p atient d evelop s tachycard ia, hyp otension, and su bsequ ent ST elevations in lead V, as w ell as a rise in the p u lm onary artery (PA) p ressu re. 115. The rise in PA pressu re is most likely caused by (A) (B) (C) (D) (E)

m yocard ial ischem ia p u lm onary em bolism hyp oventilation hyp ovolem ia light anesthesia

116. Ad m inistration of w hich one of the follow ing agents is m ost ap prop riate in this situ ation? (A) (B) (C) (D) (E)

Ep inep hrine Glycop yrrolate Phenylep hrine Sod iu m nitrop ru ssid e N icard ip ine

DIRECTION S: Use the follow ing scenario to answ er Qu estions 117-118:A 2-year-old child p resents for strabism u s rep air. The child is otherw ise healthy. After an u neventfu l ind u ction and intu bation, su rgery begins. Du ring the p roced u re the p atient’s heart rate acu tely d ecreases from sinus rhythm at 110 bpm to sinu s rhythm at 50 bp m . The blood p ressu re is norm al and stable. 117. Which one of the follow ing is the best treatm ent for this? (A) Preop erative atrop ine (B) A retrobu lbar block (C) Inform the su rgeon and requ est that he/ she stop u ntil the heart rate recovers (D) Ad m inistration of vecu roniu m (E) Ad m inistration of neostigm ine 118. Which one of the follow ing are the respective afferent and efferent lim bs of the reflex that resu lted in the p atient’s brad ycard ia? (A) Ciliary nerve and vagu s nerve (B) Trigem inal nerve and facial nerve

403

(C) Trigem inal nerve and vagu s nerve (D) Vagu s nerve and op hthalm ic nerve (E) Ciliary nerve and facial nerve 119. A 30-year-old G2P1 p atient p resents for an anesthetic consu ltation at 20 w eeks gestational age. H er p renatal cou rse is com p licated by Graves d isease. She has not u nd ergone rad ioactive iod ine treatm ent to d ate and is cu rrently on m ethim azole. Tru e statem ents regard ing her care inclu d e all of the follow ing EXCEPT (A) rad ioactive iod ine is contraind icated d u ring pregnancy (B) the m ainstay of treatm ent for hyp erthyroid ism d u ring p regnancy is the u se of antithyroid m ed ication (C) m ethim azole d oes not cross the p lacenta in ap p reciable am ou nts. (D) thyroid storm occu rs in 2-4% of p regnant p atients w ith hyp erthyroid ism (E) hyp erthyroid p atients shou ld receive glu cocorticoid su p p lem entation 120. In com p aring zalep lon and triazolam , w hich one of the follow ing ad verse effects is LEAST likely to occu r w ith zalep lon althou gh it com m only occu rs w ith the u se of triazolam ? (A) (B) (C) (D) (E)

Prolonged sed ation Abolition of REM sleep Extrap yram id al effects Ventilatory d ep ression Am nesia

121. Therap y for a p atient w ith closed head trau m a and elevated intracranial pressure may includ e (A) avoid ance of sed ation to m aintain an u nequ ivocal neurological exam (B) u se of p ressors to m aintain cerebral p erfu sion pressu re at 80 m m H g or above (C) head d ow n p osition to im p rove p erfu sion (D) end otracheal intu bation and m echanical ventilation for hyp oxem ia (E) rou tine hyp erventilation

404

20: P ra ctice Te s t

122. A new born is noted to have aspirated m econiu m . The orop harynx is su ctioned and the patient rem ains stable w ith oxygen ad m inistered by nasal cannula. Which one of the follow ing is tru e of this p atient? (A) This patient w as likely born prematurely. (B) The long-term ou tcom e for this p atient is p oor. (C) Chest p hysical therap y is not ind icated . (D) This p atient m ost likely had in u tero stress and fetal hypoxia. (E) Vagal stim u lation m ay cau se p assage of m econiu m in u tero. 123. α1-antitrypsin d eficiency is (A) (B) (C) (D)

a nonvascu lar lu ng d isease d eterm ined by a seru m assay nonfam ilial d u e to a lack of an enzym e p rod u ced by the lu ng (E) the m ost com m on cau se of COPD 124. The p revalence of obesity in the general p op u lation has been grow ing at an alarm ing rate and is reaching ep id em ic prop ortions. All of the follow ing are tru e statem ents regard ing m orbid obesity in p regnancy EXCEPT (A) there is a significantly higher failu re rate of epid u ral blocks (B) longer-than-norm al sp inal need les are m ore frequ ently requ ired for neu raxial anesthesia than longer-than-norm al ep id u ral need les (C) m orbid obesity fu rther increases the risks of m aternal m orbid ity, fetal inju ry, and anesthesia-related m aternal d eath d u ring and after cesarean section (D) there is a higher risk of both p reterm d elivery and d elivery of a low -birthw eight infant (E) m orbid ly obese p atients w ho have u nd ergone cesarean section shou ld be m onitored w ith continuou s p u lse

oxim etry after d ischarge from the postanesthesia care u nit DIRECTION S: Use the follow ing scenario to answ er Qu estions 125-126: A 52-year-old m an is ad m itted to the ICU after he w as rescued from a bu rning hom e. H e w as intubated w ithou t sed ative m ed ications in the em ergency d epartm ent and transferred to the ICU on m echanical ventilation. H e has neither evid ence of burns to his skin nor evid ence of inhalation inju ry on external exam . H e is com atose. Vitals show T 37.8°C, H R 122, BP 145/ 90, Sp O 2 100% on FIO 2 1.0. A head CT scan is u nrem arkable. 125. Which one of the follow ing is the m ost likely exp lanation for the norm al oxygen satu ration? (A) (B) (C) (D) (E)

Carboxyhem oglobin Sickle cell anem ia Methem oglobinem ia Methylene blu e ad m inistration H igh FIO 2

126. The p atient is transp orted to the hyp erbaric oxygen (H BO) chamber for therapy. Which one of the follow ing com plications of H BO treatm ent m ay be seen in the cham ber? (A) (B) (C) (D) (E)

Brad ycard ia Skin erythem a Eu p horia Seizu res Chest p ain

127. You are called to help resu scitate a new born in the d elivery room . The neonate is a 38-w eek gestational age fem ale w ith an Ap gar score of 4 at one m inu te. All of the follow ing are app rop riate interventions EXCEPT (A) clearing of the airw ay w ith a bu lb syringe (B) intu bation im m ed iately (C) bag m ask ventilation of lu ngs at 60 breaths/ m in (D) stim u lation of the neonate

Que s tions : 122–133

128. Which one of the follow ing m ed ications is MOST selective for inhibiting COX-2? (A) (B) (C) (D) (E)

Celecoxib Ibu p rofen Ind om ethacin Ketorolac Acetam inop hen

129. When properly positioned , a left-sid ed d oublelu m en tu be w ill have its lu m ens end ing (A) in the left bronchu s and the right bronchu s (B) in the left bronchu s and in the trachea (C) in the right bronchu s and in the trachea (D) in the left u p p er lobe bronchu s and in the left low er lobe bronchu s (E) both in the trachea 130. A 68-year-old p atient is on long term COX inhibitor therap y as p art of her treatm ent for severe rheumatoid arthritis. The most common ad verse effect of this class of d ru gs is (A) (B) (C) (D) (E)

gastric sid e effects ischem ic heart d isease blood p ressu re elevation analgesic nep hrop athy hyp ersensitivity

131. A 3-year-old child experiences bu rn injuries to 50% of her bod y su rface area. She also has significant inhalation inju ry. Which one of the follow ing statem ents is tru e? (A) Within 4 d an am ou nt of albu m in equal to abou t the total bod y plasm a content is lost throu gh the w ou nd . (B) Im m ed iately after inju ry, card iac ou tp u t is increased .

405

(C) The use of pu lse oxim etry is m and atory in m onitoring the p atient if carbon m onoxid e p oisoning is su spected . (D) Evap orative fluid losses are ap proxim ately 4 liters for each squ are m eter of bu rned su rface p er d ay. (E) Intubation should be avoid ed in this p atient. 132. An anesthesiologist has a d rinking p roblem . After com ing to w ork on several occasions w ith alcohol on his breath, he w as confronted by th e Physicians’ H ealth Com m ittee an d convinced to seek treatm ent. Which one of the follow ing actions that he m ight take w ou ld be least effective in h elp in g h im m ain tain sobriety? (A) H e gets an injection of naltrexone every tw o w eeks. (B) H e takes a d isu lfiram tablet each d ay. (C) H e attend s a m eeting of Alcoholics Anonym ou s several tim es p er w eek (D) H e has blood d raw n rand om ly once or tw ice a w eek for m easu rem ent of alcohol concentration. (E) H e has a 50-m in cou nseling session w ith his therapist once w eekly. 133. Sp inal cord stim u lation (A) requ ires an external generator (B) involves an electrical stim u lator p laced in the su barachnoid sp ace (C) is u sefu l for p ostop erative p ain m anagem ent (D) m ay be help fu l in p atients w ith intractable back p ain

406

20: P ra ctice Te s t

134. At the molecular level, the effect of benzodiazepines is most accurately described as increasing (A) (B) (C) (D) (E)

sod iu m cond u ctance p otassiu m cond u ctance chlorid e cond u ctance end ozep ine bind ing to its recep tor acetylcholine bind ing to its recep tor

135. You su sp ect nerve root im p ingem ent in the lu m bar sp ine in a p atient. Which one of the follow ing p hysical find ings w ou ld su p p ort this d iagnosis? (A) You susp ect L2 nerve root involvem ent and the patient has w eakness of hip flexion and sensory loss on the lateral asp ect of the calf. (B) You susp ect L4 nerve root involvem ent and the patient has w eakness of leg extension and loss of p atellar reflex. (C) You su sp ect L5 nerve root involvem ent and the patient cannot d orsiflex his big toe and has a loss of the Achilles reflex. (D) You su sp ect S3 nerve root involvem ent and the patient has loss of sensation over the bottom of the foot. The Achilles tend on reflex is norm al. 136. Fetal hemoglobin is necessary to allow transfer of oxygen from m other to fetus. Which one of the follow ing is the reason w hy this transfer is able to occu r? (A) Fetal hem oglobin has a d issociation cu rve shifted to the right relative to that for ad u lt hem oglobin. (B) Fetal hem oglobin interacts w ith 2,3-d iphosphoglycerate. (C) Fetal hem oglobin has a greater P 50 valu e than ad u lt hem oglobin. (D) Fetal hem oglobin has a greater affinity for oxygen than ad u lt hem oglobin at any given p artial p ressu re. 137. A 3-m on th -old in fan t, 46 w eeks p ostconcep tu al age, requ ires an ingu inal hernia rep air. The child w as born by a sp ontaneou s vagin al d elivery, h ad an u n com p licated

p erinatal cou rse, and w as d ischarged hom e 2 w eeks later. Which one of the follow ing statem ents is true? (A) In ord er to m inim ize d isru p tion of feed ing and sleep , the infant shou ld be op erated on as an am bu latory su rgical p atient. (B) Sp inal anesthesia is not a su itable technique. (C) Ind u ction of anesthesia w ith sevoflu rane is inad visable becau se of the relatively slow up take of inhalational agents in infants. (D) H alothane m ay cau se d ep ression of the chem oreceptor pathw ays, bu t is a suitable choice of inhalational agent. (E) A cau d al ep id u ral injection shou ld not be p erform ed d u e to the m ore cau d al location of the sp inal cord in the neonate com p ared to ad u lts and old er child ren. DIRECTION S: Use the follow ing scenario to answ er Qu estions 138-139: A p atient w ith a history of asym m etrical card iac sep tal hyp ertrophy (H OCM) is u nd ergoing a liver resection for hepatocellu lar carcinom a. Du ring the case there is su d d en large volu m e blood loss. In ad d ition to volum e rep lacem ent, the patient is started on phenylephrine, follow ed by ad d ition of ep inep hrine d u e to p ersistent hyp otension. Shortly after initiation of epinephrine infu sion, the p atient has visible cyanosis and w orsening hem od ynam ic instability. 138. An em ergent TEE p erform ed intraop eratively is most likely to reveal w hich one of the follow ing find ings? (A) Dilated right atriu m and ventricle, and collapsed left atriu m and ventricle (B) Decreased left ventricu lar fu nction (C) Severe m itral regu rgitation (D) Large VSD (E) Aortic d issection 139. Which one of the follow ing interventions is m ost likely to im p rove this p atient’s hem od ynam ic instability?

Que s tions : 134–142

(A) (B) (C) (D) (E)

Ad d ition of m ilrinone Discontinu ation of ep inep hrine Ad m inistration of IV calciu m Discontinu e volu m e rep lacem ent Ad d ition of nitroglycerin

140. A 5-year-old child w ith obstructive sleep apnea presents for tonsillectomy and ad enoid ectom y. The child has no other p ast m ed ical history. The p arents note that the p atient snores lou d ly and has frequ ent p au ses in breathing w hen sleep ing. All of the follow ing are p ossible comp lications or necessary interventions in this child EXCEPT (A) p haryngeal airw ay obstru ction m ay occu r on ind uction of anesthesia (B) u se of an oral airw ay m ay be necessary d u ring ind u ction of anesthesia (C) continu ou s p ositive airw ay p ressu re shou ld not be u sed as it m ay inflate the stom ach and m ake the patient m ore likely to have p ostop erative nau sea and vom iting (D) sm aller than exp ected d oses of op ioid m ay prod u ce pronou nced respiratory d epression (E) d ifficu lty in p rop er p lacem ent of an LMA m ay be encou ntered DIRECTION S (Questions 141-142): Each grou p of item s below consists of lettered head ings follow ed by a list of nu m bered p hrases or statem ents. For each nu m bered phrase or statem ent, select the ON E lettered head ing or com p onent that is m ost closely associated w ith it. Each lettered head ing or com p on en t m ay be selected on ce, m ore than on ce, or n ot at all. (A) (B) (C) (D) (E)

Intrap arenchym al hem orrhage Dissem inated intravascu lar coagu lation H yp otherm ia Liver failu re Urem ia

407

(F) Factor VIII d eficiency (G) Von Willebrand d isease (H ) Salicylate toxicity For each patient with head trauma, select the most appropriate diagnosis to explain the hemorrhage. 141. A mid d le-aged man is brought to the operating room em ergently for op en head inju ry after an assau lt that occurred at least three hours before presentation to the hospital. H e has a d epressed sku ll fractu re an d scalp laceration w ith exp osed brain; he is bleed ing actively from the scalp. N o previous med ical history is available. Blood alcohol content is 0.5% by volume. H eart rate is 128 and non-invasive blood p ressure is 85/ 35; esop hageal tem p eratu re is 32°C after in d u ction of gen eral an esth esia. Du rin g d ebrid em ent of the brain there is a significant brain laceration and m arked bleed ing from the scalp requ irin g rep lacem en t w ith m u ltip le u nits of p acked red blood cells to su p p ort intravascu lar volu m e; hem atocrit is norm al after transfu sion. Intraop eratively, the PT and aPTT are p rolonged ; fibrinogen and p latelet cou nt are red uced . 142. A 25-year-old m ale w ith kid ney failu re from glom eru lar nep hrop athy requ iring d ialysis three tim es w eekly is brought to the op erating room em ergently for evacu ation of a su bd u ral hem atom a that d evelop ed after a blow to the head d u rin g a softball gam e. Th e p atien t und erw ent d ialysis 36 h earlier; BUN is 67 mg/ d L and creatinine is 8.3 m g/ d L. H e presented to the em ergency d epartm ent w ith a head ache bu t su ffered a p rogressive d ecline in m ental statu s. After su ccessfu l ind u ction of general anesthesia, the craniu m w as op ened and the hem atom a rem oved . Intraop eratively the PT, aPTT, and p latelet cou nts are norm al. Profu se bleed ing from the scalp and d u ra, requ iring transfu sion of packed red blood cells, com plicates closu re.

408

20: P ra ctice Te s t

DIRECTION S (Qu estions 143-146): Each grou p of item s below consists of lettered head ings follow ed by a list of nu m bered p hrases or statem ents. For each nu m bered phrase or statem ent, select the ON E lettered head ing or com p onent that is m ost closely associated w ith it. Each lettered head ing or com p on en t m ay be selected on ce, m ore than on ce, or n ot at all. (A) (B) (C) (D)

Acu te alcohol intoxication Deliriu m Deliriu m trem ens Dem entia

For each p atient w ith agitation, select the m ost likely d iagnosis. 143. A 32-year-old fem ale is in the hosp ital recovering after rep air of a right low er extrem ity fracture sustained three d ays ago after jum ping off a tram poline. On m orning rou nd s, the patient is d isoriented , trem u lou s, and p u lling at her gow n and IV lines. H er vitals are T 37.7°C, H R 121, BP 164/ 89, RR 26, Sp O 2 96%. 144. An 83-year-old fem ale is in the ICU recovering from a colon resection for d iverticu litis. On m orning rou nd s, you find the patient talking to her incentive sp irom eter. As you approach, she is startled and asks how you got into her h ou se. H er vitals are T 37.3°C, H R 89, BP 142/ 73, RR 18, Sp O 2 97% on 2 L O 2 via nasal cannu la. When you retu rn to see the patient w ith you r attend ing, she greets you by nam e and recou nts the final m om ents of last night’s televised basketball gam e. 145. A 74-year-old female is brought to her internist’s office by her d aughter. She has becom e concerned because her mother has gotten lost while in familiar environments and has forgotten to pay several bills over the last several months.

The patient is pleasant. When asked about her symptom s, she su ggests that her d aughter is overreacting. H er Mini Mental Statu s Exam score is 20. Her vitals are T 36.9°C, HR 72, BP 128/ 78, RR 16, Sp O 2 98%. 146. A 67-year-old fem ale is brou gh t to th e Em ergency Dep artment by p aram ed ics after a m otor vehicle collision. She has a laceration of her left forearm . When she stand s to change into the hosp ital gow n, she is u nstead y and nearly falls. She falls asleep on the stretcher. When the staff w akes her to assess her vitals, sh e is u n coop erative an d con fu sed . Sh e d em and s to be left alone. DIRECTION S (Qu estions 147-148): Each grou p of item s below consists of lettered head ings follow ed by a list of nu m bered phrases or statem ents. For each nu m bered p hrase or statem ent, select the ON E lettered head ing or com p onent that is m ost closely associated w ith it. Each lettered head ing or com p on ent m ay be selected on ce, m ore than on ce, or n ot at all. (A) (B) (C) (D) (E) (F) (G) (H )

Ep inep hrine Sod iu m bicarbonate Atrop ine Calciu m Volu m e exp and er Dobu tam ine Milrinone Albu m in

For each p atient, select the m ost ap p rop riate m ed ication or therapy. 147. After birth, a new born rem ains ap neic w ith a heart rate of less than 60 beats p er m inu te d esp ite ad equ ate ven tilation an d ch est com p ressions.

Que s tions : 143–150

148. This d ru g is no longer recom m end ed d u ring the resuscitation of the new born. DIRECTION S (Qu estions 149-150): Each grou p of item s below consists of lettered head ings follow ed by a list of nu m bered p hrases or statem ents. For each nu m bered p hrase or statem ent, select the ON E lettered head ing or com p onent that is m ost closely associated w ith it. Each lettered head ing or com p on ent m ay be selected on ce, m ore than once, or not at all. (A) (B) (C) (D) (E) (F) (G) (H ) (I) (J) (K) (L) (M) (N ) (O)

Mid -esop hageal fou r cham ber Mid -esop hageal tw o cham ber Mid -esop hageal long axis Transgastric tw o cham ber Transgastric m id -p ap illary short axis Mid -esop hageal aortic valve short axis Mid -esop hageal aortic valve long axis Mid -esop hageal bicaval Mid-esophageal right ventricular inflow outflow Deep transgastric long axis Up p er esop hageal aortic valve short axis Up p er esop hageal aortic valve long axis Transgastric long axis Mid -esop hageal ascend ing aortic short axis Mid -esop hageal ascend ing aortic long axis

409

For each p hotograp h of a transesop hageal echocard iogram , select the stand ard , tw o-d im ensional tom ographic view. 149.

150.

Answe rs a nd Expla na tions

1. (E) Clam p ing of the d istal aorta in a p atient w ith no p reop erative aortic occlu sion m ost likely lead s to an increased vascu lar resistance and blood p ressu re and to d ecreased stroke volume and card iac ou tput. The heart rate usu ally is stable. (5:1025-7) 2. (A) The oculocard iac reflex (OCR) is associated w ith brad yarrhythm ias. Traction on the extraocu lar m u scles and p ressure on the eye m ay cause the reflex. The reflex exhibits fatigability. Regional anesthesia m ay be u sed to p revent the OCR but it m ay also precip itate the reflex. (5:1220) 3. (B) H etastarch is a synthetic colloid . Cau tion shou ld be exercised w hen ad m inistering hetastarch solu tions to p atien ts w ith severe sepsis/ septic shock, since it m ay increase the risk of acute kid ney inju ry. The starch molecule is m od ified to resist m etabolism by am ylase to glu cose m onom ers. The d ru g interferes w ith coagu lation by d ilu ting p latelets and coagu lation factors. The incid ence of anap hylactoid reactions is abou t 1 in 1000. H etastarch is elim inated p rim arily via the kid neys. (5:541) 4. (E) The u teru s is com p osed of sm ooth m u scle and therefore u naffected by MG. Therefore the first stage of labor shou ld be u naffected . In contrast, the second stage of labor involves m aternal expu lsive efforts and m ay be affected by the d isease p rocess. MG is associated w ith other au toim m u ne d iseases like rheu m atoid arthritis and p olym yositis. The cou rse of MG d u rin g p regn an cy is variable. (2:1059-61; 5:995-6)

410

5. (D ) Patients w ith MG have a variable response to su ccinylcholine, but it is not contraind icated in p artu rients w ith MG. In general, there is a resistance to the p aralytic effects of d ep olarizing neu rom u scu lar agents and an increased sensitivity to nond epolarizing agents. Opioid s shou ld be u sed w ith caution as these patients m ay be more susceptible to respiratory d epression. N eu raxial anesthesia is the p referred method of analgesia for labor and cesarean section, how ever, p atients w ith severe d isease w ith bu lbar and resp iratory involvem ent m ay n ot tolerate n eu raxial an esth esia and m ay require general anesthesia to protect the airw ay and p revent asp iration. (2:1060; 5:995-96) 6. (B) Most d osage gu id elin es for p ed iatric p atients have been d erived from d ata extrapolated from ad u lt stu d ies. H ow ever, in a child the d ose shou ld not exceed a total d ose of 3 m g/ kg. In this case, the child is 12 kg, and cou ld therefore receive u p to 36 m g for local infiltration. The su rgeon is u sing 0.5% bu pivacaine, and so the m axim u m volu m e is 7 m L. (5:846) 7. (A) Azithrom ycin and levofloxacin are associated w ith p rolonging the QT interval and are th erefore con train d icated . Clind am ycin is effective against the com m on skin p athogens and w ou ld be the p referred alternative to a first-generation cep halosp orin. Gentam icin is generally not effective as a single agent against the com m on Gram p ositive skin p athogens, w hile tetracycline is too broad -sp ectru m to be u sed in rou tine p eriop erative p rop hylaxis. (5:108, 232-4; 6:1897-8)

Answe rs : 1–14

8. (D ) The volu m e control, d ecelerating w aveform m od e is show n. From top to bottom , the figure d epicts flow, pressure and volum e over tim e. Pressu re su p p ort ventilation (PSV) requires patient insp iratory effort to trigger the breath and is seen as a negative d eflection in the pressure w aveform immed iately preced ing a breath; the insp iratory flow rate varies d epend ing upon patient effort. Tid al volume in pressure support may be variable and is d epend ent upon the level of pressure support, rise time, inspiratory effort of the patient, and lung m echanics. Pressure control ventilation (PCV) requ ires the clinician to set a p ressu re and a resp iratory rate. Tid al volu m e is variable. Insp iratory flow show s a d escend ing w aveform and is d etermined by the level of pressure control, resp iratory com p liance, and airw ay resistance. Volu m e control ventilation (VCV) requ ires the clinician to set a tid al volu m e, respiratory rate, flow w aveform, and inspiratory time. Tid al volume is constant in this mode w hile pressure varies d epend ing upon respiratory com p lian ce an d airw ay resistan ce. Inspiratory flow d uring VCV is fixed and m ay be set to a constant flow that creates a rectangular flow w aveform or d escend ing-ram p that creates a d ecelerating flow w aveform as d epicted in the figure. Airw ay pressure release ventilation (APRV) is a new er ventilator mod e that is an adjunct to continuous positive airw ay p ressure (CPAP). APRV ad d s alveolar ventilation to CPAP by transiently d iminishing airw ay p ressure to a low er level then restoring it to the higher level. This m od e allow s sp ontaneou s breaths at both CPAP levels, but m ay be associated w ith d yssynchrony. A theoretic ad vantage to APRV is the potential recruitm ent of atelectatic lung units at low er pressure levels than conventional ventilator mod es. N otably, new er ventilatory mod es have not been convincingly show n to improve respiratory or mortality outcom es over properly em ployed conventional ventilation. (5:1406-9, 1413; 6:2211-3) 9. (B) Althou gh carbon d ioxid e is som etim es ad d ed to the fresh gas d u ring byp ass, respiratory acid osis or alkalosis is generally corrected through changes in the fresh gas flow to the oxygenator. (5:899)

411

10. (E) The child should have an ind uction that is m ost appropriate for his clinical cond ition. If there has been a large am ou nt of blood lost, a rap id sequ ence ind u ction w ith p ropofol and su ccin ylch olin e m ay n ot be ap p rop riate. Passing a nasogastric tu be m ay not rem ove all of the stomach contents. The proced ure should not be d elayed to allow the stom ach to em pty. Once the inju ry has occu rred , the stom ach emptying probably stops, and the contents w ill still be there 6 h later. The child shou ld be allow ed to aw aken w ith the end otracheal tu be in p lace and be extu bated once p rotective reflexes are intact. (3:768-9) 11. (D ) Marfan synd rome is not associated w ith an elevated expected blood loss. The rem aining types of scoliosis d o have high expected blood loss. (3:636) 12. (A) There is no significant change in the respiratory rate at term . The increase in m inute ventilation is almost entirely related to the increase in tid al volu m e. (2:19-21; 5:290-91) 13. (A) Maintenance of norm ocarbia is recom m end ed d u ring carotid end arterectom y. While hyp ocap nia can cau se cerebral vasoconstriction, hypercapnia w ith resulting cerebral vasod ilatation cou ld cau se steal p h en om en a. Vasod ilatation has not p roven to be help fu l. Oxygen tension and blood pressure are kept at levels that are normal or slightly above normal. N orm oth erm ia is u sed , as is system ic heparinization. (5:1018-21) 14. (A) The d ifferent typ es of opioid receptors contribu te in d ifferent p rop ortions to the total op ioid receptors in the spinal cord . µ-op ioid recep tors constitu te 70%, κ-op ioid recep tors 24%, and δ-op ioid recep tors 6%. The m ain m ech an ism of sp in al op ioid an algesia is through presynaptic activation of opioid receptors. Opioid recep tors are synthesized in sm all d iam eter d orsal root ganglion cell bod ies and transp orted centrally and p erip herally. They are m ainly (70%) located p resynap tically on sm all d iam eter nociceptive prim ary afferents (C and A-δ fibers). (1:491)

412

20: P ra ctice Te s t

15. (A) The hem atocrit reaches the low est valu e betw een 8 and 12 w eeks of age in fu ll term infants. This is norm al and d oes not requ ire a blood transfu sion or iron su p p lem entation, and it is not d u e to hem od ilu tion. This su rgery, though not em ergent, is not a proced u re that need s to be d elayed u n til the h em atocrit increases. It m ay be best to avoid certain elective proced ures that m ay resu lt in significant blood loss at this tim e. (5:257-8) 16. (C) Althou gh no specific anesthetics are ind icated or contraind icated , those m ed ications that cau se release of catecholam ines shou ld be avoid ed if p ossible. Marked increases in heart rate are noted d u ring ind u ction w ith d esflu ran e anesthesia or abru p t in creases in the d elivered concentrations of d esflu rane d u e to stimulation of the sympathetic nervous system. (1:545; 5:1125-6) 17. (A) Ventilation of the patient w ith a head injury requ ires m eticu lou s attention. H ead elevation red u ces cerebral ven ou s con gestion an d thereby low ers ICP. After prolonged hyperventilation the efficacy at red u cing ICP is red u ced as the p H of CSF retu rns to norm al. While increases in intrathoracic p ressu re shou ld be avoid ed , PEEP is usefu l w hen u sed ju d iciou sly to m aintain oxygenation and avoid hyp oxia and hyp ercarbia. The u se of PEEP shou ld be w eighed against the potential to increase intracranial p ressu re. (5:880, 1360-1; 6:3377) 18. (C) Esm olol is a β 1-ad renocep tor antagonist and is contraind icated in the p atient w ith AV block. Its half-life is less than 15 m in. It is less likely than p rop ranolol to cau se bronchospasm , since it blocks β 1-ad renocep tor recep tors, not β 2-ad renocep tor recep tors. (1:327) 19. (B) The bronchial tree d ivid es into right and left bronchi, the left bronchu s being narrow er and longer. Foreign bod ies are m ore apt to go to the right sid e. The trachea moves d uring respiration and w ith m ovem ent of the head . It is lined w ith p seu d ostratified colu m nar ep itheliu m . The rings of cartilage d o not com p letely en circle the trach ea; th e p osterior w all is

m em branou s for all the rings excep t the cricoid . (5:965) 20. (D ) H yp oalbu m inem ia, w hich is a com m on find ing in critically ill p atients, can lead to u nd erestimation of the anion gap and a m issed d iagnosis of an increased anion gap m etabolic acid osis. Albu m in is a strong anion; a red u ction in seru m albu m in level throu gh d ilu tion, cap illary leak, or d ecreased hepatic p rod uction resu lts in a m etabolic alkalosis. Consequ ently, calculation of the anion gap w ithou t accou nting for the abnorm al albu m in level m ay resu lt in an anion gap in the ”norm al” range and a failu re to ap p reciate u nm easu red anions. To accou nt for this flaw in the anion gap approach to assessing metabolic acid osis, an equation for the corrected anion gap has been suggested : Anion Gap corrected = Anion Gap calcu lated + 2.5 × (N orm al albu m in g/ d L − Observed album in g/ d L) Becau se the Stew art-Fencl (strong ion d ifference) ap p roach to acid -base assessm ent incorp orates albu m in into calcu lations, it may be a m ore u sefu l tool for critically ill patients. (5:365; 6:526-7, 530-1) 21. (C) Mannitol, by d ecreasing brain size, m ay p rod uce traction and even tearing of subd ural veins that resu lt in hem atom a form ation. This is m ore of a p roblem in the eld erly. Mannitol m ay also lead to cerebral ed em a w hen the blood –brain barrier is d isru p ted . (1: 681-2; 5:880; 6:2255-7) 22. (C) The p ain of glossop haryngeal neu ralgia is very sim ilar to that of trigem inal neu ralgia but affects the posterior third of the tongue, tonsils, and p harynx. Giant cell arteritis is a com m on system ic vascu litis in the eld erly. It is com m only associated w ith visu al loss and strokes, so it m u st be d iagnosed and treated aggressively. Tem poral artery biopsy is the gold stand ard in the d iagnosis of giant cell arteritis. Glucocorticoid s are a com m on m od e of treatm ent. Cervical carotid artery d issection m ost com m only presents w ith head , facial, or neck p ain. Other com monly seen symptom s includ e

Answe rs : 15–28

H orner synd rom e, p u lsatile tinnitu s, and cranial nerve palsy. Pu re facial p ain is m ost often caused by sinu sitis and the chew ing ap paratu s, bu t also a m u ltitu d e of other cau ses. (7:246-9) 23. (C) Larger tid al volu m es increase airw ay p ressu res and lu ng com p liance w ithou t significantly im proving arterial oxygen tension but resu lt in severe h yp ocarbia th at in creases shunt fraction at a Pa CO 2 less than 30 m m H g. Arterial oxygenation d u ring lap aroscop y in m orbid ly obese patients is affected m ainly by bod y w eight and not bod y p osition, p neu m op eritoneu m , or m od e of ventilation. PEEP is the only ventilatory param eter that has been show n to im p rove resp iratory fu nction in obese p atients, bu t it m ay d ecrease venou s retu rn, card iac ou tpu t, and su bsequ ent oxygen d elivery. (5:314) 24. (B) Althou gh p lasm a cholinesterase levels d o d ecrease by roughly 25% before d elivery and by 33% on postp artu m d ay three, this u su ally d oes not resu lt in a clinically relevant p rolongation of p aralysis from a single d ose of su ccinylcholine. This m ay in p art be d u e to the larger volu m e of d istribu tion in p regnancy. The increase in blood volum e is d u e to both an increase in p lasm a and red cell volu m e. The greater in crease in p lasm a volu m e (55%) versu s red cell volu m e (30%) resu lts in the p hysiologic anem ia of pregnancy. (2:21-3) 25. (B) An FEV1 of less than 800 m L and an FVC of less than 2000 m L are associated w ith higher m orbid ity and p oorer fu nction al ou tcom e. Regional fu nction stu d ies, like the V/ Q scan, can show w hat fraction of the p reop erative lu n g fu n ction com es from the area to be resected . Patients w ith an elevated Pa CO 2 are alread y in a state of chronic respiratory failu re. Low Pa O 2 can reflect either p oor u nd erlying p u lm onary fu nction, or a shu nting of blood through the d iseased and nonventilated lung regions. When this area is resected , hypoxemia m ay resolve. (5:952-5; 6:2087-91)

413

26. (C) H yp oxia is likely cau sed by high p hysiologic shu nt in the d iseased lu ng. Physical therap y m ay help clear secretions and im p rove oxygen ation . Su p p lem en tal oxygen w ill im p rove d elivery to the good lu ng and reverse som e hyp oxic vasoconstriction on that sid e. Lateral p osition w ill im p rove flow to the good lung, as long as it d oesn’t increase soilage from th e d iseased sid e. Pn eu m on ectom y w ill rem ove the d iseased lu ng and elim inate blood flow to that sid e. An intravenou s vasod ilator, like sod iu m nitrop ru ssid e, w ill exacerbate shu nting by inhibiting hyp oxic p u lm onary vasoconstriction. (5:967-70) 27. (D ) A d ou ble lu m en end otracheal tu be fu lfills several fu nctions in a case that requires onelu ng ventilation: 1. isolation of the op erative sid e; 2. the p ossibility of ap p lication of m ild CPAP to the op erative lu ng in ord er to m aintain oxygenation (and therefore lim iting interru p tions of the su rgical p roced u re d u e to reinflation of the op erative lu ng); and 3. the p ossibility of su ctioning both the op erative and non-op erative lu ng. For these reasons, a left sid ed tu be is p referred since it is u nlikely to interfere w ith a lobectomy and its placement is significantly easier and more predictable d ue to the increased length betw een carina and left u pper lobe bronchu s as com pared to the right sid e. N either a single-lu m en end otracheal tube, nor an LMA provid e lung isolation, how ever a single-lu m en tu be w ith a balloon catheter (Univent) is an accep table alternative to a d ou ble-lu m en tu be how ever it d oes not allow for CPAP and su ctioning of the op erative lu ng. (5:963-7) 28. (E) If the right sid e cannot be ventilated , there m u st be no clear channel from the right lung to the anesthesia circu it. Tu m or or secretions cou ld obstruct the lum en. The bronchial cuff cou ld block the right sid e if the tu be is not ad vanced far enou gh. The right tu be lu m en m ight be in the left m ainstem bronchus if the tube is ad vanced too far. Placing the tube into the right sid e shou ld not stop right-sid ed ventilation. (5:963-5)

414

20: P ra ctice Te s t

29. (B) Particu larly likely cau ses of this scenario are m assive hem orrhage from the p u lm onary artery stum p or other great vessel, herniation of the heart through a p ericard ial w ind ow, or arrhythm ia from card iac irritation, hyp oxia, or right heart failu re. Ap p lying su ction to the chest tu be cou ld exacerbate the m ed iastinal shift, vascu lar com p rom ise, and card iac herniation (tension vacuthorax), as w ell as speed th e h em orrh age. Su rgical exp loration is u rgently need ed , and the patient should also receive ap p rop riate su p p ortive care. (5:1004-6) 30. (E) A history of seizu re d isord er occu rs in app roxim ately 0.5% of p artu rients. One-third of these patients w ill experience an increase in seizure frequ ency d u ring pregnancy. Proposed m echanism s inclu d e a d ecreased anticonvu lsant d rug concentration second ary to a larger volum e of d istribution, poor com pliance, and a greater clearance of the d ru g d u ring p regnancy. A high estrogen concentration is also know n to low er the seizu re threshold . Dru g concentrations shou ld be checked and the anticonvu lsant d rug continu ed , preferably in the p arenteral form , in the p erip artu m p eriod . Dru gs su ch as ketam in e an d m ep erid in e should be avoid ed as these are know n to low er the seizure threshold . (2:1061-3) 31. (A) The agents are listed in d ecreasing ord er of β -ad ren ocep tor to α -ad ren ocep tor activity. (1:201,206; 5:1401) 32. (C) This p atient has d evelop ed a severe A-a grad ient w ith a Pa O 2/ FIO 2 ratio of < 100. Since there is no evid ence of card iogenic pulm onary edema, this is consistent with ARDS. Ventilatory strategies in patients w ith ARDS aim to support the p atient w ithou t increasing lu ng inju ry throu gh alveolar overd istension or recu rrent alveolar collap se. The ARDSnet stu d y p u blished in 1999 d emonstrated red uced m ortality w hen patients w ith ARDS are ventilated w ith ”lu ng-p rotective ventilation,” inclu d ing low (6 m L/ kg id eal bod y w eight) tid al volu m es.

PEEP is titrated to m inim ize FIO 2, m axim ize Pa O 2, and optimize alveolar recruitm ent. This typically occurs at >10 mm Hg. This patient has an id eal bod y w eight of 50 kg. Male id eal bod y w eight (in kg) = 50 + 2.3 × (height in inches − 60) Female id eal bod y w eight (in kg) = 45.5 + 2.3 × (height in inches − 60) (5:1392, 1412, 1415, 1418, 1420; 6:2207-8) 33. (A) This p atient is hem od ynam ically stable. Althou gh the d ebate over transfusion triggers continu es, there is no evid ence that su pports transfusion of an otherw ise healthy critically ill p atient w ith a hem oglobin >7 g/ d L and no evid ence of hyp op erfu sion. The Transfu sion Requirements in Critical Care (TRICC) trial d id not d em onstrate a benefit for a m ore liberal transfu sion strategy in the stu d y p opu lation. Transfu sion of fresh frozen p lasm a (FFP) is necessary to avoid coagu lopathy d u ring m assive transfu sion. Once bleed ing and transfusion requ irem ents have stopped , how ever, the n eed for FFP is su bstan tially red u ced . Prothrom bin tim e is a poor p red ictor of su rgical bleed ing, and large volu m es of FFP m ay be required to im p rove a m ild ly elevated IN R. A p latelet cou nt of >50,000/ m m 3 is the u su al target level for invasive p roced ures. (5:201-3, 1394-5, 1442-3; 6:953) 34. (D ) Th e d ifferen tial d iagn osis for n on card iogenic pulm onary ed em a in an intubated patient includ es aspiration pneumonitis, pneu m onia, transfu sion related acu te lu ng inju ry (TRALI), p u lm onary em bolism , or sep sis. This p atien t’s norm al echo and h em od ynam ics, rap id recovery, lack of fever, and su bstantial exp osu re to blood p rod ucts m akes TRALI the m ost likely etiology. TRALI presents as resp iratory d istress and noncard iogenic p u lm onary ed em a that begins d uring, or w ithin 4-6 h of a transfu sion. Plasm a is generally the cu lp rit blood p rod u ct. Treatm ent is su p p ortive, and p atients u su ally recover w ithin 48 h. TRALI is

Answe rs : 29–41

the lead ing cau se of transfu sion-related m ortality. This p atient d id not have a history of trauma that w ould make pulmonary contusion likely. (5:203-4, 1400; 6:280-1, 955) 35. (A) Positive end -exp iratory p ressu re (PEEP) red u ces preload and left ventricu lar afterload by increasing intrathoracic p ressu re. (6:2237) 36. (B) Daily interru ption of sed ation and spontaneou s breathing trials have been show n to red u ce the nu m ber of d ays requ iring m echanical ventilation and ICU care in eligible patients. Lung recruitment maneuvers have been show n to open the lungs in ARDS p atients, bu t this h as n ot resu lted in an ou tcom e ben efit. Prop hylactic antibiotics p u t the p atient at higher risk of colonization w ith pathogens and have not been show n to red uce ventilator associated pneum onia. Suctioning can d am age the tracheal m ucosa, increasing the risk of tracheal colonization. Suctioning m ay also cau se au toinocu lation from the end otracheal tu be biofilm to the more d istal airw ays. N one of the ventilator m od es have been show n to im prove outcome. (5:1406, 1418, 1421-2; 6:1115, 2138, 2200-1, 2213-4) 37. (A) The atyp ical antip sychotics have less risk of extrap yram id al effects. They can cau se hyp otension and clozap ine, olanzap ine and qu etiap ine h ave been associated w ith an increased risk of new -onset typ e 2 d iabetes as w ell as increased ap p etite and w eight gain. Anticholin ergic effects are least frequ ently cau sed by p oten t an tip sych otics like halop erid ol. (1:359, 440-1) 38. (C) General anesthesia p rovid es a rapid ind u ction w ith less hypotension and less card iovascu lar instability and a more secu re airw ay once it is established . An increased risk of aspiration and the potential for a d ifficu lt intu bation lim it

415

gen eral an esth esia to situ ation s in w h ich regional anesthesia is not op tim al, su ch as w hen there is a need for em ergent d elivery in a patient w ithou t an ep id u ral in p lace or the n eed for u terine relaxation (e.g., d ifficu lt breech extraction , rep lacem en t of u terin e inversion). (2:534-5; 5:1158) 39. (C) A bacterial infection is likely to have resp ond ed to antibiotic therap y. A w heezy child m ay not be ”asthm atic” bu t m ay have asp irated a foreign bod y. In a 2-year-old child this d iagnosis should alw ays be consid ered a p ossibility even if there is no clear history of aspiration. A child presenting w ith these symp tom s and signs requ ires em ergency therap y w hatever the d iagnosis. Agitation m ay be m isinterp reted as em otional u p set w hen it is d u e to serious u nd erlying hypoxem ia. If the child is stable, x-rays m ay be help fu l in m aking the d iagnosis and in id entifying and localizing the foreign bod y. H ow ever, if the child is severely d istressed , oxygen shou ld be ad m inistered by face m ask and im m ed iate p lans m ad e for rem oval of the foreign bod y in the op erating room . (5:1002, 1250) 40. (B) The flow d u ring byp ass is virtu ally nonp ulsatile in both flow and p ressu re. This has been stu d ied to d eterm ine if a m ore p hysiologic p attern w ou ld be beneficial. (5:899) 41. (C) Generally, solu tions containing d extrose are n ot ad m in istered d u rin g n eu rosu rgery u nless ind icated for treatm ent of hyp oglycem ia. H yp erglycem ia has been im p licated in anim al exp erim ents in w orsening neu rologic ou tcom es after ischem ia. A p lasm a glu cose concentration in excess of 300 m g/ d L can p rod u ce an osm otic d iu resis. Dextrose containing solu tion s d o n ot lead to flu id reten tion . (5:1471-2)

416

20: P ra ctice Te s t

42. (B) In p op u lation -based stu d ies, m igrain e w ithou t au ra is abou t tw ice as frequ ent as m igraine w ith au ra. Major criteria for the d iagnosis of cervicogenic head ache inclu d e signs and sym p tom s of neck involvem ent su ch as the precipitation of head pain by neck m ovement or external pressure over the upper cervical or occip ital region, restricted range of m otion in the neck, u nilaterality of head pain w ith or w ithout should er or arm pain, and confirm atory evid ence by d iagnostic anesthetic blocks. Chronic tension-typ e head ache d iffers from ep isod ic tension-typ e head ache in that the average head ache frequ ency is equal to or greater than 15 d p er m onth or 180 d p er year. A shift from p eripheral to central m echanism s is believed to p lay a role in the evolu tion of ep isod ic to ch ronic tension-typ e head ache. Clu ster head aches typ ically p resent as a series of intense u nilateral head aches occu rring over a period of 2 w eeks to 3 months. They are associated w ith u nilateral autonom ic features such as nasal congestion, rhinorrhea, m iosis, or lacrim ation. The attacks are u su ally brief, lasting 150-180 m in, and occu r in the orbital, su p raorbital and/ or temporal regions. Unlike migraine head aches, tension-typ e head aches, tem p oral arteritis, and cervicogenic head aches, clu ster head aches are m ore frequ ent in m en, w ith an average m ale to fem ale ratio of 5 to 1. Tricyclic antid epressants have been show n in numerous clinical trials to be effective in the prevention of both m igraine and tension-type head aches. (7:194-201) 43. (B) Dexm ed etom id ine is an α 2-ad renocep tor agonist w ith 1600-fold greater selectivity for the α 2- than for the α 1-recep tor. It is ap p roved for short term sed ation (≤ 24 h) but can be u sed as an ad ju nct for general anesthesia as w ell w here it can m arked ly red u ce the MAC of inhalational anesthetics. It cau ses less resp iratory d epression than opioid s and benzod iazepines, and has not been associated w ith ad renal su ppression. When ad m inistered as an intravenous bolus, it w ill cause an initial increase in blood p ressu re and d ecrease in heart rate. Dexm ed etom id ine is not in d icated for the ind uction of general anesthesia. (1:548-9; 5:701)

44. (A) Preeclam p tic p atients are intravascu larly volume d epleted d espite the fact that they may exhibit ed em a and w eight gain. This m ay be d u e to a variety of reason s in clu d in g a d ecreased colloid osm otic p ressu re and an increased vascu lar p erm eability from cap illary leakage. Glom eru lar filtration rate (GFR) norm ally increases d uring pregnancy, thu s low ering concentrations of seru m m arkers of renal clearance (e.g., creatinine). In p reeclam p sia, GFR is d ecreased resu lting in an increase in creatinine concentrations. Elevated u ric acid concentrations reflect renal involvem ent w ith d ecreased renal clearance. The hem atocrit is falsely elevated d u e to a red u ction in p lasm a volum e. (2:983-4) 45. (D ) Carbon d ioxid e is som etim es ad d ed w hen the p H -stat strategy is u sed to ad ju st Pa CO 2 d u ring hyp otherm ia. (5:532) 46. (A) Absolu te contraind ications to ep id u ral steroid injections inclu d e sepsis, infection at injection site, therap eu tic anticoagu lation, and p atien t refu sal. Relative con traind ications includ e preexisting neurologic cond itions, prop hylactic low -d ose hep arin, throm bocytop enia, and u ncoop erative p atients. (A bdi S, et al., Pain Physician 2007; 10:185-212; Cannon DT, et al., A rch Phys M ed Rehabil 2000; 81:S-92) 47. (E) When ad vancing w ith a need le, the ligam entu m flavu m feels firm and cru nchy in the m id line. When the need le p asses throu gh the ligam entu m flavu m , the ep id u ral sp ace is encou ntered as a d istinct loss of resistance. (5:793) 48. (C) TURP is p erform ed by resecting p rostatic tissu e w ith a cautery loop introd u ced throu gh a sp ecial cystoscop e. Du ring the resection, venou s sinuses are opened , and the irrigation fluid can be absorbed into the system ic circulation. As a resu lt, a com p lication know n as TURP synd rome may d evelop, w ith symptoms related to flu id overload , hyp oosm olality, and hyponatremia. During TURP, system ic absorp tion of the irrigating solu tions is influ enced by the d u ration of exposu re, the num ber and size

Answe rs : 42–53

of venou s sinu ses opened , extravasation of the flu id into tissu es ou tsid e the blad d er or p rostatic cap sule, and the hyd rostatic pressure of the flu id . When large volu m es of flu id are absorbed , severe h yp on atrem ia lead in g to cerebral ed em a m ay ensu e. Ad d itionally, neu rologic m anifestations m ay also occur d u e to d irect toxic actions of som e of the solu tes u sed in irrigation. When TURP synd rom e is su sp ected , the p roced u re shou ld be stop p ed . Seru m sod iu m , p otassiu m , and osm olality shou ld be m easured in ord er to d ifferentiate betw een tru e hyp oosm olality and hyp onatrem ia in the presence of circu lating solu tes su ch as glycine. H em oglobin shou ld be m easu red , becau se it is an ind ex of the extent of flu id absorp tion. H yp onatrem ia d oes not need to be treated aggressively w hen it is not accom panied by hyp oosm olality or in the absence of neurologic symptoms. If hyponatremia needs to be treated , rap id correction shou ld be avoid ed becau se it can cau se p ontine m yelinolysis. H yp ertonic saline shou ld be u sed only in the p resence of life-th reatening m anifestations such as com a and seizu res. Otherw ise, sod ium levels can be increased by ad m inistration of n orm al saline in com bination w ith a loop d iuretic or mannitol. Sodium correction should never exceed 1 to 1.5 m Eq/ L/ h. (5:1139-40, 514-5) 49. (C) Traction on a nu m ber of stru ctu res in the p osterior fossa m ay lead to arrhythm ias, the m ost common of w hich is brad ycard ia. The su rgeon shou ld be inform ed im m ed iately. If the brad ycard ia persists, atrop ine or glycopyrrolate m ay be ind icated . Air em bolism is m ore com m only associated w ith atrial and ventricular irritability and lid ocaine m ay be necessary to treat frequent premature ventricular systoles or ventricular tachycard ia associated w ith air em bolism . Althou gh it m ay be necessary to change the anesthetic or low er the head if air em bolism is su spected , the first m aneu ver is to inform the su rgeon of the p roblem and su sp ected d iagnosis. (5:893) 50. (E) Prostagland in E1 m ay cau se hyp otension d u e to vasod ilation. The d ru g shou ld not be

417

stop p ed before ind u ction, since it m ay be requ ired to keep a d u ctu s arteriosu s op en. The d ru g is ad m inistered by infu sion, since it has a short d uration. (1:948; 5:251) 51. (E) Post-herp etic neu ralgia m ost frequ ently occurs in eld erly patients d ue to reactivation of the varicella zoster viru s. It is usu ally refractory to opioid s. Som e patients respond to tricyclic an tid ep ressan ts or to th e top ical ad m inistration of cap saicin that cau ses d ep letion of Substance P. (1:1448) 52. (B) Given that the d u ration of su rgery is pred icted to be greater than 30 m in, op tion B is likely to be most efficaciou s. Retrograd e p erfu sion is a suitable alternative for cases lasting less than 30 m in. N one of the p harm acological strategies listed is p roven to be neuroprotective against cerebral ischemia d uring operative rep airs of the aortic arch. While p acking the p atient’s head in ice is rou tinely p erform ed d u ring d eep hyp otherm ic card iac arrest, it is not sufficient in isolation to protect the brain from ischem ia. (5:916-7) 53. (D ) This p atient has the clinical featu res of ventilator-associated p neu m onia (VAP). VAP is a seriou s infection w ith high attribu table m ortality. The m ost im p ortant first step s in m anagem ent of a p atient w ith su sp ected VAP is to obtain sp ecim ens for cu ltu re, then begin ap p rop riate antibiotic coverage. If cu ltu res are not obtained , clinicians w ill be u nable to n arrow an tibiotics, in creasin g th e risk for selecting for m u ltid ru g-resistant organism s. If antibiotics are given before cu ltu res are p erform ed , the cu ltu res m ay be inaccu rate. Postsp lenectom y vaccines are a stand ard p art of p ost-sp lenectom y care, and m ay red u ce the risk of fu tu re in fection w ith en cap su lated organism s in this vu lnerable p atient p op u lation. Vaccination w ou ld not be help fu l, how ever, in the acu te setting. Chest p hysiotherapy can be help fu l in the treatm en t of p atients w ith resp iratory failu re, bu t it is not the first p riority in m anagem ent. (6:1026, 1115-6, 2130, 2137-41, 2200)

418

20: P ra ctice Te s t

54. (E) Patients with VAP should receive early broad spectrum antibiotics targeted at likely pathogens. Inappropriate selection of the initial antibiotic regimen has been associated with higher mortality. This patient has been in the hospital for >48 h and has had prior antibiotic exposure, two risk factors for infection w ith multidrug-resistant (MDR) bacteria. The initial antibiotic regimen should cover MRSA and MDR gram-negative rod s such as Pseudomonas and A cinetobacter. Vancom ycin or linezolid w ill cover MRSA. Levofloxacin or cefepime will cover MDR gramnegative rods. (6:1115-6, 2130, 2137-41) 55. (A) Tailorin g antibiotics based on cu ltu re resu lts red u ces selection p ressu re for d evelop m en t of MDR bacteria and d ecreases th e patient’s exp osure to d ru gs w ith p otential sid e effects. This patient has had a clinical resp onse and has a spu tu m cu ltu re that is p ositive for Haemophilus influenzae. Broad spectru m antibiotic coverage should be d iscontinu ed and she should be placed on a single antibiotic that w ill cover the cau sative organism at the site of infection . Th ird gen eration cep h alosp orin s (inclu d ing ceftriaxone) are the d ru gs of choice for seriou s in fection s d u e to Haemophilus. (1:1499; 6:1115-6, 2130, 2137-41) 56. (B) Becau se the am plitud e of evoked potential responses is sm all (0.1–20 m V) as com pared to the stand ard EEG (>50 m V), signal averaging is requ ired to elim inate backgrou nd noise. Mu ltip le factors influ ence evoked resp onses in clu d in g an esth etics an d tem p eratu re. Resistance to these effects varies am ong the various potentials m easu red . (5:484-9) 57. (B) A p aracervical block is effective for the first stage of labor since it helps w ith the pain associated w ith cervical d ilatation. H ow ever, paracervical block has been largely aband oned in the United States d u e to a high incid ence of rep orted fetal com p lications and the availability of other techniqu es w ith a low er com p lication rate. A p u d end al block w ill p rovid e anesthesia for the second stage and is ap p ropriate for low forcep s d elivery and episiotomy. A su barachnoid block, cau d al block, or ep id u ral block is also ap p rop riate. (2:493-501)

58. (B) The bronchod ilator terbu taline is a selective β 2-ad renocep tor agonist. It cau ses less tachycard ia than isoproterenol. It is preferred in the patient w ith heart d isease com p ared to nonselective β -ad renoceptor agonists. (1:293) 59. (D ) The celiac p lexu s block is u sefu l for p ain associated w ith u pper abd om inal m alignancy. The most common complication is hypotension. (1:735; 5:1711) 60. (D ) Factors increasing p u lm onary resistance in clu d e n itrou s oxid e, h y p oxia , a n d hypercarbia. (1:547; 5:949) 61. (D ) There are three m ajor types of fetal d ecelerations. Early d ecelerations d em onstrate a slow d rop in heart rate beginning w ith the u terine contraction (UC) w ith the nad ir coincid ing w ith the p eak of the UC. It retu rns to baseline by the end of the UC. It is a resu lt of vagal stim ulation second ary to head com p ression and is not ind icative of fetal asp hyxia. Late d ecelerations begin after the UC and retu rn to baseline after the end of the UC. They are often rep etitive an d associated w ith d ecreased fetal heart rate variability. They are associated w ith u terop lacental insu fficiency. Variable d ecelerations are variable in configu ration and bear no consistent tem p oral relationship to the onset of the UC. They are thou ght second ary to cord com p ression and u nless severe and repetitive are not thought to be ind icative of fetal com p rom ise. (2:145-6) 62. (C) See the exp lanation for Qu estion 61. 63. (B) Breakage and m igration of the lead s and bleed ing at the site have all been rep orted . There is no increased risk of d evelop ing a spinal cord tum or. (1:1457; 5:2774) 64. (D ) Reperfu sion of an extrem ity after tou rniqu et d eflation typ ically is associated w ith a d ecrease in core tem p eratu re of u p to 1.0°C. Ad d ition ally, both p reload an d afterload d ecrease as blood reenters the affected extremity, often p rod u cing hyp otension. Du ring lim b ischemia, carbon d ioxid e and lactic acid levels increase as ischemic tissues convert to anaerobic

Answe rs : 54–71

m etabolism , w ith an increm ental d ecrease in p H of the ischem ic lim b. After tou rniqu et d eflation, aerobic m etabolism resu m es, w ith increases in oxygen consu m p tion and carbon d ioxid e p rod u ction. The system ic p artial p ressu re of carbon d ioxid e increases, and pH transiently d ecreases as a resu lt of com bined m etabolic and respiratory acid osis. Du ring a general anesthetic, tou rniqu et p ain m anifests as increases in heart rate and blood pressures 45-60 min after tourniquet inflation. Increasing the depth of anesthesia or ad ministering add itional analgesics provides little relief. Tourniquet d eflation is the only factor that elim inates tou rniqu et p ain, thu s d ecreasing anesthetic requirem ents. (5:1202-3) 65. (B) The patient w ith Parkinson d isease has a d eficiency in central d op am inergic activity. Thus, phenothiazines and neuroleptanesthesia (w hich employ a d opaminergic antagonist) are relatively contraind icated . There is no contraind ication to muscle relaxants and the response to pressors is not abnormal. Since levod opa has a short d uration of action, it should be continued up until the time of surgery. (5:149) 66. (E) Treatm ent of cerebral ed em a inclu d es osm otic d iu retics, loop d iu retics, glu cocorticoid s, and su rgical d ecom pression. CSF d rainage w ill red u ce ICP and improve perfusion but has no effect on the ed em a. (5:880-1; 6:2264-5) 67. (D ) Methylergonovine is not ap p roved for intravenou s injection and is u su ally given intramuscularly or intramyometrially. It should not be u sed intravenou sly excep t in severe cases of life-threatening hemorrhage as it m ay cau se severe hyp ertension. If given intravenou sly as a lifesaving m easu re, it shou ld be ad ministered slow ly over at least 60 sec w ith close monitoring of blood pressure. All other agents are u sefu l to treat uterine atony. Rapid infusion of oxytocin can cause hypotension and p rostagland in F2α can cau se bronchoconstriction and hypertension. (2:367-8; 5:1157) 68. (C) A variable intrathoracic obstru ction lim its exp iratory flow w hile p reserving total lu ng

419

volume and inspiratory flow. Forced expiration in variable intrathoracic obstru ction resu lts in a very p ositive p leu ral p ressu re that is greater than the slightly p ositive intratracheal p ressure, resulting in an increase of the obstru ction and narrow ing of the airw ay. Forced inspiration in this situ ation w ill d ecrease the obstru ction becau se of airw ay d ilatation. (5:997) 69. (A) At the age of 6 m onths, a norm al infant m ight be exp ected to have: a heart rate of 120 ± 20, blood p ressu re 90/ 60 ± 30/ 10, O 2 consu m ption of 5 ± 0.9 m L/ kg. (5:250; 357-8) 70. (B) Cervicogenic head ache is d efined as head ache that arises from painful d isord ers of structu res in the upper neck that generates irritation of the u p p er cervical roots or their nerve branches. The cu rrent classification by the IH S and the IASP accep ts these head aches to be u nilateral or bilateral. Pain relief m ay be obtained by blocking the greater occip ital nerve. The other op tions in the qu estion are tru e. (7:263-4) 71. (A) Patients w ith acute hypocalcemia can present w ith p aresthesias, m u scle cram p s, strid or, and p ositive Chvostek and Trou sseau signs. Tracheom alacia, althou gh rarely seen, m ay resu lt from long-stand ing com p ression of the trachea by a large goiter that, w hen rem oved , m ay obstruct the trachea u pon extu bation. A m ajor com p lication of thyroid su rgery that u su ally appears early (im m ed iately or w ithin hou rs) in the p ostoperative period is airw ay obstru ction attribu table to recu rrent laryngeal nerve (RLN ) inju ry w ith resu ltant narrow ing of the glottic opening. A u nilateral RLN palsy w ou ld n ot p rod u ce sign ifican t resp iratory com prom ise as long as the contralateral nerve an d vocal ap p aratu s fu n ction n orm ally. H ow ever, bilateral nerve p alsy, as from a new u nilateral RLN inju ry in the setting of a p reexisting d eficit on the other sid e, can p rod u ce com plete closu re of the glottis and respiratory obstru ction. Rap id onset of life threatening airw ay obstru ction d u e to hem atom a is a know n com p lication of thyroid (and p arathyroid ) su rgery. (5:520, 1246)

420

20: P ra ctice Te s t

72. (C) α-adrenoceptors are stimulated by norepinep hrine cau sing vasoconstriction. Isop roterenol is a nonspecific β-adrenoceptor agonist. Metop rolol is a β 1-selective an tagon ist. Bronchioles are not affected by α-ad renoceptor receptor agonists. Adipocytes carry β 3 receptors. (1:203-4, 278, 311) 73. (E) After su barach n oid h em orrh age ECG changes consistent w ith ischem ia are com mon. A subpopulation of these patients d emonstrate elevated card iac en zym es in clu d in g CK, CK-MB, and troponin and red u ced ventricu lar con tractility w ith low ejection fraction . Pu lm onary hyp ertension is not p art of the synd rom e. (5:885-7; 6:2261) 74. (D) An oxytocin contraction test (OCT) involves the stim u lation of u terine contractions w ith either oxytocin or breast stim ulation. The presence of repetitive late d ecelerations represents a p ositive OCT, ind icative of u terop lacental insufficiency and fetal com promise. Long-term variability implies an intact fetal sympathetic/ parasympathetic nervous system . A biophysical p rofile (BPP) is an ultrasou nd that incorporates fetal movem ent, tone, breathing m otion and amniotic flu id com bined w ith a non-stress test to assess fetal wellbeing. A BPP of 8 or more is reassu ring. A non-stress test involves the observation of tw o fetal heart rate accelerations w ithin a 20-30 min period to assess fetal wellbeing. A reactive non-stress test is reassu ring. Fetal scalp pH testing involves the assessment of fetal blood p H d u ring labor. A fetal p H greater than or equal to 7.25 is reassuring. (2:94102; 5:292-3) 75. (C) At birth, p u lm onary vascu lar resistance d eclines rapid ly in response to lung expansion and exposu re of p u lm onary resistance vessels to alveolar oxygen. At the sam e tim e, system ic vascular resistance increases. Pulmonary blood flow and venou s retu rn to the left atriu m increase, and closu re of the foram en ovale occu rs w hen m ean left atrial pressu re exceed s m ean right atrial p ressu re. Functional closure of the d u ctus arteriosu s occurs in response to a rise in arterial oxygen satu ration in the first 24 h after birth. Anatom ic closu re of both the

d uctus arteriosus and the foramen ovale occurs m uch later. (5:250-1) 76. (B) It is estim ated that the risk of CRPS after fractu res is 1% to 2% and the risk increases to 12% after brain lesions. Retrosp ective stu d ies in large cohorts show s a d istribu tion in the u p p er and low er extrem ity from 1:1 to 2:1. CRPS follow ing SCI are rare. Extrem ities affected by a brain inju ry are at higher risk of d evelop ing CRPS than u naffected extrem ities. (5:1542-3; 6:3358-9) 77. (D ) An ap p rop riate laser end otracheal tu be, su ch as m etal, or a m etal-tap ed PVC tu be, shou ld be u sed . Insp ired oxygen shou ld be red u ced as tolerated by the p atient to less than 30% bu t id eally 21%. Becau se N 2O su p p orts com bu stion, either air or heliu m shou ld be u sed to d ilu te the oxygen. The ETT cu ff should be filled w ith a saline–d ye m ixture or lid ocaine jelly. H 2O-based ointm ents shou ld be used as lu bricants, as p etroleu m -based ointm ents are flam m able. Du ration and intensity of laser exp osu re shou ld be lim ited ; continu ou s m od e allow s heat bu ild u p . The fresh gas flow has no effect on p revention of airw ay fire. (5:1238-9) 78. (E) Preeclam p sia can involve all organ system s w ith associated d ecreased p latelets, abnorm al clotting stu d ies, and abnorm al liver function tests. A com bination of d ecreased plasm a colloid osm otic p ressu re and increased vascu lar permeability results in d ecreased intravascu lar volum e and extravascu lar w ater and sod ium retention. Patients are u su ally hyp erreflexic and hypertensive bu t intravascularly d epleted w ith increased sensitivity to vasop ressors. There is no contraind ication to regional anesthesia if no coagulopathy exists and the patient is p rop erly m onitored . Pu lm onary ed em a is rare, occu rring in app roxim ately 3% of w om en w ith p reeclam p sia. (2:982-7) 79. (D ) The interscalene block w ill not reliably block the C8-T1 d istribu tion of the brachial p lexu s. This w ill resu lt in inad equate blockad e of the ulnar nerve, m ed ial brachial cu taneou s nerve of the arm , and m ed ial antebrachial cutaneou s nerve of the forearm . (5:831)

Answe rs : 72–87

80. (A) End -tid al nitrogen is the m ost sensitive m onitor follow ed by end -tid al carbon d ioxid e, then blood pressu re, then esophageal stethoscop e, then ECG. (3:519-20) 81. (A) The m etabolism of brow n fat is a heatp rod ucing m echanism. All of the other options are comm on m ethod s of losing heat in an operating room . The tem peratu re m u st be m onitored and m ethod s initiated to p revent heat loss. A warming blanket placed under the infant minimizes heat loss by cond uction. (5:251-2) 82. (A) Acu p u nctu re, the insertion of need les into sp ecific points in the bod y, has the ad vantage of being a relatively benign m od e of therapy for p ain. It has been show n to have effects on the brain and end ocrine system . (5:612; 7:2366)

421

d uring open repair or replacement of the thoracic aorta. Patients undergoing successful stent graft repair of the thoracic aorta are routinely extubated at the end of the proced ure, and a high-d ose opioid techniqu e d oes not provid e spinal cord protection. (5:915, 940, 1019). 85. (A) β 2-ad renoceptors are stim ulated by isoproterenol that resu lts in p eripheral vasod ilation and increased skeletal m u scle glycogenolysis. Esm olol is a selective β 1-ad renocep tor antagonist. Clonidine is an agonist at α 2-ad renoceptors that are the principal prejunctional receptors th at in h ibit sym p ath etic n eu rotran sm itter release. (1:179, 222, 278)

83. (B) Whereas hyp otension, hyp erventilation, and hypotherm ia all d ecrease IOP, hyp oventilation , h yp oxia, an d ven ou s obstru ction increase IOP. H yp ertension m ay m arginally increase IOP. Most d ru gs u sed in anesthesia either have m inim al effect on or d ecrease IOP. Inhalational and intravenou s (IV) d ru gs have the m ost rap id and p ronou nced effect. Most sed atives and ind u ction agents (e.g., p rop ofol and thiop ental) red u ce IOP in a d ose-related m anner. Su ccinylcholine has been reported to increase IOP by 6 to 12 m m H g, bu t straining or cou gh in g r aises IO P m u ch m ore. Acetazolam id e (given IV) cau ses carbon ic anhyd rase inhibition and interferes w ith the form ation of aqu eou s hu m or and low ers IOP. β -ad renocep tor antagonist (e.g., tim olol) top ical solu tions are u sed in the treatm ent of glau com a. This class of m ed ication acts to red uce aqu eous hu m or secretion (5:1210-1, 1221)

86. (D ) Mechanically ventilated p atients are su sceptible to ventilator-ind uced lung injury that is believed d u e to overd istention of lu ng u nits, rep eated op ening and closing of atelectatic lu n g, an d likely in flam m atory m ed iators released from inju red lu ng. In p atients w ith ARDS, a m ortality benefit has been show n from a p rotective strategy u sing low er tid al volu m es for ven tilation , in th e ran ge of 6-7 m L/ kg (calcu lated based on p red icted bod y w eight). Maintenance of p lateau p ressu res < 30 cm H 2O are also recom m end ed to red u ce lu ng overd istention. Zero PEEP has been show n to be harm fu l in exp erim ental rat m od els of m echanical ventilation. In hu m ans w ith ARDS, the necessity of PEEP to p revent or red u ce lu ng d erecru itm ent is u nd isp u ted . The best m ethod to d eterm ine the op tim al PEEP level, how ever, d oes not have consensus agreem ent. Desp ite som e strong ind ivid u al p references, no sp ecific ventilator m od e has been d em onstrated su p erior to another based on ou tcom e d ata for ARDS. (5:1417-20)

84. (D ) Strategies to mitigate the risk of spinal cord ischemia in this high-risk patient includ e placement of a lumbar d rain as w ell as neurophysiologic m onitoring su ch as SSEP’s and / or MEP’s. Mild to moderate systemic hypothermia d efined as core bod y tem p eratu re of 32-35°C is an ad d itional established techniqu e to p rotect against neuronal ischemia. H ypotension is to be avoided since it w ould increase the risk of ischem ia. DH CA is a techniqu e em p loyed

87. (B) Criteria for failure of a spontaneous breathing trial have been p u blished by the Am erican College of Chest Physicians. They are resp iratory rate > 35, Sp O 2 < 90%, p u lse > 140 bp m or su stained increase by 20%, SBP > 180 m m H g or DBP > 90 m m H g, and increased anxiety or d iap horesis. If any of these criteria exist, consid eration of extu bation shou ld be d elayed u ntil a further evaluation of the patient and / or criteria occurs. (5:1421)

422

20: P ra ctice Te s t

88. (D ) There is no significant d ifference betw een general and regional anesthesia in neonatal cond ition after nonurgent cesarean section. N o neonatal d ep ression is d em onstrated if d elivery is w ithin 10 m in of ind u ction or if less than 3 m in elap ses betw een u terine incision and d elivery. (2:534-5; 5:1159) 89. (C) Ind u ced hyp otension m ay be necessary for short p eriod s of tim e d u ring a neurosu rgical proced u re. Thu s, a short-acting d ru g, su ch as sod iu m nitropru ssid e or nicard ipine, is id eal. Th e u se of h yp oten sion m ay p rod u ce an increase in d ead sp ace. H yp otension is not contraind icated in the neu rosu rgical p atient. H yp otension never im p roves ischem ia. (5:892) 90. (C) This is an ultrasound im age of an interscalene nerve block. The C8-T1 nerve roots are u su ally sp ared . Therefore th e u ln ar nerve (C8-T1) is usu ally sp ared . (5:831) 91. (C) The su p raclavicu lar nerves via the su p erficial cervical plexu s innervate the cape of the shou ld er. When p erform ing an interscalene block, these nerves are u su ally anesthetized via in d irect sp read of local an esth etic. H ow ever, d ep en d in g on th e tech n iqu es em p loyed and the volu m e of local anesthetic ad m inistered , these nerves m ay be occasionally sp ared . (5:828) 92. (D ) The child w ith an u p p er resp iratory tract infection (URI) is a d ilemma. It has consistently been observed that the likelihood of laryngosp asm , bronchosp asm , and d esatu ration is increased w hen a patient has a mild URI, especially if an end otracheal tu be is u sed . In the case of an acu te URI the risk of the above m ay be greater. Most institu tions w ill p ostp one elective su rgery if the p atient has signs and sym ptom s of an acute URI, especially if nasal d ischarge is p u ru lent, a p rod u ctive cou gh is present, the p atient is febrile, or au scu ltation of the lungs reveals rales, rhonchi, or w heezing. Th e d u ration of p ostp on in g is an oth er d ilem m a, how ever, 2 to 4 w eeks is the u su al tim e to w ait for resched uling surgery. (3:226-9)

93. (D ) Selective COX-2 inhibitors d o not have any ad vantages in term s of renal effects. COX-2 inhibitors are associated w ith less GI toxicity than nonselective COX inhibitors bu t they are m ore exp ensive. There is a p ossible increased risk of m yocard ial infarction (MI) and throm botic stroke events associated w ith the con tinu ou s long-term u se of selective COX-2 inhibitors. Those concerns led to rofecoxib and vald ecoxib being w ithd raw n from the m arket in th e years 2004 an d 2005, resp ectively. N onselective COX inhibitors inhibit the synthesis of TXA 2 by inhibiting COX-1 that is sp ared w ith the u se of COX-2 inhibitors. (5:1303) 94. (B) Althou gh m an y p eop le self-ad m inister ethanol as a bed tim e sed ative, and sed ation is a prominent effect shortly after the ingestion of m od erate am ounts of ethanol, persons d rinking ethanol ju st before bed tim e often aw aken a few hou rs later and have d ifficu lty falling asleep again. (1:634) 95. (D ) The blood supply of the posterior d escend ing artery d eterm ines the p attern of coronary d om inance: right coronary artery for right d om inance and left circu m flex artery for left d om in an ce. Most p atien ts h ave a righ td ominant, or balanced , pattern of blood supply to the posterior d escend ing artery. (5:902) 96. (B) Cau salgia is the term that is som etim es u sed to ind icate ”sym pathetically m aintained p ain” in association w ith a m ajor nerve injury. ”Com plex regional pain synd rom e” is now the accep ted term . (1:1444) 97. (B) Desp ite increased blood flow accom p anying elevated metabolic activity d u ring generalized seizu res, cerebral acid osis d evelop s. The m echanism u nd erlying this p henom enon is u nclear. Most general anesthetics suppress seizu re activity, inclu d ing barbitu rates, benzod iazep ines, and volatile anesthetics. Prolonged anesthesia w ith isoflu rane has been u sed to treat refractory seizures. (5:875-8; 6:3267-8)

Answe rs : 88–104

98. (D ) The infantile airw ay has a m ore cephalad p laced larynx, vocal cord s that are slanted , and a large tongu e in a relatively sm all jaw. The ep iglottis is lon g and n arrow and om ega shap ed . The narrow est p ortion of the infant airw ay is at the level of the cricoid cartilage. (5:254) 99. (D ) The term lu p u s anticoagu lant is a m isnom er as lu pu s anticoagulant has no tru e anticoagu lant activity in vivo. Therefore, neu raxial anesthesia is not contraind icated . The syn d rom e is characterized by recurrent pregnancy loss and/ or recurrent venous or arterial thrombosis as w ell as laboratory evid ence of either anticard iolip in antibod y or lu p u s anticoagu lant. Pregnant p atients m ay be treated w ith low -d ose asp irin therap y and hep arin in ord er to im prove fetal su rvival and d ecrease m aternal throm botic risk. Venous (e.g., d eep venou s throm bosis) and arterial (e.g., cerebral and myocard ial infarction) thrombotic events occur in patients w ith antiphospholipid synd rom e. A p atient receiving a once d aily p rop hylactic d ose of LMWH shou ld w ait at least 12 h p rior to placem ent of neu raxial anesthesia. (2:872-4; 5:216) 100. (D ) The oculocard iac reflex may be initiated by m anip u lation of any of the stru ctu res of the afferent p athw ay, inclu d ing the globe, orbital contents, ophthalm ic d ivision of cranial nerve V, and trigeminal ganglion and nerve. Tentorial m anip u lation can p rod u ce brad ycard ia and asystole. Sp inal cord rhizotom y m ay p rod u ce brad yarrhythmias. Manipulation of the carotid sinus activates afferents of the baroreflex and can ind u ce profound alterations in heart rate. Sim ple occlu sion of an arteriovenous m alform ation is n ot associated w ith card iac sym ptom s. (5:887-93; 1206) 101. (C) Preeclam p sia occu rs after the 20th w eek of gestation and requ ires at least tw o of the follow ing: systolic blood p ressu re greater than 140 m m H g or 30 m m H g above p repregnancy valu es, d iastolic blood p ressu re greater than 90 m m H g or 15 m m H g above p rep regnancy

423

valu es, generalized ed em a, or proteinu ria. It can involve all organ system s. Its etiology rem ains u nknow n. (2:975-98; 5:294-6) 102. (C) Alcohol (50% to 100%) and p henol (5% to 20%) are both neu rolytic agents. Glycerin is often ad d ed to p henol to m ake its sp ecific gravity greater than that of cerebrosp inal flu id w hen u sed for a hyp erbaric sp inal techniqu e. (1:1453; 5:2775) 103. (B) Valp roic acid com m only cau ses elevation of hepatic enzym es in plasm a. (1:596-7) 104. (D ) Sevoflu rane is rarely u sed for renal transp lantation d u e to concerns of flu orid e and com pound A toxicity. Althou gh m ost hu m an stu d ies h ave not d em onstrated d eleteriou s effects of sevoflu rane on the kid ney, m any au thors feel, given the u ncertainty su rrou nd ing the agent, as w ell as the safe alternatives available, that it shou ld be avoid ed in renal transp lant p atients. It shou ld be noted that sevoflu rane has been d em onstrated to have antiinflam m atory effects on renal tissu e that may be protective against ischemia reperfusion inju ry. Etom id ate is w ell tolerated in hem od ynam ically com prom ised patients, particu larly im p ortant in d iabetic p atients w ith au tonom ic neu rop athy. N itrou s oxid e has m inim al sid e effects of concern in these p atients, w ith no ren al toxicity an d r ap id elim in ation . Succinylcholine can be used to facilitate a rapid sequence ind uction in patients w ith full stom ach , gastrop aresis, or acid reflu x d isease. Becau se th e seru m [K+ ] can in crease 0.5 mEq/ L w ith its ad m inistration, it shou ld be u sed w ith cau tion in patients w ith renal failure w ho have an elevated p reop erative p otassiu m concentration (>5.5 m Eq/ L). Maintenance of skeletal m u scle relaxation can be p rovid ed w ith nond epolarizing m u scle relaxants that d o not d epend on the kid ney for elimination, such as cisatracu riu m , rocu roniu m , or vecu roniu m . Althou gh m etabolized in the liver, accum ulation of m etabolites excreted by kid ney m ay p rolong their d u ration if large d oses are used . (5:1097-8)

424

20: P ra ctice Te s t

105. (C) All op ioid s m u st be u sed cau tiou sly in renal transp lant recip ients, p articu larly if the graft is not fu nctioning p rop erly. A m etabolite of m orp hine, m orp hine-6-glu cu ronid e, has opioid agonist activity and is excreted by the kid neys. It can accu m u late in renal failu re and cau se resp iratory d ep ression w ith long-term u se. The m etabolism of hyd rom orphone prod u ces a neu roexcitatory com p ou nd that can accum ulate in renal failu re. H ow ever, hyd rom orphone has been used extensively in renal failu re p atients w ith no ad verse effects. In contrast, a m etabolite of m eperid ine, norm eperid ine, can accum u late in significant am ounts in patients w ith renal failu re, and this com pou nd can cau se seizures. The pharm acokinetics and p harm acod yn am ics of fentanyl, su fentan il, alfentanil, and rem ifentanil are not significantly altered by kid ney d isease. (5:1098) 106. (B) Immed iate graft function lead s to improved graft and p atient su rvival. Volu m e exp ansion, m aintenance of peripheral vascu lar resistance w ith vasopressors, and d iuresis, are all helpful. Mannitol, w hen combined w ith volume expansion, has been show n to d ecrease the incid ence of acu te tu bu lar necrosis in the transp lanted kid ney. The m echanism by w hich it d oes this m ay be related to d ecreasing tu bu lar sw elling by its osm otic effect, its action as a free-rad ical scavenger, or by flu shing aw ay slou ghed renal tubule cells before they can cause injury by second ary obstru ction. Although furosem id e can be ad m inistered to enhance d iu resis, it has not been show n to red u ce the incid ence of acu te necrosis in the transp lanted kid ney. (5:1099) 107. (D ) Only 20% of patients u nd ergo fusion to the low est lu m bar levels. Com p lications seem to occu r m ore in this patient population than in p atients w here the fu sion end s in the u p p er lu m bar sp ine. Pregnant w om en w ho have had su rgically corrected scoliosis and are w ithou t significant respiratory or card iac involvem ent tend to d o w ell. Obliteration or scarring of the epid u ral space from trau ma to the ligam entum flavu m d u ring corrective su rgery can alter the spread of local anesthetic. This can lead to a higher incid ence of failed or inad equate block and u nintentional d u ral pu nctu re. (2:1038-43)

108. (C) As oxygen is absorbed from the nonventilated lu ng, it becom es atelectatic. Persistent blood flow throu gh that lu ng is then not oxygenated and contribu tes to shu nt. Method s of im proving oxygenation includ e d ecreasing the shu nt or increasing m ixed venous oxygen concentration. The m ost d irect m ethod of d ecreasing shu nt is to m echanically stop blood flow to the atelectatic lung. Increasing oxygen flow to the circuit w ill not increase the FIO 2 so it w ill be ineffective. N itroglycerin, or increased isoflu rane, w ill im pair hypoxic p u lm onary vasoconstriction. PEEP m ay im prove V/ Q matching in the ventilated lu ng, but m ay also shift blood flow to the nonventilated lu ng. (5:971-3) 109. (C) Of all the d ru gs listed , m ethyld op a is the d ru g m ost stu d ied in p regnancy and the only Category B (see list below ) d rug. Labetalol and nifed ip ine are Category C d rugs that are often u sed in p regnancy bu t have not been stud ied as extensively as methyld opa. Diu retics are not com m only used in pregnancy and captopril is a category D d ru g. (1:773-4, 1846-7) Category A: Controlled stu d ies show no risk. Ad equ ate, w ell-controlled stu d ies in p regnant w om en have failed to d em onstrate a risk to the fetus in any trim ester of p regnancy. Category B: N o evid ence of risk in hu m ans. Ad equ ate, w ell-controlled stu d ies in p regnant w om en have not show n an increased risk of fetal abnorm alities d esp ite ad verse nd ings in anim als, or, in the absence of ad equ ate hu m an stu d ies, anim al stu d ies show no fetal risk. The chance of fetal harm is rem ote, bu t rem ains a p ossibility. Category C: Risk cannot be ru led ou t. Ad equ ate, w ell-controlled hu m an stu d ies are lacking, and anim al stu d ies have show n a risk to the fetu s or are lacking as w ell. There is a chance of fetal harm if the d ru g is ad m inistered d u ring p regnancy, bu t the p otential bene ts m ay ou tw eigh the potential risk. Category D: Positive evid ence of risk. Stu d ies in hu m ans, or investigational or p ost-m arketing d ata, have d em onstrated fetal risk. N evertheless, p otential bene ts from the

Answe rs : 105–116

u se of the d rug m ay outw eigh the potential risk. For exam p le, the d ru g m ay be accep table if need ed in a life-threatening situ ation or serious d isease for w hich safer d ru gs cannot be u sed or are ineffective. Category X: Contraind icated in p regnancy. Stu d ies in anim als or hum ans, or investigational or p ost-m arketing rep orts, have d em onstrated p ositive evid ence of fetal abnorm alities or risk that clearly ou tw eighs any possible bene t to the p atient. 110. (D ) A halo d evice can lim it neck m otion necessary for d irect laryngoscopy significantly. This necessitates alternative m eth od s of airw ay m anagem ent, includ ing aw ake fiberop tic intu bation by either a nasal or oral rou te. If the cervical sp ine is u nstable, rem oval of the halo d evice is not recom m end ed . The trau m atized p atient w ith a neck inju ry m ay have other associated inju ries, inclu d ing facial fractu res and closed head inju ry. Anesthetizing the trachea is necessary before p lacem ent of the end otracheal tu be to p revent cou ghing and straining. (5:1361-2; 6:139) 111. (C) This p atient has p ost-herp etic neu ralgia. Tricyclic antid ep ressants are often the m ost effective d ru gs in treating this d ifficu lt cond ition. The anticonvu lsant gabapentin has also been show n to be u sefu l. The u se of TEN S or opioid s is helpfu l in the occasional patient, bu t u sually the results are d isapp ointing. (1:1448) 112. (C) For many thoracic proced u res in ped iatric patients such as this one, lung isolation is not necessary. If it is necessary, either a Univent tube, bronchial blocker, or mainstem intubation m ay be u sed to p rovid e lu ng isolation. Double lumen tubes (DLT) are not available in a size small enough for this patient. The smallest DLT available in the United States is size 26-French that m ay be u sed in child ren as you ng as 8 years of age. With the Univent tu be, the blocker is less likely to becom e d islod ged because it is attached to the m ain end otracheal tube. An arterial catheter is not likely necessary in this p atient as the p atient is otherw ise healthy, is not in d istress, and is hem od ynam ically stable. If a bronchial blocker is u sed , it

425

may be passed inside of the tube with fiberoptic assistance, or, if the tube is too small, fluoroscopy m ay be u sed to guid e placem ent, or the catheter m ay be p laced w ith fiberop tic assistance outsid e of the end otracheal tube. Using a larger tube than is appropriate for the patient is not recom m end ed as this cou ld resu lt in airw ay sw elling and inflam mation. (3:281-6) 113. (A) N eural pathw ays responsible for the transm ission of p ain d uring the first stage of labor are visceral in nature and involve afferent pathw ays from T10 – L1. The pathw ays involved for the second stage of labor are som atic and prod uced by the d istension of the perineum and stretching of the fascia, skin, and su bcu taneous tissu es and involve afferent p athw ays from S2 – S4 via the pud end al nerves. (2:223-4) 114. (B) This is a com m on late presentation of an atrial sep tal d efect. The other d efects d o not cau se right atrial d ilation that lead s to atrial fibrillation. (3:479) 115. (A) The most likely d iagnosis is acute myocardial ischemia usually manifested by ST-segment changes, a rise in PA pressures, and new w all m otion abnormalities on TEE. Promoting factors for m yocard ial ischemia includ e tachycard ia, hypotension, a rise in left ventricular end d iastolic pressure (that results in a rise in pulmonary capillary w ed ge and pulmonary artery p ressu res), severe anem ia, and hyp oxem ia. While options B, C, and E m ight also lead to elevated PA pressures, they are less likely based on the information provid ed. Option D usually resu lts in d ecreased PA pressures. (5:908-9) 116. (C) Prompt treatment of hypotension is mandatory to restore coronary perfusion pressure and alleviate myocard ial ischemia. Ad ministration of the α-adrenoceptor agonist phenylephrine is the best option given; norepinephrine would be an alternative. While epinephrine can treat hypotension, it is likely to cause tachycardia and thus exacerbation of ischemia. Glycopyrrolate would cause tachycardia without associated rise in blood pressure. Arterial vasodilators are not ind icated in this situation. (1:295; 5:902-7)

426

20: P ra ctice Te s t

117. (C) If the p atient is otherw ise stable, the best treatm ent is to inform the su rgeon and requ est that he/ she stop the stim ulu s that precipitated the ocu locard iac reflex. After a brief pau se in w hich the heart rate is allow ed to recover, the su rgeon m ay resum e su rgery. The reflex d oes fatigu e early and u su ally is not p ersistent. Preoperative atropine or a regional block of the orbit are m ethod s of preventing the OCR, but are not treatm ents for it. There is som e concern that the p reop erative ad m inistration of atrop ine m ay lead to an increased incid ence of intraop erative arrhythm ias. Intravenou s atrop ine is m ore effective than intram u scu lar or oral atrop ine in p reventing this reflex. It is im p ortant to inform the su rgeon as soon as brad ycard ia is seen. (3:696; 5:1206, 1220) 118. (C) The afferent lim b of the ocu locard iac reflex is the trigem inal nerve. The efferent lim b is the vagus nerve. (5:1220) 119. (C) Methimazole d oes cross the placenta. This can result in fetal hypothyroid ism and goiter formation. Fetal goiter can be diagnosed in utero w ith ultrasound . H yperthyroid patients have d ecreased glucocorticoid reserves and should receive supplementation. Radioactive iodine is contraindicated during pregnancy. Undiagnosed or undertreated pregnant patients with Graves disease are at increased risk for thyroid storm and precipitating factors include infection, thyroid cancer, normal labor, hemorrhage, cesarean delivery, and eclampsia. Physicians should be prepared to treat thyroid storm should it occur during pregnancy. (2:923-8) 120. (E) Both d rugs are short-acting and d o not produce prolonged sedation and both drugs have little effect on REM sleep. Triazolam, like other benzodiazepines, produces anterograde amnesia. (1:463, 465, 467) 121. (D ) H yp oxem ia and intracranial hyp ertension are com m on p roblem s after closed h ead trau m a. Secu ring the airw ay and initiating m echanical ventilation m ay be requ ired to m ain tain ad equ ate oxygenation . Sed ation, w hich d ecreases cerebral m etabolic rate, is u sefu l to red u ce ICP bu t d oes obscu re the

neu rological exam . Rou tine hyp erventilation has been fou nd not to be of benefit to ou tcom e and is no longer recom m end ed . The recom m end ed low er lim it for CPP is 60 m m H g. (5:1380, 1466-72; 6:2256-8) 122. (E) Meconium aspiration usually occurs in fullterm babies and is rare in those w eighing less than 2 kg at birth. Regu lar chest p hysical therap y and p ostu ral d rainage are recom m end ed to clear resid u al m econiu m from the lu ngs. Long-term ou tcom e is good , in term s of intellectu al d evelop m ent and p u lm onary fu nction, u n less asp h yxia occu rred in th e p erinatal p eriod . Passage of m econiu m m ay occu r in the p resence or absence of fetal d istress. (5:247-8) 123. (B) α 1-antitryp sin d eficiency lead s to airw ay d isease that is fam ilial and is d eterm ined by seru m assay. The assay m easures the level of a p rotective enzym e p rod uced by the liver that acts to p revent au tod igestion of lu ng tissu e by the p roteolytic enzym es of p hagocytic cells. Only 1-2% of COPD patients are found to have severe α 1-antitryp sin d eficiency as a contribu ting cau se to their d isease. (5:188, 1090; 6:2152) 124. (D) The morbidly obese patient is at a higher risk of having a macrosomic infant and a lower risk of preterm delivery. A higher initial failure rate of epidural placement has been reported and the need for placement of a second or third catheter is more common. Morbidly obese parturients are at higher risk for undiagnosed obstructive sleep apnea (OSA). Although the ASA guidelines for the perioperative management of patients with OSA was not intended specifically for pregnant patients, they do provide some guidance for the management of the obese parturient undergoing cesarean delivery. (2:1081-90) 125. (A) Carboxyhem oglobin, w hich is p resent in cases of carbon monoxide poisoning, may cause a normal pulse oximetry read ing. The history of this patient w ho w as trapped in a burning structure w ithout burn injury and w ho is now comatose suggests carbon monoxid e poisoning with neurologic impairment. Because the absorbance spectrum of carboxyhemoglobin is similar to that of oxyhem oglobin, m ost p u lse

Answe rs : 117–133

oximeters w ill not d iscriminate carboxyhemoglobin from oxyhemoglobin and oximetry readings may be normal. An arterial blood gas w ill reflect a normal Pa O 2 since the Pa O 2 d oes not reflect the amount of oxygen bound to hemoglobin. (6:1336) 126. (D ) Seizu res are a know n com p lication of hyp erbaric oxygen therapy. Treatm ent is rapid d ecompression and removal from the chamber. (Weaver L. Crit Care M ed 2011; 39:1784-91) 127. (B) An Ap gar score of 7 to 10 ind icates a healthy infant, w ho cries after d elivery, m aintains tone and color, has a heart rate above 100 beats/ m in and requ ires only rou tine care. An infant w ith an Ap gar of 4 to 6 is d ep ressed , m ay not breathe im m ed iately and shou ld be stim u lated and have the airw ay cleared w ith a bu lb and syringe. If the heart rate is less than 100 beats/ m in the infant shou ld be ventilated w ith a bag and m ask at a rate of 60/ min. Long inflations rather than short, fast ones are op tim al. An infant w ith an Ap gar of 0 to 3 is flaccid , apneic, pale, and unresp onsive. The ones that d on’t respond to bag and mask ventilation shou ld be intu bated , and if the heart rate rem ains less than 100/ m in, chest m assage is initiated . (3:834-5) 128. (A) Celecoxib is selective for COX-2, w hile ketorolac and ind om ethacin are m ore selective for COX-1. Ibuprofen and acetaminophen have little selectivity. (1:962) 129. (B) Dou ble-lu men tu bes have one lu men in the trachea and one lu m en in a m ainstem bronchus. The hand ed ness of the tube reflects the bronchu s intu bated . (5:964) 130. (A) The most common sid e effects of chronic COX inhibitor therapy are gastric in nature such as anorexia, nausea, dyspepsia, abdominal pain, and d iarrhea. In ad d ition, 15-30% of chronic users w ill d evelop gastric ulcers. All other cond itions listed are side effects that are observed less frequently. Chronic use of COX inhibitors in the elderly must be accompanied by vigilance in m onitoring for the variou s sid e effects. This vigilance inclu d es d eterm ining (w hen

427

appropriate) liver fu nction tests, hem atocrit, renal function, and occult blood in stool. Longterm use should probably also includ e use of m isoprostol that can red uce the incid ence of COX inhibitor-ind uced ulcers; em pirical d ata suggest that other d rugs (histamine H 2 antagonists, su cralfate, antacid s, and p roton p u m p inhibitors) may have similar effects. (1:973-4) 131. (D ) Follow ing a burn injury of this magnitud e, vast amounts of fluid are lost from the circulation into the burned tissue, and thereafter are sequ estered ou tsid e the circu lation even in nonbu rned tissu es. Album in loss is usu ally at least tw ice the total bod y p lasm a content. Card iac output is strikingly d ecreased immed iately after injury because of the rapid red uction in circulating blood volume or the severe com p ressive effects of circu mferential bu rns on the abd om en and chest impairing venou s retu rn. This is d esp ite a large increase in circu lating catecholam ines. Evap orative flu id losses are abou t 4 liters per square meter per d ay. Pu lse oxim etry is not useful in monitoring oxygenation in carbon m onoxid e p oisoning becau se carboxyhem oglobin prod uces an overestim ation of oxygen satu ration; the photod etector d oes not d ifferentiate betw een oxyhemoglobin and carboxyhemoglobin. In contrast, transcutaneou s oxygen analyzers are u sefu l. As this p atient has exp erienced significant inhalation in ju ry, treatm en t in volves in tu bation and mechanical ventilation w ith aggressive pulmonary toilet. (5:1335-7) 132. (A) The efficacy of the injected d ep ot form u lation of naltrexone in m aintaining abstinence from ethanol is su rp risingly m od est. All of the other m od alities have higher efficacy. (1:642-4; 654-6; Pettinati HM , et al., A lcohol Clin Exp Res 2011; 35:1804-11) 133. (D ) Sp inal cord stim u lation involves the p lacem ent of stim u lating electrod es in the epid u ral sp ace. Typically the generators are im planted . With p rop er p atient selection, it has been show n to be usefu l in several chronic pain cond itions, inclu d ing ischem ic p ain and ”failed back synd rom e” w ith a rad icu lar comp onent. (1:1457; 5:2774)

428

20: P ra ctice Te s t

134. (C) Benzod iazep ines bind to the GABA recep tor, a chlorid e channel, and increase its affinity for GABA thereby increasing chlorid e cond u ctance. (1:461) 135. (B) The expected effects of nerve root imp ingem ent are: L2 nerve: w eakness of hip exion (iliop soas) and sensory loss on anterior groin and thigh. N o d eep tend on re ex L4 nerve: w eakness of leg extension (qu ad riceps), ankle d orsi exion (tibialis anterior); sensory loss m ed ial calf/ foot; loss of patellar re ex L5 nerve: w eakness of d orsi exion of big toe (EH L) sensory loss lateral asp ect of calf and d orsu m of foot. N o d eep tend on re ex S3 nerve: w eakness of the anal and / or u rinary sphincters and p ain or sensory loss in the d istribu tion of the p ud end al nerve (7:275) 136. (D ) N eonates have 60% to 90% hem oglobin F, and it is not u ntil they are abou t 6 m onths of age that the ad ult hemoglobin A to hemoglobin F ratio is achieved . H em oglobin F has a high affinity for oxygen, and the oxygen d issociation cu rve is shifted to the left, resu lting in d ecreased oxygen d elivery to the tissu es at a given oxygen tension, and a low er P 50 valu e com p ared to ad u lt h em oglobin . 2,3-d ip hosp hoglycerate interacts w ith hem oglobin A resu lting in d ecreased affinity for oxygen. It d oes not interact w ith hem oglobin F. (5:1175) 137. (C) Infants of u p to 50 w eeks p ost-concep tu al age are at risk of p ostoperative apnea, even if they have no history of previou s apneic episod es. Therefore, they are u sually kept in the hosp ital for the first p ostop erative night for close apnea m onitoring and are not operated on as am bulatory su rgery p atients. H alothane has been show n to d ecrease chem orecep tor sensitivity and is therefore im plicated in postoperative apnea, bu t w ith appropriate m onitoring its u se is not contraind icated . Sp inal anesthesia can be used as the sole technique for this p roced u re, bu t shou ld not be com bined

w ith a sed ative agent becau se of the risk of p ostoperative ap nea. Infants have a relatively rap id u p take of volatile agents becau se of d ecreased blood –gas p artition coefficients, d ecreased MAC, increased card iac ou tp u t p er kg bod y m ass, and relatively high blood flow to the brain. Cau d al ep id u ral injection typ ically as an ad ju nct to general anesthesia is a reasonable techniqu e. (5:1187-9) 138. (C) The m ost likely find ing on TEE based on the p atient’s sym p tom s is severe m itral regu rgitation (MR). MR in p atien ts w ith H OCM occu rs as a consequ ence of systolic anterior m otion (SAM) of the anterior m itral valve leaflet, thu s acu tely narrow ing the left ventricu lar ou tflow tract. This cond ition is exacerbated by states of increased contractility, in this case d u e to the infu sion of ep inep hrine, and hyp ovolem ia, for exam p le d u e to su d d en large volu m e blood loss. N one of the other op tions listed w ou ld exp lain w orsening hem od ynam ics in resp onse to ep inep hrine infu sion. TEE fin d in gs as d escribed in op tion A can be exp ected in acu te p u lm onary em bolu s. (5:449, 910; 6:1934, 1967) 139. (B) States of increased contractility w orsen left ven tricu lar ou tflow tract obstru ction as d escribed in the exp lanation to the p reviou s question. Epinephrine shou ld therefore be d iscontinued immed iately. Milrinone and calcium w ou ld have the sam e effect and likely w orsen this patient’s hem od ynam ics. The goal of therap y is restoration of preload w ith volu m e infusion and afterload su p p ort if ind icated w ith p u re α-ad renocep tor agonists su ch as p henylephrine. N itroglycerin, d u e to the associated red u ction in p reload w ou ld likely w orsen hem od ynam ics. (5:449, 910; 6:1934, 1967) 140. (C) Patients w ith obstru ctive sleep ap nea (OSA) m ay h ave p ron ou n ced p h aryn geal obstru ction d u ring ind u ction of anesthesia requ iring the p lacem ent of an oral or nasop haryngeal airw ay and / or CPAP to m aintain a p atent airw ay. These p atients have greatly increased sensitivity to opioids, should be monitored carefully, and w hen necessary ad m itted for observation w ith apnea monitoring. Due to

Answe rs : 134–150

th e sm all orop haryn x an d ad en otonsillar hyp ertrop hy, p lacem ent of an LMA cou ld be d ifficu lt. (3:666-8) 141. (B) Laceration of the brain com m only perm its spilling of brain tissu e throm boplastin into the system ic circu lation. This can p rod u ce th e rap id ap p earance of d issem inated intravascu lar coagu lation. Coagu lation stu d ies typ ically d em onstrate a p rolonged PT and aPTT w ith red u ced fibrin ogen an d p latelet cou n ts. D -d im ers m ay also be elevated . (5:217; 6:978) 142. (E) Urem ia from kid ney failu re is associated w ith p latelet d ysfu nction that can cau se significant bleed ing d u ring su rgery. Coagu lation stu d ies are typ ically norm al. (5:1330, 1444; 6:460, 2203) 143. (C) Deliriu m trem ens is characterized by d eliriu m , trem ors, and overactivity of the au tonom ic n ervou s system . Sym p tom s begin w ithin 5-10 h of d ecreased alcohol intake and peak on d ay 2-3. (6:3551) 144. (B) Delirium is d efined as a d isturbance of consciou sness or cognition that has d evelop ed over a short period of tim e and is caused by d irect p hysiologic consequ ences of a general med ical cond ition. The hallmark symptom is a deficit of attention that fluctuates over hours or days. The patient may also have altered sleep w ake cycles, hallucinations and affect changes. H eart rate and blood pressure instability m ay occur. Delirium is an und erd iagnosed , though com m on, find ing in hosp italized p atients (14-56%, with higher estimates in ICU patients). Mortality is estimated at 25-33% am ong d elirious inpatients. (5:1390-1; 6:196-201)

429

145. (D ) Dem entia is d efined as an acquired d eterioration in cognitive abilities that im p airs the su ccessfu l p erform ance of activities of d aily living. Mem ory is the m ost com m on, bu t not the only cognitive ability that m ay be lost. Most form s of d em entia are progressive. The Mini-Mental Status Exam ination is a screening tool u sed to confirm that a patient has cognitive im pairm ent. (6:3300-16) 146. (A) Sym p tom s of acu te alcohol intoxication inclu d e d ecreased inhibitions, a d ecrease in com p lex cognitive fu nctions, slu rred sp eech, m otor irritability, an d p oor ju d gm en t. (6: 3546-52) 147. (A) Ep inep hrine is consid ered the d ru g of choice for the treatm ent of fetal brad ycard ia d u ring the resu scitation of the new born. It is recom m end ed if the fetal heart rate rem ains less than 60 beats p er m inu te after 30 sec of ad equ ate ventilation and chest com pressions. A d ose of 0.01-0.03 mg/ kg (or 0.1-0.3 mL/ kg of a 1:10,000 solu tion) is recom m end ed every 3-5 m in. (2:164-70) 148. (C) See explanation to Qu estion 147. Atropine is no longer recom m end ed for u se d u ring the resu scitation of the new born. Ep inep hrine is consid ered the d rug of choice for the treatm ent of brad ycard ia in the new born. (2:169) 149. (M) (Lauer R, M athew JP. Transesophageal tomographic views. In: M athew JP, et al., eds., Clinical M anual and Review of Transesophageal Echocardiography, 2nd ed., New York: M cGraw-Hill, 2010, Figure 5-21) 150. (O) (Lauer R, M athew JP. Transesophageal tomographic views. In: M athew JP, et al., eds., Clinical M anual and Review of Transesophageal Echocardiography, 2nd ed., New York: M cGraw-Hill, 2010, Figure 5-32)

This page intentionally left blank

Re fe re nc e s

1. Bru nton LL, Chabner BA, Knollm ann BC, ed s. Goodman and Gilman' s The Pharmacological Basis of Therapeutics, 12th ed . N ew York: McGraw H ill, 2011. 2. Chestnu t DH , et al., ed s. Chestnut' s Obstetric A nesthesia: Principles and Practice, 4th ed . St. Lou is: C. V. Mosby, 2009. 3. Coté CJ, Lerm an J, Tod res ID, ed s. A Practice of A nesthesia for Infants and Children, 4th ed . Philad elphia: W.B. Sau nd ers Co., 2008.

4. H ad zic A, ed . N Y SORA Textbook of Regional A nesthesia and A cute Pain M anagement, N ew York: McGraw -H ill, 2007. 5. Longnecker DE, et al., ed s. A nesthesiology, 2nd ed . N ew York: McGraw -H ill, 2012. 6. Longo, DL, et al., ed s. Harrison' s Principles of Internal M edicine, 18th ed . N ew York: McGraw H ill, 2012. 7. Warfield CA, Bajw a ZH , ed s. Principles and Practice of Pain M edicine, 2nd ed . N ew York: McGraw -H ill, 2004.

431

This page intentionally left blank

Index

The Rom an nu m erals follow ing the top ic refer to Parts I or II of the book, and the nu m bers refer to sp eci c qu estions in that Part. When a “t” follow s the nu m ber, it d enotes a qu estion in one of the Practice Tests, Chapters 10 or 20 in Parts I or II, respectively. 2-Chloroprocaine, II:229, II:232, II:288 5-H T3 receptors, I:74t A Abd om inal aneurysm , II:112 Abd om inal com partm ent synd rom e, II:434, II:472 Abd om inal d istension, I:175 Absolute hum id ity, I:24 Absorp tion rate, I:583 Abstinence synd rom e, I:498 ACC/ AH A. See Am erican College of Card iology/ Am erican H eart Association Acetaminophen toxicity, II:567 Acetazolam id e, I:51t, I:522 Acetylcholine, I:1t, I:39t, I:238, I:484, II:3, II:9 Achond roplasia, II:299 Acid -base d istu rbance, II:489 Acid osis, I:132t, I:432 Acquired im m u nod e ciency synd rom e (AIDS), I:343 Acrocyanosis, II:253 Acrom egaly, I:102t, I:329 Activated p rotein C, I:100 Acup uncture, II:82t, II:582 Acute alcohol intoxication, II:146t Acute interm ittent porphyria (AIP), I:118t, I:138t, I:468 Acute tubular necrosis, I:334 Acute w ithd raw al synd rom e, II:20 Ad d ison d isease, I:6t Ad d itive effect, I:482 Ad enine, I:121t Ad enoid ectom y, I:286, II:140t Ad enosine, I:79, I:136 ADH . See Antid iu retic horm one Ad ju stable pressu re lim iting valve (APL valve), I:51 Ad renal insu f ciency, I:6t

Ad renergic nervous system , II:72t Ad renergic α 1A , II:57 Ad renergic α 2A , II:58 Ad renergic β 2, II:59 Ad renocorticotropin, I:255 Afterload , II:95 Age, I:45t, I:123, I:445 AIDS. See Acqu ired im m unod e ciency synd rom e AIP. See Acu te interm ittent porp hyria Air em bolism , II:203, II:205, II:210, II:222 Air ow, I:10 Airw ay, II:98t, II:110t Airw ay, d if cult, I:101t–102t, II:129, II:264 Airw ay, em ergency access, I:104t Airw ay, esophageal-tracheal, I:197 Airw ay, infant and ad ult, I:195 Airw ay, u pper, I:104t Airw ay collapse, II:368 Airw ay ed em a, II:458 Airw ay exchange catheter, II:128–129 Airw ay re, II:77t, II:423 Airw ay m aintenance w ith m ask, I:49t Airw ay obstruction, I:59t, I:231 Airw ay pressure, I:164, I:169 Airw ay re exes, I:538 Airw ay resistance, I:130t, I:221 Alad in vaporizer, I:42 Album in, I:304, I:558 Albuterol, II:7, II:21 Alcohol, I:300, II:27, II:132t, II:146t, II:569 Alcoholism , I:293–294, I:304, I:314, I:448, I:459, II:100 Ald osterone, I:363, II:467 Allen test, I:128 Allergic reaction, I:471, I:578, I:582 Allopurinol, I:408

433

434

Inde x

α -Ad renergic agonist, II:31t α -Ad renoceptors, I:66t, II:72t α 1-Antitrypsin d e ciency, II:123t Alpha-stat m ethod , II:140 Alpha w ave, I:251 Alveolar cells, I:192, I:200 Alveolar gas, I:219, I:483 Alveolar gas equ ation, I:1, I:34t Alveolar pressu re, I:127t, I:186–187 Alveolar proteinosis, II:156 Alveolar rupture, I:233 Alveolar tension, I:543 Alveolar ventilation, I:116t, I:260 Ambient pressure, I:127t American College of Card iology/ Am erican H eart Association (ACC/ AH A), I:76 American Society of Anesthesiologists (ASA), I:43, I:52, I:110t Aminocaproic acid , I:89 Aminoglycosid e antibiotics, II:246 Amiod arone, I:78t, I:404, I:454 Amnesia, I:460, I:538 Amniotic u id em bolu s, II:306 Amyotrophic lateral sclerosis, I:284, II:224 Analgesia, I:194, I:501, II:45 Analysis of variance, I:4 Anaphylactic reaction, I:458, II:22, II:522 Anatom ic d ead space, I:180 Anem ia, I:65t, I:364 Anesthesia, cau d al, II:292 Anesthesia, d issociative, I:470 Anesthesia, end otracheal, II:402–403 Anesthesia, general, I:31t, I:134t, I:193, I:227, I:286, I:348, I:509, I:524–558, II:10t, II:38t, II:231, II:355 Anesthesia, inhalational, I:470, I:481 Anesthesia, local, I:378, I:387, I:419 Anesthesia, neu raxial, II:118 Anesthesia, prolonged , I:474 Anesthesia, regional, I:559–600, II:38t, II:64–93, II:77 Anesthesia, spinal, I:70t, I:145t, I:262, I:561, I:562, I:569, I:570, I:574, I:591, II:67, II:118, II:304, II:334, II:349 Anesthesia circu it, I:22t, I:44 Anesthesia m achine checkout instru ctions, I:52 Anesthesia risk, I:211 Anesthetic, halogenated volatile, I:503 Anesthetic p otencies, I:381 Anesthetic toxicity, I:114t Aneu rysm , abd om inal, II:112 Aneu rysm , intracranial, II:179

Aneu rysm , thoracoabd om inal, II:148 Aneu rysm rep air, II:110, II:134–136, II:154 Aneu rysm al rebleed ing, II:487 Angina, I:305 Angiography, II:217 Angiotensin-1 receptor (AT1 recep tor), I:520 Anion gap, I:362, II:20t Ankle block, II:82 Anorexia nervosa, I:8t Antacid , I:446 Antenatal cou nseling, II:252 Anterior ischem ic op tic neu ropathy, I:42t Anterior rad icular arteries, II:64 Anterior sp inal artery, II:64 Anthrax, II:454 Antiarrhythm ic d ru gs, I:454 Antibiotics, II:55t, II:373, II:493, II:507 Anticholinergic synd rom e, I:402, II:1 Anticholinesterase d rugs, I:395 Anticoagulant d rugs, I:86t Anticoagulation, I:109t, I:414, I:418, I:559, II:10 Anticonvu lsant activity, I:379 Anticonvu lsant agent, I:504 Antid iu retic horm one (ADH ), I:255, I:337, I:363, I:517 Antiem etic effects, I:489 Antihypertensives, I:76, I:83t, I:499, I:512, II:30 Antim icrobial therapy, I:491, II:7t Antiphospholip id synd rom e, II:99t Antiplatelet therapy, I:81, II:145 Antipsychotic agents, II:37t Antithrom bin III d e ciency, I:75t, I:340 Anxiety, I:118, II:48 Anxiety d isord er, II:13 Aorta, overrid ing, I:93t Aortic regurgitation, I:87, I:143, II:132 Aortic stenosis, I:86, I:97, I:120, I:131, I:318, II:142, II:151 Aortic surgery, II:1t, II:110–111, II:134–136, II:154–155 Aortic valve rep lacem ent, II:162 Apgar score, II:75t, II:127t, II:253 APL valve. See Ad ju stable pressure lim iting valve Apnea, I:296, II:3, II:241 Apnea, obstru ctive sleep, I:338, II:140t, II:435 Apnea m onitoring, II:341 Apneic oxygenation, II:130, II:423 Aprepitant, II:40 Arachnoid m ater, I:564, II:67

Inde x

Arachnoid villi, I:67t, I:245 Argatroban, I:414, II:10 Arginine vasopressin, I:363, I:517 Argon, I:10, I:12 Arnold –Chiari m alform ation, I:283 Arrhythm ia, I:21t, I:78t, I:93, II:17, II:100t Arteria rad icu laris m agna, I:79t Arterial blood , I:237 Arterial catheter, I:100t, II:80t Arterial oxygen concentration, I:34t Arterial oxygen tension, I:98t Arterial w aveform , II:486 Arteriovenous m alform ation, I:287, I:540 Arteriovenous pressu re d ifference, I:188 Arthritis, rheum atoid , I:408, I:565, I:568 Arti cial pacing, II:99. See also Card iac p acing ASA. See Am erican Society of Anesthesiologists ASA d if cu lt airw ay algorithm , II:264 ASA Physical Statu s, I:77t Ascites, I:314 Aspiration, II:123 Aspirin, I:306, II:18, II:42, II:407, II:584 Aspirin overd ose, I:496 Asthm a, I:406, I:463–464, II:314 Asystole, I:588, II:110 AT1 receptor. See Angiotensin-1 receptor Atelectasis, I:196, II:350, II:368, II:496 Atenolol, II:30 Atracu rium , I:35t, I:85t, I:371 Atrial collapse, II:137 Atrial brillation, I:140, I:355, I:407, I:454 Atrial septal d efect, II:114t Atrioventricu lar block, I:413 Atrioventricu lar cond u ction tim e, I:103t Atrioventricu lar nod al rhythm s, II:143 Atropine, I:141t, I:398, I:431, II:43, II:148t, II:239, II:436, II:462 Au scu ltation of thorax, I:19t Au tom atic noninvasive blood pressure m easuring d evice, I:53 Au tonom ic hyperre exia, I:261 Au tonom ic nervous system , II:85t Au toregu lation, II:176 Aw ake beroptic intu bation, I:101t, I:592, I:597 Aw areness risk, I:551 Axillary block, II:69, II:79t Azygos veins, II:131 A-δ bers, II:531

B Bain circu it, I:39, I:40 Baricity, I:32, I:47t Barium sw allow, II:123 Barom etric pressure, I:35 Basal ganglia, I:248 Basilar artery, I:120t, I:264 B-cell lym phom a, I:343 Beckw ith-Wied em ann synd rom e, II:395 Bed tim e sed ative, II:44 Benzocaine, I:101t, I:592 Benzod iazepine, I:439, I:507, II:44, II:134t Benztropine, II:51 Bernou lli theorem , I:96t β-Ad renoceptor agonist, II:31t β 1-Ad renoceptor antagonism , II:47 β-Ad renoceptor antagonist, I:305, I:334, II:30, II:47 β 1-Ad renoceptor effects, II:4 β-Ad renoceptors, I:66t, I:124t β 1-Ad renoceptors, I:434 β 2-Ad renoceptors, II:85t β w ave, I:251 Bicarbonate, I:207, II:302 Bicuspid aortic valve, I:16t Bier block. See IV regional block Bind ing sites, com petition for, I:1t Biologic terrorism , II:454–456 Biopsy-proven hepatic cirrhosis, I:75t Biotransform ation, I:542 Bipolar d isord er, II:15, II:19, II:49 Birth control pills, I:306 BIS. See Bispectral ind ex m onitor Bispectral ind ex m onitor (BIS), I:36, I:62, II:451 Bisphosphonates, II:532 Blad d er function, I:569 Bleed ing, II:440 Bleom ycin, II:16 Blood coagulation, I:91t, I:399 Blood concentration of d ru g, I:440 Blood ow, II:40t Blood ow rate, II:113 Blood gas analysis, I:496 Blood glucose, I:350 Blood grou p, I:84 Blood loss, I:231, II:313, II:342 Blood p ressu re, I:25t–26t, I:139t, I:261, I:297, II:69t, II:115 Blood p ressu re cu ff, I:38, I:47 Blood p ressu re m onitor, II:107

435

436

Inde x

Blood su pply, I:38t, II:95t Blood transfu sion, I:92t, I:324, I:429 Blood volum e, II:313 Blood -brain barrier, I:90t, I:281 Blood :gas partition coef cient, I:58t, I:543 Blunt-tip need les, I:586 BN P. See B-type natriu retic p ep tid e Bod y uid , II:361 Bod y position, I:56t Bod y tem perature, I:236 Bolus intravenou s injection, I:393 Bone m arrow harvest, I:354 Bosentan, I:516 Botulinu m toxin, II:9 Botulism , II:456 Bow el obstruction, II:506–507 Bow el resection, I:66t Boyle’s law. See Gas pressu re and volum e Brachial plexu s absorp tion rate, I:583 Brachial plexu s block, I:378, I:449, I:589, II:77, II:86 Brachiocephalic vein, II:155 Brad ycard ia, I:70t, II:49t, II:386, II:578 Brain, II:175 Brain, norm al, I:280 Brain biopsy, I:239 Brain stem , I:120t, I:254 Brain sw elling, I:287 Brain tissu e, I:237 Brain tu m ors, I:288 Breathing, II:367 Breathing, w ork of, I:203, I:214 Breathlessness, I:148t Breech presentation, II:272 Bronchial blockad e, II:100 Bronchial venou s system s, II:131 Bronchoconstriction, I:488 Bronchod ilation, I:141t, II:7 Bronchod ilator therapy, II:39t Bronchopleu ral stula, II:156 Bronchoscopy, I:19t, II:125, II:380, II:506 Bronchospasm , I:412, II:152 B-typ e natriu retic peptid e (BN P), I:78 Bu d esonid e, I:406 Bu m etanid e, I:77 BUN elevation, I:349 Bu pivacaine, I:55t, I:123t, I:377, I:568, I:571, I:574, I:588, II:6t, II:72–74, II:598 Bu propion, I:97t, II:8 Bu rns, I:21t, I:28t, II:131t, II:315, II:443

Bu spirone, II:48 Bu torp hanol, I:107t, I:447 C Caffeine, I:426 Calciu m , II:23, II:477–478 Calciu m channel, II:539 Calciu m channel blockers, I:103t Calciu m chlorid e, I:41t Calories, I:60t Cancer pain, II:548, II:576 Cannon w aves, II:143 Capacitance, I:5 Capacitive reactance, I:6 Capillary perm eability, I:212 Capillary pressu re, I:212 Capnography, I:19t, I:68, II:205 Captopril, I:472, I:499 Carbam azepine, I:504, II:49 Carbon d ioxid e, I:11–12, I:14, I:17, I:73, I:155, I:170, I:207, I:218, I:289, II:45t, II:130, II:392 Carbon d ioxid e absorbent, I:48, I:48t, I:49, I:57 Carbon d ioxid e form ation, I:116t Carbon d ioxid e response cu rve, I:37t Carbon m onoxid e, I:14, I:48t, I:49, I:155, I:481, I:492 Carbon monoxide diffusion capacity (DLCO), I:81t, I:174 Carbon m onoxid e poisoning, I:182, II:437 Carboxyhem oglobin, I:155, I:182 Carcinoid synd rom e, I:325, I:336 Card iac arrest, I:479 Card iac arrhythm ias, I:10t Card iac catheterization, II:95t, II:159 Card iac contu sion, II:158 Card iac d ysrhythm ia, I:209 Card iac output, I:61t, I:97, I:141, I:271, I:434, I:536, I:543 Card iac pacing, II:94 Card iac rhythm -m anagem ent d evice (ICD/ pacem aker), II:105 Card iac shunts, I:209 Card iac toxicity, I:377 Card iogenic shock, II:520 Card iom egaly, I:102t Card iom yopathy, II:26 Card iopu lm onary bypass (CPB), I:119, II:9t, II:40t, II:45t, II:103, II:113–114, II:140, II:160, II:336 Card iopu lm onary resu scitation, I:430 Card iovascular collapse, I:488, II:318 Card iovascular d isease, I:122t Carotid bod ies, I:183

Inde x

Carotid end arterectom y, I:168, II:13t Catecholam ines, II:416 Cau d a equina synd rom e, II:225 Cau d al anesthesia, II:292, II:388 Cau d al term ination, I:561 CBF. See Cerebral blood ow CC. See Closing cap acity CDH . See Congenital d iaphragm atic hernia Cefazolin, I:491 Cefep im e, II:54t Celecoxib, II:128t Celiac plexus, I:265 Celiac plexus block, II:59t, II:559 Central d iabetes insip id us, II:516 Central line-associated blood stream infection, II:481 Central nervou s system , II:278 Central pain, II:557 Central venou s catheter, I:129, II:481 Central venou s pressure, II:143 Cephalad position, I:216 Cerebral aneu rysm clip obliteration, II:209 Cerebral artery infarction, II:528 Cerebral au toregu lation, I:285 Cerebral blood ow (CBF), I:15t, I:90t, I:99t, I:257, I:268, I:270, I:451 Cerebral blood volum e, I:288, I:289 Cerebral ed em a, I:131t, II:66t Cerebral ischem ia, I:271, II:52t Cerebral m etabolic rate of oxygen consum ption (CMRO 2), I:90t, I:268–269, I:456, II:98, II:175 Cerebral oxygen requ irem ent, II:177 Cerebral palsy, II:389 Cerebral perfusion pressure (CPP), I:259, II:189, II:192 Cerebral salt w asting, II:514 Cerebral vasospasm , I:263, II:217–219 Cerebral ventricle, I:67t Cerebrospinal uid (CSF), I:67t, I:237, I:245, I:266, I:282, I:288, I:564, II:67, II:220, II:278 Cerebrospinal uid pressure, I:87t Cerebrovascu lar accid ent, II:236 Cerebrovascu lar d ilatation, II:202 Cerebrovascu lar d isease, I:25t Cervical epid ural steroid injection, II:75 Cervical nerve root injection, II:560 Cervical spine instability, II:216 Cervical surgery, II:198 Cervicogenic head ache, II:70t Cesarean d elivery, II:88t Cesarean section, II:232, II:240–241, II:243, II:255, II:299 C- ber activity, II:535

Check valves, I:46, I:54 Chem oreceptor trigger zone, I:276, I:467 Chem oreceptors, I:17t Chest com pressions, II:509 Chest p ain, I:148t Chest rad iograph, I:19t Chest tube su ction, II:29t Chest w all com pliance, I:202 Chest w all ed em a, I:175 Chest w all transm ural p ressu re, I:127t Chest w all traum a, I:233 Chi squared test, I:4 Child birth, natu ral, II:279–280 Child -Pugh class B, II:399 China, II:41 Chlorhexid ine, II:459 Chlorid e cond uctance, I:439, II:134t Chloroprocaine, I:449 Chlorprom azine, II:37t Chlorthalid one, I:77 Cholangitis, II:406–407 Cholecystectom y, I:322, I:348 Cholinesterase, II:298 Christm as d isease. See H em ophilia B Chronic obstructive p ulm onary d isease (COPD), I:143t, I:223, I:310, I:527, II:150, II:419–420 Chronic pulm onary hypertension, I:150t Chronic renal failu re, I:291 Chvostek sign, II:71t Cim etid ine, I:437, I:486, I:550 Circle system , I:22t, I:535 Cirrhosis, I:75t, I:326, I:357, II:408–409, II:415–416 Cisatracu riu m , I:35t, I:396, I:505, I:547 Cisplatin, I:469 Clam ping of d istal aorta, II:1t Clam ping of thoracic aorta, II:110 Clarithrom ycin, I:308 Cleft palate, II:310 Clind am ycin, I:485, II:7t Clonid ine, I:506, I:568, II:31 Clopid ogrel, I:81, II:18, II:145 Closed head trau m a, II:121t Closing capacity (CC), I:45t, I:215, I:224, II:368 Closing volum e, I:13t, I:230 Clostridium perfringens infection, I:485 Clotting ability, II:475 Clotting factor, I:23t, I:331 Clozapine, II:37t Clu ster head aches, II:571

437

438

Inde x

CMP. See Com prehensive m etabolic panel CMRO 2. See Cerebral m etabolic rate of oxygen consum ption CN S excitation, II:94t CO 2 laser ablation, II:77t Coagu lation, I:100, II:141t Coagu lop athy, I:23t, II:415–416 Coarctation of aorta, II:369, II:394 Coarse hair, I:316 Cocaine, I:82t, I:425, I:573 Cod eine, II:50 Cognitive function, II:13 Colectom y, I:320–321, I:335 Collecting d u ct, I:521 Colloid solu tions, I:28 Com bined spinal-epid ural (CSE), II:290 Com bitube, I:197 Com partm ent synd rom e, II:478 Com plex regional pain synd rom e (CRPS), I:595–596, II:76t, II:96t, II:556, II:563, II:575 Com plex regional pain synd rom e typ e II (CRPS II), II:551 Com pliance cu rve, I:240 Com prehensive m etabolic panel (CMP), II:480 Com pressed spectral array (CSA), I:277 Cond uction, I:3 Cond uction anesthesia contraind ications, II:230 Congenital card iac lesion, II:114t Congenital d iaphragm atic hernia (CDH ), II:316 Congestive heart failu re, I:118, I:325, I:455, I:499, II:359–360, II:515 Constipation, II:45 Continu ou s cervical p aravertebral blocks, I:600 Continu ou s interscalene block, II:71 Continu ou s positive airw ay p ressu re (CPAP), II:102 Contractu res, I:301 Convection, I:3 COPD. See Chronic obstru ctive pulm onary d isease Cord com p ression, II:62t Core tem peratu re, I:125t, I:126t Corneal abrasion, I:529 Coronary artery d isease, I:305, I:307, II:10, II:18, II:30, II:96 Coronary blood ow, I:95, I:116, I:136, II:109 Coronary circulation, I:109 Coronary p erfu sion pressu re, I:96 Coronary steal, I:462 Corrected QT interval (QTc), I:10t Corticosteroid s, I:463 Cough, I:87t, II:92t, II:337

COX. See Cyclooxygenase COX inhibitors, II:93t, II:130t, II:585, II:588 COX-2 inhibitors, II:93t, II:128t CPAP. See Continu ous positive airw ay pressu re CPB. See Card iopulm onary bypass CPP. See Cerebral perfusion pressu re CPR, II:508–513 Cranial nerves, I:250 Craniotom y, I:451, II:193, II:199 Cranium , I:275 Craw ford classi cation, I:91, II:148 Creatinine clearance, I:303 Creu tzfeld t–Jakob d isease, I:299 Cricoid cartilage, II:536 Cricothyroid m em brane, I:104t Crohn d isease, I:577 CRPS. See Com plex regional pain synd rom e CRPS II. See Com plex regional p ain synd rom e type II Cryop recipitate, I:89, I:331, II:475 CSA. See Com pressed spectral array CSE. See Com bined spinal-epid ural CSF. See Cerebrosp inal u id Cu ff pressu res, II:192 Cu ffed end otracheal tu be, I:548 Cu taneou s innervation, II:78 Cyanosis, I:93t, I:114, I:139t Cyclooxygenase (COX), II:46, II:584 Cytochrom e P450, I:497, I:508 Cytom egaloviru s, I:299 D Dalton’s law, I:73t Dantrolene, II:23 DBS. See Dou ble-bu rst stim ulation DCD. See Organ d onation after card iac d eath DDAVP. See Desm opressin Dead space, I:184, II:294, II:331 Deep vein throm bosis, I:366 Dehyd ration, I:118t, I:132t Deliriu m , I:507, II:144t Deliriu m trem ens, II:143t Delta w ave, I:251 Dem entia, II:145t Dend rites, I:269 Depression, I:32t, I:318, II:2, II:17, II:244–245 Desaturation, II:397 Des u rane, I:14, I:22t, I:48t, I:49, I:381, I:385, I:481, I:531, I:532, II:16t Desm op ressin (DDAVP), I:89, I:443 Detached retina, II:433

Inde x

Dexam ethasone, I:409, II:201, II:480 Dexm ed etom id ine, II:43t Dextrose, II:41t DI. See Diabetes insip id us Diabetes insipid us (DI), I:351, II:516 Diabetes m ellitus, I:3t, I:25t, I:52t, I:130, I:313, I:320–321, I:323, I:330, II:10, II:18, II:30, II:53, II:104t– 106t, II:150, II:410, II:441 Dialysis, I:291, II:49 Dialysis d isequilibriu m synd rom e, II:32 Diam eter ind ex safety system (DISS), I:64 Diap hragm , I:185, I:216 Diap hragm atic hernia, II:158 Diarrhea, I:6t, I:325 Diazepam , I:424, I:486, I:557, II:34, II:42, II:48 DIC. See Dissem inated intravascu lar coagu lation Diethyl ether, I:11, I:528 Dif cu lt airw ay, I:101t–102t, II:129, II:264 Diffu se lobar in ltrates, II:482 Diffu sion barrier, I:173 Diffu sion hypoxia, I:232 Digoxin, I:455 Dip yrid am ole, I:79 Discharge hom e, II:432 Disconju gate gaze, I:538 DISS. See Diam eter ind ex safety system Dissem inated intravascular coagulation (DIC), I:333, II:141t Dissociative anesthesia, I:470 Diu retics, I:83t Diverticu litis, I:320–321, I:335 DLCO. See Carbon m onoxid e d iffu sion capacity Dobu tam ine, I:514, II:4, II:412, II:503 Dofetilid e, I:407 Dop am ine, I:140t, I:267, I:434, II:490 Dop am ine, glycine, serotonin, γ-am inobutyric acid (GABA), I:267 Dop am ine D 1 receptor agonist, II:35 Dop pler effect, I:22 Dop pler u ltrasonic transd ucer, II:210 Dorsal colum ns, I:247 Dorsal horn, II:533 Dorsal p enile block, II:322 Dorsal root ganglion, I:278 Dorsalis p ed is artery, I:26t Dose-resp onse relationship, I:63t–64t, I:382–383 Dou ble-burst stim u lation (DBS), I:397 Dou ble-lu m en tu be, I:154, II:27t, II:28t, II:102, II:124–127, II:129t Dow n synd rom e, II:163, II:216, II:344, II:386

439

Doxoru bicin, II:26 Dream ing, I:507 Droperid ol, I:550, II:38 Drug absorption, I:564 Drug abu se, I:95t Drug antagonism , I:473 Drug bioavailability, I:7t Drug clearance, I:436 Drug elim ination, I:367 Drug load ing d ose, I:453 Drug m etabolism reactions, I:373 Drug overd ose, I:496 Drug resp onse, I:445 Drug transform ation, I:4t Drying effect, II:101 Du chenne m u scu lar d ystrophy, I:342 Du ctus arteriosu s, I:108, I:114, II:326 Du ra m ater, II:193 Dw ar sm , I:302 Dysphagia, II:372–373 Dyspnea, I:61t, I:147t, I:151, I:570, II:504–505 Dysrhythm ias, I:8t, I:170, I:263 E Ear tubes, II:344 Eaton-Lam bert synd rom e, I:484. See also Myasthenic synd rom e EBP. See Epid u ral blood patch ECG. See electrocard iography Echocard iography, I:96t, II:26, II:479, II:498 Echothiophate iod id e, I:474 Eclam psia, II:78t, II:283 ECT. See Electroconvu lsive therapy Ed rop honium , I:431 EEG. See Electroencephalogram Eisenm enger synd rom e, I:144 Eld erly patient, I:45t, II:438, II:445 Electrocard iography, I:66, I:99, I:139, I:588, II:96, II:463 Electrocau tery, I:18, II:105 Electroconvulsive therapy (ECT), II:187, II:244–245 Electroencephalogram (EEG), I:251, I:263, I:273, I:277 Electrolyte abnorm alities, I:292, II:364, II:409 Electrolytes, I:416 Em ergence sequ elae, I:507 Em ergency airw ay access, I:104t Em ergency backu p pow er, I:25 Em ergency ventilation, I:228 Em physem a, I:171 En-bloc d ouble-lung transp lant, II:146 End obronchial intu bation, I:154

440

Inde x

End oscopic sphincterotom y, II:406–407 End othelin-A recep tor (ETA), I:516 End otracheal anesthesia, II:402–403 End otracheal intu bation, I:19t, I:220, II:178, II:499 End otracheal tube, I:146t, I:164, I:283, II:327, II:335, II:355 End -stage renal d isease, I:65t End -tid al carbon d ioxid e, I:47, I:207, II:511 End -tid al nitrogen, II:80t Ep id u ral absorp tion rate, I:583 Ep id u ral analgesia, I:576–577, I:590, I:594, II:47t, II:259, II:272, II:302 Ep id u ral blood patch (EBP), II:238, II:285 Ep id u ral catheter, II:65 Ep id u ral hem atom a, I:576, I:590, II:526 Ep id u ral infu sion, II:237 Ep id u ral space, I:564 Ep id u ral steroid injections, II:46t, II:558 Ep iglottitis, II:372 Ep inephrine, I:95t, I:419, I:430, I:458, I:569, I:577, II:139t, II:147t, II:302 ERV. See Expiratory reserve volum e Erythrocyte esterase, II:6 Erythropoietin, I:135 Esm olol, I:5t, II:6, II:18t Esop hageal carcinom a, I:298 Esop hageal electrod es, II:99 Esop hageal stethoscope, II:80t Esop hageal tem peratu re probe, II:428 Esop hageal-tracheal airw ay, I:197 Esop hagectom y, II:119 Esop hagoscopy, II:123 Estrogen, II:30t ESWL. See Extracorporeal shock w ave lithotripsy ETA. See End othelin-A receptor Ethacrynic acid , I:77 Ethanol, I:296, II:27, II:94t Etom id ate, I:118t, I:379, II:5 Eu phoria, II:45 Eu volem ia, I:324 Evap oration, I:3 Evoked tw itch resp onse, I:32t–33t Ex utero intrapartu m therapy (EXIT), II:300 Exenatid e, II:60 EXIT. See Ex u tero intrapartum therapy Expiratory reserve volum e (ERV), I:230, II:495 Extracorporeal shock w ave lithotripsy (ESWL), I:312, II:417–418 Extubation, I:167

F Facial nerve, I:250 Facial pain, II:22t Factor V Leid en, I:340 Fail-safe valve, I:74 Failure to thrive, II:359–360 Fasting, I:118t Fasting blood su gar, I:25t Fat em bolism , I:101, I:117t Fem oral and popliteal sciatic block, II:82 Fem oral nerve, II:66 Fem oral nerve block, I:578, I:587, II:83 Fenold op am , II:35 Fentanyl, I:94, I:300, I:308, I:479, I:486, I:505, I:509, II:20, II:133, II:359, II:430 Fetal blood capillary pH testing, II:282 Fetal circulation, II:362 Fetal concentration, II:288 Fetal hem oglobin, II:136t Fetal m onitoring, II:74t, II:297 FEV1, I:122t, I:162, I:226 Fever, I:132t, II:53t–55t, II:372–373 Fexofenad ine, I:405 FH R. See Fetal heart rate Fiberoptic bronchoscopy, I:71, II:101 Fiberoptic oxim eter probe, I:47 Fibrom yalgia, II:547 Fibrosis, I:229 First trim ester of pregnancy, I:557 First-pass effect, I:390, I:427 Fisher Grad ing Scale, II:487 Flail chest, I:175 Flecainid e, I:394 Flow m eter, I:54 Flow -volum e loop, I:204–205, II:68t Fluid m anagem ent, I:358, II:12, II:339, II:352, II:502 Flum azenil, I:402 Fluoxetine, I:318, II:2 Follicle-stim u lating horm one, I:255 Fond aparinu x, I:366 Forced exhaled vital capacity (FVC), I:217 Foreign bod y, II:39t, II:328, II:380 Fosp ropofol, I:515 Fow ler p osition, I:55 Frank-Starling curve, I:513–514 Fresh frozen plasm a transfu sions, I:41t Fu nctional resid ual cap acity (FRC), I:56t, I:89t, I:150, I:161, I:164, I:169, I:206, I:214–215, II:368, II:404 Fu rosem id e, I:306, I:514, I:523, II:48t FVC. See Forced exhaled vital capacity

Inde x

G GABA. See γ-Am inobutyric acid Gabapentin, II:19, II:595 “Gag re ex,” I:597 γ-Am inobutyric acid (GABA), I:267 Gas gangrene, I:485 Gas pressure and volum e (Boyle’s law ), I:15 Gas pressure regu lator, I:54 Gastric em p tying, I:8t, I:386 Gastrod uod enal u lcers, I:486 Gastroesop hageal re u x, I:550, II:389 Gastrointestinal sm ooth m u scle, I:438 Gastroschisis, II:352 Gastrostom y tube revision, II:381 Gate control theory, II:583 GCS. See Glasgow Com a Scale General anesthesia, I:31t, I:134t, I:193, I:227, I:286, I:348, I:509, I:524–558, II:10t, II:38t, II:231, II:355 Genitofem oral nerve, II:66 Gentam icin, I:384 Georgia valve, I:39 Gilbert synd rom e, I:354 Glasgow Com a Scale (GCS), I:256, II:471 Glau com a, I:51t, I:384, II:83t Glom eru lar ltration rate, II:352 Glossopharyngeal nerve, I:250, I:597 Glossoptosis, II:310 Glu cagon, I:438 Glu cagon-like p ep tid e-1 recep tor agonist, I:3t Glu cocorticoid s, I:335, I:457–458, II:12, II:573 Glu coneogenesis, I:311 Glu cose intolerance, I:102t Glu cose m etabolism , I:311 Glu curonid es, I:380, I:497 Glu teal sciatic block, II:80 Glybu rid e, I:52t Glycine, I:267 Glycine toxicity, II:413 Glycogen storage, I:311 Glycogenolysis, I:311 Glycop yrrolate, I:421, II:101, II:132 Goiter, I:355 Gou t, I:339 Graft reperfusion, II:408 Granisetron, I:467 Graves d isease, I:353, II:119t, II:406–407 Gravity, I:189 Ground fau lt interru pter, I:21 Grow th horm one, I:255

441

H H alf-life of d ru g, I:441 H alo brace, II:110t H alogenated volatile anesthetics, I:503 H aloperid ol, II:37t, II:51 H alothane, I:73, I:381, I:532 H arrington rod instru m entation, II:195–197 H ay fever, I:405 H BO cham ber. See H yperbaric oxygen cham ber H CG. See H u m an chorionic gonad otropin H ead com p ressions, II:271 H ead inju ry, II:185 H ead aches, II:42t, II:571, II:581 H ead -d ow n position, I:56t H eart d isease, I:78t H eart failure, I:223 H eart m u rm ur, I:147t H eart rate, I:76t, I:139t, I:261, I:370, I:577, II:69t, II:117t–118t, II:133, II:142, II:270–271 H eart transplantation, I:66t, I:138, I:150t H eat loss, operating room , I:3 H eliu m , I:10–12 H em atocrit, II:15t, II:265 H em iazygos veins, II:131 H em id iaphragm atic p aresis, I:143t H em od ynam ic m onitoring, II:80t, II:474 H em oglobin, I:82 H em oglobin level, II:340 H em oglobin satu ration, I:111t H em oglobinu ria, I:92t H em olytic transfu sion reaction, I:92t, I:332 H em op hilia B (Christm as d isease), I:23t H em op tysis, II:374 H end erson–H asselbalch equ ation, I:450 H eparin, I:91t, I:399, I:418, II:22 H eparin-ind uced throm bocytopenia (H IT), I:18t, I:80, I:366, I:495, II:10, II:159, II:473 H epatic artery, I:346 H epatic circu lation, I:346 H epatic failu re, I:327 H epatic glu coneogenesis, I:438 H epatic portal vein, I:346 H epatitis A, I:299 H epatitis C, I:304, I:327 H epatocellu lar inju ry, I:505 H epatorenal synd rom e, I:357, II:518 H epatosplenom egaly, I:302 H ering–Breuer re ex, I:29t H ernia, I:81, II:341 H erniation, II:348

442

Inde x

H eroin, I:386, I:494, II:20, II:276 H erp etic w hitlow, I:365 H etastarch, I:429, II:3t H FJV. See H igh frequency jet ventilation H igh frequ ency jet ventilation (H FJV), I:163 H igh oxygen extraction ratio, I:356 H igh-d ensity lipoprotein cholesterol, I:25t H ip prosthesis im plantation, I:297 H istam ine, I:35t, I:371 H istam ine H 2 antagonists, I:510 H IT. See H eparin-ind u ced throm bocytop enia H oarseness, II:122 H orizontal sup ine position, I:55 H orm ones, I:255, I:329 H PV. See H ypoxic pu lm onary vasoconstriction H SV-2 infection, II:295 H um an chorionic gonad otrop in (hCG), II:249 H um id i ed oxygen, II:385 H um id ity, I:24 H urler synd rom e, I:302 H yaline m embrane d isease, II:350 H yd ralazine, I:444 H yd ration, II:373 H yd rochlorothiazid e, I:77 H yd rocortisone, I:348, I:457, II:12 H yd rom orp hone, I:94, II:50 H yd roxyethyl starch, II:494 H yperald osteronism , I:61t, I:352 H yperbaric oxygen cham ber (H BO cham ber), II:126t H yperbaric solution, I:565 H ypercalcem ia, I:361 H ypercarbia, I:82, I:106t, I:119, I:122, I:184, I:209, I:222, II:330, II:525 H ypercholesterolem ia, II:115 H ypercoagu lable state, I:340 H yperd ynam ic septic shock, II:523 H yperem esis, II:266 H yperglycem ia, I:350 H yperin ation, I:171 H yperkalem ia, I:6t, I:21t, I:77, I:291, I:341, II:477 H yperosm olar com a, I:322 H yperparathyroid ism , I:328, I:361 H yperreactivity, I:122t H ypertension, I:71t, I:97t, I:102t, I:107, I:108t–109t, I:125, I:150t, I:319, I:321, I:352, I:360, I:403, I:432, II:10, II:18, II:31, II:35, II:115, II:419–420, II:444, II:447 H ypertherm ia, I:106t, I:271, I:511, II:252, II:330 H yperthyroid ism , I:30t, I:133t, I:353, I:461 H ypertonic saline, II:200

H ypertrophic obstru ctive card iom yopathy, I:145 H yperventilation, I:109, I:169–170, I:210, I:282, I:451, II:180, II:184 H ypervolem ia, I:291 H ypoalbu m inem ia, II:20t H ypobaric solu tion, I:565 H ypocalcem ia, I:290, I:328 H ypoglycem ia, I:320–321, II:395 H ypokalem ia, I:292, I:352, I:416 H ypom agnesem ia, I:459 H yponatrem ia, I:6t, I:263 H ypoperfusion, I:234 H ypotension, I:6t, I:8t, I:119, I:146t, I:149t, I:297, I:348, I:371, I:571, I:574, II:11, II:24, II:29, II:89t, II:94, II:135–136, II:139, II:241, II:281, II:287, II:354, II:421, II:450–451 H ypotherm ia, II:206, II:311, II:343, II:513 H ypothyroid ism , II:343, II:448 H ypotonic cerebral palsy, II:381 H ypoventilation, I:169, I:190, I:234, II:343, II:496 H ypovolem ia, I:76t, I:100t, I:339, I:363, I:502, II:112, II:517, II:519 H ypoxem ia, I:27t, I:92, I:117t, I:119, I:122, I:132t, I:139t, I:168–169, I:173, I:194, I:231, I:234, I:338, II:24, II:26t, II:53t–55t, II:102, II:139, II:496 H ypoxic p ulm onary vasoconstriction (H PV), I:20t, I:92, I:208, II:153 H ypoxic ventilatory d rive, I:17t I IABP. See Intraaortic balloon p um p IASP. See International Association for the Stud y of Pain Ibu profen, II:435 ICD. See Card iac rhythm -m anagem ent d evice ICP. See Intracranial pressure I:E ratio, I:50 Iliohypogastric nerve, II:66 Ilioingu inal nerve, II:66 Im m unosup pression, I:347 Inam rinone, I:518 Incisional hem orrhage, II:405 Ind u ced hypotherm ia, II:206 Ind u ced labor, II:235 Ind u ction of general anesthesia, II:188 Inert gas d ilution techniqu e, I:206 Infants, I:29t, II:81t Infection, I:97t, I:118t, II:156, II:384–385 Inferior collicu lu s, I:247 In am m ation, II:75, II:541

Inde x

In am m atory bow el d isease, I:345 Inform ed consent, I:581, I:584, I:590, II:83, II:235 Infraclavicular block, II:68, II:79t, II:92 Infrared spectrom etry, I:73 Infusion rate, I:554 Ingu inal hernia repair, II:355 Inhalation ind uction, II:354 Inhalation inju ry, II:131t Inhalational anesthesia, I:58t, I:456, I:470, I:481 Inhaled gas, I:219 Inhibitory su bstances, II:591 Injectable benzod iazepines, I:439 Innom inate artery, II:121, II:155 Inotrope, I:106 Inspiratory airw ay resistance, I:221 Inspiratory reserve volu m e, I:230, II:294 Inspiratory strid or, II:353 Inspiratory volu m e, I:205 Inspired concentration, I:543 Insu f ation w ith carbon d ioxid e, II:392 Insu lin p um p, I:323 Intercostal absorp tion rate, I:583 Intercostal blocks, II:598 Internal branch of the su perior laryngeal nerve, I:598 International Association for the Stu d y of Pain (IASP), II:70t International H ead ache Society, II:70t Interscalene block, I:55t, I:143t, II:76, II:87 Intraaortic balloon pum p (IABP), II:95, II:109, II:141 Intraarterial injection, I:555 Intracard iac shu nts, I:234 Intracerebral blood ow, II:176 Intracranial pressu re (ICP), I:72t, I:209, I:237, I:239–243, I:259, I:263, I:270–271, I:275, I:288, I:451, II:121t, II:178, II:188, II:193, II:200, II:211–213, II:383 Intracranial tissu e content, I:288 Intralip id , I:114t Intram ed u llary pressu re, II:425 Intram u scu lar injection, I:477 Intraneural injection, I:579 Intraneural need le placem ent, I:585 Intraocu lar pressu re, II:83t Intraoperative arterial oxygenation, II:23t Intraoperative aw areness, I:9t, I:537 Intraoperative u id replacem ent, I:554 Intraoperative hepatocellular injury, I:505 Intraoperative hyp otherm ia, II:311 Intrap leural catheters, II:545

443

Intrathecal d ru g-d elivery, II:577 Intravenous bolu s, I:128t-129t Intravenous d rug, I:452 Intu bation, I:20, I:46t, I:49t, I:392, I:396, I:464, II:138, II:345, II:373, II:401 In vitro fertilization (IVF), II:249 Involuntary m uscle m ovem ents, I:115t Ionization, I:465 Ipsilateral d iaphragm paralysis, II:87 Irrigating uid s, I:105t Ischem ia, II:96 Ischem ic colitis, II:411 Ischem ic heart d isease, I:417 Isobaric bu pivacaine, I:591 Isobaric solution, I:565 Iso u rane, I:35, I:44, I:57t, I:73, I:88t, I:141t, I:381, I:420, I:503, I:509, I:532, I:535, I:541, I:542, I:547, II:3, II:449 Isolated pow er su pply system , I:70, I:112t–113t Isolation m onitor alarm , I:7 Isopropanol, II:42 IV regional block (Bier block), I:593 IVF. See Vitro fertilization J Jackson–Rees system , I:39 Jehovah’s Witness, I:429, II:431 Joint pain, I:325 K K+, II:414 Kaposi’s sarcom a, I:343 Ketam ine, I:14t, I:99t, I:141t, I:239, I:370, I:391, I:507, II:359, II:587 Ketorolac, I:501 Kid ney injury, I:307 Kid ney transplant, I:347, II:104t–106t Kid neys, I:500 L Labetalol, I:401 Labor, II:275 Labor epid ural analgesia, I:563, I:582, I:599 Labor p ain, II:113t Lacrim ation, I:498 Laminar ow, I:10 Lamotrigine, I:504 Land m ark technique, II:71, II:161 LaPlace law, I:159 Laryngeal ed em a, II:370

444

Inde x

Laryngeal m ask airw ay, I:47, I:67 Laryngeal nerve injury, II:152, II:446 Laryngectom y, I:548 Laryngoscopy, I:193 Laryngospasm , I:167, II:152 Laryngotracheobronchitis, II:384 Larynx, II:120 LAST. See Local Anesthetic System ic Toxicity Late exp iratory ow, I:137t Latent error, II:452 Lateral d ecu bitu s p osition, II:157 Lateral fem oral cutaneou s nerve p alsy, II:291 Lateral ventricle, I:67t Lavage, II:126 Leakage cu rrent, I:7 LeFort I fracture of the m axilla, I:549 Left d om inant pattern of blood su pply, II:95t Left fem oral nerve block, I:585 Left sid ed d ouble-lum en tu be, II:27t, II:28t, II:129t Left ventricu lar assist d evice (LVAD), II:160 Left ventricu lar end -d iastolic pressu re (LVEDP), I:103, I:104 Lepiru d in, I:414 “Let-go” cu rrent, I:9 Levod opa, II:65t, II:441 Levo oxacin, II:54t Lid ocaine, I:422, I:437, I:449, I:471, I:566, I:567, I:593 Lighting, I:540 Lim b protection, I:560 Lim b-threatening ischem ia, I:555 Line isolation m onitor, I:4, I:112t–113t Lipid em ulsion, I:589 Lipid em ulsion therap y, I:572 Lipid solu bility, I:530 Lipolysis, I:438 Lisinopril, I:403 Lithiu m , II:15 Lithotom y position, I:55, I:69 Lithotriptor, II:418 Liver, I:4t, I:311, I:346, I:422 Liver cirrhosis, I:5t Liver transplant, I:327, II:399, II:408–409, II:440 LMWH . See Low -m olecu lar-w eight heparins Load ing d ose of d ru g, I:453 Local anesthetic absorption rate, I:583 Local Anesthetic System ic Toxicity (LAST), I:449 Loop of H enle, I:523 Lorazep am , II:13

Losartan, I:512, I:514 Low outlet forcep s d elivery, II:57t Low -m olecular-w eight heparins (LMWH ), I:495 Lu m bar d rain, II:117 Lu m bar epid ural block, I:567 Lu m bar epid ural hem atom a., I:567 Lu m bar facet joints, II:537 Lu m bar plexus block, I:588, II:66, II:80 Lu m bar pu nctu re, I:561 Lu m bar sym pathetic blockad e, II:542 Lu m bar vertebra, I:279 Lu ng, I:2t, I:136t, I:186–189, II:126, II:144 Lu ng, hyperin ation, II:348 Lu ng, ventilation d istribution, I:165 Lu ng com pliance, I:171, II:294 Lu ng sound s, II:337 Lu ng transplantation, II:146 Lu pu s anticoagulant, II:243 Lu teinizing horm one, I:255 LVAD. See Left ventricular assist d evice LVEDP. See Left ventricu lar end -d iastolic pressu re Lym phatic insu f ciency, I:212 Lym phom a, II:457 M MAC. See Minim u m alveolar concentration MAC-aw ake, I:545 Macintosh laryngoscop e, I:47, I:60 Macroglossia, I:302 Magnesium sulfate, II:235, II:246, II:250–251 Magnetic resonance im aging (MRI), I:47, II:204 Malignant hypertherm ia (MH ), II:252, II:330 Mannitol, I:131t, II:21t, II:32, II:182 MAO inhibitor. See Monoam ine oxid ase inhibitor MAP. See Mean arterial pressure Map leson D system , I:19, I:39, II:365 Marfan synd rom e, II:11t, II:52t, II:138 Mask ventilation risk, I:198 Mass spectrom etry, II:205 Maternal concentration, II:288 Maternal to fetal d rug transfer, II:258 McGill Pain Qu estionnaire, II:599 Mean arterial pressu re (MAP), I:26, I:27, I:110, I:259, II:499 Mechanical d ead space, I:20 Mechanical ventilation, I:152, I:164, I:221, II:8t Meconium , II:122t, II:351 Med iastinal veins, II:131 Med iastinoscopy, II:106–107, II:121 Med iastinu m , II:457

Inde x

Med ullary chem orecep tors, I:218 Mem antine, II:587 Meningiom a, II:211–213 Meningitis, II:529 Mep erid ine, I:94, I:415, II:50 Mep ivacaine, I:449, I:450 Meta-analysis, I:8 Metabolic acid osis, I:291, I:496, II:484 Metabolic alkalosis, I:293, I:352 Metabolic synd rom e, I:25t Metform in, I:306, II:62 Methad one, I:433, II:50, II:544, II:586 Methanol, II:27 Methem oglobin, I:111t, I:592 Methicillin-resistant Staphylococcus aureus (MRSA), I:491 Methim azole, I:133t, II:119t Methohexital, I:115t, I:389, I:555 Methyld opa, II:109t Methylergonovine, II:36, II:309 Methylm ethacrylate (MMA), I:488 Methylp araben, I:471 Methylxanthines, I:426 Metoclopram id e, I:74t, I:428, I:550 Metop rolol, I:109t, I:355, II:46, II:47, II:136 Meyer-Overton rule, I:530 MH . See Malignant hypertherm ia Micrognathia, II:310 Mid azolam , I:424, I:466, I:556, II:328, II:402–403 Mid d le cerebral artery infarction, II:528 Mid -esop hageal aortic valve long axis, II:166 Mid -esop hageal aortic valve short axis, II:174 Mid -esop hageal ascend ing aortic long axis, II:150t Mid -esop hageal bicaval, II:171 Mid -esop hageal fou r cham ber, II:170 Mid -esop hageal long axis, II:169 Mid -esop hageal right ventricular in ow -ou t ow, II:165 Mid -esop hageal tw o cham ber, II:172 Migraine, II:19 Milrinone, I:135t, I:376, II:14 Mineralocorticoid effect, I:463 Minim u m alveolar concentration (MAC), I:88t, I:525, I:541, I:545, II:278, II:357, II:449 Minu te ventilation, I:543 Miosis, II:39 Misoprostol, II:308 Mitral regurgitation, II:138t Mitral stenosis (MS), II:118, II:263 Mitral valve, I:36t

445

Mivacuriu m , I:396 MMA. See Methylm ethacrylate Mod erate sed ation, I:552 Molar pregnancy, II:267 Molecular w eight, I:531 Monoam ine oxid ase inhibitor (MAO inhibitor), I:415, II:33 Morbid obesity, I:49t, I:107, I:309, II:124t Morbid ity, I:10t, II:277 Morphine, I:94, I:468, I:494, I:497, I:498, I:501, II:105t, II:208, II:426, II:600 Mortality, I:10t, II:236, II:277 Morton, William Thom as Green, I:528 Motor cortex stim u lation, II:557 Motor evoked p otentials, I:274, II:195–197 Motor function, II:13 Motor tract blood supp ly, I:38t MRI. See Magnetic resonance im aging MRSA. See Methicillin-resistant Staphylococcus aureus MS. See Mitral stenosis Multiple sclerosis (MS), II:242 Muscarinic M 1, II:54 Muscarinic M 2, II:55 Muscarinic M 3, II:56 Muscle relaxants, I:33t, I:164, I:236, I:392, I:435, I:537, II:15, II:303, II:363 Muscular d ystrophy, I:342 Musculocutaneous nerve, II:68, II:69 Musculocutaneous nerve block, II:92 Myasthenia gravis, I:119t, I:400, II:4t–5t, II:246 Myasthenic synd rom e (Eaton-Lam bert synd rom e), II:223 Myd riasis, II:39 Myelom eningocele, II:338, II:357 Myocard ial blood ow, II:508 Myocard ial contractility, I:142 Myocard ial hyp ertrophy, I:61t Myocard ial infarction, I:108t–109t, I:139, I:146t Myocard ial ischem ia, II:115t, II:116, II:141 Myocard ial oxygenation, I:95, I:108t Myofascial pain, II:593 Myoneu ral junction, I:1t Myotonic d ystrophy, I:286, I:479 Myringotom y, II:344 Myxed em a, I:316–317 N N a +, II:420 N aloxone, I:375, I:476, II:497 N asal d ischarge, II:92t

446

Inde x

N asotracheal intubation, I:549 N ational Fire Protection Association, I:25 N atural child birth, II:279–280 N au sea, I:122t, I:539, I:574, II:40 N eck circu m ference, I:309, II:401 N eed le, contam inated , I:299 N eed le location, I:563 N egative-pressu re leak test, I:45 N eonates, II:88t, II:334, II:338, II:349 N eostigm ine, I:398, I:435, II:364, II:436 N ephrectom y, I:310 N ephrolithiasis, I:312 N erve blocks, I:49t N erve gas, II:43 N erve inju ry, I:579, II:275, II:291 N erve root im pingem ent, II:135t N erve stim ulation, I:475, I:585 N esiritid e, II:24 N eu raxial anesthesia, II:118 N eu rogenic pu lm onary ed em a, II:215 N eu rogenic shock, II:524 N eu rolep tic state, I:480 N eu rologic inju ry, I:117t, I:560, I:584, I:587 N eu rolytic block, II:102t, II:569, II:576, II:597 N eu rom u scu lar blockad e, I:28t, I:384, I:397, I:400, I:435, I:474–I:476, I:486, II:103, II:436 N eu rom u scu lar transm ission, II:9 N eu ron, I:269 N eu rosu rgery, II:89t, II:182, II:201 N eu rotransm itters, I:267, II:33 N ew York H eart Association functional class III, I:12t N ew born infant, II:127t, II:312 N icard ipine, I:513, II:29 N icotine w ithd raw al, II:8 N ifed ip ine, I:417, II:25 N ipple line, I:144t N IPPV. See N oninvasive positive pressure ventilation N itric oxid e, I:119, I:132 N itrogen, I:10, I:12 N itrogen w ashou t curve, I:13t N itroglycerin, I:109t, II:104, II:135, II:280 N itrop ru ssid e, II:447 N itrou s oxid e, I:10–13, I:50, I:53t, I:56, I:61, I:73, I:420, I:487, I:505, I:509, I:547, I:553, II:60t, II:183, II:202 N MDA antagonist, II:541, II:543 N ociceptive signals, II:553, II:591, II:594 N ociceptor, II:592

N on-d epolarizing neu rom u scu lar blockad e, I:28t, I:400, I:478 N on-H od gkin’s lym phom a, I:295 N oninvasive p ositive pressu re ventilation (N IPPV), II:497 N onparticu late antacid ad m inistration, I:446 N onshivering therm ogenesis, II:347 N on-stress test (N ST), II:228 N orepinep hrine, I:82t, I:140t, II:11, II:141, II:490 N orm ovolem ic hem od ilution, II:390 N ovocaine, I:578 N ST. See N on-stress test N u cleus pu lposu s, II:75 N u trient supp ly, I:252 N u trition, p arenteral, I:298 N ystagm u s, I:14t O Obesity, I:13t, I:49t, I:107, I:309, I:466, II:124t Obstetrical anesthetic m anagem ent, II:299 Obstetrical hysterectom y, II:290 Obstru ctive lu ng d isease, I:524, II:563 Obstru ctive shock, II:521 Obstru ctive sleep apnea, I:338, II:140t, II:435 Occipital neu ralgia, II:589 OCR. See Oculocard iac re ex OCT. See Oxytocin contraction test Octreotid e, I:336 Ocu lar akinesia, I:547 Ocu lar effects, I:547 Ocu locard iac re ex (OCR), I:254, II:2t Ocu lom otor nerve, I:250 Ohm , I:34 Olanzapine, II:37t Oligu ria, II:476 Om eprazole, I:50t, I:508 Om phalocele, II:356 Oncotic pressu re, I:212 Ond ansetron, I:4, I:489, I:550 One-lu ng ventilation, I:157, II:129, II:156 Op en globe inju ry, II:429–430 Op en w aste-gas scavenging system , I:37 Op ioid agonist–antagonist, I:447 Op ioid agonists, I:460 Op ioid antagonism , I:375, I:476 Op ioid receptors, I:253, II:45 Op ioid w ithd raw al, I:506 Op ioid s, II:14t, II:39, II:50, II:90, II:534, II:552, II:568, II:578 Op tic neu ropathy, I:42t

Inde x

Organ d onation after card iac d eath (DCD), II:460 Oropharyngeal su ctioning, II:253 Osm olality, I:19, I:29 Osteoporosis, I:8t Ovarian cysts, II:266 Overd ose, I:300 Overrid ing aorta, I:93t Oxcarbazepine, II:561 Oxycod one, II:50, II:53 Oxygen, I:10–11, I:15, I:82, I:95, I:114t, I:157, I:196 Oxygen, hum id i ed , II:385 Oxygen concentration, I:1, II:422 Oxygen consum ption, II:69t, II:358 Oxygen d elivery, II:464 Oxygen equ ipm ent, I:50, I:58–59, I:63, I:540 Oxygen reserves of brain, I:258 Oxygen satu ration, I:176, I:546, I:592, II:108t, II:125t, II:194, II:321, II:327, II:450–451 Oxygen tension, I:176 Oxygen transport, I:173 Oxygenation, II:23t Oxyhem oglobin d issociation curve, I:176, I:181, I:222 Oxytocin, I:255, II:246, II:279 Oxytocin contraction test (OCT), II:261 P Pacem aker, II:108. See also Card iac rhythm m anagem ent d evice Packed red cells, I:121t Paco 2, I:172, I:282, II:9t Pain, I:433, II:31, II:570 Palp ation, II:388 Pancreatic cyst, II:398 Pancuroniu m , I:46t, I:435, I:464, II:42 Pao 2, II:25t Paralysis, II:37, II:122 Paraplegia, II:111, II:287 Parapneum onic effusion, II:112t Parasym pathetic efferent bers, I:250 Parathyroid ectom y, I:328 Paravertebral block, II:145 Parenteral nu trition, I:298, I:345 Parietal cortex, I:278 Parietal pleura, I:233 Parkinson d isease, I:248, II:33, II:65t, II:441 Paroxysm al hem icrania, II:581 Partition coef cient, I:23

Partu rient, II:264 Patellar tw itch, I:585 Patent foram en ovale, II:227 Patient positioning, I:55, I:65, I:69 Patient-controlled analgesia (PCA), II:293, II:426, II:596 Patient-ventilator d yssynchrony, II:500 PCA. See Patient-controlled analgesia PCC. See Perioperative card iac com plications Pco 2, arterial, I:166 PDE3. See Phosphod iesterase-3 PDE5. See Phosphod iesterase-5 PDPH . See Post-d ural puncture head ache Peak inspiratory ow, II:461 Ped iatric patient, I:44t, II:323–324 PEEP. See Positive end -expiratory pressure Pelvic pain, II:546 Penicillin G, I:485 Peptic u lcer d isease, I:50t Percutaneou s d rainage, II:470 Percutaneou s nephrostom y tu be p lacem ent, II:502–503 Perfusion pressure, II:95 Pericard ial effu sion, I:301 Pericard ial tam ponad e, I:149t, II:160 Pericard iocentesis, II:504 Pericard itis, I:149 Periop erative card iac com plications (PCC), I:313 Periop erative m anagem ent, I:328–329, I:336, I:339, I:386 Peripheral chem orecep tors, I:17t Peripheral ed em a, I:61t Peripheral nerve block, II:72–74, II:84–85 Peripheral neu ropathy, I:295, I:469, II:53 Peripheral vascu lar d isease, II:53, II:150 Peripheral vasod ilation, II:14 PFTs. See Pu lm onary fu nction tests p H , I:11t, I:465 Phenobarbital, I:423, II:42 Phenol, II:569 Phenoxybenzam ine, II:424 Phenylephrine, I:114t, I:574, II:116t Phenytoin, I:423, I:504 Pheochrom ocytom a, I:83t, I:125, I:319, I:360, II:296, II:393 PH N . See Post herpetic neu ralgia; Postherpetic neuralgia Phosphod iesterase, I:376 Phosphod iesterase-3 (PDE3), I:518 Phosphod iesterase-5 (PDE5), I:519

447

448

Inde x

Phosp horus, II:478 Phrenic nerve, I:260, II:87 Physostigm ine, I:402, II:1 Pia m ater, II:67 Pierre-Robin Synd rom e, II:310, II:376 Pin ind ex safety system (PISS), I:62t, I:64 PISS. See Pin ind ex safety system Pitu itary gland , I:255, I:443 Placenta accreta, II:289 Placental abruption, II:307 Placental physiology, II:239 Placental transfer, II:303 Plasm a cholinesterase, II:298 Plasm a oncotic p ressu re, I:112 Platelet cou nt, I:102 Platelet glycoprotein IIb/ IIIa receptor, I:86t Platelets, II:154 Pleu ral pressure, I:89t, I:158 Pneu m ocephalu s, II:183 Pneu m oconiosis, I:229 Pneu m onectom y, I:171 Pneu m onia, II:25t–29t, II:459, II:470 Pneu m operitoneu m , II:422 Pneu m othorax, I:142t, I:148t, I:171, I:233, I:580, II:74, II:158 Poiseu ille’s law, I:10 Polycystic ovarian synd rom e, I:25t Polycythem ia vera (PV), I:359 PON V. See Postoperative nausea and vom iting Popliteal block, I:114t, I:584, II:82, II:89 Pop-off valve. See Ad justable pressu re lim iting valve Porphyria, I:138t Positive end -exp iratory p ressu re (PEEP), I:98t, I:119, I:201, I:214, II:35t, II:181, II:474 Post herp etic neuralgia (PH N ), II:549 Post-d u ral pu nctu re head ache (PDPH ), I:575 Posterior fossa, II:49t Posterior longitud inal ligam ent, II:65 Posterior spinal arteries, II:64 Postganglionic ber, I:246 Postherp etic neuralgia (PH N ), II:51t, II:566 Postintu bation laryngeal ed em a, II:370 Postop erative analgesia, I:576, I:577, I:590 Postop erative care, I:194, I:581 Postop erative nau sea and vom iting (PON V), I:43t, I:122t, I:409, II:391 Postop erative pain control, I:433 Postop erative pulm onary com plications, I:223 Postop erative respiratory d epression, I:283 Postop erative visual loss (POVL), I:42t, I:529, II:413

Postop erative w ound infection, I:335 Postpartum hem orrhage, II:32t–36t, II:36, II:248 Postseizu re state, II:527 Posttrau m atic stress, I:9t Potassiu m , I:71t, II:478 Potassiu m concentration, II:439 Potassiu m hyd roxid e, I:49 Potency, I:530 Pralid oxim e, II:43 Precord ial Dopp ler, II:205 Pred nisone, I:297, I:330, I:335 Preeclam psia, II:44t, II:78t, II:101t, II:234, II:250–251, II:283, II:351 Preem ptive analgesia, II:565 Preganglionic neu rotransm itter, I:238 Pregnancy, II:4t–5t, II:99t, II:107t, II:109t Pregnancy, rst trim ester of, I:557 Pregnancy, m olar, II:267 Pregnancy, m orbid obesity, II:124t Pregnancy, physiologic changes, II:12t, II:24t, II:256, II:260, II:265, II:284, II:301, II:305 Pregnancy, u teroplacental blood ow, II:262 Preop erative evaluation, I:310, I:313, I:349 Preoxygenation, I:191, II:397 Pressure assist/ control m od e ventilation, II:501 Pressure transd u cers, II:189-191 Preterm labor, II:274 Procainam id e, I:93 Prod ru g, I:515 Prolactin, I:255 Prone position, I:42t, I:55, I:56t Propofol, I:24t, I:62, I:90t, I:118t, I:270, I:388, I:412, I:479, I:486, I:490, I:502, I:505, I:509, II:3, II:180, II:211, II:221, II:319, II:402, II:430, II:458, II:482 Propranolol, I:425, I:461 Prostagland in E1, II:50t Prostatectom y, I:336 Protam ine, I:418, II:22 Protein C d e ciency, I:340 Protein catabolism , II:469 Protein S d e ciency, I:340 Proteinu ria, I:499 Proxim al airw ay pressu re, I:152 Proxim al tu bu le, I:522 Pu lm onary artery p ressu re, I:117 Pu lm onary capillary w ed ge pressure, I:103 Pu lm onary com plications, I:223

Inde x

Pu lm onary ed em a, I:59t, I:113, I:117t, I:212, II:215, II:472 Pu lm onary em bolism , I:106t, I:148, II:226, II:521 Pu lm onary brosis, I:1, I:81t, I:171, I:301 Pu lm onary fu nction tests (PFTs), I:153, I:174, I:226, II:495 Pu lm onary hem orrhage, II:156 Pu lm onary hypertension, I:150t, I:209, II:146 Pu lm onary out ow obstru ction, I:93t Pu lm onary resection, II:119 Pu lm onary toxicity, II:16 Pu lm onary vascu lar resistance, I:89t, I:178, I:516, II:60t, II:320 Pu lm onary vasoconstriction, I:208 Pu lm onary veins, II:131 Pu lm onary:system ic ow ratio, II:359–360 Pu lse oxim etry, I:27t, I:72, I:111t, I:190 Pu lse pressure, I:76t Pu lse pressure variation, I:100t, II:485 Pu lsus parad oxu s, I:127, II:504 Pu pillary d ilatation, I:526 Pu rposeless m ovem ents, I:14t, I:538 PV. See Polycythem ia vera Pylorom yotom y, II:98t Pyrid ostigm ine, II:4t–5t Q QTc. See Corrected QT interval Quaternary am ine, I:421 R Rad iation exp osure, I:3, I:16, I:31 Rad ioactive iod ine treatm ent, II:119t Rad iocontrast d ye, I:334 Rad iology room setup, I:540 Rapid ly p erfused tissu es, I:500 Rapid -sequence ind u ction, II:100 Rate of up take, I:536 Red blood cell esterases, I:5t Re ex arc, I:272 Regional anesthesia, I:559–600, II:38t, II:77 Regu rgitation of gastric contents, I:193 Relative hum id ity, I:24 Rem ifentanil, II:221 Renal blood ow, I:11t, I:307 Renal clearance, I:11t Renal failu re, I:135, I:315, I:334, I:455, I:499, II:150, II:410, II:494 Renal fu nction, I:349, II:134 Renal tubular reabsorp tion, I:373

449

Renin-angiotensin system , I:363, I:512 Repaglinid e, II:63 Resid u al volu m e, I:89t Respiratory acid osis, I:293, II:484 Respiratory alkalosis, I:496 Respiratory d ead space, II:331 Respiratory d epression, II:568 Respiratory d istress, II:24 Respiratory failure, II:34t Respiratory quotient, I:213 Restrictive lu ng d isease, I:136t Resuscitation, II:254–255, II:286, II:325–326, II:366, II:483 Retained placenta, II:269 Retinopathy of prem aturity (ROP), II:377 Rheu m atic fever, I:147t, II:162 Rheu m atoid arthritis, I:408, I:565, I:568 Rhonchi, II:53t–55t, II:506–507 Right bund le branch block, II:147, II:425 Right ventricu lar contractility, I:124 Right ventricu lar hypertrophy, I:93t Right ventricu lar strain, II:163–164 Ringer solution, I:554, I:558 Risk of aw areness, I:551 Rocu ronium , I:1t, I:372, I:396, I:486, II:3, II:345, II:430 ROP. See Retinop athy of prem atu rity Ropivacaine, I:123t, I:378, I:449, II:84–85 Rosiglitazone, II:61 Rotam eter, I:12 Ru ptu red spleen, II:158 S Sacral hiatu s, II:292 Sap henou s block, II:78–79, II:82, II:89 Sap henou s vein strip ping, I:458 SBT. See Spontaneous breathing trial SCI. See Sp inal cord inju ry Sciatic block, I:572, II:80, II:82, II:90 Sclerod erm a, I:199, I:301 Scoliosis, I:274, II:11t, II:107t Scop olam ine, I:43t, I:374, I:546, II:450 SCS. See Spinal cord stim u lation Second gas effect, I:544 Second heart sou nd , I:36t Sed ation, I:31t, II:45, II:94t, II:458 Seizu res, II:97t, II:396, II:534 Sensory transm ission, I:278 Sensory-evoked potentials, I:247 Sentinel event, II:453 Sepsis, II:11-12, II:490, II:492, II:530

450

Inde x

Septic arthritis, II:358 Serotonin, I:267 Seru m potassiu m concentration, II:439 Sevo urane, I:2, I:48t, I:54t, I:141t, I:368, I:381, I:483, I:505, I:532, I:534, II:104t, II:137t, II:354 Shivering, I:30t SIADH . See Synd rom e of inap prop riate secretion of antid iu retic horm one Sickle cell anem ia, I:132t, I:356, II:317, II:340, II:550 Sievert, I:31 SIH . See Spontaneou s intracranial hypotension Sild ena l, I:519, II:104 SIMV. See Synchronized interm ittent m and atory ventilation Sim vastatin, I:334 Single-breath nitrogen technique, I:13t Single-lum en tu be, II:107 Sinus brad ycard ia, I:455, I:588, II:94 Sinusitis, I:308 Sip p le synd rom e, I:360 Sitting position, I:56t, II:198, II:199, II:207 Skeletal m etastases, II:532 Skin graft, I:21t Skin hyperpigm entation, I:6t Skin resistance, I:30 Sm allpox, II:455 Sm oking, I:97t, I:122t, II:8, II:100, II:115, II:164 Sm ooth m u scle recep tors, II:466 Sod ium bicarbonate, II:42, II:229, II:366 Sod ium hyd roxid e, I:49 Sod ium nitrop ru ssid e, I:432, II:209 Solvent abuse, II:374 Som atic pain, II:564 Som atosensory-evoked p otentials (SSEP), I:53t, II:56t, II:195–197 Sore throat, II:372–373 Speci c gravity, I:47t, I:531 Speci c heat, I:60t Spinal anesthesia, I:70t, I:145t, I:262, I:561–562, I:569–570, I:574, I:591, II:67, II:82, II:118, II:304, II:334, II:349 Spinal cord , I:279, II:214, II:533 Spinal cord inju ry (SCI), II:84t, II:111, II:117, II:149, II:574 Spinal cord stim ulation (SCS), II:63t, II:133t, II:562, II:583 Spinal fusion, II:195–197 Spinal op ioid analgesia, II:14t Spinothalam ic tract, I:278

Spinou s process, I:599–600 Spirom etry, I:153, I:162, I:226 Spironolactone, I:521 Splenectom y, I:292 Splenic ru ptu re, I:430 Spontaneous breathing trial (SBT), II:87t Spontaneous interm ittent m and atory ventilation. See SIMV Spontaneous intracranial hypotension (SIH ), II:572 Spontaneous pneu m othorax, I:148t SSEP. See Som atosensory-evoked p otentials Stand ard d eviation, I:27 Stand ard s for Basic Intraop erative Monitoring, I:43 Statins, I:306 Statistical tests, I:4, I:8 Stellate ganglion block, I:595, II:579 Stereosp eci c bind ing, II:45 Stopcock, II:191 Strabism us repair, II:117t–118t Strid or, I:290, II:372–373 Stroke volu m e, I:97 Stu d ent’s t-test, I:4 Stu rge-Weber synd rom e, II:396 Su barachnoid hem orrhage, I:263, II:73t, II:179, II:217–219 Su barachnoid sp ace, I:67t, I:72t Su bcu taneou s injection, I:477, I:583 Su bd ural hem atom a, I:99t, II:21t Su bglottic ed em a, II:378 Su blingu al d rug ad m inistration, I:427 Su bstance abu se, I:300, II:52 Su bstance P, II:40 Su bstantia gelatinosa, I:253 Su ccinylcholine, I:21t, I:46t, I:286, I:384, I:392, I:435, I:479, I:511, II:37, II:178, II:345, II:402–403, II:414, II:430 Su ction, I:540 Su fentanil, I:94, II:402–403 Su m p su ction, II:346 Su per cial peroneal nerve, II:78 Su perior laryngeal nerve, I:598 Su pine p osition, I:56t, I:65, I:164, I:169 Su praclavicu lar block, I:142t, I:579–580, II:76, II:79t, II:81, II:86, II:91, II:93 Su prapu bic catheter, I:260–262 Su prascapular nerve, II:91t Su ral nerve, II:78 Su rfactant, I:160 SVR. See System ic vascu lar resistance Sw eating, I:124t

Inde x

Sym pathetic nervou s system , I:238, II:540 Synchronized interm ittent m and atory ventilation (SIMV), I:179, II:488 Syncope, I:84t Synd rom e of inappropriate secretion of antid iuretic horm one (SIADH ), I:344 Syphilis, I:299 System ic lu pu s erythem atosus, II:504–505 System ic sequelae, II:185 System ic toxicity, I:572 System ic vascular resistance (SVR), II:465 T T w aves, II:477 Tachycard ia, I:5t, I:68t, I:136, I:140, I:404, I:458–459, II:24–25, II:53t–55t, II:135–136, II:139 Tachyphylaxis, II:28 Tachypnea, I:117t, II:106–107 Taylor app roach, I:562 TBW. See Total bod y w ater Tec 5 vaporizer, I:69t TEF. See Tracheoesop hageal stu la TEG. See Throm boelastogram Tem peratu re, I:60t, I:124t–126t, I:236, II:81t, II:175, II:333, II:428 TEN S. See Transcutaneou s electrical nerve stim u lation Terbu taline, II:58t, II:246, II:274 Term inal half-life of d rug, I:442 Tetanu s, I:33t, I:39t Tetracaine, I:449, I:582 Tetralogy of Fallot, I:93t, I:137, II:336 Thalam u s, I:247, I:249 Therapeu tic hypotherm ia, II:513 Therm oregu lation, II:347, II:445 Theta w ave, I:251 Thiam ine d e ciency, I:294 Thiazid e d iu retics, I:416 Thiopental, I:21t, I:138t, I:369, I:389, I:412, I:445 Thoracic d eform ities, I:175 Thoracic d erm atom e, I:144t Thoracoabd om inal aneu rysm , II:148 Thoracotom y, II:598 Throm bin inhibition, I:414 Throm bocytopenia, I:495 Throm boelastogram (TEG), II:154 Throm bop hlebitis, I:556 Thyroid isthm us, II:155 Thyroid ectom y, I:290, I:355 Thyrotropin, I:255

Tid al volu m e, I:50, I:230, II:294 TIPS. See Transjugu lar intrahepatic portosystem ic shunt Tissue w ater, I:237 TN S. See Transient neurological sym ptom s Tobacco, I:97t, I:293. See also Sm oking Tonic-clonic m ovem ents, II:560 Tonic-clonic seizu res, II:30t Tonsillectom y, I:40t, I:286, II:140t Tooth extraction, I:40t Torsad es d es pointes, I:93 Total bod y tissu e perfu sion, II:468 Total bod y w ater (TBW), II:361 Total parenteral nutrition, II:38 Total pu lm onary com pliance, I:235 Tourniqu et, I:593, II:64t, II:427 Toxic goiter, I:355 T-piece, I:39 Trachea, II:155 Tracheal collapse, II:152 Tracheal ed em a, I:4, I:10 Tracheal extu bation, I:486 Tracheobronchial tree, II:19t Tracheoesophageal stula (TEF), II:346, II:379 Train-of-fou r stim ulus, I:410, I:411 Tram ad ol, II:580 Tranexam ic acid , I:89 Transcatheter aortic valve im plantation, II:151 Transcranial Doppler, II:217 Transcu taneou s electrical nerve stim u lation (TEN S), II:554 Transd erm al scopolam ine, I:43t, I:374 Transesophageal echocard iography, II:116, II:139, II:149t-150t, II:165-173, II:205 Transfusion risk, II:442 Transgastric long axis, II:149t Transgastric m id -papillary short axis, II:168 Transgastric tw o cham ber, II:167 Transient neu rological sym ptom s (TN S), I:566 Transju gular intrahepatic portosystem ic shunt (TIPS), II:444 Transm em brane transp ort m echanism , I:493 Transposition of great arteries, II:382 Transpu lm onary pressure, I:127t, I:225 Transsphenoid al hypophysectom y, I:329, II:220 Transthoracic echocard iogram , II:137 Transu rethral resection, II:419–420 Transverse process, II:71 Trau m a patient, I:324 Trend elenburg position, I:55, II:240, II:279

451

452

Inde x

Triam terene, I:77 Triazolam , II:120t Tricuspid regu rgitation, I:147, II:97 Tricyclic antid epressants, II:17, II:111t, II:590 Trigem inal nerve, I:250, I:254, II:118t Triglycerid es, I:25t Tubu lar necrosis, II:106t, II:476 Tubu lar secretion, I:380 Tunneled IV catheter, II:38 Turner synd rom e, II:394 Tw ins, II:273 U Ulnar nerve block, II:88, II:90t, II:93 Ultrasou nd , II:72–74, II:83–85, II:266–268 Ultrasou nd -gu id ed central venou s catheter placem ent, II:161 Ultrasou nd -gu id ed nerve block, I:587, II:81 Um bilicu s, I:145t Uncu ffed end otracheal tu be, II:335 Univent tu be, II:100 Upper esophageal aortic valve long axis, II:173 Upper respiratory tract infection, II:384–385 Uptake rate, I:536 Urem ia, I:291, II:142t Urine, I:11t, I:315, I:351 Urosepsis, I:333 Uterine atony, II:67t, II:248 Uterine bleed ing, II:266 Uterine inversion, II:279–280 Uterine rupture, II:257, II:277 Uterop lacental blood ow, II:262 V Vaginal birth after cesarean section (VBAC), II:247 Vaginal d elivery, II:273 Vagu s nerve, I:250, I:254, II:118t Valp roic acid , II:103t Valsartan, I:520 Vancom ycin, II:54t Vaporization, I:80t Vaporizer, I:35, I:54t, I:69t Varenicline, I:97t Variable d eceleration pattern, II:61t Vascular sm ooth m uscle, I:516 Vasoconstriction, I:61t, I:92, I:208 Vasod ilation, I:135t, I:140t, II:14, II:29, II:104 Vaso-occlusive crisis, II:550 Vasopressin, I:430, II:11–12 Vasospasm , II:219

VATS proced u re, II:112t VBAC. See Vaginal birth after cesarean section Vecu ronium , I:124t–126t, I:396, I:398, I:509 Venous access, I:49t Venous air em bolism , I:75, II:80t, II:205, II:222 Venous blood , I:237 Venous pressu re record ing, II:97 Ventilation, I:164, I:177, I:543, II:17t, II:35t–36t, II:112t, II:488, II:491, II:512 Ventilation, closed chest, II:157 Ventilation/ p erfu sion ratio (V/ Q ratio), I:41, I:156, I:196, I:210, I:234, I:524 Ventilator-associated pneu m onia, II:459 Ventilators, I:44t, I:50, I:175, II:32t, II:86t, II:461, II:483 Ventilatory control centers, I:244 Ventral ram i, II:80 Ventricular arrhythm ias, I:78t, I:123t, I:404, I:588, II:510 Ventricular fu nction curve, I:513, I:514 Ventricular septal d efect (VSD), I:93t, I:146t, II:329, II:359–360, II:375 Ventriculoperitoneal shu nt, II:383 Ventriculostom y catheter, I:72t, I:241 Verapam il, I:68t, I:140, I:413 Vertebral arteries, II:64 Vertebral colu m n, II:70 Vertebrop lasty, II:538 Vessel-rich tissues, I:500 Vestibu lar-cochlear nerve, I:247 VF. See Ventricular brillation Vincristine, I:295 Viscera, II:540 Visceral p ain, II:564 Vital capacity, I:226, I:230, II:438 Vital signs, I:581 Vitam in A d e ciency, II:400 Vitam in B12 d e ciency, I:509 Vitam in K, I:89, I:134 Vom iting, I:276, II:5, II:40, II:371 Von H ippel-Lind au synd rom e, II:393 Von Willebrand d isease (vWD), I:40t, II:233 Von Willebrand factor, I:89 V/ Q ratio. See Ventilation/ perfu sion ratio VSD. See Ventricular septal d efect VWD. See Von Willebrand d isease W “Wake up ” test, II:186 Wall connector oxygen pressu re, I:94t Warfarin, I:18t, II:41 Warm ing d evices, II:332

Inde x

Waste gas scavenging system , I:51 Water-ham m er p ulse, I:126 Watt, I:33 Wheezing, II:374, II:506–507 Whipp le p roced u re, II:147, II:508–513 Wilcoxon rank-sum test, I:4 William s synd rom e, II:387 Wind u p, II:535 Withd raw al synd rom e, II:20, II:34 Wolff-Parkinson-White synd rom e, I:84t, I:140 Work of breathing, I:203, I:214, II:367 Wou nd healing, I:122t Wou nd infections, I:97t

X Xenon, I:10, I:11 X-ray, I:220 Y Yoke-tank connections, I:62t Z Zaleplon, II:120t Ziconotid e, II:539

453